Sie sind auf Seite 1von 2086

MULTIPLE CHOICE

PAPER ONE

1
International General Certificate of Secondary Education
UNIVERSITY OF CAMBRIDGE LOCAL EXAMINATIONS SYNDICATE

PHYSICS 0625/1
PAPER 1 Multiple Choice
Wednesday 19 MAY 1999 Morning 45 minutes

Additional materials:
Electronic calculator and/or Mathematical tables
Multiple Choice answer sheet
Soft clean eraser
Soft pencil (type B or HB is recommended)

TIME 45 minutes

INSTRUCTIONS TO CANDIDATES
Do not open this booklet until you are told to do so.
Write your name, Centre number and candidate number on the answer sheet in the spaces provided
unless this has already been done for you.
There are forty questions in this paper. Answer all questions. For each question, there are four
possible answers, A, B, C and D. Choose the one you consider correct and record your choice in soft
pencil on the separate answer sheet.
Read very carefully the instructions on the answer sheet.

INFORMATION FOR CANDIDATES


Each correct answer will score one mark. A mark will not be deducted for a wrong answer.
Any rough working should be done in this booklet.

This question paper consists of 19 printed pages and 1 blank page.


SB (SC) QF91698/4
© UCLES 1999 [Turn over
1 A measuring cylinder contains some liquid.

cm3
10
9
8
7
6
5
4
3
2
1

What does the reading of the measuring cylinder scale give?

A the density of the liquid


B the height of the liquid
C the mass of the liquid
D the volume of the liquid

2 A tennis ball falls from the upstairs window of a house.

Ignoring air resistance, what happens to the acceleration of the ball?

A It decreases.
B It increases.
C It is constant.
D It is zero.

0625/1 S99
3 The graph shows how the speed of a car changes over 200 s.

20
speed
m/s

0
0 50 100 150 200
time/s

For how many seconds is the car moving at constant speed?

A 50 s B 100 s C 150 s D 200 s

4 Two apples are placed on a balance. The balance stays level.

x x

apple 1 apple 2

What does this show about the mass and the weight of the apples?

mass of apples weight of apples


A different different
B different the same
C the same different
D the same the same

0625/1 S99 [Turn over


5 The diagram shows a racing car.

PETROIL

32 V I S T

VA LV O I L

Where should the centre of mass be, and why?

centre of mass
where why
A as high as possible to give the car more acceleration
B as high as possible to give the car more stability
C as low as possible to give the car more acceleration
D as low as possible to give the car more stability

6 An object is pushed and then allowed to slide along a table.

The diagram shows the direction of the friction force acting on the object.

object sliding
object this way

table

friction force

What is the effect of the friction force on the sliding object?

A It has no effect.
B It increases the object’s weight.
C It slows the object down.
D It speeds the object up.

0625/1 S99
7 A student swings on a rope tied to a branch of a tree.

At which position is the student’s energy of motion (kinetic energy) the greatest?

A
D
B C

8 The diagram shows four types of power station.

Which type of power station uses chemical energy to generate electricity?

A B C D
geothermal coal-fired nuclear hydroelectric

hot rocks heat coal burned to heat nuclear fuel heats water from mountain
water, steam turns water, steam turns water, steam turns reservoir turns
turbine and generator turbine and generator turbine and generator turbine and generator

0625/1 S99 [Turn over


9 Four people each have to lift bricks to the same height with a rope and pulley. Some lift more
bricks, and some lift the bricks faster.

Which person develops the most power?

A B C D

h h h h

10 bricks 10 bricks 20 bricks 20 bricks


in in in in
2 seconds 4 seconds 2 seconds 4 seconds

10 The diagram shows a drawing pin (thumb tack) and a wooden board.

force

drawing pin

wooden board

The drawing pin is easily pushed into the wooden board.

Why is this?

A The flat head raises the centre of mass.


B The force produces a large moment about the point.
C The large area of the flat head increases the force.
D The small area of the point increases the pressure.

0625/1 S99
11 A liquid evaporates quickly.

How does its temperature change, and which of its molecules leave the surface?

temperature molecules
change leaving surface
A rises fastest-moving
B rises slowest-moving
C falls fastest-moving
D falls slowest-moving

12 A pupil looks through a microscope into a small, glass container in which tiny smoke particles
have been mixed with air. The container is very brightly lit and the pupil sees bright specks moving
randomly.

What are these specks?

A air molecules being hit by air molecules


B air molecules being hit by smoke particles
C smoke particles being hit by air molecules
D smoke particles being hit by smoke particles

13 A substance that melts at 60 °C is heated in a test-tube to a temperature of 95 °C. The substance


is then allowed to cool.

Which graph best shows how the temperature changes with time as the substance cools?

A B C D
temperature / °C

temperature / °C

temperature / °C

temperature / °C

100 100 100 100

0 0 0 0
0 time 0 time 0 time 0 time

0625/1 S99 [Turn over


14 The diagram shows an experiment to investigate the effect of a rise in temperature on an iron bar.

iron bar pointer

fixed support roller with


pointer attached
heat

What would be seen after heating?

15 A flask with a narrow tube contains some coloured liquid. The diagram shows the height of the
liquid column at 0°C, 100 °C and X °C.

80
70
60
50
millimetres 40
30
20
10
0

0 °C 100 °C X °C

The liquid expands uniformly with temperature.

What is temperature X?

A 40 °C B 50 °C C 60 °C D 80 °C
0625/1 S99
16 Four wires of equal length, made of different materials, are placed on heat-sensitive paper. The
four ends of the wire are put together, as shown, and are heated for three minutes.

heat-sensitive paper

heated here

Heat-sensitive paper changes colour when it gets hot. The diagram below shows the marks on the
heat-sensitive paper due to the conduction of heat energy along the wires.

Which mark was made by the best conductor of heat?

C
A
D

17 A hot-air balloon rises when the air inside it is heated.

Why does this happen?

A The air contracts and becomes less dense.


B The air contracts and becomes more dense.
C The air expands and becomes less dense.
D The air expands and becomes more dense.

0625/1 S99 [Turn over


18 Different surfaces reflect different proportions of the Sun’s radiant energy. The chart shows the
percentage of heat energy that is reflected by some surfaces.

100
% of heat
80
energy
reflected
60

40

20

0
white- red soil tar
washed brick

What is the best way to treat a flat roof so as to increase the amount of heat absorbed by the roof?

A cover it with a layer of red brick dust


B cover it with a layer of soil
C paint it with tar
D paint it with whitewash

19 A barrier lies at the mouth of a harbour. It is closer to one side of the mouth of the harbour than the
other. Sea waves come towards the harbour and pass through the two gaps.

Which diagram shows the pattern that the waves make when they pass through the gaps?

waves moving
this way A B
sea

barrier

harbour

C D

0625/1 S99
20 Which diagram shows the path of a ray of light through a glass prism and into the air beyond?

A B C D

light 45° light 45° light 45° light 45°

ray ray ray ray

21 Which diagram shows how a converging lens in a camera forms an image on the film?

object image

object image

object image

object image

22 What is heard when the frequency of a sound wave is increased at constant amplitude?

A higher pitch
B lower pitch
C louder sound
D quieter sound

0625/1 S99 [Turn over


23 A bat is flying near a house. It makes a high-pitched sound. It hears the echo 0.5 s later. The
speed of sound in air is 300 m/s.

How far away is the house from the bat?

A 600 m B 300 m C 150 m D 75 m

24 A student holds two pins on to a magnet as shown.

The student removes his thumb.

Which diagram shows the positions of the pins and the poles induced on them?

A B C D

N N N N
S S N N S S N S
N N S S N N S N

25 Different metals are separated by passing them under an electromagnet, which attracts and lifts
out some of the metals.

electromagnet
different metals

moving belt

Which line in the table shows what happens to the metals?

attracted not attracted


A copper aluminium
B copper steel
C iron aluminium
D iron steel

0625/1 S99
26 Which diagram shows the directions of the forces between two charged particles?

A + +

B + –

C + –

D – –

27 A battery is labelled ‘9 VOLTS’.

BATTERY
9 VOLTS

Which information does the label give about the battery?

A the power it can supply


B the current it can supply
C its electromotive force (e.m.f.)
D its resistance

28 The diagram shows how a student has connected some electrical components. The lamp does
not light because the circuit has not been completed.

crocodile clip
switch
battery lamp

connecting wire P
Q R S

crocodile clip

Where must the crocodile clips be connected, so that the lamp lights only when the switch is
closed?

A at P and R B at P and S C at Q and R D at Q and S


0625/1 S99 [Turn over
29 The diagram shows a circuit in which all the lamps are lit.

The lamp marked X blows and breaks the circuit.

How many lamps remain lit?

A 0 B 1 C 2 D 3

30 In which diagram is the fuse correctly connected?

A B

live live

to mains earth to mains earth


supply supply
neutral neutral

heater heater
metal case metal case

C D

live live

to mains earth to mains earth


supply supply
neutral neutral

heater heater
metal case metal case

0625/1 S99
31 The diagram shows an electrical hazard.

soft plastic insulation


bare copper wire

hard plastic lamp holder

glass bulb

Which part gives an electric shock if touched?

A soft plastic insulation


B bare copper wire
C hard plastic lamp holder
D glass bulb

32 Which diagram shows the correct wiring of a transformer?

A B

C D

33 Which object makes use of the force on a current-carrying conductor in a magnetic field?

A electric fan
B electric kettle
C lamp
D transformer

0625/1 S99 [Turn over


34 Which diagram shows an electric bell correctly connected?

A B C D

35 A straight wire connected to a milliammeter is placed between the poles of a magnet.

3 2

straight wire

milliammeter

Which change causes a current in the milliameter?

A moving the wire in direction 1


B moving the wire in direction 2
C moving the magnet in direction 2
D moving the magnet in direction 3

0625/1 S99
36 Particles given off by a hot cathode in a vacuum are attracted to a plate P. There is a high potential
difference between the cathode and plate as shown.

vacuum

hot
P
cathode

– +

What are these particles?

A α-particles
B electrons
C neutrons
D protons

37 Which of the following is a correct statement about β-particles?

A They are fast-moving electrons.


B They are more ionising than α-particles.
C They are stopped by a thin sheet of paper.
D They can pass through a thick sheet of lead.

0625/1 S99 [Turn over


38 The graph shows how the activity of a radioactive source varies with time.

count rate 400


count/minute
300

200

100

0
0 10 20 30 40 50 60 70 80 90 100
time
minute

What is the half-life of the source?

A 50 minutes
B 100 minutes
C 200 minutes
D 400 minutes

39 Hydrogen may be written as 11H.

Which line is correct for a neutral hydrogen atom?

number of number of number of


electrons protons neutrons
A 0 1 1
B 1 0 1
C 1 1 0
D 1 1 1

0625/1 S99
40 The diagram represents the radioactive decay of a nucleus.

4
2 He

238
92 U

A
Z Th

What are the values of A and Z ?

A Z
A 238 93
B 238 92
C 236 88
D 234 90

Go to answers

0625/1 S99
International General Certificate of Secondary Education
UNIVERSITY OF CAMBRIDGE LOCAL EXAMINATIONS SYNDICATE
PHYSICS 0625/1
PAPER 1 Multiple Choice
Tuesday 9 NOVEMBER 1999 Morning 45 minutes

Additional materials:
Electronic calculator and/or Mathematical tables
Multiple Choice answer sheet
Soft clean eraser
Soft pencil (type B or HB is recommended)

TIME 45 minutes

INSTRUCTIONS TO CANDIDATES
Do not open this booklet until you are told to do so.
Write your name, Centre number and candidate number on the answer sheet in the spaces provided
unless this has already been done for you.
There are forty questions in this paper. Answer all questions. For each question there are four
possible answers, A, B, C and D. Choose the one you consider correct and record your choice in soft
pencil on the separate answer sheet.
Read very carefully the instructions on the answer sheet.

INFORMATION FOR CANDIDATES


Each correct answer will score one mark. A mark will not be deducted for a wrong answer.
Any rough working should be done in this booklet.

This question paper consists of 16 printed pages.


(NH) QK93704/4
© UCLES 1999 [Turn over
2

1 The digital stopwatches show the finishing times of two runners in a race.

runner 1 runner 2

min s min s
: :

What is the time difference between the two runners?

A 1.14 s B 7.28 s C 8.42 s D 15.70 s

2 The graph shows how the speed of an object changes with time.

speed
m/s

0
0 2 4 6 8
time
s

For how long does the object move?

A 2s B 4s C 6s D 8s

3 Which statement is true about the weight of an object?

A Its weight is the same everywhere.


B Its weight is measured in kilograms.
C Its weight is the force of gravity on it.
D Its weight is zero on the Moon.

0625/1 W99
3

4 The diagram shows some liquid in a measuring cylinder. The mass of the liquid is 16 g.

cm3

30

20

10

What is the density of the liquid?

A 320 g/cm3 B 36 g/cm3 C 1.25 g/cm3 D 0.8 g/cm3

5 A heavy box is lifted by a force F, using a lever as shown.

What happens when the length x is made shorter?

distance F
moves size of F

A bigger bigger
B bigger smaller
C smaller bigger
D smaller smaller

0625/1 W99 [Turn over


4

6 In which situation is there no resultant force on the car?

40 m/s 30 m/s
A decreasing speed on a level road

m/s
30
B going at a constant speed uphill

30
m/s
40
C increasing speed downhill m/s

30 m/s 40 m/s
D increasing speed on a level road

7 A steel ball is allowed to roll along a frictionless track.

What is the highest point the ball reaches before rolling backwards? Ignore the effects of friction.

steel ball
C A

8 What is the source of geothermal energy?

A coal
B hot rocks
C tides
D waves

0625/1 W99
5

9 Which object converts sound energy into electrical energy?

A electric bell
B loudspeaker
C microphone
D radio

10 A manometer is connected to a gas tap. Diagram X shows the liquid levels before opening the gas
tap. The gas pressure is greater than atmospheric pressure.

Which diagram shows what happens to the liquid levels after opening the gas tap?

gas tap X A B C D

from gas supply

liquid

before opening
gas tap

11 The diagram shows water in an outdoor swimming pool.

Sun

water

swimming pool

The water level drops during a hot, sunny day.

What does this show?

A that water boils at only one temperature


B that water can evaporate below its boiling point
C that water can expand as it becomes warmer
D that water is a poor conductor of heat

0625/1 W99 [Turn over


6

12 A closed container full of gas is left to stand on a bench for a long time.

Which statement about the molecules of the gas is correct?

A All the molecules are at the bottom of the container.


B The fastest moving molecules are at the top of the container.
C The molecules are moving at random throughout the container.
D The pressure caused by the molecules on the container is greatest at the top of the container.

13 Metal railway lines can be laid in sections with small gaps in between, as shown. The sections are
connected by metal plates.

gap

railway line

metal plate

What is the reason for leaving the gaps?

A to allow for contraction when the temperature falls


B to allow for expansion when the temperature rises
C to prevent an electric current in the railway line
D to stop large vibrations

14 Four cooking pots are each made of iron.

Which cooking pot has the greatest heat capacity?

A B C D

mass = 2.0 kg mass = 1.8 kg mass = 1.6 kg mass = 1.4 kg


0625/1 W99
7

15 Two balloons are inflated at the same room temperature until they have the same volume of air
inside. They are then sealed. Balloon X is placed in a refrigerator for some time, while balloon Y
stays at room temperature. The diagram shows the balloons after this time.

balloon balloon
X Y

Assuming no air has leaked out of the balloons, which statement is correct?

A The air in balloon X has contracted.


B The air in balloon Y has expanded.
C The rubber of balloon Y has expanded.
D The rubber of balloon X has frozen and has compressed the air.

16 On a day when the temperature is low, the metal handlebars of a bicycle feel colder than the
plastic handlegrips.

What is the explanation for this?

A Metals are colder than plastics.


B Metals conduct heat better than plastics.
C Plastics are softer than metals.
D Plastics conduct heat better than metals.

17 A man puts a saucer on top of a cup of tea to keep the tea hot.

saucer

cup

Why does this help to keep the tea hot?

A Conduction through the cup is reduced.


B Convection in the air above the surface of the tea is reduced.
C Convection in the tea is reduced.
D Radiation from the shiny surface of the cup is reduced.
0625/1 W99 [Turn over
8

18 The following diagram represents a wave.

1 cm

1 cm

What is the wavelength of the wave?

A 8 cm B 4 cm C 2 cm D 1 cm

19 The diagram shows a wavefront striking a plane reflecting surface PQ.

P Q

wavefront

Which of the following is changed by reflection?

A direction
B frequency
C speed
D wavelength

20 A child is 1 m tall. She stands 2 m in front of a plane mirror.

mirror
2m

1m

How far away from the child is her image?


A 1m B 2m C 3m D 4m
0625/1 W99
9

21 Which diagram shows a possible path for a ray of light passing through water in a beaker with a
mirror at the bottom?

A B C D

water

mirror

22 An electric bell is suspended in a bell-jar as shown. Switch S is closed and air is then pumped
from the jar. The sound of the bell becomes quieter.

electric
bell

bell-jar

to vacuum pump

Why does the sound become quieter?

A Air is needed for an electric current in the bell.


B Air is needed for the bell’s electromagnet to operate.
C Air is needed for the bell to vibrate.
D Air is needed to transmit sound waves.

0625/1 W99 [Turn over


10

23 A student shouts when standing by a cliff. A short time later the student hears an echo.

cliff

student

Which property of sound causes the echo?

A diffraction
B dispersion
C reflection
D refraction

24 The diagram shows the circuit for an electromagnet. The core of the electromagnet can be made
from different materials.

S material under test

iron nails attracted


to material

When switch S is closed, which material attracts the largest number of iron nails?

A aluminium
B copper
C glass
D iron

0625/1 W99
11

25 A plotting compass is placed at each end of a magnet.

Which diagram shows the positions of the pointers of the plotting compasses?

26 Two very light, charged balls P and Q are hung, one above the other, from nylon threads. When a
negatively charged plastic sheet is placed alongside them, P is repelled and Q is attracted.

––
––
––
negatively ––
––
charged ––
plastic sheet –– P
––
––
––
––
––
––
––
–– Q
––

What are the original charges on P and on Q?

charge on P charge on Q

A negative negative
B negative positive
C positive negative
D positive positive

0625/1 W99 [Turn over


12

27 The meter in the circuit measures the potential difference across the resistor.

meter

Which unit is marked on the scale of the meter?

A amp
B ohm
C volt
D watt

28 Two resistors are connected in a circuit as shown.

2X

3X

What is the total resistance of the resistors?

A less than 2 Ω
B between 2 Ω and 3 Ω
C between 3 Ω and 5 Ω
D more than 5 Ω

0625/1 W99
13

29 The diagram shows a circuit with three lamps and a cell.

What is the circuit diagram for the above arrangement?

A B C D

30 Which device can be used to switch off the current in a circuit when the current becomes too
large?

A capacitor
B circuit-breaker
C light-dependent resistor
D transformer

31 A hair dryer should

(i) allow the fan to work with or without the heater,


(ii) not allow the heater to work without the fan.

Which circuit should be used?

A B
fan fan
M M

heater heater
C D
fan fan
M M

heater heater
0625/1 W99 [Turn over
14

32 A straight wire carries an electric current at right angles to the page. The black dot in each
diagram shows where the wire passes through the page.

Which diagram shows the magnetic field pattern around the wire?

A B C D

33 When electrical energy is transmitted over large distances, a high voltage is used.

Why is this better than using a low voltage?

A There is a greater current in the cables.


B There is less chance of an electric shock.
C There is less heating in the cables.
D Thicker cables can be used.

34 Which graph shows the output voltage V of an a.c. generator?

A B

V V

0 0
0 time 0 time

C D

V V

0 0
0 time 0 time

35 What is the symbol for a capacitor?

A B C D

0625/1 W99
15

36 A waveform is displayed on the screen of a cathode-ray oscilloscope (c.r.o.).

1 cm

The time base now changes from 1 ms/cm to 2 ms/cm.

Which diagram shows the new appearance of the waveform on the screen?

A B

C D

37 The half-life of a radioactive substance is 10 days.

Which of the following statements is correct?

A After 5 days, half of the original radioactive nuclei remain.


B After 5 days, a quarter of the original radioactive nuclei remain.
C After 10 days, half of the original radioactive nuclei remain.
D After 20 days, none of the original radioactive nuclei remain.

0625/1 W99 [Turn over


16

38 What can most -particles pass through?

A 2 mm of aluminium
B 2 mm of lead
C 2 cm of air
D 2 cm of concrete

39 The diagram represents an atom.

Which of the following describes the atom?

nucleus electron charge on electron

A in middle orbits round +


of atom nucleus
B in middle orbits round –
of atom nucleus
C orbits round in middle –
electron of atom
D orbits round in middle 0
electron of atom

40 Which statement about the numbers of particles in a neutral atom must be correct?

A number of protons = number of electrons


B number of protons = number of neutrons
C number of protons + number of electrons = number of neutrons
D number of protons + number of neutrons = number of electrons

Go to answers

0625/1 W99
International General Certificate of Secondary Education
UNIVERSITY OF CAMBRIDGE LOCAL EXAMINATIONS SYNDICATE
PHYSICS 0625/1
PAPER 1 Multiple Choice
MAY/JUNE SESSION 2000 45 minutes

Additional materials:
Electronic calculator and/or Mathematical tables
Multiple Choice answer sheet
Soft clean eraser
Soft pencil (type B or HB is recommended)

TIME 45 minutes

INSTRUCTIONS TO CANDIDATES
Do not open this booklet until you are told to do so.
Write your name, Centre number and candidate number on the answer sheet in the spaces provided
unless this has already been done for you.
There are forty questions in this paper. Answer all questions. For each question, there are four
possible answers, A, B, C and D. Choose the one you consider correct and record your choice in soft
pencil on the separate answer sheet.
Read very carefully the instructions on the answer sheet.

INFORMATION FOR CANDIDATES


Each correct answer will score one mark. A mark will not be deducted for a wrong answer.
Any rough working should be done in this booklet.

This question paper consists of 18 printed pages and 2 blank pages.


SB (SLC/DJ) QF05707/3
© UCLES 2000 [Turn over
2

1 A stopwatch is used to time a student running a 1500 m race.

What is the reading on the stopwatch?

A 658.31 seconds
B 6 minutes 58.31 seconds
C 6.58 minutes 31 seconds
D 6 hours 58 minutes 31 seconds

2 A student lets 100 drops of water fall into a measuring cylinder which already contains some
water.

cm3 cm3
50 50

40 40

30 30
new level
20 original level 20 of water
of water
10 10

What is the volume of one drop?

A 0.05 cm3 B 0.25 cm3 C 5.0 cm3 D 25 cm3

3 Which speed-time graph represents the motion of a car moving at constant speed?

A B C D
speed

speed

speed

speed

0 0 0 0
0 time 0 time 0 time 0 time

0625/1 S00
3

4 A student investigates the speed of a trolley and finds that it is 50 cm/s, and one second later that
it is 150 cm/s.

What is the acceleration of the trolley?

A 50 cm/s2 B 100 cm/s2 C 150 cm/s2 D 200 cm/s2

5 Which property of an object is measured in newtons?

A density
B mass
C volume
D weight

6 Which instrument can be used to give a direct measurement of the mass of a rock on the Moon?

A B C D
bathroom scales beam balance force meter top-pan balance

7 The diagrams show an empty beaker on a balance and some liquid in a measuring cylinder.

cm3
100

80

60

liquid 40

20

When all the liquid is poured into the beaker, the balance reading changes to 140 g.

What is the density of the liquid?

A (140 ÷ 60) g/cm3 B (60 ÷ 40) g/cm3 C (40 ÷ 60) g/cm3 D (60 ÷ 140) g/cm3
0625/1 S00 [Turn over
4

8 The diagrams show a brick resting on a smooth surface. Two equal forces, F, act on the brick.

Which brick does not move?

F
A B C D

F F F

F F F F

9 The diagram shows four ways of lifting a heavy box by using a lever.

Which way requires the smallest effort to lift the box?

effort
A B effort

box box

effort
effort
C D

box box

10 The diagrams show a diver climbing some steps and jumping off a diving board.

Where does the diver have the most gravitational potential energy (energy of position)?

A B C D

0625/1 S00
5

11 The diagrams show four schemes which use a barrier to convert tidal energy into electrical energy.

Which scheme has the greatest output of electrical energy?

barrier barrier
A B
turbine high turbine high
water water
level level
low
water
level low
water
level

C D
high high
water water
level level
low
water
level
low
water
level

12 The diagram shows a mercury barometer.

mercury

What is at X?

A air at atmospheric pressure


B air at high pressure
C a vacuum
D water vapour
0625/1 S00 [Turn over
6

13 Which diagram shows the best shape for a dam wall?

A B
dam wall water dam wall water

C D
dam wall water dam wall water

14 A fixed mass of gas is kept in a sealed cylinder, so that its volume does not change.

GAS

How does a change in the temperature affect the pressure of the gas?

temperature change pressure change


A decreases increases
B decreases unchanged
C increases decreases
D increases increases

0625/1 S00
7

15 The diagram shows some molecules in a liquid.

Molecules A and D are fast-moving, molecules B and C are slow-moving.

Which of these molecules is the most likely to escape from the liquid by evaporation?

A (fast) B (slow)

liquid

D (fast) C (slow)

16 A mercury thermometer without a scale is placed in pure melting ice.

mercury

pure
melting
ice

water

What does the level of mercury in the thermometer show?

A the boiling point of water


B the lower fixed point
C the melting point of mercury
D the upper fixed point

0625/1 S00 [Turn over


8

17 Some ice is heated at a constant rate in a beaker. The ice melts and later the water boils for a
short while.

Which graph shows how the temperature changes with time?

A B C D
temperature/ °C

temperature/ °C

temperature/ °C

temperature/ °C
0 0 0 0
time time time time

18 A person lifts a hot pan from a stove. Although the pan is hot, the handle is cool.

What is the handle made of?

A aluminium
B copper
C iron
D wood

19 Warm water enters a tank of cold water from the side.

warm water cold water

What happens to the warm water and why?


warm water why?
A rises It is less dense than cold water.
B rises It is more dense than cold water.
C sinks It is less dense than cold water.
D sinks It is more dense than cold water.

0625/1 S00
9

20 Plane waves produced in a ripple tank arrive at a barrier that has a small gap.

waves moving in
this direction

What causes the wave pattern that is produced beyond the barrier?

A diffraction
B dispersion
C reflection
D refraction

21 A floating cork moves up and down as a water wave passes it.

direction of water wave

movement of cork

A candle flame moves backwards and forwards as a sound wave passes it.

direction of sound wave

movement of flame

What does this show about water waves and sound waves?

water waves sound waves


A longitudinal longitudinal
B longitudinal transverse
C transverse longitudinal
D transverse transverse
0625/1 S00 [Turn over
10

22 Which diagram shows how a real image is formed by the lens?

lens

A image

lens

B image

lens

C image

lens

D image

23 A ray of light is reflected from two mirrors as shown.

60° 80°

30° 30°
40°
50°
X

What is the angle of reflection at point X?

A 40° B 50° C 60° D 80°

0625/1 S00
11

24 Two windows are double-glazed. Window X is made of two pieces of glass with a vacuum between
them. Window Y is made of two pieces of plastic with air between them.

glass plastic

vacuum air

window X window Y

Which window is better at stopping sound passing through and why?

window why?
A X Sound cannot travel through a vacuum.
B X Sound cannot travel through glass.
C Y Sound cannot travel through air.
D Y Sound cannot travel through plastic.

25 The diagram shows a student standing 165 m in front of a wall. He claps his hands once.

wall

165 m

How long after the handclap does he hear the echo?


[The speed of sound in air is 330 m/s.]

A 0.25 s B 0.50 s C 1.0 s D 2.0 s

0625/1 S00 [Turn over


12

26 A magnet is placed in a coil connected to an electrical supply.

magnet coil

electrical supply

What is the best way to demagnetise the magnet?

A Use a d.c. supply and leave the magnet inside the coil.
B Use a d.c. supply and slowly remove the magnet from the coil.
C Use an a.c. supply and leave the magnet inside the coil.
D Use an a.c. supply and slowly remove the magnet from the coil.

27 A magnet is brought near metal bars X and Y.

S N no effect X

S N Y
repulsion

What are X and Y made of?

X Y
A aluminium magnetised steel
B aluminium unmagnetised iron
C unmagnetised iron magnetised steel
D unmagnetised iron unmagnetised iron

0625/1 S00
13

28 Which diagram shows how a voltmeter and an ammeter are connected to find the value of the
resistance of resistor R?

A R V

B R V

A
R
C
V

V
D R

0625/1 S00 [Turn over


14

29 Two uncharged metal-coated polystyrene spheres, P and Q, are suspended side by side.

P Q

Both spheres are then given positive charges.

Which diagram shows the new positions of the spheres?

A B C D

P Q P Q P Q P Q

30 Component Y is placed in the circuit shown and is used to reduce the brightness of the lamp
gradually.

What is component Y?

A a fuse
B a relay
C a switch
D a variable resistor

0625/1 S00
15

31 In the circuit, which ammeter shows the greatest reading?

A
A

B
A

C
A

D
A

32 A lamp is connected across the mains supply.

Which diagram shows where the fuse should be connected?

A B
fuse
live live
fuse

neutral neutral

C D
live live
fuse

neutral neutral
fuse

33 An electric kettle takes 12 A of current from the mains supply. When the kettle’s cable needs to be
replaced, a 5 A cable is used that was originally attached to a small lamp. The original plug and
fuse from the kettle’s cable are used.

Why does this cause a problem?

A The cable becomes hot and melts the insulation.


B The fuse blows as soon as the kettle is switched on.
C The kettle becomes too hot.
D The water takes too long to reach its boiling point.

0625/1 S00 [Turn over


16

34 Which device makes use of the magnetic effect of an electric current?

A electric bell
B electric fire
C fuse
D lamp

35 The diagram shows a transformer, with 100 V applied to the primary coil.

100 V

primary coil secondary coil


(40 turns) (80 turns)

What is the voltage across the secondary coil?

A 50 V B 100 V C 200 V D 800 V

36 The diagram shows part of a cathode-ray oscilloscope.

particles
from cathode

cathode

Which particles are emitted by the cathode?

A alpha (α-) particles


B electrons
C neutrons
D protons

0625/1 S00
17

37 In the circuit shown, the brightness of the light reaching the light-dependent resistor increases.

light

light-dependant resistor

A ammeter

What happens to the current through the ammeter?

A It becomes zero.
B It decreases.
C It does not change.
D It increases.

38 Which statement about an alpha (α-) particle is correct?

A It can pass through thick paper.


B It has no charge.
C It is a helium nucleus.
D It is a type of electromagnetic radiation.

0625/1 S00 [Turn over


18

39 The graph shows the activity of a radioactive source plotted against time.

80

70

60

50

activity
40
counts/min
30

20

10

0
0 1 2 3 4 5
time/hours

After what time is the activity one quarter of its original value?

A 1 hour B 2 hours C 3 hours D 4 hours

40 A nuclide of cobalt is shown by the symbol 60


27
Co.

How many protons and neutrons are there in the nucleus of this nuclide?

protons neutrons
A 27 33
B 27 60
C 60 27
D 60 87

Go to answers

0625/1 S00
International General Certificate of Secondary Education
UNIVERSITY OF CAMBRIDGE LOCAL EXAMINATIONS SYNDICATE
PHYSICS 0625/1
PAPER 1 Multiple Choice
Tuesday 7 NOVEMBER 2000 Morning 45 minutes

Additional materials:
Electronic calculator and/or Mathematical tables
Multiple Choice answer sheet
Soft clean eraser
Soft pencil (type B or HB is recommended)

TIME 45 minutes

INSTRUCTIONS TO CANDIDATES
Do not open this booklet until you are told to do so.
Write your name, Centre number and candidate number on the answer sheet in the spaces provided
unless this has already been done for you.
There are forty questions in this paper. Answer all questions. For each question, there are four
possible answers, A, B, C and D. Choose the one you consider correct and record your choice in soft
pencil on the separate answer sheet.
Read very carefully the instructions on the answer sheet.

INFORMATION FOR CANDIDATES


Each correct answer will score one mark. A mark will not be deducted for a wrong answer.
Any rough working should be done in this booklet.

This question paper consists of 19 printed pages and 1 blank page.


SB (KN/TC) QK07573/3
© UCLES 2000 [Turn over
2

1 The diagram shows a measuring cylinder.

100
90
80
70
60
50
40
30
20
10

Which unit would be most suitable for its scale?

A mm2 B mm3 C cm2 D cm3

2 The diagrams show the times on a stopclock at the start and at the finish of an experiment.

stopclock stopclock
at start at finish

60 60
S S

45 15 45 15

30 30

How long did the experiment take?

A 10 s B 25 s C 35 s D 45 s

0625/1 W00
3

3 A ball is thrown up into the air. The diagram shows the ball’s path.

Ignoring air resistance, which statement about the acceleration of the ball is correct?

A It is least at point X.
B It is zero at point Y.
C It is greatest at point Z.
D It is the same at points X, Y and Z.

4 Which graph shows the speed of a stone, dropped from the top of a building, until it hits the
ground? (Assume there is no air resistance.)

A B C D
speed

speed

speed

speed

0 0 0 0
0 time 0 time 0 time 0 time

0625/1 W00 [Turn over


4

5 The diagram shows a book resting on a slope.

In which direction does the weight of the book act?

D C

6 An astronaut has a mass of 60 kg on the Moon.

What is the mass of this astronaut on Earth?

A 6 kg B 60 kg C 60 N D 600 N

7 The table shows information about four objects.

object mass/g volume/cm3


P 30 6
Q 40 5
R 50 10
S 50 4

Which two objects have the same density?

A P and Q B P and R C Q and S D R and S

0625/1 W00
5

8 A student tries to balance a 10 kg bag of rice on a pivoted beam, using a 5 kg bag of rice.

5 kg

10 kg
pivot

What should be done to balance the bags?

A add some rice to the 10 kg bag


B empty some rice out of the 5 kg bag
C move the pivot away from the 10 kg bag
D move the pivot towards the 10 kg bag

9 A hot-air balloon moves in the direction shown at constant speed and at constant height.
W, X, Y and Z are the forces acting on the balloon.

direction of
movement Y

X Z

Which statement about the forces is correct?

A Z is equal to X and W is equal to Y.


B Z is equal to X and W is less than Y.
C Z is less than X and W is equal to Y.
D Z is less than X and W is less than Y.

0625/1 W00 [Turn over


6

10 The diagram shows the main parts of a hydroelectric power station.

water

dam
wall
generator
turbine

Which energy change occurs in the generator?

A chemical to electrical
B electrical to chemical
C electrical to kinetic
D kinetic to electrical

11 Forces are used to move objects P, Q, R and S through the distances shown.

5N P

10 N Q

5N R

10 N S

Which statement correctly describes the work done by each force?

A Most work is done in moving R.


B Most work is done in moving S.
C The same amount of work is done in moving P and Q.
D The same amount of work is done in moving P and R.

0625/1 W00
7

12 The diagram shows a mercury barometer.

25 cm

mercury 75 cm

5 cm

Which distance can be used to find atmospheric pressure?

A 25 cm B 75 cm C 80 cm D 100 cm

13 A child has a set of identical building blocks.

Which arrangement produces the least pressure on the floor?

A
C
B
D

14 Liquid evaporates from an open dish.

Which molecules of liquid are most likely to escape?

A all molecules with a little energy


B all molecules with a lot of energy
C surface molecules with a little energy
D surface molecules with a lot of energy

0625/1 W00 [Turn over


8

15 A lorry driver checks the pressure of the air in the tyres when the temperature is 5 °C. After a long
journey, the tyres are hot and the pressure of the air in the tyres has increased.

Why has the pressure increased?

A Some air has leaked from the tyres during the journey.
B The air in the tyres is less dense.
C The average speed of the air molecules is greater.
D The tyres have expanded and have a larger volume.

16 Long pipes that carry steam often have bends in them, as shown.

Why are these bends needed?

A to allow for expansion


B to allow for pressure changes
C to make the pipes stronger
D to reduce the flow of steam

17 A solid substance is heated. The graph shows the change in temperature of the substance with
time.

At which stage does melting take place?

stage stage stage stage


A B C D

temperature

time

0625/1 W00
9

18 At the end of a long race, a runner is wrapped in a thin, plastic blanket that has a shiny, metallic
surface.

Which type of heat loss is the shiny surface intended to reduce?

A conduction
B convection
C evaporation
D radiation

19 A woman stands by a fire to warm her hands and legs.

How does most of the heat reach her hands and legs?

hands legs
A convection convection
B radiation convection
C convection radiation
D radiation radiation

0625/1 W00 [Turn over


10

20 The diagram shows waves in a ripple tank passing from deep water to shallow water.

deep water shallow water

wavefront

What happens to the speed and frequency of the waves?

speed frequency
A decreases decreases
B decreases unchanged
C increases increases
D unchanged increases

21 The diagram represents a wave.

displacement Q
P

distance
S

What is the amplitude and wavelength of the wave?

amplitude wavelength
A P Q
B P R
C S Q
D S R

0625/1 W00
11

22 A paper-clip is placed in front of a plane mirror.

paper-clip plane mirror

Which diagram shows the image formed behind the mirror?

A B C D

23 A ray of white light passes into each glass block as shown.

Which block produces dispersion of the light?

A B C D

0625/1 W00 [Turn over


12

24 The diagram shows apparatus used to find what happens to the sound from an electric bell as air
is removed from the container.

electric
bell

container

to vacuum
pump

What happens to the sound of the electric bell heard from outside the container?

A It becomes louder.
B It becomes quieter.
C It becomes quieter, then louder.
D It remains the same.

0625/1 W00
13

25 Two students stand 600 m apart.

student with student with


starting-pistol stopwatch

600 m

not to scale

They find that it takes 2 seconds for the sound from the starting pistol to travel from one student to
the other.

From these results, what is the speed of sound in air?

A 150 m/s B 300 m/s C 600 m/s D 1200 m/s

26 A piece of soft iron is placed in a coil. The switch is then closed.

soft iron

switch open

What is the condition of the soft iron when the switch is open and when the switch is closed?

switch open switch closed


A magnetised magnetised
B magnetised unmagnetised
C unmagnetised magnetised
D unmagnetised unmagnetised

0625/1 W00 [Turn over


14

27 When a magnet is brought near some metals, it induces magnetic poles in the metals.

Which diagram shows the poles induced when a magnet is brought near to an unmagnetised
piece of iron and to an unmagnetised piece of aluminium?

iron
A S N
N magnet S
S N
aluminium

iron
B S N
N magnet S
(no poles)
aluminium

iron
C N S
N magnet S
N S
aluminium

iron
D N S
N magnet S
(no poles)
aluminium

28 Which circuit shows how a voltmeter is connected to measure the potential difference across the
cell?

A B C D
V

V
V

0625/1 W00
15

29 Which diagram shows the directions of the electrostatic forces acting on two positively charged
spheres?

A B

+ + + +

C D
+ + + +

30 What is the correct symbol for a fuse?

31 In which circuit is lamp L1 lit, but lamp L2 unlit?

A B C D

L1 L2 L1 L2 L1 L2 L1 L2

0625/1 W00 [Turn over


16

32 The diagram shows the structure of a fuse.

metal end cap

ceramic case wire X

What is the purpose of wire X?

A to decrease the resistance of the circuit


B to increase the current in the circuit
C to keep the end caps on
D to melt when the current becomes too large

33 A wiring diagram for car headlamps and parking lamps is shown.

36 W

36 W

5W

5W

Why are the lamps connected in parallel?

A so that, if one lamp goes out, the others will remain lit
B so that the current in each lamp is the same
C so that the lamps operate at normal brightness, even when the battery runs down
D so that the voltage across each lamp is one quarter of the battery voltage

0625/1 W00
17

34 The diagram shows a step-up transformer.

input output
(few turns) (many turns)

What is stepped up by this type of transformer?

A current
B energy
C power
D voltage

35 Which diagram shows how a coil of wire and a magnet can be used to produce an electric current
through a meter?

magnet
A B C D
S S coil
movement glued
N on to
N magnet
coil
S S

N N

meter

0625/1 W00 [Turn over


18

36 The diagram shows part of an alarm circuit.

capacitor

What does the capacitor do?

A It detects changes in light intensity.


B It detects changes in temperature.
C It produces a magnetic field.
D It stores electrical energy.

37 The charges on the x-plates and y-plates of a cathode-ray oscilloscope cause the spot on the
screen to move to the position shown.

y-plate
screen

x-plate x-plate

spot

y-plate

Which diagram shows how the plates are charged?

A B C D
_ + + _

+ + _ _ + _ _ +

spot spot spot spot

_ + _ +

0625/1 W00
19

38 A piece of paper is placed between a radioactive source and a detector. What are the possible
radiations that can pass through the paper?

A alpha (α–) radiation only.


B alpha (α–) and gamma (γ–) radiation only.
C beta (β–) and gamma (γ–) radiation only.
D alpha (α–), beta (β–) and gamma (γ–) radiation.

39 The graph shows how the activity of a radioactive substance changes with time.

800

700

600

500
activity
counts/min
400

300

200

100

0
0 1 2 3 4 5
time/min

What is the half-life of the substance?

A 1 min B 2 min C 3 min D 4 min

40 The diagram represents a helium atom.

e key
e = electron

p p p = proton
n n n = neutron
= nucleus

What is the nucleon number (mass number) of the atom?

A 2 B 4 C 6 D 8
0625/1 W00
Go to answers
International General Certificate of Secondary Education
UNIVERSITY OF CAMBRIDGE LOCAL EXAMINATIONS SYNDICATE
PHYSICS 0625/1
PAPER 1 Multiple Choice
MAY/JUNE SESSION 2001 45 minutes

Additional materials:
Multiple Choice answer sheet
Soft clean eraser
Soft pencil (type B or HB is recommended)

TIME 45 minutes

INSTRUCTIONS TO CANDIDATES

Do not open this booklet until you are told to do so.


Write your name, Centre number and candidate number on the answer sheet in the spaces provided
unless this has already been done for you.
There are forty questions in this paper. Answer all questions. For each question, there are four
possible answers, A, B, C and D. Choose the one you consider correct and record your choice in
soft pencil on the separate answer sheet.
Read very carefully the instructions on the answer sheet.

INFORMATION FOR CANDIDATES

Each correct answer will score one mark. A mark will not be deducted for a wrong answer.
Any rough working should be done in this booklet.

This question paper consists of 19 printed pages and 1 blank page.


ND/CG QF10308/3
© UCLES 2001 [Turn over
2

1 A student measures the width of a book using a ruler.

Which diagram shows the best way to do this?

book

A 0 10 20 30 cm

B 0 10 20 30 cm

C 0 10 20 30 cm

D 0 10 20 30 cm

2 The diagrams show the times on a stopclock at the start and finish of an experiment.

stopclock stopclock
at start at finish
60 60

s s
45 15 45 15

30 30

How long did the experiment take?

A 10 s B 25 s C 35 s D 45 s

0625/1/M/J/01
3

3 An apple falls from a tree. The diagram shows the apple at P, as it starts to fall, and at Q, just
before it hits the ground. The acceleration due to gravity is g. Air resistance can be ignored.

What is the acceleration of the apple at position P and at position Q?

acceleration at P acceleration at Q

A 0 0
B 0 g
C g 0
D g g

4 In a race, a car travels 60 times round a 3.6 km track. It takes 2.4 hours.

What is the average speed of the car?


A 2.5 km / h
B 90 km / h
C 144 km / h
D 216 km / h

0625/1/M/J/01 [Turn over


4

5 A spring is stretched by hanging a piece of metal from it.

spring

metal

What is the name given to the force that stretches the spring?
A friction
B mass
C pressure
D weight

6 A child of mass 40 kg sits on one end of a see-saw. The pivot is at the centre of the see-saw.
There are four sacks of sand, each with a different mass, as shown.

40 kg

see – saw
X

30kg 20kg 10kg 5kg

pivot

How many of the sacks must be placed at X to balance the child?

A 1 B 2 C 3 D 4

7 An object has a mass of 75 g and a volume of 15 cm3.

What is its density?

A 0.2 g / cm3 B 5 g / cm3 C 60 g / cm3 D 90 g / cm3

0625/1/M/J/01
5

8 The diagrams show a brick resting on a smooth surface. Two equal forces F act on the brick.

In which diagram does the brick not move?

A B C D
F

F F F

F F F F

9 Four rings are screwed into a door, as shown. The door can be opened by putting a hook into
one of the rings and pulling.

B
A
C

pull

Which ring should be used if the pulling force is to be as small as possible?

0625/1/M/J/01 [Turn over


6

10 A stone is thrown into the air. The diagram shows the path of the stone through the air.

At which position is the potential energy of the stone greatest?

C
B

11 Four weightlifters lift weights to different heights.

Which weightlifter does the most work?

A B C D

1000 N lifted 1000 N lifted 1200 N lifted 1200 N lifted


through 1.0 m through 1.2 m through 1.0 m through 1.2 m

0625/1/M/J/01
7

12 The diagram shows a simple barometer.

Which distance should be measured to find the atmospheric pressure?

vacuum
A

mercury B C D

trough

13 Two boys X and Y each have the same total weight and are standing on snow.

X Y

Which boy is more likely to sink into the snow and why?

boy pressure on snow

A X larger than Y
B X smaller than Y
C Y larger than X
D Y smaller than X

0625/1/M/J/01 [Turn over


8

14 A student places his thumb firmly on the outlet of a bicycle pump, to stop the air coming out.

trapped air

direction of
motion

handle

What happens to the pressure and the volume of the trapped air as the pump handle is pushed
in?

pressure volume

A decreases decreases
B decreases remains the same
C increases decreases
D increases remains the same

15 To keep a bottle of milk cold without a refrigerator on a hot day, the bottle can be covered with a
damp cloth in a bowl of water.

damp cloth

bottle
milk
bowl
water

How does this method keep the milk cold?


A Milk condenses on the cloth.
B Milk evaporates from the cloth.
C Water condenses on the cloth.
D Water evaporates from the cloth.

0625/1/M/J/01
9

16 The diagram shows a liquid-in-glass thermometer.

0°C 100°C

liquid glass bulb stem

When the thermometer becomes hotter, the liquid moves further along the stem.

Why is this?
A The glass contracts.
B The glass expands.
C The liquid contracts.
D The liquid expands.

17 Two copper rods, A and C, and two copper tubes, B and D, have the same external diameter.

Which rod has the highest thermal capacity?

A
B
C
D

0625/1/M/J/01 [Turn over


10

18 On a cold night, two children sit next to a camp fire to warm their hands. Their hands are the
same distance from the fire. Child 1 holds his hands over the fire and child 2 holds her hands in
front of the fire.

child 1's hands

child 2's hands

How does the heat from the fire reach each child’s hands?

child 1 child 2

A convection only radiation only


B convection and radiation radiation only
C radiation only convection and radiation
D radiation only convection only

19 An electric kettle contains a metal heating element.

water

heating element

What are the main processes by which heat energy is transferred from the element to the water,
and throughout the water?

heat transfer process

element to water throughout water

A conduction convection
B convection radiation
C radiation conduction
D radiation convection

0625/1/M/J/01
11

20 The diagrams show four sources of waves.

Which source generates longitudinal waves?

A B C D

stick pushed up radio loudspeaker lamp


and down in water transmitter

21 The diagram shows a wave on the surface of some water.

At which two points are the molecules moving in the same vertical direction at the same time?

P Q
R T
S

A P and Q
B P and T
C Q and T
D R and S

0625/1/M/J/01 [Turn over


12

22 The diagrams show a semi-circular glass block.

Which diagram correctly shows the path of the ray of light and the critical angle c?

A B

C D

c
c

23 A student copies a diagram of the electromagnetic spectrum but makes a mistake.

radio micro- infra-red visible X- ultra-violet gamma


waves waves waves light rays waves rays

long wavelength short wavelength

Which two names should be interchanged so that the order is correct?


A infra-red waves and radio waves
B infra-red waves and ultra-violet waves
C radio waves and visible light
D ultra-violet waves and X-rays

24 An electric bell is ringing inside a glass dome.

In which situation would the bell sound quietest?

A B C D
electric electric
bell bell

glass glass
dome dome

vacuum air oxygen carbon


dioxide

0625/1/M/J/01
13

25 In a test, a car horn is found to be too loud and the pitch of the note is too high. What information
does this give about the amplitude and the frequency of the sound wave produced?

amplitude frequency

A too large too large


B too large too small
C too small too large
D too small too small

26 The diagrams show a magnetised piece of steel being moved slowly to the left.

Which diagram shows the best method of demagnetising the steel?

A B

d.c. a.c.

steel steel

C D
S magnet N magnet

N S

steel steel

0625/1/M/J/01 [Turn over


14

27 The diagram shows some nails attracted to a metal rod by magnetic induction.

S permanent
magnet
N

metal
rod

nails

From what could the metal rod be made?


A aluminium
B copper
C iron
D magnesium

28 What is the unit of potential difference?


A joule
B newton
C volt
D watt

0625/1/M/J/01
15

29 A plastic strip is rubbed on a piece of cloth and then held near water running slowly from a tap.
The water moves towards the plastic strip.

tap

plastic strip

water

Why does this happen?


A The plastic strip cools the water.
B The plastic strip warms the water.
C There is a magnetic force on the water.
D There is an electrostatic force on the water.

30 The diagrams show part of an electric circuit containing an ammeter and a voltmeter.

Which arrangement should be used to measure the p.d. across the resistor and the current
through it?

A B
A

A V V

C D

V
V

A
A

0625/1/M/J/01 [Turn over


16

31 The circuits contain identical lamps and identical cells.

In which circuit will the lamps be brightest?

A B C D

32 Why are lamps in the lighting circuit of a house connected in parallel and not in series?
A It allows them to be switched on and off independently.
B It uses less electrical energy.
C The filaments are less likely to burn out.
D The p.d. across each lamp is reduced.

33 A builder plugs an electric drill into a socket inside a house.

house

electric cable

He uses the drill outdoors. It starts to rain heavily.

Why is it dangerous to continue using the electric drill in the rain?


A The drill could give the builder an electric shock.
B The drill could overheat.
C The fuse could blow.
D The rain could rust the drill.

0625/1/M/J/01
17

34 A wire is placed between the poles of a magnet and is briefly connected to a cell. It experiences
a downwards force.

downwards
force

magnet

The cell is now reversed so that it is connected the other way round.

What happens to the direction of the magnetic field of the magnet and to the direction of the
force on the wire?

direction of magnetic field direction of force

A reversed reversed
B reversed unchanged
C unchanged reversed
D unchanged unchanged

35 The diagram represents a transformer.

input output
P S
voltage voltage

Which arrangement would make the output voltage higher than the input voltage?

number of turns on number of turns on type of input


primary coil P secondary coil S voltage

A 50 100 a.c.
B 50 100 d.c.
C 100 50 a.c.
D 100 50 d.c.

0625/1/M/J/01 [Turn over


18

36 Which circuit will act as a variable potential divider?

A B

output

input input
output

C D

input input output


output

37 The circuit shown contains a light dependent resistor (LDR) and an ammeter, in series with a
battery.

How does the circuit behave when more light shines on it?

resistance of LDR current through ammeter

A decreases decreases
B decreases increases
C increases decreases
D increases increases

0625/1/M/J/01
19

38 A thick sheet of paper is placed between a radioactive source and a detector.

radioactive
detector
source sheet of paper

Which types of radiation can pass through the paper?


A alpha-particles, beta-particles and gamma radiation
B alpha-particles and gamma radiation
C alpha-particles and beta-particles
D beta-particles and gamma radiation

39 The count rate of radiation produced by a radioactive sample is measured every minute. The
results are recorded in the table.

time / minutes count rate / per second

0 80
1 56
2 40
3 28
4 20
5 14

What is the half-life of the radioactive material?

A  minute B 2 minutes C 2  minutes D 5 minutes

40 A nuclide of lithium contains 3 protons and 4 neutrons.

Which symbol represents this nuclide?

A 4 Li 3 Li 7 Li 3 Li
3 B 4 C 3 D 7

Go to answers

0625/1/M/J/01
International General Certificate of Secondary Education
UNIVERSITY OF CAMBRIDGE LOCAL EXAMINATIONS SYNDICATE
PHYSICS 0625/1
PAPER 1 Multiple Choice
OCTOBER/NOVEMBER SESSION 2001 45 minutes

Additional materials:
Multiple Choice answer sheet
Soft clean eraser
Soft pencil (type B or HB is recommended)

TIME 45 minutes

INSTRUCTIONS TO CANDIDATES

Do not open this booklet until you are told to do so.


Write your name, Centre number and candidate number on the answer sheet in the spaces provided
unless this has already been done for you.
There are forty questions in this paper. Answer all questions. For each question, there are four
possible answers, A, B, C and D. Choose the one you consider correct and record your choice in
soft pencil on the separate answer sheet.
Read very carefully the instructions on the answer sheet.

INFORMATION FOR CANDIDATES

Each correct answer will score one mark. A mark will not be deducted for a wrong answer.
Any rough working should be done in this booklet.

This question paper consists of 21 printed pages and 3 blank pages.


SPA (ND/CG) S09093/4
© UCLES 2001 [Turn over
2

1 The diagram shows a measuring cylinder.

cm3
10

What volume of liquid does the cylinder contain?

A 5.5 cm3 B 5.7 cm3 C 6.5 cm3 D 6.7 cm3

2 The diagram shows a thick-walled tube. The thickness of the wall is 3 mm.

0 cm 1 2 3 4 5 6 7 8 9 10 11

What is the internal diameter d of the tube?

A 2.8 cm B 3.1 cm C 3.4 cm D 7.4 cm

0625/1/O/N/01
3

3 An object moves initially with constant speed and then with constant acceleration.

Which graph shows this motion?

A B

speed speed

0 0
0 0
time time

C D

speed speed

0 0
0 0
time time

4 A motorist travels 200 km.

After travelling along a fast road for 2 hours, the motorist uses a slow road for the remaining
 hour of the journey.

start

fast road 2 hours


slow road 1/2 hour

finish

What is the average speed of the car?

A 80 km / h B 100 km / h C 400 km / h D 500 km / h

0625/1/O/N/01 [Turn over


4

5 Which of the following has the same unit as weight?


A energy
B force
C mass
D power

6 The mass of an astronaut is 70 kg on the Moon.

What is the astronaut’s mass on Earth?

A 7 kg B 70 kg C 80 kg D 700 kg

7 Which of the following is a unit of density?


A cm3 / g
B g / cm2
C g / cm3
D kg / m2

8 A student carries out an experiment to plot an extension/load graph for a spring. The diagrams
show the apparatus at the start of the experiment and with a load added.

start with load added

x y

What is the extension caused by the load?

A x B y C y+x D y–x

0625/1/O/N/01
5

9 Diagram X shows the directions of the horizontal forces acting on a van when it is moving
forward at constant speed.

diagram X
(constant speed)

CIE
force Q force R

Diagram Y shows the directions of the horizontal forces acting on the same van when it is
accelerating.

diagram Y
(accelerating)

CIE
force S force T

Which of the following describes the sizes of the forces Q, R, S and T?

diagram X diagram Y
(constant speed) (accelerating)

A Q less than R S less than T


B Q less than R S equal to T
C Q equal to R S less than T
D Q equal to R S equal to T

0625/1/O/N/01 [Turn over


6

10 A pole-vaulter runs up to a jump with his pole straight. He puts one end of the pole down on the
ground and the pole bends.

before after

pole

Which form of energy is stored in the pole because it is bent?


A chemical
B gravitational
C motion
D strain

11 A ball is allowed to roll down a slope, as shown in the diagram. There is no friction.

At which point does the ball have the greatest energy of motion (kinetic energy)?

start

C
B
D

0625/1/O/N/01
7

12 A pin is squeezed between finger and thumb.

finger
pinhead
pin

thumb

Which statement is correct?


A The force of the pin is larger on the finger than on the thumb.
B The force of the pin is larger on the thumb than on the finger.
C The pressure of the pin is larger on the finger than on the thumb.
D The pressure of the pin is larger on the thumb than on the finger.

13 The diagram shows a manometer attached to an inflated balloon.

balloon

mercury

What does the difference between the mercury levels show?


A The air pressure inside the balloon is greater than atmospheric pressure.
B The air pressure outside the balloon is greater than the pressure inside the balloon.
C The air pressure in the left and right sides of the tube is equal.
D The air pressure in the left side of the tube is greater than the pressure on the right side.

0625/1/O/N/01 [Turn over


8

14 A gas is heated at constant volume.

What happens to its pressure and the speed of its molecules?

gas pressure speed of molecules

A decreases decreases
B decreases increases
C increases decreases
D increases increases

15 The table lists the melting points and the boiling points of four different substances.

Which substance is a gas at 25 °C?

melting point / °C boiling point / °C

A -219 -183
B -7 58
C 98 890
D 1083 2582

16 In an experiment, a piece of melting ice and a beaker of boiling water are in a laboratory which is
at 20 °C.

boiling water

melting ice
Bunsen burner
heating water

What is happening to the temperature of the melting ice and of the boiling water?

temperature of the temperature of the


melting ice boiling water

A constant constant
B constant increasing
C increasing constant
D increasing increasing

0625/1/O/N/01
9

17 The glass stopper in the neck of a glass bottle is too tight to come out.

Which diagram shows the best way to loosen the stopper?

A B C D

in a stream of in a stream of inside a warm inside a refrigerator


cold water for a hot water for a oven for a for a long time
short time short time long time

18 A camper warms her hands near a gas lamp.

Which waves carry most heat to her hands, and are these waves electromagnetic?

waves electromagnetic

A infra-red no
B infra-red yes
C light no
D light yes

0625/1/O/N/01 [Turn over


10

19 An experiment is set up to find out which metal is the best conductor of heat. Metal balls are
stuck with wax to rods made of different metals, as shown in diagram X.

The rods are heated at one end. Some of the balls fall off, leaving some as shown in diagram Y.

Which metal is the best conductor of heat?

diagram X diagram Y
heat heat
D

A
before heating after heating

20 Plane water waves are approaching a narrow gap in a barrier.

Which diagram shows the diffraction pattern?

A B

C D

0625/1/O/N/01
11

21 The diagram shows four waves drawn to the same scale.

P Q

0 0
0 distance 0
distance

R S

0 0
0 0
distance distance

Which statement is correct?


A The amplitude of wave P is the same as the amplitude of wave R.
B The amplitude of wave S is double the amplitude of wave Q.
C The wavelength of wave Q is double the wavelength of wave P.
D The wavelength of wave S is the same as the wavelength of wave Q.

22 Which diagram shows how a ray of light passes through a glass prism in a periscope?

A B C D

0625/1/O/N/01 [Turn over


12

23 The diagram shows a section of the electromagnetic spectrum.

gamma-rays X-rays ultra-violet rays

visible light

How do the frequency and the speed of gamma rays compare with those of visible light?

frequency speed in a vacuum

A gamma greater same


B visible greater same
C gamma greater gamma faster
D visible greater visible faster

24 Which change would make a sound louder?


A decreasing the amplitude of the wave
B increasing the amplitude of the wave
C decreasing the wavelength
D increasing the wavelength

0625/1/O/N/01
13

25 A lighted candle is put in front of a loudspeaker which is making a loud, steady note. The flame
vibrates because of the sound wave.

loudspeaker candle

Which type of waves are sound waves and in which direction does the flame vibrate?

type of wave direction of vibration

A longitudinal

B transverse

C longitudinal

D transverse

26 When a magnet is brought close to an unmagnetised piece of iron, the iron becomes
magnetised.

Which diagram correctly shows the magnetic poles induced in the iron?

A B C D

S S S S
magnet magnet

N N N N

N S
N S S N
S N

iron iron iron iron

0625/1/O/N/01 [Turn over


14

27 Two rods X and Y look the same.

P magnet

rod X N S
Q

rod Y
S

The N pole of a magnet is brought close, in turn, to each end of both rods. The results of these
four actions are shown in the table.

end tested result

P attraction
Q attraction
R attraction
S repulsion

Which of the rods is a magnet?


A neither of the rods
B both of the rods
C rod X only
D rod Y only

0625/1/O/N/01
15

28 Which circuit could be used to find the resistance of resistor R?

A B

V
R

R
V A A

C D

A
R R
V

V A

29 A polythene rod repels an inflated balloon hanging from a nylon thread.

What charges must the rod and the balloon carry?


A The rod and the balloon carry opposite charges.
B The rod and the balloon carry like charges.
C The rod is charged but the balloon is not.
D The balloon is charged but the rod is not.

30 An electrical component is to be placed in the circuit at Z, to allow the brightness of the lamp to
be varied from bright to dim.

What should be connected at Z?

A B C D

0625/1/O/N/01 [Turn over


16

31 A student uses four ammeters to measure the current in different parts of a circuit.

P Q
A A

R 10 Ω
A

S 20 Ω
A

Which two ammeters will each read the largest current?


A P and Q
B P and R
C R and Q
D R and S

32 Four lamps are each labelled 240 V, 60 W.

In which circuit would all four lamps have the correct brightness?

A B

240 V 240 V

C D

240 V 240 V

0625/1/O/N/01
17

33 A student makes the circuit shown.

5 A fuse

The fuse has blown and stopped the current.

What could have caused this?


A A 3 A fuse should have been used.
B The lamp was loose.
C The current was too large.
D The voltage was too small.

34 A coil is connected to a battery and a soft-iron bar is hung near to it.

soft iron
coil

The current is then reversed by reversing the battery connections.

How does the soft iron bar behave in the two cases?

with the battery as shown with the battery reversed

A attracted to the coil attracted to the coil


B attracted to the coil repelled from the coil
C repelled from the coil attracted to the coil
D repelled from the coil repelled from the coil

0625/1/O/N/01 [Turn over


18

35 A wire carries an electric current between the poles of a magnet. This causes a force that
pushes the wire upwards.

force on wire

S N

current into
page

The poles of the magnet and the direction of the current are both reversed.

Which arrow now shows the direction of the force on the wire?

A
force on wire

N D B S

current out
C of page

0625/1/O/N/01
19

36 Which graph shows how a thermistor could behave?

A B

resistance resistance

0 0
0 brightness of light 0 brightness of light

C D

resistance resistance

0 0
0 temperature 0 temperature

37 The diagram shows a circuit which includes a switch and a capacitor.

switch
X

capacitor

What happens to the capacitor when the switch is at X and when it is at Y?

switch at X switch at Y

A charging charging
B charging discharging
C discharging charging
D discharging discharging

0625/1/O/N/01 [Turn over


20

38 The graph shows the activity of a radioactive source over a period of time.

120
activity/
counts per 90
second
60

30

0
0 1 2 3 4 5
time/minutes

What is the half-life of the source?

A 1 minute B 2 minutes C 2  minutes D 4 minutes

39 The diagram shows five atoms in a radioactive substance. The atoms give out alpha-particles.

1st particle

atom
1

atom atom
5 2

atom atom
4 3

2nd particle

Atom 1 is the first to give out a particle. Atom 3 is the second to give out a particle.

Which atom will give out the next particle?


A atom 2
B atom 4
C atom 5
D it is impossible to tell

0625/1/O/N/01
21

40 The diagram shows the particles in an atom. Charged particles are marked + or –.

What is the nucleon number (mass number) of the atom?

A 4 B 5 C 9 D 13

Go to answers

0625/1/O/N/01
w
w
w
.X
tr
me
eP
ap
er
s.c
om
International General Certificate of Secondary Education
CAMBRIDGE INTERNATIONAL EXAMINATIONS
PHYSICS 0625/1
PAPER 1 Multiple Choice
OCTOBER/NOVEMBER SESSION 2002
45 minutes
Additional materials:
Multiple Choice answer sheet
Soft clean eraser
Soft pencil (type B or HB is recommended)

TIME 45 minutes

INSTRUCTIONS TO CANDIDATES

Do not open this booklet until you are told to do so.


Write your name, Centre number and candidate number on the answer sheet in the spaces provided
unless this has already been done for you.
There are forty questions in this paper. Answer all questions. For each question, there are four
possible answers, A, B, C and D. Choose the one you consider correct and record your choice in
soft pencil on the separate answer sheet.
Read very carefully the instructions on the answer sheet.

INFORMATION FOR CANDIDATES

Each correct answer will score one mark. A mark will not be deducted for a wrong answer.
Any rough working should be done in this booklet.

This question paper consists of 18 printed pages and 2 blank pages.


SP (AT/KN) S24280/2
© CIE 2002 [Turn over
2

1 The diagram shows a stopwatch, originally set at 00:00.

When a car was first seen, the stop-start button was pressed. When the car passed the observer,
the stopwatch showed 01:06.

stop-start button
min s
01 : 06

How long did the car take to reach the observer?

A 1.06 seconds
B 6 seconds
C 66 seconds
D 106 seconds

2 A girl uses a rule to measure the length of a metal rod. Because the end of the rule is damaged,
she places one end of the rod at the 1 cm mark as shown.

metal rod

damaged
end 1 2 3 4 5 6 7
cm

How long is the metal rod?

A 43 mm B 46 mm C 53 mm D 56 mm

0625/1/O/N/02
3

3 A stone falls freely from the top of a cliff into the sea. Air resistance may be ignored.

Which graph shows how the acceleration of the stone varies with time as it falls?

A B

acceleration acceleration

0 0
0 time 0 time

C D

acceleration acceleration

0 0
0 time 0 time

4 A child is standing on the platform of a station, watching the trains.

A train travelling at 30 m / s takes 3 s to pass the child.

What is the length of the train?

A 10 m B 30 m C 90 m D 270 m

5 Which of the following statements is correct?

A Mass and weight are different names for the same thing.
B The mass of an object is different if the object is taken to the Moon.
C The weight of a car is one of the forces acting on the car.
D The weight of a chocolate bar is measured in kilograms.

0625/1/O/N/02 [Turn over


4

6 In an experiment, six identical bags of flour are balanced by a 9 kg mass.

six bags 9 kg mass


of flour

pivot

Two bags of flour are removed. What mass will balance the remaining bags?

A 3 kg B 6 kg C 7 kg D 9 kg

7 The masses of a measuring cylinder before and after pouring some liquid are shown in the
diagram.

cm3 cm3

100 100

liquid
50 50

mass = 140 g mass = 217 g

What is the density of the liquid?

A 217 g / cm3 B 217 g / cm3 C 77 g / cm3 D 77 g / cm3


52 70 52 70

8 In which of these situations is no resultant force needed?

A a car changing direction


B a car moving in a straight line at a steady speed
C a car slowing down
D a car speeding up

0625/1/O/N/02
5

9 The diagram shows sections of four objects, all of equal mass. The position of the centre of mass
of each object has been marked with a cross.

Which object is the most stable?

A B C D

10 A car accelerates along a road as it rises uphill.

Which energy changes are taking place?

energy of motion energy of position


(kinetic energy) (gravitational potential energy)

A decreasing decreasing

B decreasing increasing

C increasing decreasing

D increasing increasing

11 In a car engine, energy stored in the fuel is converted into thermal energy (heat energy) and
energy of motion (kinetic energy).

In which form is the energy stored in the fuel?

A chemical
B geothermal
C hydroelectric
D nuclear

0625/1/O/N/02 [Turn over


6

12 The diagram shows a simple mercury barometer.

vacuum

level X

level Y
mercury

If atmospheric pressure increases, what happens to level X and to level Y?

level X level Y

A goes down goes down

B goes down goes up

C goes up goes down

D goes up goes up

13 Four flower vases have circular bases. They are filled with water so that they all have the same
weight.

Which vase exerts the greatest pressure on its base?

A B C D

14 Some gas in a sealed plastic bag is placed in a refrigerator to cool down.

How do the gas molecules behave when this happens?

A They move more quickly and are closer together.


B They move more quickly and are further apart.
C They move more slowly and are closer together.
D They move more slowly and are further apart.

0625/1/O/N/02
7

15 A measured mass of gas is placed in a cylinder at atmospheric pressure and is then slowly
compressed.

piston

gas
piston pushed in

If the temperature of the gas does not change, what happens to the pressure of the gas?

A It drops to zero.
B It decreases, but not to zero.
C It stays the same.
D It increases.

16 A mercury-in-glass thermometer is to be used to measure temperatures from 0 °C to 100 °C.

Why is mercury suitable?

A Mercury expands when heated.


B Mercury has a boiling point below 100 °C.
C Mercury has a melting point above 0 °C.
D Mercury is a poor conductor.

17 A block of ice is heated at a constant rate. Eventually the melted ice boils.

The graph shows how the temperature changes with time.

100
temperature / °C

0 5 10
time / minutes

How many minutes did it take to melt all the ice?

A 4 B 7 C 11 D 13

0625/1/O/N/02 [Turn over


8

18 How does thermal energy (heat energy) travel through the vacuum between the Earth and the
Sun?

A by conduction
B by convection
C by radiation
D by radioactive decay

19 Two plastic cups are placed one inside the other. Hot water is poured into the inner cup and a lid
is put on top as shown.

lid

small spacer
small air gap
hot water
bench

Which statement is correct?

A Heat loss by radiation is prevented by the small air gap.


B No heat passes through the sides of either cup.
C The bench is heated by convection from the bottom of the outer cup.
D The lid is used to reduce heat loss by convection.

0625/1/O/N/02
9

20 Waves are sent along two long springs X and Y as shown.

spring X
direction of
side to wave travel
side
movement

backward spring Y
and direction of
wave travel
forward
movement

How should the wave motions in X and Y be described?

spring X spring Y

A longitudinal longitudinal

B longitudinal transverse

C transverse longitudinal

D transverse transverse

21 Radio waves are received at a house at the bottom of a hill.

hill

The waves reach the house because the hill has caused them to be

A diffracted.
B radiated.
C reflected.
D refracted.

0625/1/O/N/02 [Turn over


10

22 One of the effects of passing a ray of white light through a prism is to split the light into colours.

prism

white light
coloured light

What is the name given to this effect?

A deviation
B dispersion
C reflection
D refraction

23 A student looks at the letter P on a piece of paper, and at its reflection in a mirror.

What does he see?

A B C D
mirror mirror

paper paper

24 Which of the following can be heard by the human ear?

A A whistle emitting a wave of frequency 50 kHz.


B A bat emitting a wave of frequency of 30 kHz.
C An insect emitting a wave of 300 Hz.
D A vibrating spring emitting a wave of frequency of 5 Hz.

0625/1/O/N/02
11

25 Two sound waves P and Q are displayed on an oscilloscope with the same time-base and
Y-plate settings for each.

P Q

Which statement correctly describes the pitch and the loudness of the two sounds?

A P has a higher pitch and is louder than Q.


B P has a higher pitch and is quieter than Q.
C P has a lower pitch and is louder than Q.
D P has a lower pitch and is quieter than Q.

26 A piece of magnetised steel is placed inside a coil of wire that has a large alternating current
passing through it. The magnet is slowly moved out of the coil to position P.

steel magnet moved


out of coil P

12 V a.c.

How has the steel changed when it reaches P?

A It has become demagnetised.


B There has been no change.
C It has become a stronger magnet.
D The poles have changed ends.

0625/1/O/N/02 [Turn over


12

27 A permanent magnet is placed close to a bar of soft iron PQ.

permanent
S N P Q soft-iron bar
magnet

What happens?

A P becomes a north pole.


B P becomes a south pole.
C PQ does not become magnetised.
D The poles of the magnet are reversed.

28 In which circuit does the ammeter read the total current through both resistors?

B
A

A
A

C D

0625/1/O/N/02
13

29 The table shows the voltage and current ratings for four light bulbs.

Which bulb has the greatest resistance when used normally?

voltage / V current / A

A 2 0.5

B 3 0.2

C 6 12

D 12 1.0

30 In the following circuits, the resistors have the same value and the cells are identical.

Which circuit has the smallest resistance?

A B

C D

31 The diagram shows a circuit, with four possible positions to place a switch.

A lamp 1
B
C

D
lamp 2

At which labelled point should a switch be placed so that lamp 1 remains on all the time and
lamp 2 can be switched on and off?

0625/1/O/N/02 [Turn over


14

32 The device X in this circuit is designed to cut off the electricity supply automatically if too much
current flows.

What is device X?

A a fuse
B a relay
C a resistor
D an ammeter

33 A classroom has four lights, each fitted with a lamp marked 240 V, 100 W.

Which circuit is most suitable for the classroom?

A B C D

240 V 240 V

240 V 240 V

0625/1/O/N/02
15

34 Which graph shows the output of a simple a.c. generator?

A B

e.m.f. e.m.f.

0 0
time time

C D

e.m.f. e.m.f.

0 0
time time

0625/1/O/N/02 [Turn over


16

35 When the electric current in wire XY is in the direction shown, there is an upward force on the
wire.

current

force

magnet magnet

N S

If the north and south poles of the magnet exchange positions, in which direction will the force on
the wire act?

A downwards
B upwards
C to the left
D to the right

36 Charged particles are emitted from the cathode of an oscilloscope.

What is the name and charge of these particles?

name of particles charge of particles

A electrons negative

B electrons positive

C protons negative

D protons positive

0625/1/O/N/02
17

37 Diagram 1 shows a potential divider circuit containing two 100 Ω resistors.

100 Ω

100 Ω V

Diagram 1

One of the resistors is changed to 90 Ω, as shown in diagram 2.

90 Ω

100 Ω V

Diagram 2

How does the reading on the voltmeter change when this is done?

A It becomes zero.
B It decreases a little.
C It increases a little.
D It stays the same.

38 A radioactive source emits radiation that can pass through a sheet of paper but not through thick
aluminium.

paper thick aluminium

radiation

all radiation no radiation


passes through passes through

What does this show about the radiation?

A It is alpha-particles.
B It is beta-particles.
C It is gamma-rays.
D It is a mixture of alpha-particles and gamma-rays.

0625/1/O/N/02 [Turn over


18

39 A sample of a radioactive isotope is decaying.

Which atoms will decay first?

A impossible to know, because radioactive decay is random


B impossible to know, unless the age of the material is known
C atoms near the centre, because they are surrounded by more atoms
D atoms near the surface, because the radiation can escape more easily

40 Which line in the table gives the numbers of protons and neutrons in the nuclide 73 Li?

protons neutrons

A 3 4

B 3 7

C 4 3

D 7 3

0625/1/O/N/02
19

BLANK PAGE

0625/1/O/N/02
20

BLANK PAGE

0625/1/O/N/02
CAMBRIDGE INTERNATIONAL EXAMINATIONS
International General Certificate of Secondary Education
PHYSICS 0625/01
Paper 1 Multiple Choice
May/June 2003

45 minutes
Additional Materials: Multiple Choice Answer Sheet
Soft clean eraser
Soft pencil (type B or HB is recommended)

READ THESE INSTRUCTIONS FIRST

Write in soft pencil.


Do not use staples, paper clips, highlighters, glue or correction fluid.
Write your name, Centre number and candidate number on the Answer Sheet in the spaces provided
unless this has been done for you.

There are forty questions on this paper. Answer all questions. For each question there are four possible
answers A, B, C, and D.
Choose the one you consider correct and record your choice in soft pencil on the separate Answer Sheet.

Read the instructions on the Answer Sheet very carefully.

Each correct answer will score one mark. A mark will not be deducted for a wrong answer.
Any rough working should be done in this booklet.

This document consists of 18 printed pages and 2 blank pages.


SP (SLC/KN) S45991/2
© CIE 2003 [Turn over
2

1 A glass tank contains some water.

V
water
T
Q
U
S

The length QR and the width RS of the tank are known.

What other distance needs to be measured in order to be able to calculate the volume of the
water?

A ST B SV C TU D TV

2 A stopwatch is used to time a race. The diagrams show the watch at the start and at the end of the
race.

start end

60 60
55 5 55 5
50 10 50 10

45 15 45 15

40 seconds 20 40 seconds 20
35 25 35 25
30 30

How long did the race take?

A 45.7 s B 46.0 s C 46.5 s D 47.0 s

0625/01/M/J/03
3

3 The diagram shows a speed-time graph for a body moving with constant acceleration.

speed

0
0 time

What is represented by the shaded area under the graph?

A acceleration
B distance
C speed
D time

4 A tunnel has a length of 50 km. A car takes 20 min to travel between the two ends of the tunnel.

What is the average speed of the car?

A 2.5 km / h
B 16.6 km / h
C 150 km / h
D 1000 km / h

5 Which statement is correct?

A Mass is a force, measured in kilograms.


B Mass is a force, measured in newtons.
C Weight is a force, measured in kilograms.
D Weight is a force, measured in newtons.

0625/01/M/J/03 [Turn over


4

6 Three children, X, Y and Z, are using a see-saw to compare their weights.

X Y Y Z X Z

Which line in the table shows the correct order of the children’s weights?

heaviest ←→ lightest


A X Y Z
B X Z Y
C Y X Z
D Y Z X

7 What apparatus is needed to determine the density of a regularly-shaped block?

A a balance and a ruler


B a balance and a forcemeter (spring balance)
C a measuring cylinder and a ruler
D a measuring cylinder and a beaker

8 A spring is suspended from a stand. Loads are added and the extensions are measured.

spring

stand
loads rule

Which graph shows the result of plotting extension against load?

A B C D
extension

extension

extension

extension

0 0 0 0
0 load 0 load 0 load 0 load

0625/01/M/J/03
5

9 A student uses a stand and clamp to hold a flask of liquid.

Which diagram shows the most stable arrangement?

A B C D

10 What is the source of the energy converted by a hydro-electric power station?

A hot rocks
B falling water
C oil
D waves

11 A labourer on a building site lifts heavy concrete blocks onto a lorry. Lighter blocks are now lifted
the same distance in the same time.

What happens to the work done in lifting each block and the power exerted by the labourer?

work done in power exerted by


lifting each block labourer
A decreases decreases
B decreases remains the same
C increases increases
D remains the same increases

0625/01/M/J/03 [Turn over


6

12 The diagram shows an instrument used to measure gas pressure.

liquid

What is the instrument called?

A ammeter
B barometer
C manometer
D thermometer

13 The diagrams show two divers swimming in the sea and two divers swimming in fresh water. Sea
water is more dense than fresh water.

On which diver is there the greatest pressure?

0m 0m
A sea water C fresh water

2m 2m

4m 4m
B D

6m 6m

14 When water evaporates, some molecules escape.

Which molecules escape?

A the molecules at the bottom of the liquid with less energy than others
B the molecules at the bottom of the liquid with more energy than others
C the molecules at the surface with less energy than others
D the molecules at the surface with more energy than others

0625/01/M/J/03
7

15 Two metal boxes containing air are standing in a room. Box X is on top of a heater. Box Y is on a
bench. The boxes are left for a long time.

X Y

heater bench

Which line in the table best describes the average speed of the molecules in the containers?

box X box Y
A fast zero
B fast slow
C slow fast
D zero fast

16 The top of the mercury thread in a mercury-in-glass thermometer reaches point X at 0 °C and
point Z at 100 °C.

Z
Y

X
W

Where might it be at a temperature below the ice-point?

A point W
B point X
C point Y
D point Z

0625/01/M/J/03 [Turn over


8

17 The same quantity of heat energy is applied to four different blocks. The temperature rise
produced is shown on each block.

Which block has the highest thermal capacity?

A B

temperature
temperature
rise is
rise is
3 °C
6 °C

C D

temperature
temperature rise is
rise is 18 °C
9 °C

18 A person holds a glass beaker in one hand and fills it quickly with hot water. It takes several
seconds before his hand starts to feel the heat.

Why is there this delay?

A Glass is a poor conductor of heat.


B Glass is a good conductor of heat.
C Water is a poor conductor of heat.
D Water is a good conductor of heat.

0625/01/M/J/03
9

19 The diagram shows a heater used to heat a tank of cold water.

water

lagging

tank
heater

What is the main process by which heat moves through the water?

A conduction
B convection
C evaporation
D radiation

20 What causes refraction when light travels from air into glass?

A The amplitude of the light waves changes.


B The colour of the light changes.
C The frequency of the light waves changes.
D The speed of the light changes.

21 A woman tunes her radio to a station broadcasting on 200 m.

What does the 200 m tell her about the radio wave?

A its amplitude
B its frequency
C its speed
D its wavelength

0625/01/M/J/03 [Turn over


10

22 Which statement is correct about the speed of electromagnetic waves in a vacuum?

A Ultra-violet waves have the greatest speed.


B Visible light waves have the greatest speed.
C Infra-red waves have the greatest speed.
D All electromagnetic waves have the same speed.

23 Which diagram correctly shows rays passing through a camera lens?

A camera B camera

film film

object object

lens image lens image

C camera D camera

film film

object object

lens image lens image

0625/01/M/J/03
11

24 A sound wave passes through the air, in the direction shown.

→
direction of travel of sound wave

How does a particle of air move as the sound wave passes?

A moves to the right and stays there •→

B moves left and right ←•→



C moves up and stays there •

D moves up and down •

25 A boy is stranded on an island 500 m from the shore.

500 m
cliffs

island

He shouts for help, but all he can hear in reply is the echo of his shout from some cliffs.

Sound travels at 340 m / s through the air.

What is the time interval between the boy shouting and hearing the echo?
500 2 × 500 340 2 × 340
A s B s C s D s
340 340 500 500

0625/01/M/J/03 [Turn over


12

26 A student wishes to use a magnetising coil to make a permanent magnet from a piece of metal.

metal

Which metal should she use?

A aluminium
B copper
C iron
D steel

27 A metal rod XY is placed near a magnet. End X is attracted when it is placed near to the north pole
of the magnet, and also when it is placed near to the south pole.

X Y
N N
attraction
X Y
S S
attraction

How does end Y behave when it is placed, in turn, near to the two poles of the magnet?

Y near north pole Y near south pole


A attraction attraction
B attraction repulsion
C repulsion attraction
D repulsion repulsion

0625/01/M/J/03
13

28 When the potential difference (p.d.) across a piece of resistance wire is changed, the current
through the wire also changes.

The temperature of the wire is kept the same.

Which graph shows how the p.d. and current are related?

A B C D

current current current current

0 0 0 0
0 p.d. 0 p.d. 0 p.d. 0 p.d.

29 Two faulty ammeters and two perfect ammeters are connected in series in the circuit shown.

A1 A2 A3 A4

The readings on the ammeters are

A1 2.9 A
A2 3.1 A
A3 3.1 A
A4 3.3 A

Which two ammeters are faulty?

A A1 and A2 B A1 and A4 C A2 and A3 D A3 and A4

30 Which electrical component would not normally be found in a battery-operated torch (flashlight)?

A B C D

0625/01/M/J/03 [Turn over


14

31 A student connects two lamps in the circuit shown.

Which switches must he close to light both lamps?

A 1 and 2
B 1, 2 and 3
C 1 and 3
D 2 and 3

32 A student makes four circuits.

In which circuit are both lamps protected by the fuse?

A B

C D

0625/01/M/J/03
15

33 Four lamps are labelled ‘60 W 240 V’.

In which circuit are the lamps connected so that they all work at normal brightness?

A B C D
240 V 240 V 240 V 240 V

34 The diagram shows a solenoid connected to a sensitive voltmeter.

S
magnet

solenoid
V

Which of the following would give a zero reading on the voltmeter?

A holding the magnet stationary inside the solenoid


B moving the magnet away from the solenoid
C moving the magnet towards the solenoid
D moving the solenoid towards the magnet

0625/01/M/J/03 [Turn over


16

35 The diagram shows a transformer with an alternating voltage of 100 V applied to the primary coil.

secondary coil
primary coil

100 V (40 turns) (80 turns)

What is the voltage produced across the secondary coil?

A 50 V B 100 V C 200 V D 8000 V

36 The diagram below shows the screen of a cathode-ray oscilloscope tube.

spot of light

The tube is placed between a pair of charged plates.

Which diagram shows the new position of the spot?

A B

+ – + –

+ – + –

+ – + –

+ – + –

+ – + –

C D

+ – + –

+ – + –

+ – + –

+ – + –

+ – + –

0625/01/M/J/03
17

37 An electrical component X is placed in water, as shown.

thermometer

water

When the temperature of the water is increased, the reading on the ammeter increases.

What is component X?

A a capacitor
B a light-dependent resistor
C a reed relay
D a thermistor

38 Which type of radiation can be stopped by a sheet of paper?

A α-particles
B β-particles
C γ-rays
D X-rays

39 The half-life of a radioactive substance is 5 hours. A sample is tested and found to contain 0.48 g
of the substance.

How much of the substance was present in the sample 20 hours before the sample was tested?

A 0.03 g
B 0.12 g
C 1.92 g
D 7.68 g

0625/01/M/J/03 [Turn over


18

40 An atom of lithium contains three protons and three electrons.

The nucleon number (mass number) of the atom is 7.

How many neutrons are there in the atom?

A 3 B 4 C 7 D 10

Go to answers

0625/01/M/J/03
CAMBRIDGE INTERNATIONAL EXAMINATIONS
International General Certificate of Secondary Education

PHYSICS 0625/01
Paper 1 Multiple Choice
October/November 2003

45 minutes
Additional Materials: Multiple Choice Answer Sheet
Soft clean eraser
Soft pencil (type B or HB is recommended)

READ THESE INSTRUCTIONS FIRST

Write in soft pencil.


Do not use staples, paper clips, highlighters, glue or correction fluid.
Write your name, Centre number and candidate number on the Answer Sheet in the spaces provided
unless this has been done for you.

There are forty questions on this paper. Answer all questions. For each question there are four possible
answers A, B, C, and D.
Choose the one you consider correct and record your choice in soft pencil on the separate Answer Sheet.

Read the instructions on the Answer Sheet very carefully.

Each correct answer will score one mark. A mark will not be deducted for a wrong answer.
Any rough working should be done in this booklet.

This document consists of 18 printed pages and 2 blank pages.


MML 4504 11/02 S53619/2 UNIVERSITY of CAMBRIDGE
© UCLES 2003 Local Examinations Syndicate [Turn over
2

1 Which of the following is not necessary when using a measuring cylinder to measure the volume
of a quantity of water?

A making sure that the measuring cylinder is vertical


B making sure that your eye is level with the liquid surface
C reading the bottom of the meniscus
D using the largest measuring cylinder possible

2 A pendulum is set in motion and 20 complete swings are timed. The time measured is 30 s.

What is the time for one complete swing of the pendulum?

A 0.67 s B 0.75 s C 1.5 s D 3.0 s

3 Five telegraph poles are positioned at equal distances along the side of a road.

pole 1 pole 2 pole 3 pole 4 pole 5

A car accelerates until it is level with pole 4. The car then continues along the road at a steady
speed. The times taken to travel between one pole and the next are measured.

Which time is the greatest?

The time between

A pole 1 and pole 2.


B pole 2 and pole 3.
C pole 3 and pole 4.
D pole 4 and pole 5.

0625/01/O/N/03
3

4 A train travels along a track from Aytown to Beetown. The map shows the route.

Aytown

Beetown

The distance travelled by the train between the towns is 210 km. It moves at an average speed of
70 km / h.

How long does the journey take?


70
A less than ––– hours, because the journey is not in a straight line
210
70
B exactly ––– hours
210
210
C exactly ––– hours
70
210
D more than ––– hours, because the journey is not in a straight line
70

5 A beam is pivoted at its centre. Two masses are suspended at equal distances from the pivot as
shown in the diagram.

pivot

block X 2 kg mass

Which statement is correct?

A If X has a mass of exactly 2 kg, it will rise.


B If X has a mass of less than 2 kg, it will fall.
C If X has a mass of more than 2 kg, it will fall.
D If X has a mass of more than 2 kg, it will rise.

0625/01/O/N/03 [Turn over


4

6 The mass of a full bottle of cooking oil is 1.30 kg.

When exactly half of the oil has been used, the mass of the bottle plus the remaining oil is
0.90 kg.

What is the mass of the bottle?

A 0.40 kg B 0.50 kg C 0.65 kg D 0.80 kg

7 A student tries to find the density of a metal block. First he measures the weight with a
forcemeter (spring balance). Next he measures the sides of the block using a rule, in order to
calculate the volume of the block. Finally he divides the weight by the volume to find the density.

The student has made a mistake.

Why does his method not give the density?

A Density is volume divided by weight.


B He should have measured the surface area, not the volume.
C He should have used the mass in his calculation, not the weight.
D Weight is not measured with a forcemeter (spring balance).

8 Two forces act on an object.

In which situation is it impossible for the object to be in equilibrium?

A The two forces act in the same direction.


B The two forces act through the same point.
C The two forces are of the same type.
D The two forces are the same size.

0625/01/O/N/03
5

9 The table below shows the length of a wire as the load on it is increased.

load / N 0 10 20 30

length / cm 50.0 52.1 54.1 56.3

Which graph correctly shows the extension of the wire plotted against load?

A B

60 3
extension / cm extension / cm
40 2

20 1

0 0
0 10 20 30 0 10 20 30
load / N load / N

C D

6 56.3
extension / cm extension / cm
4 54.1

2 52.1

0 50.0
0 10 20 30 0 10 20 30
load / N load / N

10 A large electric motor is used to lift a container off a ship.

Which of the following values are enough to allow the power of the motor to be calculated?

A the mass of the container and the distance moved


B the force used and the distance moved
C the current used and the work done
D the work done and the time taken

0625/01/O/N/03 [Turn over


6

11 A tidal power station is made by building a barrage across the mouth of a river. At high tide the
sea water is trapped behind the barrage.

barrage

turbine
trapped sea water

sea at low
tide

At low tide the water is allowed to flow back into the sea through a turbine.

What is the useful energy change in a tidal power station?

A electrical energy → energy of position (potential)


B electrical energy → energy of motion (kinetic)
C energy of motion (kinetic) → energy of position (potential)
D energy of position (potential) → electrical energy

12 Which diagram shows the child exerting least pressure on the ground?

A B C D

0625/01/O/N/03
7

13 A manometer is being used to measure the pressure of the gas inside a tank. A, B, C and D
show the manometer at different times.

At which time is the gas pressure inside the tank greatest?

A B C D

gas

14 Which line in the table correctly describes whether the molecules of a solid, liquid and gas are
moving or stationary?

solid liquid gas


A stationary stationary stationary
B stationary stationary moving
C stationary moving moving
D moving moving moving

15 Driving a car raises the temperature of the tyres.

This causes the pressure of the air in the tyres to increase.

Why is this?

A Air molecules break up to form separate atoms.


B Air molecules expand with the rise in temperature.
C The force between the air molecules increases.
D The speed of the air molecules increases.

0625/01/O/N/03 [Turn over


8

16 The diagram shows how the atoms in a substance rearrange themselves during a change of
state.

Which change of state is shown?

A gas to liquid
B liquid to gas
C liquid to solid
D solid to liquid

17 Equal masses of two different liquids are put into identical beakers. They are heated from 20 °C
to 30 °C by heaters of the same power. Liquid 2 takes twice as long to heat as liquid 1.

same mass of
different liquids
liquid 1 liquid 2

heaters of
same power

heating time = t heating time = 2t

Which statement is correct?

A Both liquids receive the same amount of energy.


B Liquid 1 receives more energy than liquid 2.
C The thermal capacity of liquid 1 is equal to the thermal capacity of liquid 2.
D The thermal capacity of liquid 1 is less than the thermal capacity of liquid 2.

0625/01/O/N/03
9

18 There is a vacuum between the double walls of a vacuum flask.

Which types of heat transfer are reduced by the vacuum?

A conduction and convection


B conduction and radiation
C convection and radiation
D conduction, convection and radiation

19 The diagrams show four identical pieces of ice that are heated in test-tubes of water.

In which test-tube will the ice take the longest time to melt?

A B

ice water

water ice wrapped


in lead wire

C D

water
water ice
ice wrapped
in lead wire

0625/01/O/N/03 [Turn over


10

20 The diagram shows a cork with a weight attached so that the cork floats upright.

cork water surface

X Y

weight

Transverse waves travel across the water from X to Y.

Which way do the waves make the cork move?

A → ← right and left


B ↑↓ up and down
C → only to the right
D ← only to the left

21 Waves travel more slowly on the surface of water when the water is shallow.

A person drops a stone into a pool at X. The diagram shows the first wavefront on the surface of
the pool.

Which region of the pool is likely to be most shallow?

A X
C

wavefront

0625/01/O/N/03
11

22 Which diagram correctly shows the paths taken by red and blue light when a beam of white light
enters a glass prism?

A B

red blue
blue red
white white
light light

C D

blue red
red blue
white white
light light

23 Which diagram shows the correct order of the waves in the electromagnetic spectrum?

visible
infra ultra
radio red violet X-ray
A

increasing wavelength

visible
ultra infra
radio violet red X-ray
B

increasing wavelength

visible
ultra infra
X-ray violet red radio
C

increasing wavelength

visible
infra ultra
X-ray red violet radio
D

increasing wavelength

0625/01/O/N/03 [Turn over


12

24 Which change will lower the pitch of a sound?

A decreasing its amplitude


B decreasing its frequency
C increasing its amplitude
D increasing its frequency

25 Astronaut 1 uses a hammer to mend a satellite in space. Astronaut 2 is nearby. There is no


atmosphere in space.

hammer
astronaut 1 astronaut 2

Compared with the sound heard if they were working on Earth, what does astronaut 2 hear?

A no sound at all
B a quieter sound
C a sound of the same loudness
D a louder sound

0625/01/O/N/03
13

26 A steel bar is magnetised by stroking it several times with the south pole of a magnet, as shown.

X Y

steel bar

Which poles are formed at X and Y?

X Y
A north north
B north south
C south north
D south south

27 A steel ball on a horizontal wooden table rolls near the north pole of a bar magnet that is lying on
the table.

Which diagram shows the most likely path of the ball, as seen from above the table?

A B
S S
magnet magnet
N N

ball ball

C D

magnet S magnet S
N N

ball ball stops


here

0625/01/O/N/03 [Turn over


14

28 A student wants to find the resistance of resistor R using a voltmeter and an ammeter.

Which circuit should the student use?

A B C D

V A V A
V
R R R R

A V A

29 Two very light, charged balls P and Q are hung, one above the other, from nylon threads. When a
negatively charged plastic sheet is placed alongside them, P is repelled and Q is attracted.

negatively
charged
plastic sheet

What are the original charges on P and on Q?

charge on P charge on Q
A negative negative
B negative positive
C positive negative
D positive positive

0625/01/O/N/03
15

30 Four students are asked to draw a circuit showing three lamps working in parallel, a cell, and a
switch that controls all three lamps.

Which student is correct?

A B

C D

31 A 3.0  lamp and a 6.0  lamp are connected in series.

What is the total resistance of the combination?

A 0.5 
B 2.0 
C 9.0 
D 18.0 

32 Why is a circuit breaker or a fuse used in a mains electric circuit?

A It allows spare current to be returned to the mains.


B It improves the insulation of the wiring.
C It protects the mains wiring from current overload.
D It saves energy by reducing the current.
0625/01/O/N/03 [Turn over
16

33 The diagrams show two ways in which three lamps may be connected.

Y X Y Z

circuit 1 circuit 2

Which statement is correct?

A If lamp Y breaks in circuit 1, both the other lamps will go out.


B If lamp Y breaks in circuit 2, both the other lamps will go out.
C If lamp Y breaks in circuit 1, lamp Z will go out, but lamp X will remain on.
D If lamp Y breaks in circuit 2, lamp Z will go out, but lamp X will remain on.

34 How is electricity transmitted over large distances and why is it transmitted in this way?

how why
A at high voltage for safety
B at high voltage to reduce energy loss
C at low voltage for safety
D at low voltage to reduce energy loss

35 In a cathode-ray tube, particles are given off from a hot cathode by thermionic emission.

Which particles are given off?

A atoms
B electrons
C ions
D protons

0625/01/O/N/03
17

36 When the thermistor in the circuit below is heated, the lamp becomes brighter.

Why does this happen?

A The resistance of the lamp decreases.


B The resistance of the lamp increases.
C The resistance of the thermistor decreases.
D The resistance of the thermistor increases.

37 Which line in the table describes the nature of an -particle and a -ray?

-particle -ray
A helium nucleus electromagnetic radiation
B helium nucleus electron
C proton electromagnetic radiation
D proton electron

38 A radioactive nucleus R decays with the emission of a -particle as shown.

x
yR → qpS + 

Which equation is correct?

A x=p
B y=q
C p=x–1
D q=y–1

0625/01/O/N/03 [Turn over


18

39 Which line in the table shows the structure of the nucleus of a helium atom 24 He?

electrons neutrons protons


A 2 2 0
B 2 0 2
C 0 2 2
D 2 2 2

26 X.
40 A nucleus of substance X has the symbol 12

How many electrons orbit around the nucleus of a neutral atom of substance X?

A 12
B 14
C 26
D 38

Go to answers

0625/01/O/N/03
UNIVERSITY OF CAMBRIDGE INTERNATIONAL EXAMINATIONS
International General Certificate of Secondary Education

PHYSICS 0625/01
Paper 1 Multiple Choice
May/June 2004

45 minutes
Additional Materials: Multiple Choice Answer Sheet
Soft clean eraser
Soft pencil (type B or HB is recommended)

READ THESE INSTRUCTIONS FIRST

Write in soft pencil.


Do not use staples, paper clips, highlighters, glue or correction fluid.
Write your name, Centre number and candidate number on the Answer Sheet in the spaces provided
unless this has been done for you.

There are forty questions on this paper. Answer all questions. For each question there are four possible
answers A, B, C, and D.
Choose the one you consider correct and record your choice in soft pencil on the separate Answer Sheet.

Read the instructions on the Answer Sheet very carefully.

Each correct answer will score one mark. A mark will not be deducted for a wrong answer.
Any rough working should be done in this booklet.

This document consists of 18 printed pages and 2 blank pages.


IB04 06_0625_01/3RP
 UCLES 2004 [Turn over
2

1 The diagram shows a measuring cylinder.

100
90
80
70
60
50
40
30
20
10

Which unit would be most suitable for its scale?

A mm2 B mm3 C cm2 D cm3

2 A piece of cotton is measured between two points on a ruler.

cotton

cm 1 2 3 4 5 6 7 8 9 10 11 12 13 14 15 16

When the length of cotton is wound closely around a pen, it goes round six times.

six turns of cotton


pen

What is the distance once round the pen?

A 2.2 cm B 2.6 cm C 13.2 cm D 15.6 cm

 UCLES 2004 0625/01/M/J/04


3

3 The diagram shows the speed-time graph for an object moving at constant speed.

2
speed
m/s
1

0
0 1 2 3 4
time / s

What is the distance travelled by the object in the first 3 s?

A 1.5 m B 2.0 m C 3.0 m D 6.0 m

4 A small steel ball is dropped from a low balcony.

Ignoring air resistance, which statement describes its motion?

A It falls with constant acceleration.


B It falls with constant speed.
C It falls with decreasing acceleration.
D It falls with decreasing speed.

5 Which statement about the mass of a falling object is correct?

A It decreases as the object falls.

B It is equal to the weight of the object.


C It is measured in newtons.
D It stays the same as the object falls.

 UCLES 2004 0625/01/M/J/04 [Turn over


4

6 The weights of four objects, 1 to 4, are compared using a balance.

2 2 4

1 3 2

Which object is the lightest?

A object 1 B object 2 C object 3 D object 4

7 Which of the following is a unit of density?

A cm3 / g

B g / cm2
C g / cm3
D kg / m2

8 A piece of card has its centre of mass at M.

Which diagram shows how it hangs when suspended by a thread?

A B C D

M
M
M M

9 An experiment is carried out to measure the extension of a rubber band for different loads.

The results are shown below.

load / N 0 1 2 3
length / cm 15.2 16.2 18.6
extension / cm 0 1.0 2.1 3.4

Which figure is missing from the table?

A 16.5 B 17.3 C 17.4 D 18.3

 UCLES 2004 0625/01/M/J/04


5

10 The diagram shows a man diving into water.

Which form of energy is increasing as he falls?

A chemical
B gravitational
C kinetic
D strain

11 A boy and a girl run up a hill in the same time.

boy weighs 600 N girl weighs 500 N

The boy weighs more than the girl.

Which statement is true about the power produced?

A The boy produces more power.

B The girl produces more power.


C They both produce the same power.
D It is impossible to tell who produces more power.

 UCLES 2004 0625/01/M/J/04 [Turn over


6

12 The diagram shows a simple mercury barometer. The barometer reading is h cm of mercury.

mercury

What is the pressure at S?

A approximately zero
B atmospheric pressure
C atmospheric pressure + h cm of mercury
D h cm of mercury

13 Two boys X and Y each have the same total weight and are standing on soft ground.

X Y

Which boy is more likely to sink into the soft ground and why?

boy more pressure on soft


likely to sink ground

A X larger than Y
B X smaller than Y
C Y larger than X
D Y smaller than X

 UCLES 2004 0625/01/M/J/04


7

14 A student places his thumb firmly on the outlet of a bicycle pump, to stop the air coming out.

trapped air

direction of
motion

handle

What happens to the pressure and to the volume of the trapped air as the pump handle is pushed
in?

pressure volume

A decreases decreases
B decreases remains the same
C increases decreases
D increases remains the same

15 A balloon is inflated in a cold room. When the room becomes much warmer, the balloon becomes
larger.

How does the behaviour of the air molecules in the balloon explain this?

A The molecules become larger.


B The molecules evaporate.
C The molecules move more quickly.
D The molecules repel each other.

 UCLES 2004 0625/01/M/J/04 [Turn over


8

16 A substance is heated at a steady rate. It changes from a solid to a liquid, and then to a gas.

The graph shows how its temperature changes with time.

S
temperature
R

time

Which parts of the graph show a change of state taking place?

A P and R
B P and S
C Q and R
D Q and S

17 An engineer wants to fix a steel washer on to a steel rod. The rod is just too big to fit into the hole
of the washer.

steel steel rod


washer

How can the engineer fit the washer onto the rod?

A cool the washer and put it over the rod


B cool the washer and rod to the same temperature and push them together
C heat the rod and then place it in the hole
D heat the washer and place it over the rod

 UCLES 2004 0625/01/M/J/04


9

18 An experiment is set up to find out which metal is the best conductor of heat. Balls are stuck with
wax to rods made from different metals, as shown in diagram X.

The rods are heated at one end. Some of the balls fall off, leaving some as shown in diagram Y.

Which labelled metal is the best conductor of heat?

diagram X diagram Y
A B C D

h e a t h e a t
before heating after heating

19 Thermometer X is held above an ice cube and thermometer Y is held the same distance below
the ice cube. After several minutes, the reading on one thermometer changes. The ice cube does
not melt.

thermometer X

ice cube

thermometer Y

Which thermometer reading changes and why?

thermometer reason

A X cool air rises from the ice cube


B X warm air rises from the ice cube
C Y cool air falls from the ice cube
D Y warm air falls from the ice cube

 UCLES 2004 0625/01/M/J/04 [Turn over


10

20 Water waves change direction when they move from shallow water to deep water.

new wave
direction

original
wave
direction
deep
water

shallow
water

What is the name of this effect?

A diffraction
B dispersion
C reflection
D refraction

21 A vertical stick is dipped up and down in water at P. In two seconds, three wave crests are
produced on the surface of the water.

wave P
crests

Which statement is true?

A Distance X is the amplitude of the waves.


B Distance Y is the wavelength of the waves.
C Each circle represents a wavefront.
D The frequency of the waves is 3 Hz.

 UCLES 2004 0625/01/M/J/04


11

22 A plane mirror is on a wall.

Which is a correct description of the image formed by the mirror?

A the right way up and smaller than the object


B the right way up and the same size as the object
C upside down and smaller than the object
D upside down and the same size as the object

23 The diagram shows a ray of light entering a block of glass.

normal
ray of
light

2
air 1
glass 3

Which numbered angles are the angles of incidence and of refraction?

angle angle
of incidence of refraction

A 1 3
B 1 4
C 2 3
D 2 4

 UCLES 2004 0625/01/M/J/04 [Turn over


12

24 Three rays of light fall on a converging lens as shown.

lens

Which diagram shows the path of the rays after passing through the lens?

A B

C D

25 Which type of wave cannot travel through a vacuum?

A infra-red radiation
B microwaves
C sound waves
D X-rays

 UCLES 2004 0625/01/M/J/04


13

26 An engineer standing at P hears the sound of an explosion at X.

P Z

Y
DANGER - X
BLASTING

V W

After the explosion, she hears two bangs. One bang is heard a fraction of a second after the
other.

The second bang is an echo from

A XY.
B PV.
C ZY.
D WX.

27 How can a permanent magnet be demagnetised?

A cool the magnet for a long time


B hit the magnet repeatedly with a hammer
C leave the magnet in a coil which carries direct current
D pass a small current through the magnet

28 An electromagnet is used to separate magnetic metals from non-magnetic metals.

Why is steel unsuitable as the core of the electromagnet?

A It is a good conductor of electricity.

B It forms a permanent magnet.


C It has a high density.
D It has a high thermal capacity.

 UCLES 2004 0625/01/M/J/04 [Turn over


14

29 Which circuit shows how a voltmeter is connected to measure the potential difference across the
cell?

A B C D

V V
V

30 A polythene rod repels an inflated balloon hanging from a nylon thread.

What charges must the rod and the balloon carry?

A The rod and the balloon carry opposite charges.


B The rod and the balloon carry like charges.
C The rod is charged but the balloon is not.
D The balloon is charged but the rod is not.

31 An electrical component is to be placed in the circuit at Z, to allow the brightness of the lamp to
be varied from bright to dim.

What should be connected at Z?

A B C D

 UCLES 2004 0625/01/M/J/04


15

32 The circuit shown contains four lamps and three switches.

switch 1
lamp 1

switch 2 lamp 2

lamp 3

switch 3
lamp 4

Which switches must be closed to light only lamps 1 and 3?

A switch 1 only
B switch 1 and switch 2 only
C switch 1 and switch 3 only
D switch 2 and switch 3 only

33 The diagram shows a torch containing two 2 V cells, a switch and a lamp.

plastic
case

brass
connecting switch
strip lamp

What is the circuit diagram for the torch?

A B C D

 UCLES 2004 0625/01/M/J/04 [Turn over


16

34 Which statement is correct?

A A fuse is included in a circuit to prevent the current becoming too high.


B A fuse should be connected to the neutral wire in a plug.
C An electric circuit will only work if it includes a fuse.
D An earth wire is needed to prevent the fuse blowing.

35 A straight wire carrying a current produces a magnetic field.

Which diagram shows the correct shape of the field?

A B
current current

C D
current current

 UCLES 2004 0625/01/M/J/04


17

36 A student carries out an experiment to see the effect of a magnetic field on a wire carrying a
current.

The wire moves upwards as shown.

wire moves
upwards

N S
direction
of current

What should the student do to make the wire move downwards?

A change the direction of the current


B move the poles of the magnet closer together
C send a smaller current through the wire
D use a stronger magnet

37 A beam of cathode rays passes through an electric field between two parallel plates.

+ + + + + +

cathode rays
_ _ _ _ _ _

In which direction is the beam deflected?

A into the page


B out of the page
C towards the bottom of the page
D towards the top of the page

 UCLES 2004 0625/01/M/J/04 [Turn over


18

38 Which line correctly describes α-particles?

penetrates 1 cm
electric charge
of aluminium?

A negative yes
B negative no
C positive yes
D positive no

39 A small amount of a radioactive isotope contains 72 billion unstable nuclei. The half-life of the
isotope is 4 hours.

How many unstable nuclei would remain after 12 hours?

A 6 billion
B 9 billion
C 18 billion
D 24 billion

39
40 How many nucleons are in a nucleus of 19 K ?

A 19 B 20 C 39 D 58

Go to answers

 UCLES 2004 0625/01/M/J/04


UNIVERSITY OF CAMBRIDGE INTERNATIONAL EXAMINATIONS
International General Certificate of Secondary Education

PHYSICS 0625/01
Paper 1 Multiple Choice
October/November 2004

45 minutes
Additional Materials: Multiple Choice Answer Sheet
Soft clean eraser
Soft pencil (type B or HB is recommended)

READ THESE INSTRUCTIONS FIRST

Write in soft pencil.


Do not use staples, paper clips, highlighters, glue or correction fluid.
Write your name, Centre number and candidate number on the Answer Sheet in the spaces provided
unless this has been done for you.

There are forty questions on this paper. Answer all questions. For each question there are four possible
answers A, B, C, and D.
Choose the one you consider correct and record your choice in soft pencil on the separate Answer Sheet.

Read the instructions on the Answer Sheet very carefully.

Each correct answer will score one mark. A mark will not be deducted for a wrong answer.
Any rough working should be done in this booklet.

This document consists of 14 printed pages and 2 blank pages.


IB04 11_0625_01/2RP
 UCLES 2004 [Turn over
2

1 A floor is covered with square tiles. The diagram shows a ruler on the tiles.

0 1 2 3 4 5 6 7 8 9 10 11 12 13 14 15
cm

How long is one tile?

A 3 cm B 6 cm C 9 cm D 12 cm

2 The diagrams show the times on a stopclock at the beginning and at the end of an experiment.

stopclock stopclock
at beginning at end
0 0

s s
45 15 45 15

30 30

How long did the experiment take?

A 10 s B 25 s C 35 s D 45 s

3 Which speed / time graph applies to an object at rest?

A B C D

speed speed speed speed

0 0 0 0
0 time 0 time 0 time 0 time

© UCLES 2004 0625/01/O/N/04


3

4 A racing car is fitted with an on-board computer. Every time the car passes the starting line, the
computer records the distance travelled in the next 2 seconds.

Which set of data shows that the car is increasing in speed during the 2 seconds?

A B
distance distance
time / s time / s
travelled / m travelled / m

0 0 0 0
1 100 1 90
2 200 2 180

C D
distance distance
time / s time / s
travelled / m travelled / m

0 0 0 0
1 80 1 100
2 190 2 180

5 A spring is stretched by hanging a piece of metal from it.

spring

metal

What is the name given to the force that stretches the spring?

A friction
B mass
C pressure
D weight

© UCLES 2004 0625/01/O/N/04 [Turn over


4

6 The diagram shows some liquid in a measuring cylinder. The mass of the liquid is 16 g.

25
cm3
20

15
10
5

What is the density of the liquid?

A 320 g / cm 3 B 36 g / cm 3 C 1.25 g / cm 3 D 0.8 g / cm 3

7 A student carries out an experiment to plot an extension / load graph for a spring. The diagrams
show the apparatus at the start of the experiment and with a load added.

start with load added

x
y

What is the extension caused by the load?

A x B y C y+x D y–x

8 Three horizontal forces act on a car that is moving along a straight, level road.

air resistance

friction driving force

Which combination of forces would result in the car moving at constant speed?

air resistance friction driving force

A 200 N 1000 N 800 N


B 800 N 1000 N 200 N
C 800 N 200 N 1000 N
D 1000 N 200 N 800 N

© UCLES 2004 0625/01/O/N/04


5

9 A child pushes a toy car along a level floor and then lets it go.

As the car slows down, what is the main energy change?

A from chemical to heat


B from chemical to kinetic
C from kinetic to gravitational (potential)
D from kinetic to heat

10 The diagram shows a cyclist riding along a hilly road.

At which position does the cyclist have the least gravitational (potential) energy?

D
A

11 A pin is squeezed between finger and thumb.

finger

pinhead

pin

thumb

Which statement is correct?

A The force of the pin is larger on the finger than on the thumb.

B The force of the pin is larger on the thumb than on the finger.
C The pressure of the pin is larger on the finger than on the thumb.
D The pressure of the pin is larger on the thumb than on the finger.

© UCLES 2004 0625/01/O/N/04 [Turn over


6

12 Liquid X has a density of 1010 kg / m3. Liquid Y has a density of 950 kg / m3.

The liquids are poured into tubes as shown.

Which tube has the greatest pressure on its base?

A B C D

liquid X liquid Y liquid X liquid Y

13 Some water molecules escape from the surface of a lake.

Which name is given to this process?

A boiling

B convection
C evaporation
D radiation

14 The diagram represents gas molecules contained in a cylinder. The piston is moved slowly
downwards and the temperature of the gas stays the same.

piston

cylinder

gas molecule

Why does the pressure of the gas increase?

A The molecules collide harder with the walls.

B The molecules collide more often with the walls.


C The molecules move more quickly.
D The number of molecules increases.

© UCLES 2004 0625/01/O/N/04


7

15 To mark the lower fixed point of a Celsius scale on a thermometer, the thermometer should be
placed in

A pure alcohol.
B pure distilled water.
C pure melting ice.
D pure mercury.

16 A glass flask full of cool water is placed in a container of hot water.

container of glass flask full


hot water of cool water

What will happen to the level of water at X as the cool water becomes warmer?

A It will fall.
B It will rise.
C It will rise then fall.
D It will stay the same.

17 A beaker of water is heated at its base.

Why does the water at the base rise?

A It contracts and becomes less dense.


B It contracts and becomes more dense.
C It expands and becomes less dense.
D It expands and becomes more dense.

© UCLES 2004 0625/01/O/N/04 [Turn over


8

18 Which of these waves is longitudinal?

A light waves
B sound waves
C water waves
D X-ray waves

19 Waves move from deep water to shallow water where they are slower.

Which diagram shows what happens to the waves?

A B C D

deep deep deep deep


water water water water

shallow shallow shallow shallow


water water water water

20 Which type of radiation lies between visible light and microwaves in the electromagnetic
spectrum?

A infra-red
B radio waves
C ultra-violet
D X-rays

21 The critical angle for a glass / air boundary is C.

Which diagram shows the correct path of the light ray?

A B C D

air air air air


glass glass glass glass
C C C C

© UCLES 2004 0625/01/O/N/04


9

22 The diagram shows the image of a clockface in a plane mirror.

Which of these times is shown?

A 02.25 B 02.35 C 09.25 D 09.35

23 What is the approximate range of audible frequencies for most humans?

A 10 Hz to 10 000 Hz
B 20 Hz to 20 000 Hz
C 10 kHz to 10 000 kHz
D 20 kHz to 20 000 kHz

24 A 100 metre race is started by firing a gun. The gun makes a bang and a puff of smoke comes
out of the gun as shown.

starter finishing
judge

100 m

When does the finishing judge see the smoke and hear the bang?

sees the smoke hears the bang

A immediately immediately
B immediately after about 0.3 s
C after about 0.3 s immediately
D after about 0.3 s after about 0.3 s

© UCLES 2004 0625/01/O/N/04 [Turn over


10

25 Six small compasses are placed around a bar magnet.

Which diagram shows the directions in which the compass needles point?

A B

N S N S

C D

N S N S

26 A student carries out four tests with a magnet.

Which result shown is not correct?

arrangement result

A S magnet N iron bar attracts

B S magnet N S magnet N attracts

C N magnet S copper bar no effect

D N magnet S N magnet S repels

27 A circuit is set up with a gap between two terminals X and Y. The four strips of material shown in
the diagram are connected in turn across the gap.

Which strip completes the circuit so that the lamp lights?

A B C D
wood glass steel rubber
X

© UCLES 2004 0625/01/O/N/04


11

28 A pupil measures the potential difference across a device and the current in it.

Which calculation gives the resistance of the device?

A current + potential difference


B current ÷ potential difference
C potential difference ÷ current
D potential difference x current

29 Which circuit could be used to find the resistance of resistor R?

A B

V
R
R
A A

C D

A
R R
V

A V

30 In the circuit shown, the reading on ammeter 1 is 2 A.

2Ω

2Ω

ammeter 1 ammeter 2
reading = 2 A A A reading = ?

What is the reading on ammeter 2?

A 0A B 1A C 2A D 4A

© UCLES 2004 0625/01/O/N/04 [Turn over


12

31 In which position in the circuit shown should a switch be placed so that both lamps can be
switched on or off at the same time?

A
B

C
D

32 In the circuits shown, all the resistors are identical.

Which circuit has the least resistance?

A B

C D

33 Why are electric circuits often fitted with fuses?

A Fuses break the circuit if the current is too high.


B Fuses only allow the current through in the correct direction.
C Fuses return any excess current to earth.
D Fuses use up any spare current.

34 A mains circuit can safely supply a current of 40 A.

A hair-drier takes 2 A. It is connected to the circuit by a lead which can carry up to 5 A.

Which of these fuses would be best to use in the plug fitted to the hair-drier lead?

A 1 A fuse B 3 A fuse C 10 A fuse D 50 A fuse

© UCLES 2004 0625/01/O/N/04


13

35 The diagram represents a transformer.

input output
P S
voltage voltage

Which arrangement could be used to make the output voltage higher than the input voltage?

number of turns on number of turns on


type of input
primary coil P secondary coil S

A 100 50 a.c.
B 100 50 d.c.
C 50 100 a.c.
D 50 100 d.c.

36 A student sets up the apparatus shown in order to make a relay.

contact core

spring

Which metal should be used to make the core?

A aluminium
B copper
C iron
D steel

© UCLES 2004 0625/01/O/N/04 [Turn over


14

37 Which particles are emitted during thermionic emission?

A electrons
B ions
C neutrons
D protons

38 The equation shows the decay of the nuclide X.

226
88 X → QP Y + 4

What are the values of P and Q?

P Q

A 230 90
B 230 86
C 222 90
D 222 86

39 During a fire in a laboratory storeroom, some radioactive material was spilled. A firefighter
detected radiation through the lead-lined walls of the storeroom. The radiation was emitted by the
radioactive material.

lead-lined
storeroom
firefighter

detector radioactive
material

Which type of radiation was being detected?

A α-particles
B β-particles
C γ-rays
D X-rays

40 How many neutrons are in a nucleus of 146C ?

A 0 B 6 C 8 D 14

© UCLES 2004 0625/01/O/N/04


Go to answers
UNIVERSITY OF CAMBRIDGE INTERNATIONAL EXAMINATIONS
International General Certificate of Secondary Education

PHYSICS 0625/01
Paper 1 Multiple Choice
May/June 2005

45 minutes
Additional Materials: Multiple Choice Answer Sheet
Soft clean eraser
Soft pencil (type B or HB is recommended)

READ THESE INSTRUCTIONS FIRST

Write in soft pencil.


Do not use staples, paper clips, highlighters, glue or correction fluid.
Write your name, Centre number and candidate number on the Answer Sheet in the spaces provided unless
this has been done for you.

There are forty questions on this paper. Answer all questions.


For each question there are four possible answers A, B, C and D. Choose the one you consider correct and
record your choice in soft pencil on the separate Answer Sheet.

Read the instructions on the Answer Sheet very carefully.

Each correct answer will score one mark. A mark will not be deducted for a wrong answer.
Any rough working should be done in this booklet.

This document consists of 17 printed pages and 3 blank pages.


IB05 06_0625_01/RP
 UCLES 2005 [Turn over
2

1 A decorator wishes to calculate the area of a bathroom tile so that he can estimate the amount of
adhesive that he needs to buy.

What must he use?

A a measuring cylinder only


B a ruler only
C a measuring cylinder and a clock only
D a measuring cylinder and a ruler only

2 The three balls shown are dropped from a bench.

aluminium lead wood

Which balls have the same acceleration?

A aluminium and lead only


B aluminium and wood only
C lead and wood only
D aluminium, lead and wood

3 A car accelerates from traffic lights. The graph shows how the car’s speed changes with time.

speed
m/s
20

0
0 10 time / s

How far does the car travel before it reaches a steady speed?

A 10 m B 20 m C 100 m D 200 m

© UCLES 2005 0625/01/M/J/05


3

4 Which statement is correct?

A The mass of a bottle of water at the North Pole is different from its mass at the Equator.
B The mass of a bottle of water is measured in newtons.
C The weight of a bottle of water and its mass are the same thing.
D The weight of a bottle of water is one of the forces acting on it.

5 Two blocks X and Y are placed on a beam as shown. The beam balances on a pivot at its centre.

Y
X

pivot

What does this show about X and Y?

A They have the same mass and the same density.


B They have the same mass and the same weight.
C They have the same volume and the same density.
D They have the same volume and the same weight.

6 The masses of a measuring cylinder before and after pouring some liquid into it are shown in the
diagram.

cm3 cm3
200 200

100 100
liquid

mass = 80 g mass = 180 g

What is the density of the liquid?

A 100 g / cm3 B 100 g / cm3 C 180 g / cm3 D 180 g / cm3


120 140 120 140

© UCLES 2005 0625/01/M/J/05 [Turn over


4

7 A girl and a boy are pulling in opposite directions on a rope. The forces acting on the rope are
shown in the diagram.

girl boy
200 N 150 N

rope

Which single force has the same effect as the two forces shown?

A 50 N acting towards the girl


B 350 N acting towards the girl
C 50 N acting towards the boy
D 350 N acting towards the boy

8 Objects with different masses are hung on a 10 cm spring. The diagram shows how much the
spring stretches.

10 cm

20 cm

30 cm

100 g

The extension of the spring is directly proportional to the mass hung on it.

What is the mass of object M?

A 110 g B 150 g C 200 g D 300 g

© UCLES 2005 0625/01/M/J/05


5

9 What is designed to change electrical energy into kinetic energy?

A capacitor
B generator
C motor
D transformer

10 A power station uses nuclear fission to obtain energy.

In this process, nuclear energy is first changed into

A chemical energy.
B electrical energy.
C gravitational energy.
D internal energy.

11 A ball is released from rest and rolls down a track from the position shown.

What is the furthest position the ball could reach?

ball C
starts
here B

© UCLES 2005 0625/01/M/J/05 [Turn over


6

12 A water manometer is used to measure the pressure of a gas supply to a house. It gives a
reading of h cm of water.

gas
supply

h cm

Why is it better to use water rather than mercury in this manometer?

A h would be too large if mercury were used.


B h would be too small if mercury were used.
C The tube would need to be narrower if mercury were used.
D The tube would need to be wider if mercury were used.

13 A farmer has two carts. The carts have the same weight, but one has four narrow wheels and the
other has four wide wheels.

narrow wheel wide wheel

In rainy weather, which cart sinks less into soft ground, and why?

cart wheels why

A narrow greater pressure on the ground


B narrow less pressure on the ground
C wide greater pressure on the ground
D wide less pressure on the ground

© UCLES 2005 0625/01/M/J/05


7

14 Viewed through a microscope, very small particles can be seen moving with Brownian motion.

Which line in the table is correct?

type of motion particles are


of particles suspended in

A vibration a liquid or a gas


B vibration a solid, a liquid or a gas
C random a liquid or a gas
D random a solid, a liquid or a gas

15 A measured mass of gas is placed in a cylinder at atmospheric pressure and is then slowly
compressed.

piston
gas

piston pushed in

The temperature of the gas does not change.

What happens to the pressure of the gas?

A It drops to zero.
B It decreases, but not to zero.
C It stays the same.
D It increases.

16 The graph shows the change in temperature of a material as it is heated.

Which part on the graph shows when the material is boiling?

D
temperature
C

time

© UCLES 2005 0625/01/M/J/05 [Turn over


8

17 An experiment is set up as shown.

pressure gauge

air
flask

water

heat

What does the pressure gauge show as the air in the flask becomes hotter?

A a steady pressure
B a decrease in pressure
C an increase in pressure
D an increase and then a decrease in pressure

18 An iron bar is held with one end in a fire. The other end soon becomes too hot to hold.

hand fire
iron bar

How has the heat travelled along the iron bar?

A by conduction
B by convection
C by expansion
D by radiation

© UCLES 2005 0625/01/M/J/05


9

19 The diagram shows a block of ice placed in a warm room.

At which point is the temperature the lowest?

A
B

clamp C
ice

table D

20 The drawing shows a wave.

Which labelled distance is the wavelength?

A
B
D

21 Radio waves are received at a house at the bottom of a hill.

hill

The waves reach the house because the hill has caused them to be

A diffracted.
B radiated.
C reflected.
D refracted.

© UCLES 2005 0625/01/M/J/05 [Turn over


10

22 Which diagram correctly shows a ray of light passing through a rectangular glass block?

A B

C D

23 The ray diagram shows how an image is formed by a converging lens.

24 cm 10 cm 8 cm

What is the focal length of this lens?

A 8 cm B 10 cm C 18 cm D 24 cm

© UCLES 2005 0625/01/M/J/05


11

24 A fire alarm is not loud enough. An engineer adjusts it so that it produces a note of the same pitch
which is louder.

What effect does this have on the amplitude and on the frequency of the sound?

amplitude frequency

A larger larger
B larger same
C same larger
D same same

25 To estimate the width of a valley, a climber starts a stopwatch as he shouts. He hears an echo
from the opposite side of the valley after 1.0 s.

sound

climber

valley

The sound travels at 340 m / s.

What is the width of the valley?

A 85 m B 170 m C 340 m D 680 m

26 Which material is used for the core of an electromagnet?

A aluminium
B copper
C iron
D steel

© UCLES 2005 0625/01/M/J/05 [Turn over


12

27 A brass rod is positioned in an east-west direction and a plotting compass is placed at each end.

brass rod
N
plotting
compass

Which diagram shows the positions of the needles of the plotting compasses?

28 How many of the following materials conduct electricity?

aluminium
silver
iron
plastic

A 1 B 2 C 3 D 4

© UCLES 2005 0625/01/M/J/05


13

29 In which circuit does the voltmeter read the potential difference across the lamp?

A B

V V

C D

30 In the circuit below, X and Y are identical 6 V lamps.

6V

switch
X

What happens when the switch is closed?

A X lights more brightly than Y.


B Y lights more brightly than X.
C X and Y light with equal brightness.
D Neither X nor Y light.

© UCLES 2005 0625/01/M/J/05 [Turn over


14

31 The diagram shows a circuit with three ammeters, X, Y and Z.

A X

Y
A

Z
A

Which set of readings is possible?

X Y Z

A 2A 3A 5A
B 3A 2A 5A
C 3A 3A 3A
D 5A 2A 3A

32 A lamp is to be connected in a circuit so that the p.d. across it can be varied from 0 to 6 V.

Which circuit would be most suitable?

A B

6V 6V

C D

6V 6V

© UCLES 2005 0625/01/M/J/05


15

33 A student makes the circuit shown.

5 A fuse

The fuse has blown and stopped the current.

What could have caused this?

A The current rating of the fuse was too high.


B The current was too large.
C The lamp was loose.
D The voltage was too small.

34 Which graph shows the output voltage from a simple a.c. generator?

voltage

A 0
time

voltage

B 0
time

voltage

C 0
time

voltage

D 0
time

© UCLES 2005 0625/01/M/J/05 [Turn over


16

35 A transformer has 50 turns on its primary coil and 100 turns on its secondary coil. An a.c. voltage
of 25.0 V is connected across the primary coil.

25.0 V

primary coil
secondary coil
50 turns
100 turns

What is the voltage across the secondary coil?

A 12.5 V B 50.0 V C 175 V D 200 V

36 Two circuits are set up as shown. The iron rods are placed close together, and are free to move.

iron rod iron rod


X

What happens to the size of the gap at X when switch S is closed?

A It decreases.
B It decreases then increases.
C It increases.
D It does not change.

37 The diagram shows a simple cathode-ray tube.

Which part emits the electrons?

– +

A B C

© UCLES 2005 0625/01/M/J/05


17

38 Which type of radiation has the greatest ionising effect?

A α-particles

B β-particles

C γ-rays
D all have the same ionising effect

39 A powder contains 400 mg of a radioactive material that emits α-particles.

The half-life of the material is 5 days.

What mass of that material remains after 10 days?

A 0 mg B 40 mg C 100 mg D 200 mg

40 In the symbol below, A is the nucleon number and Z is the proton number.

Z
X
What is represented by the symbol?

A an electron
B a neutron
C a nuclide
D an X-ray

© UCLES 2005 0625/01/M/J/05


18

BLANK PAGE

0625/01/M/J/05
19

BLANK PAGE

0625/01/M/J/05
20

BLANK PAGE

Every reasonable effort has been made to trace all copyright holders where the publishers (i.e. UCLES) are aware that third-party material has been
reproduced. The publishers would be pleased to hear from anyone whose rights they have unwittingly infringed.

University of Cambridge International Examinations is part of the University of Cambridge Local Examinations Syndicate (UCLES), which is itself a department
of the University of Cambridge.
0625/01/M/J/05
UNIVERSITY OF CAMBRIDGE INTERNATIONAL EXAMINATIONS
International General Certificate of Secondary Education

PHYSICS 0625/01
Paper 1 Multiple Choice (Core)
October/November 2005

45 minutes
Additional Materials: Multiple Choice Answer Sheet
Soft clean eraser
Soft pencil (type B or HB is recommended)

READ THESE INSTRUCTIONS FIRST

Write in soft pencil.


Do not use staples, paper clips, highlighters, glue or correction fluid.
Write your name, Centre number and candidate number on the Answer Sheet in the spaces provided unless
this has been done for you.

There are forty questions on this paper. Answer all questions.


For each question there are four possible answers A, B, C and D. Choose the one you consider correct and
record your choice in soft pencil on the separate Answer Sheet.

Read the instructions on the Answer Sheet very carefully.

Each correct answer will score one mark. A mark will not be deducted for a wrong answer.
Any rough working should be done in this booklet.

This document consists of 17 printed pages and 3 blank pages.


IB05 11_0625_01/RP
 UCLES 2005 [Turn over
2

1 A measuring cylinder is used to measure the volume of a liquid.

cm3

50

40
50
30
40
20

10

What is the volume of the liquid?

A 43 cm3 B 46 cm3 C 48 cm3 D 54 cm3

2 The graph represents part of the journey of a car.

30

speed
m / s 20

10

0
0 10 20 30 40
time / s

What distance does the car travel during this part of the journey?

A 150 m B 300 m C 600 m D 1200 m

3 A man crosses a road 8.0 m wide at a speed of 2.0 m / s.

movement
of man
man

8.0 m

How long does the man take to cross the road?

A 4.0 s B 6.0 s C 10 s D 16 s

© UCLES 2005 0625/01/O/N/05


3

4 What is the gravitational force that the Earth exerts on an object?

A the density of the object


B the mass of the object
C the volume of the object
D the weight of the object

5 In an experiment, five identical bags of rice are balanced by a 10 kg mass.

five bags
of rice 10 kg mass

pivot

Two bags of rice are added to the other five.

What mass will now balance the bags?

A 3.5 kg B 7.0 kg C 10 kg D 14 kg

6 The same mass of four different liquids is placed in some measuring cylinders.

Which measuring cylinder contains the liquid with the greatest density?

A B C D

cm3 cm3 cm3 cm3


25 25
10 10
20 20
8 8
15 15
6 6
10 10
4 4

2 2 5 5

© UCLES 2005 0625/01/O/N/05 [Turn over


4

7 The diagram shows a flat metal plate that may be hung from a nail so that it can rotate about any
of four holes.

holes holes

flat metal plate

What is the smallest number of holes from which the flat metal plate should be hung in order to
find its centre of gravity?

A 1 B 2 C 3 D 4

8 Two equal forces F act on each of four planks.

Which plank turns?

A B C D
F F F F

pivot pivot pivot pivot


F F F F

9 Which type of power station does not use steam from boiling water to generate electricity?

A geothermal
B hydroelectric
C nuclear
D oil-fired

© UCLES 2005 0625/01/O/N/05


5

10 A man standing at the top of a cliff throws a stone.

Which forms of energy does the stone have at X and at Y?

energy at X energy at Y

A gravitational only kinetic only


B kinetic only gravitational only
C gravitational only gravitational and kinetic
D gravitational and kinetic gravitational and kinetic

11 Four blocks, each weighing 10 N, rest on a horizontal table.

Which block applies the greatest pressure on the table?

A B C D

table

10 N 10 N 10 N 10 N

© UCLES 2005 0625/01/O/N/05 [Turn over


6

12 The diagram shows a mercury barometer.

25 cm

mercury

75 cm

5 cm

Which distance is used to calculate the pressure of the atmosphere?

A 25 cm B 75 cm C 80 cm D 100 cm

13 A drop of liquid falls on a student’s skin and quickly evaporates.

What is the effect on the skin and the reason?

A The skin cools because the most energetic molecules escape from the liquid.
B The skin cools because the most energetic molecules remain in the liquid.
C The skin warms because the most energetic molecules escape from the liquid.
D The skin warms because the most energetic molecules remain in the liquid.

14 A suspension of pollen grains in water is observed under a microscope. The pollen grains are
seen to be moving all the time.

Which diagram illustrates this motion?

A B C D

© UCLES 2005 0625/01/O/N/05


7

15 A knife is being sharpened on a rotating sharpening-stone. A spark flies off and lands on the
operator’s hand. The spark is a very hot, very small piece of metal. The operator feels nothing.

What does this show about the piece of metal?

A It has a high thermal capacity.


B It has a low thermal capacity.
C It is a good conductor of heat.
D It is a poor conductor of heat.

16 Which substance is a liquid at a room temperature of 25 oC?

substance melting point / oC boiling point / oC

A –218 –183
B –39 357
C 44 280
D 119 444

17 The diagram shows a cooling unit in a refrigerator.

cooling
unit

Why is the cooling unit placed at the top?

A Cold air falls and warm air is displaced upwards.


B Cold air is a bad conductor so heat is not conducted into the refrigerator.
C Cold air is a good conductor so heat is conducted out of the refrigerator.
D Cold air stops at the top and so prevents convection.

© UCLES 2005 0625/01/O/N/05 [Turn over


8

18 How does heat from the Sun reach the Earth?

A conduction only
B convection only
C radiation only
D conduction, convection and radiation

19 The diagrams show four sources of waves.

Which source generates longitudinal waves?

A B C D

stick pushed up radio loudspeaker lamp


and down in water transmitter

© UCLES 2005 0625/01/O/N/05


9

20 In a ripple tank, water waves move towards a barrier with a narrow gap.

barrier

water
waves

Which diagram best shows the waves beyond the barrier?

A B

C D

© UCLES 2005 0625/01/O/N/05 [Turn over


10

21 A ray of light passes from glass into air at an angle of incidence of 40o. The glass has a critical
angle of 42 o.

Which diagram shows what happens to the ray?

A B

air air
glass glass
40o 40o

C D

air air
glass glass
40o 40o

22 Rays of light enter and leave a box.

ray 1
ray 1
ray 2
ray 2

What could be inside the box to make the rays behave as shown?

A a converging lens
B a parallel-sided glass block
C a plane mirror
D a triangular prism

© UCLES 2005 0625/01/O/N/05


11

23 A thin converging lens is used to produce, on a screen, a focused image of a candle.

screen
image
lens

candle

The screen and the lens are moved back and forth and various focused images are produced on
the screen.

Which statement is always true?

A The image is at the principal focus (focal point) of the lens.


B The image is bigger than the object.
C The image is closer to the lens than the object is.
D The image is inverted.

24 Two astronauts without radios can only communicate in space if their helmets are touching.
There is no air in space.

What does this show about sound?

through a solid through a vacuum

A can travel can travel


B can travel cannot travel
C cannot travel can travel
D cannot travel cannot travel

© UCLES 2005 0625/01/O/N/05 [Turn over


12

25 When the horn on a ship is sounded, the passengers hear an echo from a cliff after 4.0 s.

If the speed of sound is 340 m / s, how far away is the cliff?

A 170 m B 340 m C 680 m D 1360 m

26 How many of the following methods could be used to demagnetise a piece of steel?

heating it until it is red hot


pulling it from a coil that is carrying an alternating current
placing it in an east-west direction and hammering it
putting it in a coil which is carrying a direct current

A 1 B 2 C 3 D 4

27 Two rods X and Y look the same.

P magnet

rod X N S
Q

rod Y
S

The N pole of a magnet is brought close, in turn, to each end of both rods. The results of these
four actions are shown in the table.

end tested result

P attraction
Q attraction
R attraction
S repulsion

Which of the rods is a permanent magnet?

A neither of the rods


B both of the rods
C rod X only
D rod Y only

© UCLES 2005 0625/01/O/N/05


13

28 Which circuit should be used to find the resistance of a lamp?

A B

A
V A
V

C D
V

V
A A

29 The table shows the voltage and current ratings for four electric heaters.

Which heater has the least resistance?

voltage / V current / A

A 110 5.0
B 110 10
C 230 5.0
D 230 10

30 Which component can store energy for use in time delay circuits?

A capacitor
B potentiometer
C resistor
D thermistor

© UCLES 2005 0625/01/O/N/05 [Turn over


14

31 When the circuit shown is connected with switch S open, the 6 V lamp glows.

6V

6 V lamp

What happens to the brightness of the lamp when switch S is closed?

A It becomes brighter.
B It remains the same.
C It becomes dimmer.
D It goes off.

32 Why are the electric lamps in a house lighting circuit normally connected in parallel?

A The current in every circuit must be the same.


B The lamps are always switched on and off at the same time.
C The voltage across each lamp must be the mains voltage.
D When one of the lamps blows, all the others go out.

33 In the circuit shown, one of the fuses blows and all the lamps go out.

Which fuse blows?

B C
D

© UCLES 2005 0625/01/O/N/05


15

34 Which arrangement may be used to step up a voltage?

A iron core B iron core

a.c. output d.c. output


input input

C iron core D iron core

a.c. d.c.
output output
input input

35 The diagrams show a straight wire carrying a current into the paper.

Which diagram shows the magnetic field pattern due to this current?

A B C D

wire wire wire wire

© UCLES 2005 0625/01/O/N/05 [Turn over


16

36 A magnet is suspended from a spring so that it can move freely inside a coil which is connected
to a sensitive centre-zero ammeter.

spring

centre-zero
ammeter

What does the ammeter show when the magnet vibrates slowly up and down?

A a reading constantly changing from left to right and right to left


B a steady reading to the left
C a steady reading to the right
D a steady zero reading

37 Charged particles are emitted from the cathode of an oscilloscope.

What is the name and the charge of these particles?

name of particles charge of particles

A electrons negative
B electrons positive
C protons negative
D protons positive

© UCLES 2005 0625/01/O/N/05


17

38 A radioactive source emits radiation that can pass through a sheet of paper but not through thick
aluminium.

paper thick aluminium


(all the radiation (none of the radiation
passes through) passes through)

radiation

What does this show about the radiation?

A It is α-particles.

B It is β-particles.

C It is γ-rays.

D It is a mixture of α-particles and γ-rays.

39 An unstable nucleus has 145 neutrons and 92 protons. It emits a β-particle.

How many neutrons and protons does the nucleus have after emitting the β-particle?

neutrons protons

A 144 92
B 144 93
C 145 91
D 145 93

40 Which particles are found in the nucleus of an atom?

A neutrons and protons only


B neutrons only
C protons and electrons only
D protons, electrons and neutrons

© UCLES 2005 0625/01/O/N/05


18

BLANK PAGE

0625/01/O/N/05
19

BLANK PAGE

0625/01/O/N/05
20

BLANK PAGE

Permission to reproduce items where third-party owned material protected by copyright is included has been sought and cleared where possible. Every
reasonable effort has been made by the publisher (UCLES) to trace copyright holders, but if any items requiring clearance have unwittingly been included, the
publisher will be pleased to make amends at the earliest possible opportunity.

University of Cambridge International Examinations is part of the University of Cambridge Local Examinations Syndicate (UCLES), which is itself a department
of the University of Cambridge.

0625/01/O/N/05
UNIVERSITY OF CAMBRIDGE INTERNATIONAL EXAMINATIONS
International General Certificate of Secondary Education

PHYSICS 0625/01
Paper 1 Multiple Choice
May/June 2006

45 minutes
Additional Materials: Multiple Choice Answer Sheet
Soft clean eraser
Soft pencil (type B or HB is recommended)

READ THESE INSTRUCTIONS FIRST

Write in soft pencil.


Do not use staples, paper clips, highlighters, glue or correction fluid.
Write your name, Centre number and candidate number on the Answer Sheet in the spaces provided unless
this has been done for you.

There are forty questions on this paper. Answer all questions. For each question there are four possible
answers A, B, C and D.
Choose the one you consider correct and record your choice in soft pencil on the separate Answer Sheet.

Read the instructions on the Answer Sheet very carefully.

Each correct answer will score one mark. A mark will not be deducted for a wrong answer.
Any rough working should be done in this booklet.

This document consists of 16 printed pages.


IB06 06_0625_01/3RP
 UCLES 2006 [Turn over
2

1 A measuring cylinder contains some water. When a stone is put in the water, the level rises.

cm3 cm3
200 200

150 150

100 100

50 50
stone

What is the volume of the stone?

A 50 cm3 B 70 cm3 C 75 cm3 D 125 cm3

2 The graph represents the movement of a body accelerating from rest.

10
speed
m/s 8

0
1 2 3 4 5
time / s

After 5 seconds how far has the body moved?

A 2m B 10 m C 25 m D 50 m

3 A child is standing on the platform of a station, watching the trains.

A train travelling at 30 m / s takes 3 s to pass the child.

What is the length of the train?

A 10 m B 30 m C 90 m D 135 m

© UCLES 2006 0625/01/M/J/06


3

4 Below are four statements about the effects of forces on objects.

Three of the statements are correct.

Which statement is incorrect?

A A force can change the length of an object.


B A force can change the mass of an object.
C A force can change the shape of an object.
D A force can change the speed of an object.

5 A simple balance has two pans suspended from the ends of arms of equal length. When it is
balanced, the pointer is at 0.

pivot arm

pointer

pan X pan Y

Four masses (in total) are placed on the pans, with one or more on pan X and the rest on pan Y.

Which combination of masses can be used to balance the pans?

A 1 g, 1 g, 5 g, 10 g
B 1 g, 2 g, 2 g, 5 g
C 2 g, 5 g, 5 g, 10 g
D 2 g, 5 g, 10 g, 10 g

6 A person measures the length, width, height and mass of a rectangular metal block.

Which of these measurements are needed in order to calculate the density of the metal?

A mass only
B height and mass only
C length, width and height only
D length, width, height and mass

© UCLES 2006 0625/01/M/J/06 [Turn over


4

7 Two forces act on an object.

In which situation is it impossible for the object to be in equilibrium?

A The two forces act in the same direction.


B The two forces act through the same point.
C The two forces are of the same type.
D The two forces are the same size.

8 The diagram shows four models of buses placed on different ramps.

centre centre centre centre


of mass of mass of mass of mass

How many of these models will fall over?

A 1 B 2 C 3 D 4

9 Which form of energy do we receive directly from the Sun?

A chemical
B light
C nuclear
D sound

10 A labourer on a building site lifts a heavy concrete block onto a lorry. He then lifts a light block the
same distance in the same time.

Which of the following is true?

work done in lifting the


power exerted by labourer
blocks

A less for the light block less for the light block
B less for the light block the same for both blocks
C more for the light block more for the light block
D the same for both blocks more for the light block

© UCLES 2006 0625/01/M/J/06


5

11 The diagram shows a thick sheet of glass.

Which edge must it stand on to cause the greatest pressure?

A
B

12 A manometer is being used to measure the pressure of the gas inside a tank. A, B, C and D
show the manometer at different times.

At which time is the gas pressure inside the tank greatest?

A B C D

gas

13 Brownian motion is seen by looking at smoke particles through a microscope.

How do the smoke particles move in Brownian motion?

A all in the same direction


B at random
C in circles
D vibrating about fixed points

© UCLES 2006 0625/01/M/J/06 [Turn over


6

14 Driving a car raises the temperature of the tyres.

This causes the pressure of the air in the tyres to increase.

Why is this?

A Air molecules break up to form separate atoms.


B Air molecules expand with the rise in temperature.
C The force between the air molecules increases.
D The speed of the air molecules increases.

15 To mark a temperature scale on a thermometer, fixed points are needed.

Which is a fixed point?

A the bottom end of the thermometer tube


B the top end of the thermometer tube
C the temperature of pure melting ice
D the temperature of pure warm water

16 Four blocks, made of different materials, are each given the same quantity of internal (heat)
energy.

Which block has the greatest thermal capacity?

A B C D

temperature temperature temperature temperature


rise = 2 oC rise = 4 oC rise = 6 oC rise = 8 oC

© UCLES 2006 0625/01/M/J/06


7

17 A long thin bar of copper is heated evenly along its length.

copper bar

heat

What happens to the bar?

A It becomes lighter.
B It becomes longer.
C It becomes shorter.
D It bends at the ends.

18 A beaker contains water at room temperature.

X water

How could a convection current be set up in the water?

A cool the water at X


B cool the water at Y
C stir the water at X
D stir the water at Y

© UCLES 2006 0625/01/M/J/06 [Turn over


8

19 Two plastic cups are placed one inside the other. Hot water is poured into the inner cup and a lid
is put on top as shown.

lid

small spacer
small air gap
hot water
bench

Which statement is correct?

A Heat loss by radiation is prevented by the small air gap.


B No heat passes through the sides of either cup.
C The bench is heated by convection from the bottom of the outer cup.
D The lid is used to reduce heat loss by convection.

20 Which is the best description of the speed of a water wave?

A the distance between one wave crest and the next


B the distance between the crest of a wave and a trough
C the distance that a particle of water moves up and down in one second
D the distance that a wavefront moves along the surface in one second

© UCLES 2006 0625/01/M/J/06


9

21 Water waves travel more slowly in shallow water than in deep water.

Which diagram shows what will happen to plane waves in deep water when they enter shallow
water?

A B
deep shallow deep shallow

C D
deep shallow deep shallow

22 A ray of light passes through a window.

Which path does it take?

air glass air A


B
C
D

© UCLES 2006 0625/01/M/J/06 [Turn over


10

23 The diagram shows the image of a clock in a plane mirror.

What time is shown?

A 02:25 B 02:35 C 09:25 D 09:35

24 The diagram shows a man standing at X who shouts to a man standing at Y.

X
N

W E

The man’s voice will be heard sooner and more clearly if the wind is blowing towards the

A north.
B south.
C east.
D west.

25 Sounds are made by vibrating objects. A certain object vibrates but a person nearby cannot hear
any sound.

Which statement might explain why nothing is heard?

A The amplitude of the sound waves is too large.


B The frequency of the vibration is too high.
C The sound waves are transverse.
D The speed of the sound waves is too high.

© UCLES 2006 0625/01/M/J/06


11

26 A student investigates which end of a magnetic compass needle is attracted to a bar magnet.

What does the investigation show?

A Both ends of the compass needle are attracted by the north pole of the magnet.
B Both ends of the compass needle are attracted by the south pole of the magnet.
C One end of the compass needle is attracted by the north pole and the other end by the south
pole.
D The compass needle is not attracted by either end of the magnet.

27 From which materials are the coil and the core of an electromagnet made?

coil core

A copper copper
B copper iron
C iron copper
D iron iron

28 What are the symbols used for the units of current and resistance?

unit of current unit of resistance

A A W
B A Ω
C V W
D V Ω

29 When a plastic comb is placed next to a small piece of aluminium foil hanging from a nylon
thread, the foil is repelled by the comb.

Why is this?

A The comb is charged and the foil is uncharged.


B The comb is uncharged and the foil is charged.
C The comb and the foil have charge of opposite signs.
D The comb and the foil have charge of the same sign.

© UCLES 2006 0625/01/M/J/06 [Turn over


12

30 Which symbol represents an electrical component used to store energy?

A B C D

31 Four lamps and four switches are connected to a power supply as shown in the circuit diagram.

When all the switches are closed, all the lamps are lit.

When one of the switches is then opened, only one lamp goes out.

Which switch is opened?

A B

C D

32 Four resistors and an ammeter are connected to a battery as shown.

The ammeter reads 2 A.

Which of the four labelled points in the circuit is the only one where the current is less than 2 A?

A
A

C D

© UCLES 2006 0625/01/M/J/06


13

33 Why is a fuse used in an electrical circuit in a house?

A to increase the circuit resistance


B to keep the power used to a minimum value
C to prevent a short-circuit from occurring
D to stop the cables from carrying too much current

34 An electric power tool is being used outdoors in a shower of rain.

What is the greatest hazard to the user?

A The cable gets hot and causes burns.


B The circuit-breaker cuts off the current.
C The current passes through water and causes a shock.
D The tool rusts.

35 A current-carrying coil in a magnetic field experiences a turning effect.

variable power supply

N S

How can the turning effect be increased?

A increase the number of turns on the coil


B reduce the size of the current
C reverse the direction of the magnetic field
D use thinner wire for the coil

© UCLES 2006 0625/01/M/J/06 [Turn over


14

36 A transformer is to be used to produce a 6 V output from a 24 V input.

coil X coil Y

24 V 6V

What are suitable numbers of turns for coil X and for coil Y?

number of turns number of turns


on coil X on coil Y

A 240 60
B 240 240
C 240 960
D 960 60

37 A cathode-ray tube has an anode and an earthed cathode.

Which line in the table shows the charge and the temperature of the anode?

anode charge anode temperature

A negative cool
B negative hot
C positive cool
D positive hot

© UCLES 2006 0625/01/M/J/06


15

38 The diagram shows five atoms in a radioactive substance. The atoms each give out an α-particle.

1st particle
atom
1

atom
atom 2
5

atom atom
4 3
2nd particle

Atom 1 is the first to give out a particle. Atom 3 is the second to give out a particle.

Which atom will give out the next particle?

A atom 2
B atom 4
C atom 5
D impossible to tell

39 A Geiger counter detects radiation from radioactive sources.

A radioactive source is inside a thick aluminium container as shown.

radioactive source

2m
Geiger counter

thick aluminium container

Which type of radiation from this source is being detected?

A α-particles

B β-particles

C γ-rays
D radio waves

© UCLES 2006 0625/01/M/J/06 [Turn over


16

40 The nucleus of a neutral atom of lithium is represented by 73 Li.

How many protons, electrons and neutrons does the atom contain?

protons electrons neutrons

A 7 7 3
B 3 7 3
C 3 4 4
D 3 3 4

Permission to reproduce items where third-party owned material protected by copyright is included has been sought and cleared where possible. Every
reasonable effort has been made by the publisher (UCLES) to trace copyright holders, but if any items requiring clearance have unwittingly been included, the
publisher will be pleased to make amends at the earliest possible opportunity.

University of Cambridge International Examinations is part of the University of Cambridge Local Examinations Syndicate (UCLES), which is itself a department
of the University of Cambridge.

© UCLES 2006 0625/01/M/J/06


UNIVERSITY OF CAMBRIDGE INTERNATIONAL EXAMINATIONS
International General Certificate of Secondary Education

PHYSICS 0625/01
Paper 1 Multiple Choice
October/November 2006

45 minutes
Additional Materials: Multiple Choice Answer Sheet
Soft clean eraser
Soft pencil (type B or HB is recommended)

READ THESE INSTRUCTIONS FIRST

Write in soft pencil.


Do not use staples, paper clips, highlighters, glue or correction fluid.
Write your name, Centre number and candidate number on the Answer Sheet in the spaces provided unless
this has been done for you.

There are forty questions on this paper. Answer all questions. For each question there are four possible
answers A, B, C and D.
Choose the one you consider correct and record your choice in soft pencil on the separate Answer Sheet.

Read the instructions on the Answer Sheet very carefully.

Each correct answer will score one mark. A mark will not be deducted for a wrong answer.
Any rough working should be done in this booklet.

This document consists of 18 printed pages and 2 blank pages.


IB06 11_0625_01/RP
 UCLES 2006 [Turn over
2

1 A ruler is used to measure the length of a nail.

5.0 6.0 7.0 8.0


cm

What is the length of the nail?

A 1.3 cm B 2.9 cm C 5.2 cm D 8.1 cm

2 A cyclist is riding along a road when an animal runs in front of him. The graph shows the cyclist’s
motion. He sees the animal at P, starts to brake at Q and stops at R.

speed
P Q

R
0
0 time

What is used to find the distance travelled after he applies the brakes?

A the area under line PQ


B the area under line QR
C the gradient of line PQ
D the gradient of line QR

© UCLES 2006 0625/01/O/N/06


3

3 A car travels along the route PQRST in 30 minutes.

5 km

Q 10 km

T
5 km
10 km

R
P

What is the average speed of the car?

A 10 km / hour
B 20 km / hour
C 30 km / hour
D 60 km / hour

4 A newton is a unit of force.

Which quantity is measured in newtons?

A acceleration
B density
C mass
D weight

© UCLES 2006 0625/01/O/N/06 [Turn over


4

5 A student pours liquid into a measuring cylinder.

100
90
measuring
cylinder
80
70
60
50
liquid
40 digital
30 balance
20
10

The student records the volume of the liquid from the scale on the measuring cylinder. He then
puts the measuring cylinder containing the liquid on a balance and records the mass.

What else needs to be measured before the density of the liquid can be calculated?

A the depth of the liquid in the measuring cylinder


B the mass of the empty measuring cylinder
C the temperature of the liquid in the measuring cylinder
D the volume of the empty measuring cylinder

© UCLES 2006 0625/01/O/N/06


5

6 The diagram shows four blocks, each made of glass of density 2.6 g / cm3.

The top of each block has an area of 1 cm2.

Which block has a mass of 13 g?

15 cm

10 cm

5 cm

2 cm

A B C D

7 A heavy beam is resting on two supports, so that there are three forces acting on it.

R1 R2

The beam is in equilibrium.

Which statement is correct?

A All the forces are equal in value.


B The forces are in one direction and their turning effects are in the opposite direction.
C The resultant force is zero and the resultant turning effect is zero.
D The total upward force is twice the total downward force.

© UCLES 2006 0625/01/O/N/06 [Turn over


6

8 The diagram shows sections of four objects of equal mass. The position of the centre of mass of
each object has been marked with a cross.

Which object is the most stable?

A B C D

9 Which source of energy uses the production of steam to generate electricity?

A hydroelectric
B nuclear
C tides
D waves

10 A cyclist travels down a hill from rest at point X without pedalling.

The cyclist applies his brakes and the cycle stops at point Y.

hill

Which energy changes have taken place between X and Y?

A kinetic → internal (heat) → gravitational potential

B kinetic → gravitational potential → internal (heat)

C gravitational potential → internal (heat) → kinetic

D gravitational potential → kinetic → internal (heat)

© UCLES 2006 0625/01/O/N/06


7

11 To prevent a cement mixer sinking into soft ground, the mixer is placed on a large flat board.

cement mixer

board
soft ground

Why does this prevent the mixer sinking?

A The large area decreases the pressure on the ground.


B The large area increases the pressure on the ground.
C The large area decreases the weight on the ground.
D The large area increases the weight on the ground.

12 The diagram shows a simple mercury barometer.

mercury

X
Y
Z

The atmospheric pressure increases.

Which distance increases?

A VW B WY C XY D XZ

© UCLES 2006 0625/01/O/N/06 [Turn over


8

13 A gas cylinder is left outside on a sunny day.

The Sun heats the gas inside the cylinder.

What happens to the gas molecules?

A They collide less often.


B They expand.
C They move closer together.
D They move more rapidly.

14 Water spilled on the ground on a hot day evaporates.

Which diagram represents the change in arrangement of the particles in the water as it
evaporates?

© UCLES 2006 0625/01/O/N/06


9

15 A block of ice is heated until it has all melted. The water that is produced is then heated until it
boils.

Which line in the table states what happens to the temperature of the ice while it is melting, and
to the temperature of the water while it is boiling?

temperature of ice temperature of water


while it is melting while it is boiling

A increases increases
B increases stays the same
C stays the same increases
D stays the same stays the same

16 A thermometer with no scale is taped to a ruler as shown. When placed in steam, the mercury
level rises to 22 cm. When placed in pure melting ice, the mercury level falls to 2 cm.

cm
30
28
26
24
22
20
18
16
14
12
10
mercury
8
6
4
2
0

Which temperature is shown by the mercury level in the diagram?

A 6 °C B 8 °C C 30 °C D 40 °C

© UCLES 2006 0625/01/O/N/06 [Turn over


10

17 Which line in the table is correct about conduction and convection?

conduction convection

A can happen in a solid can happen in a solid


B can happen in a solid only happens in fluids
C only happens in fluids can happen in a solid
D only happens in fluids only happens in fluids

18 A heating element is positioned in a narrow sealed tube of liquid.

What would be the best place to position the heating element in order to obtain the best
circulation of the liquid through the tube?

A B liquid

D C

© UCLES 2006 0625/01/O/N/06


11

19 In a ripple tank experiment, plane water-waves meet a straight barrier with a wide gap in it.

wave
motion

barrier

Which diagram shows the wave pattern beyond the barrier?

A B

C D

20 The diagram shows a wave.

displacement

0
distance

X Y

How many wavelengths are there between X and Y?


2
A 3
B 1 C 1 21 D 3

© UCLES 2006 0625/01/O/N/06 [Turn over


12

21 A parallel beam of light falls on a converging lens.

Which diagram shows what happens to the beam of light?

A B C D

22 The image of a clock face as seen in a plane mirror is shown.

21

3 9

What is the time on the clock?

A 1.25 B 1.35 C 10.25 D 10.35

23 A police car with its siren sounding is stationary in heavy traffic. A pedestrian notices that,
although the loudness of the sound produced does not change, the pitch varies.

Which line in the table describes the amplitude and the frequency of the sound?

amplitude frequency

A constant varying
B constant constant
C varying constant
D varying varying

© UCLES 2006 0625/01/O/N/06


13

24 A sports field is next to a large school building. At the far side of the sports field, a student sees a
groundsman hammer a pole into the ground.

school building

student
hammer

pole

about 400 m

The student hears two bangs each time the hammer hits the pole.

Why does the student hear two bangs?

first bang caused by second bang caused by

A sound of hammer hitting pole sound of pole hitting hammer


B sound reaching left ear sound reaching right ear
C sound reaching student directly sound due to echo from school building
D sound reflected back from school building sound reaching student directly

25 A student uses three small plotting compasses to investigate the magnetic field around a bar
magnet.

Which diagram shows the directions in which the compass needles point?

A B

N S N S

C D

N S N S

© UCLES 2006 0625/01/O/N/06 [Turn over


14

26 The ends of three metal rods are tested by holding end Q of rod 1 close to the others in turn.

R T

Q S U

rod 1 rod 2 rod 3

The results are as follows.

End Q: attracts end R,


attracts end S,
attracts end T,
repels end U.

Which of the metal rods is a magnet?

A rod 1 only
B rod 1 and rod 2 only
C rod 1 and rod 3 only
D rod 3 only

27 A student wishes to measure the electromotive force (e.m.f.) of a battery and the potential
difference (p.d.) across a resistor.

She has the resistor, the battery and some connecting wires.

What else does she need?

A a voltmeter only
B an ammeter only
C an ammeter and a voltmeter
D a force meter (newton meter) and a voltmeter

28 Which particle does not experience a force due to an electric field?

A α-particle
B electron
C neutron
D proton

© UCLES 2006 0625/01/O/N/06


15

29 A student uses a length of wire as a resistor. He discovers that the resistance of the wire is too
small.

To be certain of making a resistor of higher value, he should use a piece of wire that is

A longer and thicker.


B longer and thinner.
C shorter and thicker.
D shorter and thinner.

30 Which graph shows how the resistance of a thermistor changes with temperature?

A B

resistance / Ω resistance / Ω

0 0
0 temperature / °C 0 temperature / °C

C D

resistance / Ω resistance / Ω

0 0
0 temperature / °C 0 temperature / °C

31 In the circuit below, one of the lamps breaks, causing all the other lamps to go out.

Which lamp breaks?

A C D

© UCLES 2006 0625/01/O/N/06 [Turn over


16

32 Circuit-breakers are used with electrical appliances as safety devices.

Which description is correct for a circuit-breaker?

position action when overloaded

A connected in the live wire melts


B connected in the live wire operates an electromagnet
C connected to the casing of the appliance melts
D connected to the casing of the appliance operates an electromagnet

33 An electric heater is connected to the mains using insulated copper wires. The wires become
very warm.

What can be done to prevent so much heat being produced in the connecting wires?

A Use thicker copper wires.


B Use thinner copper wires.
C Use thicker insulation.
D Use thinner insulation.

34 The diagram shows an electrical device.

rotation
magnet

S
commutator
N
carbon brush

battery

What is this electrical device?

A a d.c. motor
B an a.c. generator
C a magnetising coil
D a transformer

© UCLES 2006 0625/01/O/N/06


17

35 The graph shows the voltage input to a step-down transformer.

12 V
voltage
input
0
time

Which diagram shows the voltage output from the transformer?

A B
12 V 12 V
voltage voltage
output output
0 0
time time

C D
12 V 12 V
voltage voltage
output output
0 0
time time

36 Particles are emitted by a heated cathode in a cathode-ray tube.

heater particles
cathode

anode

What are these particles?

A atoms
B electrons
C neutrons
D protons

© UCLES 2006 0625/01/O/N/06 [Turn over


18

37 Which line in the table describes the nature of an α-particle and of a γ-ray?

α-particle γ-ray

A helium nucleus electromagnetic radiation


B helium nucleus electron
C proton electromagnetic radiation
D proton electron

38 The count rates of four radioactive sources were measured at the same time on three
consecutive days.

Which source has a half-life of two days?

Monday Tuesday Wednesday

A 100 50 25
B 200 140 100
C 300 300 300
D 400 200 100

39 Which statement is true of all neutral atoms?

A The number of electrons equals the number of nucleons.


B The number of neutrons equals the number of protons.
C The number of nucleons equals the number of neutrons.
D The number of protons equals the number of electrons.

40 There are three nuclides of hydrogen.

nuclide 1 nuclide 2 nuclide 3

1 2 3
1H H
1 1H

Which of these nuclides have the same number of protons in their nuclei?

A 1 and 2 only
B 2 and 3 only
C all of them
D none of them

© UCLES 2006 0625/01/O/N/06


19

BLANK PAGE

0625/01/O/N/06
20

BLANK PAGE

Permission to reproduce items where third-party owned material protected by copyright is included has been sought and cleared where possible. Every
reasonable effort has been made by the publisher (UCLES) to trace copyright holders, but if any items requiring clearance have unwittingly been included, the
publisher will be pleased to make amends at the earliest possible opportunity.

University of Cambridge International Examinations is part of the University of Cambridge Local Examinations Syndicate (UCLES), which is itself a department
of the University of Cambridge.

0625/01/O/N/06
UNIVERSITY OF CAMBRIDGE INTERNATIONAL EXAMINATIONS
International General Certificate of Secondary Education

PHYSICS 0625/01
Paper 1 Multiple Choice May/June 2007
45 minutes
Additional Materials: Multiple Choice Answer Sheet
Soft clean eraser
*4300881997*

Soft pencil (type B or HB is recommended)

READ THESE INSTRUCTIONS FIRST

Write in soft pencil.


Do not use staples, paper clips, highlighters, glue or correction fluid.
Write your name, Centre number and candidate number on the Answer Sheet in the spaces provided
unless this has been done for you.

There are forty questions on this paper. Answer all questions. For each question there are four possible
answers A, B, C and D.
Choose the one you consider correct and record your choice in soft pencil on the separate Answer Sheet.

Read the instructions on the Answer Sheet very carefully.

Each correct answer will score one mark. A mark will not be deducted for a wrong answer.
Any rough working should be done in this booklet.

This document consists of 18 printed pages and 2 blank pages.

IB07 06_0625_01/3RP
© UCLES 2007 [Turn over
2

1 The diagram shows a thick-walled tube. The thickness of the wall is 3 mm.

0 cm 1 2 3 4 5 6 7 8 9 10 11

What is the internal diameter d of the tube?

A 2.8 cm B 3.1 cm C 3.4 cm D 7.4 cm

2 The graph shows the progress of an athlete in a 100 m race.

100

distance
travelled / m
80

60

40

20

0
0 1 2 3 4 5 6 7 8 9 10 11 12 13 14
time / s

What time was taken to travel 10 m from the start?

A 2.4 s B 2.8 s C 65 s D 70 s

© UCLES 2007 0625/01/M/J/07


3

3 Two stones of different weight fall at the same time from a table. Air resistance may be ignored.

What will happen and why?

what will happen why

A both stones hit the floor at the same time the acceleration of free fall is constant
B both stones hit the floor at the same time they fall at constant speed
C the heavier stone hits the floor first acceleration increases with weight
D the heavier stone hits the floor first speed increases with weight

4 The diagram shows a bird in flight.

In which direction does the weight of the bird act?

A C

© UCLES 2007 0625/01/M/J/07 [Turn over


4

5 The mass of a full bottle of cooking oil is 1.30 kg.

When exactly half of the oil has been used, the mass of the bottle plus the remaining oil is
0.90 kg.

What is the mass of the empty bottle?

A 0.40 kg B 0.50 kg C 0.65 kg D 0.80 kg

6 Which of the following is a unit of density?

A cm3 / g B g / cm2 C g / cm3 D kg / m2

7 Each of the solids shown in the diagram has the same mass.

Which solid has the greatest density?

A B C D

2 cm
1 cm 2 cm
1 cm

2 cm 2 cm 2 cm 2 cm
1 cm 1 cm 1 cm 1 cm

© UCLES 2007 0625/01/M/J/07


5

8 A force acts on a moving rubber ball.

How many of the following changes could happen to the ball because of the force?

• a change in direction

• a change in shape

• a change in mass

• a change in speed

A 1 B 2 C 3 D 4

9 A light aircraft stands at rest on the ground. It stands on three wheels, one at the front and two
further back.

Which point could be its centre of mass?

A B C D

10 Which form of energy is used to generate electrical energy in a tidal power station?

A chemical energy
B gravitational energy
C internal energy (thermal energy)
D nuclear energy

11 Which line in the table gives an example of the stated form of energy?

form of energy example

A gravitational the energy due to the movement of a train


B internal the energy due to the flow of cathode rays in a cathode ray tube
C kinetic the energy due to the position of a swimmer standing on a
high diving board
D strain the energy due to the compression of springs in a car seat

© UCLES 2007 0625/01/M/J/07 [Turn over


6

12 A pressure is measured using a manometer as shown in the diagram.

pressure to
be measured

water

The water in the manometer is replaced with a liquid which is more dense.

How does the value of h change?

A It becomes zero.
B It decreases, but not to zero.
C It stays the same.
D It increases.

13 A cylinder is filled with a gas and then sealed, so that the gas has a fixed volume.

The gas molecules are given energy so that their average speed increases.

What happens to the pressure and to the temperature of the gas in the cylinder?

pressure temperature

A decreases decreases
B decreases increases
C increases decreases
D increases increases

© UCLES 2007 0625/01/M/J/07


7

14 Diagram 1 shows apparatus being used to observe smoke particles.

Diagram 2 shows how a smoke particle moves randomly.

diagram 1 diagram 2

random
movement
microscope

air molecules
light and
smoke particles

Why do the smoke particles move randomly?

A They are hit by air molecules.


B They are less dense than air.
C They are moved by convection currents.
D They gain energy from the light.

15 The graph shows how the temperature of hot liquid wax changes with time as the wax is allowed
to cool.

At which labelled point on the graph are both liquid wax and solid wax present?

temperature
A
B C

0
0 time

© UCLES 2007 0625/01/M/J/07 [Turn over


8

16 1 kg of water and 1 kg of aluminium are heated to the same temperature and then allowed to cool
in a room.

Why does the aluminium cool more quickly than the water?

A Aluminium contracts more than water.


B Aluminium does not evaporate but water does.
C Aluminium has a higher thermal capacity than water.
D Aluminium has a lower thermal capacity than water.

17 Bread can be cooked by placing it below, but not touching, a heating element.

heating element

bread

Which process transfers thermal energy from the heating element to the bread?

A conduction
B convection
C insulation
D radiation

© UCLES 2007 0625/01/M/J/07


9

18 The diagram shows a refrigerator. The cooling unit is placed at the top. The cooling unit cools the
air near it.

cooling unit

What happens to the density of this air as it cools and how does it move?

density of the air movement of the air

A decreases moves down


B decreases stays where it is
C increases moves down
D increases stays where it is

19 Water waves are reflected at a plane surface.

Which property of the waves is changed by the reflection?

A direction
B frequency
C speed
D wavelength

© UCLES 2007 0625/01/M/J/07 [Turn over


10

20 Plane water waves travel from a shallow region into a deeper region. They travel more quickly in
the deeper water.

shallow water

boundary
wave
direction
deep water

Which diagram shows the wave pattern in the deeper water?

A B

C D

© UCLES 2007 0625/01/M/J/07


11

21 A man sees a stone at the bottom of a pool of water.

Which path could be taken by light from the stone to the man?

man

A B C D
air

water

stone

22 A ray of light is reflected by two parallel plane mirrors X and Y.

mirror X

30°

mirror Y

Which statement is correct?

A The angle of incidence at mirror X is 30°.

B The angle of incidence at mirror Y is 60°.

C The angle of reflection at mirror X is 120°.

D The angle of reflection at mirror Y is 0°.

© UCLES 2007 0625/01/M/J/07 [Turn over


12

23 Music is produced by the loudspeaker of a radio.

Which property of the sound wave increases when the music is made louder?

A amplitude
B frequency
C speed
D wavelength

24 A starting pistol is fired 640 m away from a spectator.

640 m

spectator

The spectator hears the sound of the starting pistol two seconds after seeing the flash from the
gun.

What is the speed of sound in air?

A 160 m / s B 320 m / s C 640 m / s D 1280 m / s

25 Small particles of metal are scattered near a bar magnet to show the pattern of the magnetic
field.

Which metal is suitable?

A aluminium
B brass
C copper
D iron

© UCLES 2007 0625/01/M/J/07


13

26 A strong electromagnet is used to attract pins.

core

pins
coil

What happens when the current in the coil is halved?

A No pins are attracted.


B Fewer pins are attracted.
C The same number of pins is attracted.
D Many more pins are attracted.

27 Four plotting compasses are placed near a bar magnet.

Which plotting compass is shown pointing in the correct direction?

A B

N
C

D S

28 Four wires are made from the same material.

Which wire has the greatest resistance?

length of wire / cm diameter of wire / mm

A 50 0.1
B 50 0.2
C 100 0.1
D 100 0.2

© UCLES 2007 0625/01/M/J/07 [Turn over


14

29 A plastic rod is rubbed with a dry cloth and becomes positively charged.

Why has the rod become positively charged?

A It has gained electrons.


B It has gained neutrons.
C It has lost electrons.
D It has lost neutrons.

30 Which circuit is a variable potential divider (potentiometer)?

A B

output

output

C D

output
output

© UCLES 2007 0625/01/M/J/07


15

31 The circuit contains a relay.

relay lamp 2

switch S

lamp 1

When switch S is closed, what is the state of the lamps?

lamp 1 lamp 2

A on on
B on off
C off on
D off off

32 Which diagram shows a circuit that will allow the lamps to be switched on and off independently?

A B C D

33 What is the symbol for a fuse?

A B C D

© UCLES 2007 0625/01/M/J/07 [Turn over


16

34 A cable in a house is carrying too much current.

What is the greatest danger?

A appliances not working


B electric shock
C fire
D low power

35 A coil is rotated steadily between the poles of a magnet. The coil is connected to an oscilloscope,
which shows a graph of voltage output against time.

N S oscilloscope

contact
contact

Which graph shows the voltage output against time?

A B C D

36 The diagram shows a lamp connected to a transformer.

40 turns 200 turns

12 V a.c.

What is the potential difference across the lamp?

A 2.4 V B 12 V C 60 V D 240 V

© UCLES 2007 0625/01/M/J/07


17

37 A beam of cathode rays passes through an electric field between two parallel plates.

+ + + + + +

cathode rays

– – – – – –

In which direction is the beam deflected?

A into the page


B out of the page
C towards the bottom of the page
D towards the top of the page

38 What are the most penetrating and the least penetrating types of radiation?

most penetrating least penetrating

A α-particles β-particles
B β-particles α-particles
C γ-rays α-particles
D γ-rays β-particles

39 The half-life of a radioactive substance is 5 hours. A sample is tested and found to contain 0.48 g
of the substance.

How much of the substance was present in the sample 20 hours before the sample was tested?

A 0.03 g B 0.12 g C 1.92 g D 7.68 g

© UCLES 2007 0625/01/M/J/07 [Turn over


18

40 The data below relates to the nucleus of a particular neutral atom of nitrogen.

proton number Z=7


nucleon number A = 17

Which row represents the correct number of neutrons and electrons in this atom?

number of neutrons number of electrons

A 10 7
B 10 10
C 17 7
D 17 10

© UCLES 2007 0625/01/M/J/07


19

BLANK PAGE

0625/01/M/J/07
20

BLANK PAGE

Permission to reproduce items where third-party owned material protected by copyright is included has been sought and cleared where possible. Every
reasonable effort has been made by the publisher (UCLES) to trace copyright holders, but if any items requiring clearance have unwittingly been included, the
publisher will be pleased to make amends at the earliest possible opportunity.

University of Cambridge International Examinations is part of the Cambridge Assessment Group. Cambridge Assessment is the brand name of University of
Cambridge Local Examinations Syndicate (UCLES), which is itself a department of the University of Cambridge.

0625/01/M/J/07
UNIVERSITY OF CAMBRIDGE INTERNATIONAL EXAMINATIONS
International General Certificate of Secondary Education

PHYSICS 0625/01
Paper 1 Multiple Choice October/November 2007
45 minutes
Additional Materials: Multiple Choice Answer Sheet
Soft clean eraser
*9483001639*

Soft pencil (type B or HB is recommended)

READ THESE INSTRUCTIONS FIRST

Write in soft pencil.


Do not use staples, paper clips, highlighters, glue or correction fluid.
Write your name, Centre number and candidate number on the Answer Sheet in the spaces provided unless
this has been done for you.

There are forty questions on this paper. Answer all questions. For each question there are four possible
answers A, B, C, and D.
Choose the one you consider correct and record your choice in soft pencil on the separate Answer Sheet.

Read the instructions on the Answer Sheet very carefully.

Each correct answer will score one mark. A mark will not be deducted for a wrong answer.
Any rough working should be done in this booklet.

This document consists of 18 printed pages and 2 blank pages.

IB07 11_0625_01/3RP
© UCLES 2007 [Turn over
2

1 Some water is poured into four tubes of different cross-sectional areas.

Which tube contains the largest volume of water?

A B C D

area = 10 cm2 area = 20 cm2 area = 30 cm2 area = 40 cm2

6 cm
4 cm
3 cm
2 cm

2 Four students try to explain what is meant by acceleration.

Which student makes a correct statement?

A It is related to the changing speed of an object.


B It is the distance an object travels in one second.
C It is the force acting on an object divided by the distance it travels in one second.
D It is the force acting on an object when it is near to the Earth.

3 A tennis player hits a ball hard and 0.4 s later hears an echo from a wall.

The speed of sound in air is 330 m / s.

How far away is the player from the wall?

A 66 m B 132 m C 264 m D 825 m

 UCLES 2007 0625/01/O/N/07


3

4 What are the correct units for force and for weight?

force weight

A kg kg
B kg N
C N kg
D N N

5 The table shows the weight of a 10 kg mass on each of five planets.

planet weight of a 10 kg mass / N

Mercury 40
Venus 90
Earth 100
Mars 40
Jupiter 250

On which planets would an astronaut have a smaller weight than on Earth?

A Mercury, Mars and Jupiter


B Mercury, Venus and Mars
C Mercury, Venus and Jupiter
D Venus, Mars and Jupiter

6 A student is trying to find the density of water and of a large, regularly-shaped solid.

Which apparatus is needed to find the density of both?

A balance, clock, ruler


B balance, measuring cylinder, ruler
C balance, measuring cylinder, string
D clock, ruler, string

 UCLES 2007 0625/01/O/N/07 [Turn over


4

7 A metal drum has a mass of 200 kg when empty and 1000 kg when filled with 1.0 m3 of
methylated spirit.

What is the density of methylated spirit?

A 0.0050 kg / m3
B 0.11 kg / m3
C 800 kg / m3
D 1000 kg / m3

8 An empty glass is placed on a join between two tables as shown. The glass remains stable.

Which point is the centre of mass of the glass?

B
C

 UCLES 2007 0625/01/O/N/07


5

9 An aeroplane is in equilibrium.

The diagram shows the forces acting on the aeroplane.

lift force from wings

force from air


engine resistance

weight

Which statement about the forces is correct?

force from engine lift force from wings

A equal to air resistance equal to weight


B equal to air resistance greater than weight
C greater than air resistance equal to weight
D greater than air resistance greater than weight

10 A student adds loads to an elastic cord. He measures the length of the cord for each load.

He then plots a graph from the results.

0
0 1 2 3 4 load / N

Which length is plotted on the vertical axis?

A measured length
B original length
C (measured length – original length)
D (measured length + original length)

 UCLES 2007 0625/01/O/N/07 [Turn over


6

11 A person uses chemical energy to run up some stairs.

She stops at the top of the stairs.

What has the chemical energy been converted to when she is at the top of the stairs?

A kinetic energy and gravitational energy


B kinetic energy and strain energy
C gravitational energy and heat energy
D strain energy and heat energy

12 Electrical energy may be obtained from nuclear fission.

In what order is the energy transferred in this process?

A nuclear fuel → generator → reactor and boiler → turbines

B nuclear fuel → generator → turbines → reactor and boiler

C nuclear fuel → reactor and boiler → generator → turbines

D nuclear fuel → reactor and boiler → turbines → generator

 UCLES 2007 0625/01/O/N/07


7

13 The diagram shows a simple mercury barometer, used to measure atmospheric pressure.

P
L

mercury

Atmospheric pressure increases.

What happens to the level L and to the pressure at P?

level L pressure at P

A falls increases
B falls stays the same
C rises increases
D rises stays the same

14 The gas in a container is heated but is kept at constant volume.

Why does the gas pressure increase?

A The molecules expand.


B The molecules increase in mass.
C The molecules move further apart.
D The molecules move more rapidly.

 UCLES 2007 0625/01/O/N/07 [Turn over


8

15 The table lists the melting points and the boiling points of four different substances A, B, C and D.

Which substance is a gas at 25 °C?

substance melting point / °C boiling point / °C

A –219 –183
B –7 58
C 98 890
D 1083 2582

16 The diagram shows four blocks of steel. The same quantity of heat is given to each block.

Which block shows the greatest rise in temperature?

A B C D

17 A wooden wheel can be strengthened by putting a tight circle of iron around it.

wooden wheel

iron circle

Which action would make it easier to fit the circle over the wood?

A cooling the iron circle


B heating the iron circle
C heating the wooden wheel
D heating the wooden wheel and cooling the iron circle

 UCLES 2007 0625/01/O/N/07


9

18 Which statement refers to convection?

A It does not involve energy transfer.


B It is the transfer of heat energy without the movement of particles.
C It only occurs in liquids or gases.
D It only occurs in solids.

19 Spoons made of different materials were placed in four cups of coffee poured from the same jug.

Which spoon will be hottest to touch at end X?

experiment A experiment B experiment C experiment D

wooden X steel X white plastic X black plastic X


spoon spoon spoon spoon

20 Which line gives an example of a longitudinal wave and describes its vibrations?

example of a
vibrations
longitudinal wave

A light wave at right angles to the direction the wave travels


B Iight wave in the same direction as the wave travels
C sound wave at right angles to the direction the wave travels
D sound wave in the same direction as the wave travels

21 The diagram shows a section through a series of waves on water.

Which dotted line shows the position of the still water surface after the waves have passed?

A
B
C
D

 UCLES 2007 0625/01/O/N/07 [Turn over


10

22 A student shines a narrow beam of white light into a prism as shown in the diagram. He sees a
spectrum of colours emerging from the prism.

narrow beam X
Y spectrum
of white light
Z of colours

Which three colours does he see at X, at Y and at Z?

X Y Z

A blue yellow red


B red blue yellow
C red yellow blue
D yellow red blue

23 An object placed in front of a plane mirror at O produces an image at I.

If the object moves towards the mirror in the direction shown by the arrow, in which direction does
the image move?

D
C
B

A
I

 UCLES 2007 0625/01/O/N/07


11

24 A girl stands at a distance from a large building. She claps her hands and a short time later hears
an echo.

Why is an echo produced when the sound waves hit the building?

A The sound waves are absorbed.


B The sound waves are diffracted.
C The sound waves are reflected.
D The sound waves are refracted.

25 The graph represents a sound wave. The horizontal (x) axis represents time.

The frequency of the sound is increased.

The graphs below are shown to the same scale. Which graph represents the new sound wave?

A B
y y

x x
C D
y y

x x

 UCLES 2007 0625/01/O/N/07 [Turn over


12

26 Which materials are suitable for making a permanent magnet and the core of an electromagnet?

permanent magnet core of an electromagnet

A iron iron
B iron steel
C steel iron
D steel steel

27 Three charged balls, P, Q and R, are suspended by insulated threads. Ball P is negatively
charged.

insulated thread

P – Q

Q R

What are the charges on Q and on R?

Q R

A positive positive
B positive negative
C negative positive
D negative negative

 UCLES 2007 0625/01/O/N/07


13

28 The circuit shows a current I in a resistor of resistance R.

3.0 V

Which line gives possible values of I and of R?

I/A R/Ω

A 1.5 1.5
B 1.5 2.0
C 6.0 2.0
D 4.0 12.0

29 The circuit shown in the diagram contains an unknown component X, hidden in a box.

The voltage-current graph for X is as shown.

variable voltage supply


+ –
voltage

X
0
V 0 current

What is the component X?

A a capacitor
B a closed switch
C a metallic resistor
D an open switch

 UCLES 2007 0625/01/O/N/07 [Turn over


14

30 Four lamps are connected in a circuit as shown in the diagram.

Each lamp is designed to operate at 12 V.

12 V

The circuit is now switched on.

Which statement is correct?

A Each lamp can be switched off independently.


B If one lamp breaks all the others will stay alight.
C The current is the same in all the lamps.
D The lamps will all light at normal brightness.

31 The diagram shows a battery connected to three identical resistors. Four ammeters A, B, C and
D are connected in the circuit.

Which ammeter shows the smallest reading?

A A A D

A
B

A
C

 UCLES 2007 0625/01/O/N/07


15

32 In the circuit shown, the switch is closed for a long time, then opened.

voltmeter 1
V

V V
voltmeter 2 voltmeter 3

Immediately after the switch is opened, which voltmeters will read zero?

A voltmeter 1 only
B voltmeter 2 only
C voltmeter 3 only
D voltmeter 1, voltmeter 2 and voltmeter 3

33 A householder asks an electrician to install a mains electrical socket in her bathroom so that she
may use a hairdryer there. The electrician refuses to do this because it would be dangerous.

Why would installing the socket be dangerous?

A The current drawn by the hairdryer would cause overheating in the cables.
B The handling of electrical equipment in damp conditions could cause an electric shock.
C The hot air produced by the hairdryer would cause the fuse to melt.
D The temperature in the bathroom would damage the insulation.

34 A mains electrical circuit uses insulated copper cable and the cable overheats.

To prevent the cable overheating, how should the cable be changed, and why?

A Use thicker copper cable which has less resistance.


B Use thicker insulation which stops the heat escaping.
C Use thinner copper cable which has more resistance.
D Use thinner insulation which allows less heat to escape.

 UCLES 2007 0625/01/O/N/07 [Turn over


16

35 How is electricity transmitted over large distances and why is it transmitted in this way?

how why

A at high voltage for safety


B at high voltage to reduce energy loss
C at low voltage for safety
D at low voltage to reduce energy loss

36 Which parts of an a.c. generator slide past each other when the generator is working?

A brushes and coil


B coil and magnets
C magnets and slip rings
D slip rings and brushes

37 The diagram shows a cathode-ray tube.

electron top
beam

screen
Y2 X2
heater
Y1 X1

cathode anode Y-plates X-plates bottom

What must be done to deflect the electron beam upwards?

A make X1 more positive than X2


B make X2 more positive than X1
C make Y1 more positive than Y2
D make Y2 more positive than Y1

 UCLES 2007 0625/01/O/N/07


17

38 A sheet of paper is placed between a radioactive source and a detector.

radioactive detector
source sheet of paper

Which types of radiation can pass through the paper?

A α-particles and β-particles only

B α-particles and γ-rays only

C β-particles and γ-rays only

D α-particles, β-particles and γ-rays

39 A sample of radioactive isotope is decaying.

The nuclei of which atoms will decay first?

A impossible to know, because radioactive decay is random


B impossible to know, unless the age of the material is known
C atoms near the centre, because they are surrounded by more atoms
D atoms near the surface, because the radiation can escape more easily

 UCLES 2007 0625/01/O/N/07 [Turn over


18

40 An atom of the element lithium has a nucleon number of 7 and a proton number of 3.

Which diagram represents a neutral atom of lithium?

A B

– –

n + +
+ + +
+ n n key
n = a neutron
– –
+ = a proton

– = an electron

C D (not to scale)

– –
– –
n n
+n+ +n+
n n n n
+ +

– –
– –

 UCLES 2007 0625/01/O/N/07


19

BLANK PAGE

0625/01/O/N/07
20

BLANK PAGE

Permission to reproduce items where third-party owned material protected by copyright is included has been sought and cleared where possible. Every
reasonable effort has been made by the publisher (UCLES) to trace copyright holders, but if any items requiring clearance have unwittingly been included, the
publisher will be pleased to make amends at the earliest possible opportunity.

University of Cambridge International Examinations is part of the Cambridge Assessment Group. Cambridge Assessment is the brand name of University of
Cambridge Local Examinations Syndicate (UCLES), which is itself a department of the University of Cambridge.

0625/01/O/N/07
UNIVERSITY OF CAMBRIDGE INTERNATIONAL EXAMINATIONS
International General Certificate of Secondary Education

PHYSICS 0625/01
Paper 1 Multiple Choice May/June 2008
45 minutes
Additional Materials: Multiple Choice Answer Sheet
Soft clean eraser
*1101906355*

Soft pencil (type B or HB recommended)

READ THESE INSTRUCTIONS FIRST

Write in soft pencil.


Do not use staples, paper clips, highlighters, glue or correction fluid.
Write your name, Centre number and candidate number on the Answer Sheet in the spaces provided
unless this has been done for you.

There are forty questions on this paper. Answer all questions. For each question there are four possible
answers A, B, C and D.
Choose the one you consider correct and record your choice in soft pencil on the separate Answer Sheet.

Read the instructions on the Answer Sheet very carefully.

Each correct answer will score one mark. A mark will not be deducted for a wrong answer.
Any rough working should be done in this booklet.

This document consists of 18 printed pages and 2 blank pages.

IB08 06_0625_01/3RP
© UCLES 2008 [Turn over
2

1 Some liquid is heated in a flask.

The diagrams show the height of the liquid in the tube when the liquid is cold and when it is hot.

cold hot

5 5
tube

4 4

3 3

2 2

scale 1 1
cm cm

liquid

flask

What is the difference in the heights?

A 1.7 cm B 2.8 cm C 3.2 cm D 4.5 cm

2 The speed-time graph shown is for a bus travelling between stops.

Where on the graph is the acceleration of the bus the greatest?

C
speed

A D

time

© UCLES 2008 0625/01/M/J/08


3

3 The circuit of a motor racing track is 3 km in length. In a race, a car goes 25 times round the
circuit in 30 minutes.

What is the average speed of the car?

A 75 km / hour
B 90 km / hour
C 150 km / hour
D 750 km / hour

4 The force of gravity acting on an astronaut in an orbiting spacecraft is less than when she is on
the Earth’s surface.

Compared with being on the Earth’s surface, how do her mass and weight change when she
goes into orbit?

mass in orbit weight in orbit

A decreases decreases
B decreases unchanged
C unchanged decreases
D unchanged unchanged

5 When two blocks X and Y are placed on a uniform beam, the beam balances on a pivot at its
centre as shown.

Y
X

pivot

What does this show about X and Y?

A They have the same mass and the same density.


B They have the same mass and the same weight.
C They have the same volume and the same density.
D They have the same volume and the same weight.

© UCLES 2008 0625/01/M/J/08 [Turn over


4

6 The diagram shows a rectangular metal block measuring 10 cm × 5.0 cm × 2.0 cm.

10 cm
5.0 cm
2.0 cm

Its mass is 250 g.

What is the density of the metal?

A 0.20 g / cm3 B 0.40 g / cm3 C 2.5 g / cm3 D 5.0 g / cm3

7 The diagram shows an experiment to find the density of a liquid.

cm3 cm3
50 50
40 measuring 40
cylinder
30 30
liquid
20 20
10 balance 10

200 g 250 g

What is the density of the liquid?

A 0.5 g / cm3 B 2.0 g / cm3 C 8.0 g / cm3 D 10.0 g / cm3

8 What are the conditions for equilibrium?

resultant force resultant turning


acting effect acting

A yes yes
B yes no
C no yes
D no no

© UCLES 2008 0625/01/M/J/08


5

9 The diagram shows four objects standing on a flat surface.

The centre of mass of each object is marked M.

Which object will fall over?

A B C D

M M M M

10 A stone is thrown from the edge of a cliff. Its path is shown in the diagram.

3
ground

In which position does the stone have its greatest kinetic energy and in which position does it
have its least gravitational energy?

greatest kinetic least gravitational


energy energy

A 1 2
B 2 3
C 3 1
D 3 3

© UCLES 2008 0625/01/M/J/08 [Turn over


6

11 A worker is lifting boxes of identical weight from the ground onto a moving belt.

At first, it takes him 2 s to lift each box. Later in the day, it takes him 3 s.

Which statement is correct?

A Later in the day, less work is done in lifting each box.


B Later in the day, more work is done in lifting each box.
C Later in the day, less power is developed in lifting each box.
D Later in the day, more power is developed in lifting each box.

12 A manometer is used to indicate the pressure in a steel vessel, as shown in the diagram.

liquid
manometer
steel vessel

What value does the liquid manometer give for the pressure in the vessel?

A It is zero.
B It is between zero and atmospheric pressure.
C It is equal to atmospheric pressure.
D It is greater than atmospheric pressure.

13 Four glass tanks contain water.

In which tank is the pressure of the water on the base greatest?

A B C D

4m 3m
2m 6m
3m 4m 1m 4m
2m 2m 2m 6m

© UCLES 2008 0625/01/M/J/08


7

14 Which line in the table describes the properties of solids and of liquids at a fixed temperature?

solids liquids

A definite volume and definite shape no definite volume but definite shape
B no definite volume but definite shape definite volume and definite shape
C definite volume and definite shape definite volume but no definite shape
D no definite volume but definite shape no definite volume and no definite shape

15 Air is pumped slowly into a car tyre to increase the pressure. The temperature of the air does not
change.

Which line in the table is correct?

number of molecules
average speed at which
hitting 1 cm2 of the
molecules hit the tyre
tyre each second

A increases increases
B increases unchanged
C unchanged increases
D unchanged unchanged

16 The thermometer in the diagram has no scale.

bulb

Where must the bulb be placed so that 0 °C can be marked on the stem?

A in boiling water
B in cold water
C in a freezer
D in melting ice

© UCLES 2008 0625/01/M/J/08 [Turn over


8

17 A sample of a solid is heated for 12 minutes and its temperature noted every minute.

The results are shown in the table.

time / min 0 1 2 3 4 5 6 7 8 9 10 11 12

temperature / °C 11.5 16.1 22.1 31.0 31.1 31.1 31.1 31.3 45.0 65.2 66.2 66.3 66.3

How should the sample be described at the end of the 12 minutes?

A all solid
B in the process of melting
C all liquid
D in the process of boiling

18 A heater is placed in a room.

Which diagram shows the movement of air as the room is heated?

A B

C D

© UCLES 2008 0625/01/M/J/08


9

19 The diagrams show four identical pieces of ice that are heated in test-tubes of water.

In which test-tube will the ice take the longest time to melt?

A B

ice
water water

ice wrapped
in lead wire

C D

ice
water
water
ice wrapped
in lead wire

20 The diagram represents water waves about to move into shallow water from deep water.

wavefront

deep water
shallow water

Which property of the waves remains the same after the waves move into shallow water?

A frequency
B speed
C wavefront direction
D wavelength

© UCLES 2008 0625/01/M/J/08 [Turn over


10

21 Two sets of water waves overlap as shown in the diagram.

U Q

S
T R

From which two points are the sets of waves coming?

A P and S B T and R C Q and T D U and Q

22 Which diagram shows total internal reflection of light?

A B C D
mirror glass prism

© UCLES 2008 0625/01/M/J/08


11

23 Light from the Sun passes through a prism and a spectrum is produced on a screen.

screen

narrow slit

light from P
red
the Sun
violet

A thermometer placed at P shows a large temperature rise.

Which type of radiation causes this?

A infra-red
B microwave
C ultra-violet
D visible light

24 Two thin converging lenses X and Y are used as shown to give a focused image of an illuminated
slit. The rays shown are parallel between X and Y.

X Y
screen
slit image

source
of light

30 cm 20 cm 15 cm

What are the correct values for the focal lengths of X and of Y?

focal length focal length


of X / cm of Y / cm

A 50 35
B 30 20
C 30 15
D 20 20

© UCLES 2008 0625/01/M/J/08 [Turn over


12

25 Which equation can be used to calculate the speed of sound?

A speed = distance
time

B speed = distance × time

C speed = time
distance

D speed = time + distance

26 A battery-operated bell is surrounded by a box with double walls.

air
bell box walls

gap

The bell is ringing but no sound at all is heard outside the box.

What is in the gap?

A a solid
B a liquid
C a gas
D a vacuum

27 Which line in the table shows whether iron and steel are ferrous or non-ferrous materials?

iron steel

A ferrous ferrous
B ferrous non-ferrous
C non-ferrous ferrous
D non-ferrous non-ferrous

© UCLES 2008 0625/01/M/J/08


13

28 A solenoid carrying a current is used to demagnetise a bar magnet.

solenoid
bar magnet

power
supply

Which conditions achieve demagnetisation?

current through
movement of bar magnet
solenoid

A a.c. around the solenoid quickly


B a.c. through the solenoid slowly
C d.c. around the solenoid quickly
D d.c. through the solenoid slowly

29 A polythene rod repels an inflated balloon hanging from a nylon thread.

What charges must the rod and the balloon carry?

A The rod and the balloon carry opposite charges.


B The rod and the balloon carry like charges.
C The rod is charged but the balloon is not.
D The balloon is charged but the rod is not.

30 Which circuit symbol represents a component used to store energy?

A B C D

© UCLES 2008 0625/01/M/J/08 [Turn over


14

31 In the circuit shown, the switches S1 and S2 may be open (off) or closed (on).

V
+ switches shown
12 V open (off)
– S2
S1

Which line in the table shows the voltmeter reading for the switch positions given?

S1 S2 voltmeter reading / V

A open open 12
B closed closed 12
C open closed 0
D closed open 12

32 The device X in this circuit is designed to cut off the electricity supply automatically if too much
current flows.

What is device X?

A a fuse
B a switch
C a resistor
D an ammeter

© UCLES 2008 0625/01/M/J/08


15

33 The diagrams show two ways in which three lamps may be connected.

X Y Z
Z

circuit 1 circuit 2

Which statement is correct?

A If lamp Y breaks in circuit 1, both the other lamps will go out.


B If lamp Y breaks in circuit 2, both the other lamps will go out.
C If lamp Y breaks in circuit 1, lamp Z will go out, but lamp X will remain on.
D If lamp Y breaks in circuit 2, lamp Z will go out, but lamp X will remain on.

34 The wire XY shown in the diagram is connected to a sensitive voltmeter with a centre zero. XY is
then moved quickly once through the magnetic field.

S
V
N
movement

What is observed on the voltmeter?

A The needle moves briefly in one direction and then returns to the centre.
B The needle moves quickly in one direction and stays deflected.
C The needle vibrates rapidly from side to side whilst XY is moving.
D The needle stays still.

© UCLES 2008 0625/01/M/J/08 [Turn over


16

35 The diagram shows a transformer connected to a 240 V a.c. supply.

soft-iron core
primary coil
secondary coil
60 turns
30 turns

240 V
a.c.

What is the potential difference across the secondary coil of the transformer?

A 30 V B 120 V C 240 V D 480 V

36 In order to produce a beam of cathode rays, a heated filament is placed near to an anode in an
evacuated glass vessel.

evacuated glass vessel

anode

heated filament

What is the type of charge on the anode and why is this charge chosen?

charge reason

A negative to attract electrons


B negative to repel electrons
C positive to attract electrons
D positive to repel electrons

© UCLES 2008 0625/01/M/J/08


17

37 Diagram 1 shows a potential divider circuit containing two 100 Ω resistors.

One of the resistors is changed to 90 Ω, as shown in diagram 2.

diagram 1 diagram 2

100 Ω 90 Ω

100 Ω V 100 Ω V

How does the reading on the voltmeter change when this is done?

A It becomes zero.
B It decreases a little.
C It increases a little.
D It stays the same.

38 What is a β-particle and from which part of a radioactive atom is it emitted?

β-particle emitted from

A electron nucleus
B electron outer orbits
C helium nucleus nucleus
D helium nucleus outer orbits

39 A sample of radioactive uranium has mass 1 g. Another sample of the same material has
mass 2 g.

Which property is the same for both samples?

A the amount of radiation emitted per second


B the half-life
C the number of uranium atoms
D the volume

© UCLES 2008 0625/01/M/J/08 [Turn over


18

37
40 A particular nuclide has the symbol 17 Cl .

What is true for atoms of this nuclide?

A There are 17 nucleons in the nucleus.


B There are 17 protons in the nucleus.
C There are 37 electrons in the nucleus.
D There are 37 neutrons in the nucleus.

© UCLES 2008 0625/01/M/J/08


19

BLANK PAGE

0625/01/M/J/08
20

BLANK PAGE

Permission to reproduce items where third-party owned material protected by copyright is included has been sought and cleared where possible. Every
reasonable effort has been made by the publisher (UCLES) to trace copyright holders, but if any items requiring clearance have unwittingly been included, the
publisher will be pleased to make amends at the earliest possible opportunity.

University of Cambridge International Examinations is part of the Cambridge Assessment Group. Cambridge Assessment is the brand name of University of
Cambridge Local Examinations Syndicate (UCLES), which is itself a department of the University of Cambridge.

0625/01/M/J/08
UNIVERSITY OF CAMBRIDGE INTERNATIONAL EXAMINATIONS
International General Certificate of Secondary Education

PHYSICS 0625/01
Paper 1 Multiple Choice October/November 2008
45 minutes
Additional Materials: Multiple Choice Answer Sheet
Soft clean eraser
*7002360314*

Soft pencil (type B or HB recommended)

READ THESE INSTRUCTIONS FIRST

Write in soft pencil.


Do not use staples, paper clips, highlighters, glue or correction fluid.
Write your name, Centre number and candidate number on the Answer Sheet in the spaces provided
unless this has been done for you.

There are forty questions on this paper. Answer all questions. For each question there are four possible
answers A, B, C and D.
Choose the one you consider correct and record your choice in soft pencil on the separate Answer Sheet.

Read the instructions on the Answer Sheet very carefully.

Each correct answer will score one mark. A mark will not be deducted for a wrong answer.
Any rough working should be done in this booklet.

This document consists of 20 printed pages.

IB08 11_0625_01/3RP
© UCLES 2008 [Turn over
2

1 Two digital stopwatches X and Y, which record in minutes and seconds, are used to time a race.

The readings of the two stopwatches, at the start and at the end of the race, are shown.

start end

stopwatch X 00:00 00:40

start end

stopwatch Y 01:30 02:20

Which statement about the time of the race is correct?

A Both stopwatches record the same time interval.


B Stopwatch X recorded 10 s longer than stopwatch Y.
C Stopwatch Y recorded 10 s longer than stopwatch X.
D Stopwatch Y recorded 50 s longer than stopwatch X.

2 A tennis player hits a ball over the net.

In which position is the ball accelerating?

A P and Q only
B P and R only
C Q and R only
D P, Q and R

© UCLES 2008 0625/01/O/N/08


3

3 A car travels at various speeds during a short journey.

The table shows the distances travelled and the time taken during each of four stages
P, Q, R and S.

stage P Q R S

distance travelled / km 1.8 3.6 2.7 2.7


time taken / minutes 2 2 4 3

During which two stages is the car travelling at the same speed?

A P and Q B P and S C Q and R D R and S

4 What is the meaning of the weight of an object?

A the density of the material from which it is made


B the force exerted on it by gravity
C the mass of the matter it contains
D the pressure it exerts on the floor

5 The weight of an object is to be found using the beam balance shown in the diagram.

object
?
The object is put in the left-hand pan and various standard weights are put in the right-hand pan,
with the following results.

weights in the right hand pan effect

0.1 N, 0.1 N, 0.05 N, 0.02 N balance tips down slightly on the left-hand side
0.2 N, 0.1 N, 0.01 N balance tips down slightly on the right-hand side

What is the best estimate of the weight of the object?

A 0.27 N B 0.29 N C 0.31 N D 0.58 N

© UCLES 2008 0625/01/O/N/08 [Turn over


4

6 A student needs to find the density of a cubic block of wood.

Which two pieces of apparatus should she use?

A balance and metre rule


B balance and thermometer
C measuring cylinder and metre rule
D measuring cylinder and thermometer

7 Two identical measuring cylinders containing different liquids are placed on a simple balance.

They balance as shown.

liquid X
volume = 200 cm3 liquid Y
volume = 100 cm3

How does the density of X compare with the density of Y?


1
A density of X = 2
× density of Y

B density of X = density of Y

C density of X = 2 × density of Y

D density of X = 4 × density of Y

© UCLES 2008 0625/01/O/N/08


5

8 The diagrams show a uniform rod with its midpoint on a pivot.

Two equal forces F are applied to the rod, as shown.

Which diagram shows the rod in equilibrium?

A B
F F F

F
pivot pivot

C F D

F F F
pivot pivot

9 A train is travelling along a horizontal track at constant speed. Two of the forces acting on the
train are shown in the diagram.

forwards
force from engine
friction 60 000 N
20 000 N

A force of air resistance is also acting on the train to give it a resultant force of zero.

What is this air resistance force?

A 40 000 N backwards
B 80 000 N backwards
C 40 000 N forwards
D 80 000 N forwards

© UCLES 2008 0625/01/O/N/08 [Turn over


6

10 The diagram shows water stored behind a dam.

dam

water
turbine
generator

water

The water flows to a turbine and turns a generator.

Which sequence for the conversion of energy is correct?

A gravitational energy → kinetic energy → electrical energy

B kinetic energy → gravitational energy → electrical energy

C gravitational energy → electrical energy → kinetic energy

D kinetic energy → electrical energy → gravitational energy

11 A rubber ball is dropped from a height of 2 m on to a table.

Whilst in contact with the table, some of its energy is converted into internal energy.

What is the highest possible point the ball could reach after bouncing?

4m A
height

B
ball released
from this height
C
2m
D

0
table

© UCLES 2008 0625/01/O/N/08


7

12 The diagram shows a simple mercury barometer used to measure atmospheric pressure.

mercury
Q

Which statement is correct?

A The pressure at P is atmospheric pressure.


B The pressure at P is nearly zero.
C The pressure at Q is lower than the pressure at P.
D The pressure at Q is nearly zero.

© UCLES 2008 0625/01/O/N/08 [Turn over


8

13 A brick with rectangular sides rests on a table.

brick
table

The brick is now turned so that it rests on the table on its smallest face.

How has this change affected the force and the pressure exerted by the brick on the table?

force pressure

A unchanged unchanged
B increased unchanged
C unchanged increased
D increased increased

14 Viewed through a microscope, very small particles can be seen moving with Brownian motion.

Which line in the table is correct?

type of motion particles are


of particles suspended in

A vibration a liquid or a gas


B vibration a solid, a liquid or a gas
C random a liquid or a gas
D random a solid, a liquid or a gas

© UCLES 2008 0625/01/O/N/08


9

15 The pressure of a fixed mass of gas in a cylinder is measured. The volume of the gas in the
cylinder is then slowly decreased.

Which graph could show the change of pressure of the gas during this process?

A B

pressure pressure

0 0
0 time 0 time

C D

pressure pressure

0 0
0 time 0 time

© UCLES 2008 0625/01/O/N/08 [Turn over


10

16 Equal masses of two different liquids are heated using the same heater. The graph shows how
the temperature of each liquid changes with time.

temperature
liquid 1

liquid 2

0
0 time

What does the graph tell us about the liquids?

A Liquid 1 has a higher melting point than liquid 2.


B Liquid 1 has a higher boiling point than liquid 2.
C Liquid 1 starts to melt sooner than liquid 2.
D Liquid 1 starts to boil sooner than liquid 2.

17 An engineer wants to fix a steel washer on to a steel rod. The rod is just too big to fit into the hole
of the washer.

steel steel rod


washer

How can the engineer fit the washer on to the rod?

A Cool the washer and put it over the rod.


B Cool the washer and rod to the same temperature and push them together.
C Heat the rod and then place it in the hole.
D Heat the washer and then place it over the rod.

© UCLES 2008 0625/01/O/N/08


11

18 A cup with a lid contains a hot drink.

lid

hot drink

When the lid is removed, the rate of heat loss from the drink increases.

What causes this?

A convection only
B evaporation only
C both convection and evaporation
D neither convection nor evaporation

19 A cupboard is placed in front of a heater. Air can move through a gap under the cupboard.

wall

cupboard
heater

air moves through gap

floor

Which line in the table describes the temperature and the direction of the air that moves through
the gap?

air temperature air direction

A cool away from the heater


B cool towards the heater
C warm away from the heater
D warm towards the heater

© UCLES 2008 0625/01/O/N/08 [Turn over


12

20 Waves in a tank pass from shallow to deep water.

The wavefront diagram is shown.

wavefront

shallow deep
water water

Which quantity increases as the waves enter the deep water?

A amplitude
B frequency
C wave energy
D wavelength

21 The diagrams represent water waves in a tank.

Which diagram represents waves that change speed?

A B C D
barrier barrier barrier
deeper shallower
water water

© UCLES 2008 0625/01/O/N/08


13

22 Which diagram shows the dispersion of white light?

A B
glass glass

white white
light light

C D
glass glass

white white
light light

23 In the diagram, the distance OP is the focal length of the lens.

Through which point will the ray shown pass, after refraction by the lens?

lens
A

O P D B

focal length focal length


C

24 Which word correctly completes the sentence below?

An echo is a sound wave which is ………… by a large obstacle.

A absorbed
B dispersed
C reflected
D refracted

© UCLES 2008 0625/01/O/N/08 [Turn over


14

25 In an experiment to measure the speed of sound, a student uses a stopwatch to find how long a
sound takes to travel from X to Y. She does this six times.

sound travels from X to Y

X Y

The table shows her results.

time / s

first 0.5
second 0.7
third 0.6
fourth 0.4
fifth 0.9
sixth 0.5

What value for the time should be used to calculate the speed of sound?

A 0.4 s B 0.5 s C 0.6 s D 0.9 s

26 A student wishes to demagnetise a steel bar. He uses the apparatus shown.

coil

electrical
supply

steel bar

Which type of electrical supply should the student use and what should he do with the steel bar?

supply what to do with the steel bar

A a.c. keep it inside the coil


B a.c. slowly remove it from the coil
C d.c. keep it inside the coil
D d.c. slowly remove it from the coil

© UCLES 2008 0625/01/O/N/08


15

27 Two metal bars are held together. At least one of the bars is a magnet. The bars repel each
other.

What does this show about the bars and why?

what it shows why

A only one of the bars is a magnet two magnets always attract each other

B only one of the bars is a magnet induced magnetism in the other bar makes it repel

C they are both magnets there must be like poles facing each other

D they are both magnets there must be opposite poles facing each other

28 When there is an electric current in a metal wire, what flows through the wire?

A atoms
B electrons
C neutrons
D protons

29 The table shows the voltage and current ratings for four electric heaters.

Which heater has the least resistance?

voltage / V current / A

A 110 5.0
B 110 10.0
C 230 5.0
D 230 10.0

© UCLES 2008 0625/01/O/N/08 [Turn over


16

30 In the circuit below, X and Y are identical 6 V lamps.

6V

switch
X

What happens when the switch is closed (the current is switched on)?

A X lights more brightly than Y.


B Y lights more brightly than X.
C X and Y both light with full brightness.
D X and Y both light with half brightness.

31 The diagram shows four different circuits.

P Q

3Ω 5Ω

R S

3Ω
3Ω 5Ω
5Ω

What is the order of increasing resistance of these circuits?

lowest highest
resistance resistance

A P Q R S
B R S P Q
C S P Q R
D P R Q S

© UCLES 2008 0625/01/O/N/08


17

32 A student makes four circuits.

In which circuit are both lamps protected by the fuse?

A B

C D

33 A mains circuit can safely supply a current of 40 A.

A hairdryer takes 2 A. It is connected to the circuit by a lead which can safely carry up to 5 A.

Which fuse is best to use in the plug fitted to the hairdryer lead?

A 1 A fuse B 3 A fuse C 10 A fuse D 50 A fuse

34 Two different systems are used to transmit equal amounts of electrical power from one building to
another.

One system uses low voltage and the other uses high voltage. Both systems use identical wires.

Which line in the table is correct about which system wastes least energy and why?

least energy wasted why

A high voltage system the current in the wires is bigger


B high voltage system the current in the wires is smaller
C low voltage system the current in the wires is bigger
D low voltage system the current in the wires is smaller

© UCLES 2008 0625/01/O/N/08 [Turn over


18

35 When the electric current in wire XY is in the direction shown, there is an upward force on the
wire.

current

force
magnet magnet

N S

If the north and south poles of the magnet exchange positions, in which direction will the force on
the wire act?

A downwards
B upwards
C to the left
D to the right

36 A cathode-ray beam passes through an electric field between charged parallel plates.

+
cathode-ray beam

In which direction is the beam deflected?

A towards the negative plate


B towards the positive plate
C into the page
D out of the page

© UCLES 2008 0625/01/O/N/08


19

37 When the thermistor in the circuit below is heated, the lamp becomes brighter.

Why does this happen?

A The resistance of the lamp decreases.


B The resistance of the lamp increases.
C The resistance of the thermistor decreases.
D The resistance of the thermistor increases.

38 The diagram shows an experiment to monitor the radiation from a radioactive gas. The counter
readings are corrected for background radiation.

counter
000.0

radioactive gas

The table shows how the counter reading varies with time.

time / seconds 0 20 40 60 80 100 120 140 160 180


counter reading /
140 105 82 61 44 36 27 20 15 10
counts per minute

What is the half-life of the gas?

A between 20 and 40 seconds


B between 40 and 60 seconds
C between 60 and 140 seconds
D between 140 and 180 seconds

© UCLES 2008 0625/01/O/N/08 [Turn over


20

39 Which material is commonly used as a lining for a box for storing radioactive samples?

A aluminium
B copper
C lead
D uranium

238
40 A uranium 92 U nucleus emits an α-particle.

What are the new nucleon and proton numbers?

nucleon number proton number

A 238 88
B 236 90
C 234 92
D 234 90

Permission to reproduce items where third-party owned material protected by copyright is included has been sought and cleared where possible. Every
reasonable effort has been made by the publisher (UCLES) to trace copyright holders, but if any items requiring clearance have unwittingly been included, the
publisher will be pleased to make amends at the earliest possible opportunity.

University of Cambridge International Examinations is part of the Cambridge Assessment Group. Cambridge Assessment is the brand name of University of
Cambridge Local Examinations Syndicate (UCLES), which is itself a department of the University of Cambridge.

© UCLES 2008 0625/01/O/N/08


First Variant Question Paper

UNIVERSITY OF CAMBRIDGE INTERNATIONAL EXAMINATIONS


International General Certificate of Secondary Education

PHYSICS 0625/11
Paper 1 Multiple Choice May/June 2009
45 minutes
Additional Materials: Multiple Choice Answer Sheet
Soft clean eraser
*7150187285*

Soft pencil (type B or HB recommended)

READ THESE INSTRUCTIONS FIRST

Write in soft pencil.


Do not use staples, paper clips, highlighters, glue or correction fluid.
Write your name, Centre number and candidate number on the Answer Sheet in the spaces provided
unless this has been done for you.

There are forty questions on this paper. Answer all questions. For each question there are four possible
answers A, B, C and D.
Choose the one you consider correct and record your choice in soft pencil on the separate Answer Sheet.

Read the instructions on the Answer Sheet very carefully.

Each correct answer will score one mark. A mark will not be deducted for a wrong answer.
Any rough working should be done in this booklet.

This document consists of 19 printed pages and 1 blank page.

IB09 06_0625_01/3RP
© UCLES 2009 [Turn over
2

1 A metre rule is used to measure a length.

Which reading is shown to the nearest millimetre?

A 0.7 m B 0.76 m C 0.761 m D 0.7614 m

2 The graph represents the movement of a body.

10
speed
m/s 8

0
0 1 2 3 4 5
time / s

How far has the body moved after 5 s?

A 2m B 10 m C 25 m D 50 m

3 A car travels 100 km. The highest speed of the car is 90 km/h, and the lowest speed is
30 km/h. The journey takes two hours.

What is the average speed for the journey?

A 30 km / h B 50 km / h C 60 km / h D 90 km / h

4 Which statement about the masses and weights of objects on the Earth is correct?

A A balance can only be used to compare weights, not masses.


B Heavy objects always have more mass than light ones.
C Large objects always have more mass than small ones.
D Mass is a force but weight is not.

© UCLES 2009 0625/11/M/J/09


3

5 The table shows the weight in newtons of a10 kg mass on each of four planets.

planet weight of a 10 kg mass / N

Earth 100
Jupiter 250
Mercury 40
Venus 90

The diagram shows a force meter (spring balance) being used.

18 N

2.0 kg

On which planet is the force meter (spring balance) being used?

A Earth
B Jupiter
C Mercury
D Venus

6 Which items of apparatus are required to determine the density of a liquid?

A balance and measuring cylinder


B balance and thermometer
C metre rule and measuring cylinder
D metre rule and thermometer

© UCLES 2009 0625/11/M/J/09 [Turn over


4

7 Which property of an object cannot be changed by a force?

A its mass
B its motion
C its shape
D its size

8 A wooden plank rests in equilibrium on two boulders on opposite sides of a narrow stream. Three
forces of size P, Q and R act on the plank.

P R
plank

How are the sizes of the forces related?

A P+Q=R
B P+R=Q
C P=Q=R
D P=Q+R

9 Electricity can be obtained from different energy resources.

Which energy resource is used to obtain electricity without producing heat to boil water?

A coal
B geothermal
C hydroelectric
D nuclear

© UCLES 2009 0625/11/M/J/09


5

10 Four people of equal weight on a beach use different routes to get to the top of a sea wall.

pavement

slipway

sea wall

beach

Which person produces the greatest average power?

person route time taken

A runs across the beach, then climbs the ladder 8s


B walks across the beach, then climbs the ladder 16 s
C runs up the slipway 5s
D walks up the slipway 10 s

11 The diagrams show two mercury barometers standing side by side. The right-hand diagram
shows a tube of bigger diameter, but the diagram is incomplete. There is a vacuum above the
mercury in both tubes.

Which labelled position on the right-hand tube could show the mercury level in that tube?

vacuum A
B
C

glass
tubes

D
dish
mercury dish

© UCLES 2009 0625/11/M/J/09 [Turn over


6

12 A student fills two containers with water (density 1.0 g / cm3) and two with oil (density 0.8 g / cm3),
as shown in the diagrams.

In which container is the pressure on the base the greatest?

A B C D

water oil

water oil

13 In an experiment, some of a substance changes from a liquid to a gas. The temperature of the
remaining liquid changes because of this.

What is the name for this change of state and how does the temperature change?

change of state how temperature changes

A condensation decreases
B condensation increases
C evaporation decreases
D evaporation increases

© UCLES 2009 0625/11/M/J/09


7

14 A piston traps a certain mass of gas inside a cylinder. Initially the piston is halfway along the
length of the cylinder.

The piston is now moved towards the open end of the cylinder. The temperature of the gas
remains constant.

initial position

final position

How are the density and the pressure of the gas affected by moving the piston?

density pressure

A decreases decreases
B decreases unchanged
C increases decreases
D increases unchanged

15 The graph shows the change in temperature of a substance as it is heated steadily.

Which part of the graph shows when the substance is boiling?

D
temperature
C

time

© UCLES 2009 0625/11/M/J/09 [Turn over


8

16 Four blocks, made of different materials, are each given the same quantity of internal energy
(heat).

Which block has the greatest thermal capacity?

A B C D

temperature temperature temperature temperature


rise = 2 oC rise = 4 oC rise = 6 oC rise = 8 oC

17 A rod is made up of copper and wood joined together.

After the rod is heated at the join in the centre for about a minute, where would the lowest
temperature be?

B C

A D

wood copper

18 The diagram shows a fire.

Why does the smoke rise above the fire?

A Smoke evaporates more quickly at higher temperatures.


B Smoke molecules diffuse more quickly at higher temperatures.
C The density of the air is lower at higher temperatures.
D The pressure of the air is greater at higher temperatures.

© UCLES 2009 0625/11/M/J/09


9

19 The diagrams show water waves that move more slowly after passing into shallow water at the
broken line.

Which diagram shows what happens to the waves?

A B
fast slow fast slow

deep shallow deep shallow


water water water water

C D
fast slow fast slow

deep shallow deep shallow


water water water water

© UCLES 2009 0625/11/M/J/09 [Turn over


10

20 The diagrams show examples of wave motion.

1 2
ripple tank

drum

waves on water waves in air

3 4

waves on a rope waves in a spring


(as shown)

Which are longitudinal waves?

A 1 only
B 1, 2 and 4
C 2 and 3 only
D 2 and 4 only

© UCLES 2009 0625/11/M/J/09


11

21 Which diagram shows what happens when a ray of white light passes through a prism?

A B

spectrum
white white
light light
spectrum

C D
spectrum

white white
light light spectrum

22 The diagram shows the path of a ray of light passing through a principal focus F of a lens.

Which broken line shows the direction of the ray after it leaves the lens?

lens

ray
of
lig
ht
A

F F’

D C

© UCLES 2009 0625/11/M/J/09 [Turn over


12

23 A small boat in a harbour is protected from waves on the sea by harbour walls.

land sea
waves
small
boat
harbour

harbour
walls

Some waves can curve round the harbour walls and reach the boat.

What is the name for this effect?

A diffraction
B dispersion
C reflection
D refraction

24 An engineer standing at P sees an explosion at X.

P Z

Y
DANGER - X
BLASTING

V W

After the explosion, she hears two bangs. One bang is heard a fraction of a second after the
other. The second bang is an echo.

From which surface has the sound reflected to cause this echo?

A XY B PV C ZY D WX

© UCLES 2009 0625/11/M/J/09


13

25 The north pole of a bar magnet is placed next to end P of an iron bar PQ, as shown. As a result,
magnetic poles are induced in the iron bar.

S N P Q

magnet iron bar

What are the magnetic poles induced at P and at Q?

magnetic pole at P magnetic pole at Q

A north north
B north south
C south north
D south south

26 An electromagnet is used to separate magnetic metals from non-magnetic metals.

Why is steel unsuitable as the core of the electromagnet?

A It forms a permanent magnet.


B It has a high density.
C It has a high thermal capacity.
D It is a good conductor of electricity.

27 Which particle does not experience a force due to an electric field?

A α-particle
B electron
C neutron
D proton

© UCLES 2009 0625/11/M/J/09 [Turn over


14

28 Using the circuit shown, the current I is found for various voltages V. The temperature of the
resistor does not change.

variable direct
voltage
V

Which graph shows the results obtained?

A B C D

I I I I

0 0 0 0
0 V 0 V 0 V 0 V

29 In the circuit shown, ammeter X reads 0.5 A.

4.0 V

X A A Y

3.0 Ω 5.0 Ω

What does ammeter Y read?

A 0 B 0.5 A C 3.5 A D 4.0 A

© UCLES 2009 0625/11/M/J/09


15

30 In the circuits shown, all the resistors are identical.

Which circuit has the least resistance?

A B

C D

31 In the circuit below, one of the lamps breaks, causing all the other lamps to go out.

Which lamp breaks?

A C D

© UCLES 2009 0625/11/M/J/09 [Turn over


16

32 Either a fuse or a circuit-breaker can be used to protect electrical cables from large currents that
could cause overheating.

X cable
live

electrical
Y appliance
supply

neutral
cable

If a fuse is used, in which position in the circuit should it be connected, and if a circuit-breaker is
used, in which position should it be connected?

position position of
of fuse circuit-breaker

A X X
B X Y
C Y X
D Y Y

33 The current in a lamp at full brightness is 0.25 A. The flexible cable to the lamp is designed for
currents up to 5.0 A, so it can safely carry the 0.25 A taken by the lamp.

Which fuse should be inserted in the plug at the other end of the flexible cable?

A 0.2 A B 1.0 A C 5.0 A D 10.0 A

© UCLES 2009 0625/11/M/J/09


17

34 A wire perpendicular to the page carries an electric current in a direction out of the page. There
are four compasses near the wire.

Which compass shows the direction of the magnetic field caused by the current?

A C

wire with
current out
of page

35 A transformer has 50 turns on its primary coil and 100 turns on its secondary coil. An alternating
voltage of 25.0 V is connected across the primary coil.

25.0 V

primary coil secondary coil


50 turns 100 turns

What is the voltage across the secondary coil?

A 12.5 V B 50.0 V C 175 V D 200 V

36 In a cathode-ray tube, a hot tungsten cathode releases particles by thermionic emission.

What are these particles?

A α-particles
B electrons
C protons
D tungsten atoms

© UCLES 2009 0625/11/M/J/09 [Turn over


18

37 The diagram shows a cathode-ray tube.

cathode anode top Y plate


P

screen

bottom Y plate

A student wants the cathode rays to make a spot at P on the screen.

Which parts of the cathode-ray tube should be positive?

A anode and top Y plate


B anode and bottom Y plate
C cathode and top Y plate
D cathode and bottom Y plate

38 A radioactive nucleus contains 138 neutrons. The nucleus emits an α-particle.

How many neutrons are in the nucleus after it has emitted the α-particle?

A 134 B 136 C 138 D 139

© UCLES 2009 0625/11/M/J/09


19

39 The graph shows the decay curve for one particular radioactive nuclide.

2500
count rate
counts / min

2000

1500

1000

500

0
0 1 2 3 4 5
time / days

What is the half-life of this nuclide?

A 1.0 day B 1.5 days C 2.0 days D 2.5 days

16
40 7N is the symbol for a particular nuclide of nitrogen.

How many nucleons does this nuclide contain?

A 7 B 9 C 16 D 23

© UCLES 2009 0625/11/M/J/09


20

BLANK PAGE

Permission to reproduce items where third-party owned material protected by copyright is included has been sought and cleared where possible. Every
reasonable effort has been made by the publisher (UCLES) to trace copyright holders, but if any items requiring clearance have unwittingly been included, the
publisher will be pleased to make amends at the earliest possible opportunity.

University of Cambridge International Examinations is part of the Cambridge Assessment Group. Cambridge Assessment is the brand name of University of
Cambridge Local Examinations Syndicate (UCLES), which is itself a department of the University of Cambridge.

0625/11/M/J/09
UNIVERSITY OF CAMBRIDGE INTERNATIONAL EXAMINATIONS
International General Certificate of Secondary Education

PHYSICS 0625/11
Paper 1 Multiple Choice October/November 2009
45 minutes
Additional Materials: Multiple Choice Answer Sheet
Soft clean eraser
*7565115686*

Soft pencil (type B or HB recommended)

READ THESE INSTRUCTIONS FIRST

Write in soft pencil.


Do not use staples, paper clips, highlighters, glue or correction fluid.
Write your name, Centre number and candidate number on the Answer Sheet in the spaces provided
unless this has been done for you.

There are forty questions on this paper. Answer all questions. For each question there are four possible
answers A, B, C and D.
Choose the one you consider correct and record your choice in soft pencil on the separate Answer Sheet.

Read the instructions on the Answer Sheet very carefully.

Each correct answer will score one mark. A mark will not be deducted for a wrong answer.
Any rough working should be done in this booklet.

This document consists of 16 printed pages.

IB09 11_0625_01/4RP
© UCLES 2009 [Turn over
2

1 A stopwatch is used to time an athlete running 100 m. The timekeeper forgets to reset the watch
to zero before using it to time another athlete running 100 m.

60 60
55 5 55 5

50 10 50 10

45 15 45 15

40 seconds 20 40 seconds 20

35 25 35 25
30 30

stopwatch at stopwatch at
end of first end of second
athlete’s run athlete’s run

How long does the second athlete take to run 100 m?

A 11.2 s B 11.4 s C 12.4 s D 23.8 s

2 A ball is thrown upwards.

What effect does the force of gravity have on the ball?

A It produces a constant acceleration downwards.


B It produces a constant acceleration upwards.
C It produces a decreasing acceleration upwards.
D It produces an increasing acceleration downwards.

3 A car accelerates from traffic lights. The graph shows the car’s speed plotted against time.

speed
m/s
20

0
0 10 time / s

How far does the car travel before it reaches a constant speed?

A 10 m B 20 m C 100 m D 200 m

© UCLES 2009 0625/11/O/N/09


3

4 Which property of a body can be measured in newtons?

A density
B mass
C volume
D weight

5 A cup contains hot liquid.

Some of the liquid evaporates as it cools.

What happens to the mass and to the weight of the liquid in the cup as it cools?

mass weight

A decreases decreases
B decreases stays the same
C stays the same decreases
D stays the same stays the same

6 The diagrams show a rectangular box with inside measurements of 5 cm × 6 cm × 4 cm.

liquid

4 cm
6 cm

5 cm
mass = 40 g total mass = 220 g

The box has a mass of 40 g when empty. When filled with a liquid, it has a total mass of 220 g.

What is the density of the liquid?

A 220 g / cm3
(5 × 6 × 4)

(220 − 40)
B g / cm3
(5 × 6 × 4)

C (5 × 6 × 4) g / cm3
220

(5 × 6 × 4)
D g / cm3
(220 − 40)

© UCLES 2009 0625/11/O/N/09 [Turn over


4

7 Passengers are not allowed to stand on the upper deck of double-decker buses.

upper deck

lower deck

Why is this?

A They would cause the bus to become unstable.


B They would cause the bus to slow down.
C They would increase the kinetic energy of the bus.
D They would lower the centre of mass of the bus.

8 The object in the diagram is acted upon by the two forces shown.

3N 2N
object

What is the effect of these forces?

A The object moves to the left with constant speed.


B The object moves to the left with constant acceleration.
C The object moves to the right with constant speed.
D The object moves to the right with constant acceleration.

© UCLES 2009 0625/11/O/N/09


5

9 The diagram shows a microphone being used in an interview.

microphone

Which energy change takes place in the microphone?

input energy output energy

A chemical electrical
B electrical chemical
C electrical sound
D sound electrical

10 The table shows the times taken for four children to run up a set of stairs.

Which child’s power is greatest?

mass of child / kg time / s

A 40 10
B 40 20
C 60 10
D 60 20

© UCLES 2009 0625/11/O/N/09 [Turn over


6

11 A water manometer is used to measure the pressure of a gas supply to a house. It gives a
reading of h cm of water.

gas
supply

h cm

Why is it better to use water rather than mercury in this manometer?

A h would be too large if mercury were used.


B h would be too small if mercury were used.
C The tube would need to be narrower if mercury were used.
D The tube would need to be wider if mercury were used.

12 The diagrams show two swimming pools. One contains fresh water and the other contains salt
water. Salt water is more dense than fresh water.

At which labelled point is the pressure the greatest?

water surface water surface

A C

B D
fresh water (less dense) salt water (more dense)

13 Brownian motion is observed by looking at smoke particles through a microscope.

How do the smoke particles move in Brownian motion?

A all in the same direction


B at random
C in circles
D vibrating about fixed points

© UCLES 2009 0625/11/O/N/09


7

14 A small amount of air is introduced into the vacuum above the mercury in a mercury barometer
tube.

The mercury level goes down.

Why does the mercury level go down?

A The air molecules cool the mercury and make it contract.


B The air molecules decrease the pressure above the mercury.
C The air molecules heat the mercury and make it expand.
D The air molecules increase the pressure above the mercury.

15 To mark a temperature scale on a thermometer, standard temperatures known as fixed points are
needed.

Which of these is a fixed point?

A room temperature
B the temperature inside a freezer
C the temperature of pure melting ice
D the temperature of pure warm water

16 The table gives the melting points and boiling points of glycerine and benzene.

melting point boiling point

glycerine 18 °C 290 °C
benzene 5.4 °C 80 °C

At which temperature will both glycerine and benzene be liquid?

A 0 °C B 50 °C C 100 °C D 150 °C

© UCLES 2009 0625/11/O/N/09 [Turn over


8

17 A beaker contains water at room temperature.

X water

How could a convection current be set up in the water?

A cool the water at X


B cool the water at Y
C stir the water at X
D stir the water at Y

18 A student warms her hands near a fire.

Which waves carry most heat to her hands and are these waves electromagnetic?

waves electromagnetic

A infra-red no
B infra-red yes
C visible light no
D visible light yes

19 The drawing shows a wave.

Which labelled distance is the wavelength?

A
B
D

© UCLES 2009 0625/11/O/N/09


9

20 Sound waves travel from a point X to another point Y.

X Y

Which diagram represents the movement of the air molecules, caused by the sound waves, in the
region between X and Y.

A B C D

X Y

21 A ray of light in glass is incident on a boundary with air.

Which path does the ray of light take when the angle of incidence i is less than the critical angle?

air
glass C

i D

ray of
light

22 An object O is placed in front of a converging lens of focal length f.

At which point will the top of the image be seen?

A
B

f f
C
D

© UCLES 2009 0625/11/O/N/09 [Turn over


10

23 Which range of frequencies typically can be heard by a 10 year-old child?

A 20 Hz – 2000 Hz
B 20 Hz – 20 000 Hz
C 200 Hz – 2000 Hz
D 200 Hz – 20 000 Hz

24 Astronaut 1 uses a hammer to mend a satellite in space. Astronaut 2 is nearby. There is no air in
space.

hammer
astronaut 1 astronaut 2

Compared with the sound heard if they were working on Earth, what does astronaut 2 hear?

A a louder sound
B a quieter sound
C a sound of the same loudness
D no sound at all

25 A permanent magnet is brought near to a piece of copper. The copper is not attracted by the
magnet.

Why is there no attraction?

A Copper is ferrous but is only attracted by an electromagnet.


B Copper is ferrous but is not attracted by any type of magnet.
C Copper is not ferrous and is only attracted by an electromagnet.
D Copper is not ferrous and is not attracted by any type of magnet.

© UCLES 2009 0625/11/O/N/09


11

26 Four plotting compasses are placed in the magnetic field of two identical bar magnets as shown
in the diagram.

Which compass is shown pointing in the wrong direction?

A D

S N N S

27 A pupil measures the potential difference across a device and the current in it.

Which calculation gives the resistance of the device?

A current + potential difference

B current ÷ potential difference

C potential difference ÷ current

D potential difference × current

28 A student uses a length of wire as a resistor. He discovers that the resistance of the wire is too
small.

To be certain of making a resistor of higher value, he should use a piece of wire that is

A longer and thicker.


B longer and thinner.
C shorter and thicker.
D shorter and thinner.

© UCLES 2009 0625/11/O/N/09 [Turn over


12

29 The diagram shows a battery connected to two identical resistors. Three ammeters M1, M2 and
M3 are connected in the circuit.

M1 A A M3

A
M2

Meter M1 reads 1.0 A.

What are the readings on M2 and on M3?

reading on M2 / A reading on M3 / A

A 0.5 0.0
B 0.5 0.5
C 0.5 1.0
D 1.0 1.0

30 The diagram shows a torch containing two cells, a switch and a lamp.

plastic
case

brass
connecting switch
strip lamp

What is the circuit diagram for the torch?

A B C D

© UCLES 2009 0625/11/O/N/09


13

31 An electrical component X is placed in water, as shown.

thermometer

water

When the temperature of the water is increased, the reading on the ammeter increases.

What is component X?

A a capacitor
B a light-dependent resistor
C a relay
D a thermistor

32 A certain electrical appliance is powered from a mains supply. The appliance normally uses a
current of 3 A, but the current briefly rises to 4 A at the instant the appliance is switched on. The
cable to the appliance is designed for currents up to 6 A.

The fuses available to protect the cable are rated at 1 A, 3 A, 5 A and 13 A.

Which fuse should be used?

A 1A B 3A C 5A D 13 A

33 On a building site, metal scaffolding is firmly embedded in the damp ground. A builder holds a
mains-operated electric drill in one hand. With his other hand he holds on to the scaffolding.

The power cable of the drill is damaged where it enters the metal casing of the drill.

What danger does this present to the builder?

A A current could pass through the builder and electrocute him.


B A current in the scaffolding could heat it up and burn him.
C The large current could blow the fuse and damage the drill.
D The large current could make the motor spin too quickly.

© UCLES 2009 0625/11/O/N/09 [Turn over


14

34 Which device is designed to allow a small direct current (d.c.) to control a large direct current
(d.c.)?

A a generator
B a motor
C a relay
D a transformer

35 A transformer is to be used to provide a 10 V output from a 100 V supply.

100 V 10 V

primary coil secondary coil

What are suitable numbers of turns for the primary coil and for the secondary coil?

number of turns on number of turns on


the primary coil the secondary coil

A 100 1000
B 200 110
C 400 490
D 800 80

36 The diagram shows a beam of cathode rays entering an electric field.

beam

In which direction is the beam deflected by the field?

A downwards
B upwards
C into the page
D out of the page

© UCLES 2009 0625/11/O/N/09


15

37 To create cathode rays in a vacuum, what might be a suitable potential difference to connect
between the cathode and anode?

A 6 V a.c.
B 6 V d.c.
C 600 V a.c.
D 600 V d.c.

38 Which statement explains the meaning of the half-life of a radioactive substance?

A half the time taken for half the substance to decay


B half the time taken for the substance to decay completely
C the time taken for half the substance to decay
D the time taken for the substance to decay completely

39 The diagram shows the paths of three different types of radiation, X, Y and Z.

2 mm of 10 mm lead
plastic of aluminium

Which row in the table correctly identifies X, Y and Z?

X Y Z

A α-particles β-particles γ-rays


B β-particles α-particles γ-rays
C β-particles γ-rays α-particles
D γ-rays α-particles β-particles

Question 40 is on the next page.

© UCLES 2009 0625/11/O/N/09 [Turn over


16

238
40 How many neutrons and how many protons are contained in a nucleus of 92 U ?

neutrons protons

A 92 146
B 146 92
C 146 238
D 238 92

Permission to reproduce items where third-party owned material protected by copyright is included has been sought and cleared where possible. Every
reasonable effort has been made by the publisher (UCLES) to trace copyright holders, but if any items requiring clearance have unwittingly been included, the
publisher will be pleased to make amends at the earliest possible opportunity.

University of Cambridge International Examinations is part of the Cambridge Assessment Group. Cambridge Assessment is the brand name of University of
Cambridge Local Examinations Syndicate (UCLES), which is itself a department of the University of Cambridge.

© UCLES 2009 0625/11/O/N/09


UNIVERSITY OF CAMBRIDGE INTERNATIONAL EXAMINATIONS
International General Certificate of Secondary Education

PHYSICS 0625/11
Paper 1 Multiple Choice May/June 2010
45 minutes
Additional Materials: Multiple Choice Answer Sheet
Soft clean eraser
*0334676086*

Soft pencil (type B or HB recommended)

READ THESE INSTRUCTIONS FIRST

Write in soft pencil.


Do not use staples, paper clips, highlighters, glue or correction fluid.
Write your name, Centre number and candidate number on the Answer Sheet in the spaces provided
unless this has been done for you.

There are forty questions on this paper. Answer all questions. For each question there are four possible
answers A, B, C and D.
Choose the one you consider correct and record your choice in soft pencil on the separate Answer Sheet.

Read the instructions on the Answer Sheet very carefully.

Each correct answer will score one mark. A mark will not be deducted for a wrong answer.
Any rough working should be done in this booklet.

This document consists of 17 printed pages and 3 blank pages.

IB10 06_0625_11/4RP
© UCLES 2010 [Turn over
2

1 A scientist needs to determine the volume of a small, irregularly shaped rock sample. Only a rule
and a measuring cylinder, partially filled with water, are available.

cm3

cm

rock sample rule

measuring
cylinder

To determine the volume, which apparatus should the scientist use?

A both the measuring cylinder and the rule


B neither the measuring cylinder nor the rule
C the measuring cylinder only
D the rule only

2 A student uses a stopwatch to time a runner running around a circular track. The runner runs two
laps (twice around the track). The diagrams show the reading on the stopwatch when the runner
starts running, at the end of the first lap, and at the end of the second lap.

reading when reading at end reading at end


runner starts of first lap of second lap

What is the time taken for the runner to run the second lap?

A 0 min 50 s B 1 min 10 s C 1 min 13 s D 2 min 03 s

© UCLES 2010 0625/11/M/J/10


3

3 Two distance / time graphs and two speed / time graphs are shown.

Which graph represents an object that is at rest?

A B

distance distance

0 0
0 time 0 time

C D

speed speed

0 0
0 time 0 time

4 A snail moves along a ruler. It takes 20 s to move from Q to R.

P Q R

0 2 cm 12

What is its average speed from Q to R?

A 12 cm / s
20

B 12 − 2 cm / s
20

C 20 cm / s
12

D 20 cm / s
12 − 2

© UCLES 2010 0625/11/M/J/10 [Turn over


4

5 Two blocks of metal X and Y hang from spring balances as shown in the diagram.

N0 N0
1 1
2 2
3 3
4 4
5 5

X
Y

What does the diagram show about X and Y?

A They have the same mass and the same volume but different weights.
B They have the same mass and the same weight but different volumes.
C They have the same mass, the same volume and the same weight.
D They have the same weight and the same volume but different masses.

© UCLES 2010 0625/11/M/J/10


5

6 The graph shows how weight varies with mass on planet P and on planet Q.

400
planet P
weight / N
300
planet Q

200

100

0
0 20 40 60 80
mass / kg

An object weighs 400 N on planet P. The object is taken to planet Q.

Which row is correct?

mass of object weight of object


on planet Q / kg on planet Q / N

A 40 200
B 40 400
C 80 200
D 80 400

7 The diagram shows a rectangular block of density 2 g / cm3.

3 cm
2 cm

2 cm

What is the mass of the block?

A 2g B 6g C 14 g D 24 g

© UCLES 2010 0625/11/M/J/10 [Turn over


6

8 Which statement about a moving object is correct?

A When an object is accelerating, the resultant force acting on it must equal zero.
B When an object is moving at a steady speed, the air resistance acting on it must equal zero.
C When an object is moving at a steady speed, the resultant force acting on it must equal zero.
D When an object is moving, there must be a resultant force acting on it.

9 An experiment is carried out to measure the extension of a rubber band for different loads.

The results are shown below.

load / N 0 1 2 3
length / cm 15.2 16.2 18.6
extension / cm 0 1.0 2.1 3.4

Which figure is missing from the table?

A 17.2 B 17.3 C 17.4 D 17.6

10 Energy is stored in a battery and in a box of matches.

Which type of energy is stored in each of them?

a battery a box of matches

A chemical chemical
B chemical internal (thermal)
C electrical chemical
D electrical internal (thermal)

11 A man lifts 20 bricks, each of weight 6 N.

What other information is needed to calculate the useful work done in lifting the bricks?

A the distance he lifts the bricks


B the mass of the bricks
C the time taken to lift the bricks
D the volume of the bricks

© UCLES 2010 0625/11/M/J/10


7

12 To prevent a cement mixer sinking into soft ground, the mixer is placed on a large flat board.

cement mixer

board
soft ground

Why does this prevent the mixer sinking?

A The large area decreases the pressure on the ground.


B The large area increases the pressure on the ground.
C The large area decreases the weight on the ground.
D The large area increases the weight on the ground.

13 The pressure of a gas is measured using a manometer as shown in the diagram.

pressure to
be measured

h
mercury

The mercury in the manometer is replaced with a liquid which is less dense.

How does the value of h change?

A It becomes zero.
B It decreases, but not to zero.
C It stays the same.
D It increases.

© UCLES 2010 0625/11/M/J/10 [Turn over


8

14 The diagram represents the molecules of a gas in a closed container of constant volume.

What happens to the molecules when the gas is heated?

A They expand.
B They hit the walls less often.
C They move further apart.
D They move more quickly.

15 A liquid is left in an open dish. After several days there is less liquid in the dish.

Which statement explains this?

A The least energetic molecules leave the surface and escape into the air.
B The least energetic molecules leave the surface and return.
C The most energetic molecules leave the surface and escape into the air.
D The most energetic molecules leave the surface and return.

16 A thermometer has a scale which starts at –10 °C and ends at 110 °C.

–10 0 100 110

°C

What is the value of the lower fixed point and of the upper fixed point of the scale?

lower fixed point upper fixed point


/ °C / °C

A –10 100
B –10 110
C 0 100
D 0 110

© UCLES 2010 0625/11/M/J/10


9

17 A student carries out an experiment to find the melting point of wax. The graph shows how the
temperature of the wax changes as it cools.

X
temperature
/ °C
Y
melting point
of wax Z

0
0
time

Which statement is correct?

A At X the temperature drops more slowly than at Z.


B At Y all the wax is solid.
C At Y thermal energy is being given out by the wax.
D At Z the wax molecules are not moving.

18 Hot liquid in a vacuum flask cools extremely slowly. This is because some methods of heat
transfer cannot take place in a vacuum.

Which methods cannot take place in a vacuum?

A conduction and convection only


B conduction and radiation only
C convection and radiation only
D conduction, convection and radiation

© UCLES 2010 0625/11/M/J/10 [Turn over


10

19 A teacher demonstrates convection currents using a box with two chimneys and a lighted candle.
She holds a smoking taper at point P.

In which direction does the convection current cause the smoke to move?

P
A C

chimney chimney D

20 Which waves are longitudinal?

A B C D

88:88

light waves microwaves sound waves water waves


from a lamp in an oven from a trumpet on a pond

21 A navigation buoy floating on the sea oscillates up and down as a wave passes.

navigation
buoy

In exactly two minutes, six complete wavelengths pass the buoy.

What is the frequency of the waves?

A 0.050 Hz B 0.33 Hz C 3.0 Hz D 20 Hz

© UCLES 2010 0625/11/M/J/10


11

22 The diagram shows a ray of light incident on the edge of a piece of glass. The angle i is bigger
than the critical angle.

Which arrow correctly shows the direction of the ray after it leaves the edge of the glass?

normal
ray of
D
light
i
glass
air
C

A
B

23 Which diagram correctly shows rays of light passing through a converging lens in a camera?

A B
camera camera

object object
image image
lens lens

C D
camera camera

object object
image image
lens lens

24 What is the approximate value of the highest frequency that can be heard by a young person?

A 20 Hz B 200 Hz C 2000 Hz D 20 000 Hz

© UCLES 2010 0625/11/M/J/10 [Turn over


12

25 A police car siren emits two different sounds P and Q. These are produced alternately. The
diagram represents the sounds emitted.

P P P

displacement
Q Q

time

Which sound is the louder and which has the lower pitch?

louder lower pitch

A P P
B P Q
C Q P
D Q Q

26 A magnet attracts two pieces of iron.

iron

N S

What is the arrangement of the induced poles in the pieces of iron?

A N S S N

B N S N S

C S N S N

D S N N S

© UCLES 2010 0625/11/M/J/10


13

27 A piece of iron and a piece of steel are picked up by an electromagnet as shown.

electromagnet

N S

steel iron

The current to the electromagnet is switched off.

What happens?

A Both the iron and the steel remain magnetised.


B Neither the iron nor the steel remain magnetised.
C Only the iron remains magnetised.
D Only the steel remains magnetised.

28 The table shows the lengths and diameters of four copper wires.

Which wire has the least resistance?

length / m diameter / mm

A 0.50 1.0
B 0.50 2.5
C 0.75 1.0
D 0.75 2.5

29 A circuit is set up to measure the resistance of a resistor R. The meter readings are 2.0 A and
3.0 V.

A
V

What is the resistance of the resistor R?

A 0.67 Ω B 1.5 Ω C 5.0 Ω D 6.0 Ω

© UCLES 2010 0625/11/M/J/10 [Turn over


14

30 The circuit shown is a potential divider.

X
output

What is component X?

A a light-dependent resistor
B a relay
C a thermistor
D a variable resistor

31 A lamp is connected in four circuits in turn, each using identical batteries.

The resistors are all identical.

In which circuit will the lamp be brightest?

A B

C D

© UCLES 2010 0625/11/M/J/10


15

32 Which diagram shows the correct positions for both the switch and the fuse?

live wire switch

A lamp
neutral wire
fuse

fuse
live wire switch

B lamp
neutral wire

fuse
live wire

C lamp
neutral wire

switch

live wire

D lamp
neutral wire

fuse switch

33 After some building work in a house, a bare (uninsulated) live wire is left protruding from a wall.

What is the greatest hazard?

A a fire
B a fuse will blow
C an electric shock
D no current will flow

© UCLES 2010 0625/11/M/J/10 [Turn over


16

34 A coil is connected to a battery and a soft iron bar is hung near to it.

soft iron
coil

The current is then reversed by reversing the battery connections.

How does the soft iron bar behave in the two cases?

with the battery as shown with the battery reversed

A attracted to the coil attracted to the coil


B attracted to the coil repelled from the coil
C repelled from the coil attracted to the coil
D repelled from the coil repelled from the coil

35 A transformer has 15 000 turns on its primary coil and 750 turns on its secondary coil.

Connected in this way, for what purpose could this transformer be used?

A to convert the 8000 V a.c. output of a power station to 160 000 V for long-distance power
transmission
B to convert the 160 000 V d.c. supply from a power line to 8000 V for local power transmission
C to use a 12 V d.c. supply to operate a 240 V razor
D to use a 240 V a.c. mains supply to operate a 12 V motor

36 What are cathode rays?

A a beam of electrons
B a beam of neutrons
C a beam of protons
D electromagnetic waves

© UCLES 2010 0625/11/M/J/10


17

37 A cathode-ray tube has an anode and an earthed cathode.

Which row shows the charge on the anode and the temperature of the cathode?

cathode
anode charge
temperature

A negative cool
B negative hot
C positive cool
D positive hot

38 Which row describes the properties of α-particles?

ionizing radiation stopped


effect by aluminium?

A large no
B large yes
C small no
D small yes

39 A radioactive substance has a half-life of 2 weeks. At the beginning of an investigation the


substance emits 3000 β-particles per minute.

How many β-particles will it emit per minute after 6 weeks?

A 0 B 375 C 500 D 1500

40 The nuclide notation for radium-226 is 226


88 Ra .

How many electrons orbit the nucleus of a neutral atom of radium-226?

A 0 B 88 C 138 D 226

© UCLES 2010 0625/11/M/J/10


18

BLANK PAGE

© UCLES 2010 0625/11/M/J/10


19

BLANK PAGE

© UCLES 2010 0625/11/M/J/10


20

BLANK PAGE

Permission to reproduce items where third-party owned material protected by copyright is included has been sought and cleared where possible. Every
reasonable effort has been made by the publisher (UCLES) to trace copyright holders, but if any items requiring clearance have unwittingly been included, the
publisher will be pleased to make amends at the earliest possible opportunity.

University of Cambridge International Examinations is part of the Cambridge Assessment Group. Cambridge Assessment is the brand name of University of
Cambridge Local Examinations Syndicate (UCLES), which is itself a department of the University of Cambridge.

© UCLES 2010 0625/11/M/J/10


w
w
w
.X
tr
me
eP
ap
UNIVERSITY OF CAMBRIDGE INTERNATIONAL EXAMINATIONS

er
s
International General Certificate of Secondary Education

.c
om
PHYSICS 0625/11
Paper 1 Multiple Choice October/November 2010
45 minutes
Additional Materials: Multiple Choice Answer Sheet
Soft clean eraser
*0588341704*

Soft pencil (type B or HB recommended)

READ THESE INSTRUCTIONS FIRST

Write in soft pencil.


Do not use staples, paper clips, highlighters, glue or correction fluid.
Write your name, Centre number and candidate number on the Answer Sheet in the spaces provided
unless this has been done for you.

There are forty questions on this paper. Answer all questions. For each question there are four possible
answers A, B, C and D.
Choose the one you consider correct and record your choice in soft pencil on the separate Answer Sheet.

Read the instructions on the Answer Sheet very carefully.

Each correct answer will score one mark. A mark will not be deducted for a wrong answer.
Any rough working should be done in this booklet.

This document consists of 18 printed pages and 2 blank pages.

IB10 11_0625_11/3RP
© UCLES 2010 [Turn over
2

1 The diagrams show an experiment to determine the volume of a stone.

without stone with stone

10 cm3 10 cm3
9 9
8 8
7 7
6 6
5 5
4 4
3 3
2 2 stone
1 1

What is the volume of the stone?

A 3 cm3 B 4 cm3 C 7 cm3 D 11 cm3

2 Four athletes run twice around a track. The table shows their times at the end of each lap.

Which athlete runs the second lap the fastest?

athlete time at end of first lap / s time at end of second lap / s

A 22.99 47.04
B 23.04 47.00
C 23.16 47.18
D 23.39 47.24

© UCLES 2010 0625/11/O/N/10


3

3 The diagram shows the speed / time graph for an object moving at constant speed.

2
speed
m/s
1

0
0 1 2 3 4
time / s

What is the distance travelled by the object in the first 3 s?

A 1.5 m B 2.0 m C 3.0 m D 6.0 m

4 A car travels along the route PQRST in 30 minutes.

5 km

Q 10 km

T
5 km
10 km

R
P

What is the average speed of the car?

A 10 km / hour B 20 km / hour C 30 km / hour D 60 km / hour

5 Which list contains the name of a force?

A acceleration, charge, temperature


B density, resistance, speed
C distance, frequency, mass
D energy, power, weight

© UCLES 2010 0625/11/O/N/10 [Turn over


4

6 The reading on a spring balance with a holder and eight identical discs is 3.0 N.

Six discs are removed and the reading becomes 1.2 N.

N0 spring N0
balance
1 1
2 2
3 3
4 4

holder
for discs
two
discs
eight
discs

What is the weight of one disc?

A 0.2 N B 0.3 N C 0.5 N D 0.6 N

7 A student is trying to find the density of water and of a large, regularly shaped concrete block.

Which apparatus is needed to find the density of both the water and the concrete block?

A balance, clock, measuring cylinder


B balance, clock, ruler
C balance, measuring cylinder, ruler
D clock, measuring cylinder, ruler

8 A force acts on a moving rubber ball.

Which of these changes could not happen to the ball because of the force?

A a change in direction
B a change in mass
C a change in shape
D a change in speed

© UCLES 2010 0625/11/O/N/10


5

9 The extension / load graph for a spring is shown. The unloaded length of the spring is 15.0 cm.

3
extension / cm

0
0 1 2 3 4 5
load / N

When an object of unknown weight is hung on the spring, the length of the spring is 16.4 cm.

What is the weight of the object?

A 0.55 N B 0.67 N C 3.5 N D 4.1 N

10 Which of these is designed to change electrical energy into kinetic energy?

A a capacitor
B a generator
C a motor
D a transformer

11 A car is driven on a long journey along a flat, horizontal road. The car stops several times on the
journey and its engine becomes hot.

Which type of energy does not change during the journey?

A the chemical energy in the fuel tank


B the gravitational energy of the car
C the internal (thermal) energy of the engine
D the kinetic energy of the car

© UCLES 2010 0625/11/O/N/10 [Turn over


6

12 What is a simple mercury barometer designed to measure?

A the pressure beneath a liquid


B the pressure of a gas supply
C the pressure of car tyres
D the pressure of the atmosphere

13 Liquid X has a density of 1010 kg / m3. Liquid Y has a density of 950 kg / m3.

The liquids are poured into tubes as shown.

Which tube has the greatest pressure on its base?

A B C D

liquid X liquid Y liquid X liquid Y

14 Some gas in a sealed plastic bag is cooled.

How do the gas molecules behave when this happens?

A They move more quickly and become closer together.


B They move more quickly and become further apart.
C They move more slowly and become closer together.
D They move more slowly and become further apart.

© UCLES 2010 0625/11/O/N/10


7

15 A block of ice cream is prevented from melting by wrapping it in newspaper soaked in water. The
water evaporates from the newspaper.

Which molecules escape from the water and what happens to the average speed of the water
molecules that remain in the newspaper?

average speed of the


escaping molecules
remaining water molecules

A the more energetic ones decreases


B the more energetic ones increases
C the less energetic ones decreases
D the less energetic ones increases

16 Which change is condensation?

A B
solid liquid gas
C D

© UCLES 2010 0625/11/O/N/10 [Turn over


8

17 A beaker containing ice and a thermometer is left in a warm room for 15 minutes.

No water is visible in the beaker until 5 minutes has passed. After 15 minutes some ice is still
visible.

ice ice

water

start of after after


experiment 5 minutes 15 minutes

Which graph shows how the thermometer reading changes?

A B

temperature temperature
/ °C / °C

0 0

0 5 10 15 0 5 10 15
time / min time / min

C D

temperature temperature
/ °C / °C

0 0

0 5 10 15 0 5 10 15
time / min time / min

© UCLES 2010 0625/11/O/N/10


9

18 A piece of wood has some iron nails pushed through it. One side of the wood is covered with heat
sensitive paper which turns from pink to blue when heated. The wood is heated as shown for a
few minutes and blue dots appear on the heat sensitive paper where it touches the nails.

wood heat sensitive paper

iron blue
heat
nails dots

This experiment shows that, compared to wood, iron is a good

A absorber of heat.
B conductor of heat.
C convector of heat.
D emitter of heat.

19 An electric heater is placed inside a metal box which has one side open. The diagram shows four
possible positions for the box.

The heater is switched on for several minutes.

In which position does the box become the hottest?

A B C D

heater heater
box box

© UCLES 2010 0625/11/O/N/10 [Turn over


10

20 Which group contains only transverse waves?

A infra-red waves, light waves, sound waves


B infra-red waves, light waves, ultra-violet waves
C infra-red waves, ultra-violet waves, sound waves
D light waves, sound waves, ultra-violet waves

21 Water waves in a tank pass over a thin plastic block as shown.

plastic block

tank

What happens to the waves as they reach the plastic block?

A They are diffracted because they slow down.


B They are diffracted because they speed up.
C They are refracted because they slow down.
D They are refracted because they speed up.

22 A girl writes the word LEFT on a piece of card.

LEFT
She looks at the image of this card, made by reflection by a plane mirror.

What does she see?

A B C D

© UCLES 2010 0625/11/O/N/10


11

23 A scientist is trying to direct a ray of light through a glass block without any light leaving the top of
the block. However, some light does leave the top.

light leaving top


top of block

glass block
X
ray of
light

The scientist changes angle X and stops the ray of light leaving the top.

Which row in the table describes the change to angle X and the name of the effect produced?

change to angle X name of effect produced

A decrease total internal reflection


B decrease total internal refraction
C increase total internal reflection
D increase total internal refraction

24 The diagrams represent two different sound waves.

wave P wave Q

displacement displacement

time time

How do the frequency and pitch of P compare with the frequency and pitch of Q?

frequency of P pitch of P

A greater than Q higher than Q


B greater than Q same as Q
C same as Q higher than Q
D same as Q same as Q

© UCLES 2010 0625/11/O/N/10 [Turn over


12

25 A ship sends a pulse of sound vertically downwards to the sea bed. An echo is heard
0.4 seconds later.

If the speed of sound in the water is 1200 m / s, how deep is the water below the ship?

A 240 m B 480 m C 1500 m D 3000 m

26 Which statement about a magnet is not correct?

A It can attract another magnet.


B It can attract an unmagnetised piece of iron.
C It can repel another magnet.
D It can repel an unmagnetised piece of iron.

27 A chain of steel nails and a chain of iron nails hang from a strong magnet. The chains are then
carefully removed from the magnet.

magnet

steel iron

What happens to the chains?

A Both chains fall apart.


B Both chains stay together.
C Only the chain of iron nails falls apart.
D Only the chain of steel nails falls apart.

© UCLES 2010 0625/11/O/N/10


13

28 A potential difference (p.d.) across a resistor causes a current in it.

resistor
current

p.d.

The p.d. and the resistance of the resistor can both be changed.

Which row shows two changes that will both increase the current in the resistor?

change change

A decrease p.d. decrease resistance


B decrease p.d. increase resistance
C increase p.d. decrease resistance
D increase p.d. increase resistance

29 The diagram shows a circuit containing two ammeters and three resistors.

ammeter ammeter
A A
X Y

R1 R2 R3

Which of the ammeters will show the current in resistor R2?

A ammeter X only
B ammeter Y only
C both ammeter X and ammeter Y
D neither ammeter X nor ammeter Y

30 Which component can store energy and can be used in time-delay circuits?

A a capacitor
B a potentiometer
C a resistor
D a thermistor

© UCLES 2010 0625/11/O/N/10 [Turn over


14

31 A circuit contains two resistors connected in parallel with a battery.

R
P
2Ω

Q
4Ω

Which of the following statements about the currents at P, Q and R is true?

A The current at P is the greatest.


B The current at Q is the greatest.
C The current at R is the greatest.
D The current is the same at points P, Q and R.

32 The potential divider shown is connected across a constant 12 V supply.

V1 20 Ω

12 V

V2 R

When R has a value of 20 Ω, the voltmeter readings are equal.

How do these readings change when the value of R is reduced to 10 Ω?

reading on V1 reading on V2

A decreases decreases
B decreases increases
C increases decreases
D increases increases

© UCLES 2010 0625/11/O/N/10


15

33 A fuse is a safety device for use in an electrical appliance.

How does a fuse affect a circuit when the current in it becomes higher than the correct value for
the appliance?

A It completely stops the current.


B It reduces the current to the correct value for the appliance.
C It sends the current to the outer case of the appliance.
D It sends the excess current to the earth wire.

34 Some electrical equipment is connected to a 230 V supply. It is kept inside a metal case which is
not earthed. The case is fixed to a plastic support. A strand of wire has become loose and
touches the metal case as shown.

cable

metal case on plastic support

strand of wire

electrical
equipment

Which statement about this situation is correct?

A An electric current is passing through the metal case.


B A fuse in the live wire will blow.
C Someone touching the case would receive an electric shock.
D The metal case is at 0 V.

© UCLES 2010 0625/11/O/N/10 [Turn over


16

35 A coil is rotated steadily between the poles of a magnet. The coil is connected to an oscilloscope.

N S oscilloscope

contact
contact

Which graph shows the output voltage V against time t ?

A B C D

V V V V
t t t t

36 A village has to be supplied with electricity from a power station that is a long way from the
village.

Which type of current should be used, and at which voltage?

type of current voltage

A alternating current high voltage


B alternating current low voltage
C direct current high voltage
D direct current low voltage

© UCLES 2010 0625/11/O/N/10


17

37 An electric field is set up between two parallel plates.

Cathode rays are directed into this field, parallel to the plates.

cathode
rays

In which direction are the cathode rays deflected by the electric field?

A downwards
B upwards
C into the page
D out of the page

38 A radioactive element has a half-life of 70 s.

The number of emissions per second, N, of a sample of the element is measured at a certain
time.

What was the number of emissions per second 70 s earlier?

A 0 B N/2 C N D 2N

39 S is a radioactive source emitting α-particles, β-particles and γ-rays. A detector is placed 5 cm


away from S. A thin sheet of paper is placed as shown in the diagram.

thin sheet of paper

S detector

5 cm

Which radiations can be detected?

A α-particles and β-particles only

B α-particles and γ-rays only

C β-particles and γ-rays only

D α-particles, β-particles and γ-rays

© UCLES 2010 0625/11/O/N/10 [Turn over


18

40 In the atomic model, an atom consists of a central mass, orbited by much smaller particles.

central
mass

orbiting
particles

What is the name of the central mass and of the orbiting particles?

central mass orbiting particles

A neutron α-particles
B neutron electrons
C nucleus α-particles
D nucleus electrons

© UCLES 2010 0625/11/O/N/10


19

BLANK PAGE

© UCLES 2010 0625/11/O/N/10


20

BLANK PAGE

Permission to reproduce items where third-party owned material protected by copyright is included has been sought and cleared where possible. Every
reasonable effort has been made by the publisher (UCLES) to trace copyright holders, but if any items requiring clearance have unwittingly been included, the
publisher will be pleased to make amends at the earliest possible opportunity.

University of Cambridge International Examinations is part of the Cambridge Assessment Group. Cambridge Assessment is the brand name of University of
Cambridge Local Examinations Syndicate (UCLES), which is itself a department of the University of Cambridge.

© UCLES 2010 0625/11/O/N/10


w
w
w
.X
tr
me
eP
ap
UNIVERSITY OF CAMBRIDGE INTERNATIONAL EXAMINATIONS

er
s
International General Certificate of Secondary Education

.c
om
PHYSICS 0625/11
Paper 1 Multiple Choice May/June 2011
45 minutes
Additional Materials: Multiple Choice Answer Sheet
Soft clean eraser
*3129497903*

Soft pencil (type B or HB recommended)

READ THESE INSTRUCTIONS FIRST

Write in soft pencil.


Do not use staples, paper clips, highlighters, glue or correction fluid.
Write your name, Centre number and candidate number on the Answer Sheet in the spaces provided
unless this has been done for you.

There are forty questions on this paper. Answer all questions. For each question there are four possible
answers A, B, C and D.
Choose the one you consider correct and record your choice in soft pencil on the separate Answer Sheet.

Read the instructions on the Answer Sheet very carefully.

Each correct answer will score one mark. A mark will not be deducted for a wrong answer.
Any rough working should be done in this booklet.

This document consists of 19 printed pages and 1 blank page.

IB11 06_0625_11/4RP
© UCLES 2011 [Turn over
2

1 The diagram shows an enlarged drawing of the end of a metre rule. It is being used to measure
the length of a small feather.

10 20 30
mm
cm 1 2 3

What is the length of the feather?

A 19 mm B 29 mm C 19 cm D 29 cm

2 An object moves initially with constant speed and then with constant acceleration.

Which graph shows this motion?

A B

speed speed

0 0
0 time 0 time

C D

speed speed

0 0
0 time 0 time

© UCLES 2011 0625/11/M/J/11


3

3 A tennis player hits a ball hard and 0.40 s later hears the echo from a wall.

The speed of sound in air is 330 m / s.

How far away is the player from the wall?

A 66 m B 132 m C 264 m D 825 m

4 Which statement about mass and weight is correct?

A Mass and weight are both forces.


B Neither mass nor weight is a force.
C Only mass is a force.
D Only weight is a force.

© UCLES 2011 0625/11/M/J/11 [Turn over


4

5 The diagram shows a balance being used to find the weight of a baby. The weight of the basket
can be ignored.

At equilibrium, the pivot is nearer to the weight W than to the baby.

centre of mass
of beam

pivot
W

baby

basket

What is the weight of the baby?

A less than W
B more than W
C W
D impossible to tell

6 A cube of side 2.0 cm is placed on a balance.

2.0 cm
balance

7.2 g

What is the density of the cube?

A 0.90 g / cm3 B 1.2 g / cm3 C 1.8 g / cm3 D 3.6 g / cm3

© UCLES 2011 0625/11/M/J/11


5

7 Objects with different masses are hung on a spring. The diagram shows how much the spring
stretches.

10 cm

20 cm

30 cm

100 g

The extension of the spring is directly proportional to the mass hung on it.

What is the mass of object M?

A 110 g B 150 g C 200 g D 300 g

8 Which row gives an example of the stated form of energy?

form of energy example

A gravitational the energy due to the movement of a train along a level track
B internal the energy due to the flow of cathode rays in a cathode-ray tube
C kinetic the energy due to the position of a swimmer standing on a
high diving board
D strain the energy due to the compression of springs in a car seat

9 Which energy resource is used to generate electricity by first boiling water?

A hydroelectric
B nuclear fission
C tides
D waves

© UCLES 2011 0625/11/M/J/11 [Turn over


6

10 Two farmers use an electrically powered elevator to lift bales of hay. All the bales of hay have the
same mass.

bale of hay

As sunset approaches, they increase the speed of the motor so that more bales are lifted up in a
given time.

How does this affect the work done in lifting each bale and the useful output power of the motor?

work done in useful output power


lifting each bale of the motor

A increases decreases
B increases increases
C no change decreases
D no change increases

© UCLES 2011 0625/11/M/J/11


7

11 A brick with flat, rectangular sides rests on a table.

brick
table

The brick is now turned so that it rests on the table on its smallest face.

How has this affected the force and the pressure exerted by the brick on the table?

force pressure

A increased increased
B increased unchanged
C unchanged increased
D unchanged unchanged

© UCLES 2011 0625/11/M/J/11 [Turn over


8

12 The diagram shows two mercury barometers.

Barometer 1 is measuring atmospheric pressure on day 1.

Barometer 2 is measuring atmospheric pressure on day 2.

mercury

point X point Y

barometer 1, day 1 barometer 2, day 2

Which statement is true?

A The atmospheric pressure on day 1 is less than the atmospheric pressure on day 2.
B The atmospheric pressure on day 1 is the same as the atmospheric pressure on day 2.
C The pressure at point X is less than the pressure at point Y.
D The pressure at point X is the same as the pressure at point Y.

13 A sealed gas cylinder is left outside on a hot, sunny day.

What happens to the average speed of the molecules and to the pressure of the gas in the
cylinder as the temperature rises?

average speed of
gas pressure
the gas molecules

A falls falls
B falls rises
C rises falls
D rises rises

© UCLES 2011 0625/11/M/J/11


9

14 When a liquid evaporates, some molecules escape from it and its temperature changes.

From where do the molecules escape and what is the effect on the temperature of the liquid?

molecules escape from temperature of liquid

A all parts of the liquid decreases


B all parts of the liquid increases
C only the liquid surface decreases
D only the liquid surface increases

15 The diagram shows a thermometer calibrated in degrees Celsius.

°C
110
100
90
80
70
60
50
40
30
20
10
0
–10

What are the values of the lower fixed point and of the upper fixed point on the Celsius scale?

lower fixed upper fixed


point / °C point / °C

A –10 110
B 0 20
C 0 100
D 20 100

16 An ice cube at a temperature of 0 °C is put into a drink at a temperature of 10 °C.

After a short time, some of the ice has melted and the drink has cooled to a temperature of 8 °C.

What is the temperature of the remaining ice?

A 0 °C B 2 °C C 4 °C D 8 °C

© UCLES 2011 0625/11/M/J/11 [Turn over


10

17 An experiment is set up to find out which metal is the best conductor of heat.

Balls are stuck with wax to rods made from different metals, as shown in diagram 1.

The rods are heated at one end. Some of the balls fall off, leaving some as shown in diagram 2.

Which labelled metal is the best conductor of heat?

diagram 1 diagram 2
A B C D

heated end heated end


before heating after heating

18 Food is kept in a cool-box which uses two ice packs to keep it cool.

Where should the ice packs be placed to keep all the food as cool as possible?

A both at the bottom of the box


B both at the top of the box
C one at the front and one at the back of the box
D one on the left and one on the right of the box

19 Water waves can be used to show reflection, refraction and diffraction.

For each of these, which row shows whether or not the speed of the water waves changes?

reflection refraction diffraction

A no no yes
B no yes no
C yes no no
D yes yes yes

© UCLES 2011 0625/11/M/J/11


11

20 A vertical stick is dipped up and down in water at P.

In two seconds, three wave crests are produced on the surface of the water.

wave P
crests

Which statement is correct?

A Distance X is the amplitude of the waves.


B Distance Y is the wavelength of the waves.
C Each circle represents a wavefront.
D The frequency of the waves is 3 Hz.

21 The diagram shows the dispersion of white light by a glass prism.

white light red light

glass prism violet light

Why does dispersion occur when white light enters the glass?

A The frequency of red light decreases more than that of violet light.
B The frequency of violet light decreases more than that of red light.
C The speed of red light decreases more than that of violet light.
D The speed of violet light decreases more than that of red light.

© UCLES 2011 0625/11/M/J/11 [Turn over


12

22 A thin converging lens is used to produce, on a screen, a focused image of a candle.

screen
image

lens

candle

Various focused images are produced on the screen by moving the lens and the screen
backwards and forwards.

Which statement is always correct?

A The image is at the principal focus (focal point) of the lens.


B The image is bigger than the object.
C The image is closer to the lens than the object is.
D The image is inverted.

23 Sound travels by wave motion.

Which property of waves causes echoes?

A diffraction
B dispersion
C reflection
D refraction

24 A student listens to a machine that makes sounds of different frequencies. He can only hear one
of the sounds.

Which frequency of sound is the student able to hear?

A 2 Hz B 10 Hz C 2 kHz D 30 kHz

© UCLES 2011 0625/11/M/J/11


13

25 Which test could be used to find which end of a magnet is the north pole?

A putting it near a compass needle


B putting it near a ferrous metal
C putting it near a non-ferrous metal
D putting it near a steel spoon

26 In two separate experiments, a magnet is brought near to an unmagnetised iron bar. This causes
the bar to become magnetised.

experiment 1 N magnet S X iron bar

experiment 2 S magnet N iron bar Y

Which magnetic poles are induced at X and at Y?

pole induced at X pole induced at Y

A N N
B N S
C S N
D S S

27 An ammeter and an 18 Ω resistor are connected in series with a battery. The reading on the
ammeter is 0.50 A. The resistance of the battery and the ammeter can be ignored.

18 Ω

What is the electromotive force (e.m.f.) of the battery?

A 9.0 N B 9.0 V C 36 N D 36 V

© UCLES 2011 0625/11/M/J/11 [Turn over


14

28 A polythene rod repels an inflated balloon hanging from a nylon thread.

What charges must the rod and the balloon carry?

A The rod and the balloon carry opposite charges.


B The rod and the balloon carry like charges.
C The rod is charged but the balloon is not.
D The balloon is charged but the rod is not.

29 Which circuit includes a capacitor and what does the capacitor do in this circuit?

what the
circuit
capacitor does

A potential divider stores current


B potential divider stores energy
C time delay stores current
D time delay stores energy

30 A student sets up the circuit shown. The switch is open (off).

Which lamps are on and which lamps are off?

lamp X lamp Y lamp Z

A off off off


B on off off
C on off on
D on on on

© UCLES 2011 0625/11/M/J/11


15

31 The diagram shows a thermistor in a potential divider. A voltmeter is connected across the
thermistor.

The graph shows how the resistance of the thermistor changes with temperature.

resistance

temperature

As the thermistor becomes warmer, what happens to its resistance and what happens to the
reading on the voltmeter?

resistance voltmeter reading

A decreases decreases
B decreases increases
C increases decreases
D increases increases

© UCLES 2011 0625/11/M/J/11 [Turn over


16

32 In this circuit, a component at X automatically protects the wiring from overheating if there is a
fault.

electrical supply

Which components are suitable to use at X?

A a circuit-breaker, a fuse or a switch


B only a circuit-breaker or a fuse
C only a circuit-breaker or a switch
D only a fuse

33 Which graph shows how the output voltage varies with time for a simple a.c. generator?

A B

voltage voltage

0 0
time time

C D

voltage voltage

0 0
time time

© UCLES 2011 0625/11/M/J/11


17

34 A magnet is suspended from a spring so that it can move freely inside a coil. The coil is
connected to a sensitive centre-zero ammeter.

spring

moving magnet
N

centre-zero ammeter

stationary coil S

What does the ammeter show when the magnet repeatedly moves slowly up and down?

A a reading constantly changing from left to right and right to left


B a steady reading to the left
C a steady reading to the right
D a steady reading of zero

35 The diagram shows a simple step-down transformer used to decrease a voltage.

Which part is the primary coil?

B C D

© UCLES 2011 0625/11/M/J/11 [Turn over


18

36 The diagram shows a cathode-ray tube.

power
supply
Y vacuum

screen
X

What are the correct labels for X and for Y?

X Y

A negative anode positive cathode


B negative cathode positive anode
C positive anode negative cathode
D positive cathode negative anode

37 A beam of cathode rays passes between two parallel metal plates connected to a high-voltage
d.c. power supply.

Which path does the beam follow?

C
+ + + + + +
D
cathode B
rays

38 Which row shows the relative ionising effects and penetrating abilities of α-particles and
β-particles?

ionising effect penetrating ability

A α greater than β α greater than β


B α greater than β α less than β
C α less than β α greater than β
D α less than β α less than β

© UCLES 2011 0625/11/M/J/11


19

39 A powder contains 400 mg of a radioactive material that emits α-particles.

The half-life of the material is 5 days.

What mass of that material remains after 10 days?

A 0 mg B 40 mg C 100 mg D 200 mg

40 An atom of the element lithium has a nucleon number of 7 and a proton number of 3.

Which diagram represents a neutral atom of lithium?

A B

– –

n + +
+ + +
+ n n key
n = a neutron
– –
+ = a proton

– = an electron

C D (not to scale)

– –
– –
n n
+n+ +n+
n n n n
+ +

– –
– –

© UCLES 2011 0625/11/M/J/11


20

BLANK PAGE

Permission to reproduce items where third-party owned material protected by copyright is included has been sought and cleared where possible. Every
reasonable effort has been made by the publisher (UCLES) to trace copyright holders, but if any items requiring clearance have unwittingly been included, the
publisher will be pleased to make amends at the earliest possible opportunity.

University of Cambridge International Examinations is part of the Cambridge Assessment Group. Cambridge Assessment is the brand name of University of
Cambridge Local Examinations Syndicate (UCLES), which is itself a department of the University of Cambridge.

© UCLES 2011 0625/11/M/J/11


w
w
w
.X
tr
me
eP
ap
UNIVERSITY OF CAMBRIDGE INTERNATIONAL EXAMINATIONS

er
s
International General Certificate of Secondary Education

.c
om
PHYSICS 0625/11
Paper 1 Multiple Choice October/November 2011
45 minutes
Additional Materials: Multiple Choice Answer Sheet
Soft clean eraser
*1240126518*

Soft pencil (type B or HB recommended)

READ THESE INSTRUCTIONS FIRST

Write in soft pencil.


Do not use staples, paper clips, highlighters, glue or correction fluid.
Write your name, Centre number and candidate number on the Answer Sheet in the spaces provided
unless this has been done for you.

There are forty questions on this paper. Answer all questions. For each question there are four possible
answers A, B, C and D.
Choose the one you consider correct and record your choice in soft pencil on the separate Answer Sheet.

Read the instructions on the Answer Sheet very carefully.

Each correct answer will score one mark. A mark will not be deducted for a wrong answer.
Any rough working should be done in this booklet.

This document consists of 20 printed pages.

IB11 11_0625_11/3RP
© UCLES 2011 [Turn over
2

1 A ruler is used to measure the length of an object.

object

1 2 3 4 5 6 7 8 9 10
cm

What is the length of the object?

A 3.0 cm B 4.0 cm C 5.0 cm D 6.5 cm

2 The graph shows how the speed of a car changes with time.

speed

0
U V W X Y time

Between which two times is the car stationary?

A U and V B V and W C W and X D X and Y

3 A child is standing on the platform of a station.

A train travelling at 30 m / s takes 3.0 s to pass the child.

What is the length of the train?

A 10 m B 27 m C 30 m D 90 m

© UCLES 2011 0625/11/O/N/11


3

4 The weight of an object is to be found using the balance shown in the diagram.

object

The object is put in the left-hand pan and various standard weights are put in the right-hand pan.
These are the results.

weights in the right-hand pan effect

0.1 N, 0.1 N, 0.05 N, 0.02 N balance tips down slightly on the left-hand side

0.2 N, 0.1 N, 0.01 N balance tips down slightly on the right-hand side

What is the best estimate of the weight of the object?

A 0.27 N B 0.29 N C 0.31 N D 0.58 N

© UCLES 2011 0625/11/O/N/11 [Turn over


4

5 The diagrams show four blocks with the same mass.

Which block is made from the least dense material?

A B

8 cm 3 cm

1 cm 3 cm
2 cm 2 cm

C D

5 cm
2 cm
4 cm

3 cm
2 cm
2 cm

6 A student is told to measure the density of a liquid and also of a large cube of metal.

Which pieces of equipment are sufficient to be able to take the measurements needed?

A balance, measuring cylinder and ruler


B balance and thermometer
C measuring cylinder and ruler
D measuring cylinder, ruler and thermometer

© UCLES 2011 0625/11/O/N/11


5

7 A uniform rod rests on a pivot at its centre. The rod is not attached to the pivot. Forces are then
applied to the rod in four different ways, as shown. The weight of the rod can be ignored.

Which diagram shows the rod in equilibrium?

A B

100 N 100 N 100 N

100 N

C D

100 N

100 N 100 N 100 N 100 N

8 Which combination of forces produces a resultant force acting towards the right?

A B C D
5N 4N 6N 5N
10 N 5N 10 N 7N

2N 3N 4N 3N

9 A student adds weights to an elastic cord. He measures the length of the cord for each weight.

He then plots a graph from the results, as shown.

0
0 1 2 3 4 weight / N

Which length has he plotted on the vertical axis?

A measured length
B original length
C (measured length – original length)
D (measured length + original length)

© UCLES 2011 0625/11/O/N/11 [Turn over


6

10 A force F moves a load from the bottom of a slope to the top.

F p

d
loa

The work done by the force depends on the size of the force, and on a distance.

What is this distance?

A p B q C r D p+q

11 A water manometer is used to measure the pressure of a gas supply.

gas
supply

initial level 10 cm
of water 10 cm

water

When it is attached to the gas supply, the water falls on the left side and rises on the right side.
The difference in the levels of water on the two sides is now 20 cm.

What is the pressure of the gas supply?

A the pressure due to 10 cm depth of water


B the pressure due to 20 cm depth of water
C the pressure due to 10 cm depth of water plus atmospheric pressure
D the pressure due to 20 cm depth of water plus atmospheric pressure

© UCLES 2011 0625/11/O/N/11


7

12 A chair is placed on protective cups to prevent damage to the carpet underneath it.

chair

carpet

protective cups

How do the cups change the area of contact with the carpet and the pressure on it?

area of contact pressure

A decreased decreased
B decreased increased
C increased decreased
D increased increased

13 Evaporation occurs when molecules escape from a liquid surface into the air above it. During this
process the temperature of the liquid falls.

Why does the temperature of the liquid fall?

A The molecules in the vapour expand because the pressure is less.


B The molecules left in the liquid have more space to move around.
C The molecules move more slowly when they escape into the air.
D The molecules with the highest energies escape into the air.

© UCLES 2011 0625/11/O/N/11 [Turn over


8

14 The pressure of a fixed mass of gas in a cylinder is measured. The volume of the gas in the
cylinder is then slowly decreased. The temperature of the gas does not change.

Which graph could show the change of pressure of the gas during this process?

A B

pressure pressure

0 0
0 time 0 time

C D

pressure pressure

0 0
0 time 0 time

15 To mark the lower fixed point of a Celsius scale on a thermometer, the thermometer should be
placed in

A pure alcohol.
B pure distilled water.
C pure melting ice.
D pure mercury.

© UCLES 2011 0625/11/O/N/11


9

16 The diagram shows an electric heater being used to heat a beaker of water and an identical
beaker of oil for several minutes.

water oil

electric heater

The temperature of the water and the temperature of the oil increase constantly. The rise in
temperature of the oil is much greater than that of the water.

Why is this?

A The oil has a higher boiling point than water.


B The oil has a higher thermal capacity than water.
C The oil has a lower boiling point than water.
D The oil has a lower thermal capacity than water.

17 The diagram shows a cooling unit in a refrigerator.

cooling
unit

Why is the cooling unit placed at the top?

A Cold air falls and warm air is displaced upwards.


B Cold air is a bad conductor so heat is not conducted into the refrigerator.
C Cold air is a good conductor so heat is conducted out of the refrigerator.
D Cold air remains at the top and so prevents convection.

© UCLES 2011 0625/11/O/N/11 [Turn over


10

18 Two identical copper cans are filled with boiling water.

thermometer

hot water

copper can

wool

uninsulated can insulated can

One can is insulated with wool. The temperature of the water in each can is taken every minute
for several minutes. Graphs of the results are plotted.

Which graph shows the results obtained?

A B

temperature temperature
insulated

insulated uninsulated
uninsulated

time time

C D
insulated
temperature temperature
uninsulated
uninsulated
insulated

time time

19 Which of these waves is longitudinal?

A infra-red
B radio
C sound
D water

© UCLES 2011 0625/11/O/N/11


11

20 Radio waves are received at a house at the bottom of a hill.

radio
waves

house
hill

The waves reach the house because the hill has caused them to be

A diffracted.
B radiated.
C reflected.
D refracted.

21 The diagram shows a ray of light travelling from X. Angle P is less than the critical angle.

In which direction does the ray continue?

P
glass A
air
B

D
C

22 The diagram shows the image of a clock in a plane mirror.

What time is shown?

A 02:25 B 02:35 C 09:25 D 09:35

© UCLES 2011 0625/11/O/N/11 [Turn over


12

23 A student wishes to measure the speed of sound in air. She plans to measure the time between
making a sound and hearing the echo from a cliff.

cliff

student

She will use the equation: speed = distance .


time
Which type of sound should she make and which distance should she use in her calculation?

type of sound distance to use


distance to cliff
A continuous sound 2
B continuous sound distance to cliff × 2
distance to cliff
C short, sharp sound 2
D short, sharp sound distance to cliff × 2

24 The diagrams show the wave shapes of two different sounds. The scales are the same in each
diagram.

time time

sound 1 sound 2

How does sound 2 compare with sound 1?

A Sound 2 is louder than sound 1.


B Sound 2 is quieter than sound 1.
C Sound 2 has a higher pitch than sound 1.
D Sound 2 has a lower pitch than sound 1.

© UCLES 2011 0625/11/O/N/11


13

25 A horseshoe magnet is brought near to an unmagnetised iron bar.

N S

iron bar

Which row in the table shows the magnetic poles induced in the iron bar and the direction of the
forces between the bar and the magnet?

magnetic poles induced force between iron bar


in iron bar and magnet

A N S attraction

B N S repulsion

C S N attraction

D S N repulsion

© UCLES 2011 0625/11/O/N/11 [Turn over


14

26 Four nails, A, B, C and D, are tested to find which makes the strongest permanent magnet.

nail
N S

bar magnet

paper clips

One of the nails is placed against a bar magnet and the number of paper clips which the nail can
support is recorded. The bar magnet is then removed and the number of paper clips remaining
attached to the nail is recorded. Each nail is tested in turn.

Which nail becomes the strongest permanent magnet?

number of paper clips attached to the nail


nail
bar magnet present bar magnet removed

A 2 0
B 2 1
C 4 3
D 5 2

27 Which symbols are used for the units of current and of resistance?

unit of current unit of resistance

A A W
B A Ω
C C W
D C Ω

© UCLES 2011 0625/11/O/N/11


15

28 The diagram shows a lamp in a circuit.

Which change to the circuit would increase the current in the lamp?

A adding another resistor in parallel with the one in the circuit


B adding another resistor in series with the one in the circuit
C decreasing the electromotive force (e.m.f.) of the battery in the circuit
D moving the lamp to point P in the circuit

29 The diagram shows an electric circuit containing three meters, X, Y and Z, all connected
correctly.

X Z

What are meters X, Y and Z?

X Y Z

A ammeter ammeter ammeter


B ammeter voltmeter ammeter
C voltmeter ammeter voltmeter
D voltmeter voltmeter voltmeter

© UCLES 2011 0625/11/O/N/11 [Turn over


16

30 The diagrams show four arrangements of resistors.

Which arrangement has the smallest total resistance?

A B
4Ω 2Ω 2Ω

C D
4Ω

4Ω 4Ω

4Ω

31 The diagram shows a potential divider circuit with two identical lamps L1 and L2.

X L1
K

Y L2

The contact K is halfway between X and Y and the lamps are equally bright.

What will happen to the brightness of the lamps when contact K is moved a short distance
towards X?

lamp L1 lamp L2

A brighter brighter
B brighter dimmer
C dimmer brighter
D dimmer dimmer

© UCLES 2011 0625/11/O/N/11


17

32 A fuse and a relay each use an effect of an electric current.

Which effect of an electric current is used by a fuse and which effect is used by a relay?

effect used by effect used by


a fuse a relay

A heating effect heating effect


B heating effect magnetic effect
C magnetic effect heating effect
D magnetic effect magnetic effect

33 Which diagram represents the direction of the magnetic field around a straight wire carrying a
current out of the page?

A B
current out
of page

C D
current out
of page

© UCLES 2011 0625/11/O/N/11 [Turn over


18

34 The diagram shows an a.c. generator.

rotation
of coil
coil

N S

output voltage

With the coil in the position shown, the output voltage is +10 V.

When does the output voltage become –10 V?

A when the coil has turned 90°

B when the coil has turned 180°

C when the coil has turned 270°

D when the coil has turned 360°

35 The diagram shows a simple transformer.

core

input output

From which material should the core be made?

A aluminium
B copper
C iron
D steel

© UCLES 2011 0625/11/O/N/11


19

36 Cathode rays are emitted in a vacuum tube. They consist of particles that are found in atoms.

What is the name of the particles and how are the cathode rays produced?

how the cathode rays


name of particles
are produced

A electrons electromagnetic induction


B electrons thermionic emission
C protons electromagnetic induction
D protons thermionic emission

37 Two parallel metal plates in a vacuum are connected to the terminals of a high-voltage power
supply. A beam of cathode rays is passed into the space between the two plates, as shown.

In which direction does the beam of cathode rays deflect?

high-voltage
power supply
+ –
metal plate
B

A
C metal plate

cathode rays D

38 A scientist needs to use a source of γ-rays as safely as possible.

Which action will not reduce the amount of radiation that reaches the scientist?

A keeping the distance between the source and the scientist as large as possible
B keeping the temperature of the source as low as possible
C keeping the time for which the scientist uses the source as small as possible
D placing a lead screen between the scientist and the source

© UCLES 2011 0625/11/O/N/11 [Turn over


20

39 The graph shows the activity of a radioactive source over a period of time.

2000
activity
/ counts per
second
1500

1000

500

0
0 1 2 3
time / hour

What is the half-life of the source?

1
A hour B 1 hour C 1 1 hours D 3 hours
2 2

26
40 A nuclide of substance X has the symbol 12 X .

How many electrons are there in a neutral atom of substance X?

A 12 B 14 C 26 D 38

Permission to reproduce items where third-party owned material protected by copyright is included has been sought and cleared where possible. Every
reasonable effort has been made by the publisher (UCLES) to trace copyright holders, but if any items requiring clearance have unwittingly been included, the
publisher will be pleased to make amends at the earliest possible opportunity.

University of Cambridge International Examinations is part of the Cambridge Assessment Group. Cambridge Assessment is the brand name of University of
Cambridge Local Examinations Syndicate (UCLES), which is itself a department of the University of Cambridge.

© UCLES 2011 0625/11/O/N/11


w
w
w
.X
tr
me
eP
ap
UNIVERSITY OF CAMBRIDGE INTERNATIONAL EXAMINATIONS

er
s
International General Certificate of Secondary Education

.c
om
PHYSICS 0625/11
Paper 1 Multiple Choice May/June 2012
45 minutes
Additional Materials: Multiple Choice Answer Sheet
Soft clean eraser
*9772978782*

Soft pencil (type B or HB recommended)

READ THESE INSTRUCTIONS FIRST

Write in soft pencil.


Do not use staples, paper clips, highlighters, glue or correction fluid.
Write your name, Centre number and candidate number on the Answer Sheet in the spaces provided
unless this has been done for you.

There are forty questions on this paper. Answer all questions. For each question there are four possible
answers A, B, C and D.
Choose the one you consider correct and record your choice in soft pencil on the separate Answer Sheet.

Read the instructions on the Answer Sheet very carefully.

Each correct answer will score one mark. A mark will not be deducted for a wrong answer.
Any rough working should be done in this booklet.

This document consists of 17 printed pages and 3 blank pages.

IB12 06_0625_11/3RP
© UCLES 2012 [Turn over
2

1 A cylindrical can is rolled along the ruler shown in the diagram.

starting position final position

can rolled
mark on
can

0 cm 5 10 15 20 25 30 cm

The can rolls over twice.

What is the circumference (distance all round) of the can?


A 13 cm B 14 cm C 26 cm D 28 cm

2 A car is moving downhill along a road at a constant speed.

Which graph is the speed / time graph for the car?

A B

speed speed

0 0
0 time 0 time

C D

speed speed

0 0
0 time 0 time

3 In a race, a car travels 60 times around a 3.6 km track. This takes 2.4 hours.

What is the average speed of the car?

A 1.5 km / h B 90 km / h C 144 km / h D 216 km / h

© UCLES 2012 0625/11/M/J/12


3

4 Which quantity is measured in newtons?

A density
B energy
C pressure
D weight

5 A geologist places a small rock on the left-hand pan of a balance. The two pans are level as
shown when masses with a total weight of 23 N are placed on the right-hand pan.
Take the weight of 1.0 kg to be 10 N.

rock masses

What is the mass of the small rock?

A 0.023 kg B 2.3 kg C 23 kg D 230 kg

6 A stone has a volume of 0.50 cm3 and a mass of 2.0 g.

What is the density of the stone?

A 0.25 g / cm3
B 1.5 g / cm3
C 2.5 g / cm3
D 4.0 g / cm3

© UCLES 2012 0625/11/M/J/12 [Turn over


4

7 Passengers are not allowed to stand on the upper deck of double-decker buses.

upper deck

lower deck

Why is this?

A They would cause the bus to become less stable.


B They would cause the bus to slow down.
C They would increase the kinetic energy of the bus.
D They would lower the centre of mass of the bus.

8 The diagram shows a handle with three forces, each 100 N, applied to it. The handle is free to
move.

100 N

100 N

100 N

handle
pivot

What is the effect of the forces on the handle?

A The handle will move downwards.


B The handle will not move.
C The handle will turn anticlockwise (to the left).
D The handle will turn clockwise (to the right).

9 In which pair of energy sources are both sources renewable?

A oil and coal


B oil and tidal
C tidal and geothermal
D tidal and nuclear fission

© UCLES 2012 0625/11/M/J/12


5

10 An object on a thread is swinging between X and Z, as shown in the diagram. It is momentarily at


rest at X and at Z.

thread

X Z
Y

An incomplete word equation about the energy of the object is shown below.

gravitational potential energy = kinetic energy + ……… energy + energy losses


at X at Y at Y

Which form of energy is needed to complete the word equation?

A chemical
B gravitational potential
C internal
D strain

11 Which statement is explained by reference to pressure?

A Objects with greater mass have greater weight.


B One kilogram of water occupies more volume than one kilogram of lead.
C Spikes on running-shoes sink into the ground.
D Water cooled to a low enough temperature turns to ice.

© UCLES 2012 0625/11/M/J/12 [Turn over


6

12 The diagram shows a mercury manometer used to measure the pressure of gas in a container.
Atmospheric pressure is 76 cm of mercury.

gas

mercury
20 cm
12 cm

What is the pressure of the gas?

A 56 cm of mercury
B 68 cm of mercury
C 84 cm of mercury
D 96 cm of mercury

13 Brownian motion is observed when looking at smoke particles in air using a microscope.

What causes the smoke particles to move at random?

A Smoke particles are hit by air molecules.


B Smoke particles are moved by convection currents in the air.
C Smoke particles have different weights and fall at different speeds.
D Smoke particles hit the walls of the container.

14 The molecules of a substance become more closely packed and move more quickly.

What is happening to the substance?

A A gas is being heated and compressed.


B A gas is being heated and is expanding.
C A liquid is boiling.
D A liquid is evaporating at room temperature.

© UCLES 2012 0625/11/M/J/12


7

15 Which pair contains only physical quantities that vary with temperature and so could be used in
making a thermometer?

A activity of a radioactive source, volume of a gas


B mass of a liquid, volume of a liquid
C activity of a radioactive source, mass of a solid
D volume of a gas, volume of a liquid

16 A heater supplies 80 J of energy to a block of metal. The temperature of the block rises by 20 °C.

What happens to the block of metal when its temperature falls by 10 °C?

A Its internal energy decreases by 40 J.


B Its internal energy decreases by 160 J.
C Its internal energy increases by 40 J.
D Its internal energy increases by 160 J.

17 An engineer wants to fix a steel washer on to a steel rod. The rod is just too big to fit into the hole
of the washer.

steel steel rod


washer

How can the engineer fit the washer on to the rod?

A Cool the washer and put it over the rod.


B Cool the washer and rod to the same temperature and push them together.
C Heat the rod and then place it in the hole.
D Heat the washer and then place it over the rod.

18 Why does convection take place in a liquid when it is heated?

A Liquids expand when they are heated.


B Liquids start to bubble when they get close to boiling point.
C Molecules in the liquid expand when they are heated.
D Molecules near to the surface of the liquid escape into the air.

© UCLES 2012 0625/11/M/J/12 [Turn over


8

19 Two plastic cups are placed one inside the other. Hot water is poured into the inner cup and a lid
is put on top, as shown.

lid

small spacer
small air gap
hot water
bench

Which statement is correct?

A Heat loss by radiation is prevented by the small air gap.


B No heat passes through the sides of either cup.
C The bench is heated by convection from the bottom of the outer cup.
D The lid is used to reduce heat loss by convection.

20 What is the unit of wavelength?

A hertz
B metre
C metre per second
D second

21 Which row correctly describes light waves and radio waves?

light waves radio waves

A longitudinal longitudinal
B longitudinal transverse
C transverse longitudinal
D transverse transverse

© UCLES 2012 0625/11/M/J/12


9

22 The diagram shows water waves passing through a gap in a harbour wall. The waves curve
round the wall and reach a small boat in the harbour.

harbour wall

waves

harbour

gap
boat

What is the name of this curving effect, and how can the gap be changed so that the waves are
less likely to reach the boat?

name of effect change to the gap

A diffraction make the gap slightly bigger


B diffraction make the gap slightly smaller
C refraction make the gap slightly bigger
D refraction make the gap slightly smaller

© UCLES 2012 0625/11/M/J/12 [Turn over


10

23 The image formed by a plane mirror is upright.

position
object of image

eye

What are the other characteristics of the image?

laterally inverted magnified


virtual
(left to right) (larger than the object)

A no yes yes
B yes no no
C yes no yes
D yes yes no

24 A student draws three rays of light from point P through a converging lens.

Each point labelled F is a principal focus of the lens.

P ray X

F F

ray Z ray Y

Which of the rays are drawn correctly?

A ray Y only
B ray Z only
C ray X and ray Y
D ray X and ray Z

© UCLES 2012 0625/11/M/J/12


11

25 A girl stands at a distance from a large building. She claps her hands and a short time later hears
an echo.

Why is an echo produced when the sound waves hit the building?

A The sound waves are absorbed.


B The sound waves are diffracted.
C The sound waves are reflected.
D The sound waves are refracted.

26 The diagrams represent the waves produced by four sources of sound. The scales are the same
for all the diagrams.

Which sound has the highest frequency?

A time

B time

C time

D time

© UCLES 2012 0625/11/M/J/12 [Turn over


12

27 Which statement describes a property of a magnet?

A It attracts ferrous materials.


B It could have only one pole (north or south).
C It points in a random direction when suspended.
D It repels non-ferrous materials.

28 Which procedure may be used to demagnetise a steel bar?

A cooling it in a freezer
B earthing it with a copper wire
C placing it in a solenoid carrying a large direct current (d.c.)
D striking it repeatedly with a hammer

29 In which unit is potential difference measured?

A ampere
B ohm
C volt
D watt

30 The circuit shown in the diagram contains an unknown component X, hidden in a box.

The voltage-current graph for X is as shown.

variable voltage supply


+ –
voltage

X
0
V 0 current

What is the component X?

A a capacitor
B a closed switch
C an open switch
D a resistor of constant resistance

© UCLES 2012 0625/11/M/J/12


13

31 Which circuit contains a fuse?

A B

C D

32 A thermistor is used in a circuit to control a piece of equipment automatically.

What might this circuit be used for?

A lighting an electric lamp as it becomes darker


B ringing an alarm bell if a locked door is opened
C switching on a water heater at a pre-determined time
D turning on an air conditioner when the temperature rises

© UCLES 2012 0625/11/M/J/12 [Turn over


14

33 A student connects a variable potential divider (potentiometer) circuit.

R
V
T

12 V

What happens to the reading on the voltmeter as the sliding terminal T is moved from R to S?

A It decreases from 12 V to 0 V.
B It increases from 0 V to 12 V.
C It remains at 0 V.
D It remains at 12 V.

© UCLES 2012 0625/11/M/J/12


15

34 A circuit-breaker is designed to protect a circuit which usually carries a current of 2 A.

The time taken to break the circuit depends on the current, as shown in the graph.

160
time taken
140
to break the
circuit / s 120
100
80
60
40
20
0
0 2 4 6 8 10 12 14 16 18 20

current / A

What happens when the current in the circuit is 2 A and what happens when the current 18 A?

when the current is 2 A when the current is 18 A

A the circuit breaks in less than 5 seconds the circuit breaks in less than 5 seconds
B the circuit breaks in less than 5 seconds the circuit does not break
C the circuit does not break the circuit breaks in less than 5 seconds
D the circuit does not break the circuit does not break

35 The diagram shows a coil connected to a battery and a switch. Two unmagnetised iron bars hang
freely near opposite ends of the coil.

coil

X Y

iron bar iron bar

What happens to the iron bars when the switch is closed?

A Both X and Y move away from the coil.


B Both X and Y move towards the coil.
C X moves towards the coil, Y moves away from the coil.
D Y moves towards the coil, X moves away from the coil.

© UCLES 2012 0625/11/M/J/12 [Turn over


16

36 A wire passes between the poles of a horseshoe magnet. There is a current in the wire in the
direction shown, and this causes a force to act on the wire.

magnet

current
N
wire S

force

Three other arrangements, P, Q and R, of the wire and magnet are set up as shown.

P Q R

S N S
N S N

magnet turned around current direction reversed current direction reversed


and magnet turned around

Which arrangement or arrangements will cause a force in the same direction as the original
arrangement?

A P, Q and R B P and Q only C P only D R only

37 The diagram shows a device to produce cathode rays.

evacuated glass container

cathode
anode

Which part of the device is heated and why?

part heated reason

A anode to emit electrons


B anode to emit protons
C cathode to emit electrons
D cathode to emit protons

© UCLES 2012 0625/11/M/J/12


17

38 A radioactive nucleus emits a β-particle.

β-particle

nucleus

What happens to the proton number (atomic number) of the nucleus?

A It stays the same.


B It increases by 1.
C It decreases by 2.
D It decreases by 4.

39 The diagram shows the paths of three different types of radiation, X, Y and Z.

2 mm of 10 mm of 50 mm
plastic aluminium of lead

Which row in the table correctly identifies X, Y and Z?

X Y Z

A α-particles β-particles γ-rays


B β-particles α-particles γ-rays
C β-particles γ-rays α-particles
D γ-rays α-particles β-particles

40 Which diagram could represent the structure of a neutral atom?

A B C D
key
– – – –
neutron

++ ++ + proton
+ ++
+ – electron

– – –

© UCLES 2012 0625/11/M/J/12


18

BLANK PAGE

© UCLES 2012 0625/11/M/J/12


19

BLANK PAGE

© UCLES 2012 0625/11/M/J/12


20

BLANK PAGE

Permission to reproduce items where third-party owned material protected by copyright is included has been sought and cleared where possible. Every
reasonable effort has been made by the publisher (UCLES) to trace copyright holders, but if any items requiring clearance have unwittingly been included, the
publisher will be pleased to make amends at the earliest possible opportunity.

University of Cambridge International Examinations is part of the Cambridge Assessment Group. Cambridge Assessment is the brand name of University of
Cambridge Local Examinations Syndicate (UCLES), which is itself a department of the University of Cambridge.

© UCLES 2012 0625/11/M/J/12


w
w
w
.X
tr
me
eP
ap
UNIVERSITY OF CAMBRIDGE INTERNATIONAL EXAMINATIONS

er
s
International General Certificate of Secondary Education

.c
om
PHYSICS 0625/11
Paper 1 Multiple Choice October/November 2012
45 minutes
Additional Materials: Multiple Choice Answer Sheet
Soft clean eraser
*8913251826*

Soft pencil (type B or HB recommended)

READ THESE INSTRUCTIONS FIRST

Write in soft pencil.


Do not use staples, paper clips, highlighters, glue or correction fluid.
Write your name, Centre number and candidate number on the Answer Sheet in the spaces provided
unless this has been done for you.

There are forty questions on this paper. Answer all questions. For each question there are four possible
answers A, B, C and D.
Choose the one you consider correct and record your choice in soft pencil on the separate Answer Sheet.

Read the instructions on the Answer Sheet very carefully.

Each correct answer will score one mark. A mark will not be deducted for a wrong answer.
Any rough working should be done in this booklet.

This document consists of 18 printed pages and 2 blank pages.

IB12 11_0625_11/3RP
© UCLES 2012 [Turn over
2

1 A pendulum is set in motion and timed. The time measured for 20 complete swings is 30 s.

What is the time for one complete swing of the pendulum?

A 0.67 s B 0.75 s C 1.5 s D 3.0 s

2 Two stones of different weight fall at the same time from a table. Air resistance may be ignored.

What will happen and why?

what will happen why

A both stones hit the floor at the same time acceleration of free fall is constant
B both stones hit the floor at the same time they fall at constant speed
C the heavier stone hits the floor first acceleration increases with weight
D the heavier stone hits the floor first speed increases with weight

3 The speed-time graph shown is for a bus travelling between stops.

Where on the graph is the acceleration of the bus greatest?

C
speed

A D

time

© UCLES 2012 0625/11/O/N/12


3

4 A large bag of feathers and a steel block balance each other on some scales.

bag of steel block


feathers

What does this show about the masses and the weights of the bag of feathers and the steel
block?

A It shows that the masses are equal and the weights are equal.
B It shows that the masses are equal, but the weights might be different.
C It shows that the masses might be different and the weights might be different.
D It shows that the weights are equal, but the masses might be different.

5 A parachutist inside an aeroplane has a mass of 70 kg.

What is his mass after he has jumped from the aeroplane?

A 0 kg
B between 0 kg and 70 kg
C 70 kg
D greater than 70 kg

© UCLES 2012 0625/11/O/N/12 [Turn over


4

6 A liquid has a density of 0.80 g / cm3.

Which could be the volume and mass of this liquid?

volume / cm3 mass / g

A 2.0 16
B 8.0 10
C 10 8.0
D 16 2.0

7 The diagram shows sections of four objects of equal mass. The position of the centre of mass of
each object has been marked with a cross.

Which object is the most stable?

A B C D

8 A see-saw is made by resting a long plank of wood with its centre of mass on a barrel.

A boy sits on one side of the barrel and a girl sits on the other side so that the see-saw is
balanced.

x y
boy girl

boy’s weight girl’s weight

Which statement must be true?

A boy’s weight = girl’s weight


B distance x = distance y
C total downward force = total moment about the barrel
D resultant force and resultant moment are both zero

© UCLES 2012 0625/11/O/N/12


5

9 A power station uses nuclear fission to obtain energy.

In this process, nuclear energy is first changed into

A chemical energy.
B electrical energy.
C gravitational energy.
D thermal (heat) energy.

10 A person lifts boxes of equal weight on to a platform.

boxes

platform

Which quantity will not affect the work done by the person?

A the height of the platform above the ground


B the number of boxes lifted
C the time taken to lift the boxes
D the weight of the boxes

11 A skier walks from the bottom of a ski slope to the top and gains 10 000 J of gravitational potential
energy.

She skis down the slope. At the bottom of the slope, her kinetic energy is 2000 J.

How much energy was converted into thermal energy and sound energy as the skier moved
down the slope?

A 2000 J B 8000 J C 10 000 J D 12 000 J

© UCLES 2012 0625/11/O/N/12 [Turn over


6

12 The diagram shows a mercury barometer.

25 cm

mercury
75 cm

5 cm

Which distance is used to calculate the pressure of the atmosphere?

A 25 cm B 75 cm C 80 cm D 100 cm

13 A heavy table has six legs. The area of cross-section of each leg is X.

The legs of the table make marks in a carpet. These marks become deeper with increased
pressure.

What would reduce the depth of the marks for a table of a fixed weight?

A using three legs, each of an area smaller than X


B using four legs, each of an area the same as X
C using six legs, each of an area smaller than X
D using eight legs, each of an area the same as X

© UCLES 2012 0625/11/O/N/12


7

14 Very small pollen grains are suspended in water. A bright light shines from the side.

Looking through a microscope, small specks of light are seen to be moving in a random, jerky
manner.

eye

microscope

light

pollen grains
in water

What are the moving specks of light?

A pollen grains being hit by other pollen grains


B pollen grains being hit by water molecules
C water molecules being hit by other water molecules
D water molecules being hit by pollen grains

15 A swimmer feels cold after leaving warm water on a warm, windy day.

Why does she feel cold even though the air is warm?

A The less energetic water molecules on her skin escape quickly.


B The more energetic water molecules on her skin do not escape quickly.
C The water on her skin does not evaporate quickly enough to keep her warm.
D The water on her skin evaporates quickly and cools her skin.

16 Which physical property is used to measure temperature in a liquid-in-glass thermometer?

A the length of the thermometer


B the thickness of the glass bulb
C the volume of the glass bulb
D the volume of the liquid

© UCLES 2012 0625/11/O/N/12 [Turn over


8

17 The graph shows the temperature of a substance as it is heated steadily.

In which part of the graph is the substance boiling?

D
temperature
C

time

18 A night storage heater contains a large block of material that is heated electrically during the
night. During the day the block cools down, releasing thermal energy into the room.

Which thermal capacity and which night-time temperature increase will cause the most energy to
be stored by the block?

thermal capacity night-time


of block temperature increase

A large large
B large small
C small large
D small small

19 After a sheep has its wool cut off, it is harder for it to stay warm when the air temperature falls.

How does the wool help the sheep to stay warm?

A Air can circulate between the wool fibres and heat up the skin by convection.
B Air trapped by the wool fibres reduces heat losses from the skin by convection.
C The wool fibres are curly so it takes longer for heat to be conducted away from the skin.
D The wool fibres conduct heat to the skin from the air outside.

© UCLES 2012 0625/11/O/N/12


9

20 A boy sits near a campfire. He pokes the fire with an iron bar. His hand becomes hot.

iron bar

In which ways does thermal energy (heat) from the fire reach his hand?

A conduction and convection only


B conduction and radiation only
C convection and radiation only
D conduction, convection and radiation

21 The diagram shows plane waves reflected by a plane surface.

Which line represents a wavefront?

A
D

22 A swimmer is sitting on a rock at the sea shore looking at passing waves. He notices that five
complete wavelengths pass him in 20 s.

What is the frequency of this wave?

A 0.25 Hz B 4.0 Hz C 15 Hz D 100 Hz

© UCLES 2012 0625/11/O/N/12 [Turn over


10

23 A thin converging lens forms an image.

object

image
F F

What is the nature of this image and can it be formed on a screen?

can be formed
nature of image
on a screen?

A not real no
B not real yes
C real no
D real yes

24 A piece of paper has ‘PAL’ written on it.

A student holds the paper in front of a plane mirror.

back of
mirror

PAL

What does the student see?

A B C D

PAL PAL
PAL LAP

© UCLES 2012 0625/11/O/N/12


11

25 A girl notices that when she shouts into a cave she hears an echo.

Which wave property causes the echo?

A diffraction
B dispersion
C reflection
D refraction

26 In a test, a car horn is found to be too loud and the pitch of the note is too high.

What information does this give about the amplitude and the frequency of the sound wave
produced?

amplitude frequency

A too large too large


B too large too small
C too small too large
D too small too small

27 Two bars of soft iron are placed near a bar magnet.

bar magnet soft iron soft iron

S N P Q

Which row states and explains the behaviour of poles P and Q of the soft iron bars?

P and Q reason

A attract P and Q are like poles


B attract P and Q are unlike poles
C repel P and Q are like poles
D repel P and Q are unlike poles

© UCLES 2012 0625/11/O/N/12 [Turn over


12

28 Some electrical devices require a magnet which may be switched on and off many times in a
second.

Which type of magnet may be used?

A an electromagnet only
B a permanent magnet only
C either a permanent magnet or an electromagnet
D neither a permanent magnet nor an electromagnet

29 Which of these is an electric current?

A a beam of atoms
B a beam of electrons
C a beam of molecules
D a beam of neutrons

30 The diagram shows a circuit used to find the resistance of lamp L.

Blocks P, Q and R represent the different components used.

12 V

P R

Which is a correct possible choice of components to use for P, Q and R?

P Q R

A ammeter variable resistor voltmeter


B variable resistor voltmeter ammeter
C voltmeter ammeter variable resistor
D voltmeter variable resistor ammeter

© UCLES 2012 0625/11/O/N/12


13

31 A student connects the circuit shown.

Which switches must be closed for the bell to ring without lighting the lamp?

A 1 and 2 only
B 1 and 3 only
C 1, 3 and 4 only
D 2, 3 and 4 only

32 Which row shows a use of a capacitor and a use of a relay?

use of a capacitor use of a relay

A switching circuit voltage transformation


B time-delay circuit switching circuit
C voltage transformation switching circuit
D voltage transformation time-delay circuit

© UCLES 2012 0625/11/O/N/12 [Turn over


14

33 The diagram shows two voltmeters, P and Q, connected to a potential divider.

V voltmeter P
X

V voltmeter Q

The sliding connection at point X is moved towards the top of the diagram.

What happens to the reading on P and to the reading on Q?

reading on P reading on Q

A decreases decreases
B decreases increases
C increases decreases
D increases increases

34 An electric oven is connected to the mains supply using insulated copper wires. The wires
become very warm.

What can be done to prevent so much heat being produced in the connecting wires?

A Use thicker copper wires.


B Use thinner copper wires.
C Use thicker insulation.
D Use thinner insulation.

© UCLES 2012 0625/11/O/N/12


15

35 Which graph shows how the voltage of a simple a.c. generator varies with time?

A B

voltage voltage

0 0
0 0
time time

C D

voltage voltage

0 0
0 time 0 time

© UCLES 2012 0625/11/O/N/12 [Turn over


16

36 The diagram shows a wire in the magnetic field between two poles of a magnet.

magnet

N S

wire

The current in the wire repeatedly changes between a constant value in one direction and a
constant value in the opposite direction. This is shown on the graph.

current

0
0 time

What is the effect on the wire?

A The force on the wire alternates between one direction and the opposite direction.
B The force on the wire is constant in size and direction.
C There is no force acting on the wire at any time.
D There is only a force on the wire when the current reverses.

© UCLES 2012 0625/11/O/N/12


17

37 A beam of cathode rays passes between two parallel, charged metal plates in a vacuum.

cathode rays

– +
– +
– +
– +
– +
– +
– +
– +
– +
– +

In which direction is the beam deflected?

A into the page


B out of the page
C to the left of the page
D to the right of the page

38 How does the ionising effect of α-particles compare with that of β-particles and γ-rays?

compared with β-particles compared with γ-rays

A α-particles are less strongly ionising α-particles are less strongly ionising
B α-particles are less strongly ionising α-particles are more strongly ionising
C α-particles are more strongly ionising α-particles are less strongly ionising
D α-particles are more strongly ionising α-particles are more strongly ionising

39 The table shows the count rates obtained from four radioactive sources. The measurements were
taken at noon on four consecutive days.

Which source has the longest half-life?

count rate /.counts per second


day 1 day 2 day 3 day 4

A 100 48 27 11
B 200 142 99 69
C 300 297 292 290
D 400 202 99 48

© UCLES 2012 0625/11/O/N/12 [Turn over


18

40 Which statement about a carbon nucleus represented by 146 C is correct?

A It contains 6 neutrons.
B It contains 6 electrons.
C It contains 8 protons.
D It contains 14 nucleons.

© UCLES 2012 0625/11/O/N/12


19

BLANK PAGE

© UCLES 2012 0625/11/O/N/12


20

BLANK PAGE

Permission to reproduce items where third-party owned material protected by copyright is included has been sought and cleared where possible. Every
reasonable effort has been made by the publisher (UCLES) to trace copyright holders, but if any items requiring clearance have unwittingly been included, the
publisher will be pleased to make amends at the earliest possible opportunity.

University of Cambridge International Examinations is part of the Cambridge Assessment Group. Cambridge Assessment is the brand name of University of
Cambridge Local Examinations Syndicate (UCLES), which is itself a department of the University of Cambridge.

© UCLES 2012 0625/11/O/N/12


CORE
PAPER TWO

2
Candidate
Centre Number Number

Candidate Name

International General Certificate of Secondary Education


UNIVERSITY OF CAMBRIDGE LOCAL EXAMINATIONS SYNDICATE

PHYSICS 0625/2
PAPER 2
Monday 24 MAY 1999 Afternoon 1 hour

Candidates answer on the question paper.


Additional materials:
Electronic calculator and/or Mathematical tables

TIME 1 hour

INSTRUCTIONS TO CANDIDATES
Write your name, Centre number and candidate number in the spaces at the top of this page.
Answer all questions.
Write your answers in the spaces provided on the question paper.

INFORMATION FOR CANDIDATES


The number of marks is given in brackets [ ] at the end of each question or part question.
You may lose marks if you do not show your working or if you do not use appropriate units.
Take the weight of 1 kg to be 10 N (i.e. acceleration of free fall = 10 m/s2).

FOR EXAMINER’S USE

This question paper consists of 15 printed pages and 1 blank page.


MML LOC 864 4/98 QF91699
© UCLES 1999 [Turn over
2 For
Examiner's
Use
1 An insect lands on a 30 cm ruler and walks along the edge, as shown in Fig. 1.1.
topicvolume

0 cm 5 10 15 20 25 30

Fig. 1.1

A child measures the time the insect takes to walk from the 5 cm mark to the 25 cm mark. It
takes 50 s to do this.

What is the average speed, in cm/s, of the insect?

speed = ...................................... [3]

0625/2/S99
3 For
Examiner's
Use
2 (a) A uniform beam AB of weight W is balanced at its midpoint on a pivot. Two weights W1
topiccentreofmass
topiccentre
topicmass
and W2 are then hung at equal distances from the midpoint of the beam.

A B

pivot

W2 W1

Fig. 2.1

When this is done, the end B moves down.

(i) Which is the heavier weight?

..................................................................................................................................
(ii) Which way would W1 have to be moved so that the beam is again balanced?

..................................................................................................................................
[2]

(b) W2 is removed from the beam. This means that the only forces acting downwards on
the beam are the weight W of the beam and W1.

W is much greater than W1.

A B

W W1

Fig. 2.2

On Fig. 2.2, mark a possible position for the pivot to be placed so that the beam is
again balanced. [2]

0625/2/S99 [Turn over


4 For
Examiner's
Use
3 (a) How does the separation of the molecules of substance X in the solid state compare
topicmolecularmodels
topicmolecular
topicmodels
with the separation when substance X is in the gaseous state? Tick one box.

molecules further apart in the solid state

molecules further apart in the gaseous state

molecules same distance apart in both cases [1]

(b) State how the molecules of substance X move in the solid state and in the gaseous
state.

solid state .........................................................................................................................

gaseous state ............................................................................................................ [2]

4 Here are some statements about energy. Complete the statements using words from the
topicwork
topicworkandpower
topicpower
following list.

chemical, electrical, geothermal, heat, hydroelectric, light,

movement (kinetic), position (potential), strain, tidal, wave

(a) A coal fire converts .............................................. energy into

.............................................. energy and .............................................. energy. [3]

(b) When a ball falls from rest, its .............................................. energy increases

and its .............................................. energy decreases. [2]

(c) The source of energy, in which hot rocks under the Earth’s surface heat water

to produce steam, is referred to as .............................................. energy. [1]

0625/2/S99
5 For
Examiner's
Use
5 (a) On Fig. 5.1, sketch the graph you would expect to get if you plotted values of the
topiccircuits
potential difference V across a metallic conductor at constant temperature and the
current I through it.
[2]

V/V

0
0 I/A

Fig. 5.1

(b) How would you use the graph to find the resistance of the conductor?

..........................................................................................................................................

..........................................................................................................................................

.................................................................................................................................... [1]

0625/2/S99 [Turn over


6 For
Examiner's
Use
6 A narrow beam of white light passes through a glass prism and is split into a band of
topicrefraction
colours, which is seen on a screen AB. This is illustrated in Fig. 6.1.

glass prism

narrow beam
of white light

Fig. 6.1

(a) What name do we give to the process by which the white light is split up into colours?

.................................................................................................................................... [1]

(b) What name do we give to the band of colours seen on the screen?

.................................................................................................................................... [1]

(c) What colour is seen at the edge of the band of colours closer to A?

.................................................................................................................................... [1]

(d) A thin sheet of clear red plastic is put in the path of the light before the light reaches
the prism. What is now seen on the screen?

.................................................................................................................................... [1]

0625/2/S99
7 For
Examiner's
Use
7 A student wraps a length of fine wire around a wood block and hangs the block between
topicforce
topicforceonaconductorinamagneticfield
topicfield
topicconductor
topicmagnetic
the poles of a magnet, as shown in Fig. 7.1.

fine wire

wood block

Fig. 7.1

(a) What is seen to happen when the student passes a current through the fine wire?

.................................................................................................................................... [1]

(b) Why does this happen?

..........................................................................................................................................

.................................................................................................................................... [2]

(c) Name a device which makes use of this effect.

.................................................................................................................................... [1]

8 State one safety reason why


topicradioactivity

(a) radioactive sources should not be touched with bare hands,

.................................................................................................................................... [1]

(b) radioactive sources emitting -rays should be stored in lead boxes with thick sides,

.................................................................................................................................... [1]

(c) the radiation symbol should be displayed on the cupboard or drawer in which
radioactive materials are kept.

.................................................................................................................................... [1]

0625/2/S99 [Turn over


8 For
Examiner's
Use
9 In a factory which makes paper, the sheets are packed in piles of 500. One pile (of
topicconverging
topicthinconverginglens
topicthin
500 sheets) has a mass of 2.4 kg, and is 0.05 m thick. The sheets measure 0.3 m x 0.2 m.
The pile is illustrated in Fig. 9.1.

500 sheets
of paper
0.3 m 0.2 m

0.05 m

Fig. 9.1

(a) Calculate
(i) the volume of the pile of 500 sheets,

volume = ............................................
(ii) the density of the paper.

density = ............................................
[6]

(b) An object of mass 1 kg has a weight of 10 N. Calculate the weight of one pile of paper.

weight = ....................................... [1]

0625/2/S99
9 For
Examiner's
Use
(c) The pressure exerted by the pile of paper when it is on a table is given by

force
pressure = –––– .
area

Calculate this pressure.

pressure = .................................... [4]

(d) Another pile of the same paper contains only 250 sheets.
(i) How does the mass of this pile compare with that of the first pile?

..................................................................................................................................
(ii) How does the density of the paper in this pile compare with that of paper in the
first pile?

..................................................................................................................................
(iii) How does the weight of this pile compare with that of the first pile?

..................................................................................................................................
(iv) How does the pressure exerted by this pile compare with that of the first pile?

..................................................................................................................................
[4]

0625/2/S99 [Turn over


10 For
Examiner's
Use
10 You are given a steel spring hanging from a support, a load and a 30 cm rule.
topictransformer

spring load 30 cm rule

Fig. 10.1

(a) In the space below, describe carefully the steps you would take in order to measure
the extension of the spring when the load is hanging on it. You may draw a diagram if
this helps you to answer the question.

.................................................................................................

.................................................................................................

.................................................................................................

.................................................................................................

.................................................................................................

.................................................................................................
[5]

(b) You also have a range of other loads which you can hang on to the spring. Equal
increases in load cause equal increases in length. Using the maximum load from this
range, the spring returns to its original length when this load is removed. On Fig. 10.2,
sketch the extension / load graph you would expect to obtain with the spring as the load
is increased. Label this line “GRAPH 1”. [2]

extension

0
0
load

Fig. 10.2
0625/2/S99
11 For
Examiner's
Use
(c) The first spring is replaced with a second spring and the experiment is repeated. The
second spring does not stretch as much as the first spring when the same load is hung
on it.

On Fig.10.2, sketch the graph you would expect to obtain.

Label this line “GRAPH 2”. [1]

(d) A wooden trolley is placed on a horizontal bench.

Fig. 10.3

The first spring is attached to the trolley, as shown in Fig.10.3.


(i) A force F keeps the spring slightly extended and the trolley moves. Describe the
motion of the trolley.

..................................................................................................................................
(ii) The first spring is replaced by the second spring, and the same force F stretches
the spring. How does the motion of the trolley compare with your answer to (d)(i)?
Explain your answer.

..................................................................................................................................

..................................................................................................................................

..................................................................................................................................
[3]

0625/2/S99 [Turn over


12 For
Examiner's
Use
11 (a) (i) Draw a clear diagram of a simple mercury barometer. [4]
topicmercurybarometer
topicbarometer
topicmanometer
topicmercury

Now fully label your diagram. [2]


(ii) State the physical quantity that can be determined by using a mercury barometer.

............................................................................................................................ [1]
(iii) On your diagram in (i), mark clearly, using the letter h, the length you would
measure to determine the physical quantity named in (ii). [2]

0625/2/S99
13 For
Examiner's
Use
(b) Fig. 11.1 shows a manometer being used to measure the pressure of the gas in a
container.

container 30 cm rule glass tube

30
25
20
15
gas

10
5
mercury

Fig. 11.1

(i) What is the difference in height of the two mercury levels?

............................. cm [1]
(ii) What does the difference in height of the mercury levels indicate?

............................................................................................................................ [1]
(iii) State whether the gas pressure is greater than or less than the atmospheric
pressure and how you know this.

..................................................................................................................................

............................................................................................................................ [2]
(iv) What would happen to the two mercury levels if the gas pressure increased
slightly?

..................................................................................................................................

............................................................................................................................ [1]
(v) The mercury manometer is replaced by another manometer that contains a liquid
of lower density. How, if at all, does this affect the difference between the liquid
levels?

............................................................................................................................ [1]

0625/2/S99 [Turn over


14 For
Examiner's
Use
12 (a) Fig. 12.1 illustrates a cathode-ray tube.
topicmolecularmodel
topicmolecular
topicmodel
topictube
topicoscilloscope

cathode anode B C
screen
A
filament

H
G F E D

Fig. 12.1

(i) Between which two points would you connect a low potential difference in order to
heat the cathode?

Between ................................... and ..................................


(ii) Between which two points would you connect a high potential difference in order
to produce cathode rays?

Between ................................... and ..................................


(iii) Between which two points would you connect a potential difference in order to
deflect the cathode rays upwards?

Between ................................... and ..................................


[3]

0625/2/S99
15 For
Examiner's
Use
(b) When the time base of a cathode-ray oscilloscope is turned on, there is a horizontal
trace across the screen, as shown in Fig. 12.2.

Fig. 12.2

(i) An alternating potential difference of constant frequency and constant amplitude


is connected to the Y-input of the oscilloscope.
On Fig. 12.2, sketch the trace which might be obtained.
(ii) The time base is switched off but the alternating potential difference is left
connected. Describe what would be seen on the screen.

..................................................................................................................................

..................................................................................................................................
[4]

(c) A microphone is connected to another cathode-ray oscilloscope, with the time base
switched to a suitable setting. First, a lady with a high-pitched voice sings into the
microphone. Then a man with a low-pitched voice sings into the microphone. Describe
how the traces seen on the screen would differ.

..........................................................................................................................................

.................................................................................................................................... [2]

0625/2/S99
Candidate
Centre Number Number

Candidate Name

International General Certificate of Secondary Education


UNIVERSITY OF CAMBRIDGE LOCAL EXAMINATIONS SYNDICATE

PHYSICS 0625/2
PAPER 2
Friday 12 NOVEMBER 1999 Morning 1 hour

Candidates answer on the question paper.


Additional materials:
Electronic calculator and/or Mathematical tables
300 mm ruler

TIME 1 hour

INSTRUCTIONS TO CANDIDATES
Write your name, Centre number and candidate number in the spaces at the top of this page.
Answer all questions.
Write your answers in the spaces provided on the question paper.

INFORMATION FOR CANDIDATES


The number of marks is given in brackets [ ] at the end of each question or part question.
You may lose marks if you do not show your working or if you do not use appropriate units.
Take the weight of 1 kg to be 10 N (i.e. acceleration of free fall = 10 m/s2).

FOR EXAMINER’S USE

This question paper consists of 15 printed pages and 1 blank page.


MML LOC 864 4/98 QK93705
© UCLES 1999 [Turn over
2 For
Examiner's
Use
1 A small tank contains water of depth 2 cm, as shown in Fig. 1.1.
topicspeed

2 cm

8 cm
5 cm

Fig. 1.1

(a) Show that the volume of water in the tank is 80 cm3.

[3]

0625/2/W99
3 For
Examiner's
Use
(b) The water is poured into the measuring cylinder shown in Fig. 1.2. On Fig. 1.2, mark
the level of the water surface in the measuring cylinder when this has been done. [1]

cm3
100

90

80

70

60

50

40

30

20

10

Fig. 1.2

0625/2/W99 [Turn over


4 For
Examiner's
Use
2 When the cardboard shape in Fig. 2.1 is freely hung from A, line AX is vertical.
topicturningeffect
topiceffect
topicturning

Y
B
C

Fig. 2.1

When it is freely hung from B, line BY is vertical.

(a) On Fig. 2.1, mark the position of the centre of mass of the shape, using a clear dot (●).
[1]

(b) On Fig. 2.1, draw a line through C which would be vertical if the shape were to be
freely hung from C. [1]

0625/2/W99
5 For
Examiner's
Use
3 Magnet A is put on a smooth (frictionless) horizontal table, as shown in Fig. 3.1.
topicmagnet

N S
magnet A

Fig. 3.1

(a) A second magnet is moved towards magnet A, as shown in Fig. 3.2.

N S N S
magnet A magnet B

Fig. 3.2

What happens to magnet A?

.................................................................................................................................... [1]

(b) Magnet C is now moved towards magnet A, as shown in Fig. 3.3.

N S S N
magnet A magnet C

Fig. 3.3

What happens to magnet A?

.................................................................................................................................... [1]

0625/2/W99 [Turn over


6 For
Examiner's
Use
(c) An iron bar is moved towards magnet A, as shown in in Fig. 3.4.

N S
magnet A iron bar

Fig. 3.4

What happens to magnet A?

.................................................................................................................................... [1]

(d) A plastic rod is moved towards magnet A, as shown in Fig. 3.5.

N S
magnet A plastic rod

Fig. 3.5

What happens to magnet A?

.................................................................................................................................... [1]

0625/2/W99
7 For
Examiner's
Use
4 A person walks from A to E, a journey which goes over the top of the hill BCD, as shown in
topicenergytransformations
topicenergy
in Fig. 4.1.
topictransformations

A B D E

Fig. 4.1

(a) (i) During which part of the walk does the person do most work? Tick one box.

AB

BC

CD

DE
(ii) Explain your answer to (a)(i).

..................................................................................................................................

..................................................................................................................................
[2]

(b) (i) The person now runs over the hill from A to E. How does the average power
developed by the person compare with that when the person walked? Tick one box.

greater than when walking

same as when walking

less than when walking


(ii) Explain your answer to (b)(i).

..................................................................................................................................

..................................................................................................................................
[3]

0625/2/W99 [Turn over


8 For
Examiner's
Use
5 Fig. 5.1 shows a battery, a switch and a bell connected so that the bell rings when the
topicresistance
switch is pushed.

battery

switch
bell

Fig. 5.1

(a) Draw the circuit diagram for this arrangement. Use standard circuit symbols.

[3]

(b) A second bell is now connected in parallel with the first bell.
(i) Copy your circuit diagram from (a) and add the second bell.

(ii) Why will the battery run out more quickly when the switch has been pushed?

..................................................................................................................................

..................................................................................................................................
[2]

0625/2/W99
9 For
Examiner's
Use
6 A ray of yellow light enters a rectangular glass block at A and leaves it at B, as shown in
topicdispersion
Fig. 6.1.

glass block

ray of
yellow light

Fig. 6.1

(a) At A on Fig. 6.1,


(i) draw the normal,
(ii) mark carefully and label clearly the angle of incidence i and the angle of refraction r.
[3]

(b) At B on Fig. 6.1, draw carefully the ray of light which emerges from the glass block. [2]

7 Fig. 7.1 shows an electric kettle.


topicconvection

element

Fig. 7.1

Explain why the heating element is placed near the bottom of the kettle.

.................................................................................................................................................

.................................................................................................................................................

........................................................................................................................................... [2]

0625/2/W99 [Turn over


10 For
Examiner's
Use
8 Describe the structure of an atom in terms of its nucleus and electrons. Do not mention
topicstructure
topicatomic
topicatomicstructure
protons or neutrons.

.........................................................................

.........................................................................

.........................................................................

.........................................................................

.........................................................................

................................................................... [3]

9 (a) Light from an illuminated slit passes through two lenses A and B and forms a focused
topicdensity
topicpressure
topicvolume
topiclens
topicrefraction
image on a screen, as shown in Fig. 9.1.

screen
lens lens
A B
slit

source
of
light

Fig. 9.1

On Fig. 9.1, indicate clearly


(i) the focal length of lens A, using the symbol fA,
(ii) the focal length of lens B, using the symbol fB.
[4]

0625/2/W99
11 For
Examiner's
Use
(b) In Fig. 9.2, O is an object placed with its base on the axis of a lens, and PF is the focal
length of the lens.

P F

Fig. 9.2

(i) Use your rule to draw two rays from the top of the object, through the lens, which
meet at the top of the image. [5]
(ii) Draw in the image, and label it I. [1]
(iii) Put ticks in the boxes alongside the statements which correctly describe your
image.

real

upright

inverted

larger than object

smaller than object

same size as object


[3]

0625/2/W99 [Turn over


12 For
Examiner's
Use
10 (a) Fig. 10.1 shows the structure of a simple transformer.
topictension

Fig. 10.1

On Fig. 10.1, label the important components of the transformer. [5]

(b) Explain why the coils of a transformer are made of copper, rather than some other
metal.

..........................................................................................................................................

.................................................................................................................................... [1]

(c) A transformer has 500 turns in its primary coil, and 1500 turns in its secondary coil.
Energy losses from the transformer are so small that they may be neglected.
V N
(i) Use the equation ––s = ––s to calculate the potential difference across the
s e c o n d a r y
Vp Np
coil when an alternating potential difference of 10 V is supplied to the primary coil.

(ii) State the value of the potential difference across the secondary coil when a
steady (d.c.) potential difference of 10 V is supplied to the primary coil.

..................................................................................................................................
[4]

(d) Another transformer has the same number of turns on its primary coil as it has on its
secondary coil. An alternating potential difference is supplied across the primary coil.
State the size of the output potential difference compared with the input potential
difference.

.................................................................................................................................... [1]

0625/2/W99
13 For
Examiner's
Use
11 (a) In the table below, write two different physical properties which may be used to
topicthermometer
measure temperature. An example has been given to help you.

The change in volume OF a liquid


OF

OF
[4]

(b) When creating a temperature scale, fixed points are needed.

Explain what is meant by a fixed point.

..........................................................................................................................................

..........................................................................................................................................

.................................................................................................................................... [2]

(c) In the table below, state the upper and lower fixed points used when calibrating a
liquid-in-glass thermometer with a centigrade temperature scale.

THE UPPER FIXED POINT IS ITS VALUE IS


THE TEMPERATURE OF

THE LOWER FIXED POINT IS ITS VALUE IS


THE TEMPERATURE OF

[5]

0625/2/W99 [Turn over


14 For
Examiner's
Use
(d) Fig. 11.1 shows how the temperature changes with time for a substance as it is heated
steadily from a solid to a liquid and then to a gas.

temperature / °C

0
time / min

Fig. 11.1

On Fig. 11.1,
(i) label the melting point and the boiling point of the substance,
(ii) indicate the time when the substance is completely liquid.
[3]

0625/2/W99
15 For
Examiner's
Use
12 A football is inflated by pumping air into it.
topiccathoderaytube
topiccathode
topicray

(a) Describe the behaviour of an air molecule in the middle of the football.

..........................................................................................................................................

.................................................................................................................................... [3]

(b) Using a diagram, describe the behaviour of the molecule near the inside surface of the
football.

..........................................................................................................................................

.................................................................................................................................... [2]

(c) Use your answer to explain how air molecules create the pressure on the inside of the
football.

..........................................................................................................................................

..........................................................................................................................................

.................................................................................................................................... [4]

(d) When the football is left in the Sun’s rays, it gets hot. Describe what happens to the air
molecules, and how this affects the pressure of the football.

..........................................................................................................................................

..........................................................................................................................................

.................................................................................................................................... [3]

0625/2/W99
Candidate
Centre Number Number

Candidate Name

International General Certificate of Secondary Education


UNIVERSITY OF CAMBRIDGE LOCAL EXAMINATIONS SYNDICATE

PHYSICS 0625/2
PAPER 2
MAY/JUNE SESSION 2000 1 hour

Candidates answer on the question paper.


Additional materials:
Electronic calculator and/or Mathematical tables
Ruler (30 cm)

TIME 1 hour

INSTRUCTIONS TO CANDIDATES
Write your name, Centre number and candidate number in the spaces at the top of this page.
Answer all questions.
Write your answers in the spaces provided on the question paper.

INFORMATION FOR CANDIDATES


The number of marks is given in brackets [ ] at the end of each question or part question.
You may lose marks if you do not show your working or if you do not use appropriate units.
Take the weight of 1 kg to be 10 N (i.e. acceleration of free fall = 10 m/s2).

FOR EXAMINER’S USE

This question paper consists of 15 printed pages and 1 blank page.


SB (SM/JG) QF05708/3
© UCLES 2000 [Turn over
2 For
Examiner’s
Use
1 Read the sentences below and then answer the questions which follow.
topicvolume
topicweight
topicmass
topicdensity

“When potatoes are bought in a market, the weight of a bag full of potatoes is affected by the
density of the potatoes. A lady fills her bag when she buys 5 kg of large potatoes. A man
buys 5 kg of small potatoes. He puts them in a bag of the same size as the lady’s, but his bag
is not filled.”

(a) Which word in these sentences describes a quantity which is a force?

......................................................................................................................................[1]

(b) What does the 5 kg measure? Tick one box.

the density of the potatoes

the mass of the potatoes

the volume of the potatoes

the weight of the potatoes [1]

topictransfer topicthermal topicenergy

(c) Suggest one reason why the man’s 5 kg of potatoes occupies less volume than the
lady’s potatoes.

......................................................................................................................................[1]

2 A dish of hot food is put on a wooden table.


topicturningeffect
topiceffect
topicturning
topicenergy

Fig. 2.1

(a) State three processes by which the dish and its contents could lose heat to the
surroundings.

1. ...................................................................................................................................

2. ...................................................................................................................................

3. ...............................................................................................................................[3]

(b) (i) Describe one way of reducing the heat loss to the surroundings.

...................................................................................................................................

...................................................................................................................................

(ii) Which form of heat loss would this reduce?

...............................................................................................................................[2]

0625/2 S00
3 For
Examiner’s
Use
3 You are given an iron bar, a reel of insulated wire, a battery and some wire cutters.
topicmagnet

iron bar reel of battery wire cutters


insulated
wire
Fig. 3.1

(a) In the space below, describe how you would make an electromagnet. You may use a
labelled diagram if it helps you to answer the question.

..........................................................................................................................................

..........................................................................................................................................

......................................................................................................................................[3]

(b) How would you check that your electromagnet actually works?

..........................................................................................................................................

..........................................................................................................................................

......................................................................................................................................[1]

0625/2 S00 [Turn over


4 For
Examiner’s
Use
4 At night, the light beam from a torch is shone into a swimming pool along the line TSA.
topicenergy
Instead of striking the bottom of the pool at A, the beam travels to B, as shown on Fig. 4.1.
topicenergytransformations
topictransformations
topicapparentdepth

T torch

S
water

39° 55°
A B

Fig. 4.1
(a) At S, the direction of the beam changes. State the name we use to describe this change.

......................................................................................................................................[1]

(b) (i) On Fig. 4.1, draw the normal to the surface at S.


(ii) Clearly mark and label the angle of incidence. [2]

(c) Fig. 4.2 shows the same pool and the same points A, B, S and T. The critical angle for
the water is 50°.

S
water

39° 55°
A B
Fig. 4.2
(i) A beam of light is directed up from B to S. On Fig. 4.2, carefully draw the path of
the ray from B to S and then out into the air.
(ii) 1. A beam of light is directed up from A to S. Describe what happens to the beam
at S.

...........................................................................................................................

...........................................................................................................................

2. Explain why this happens.

...........................................................................................................................

.......................................................................................................................[4]
0625/2 S00
5 For
Examiner’s
Use
5 A man is watching a thunderstorm which is directly over a village. Some distance behind the
topiclight
topicsound
topicspeedofsoundandlight
topicspeed
village is a mountain.

cloud mountain

lightning

village

man

Fig. 5.1

(a) Thunder is created at the same time as the lightning flash but, after the man sees a
lightning flash, he has to wait a short time before he hears the thunder.
Why is there this delay?

..........................................................................................................................................

..........................................................................................................................................

..........................................................................................................................................

......................................................................................................................................[2]

(b) When he listens carefully, the man realises that, for each lightning flash, he can hear a
loud sound of thunder followed by a quieter one.

(i) After studying Fig. 5.1, explain why he hears two sounds for each lightning flash.

...................................................................................................................................

...................................................................................................................................

(ii) Suggest why the second sound is quieter.

...................................................................................................................................

...............................................................................................................................[2]

0625/2 S00 [Turn over


6 For
Examiner’s
Use
(c) The man measures the time between seeing a flash of lightning over the village, and
hearing the first sound of thunder. The time is 4 s.
The speed of sound in air is 330 m/s.
How far away is the village?

ANSWER: ................................. m [3]

6 Some smoke is mixed with the air in a glass box. The box is lit brightly from the side and its
topicbrownian
topicbrownianmotion
topicmotion
contents studied from above through a microscope.

glass box containing


air and smoke
mixed together

bright light

Fig. 6.1

(a) Bright specks are seen moving in continuous and jerky random movement.

(i) What are the bright specks? Tick one box.

air molecules

smoke molecules

smoke particles

0625/2 S00
7 For
Examiner’s
Use
(ii) What is the explanation for the jerky random movement? Tick one box.

The air molecules bombard each other.

The smoke particles bombard each other.

The air molecules bombard the smoke particles.

The air molecules bombard the glass.

The smoke particles bombard the glass. [2]

(b) The contents of the glass box exert a pressure on the glass walls.
Tick any of the following sentences which might help explain this pressure.

The air molecules bombard each other.

The smoke particles bombard each other.

The air molecules bombard the smoke particles.

The air molecules bombard the glass.

The smoke particles bombard the glass. [2]

0625/2 S00 [Turn over


8 For
Examiner’s
Use
7 Fig. 7.1 shows one way of using water to generate electricity.
topicforce
topicconductor
topicforceonaconductorinamagneticfield

(a) Fill in the missing words in the boxes. [4]

reservoir
dam
(iii) The water rotates the
pipe
………………………………
(i) This water has so that the generator
produces
………………………
energy because ………………………………
of its height. energy.

(ii) This moving water


has increased
generator
………………………… house
energy.

Fig. 7.1

(b) In other places, water is used in different ways to generate electricity.


State two of these ways.

1. ...................................................................................................................................

2. ...............................................................................................................................[2]

0625/2 S00
9 For
Examiner’s
Use
8 This question deals with the decay of a radioactive source.
topicradioactivity
The radioactive source has a count rate of 640 counts/minute at the start of an experiment.
This value has been plotted on Fig. 8.1.

600

500
counts / minute

400
count rate

300

200

100

0
0 10 20 30 40 50 60 70 80
time / minutes
Fig. 8.1

The source has a half-life of 20 minutes.

(a) (i) What would you expect the count rate to be after 20 minutes?

............................................... counts/minute

(ii) Plot this value on the graph. [2]

(b) (i) What would you expect the count rate to be after a further 20 minutes
(i.e. 40 minutes after the start of the experiment)?

............................................... counts/minute

(ii) Plot this value on the graph. [2]

(c) Plot two further points which might be expected if the decay curve were perfect. [1]

(d) Draw a smooth curve through all five points on your graph. [1]

0625/2 S00 [Turn over


10 For
Examiner’s
Use
(e) If this perfect decay continued, how long would it take from the beginning of the
experiment for the count rate to decrease to zero?
Tick one answer.

90 minutes

100 minutes

120 minutes

a very long time

an infinite time [1]

(f) In a real experiment, the values found for the count rates might not all lie exactly on a
smooth curve. One reason for this might be experimental error. State one other reason.

......................................................................................................................................[1]

9 (a) Heavy furniture sometimes marks the floor on which it stands.


topicpressure
Four tables of the same weight each have four legs. Fig. 9.1 shows part of a leg from
each table.

A B C D

Fig. 9.1

(i) Which leg is least likely to mark the floor underneath it? ...............
(ii) Explain your answer.

...................................................................................................................................

...............................................................................................................................[3]

0625/2 S00
11 For
Examiner’s
Use
(b) A hot flat metal sheet is placed on a horizontal surface.

A C

Fig. 9.2

As the hot metal sheet cools, what happens to the quantities in the list below?
Tick one answer for each.

increases decreases stays the same

length AB

width BC

thickness CD
area touching the
horizontal surface
mass of sheet

weight of sheet

density of metal
pressure on
horizontal surface
[6]

0625/2 S00 [Turn over


12 For
Examiner’s
Use
10 A laboratory technician wants to make a resistor of value 64 Ω, using some resistance wire.
He takes 1.0 m of this wire. The wire is shown in Fig. 10.1 as AC. He connects up the circuit
topictension

shown.

V
uniform
resistance
wire

A 0.5 m B 0.5 m C
crocodile clip

Fig. 10.1

(a) He connects the crocodile clip at B, which is 0.5 m from A.


Here are the readings he gets.

voltmeter reading 12 V

ammeter reading 1.5 A

Calculate the resistance of wire AB.

ANSWER: resistance of AB = ................... Ω [3]

(b) The laboratory technician now connects the crocodile clip to C, to measure the
resistance of 1 m of the wire. The wire has constant thickness.

(i) In the spaces below, write the readings he obtains. Ignore the effects of the
resistance of the ammeter, voltmeter and battery.

voltmeter reading ............... V

ammeter reading ............... A

0625/2 S00
13 For
Examiner’s
Use
(ii) What is the resistance of wire AC?

ANSWER: resistance of AC = ................ Ω [3]

(c) Use your answer to (b) to answer the following questions.

(i) What is the resistance per metre of this wire?

ANSWER: resistance per metre = ........... Ω/m

(ii) What length of wire does the laboratory technician need for the 64 Ω resistor?

ANSWER: length needed = .................. m [3]

0625/2 S00 [Turn over


14 For
Examiner’s
Use
11 Here is a list of different types of waves.
topicmercury
topicbarometer
topicmercurybarometer

gamma (γ)
infra-red
radio
sound
ultra-violet
visible
X-rays

(a) Which one of these is the only one which is not part of the electromagnetic spectrum?

......................................................................................................................................[1]

(b) Which one of these makes us feel warm when the Sun shines?

......................................................................................................................................[1]

(c) Which one of these do doctors use to detect broken bones?

......................................................................................................................................[1]

(d) (i)

Fig. 11.1

On the moon, two astronauts cannot hear each other, even when they shout,
unless they have their radios switched on.

1. Why cannot they hear each other even when they shout?

...........................................................................................................................

...........................................................................................................................

2. Why can they hear each other using their radios?

...........................................................................................................................

...........................................................................................................................

(ii) Which type of wave is used to carry messages from the astronauts to mission
control on Earth?

...............................................................................................................................[4]

0625/2 S00
15 For
Examiner’s
Use
12 The hammers A and B shown in Fig. 12.1 consist of steel hammer-heads of different weights
fitted to identical wooden handles.
topictube
topiccathoderaytube
topicray
topiccathode

A B

hammer handle
-head

nail

Fig. 12.1

(a) (i) Which hammer-head causes the greater moment about the end of the handle
when the hammer is held horizontally, as shown in Fig. 12.1?

ANSWER: hammer ............

(ii) Explain your answer.

...................................................................................................................................

...............................................................................................................................[3]

(b) (i) Which hammer-head requires the greater work to lift it a distance of 30 cm from the
position shown?

ANSWER: hammer ............


(ii) Explain your answer.

...................................................................................................................................

...............................................................................................................................[3]

(c) If you wanted to estimate the power developed when lifting hammer A through 30 cm,
what two other quantities would you need to measure?

1. ...................................................................................................................................

2. ...............................................................................................................................[2]

(d) One of the hammers hits the nail and comes to rest without bouncing.

(i) What form of energy did the hammer lose when it came into contact with the nail?

...................................................................................................................................

(ii) State two forms of energy into which this “lost” energy is changed.

1. ...........................................................................................................................

2. .......................................................................................................................[3]

0625/2 S00
Candidate
Centre Number Number

Candidate Name

International General Certificate of Secondary Education


UNIVERSITY OF CAMBRIDGE LOCAL EXAMINATIONS SYNDICATE

PHYSICS 0625/2
PAPER 2
Friday 10 NOVEMBER 2000 Morning 1 hour

Candidates answer on the question paper.


Additional materials:
Electronic calculator and/or Mathematical tables
Ruler (30 cm)

TIME 1 hour

INSTRUCTIONS TO CANDIDATES
Write your name, Centre number and candidate number in the spaces at the top of this page.
Answer all questions.
Write your answers in the spaces provided on the question paper.

INFORMATION FOR CANDIDATES


The number of marks is given in brackets [ ] at the end of each question or part question.
You may lose marks if you do not show your working or if you do not use appropriate units.
Take the weight of 1 kg to be 10 N (i.e. acceleration of free fall = 10 m/s2).

FOR EXAMINER’S USE

This question paper consists of 16 printed pages.


SB (KN/DJ) QK07574/3
© UCLES 2000 [Turn over
2 For
Examiner’s
Use
1 Moving cars always experience friction. A driver goes on a short journey in a car.
topicweight
topicmass
topicdensity
topicvolume

Fig. 1.1 shows the car at four places during the journey. The arrows represent the size and
direction of the horizontal forces on the car.

driving friction driving friction


force force force force

The car is ............................... . The car is ............................... .

driving friction
force force

The car is ............................... . The car is ............................... .

Fig. 1.1

On the line underneath each picture, state whether the car is

at rest,
speeding up,
going at steady speed,
slowing down. [4]

0625/2 W00
3 For
Examiner’s
Use
2 (a) Fig. 2.1 shows three types of water tap (faucet).
topictransfer topicthermal topicenergy

type A type B type C

PUSH KNOB
DOWN

Fig. 2.1

In the box underneath each tap, write YES if a person would need to cause a moment
on the handle in order to make the water flow, or NO if the person would not need to
cause a moment. [3]

(b) A old person has taps of type A in the kitchen. The person has difficulty operating the
tap. What could be done to make it easier to operate the tap?

......................................................................................................................................[1]

0625/2 W00 [Turn over


4 For
Examiner’s
Use
3 (a) Fig. 3.1 shows the speed/time graph for a motorcycle.
topicmolecularmodels
topicmodels
topicmolecular

35

30

25

20
speed
m/s
15

10

0
0 5 10 15 20 25 30 35 40
time/s

Fig. 3.1

(i) What is the maximum speed of the motorcycle? ..................... m/s

(ii) Whilst accelerating, the motorcycle changes gear three times.

State one of the speeds at which the gear is changed. ............ m/s

(iii) For how long is the motorcycle slowing down? ...................... s


[3]

0625/2 W00
5 For
Examiner’s
Use
(b) On another occasion, the motorcycle is made to increase its speed at a constant rate for
10 s. The speed/time graph for this is shown in Fig. 3.2.

20

15

speed
10
m/s
5

0
0 5 10
time/s

Fig. 3.2

How far does the motorcycle travel in these 10 s?

distance travelled = ................ m [3]

0625/2 W00 [Turn over


6 For
Examiner’s
Use
4 (a) Some students are asked to write down what they know about evaporation of a liquid.
topicevaporation
Here are their statements, some of which are correct and some incorrect.

Put a tick alongside those statements which are correct.

A “Evaporation occurs at any temperature.”

B “Evaporation only occurs at the boiling point.”

C “Evaporation occurs where the liquid touches the bottom of the container.”

D “Evaporation occurs at the surface of the liquid.”

E “It is the higher energy molecules which escape.”

F “The molecules gain energy as they escape.”

G “The liquid temperature always rises when evaporation occurs.”

H “Rapid evaporation produces cooling.”

[4]

(b) Sometimes after shaving, men splash a liquid, called an aftershave, over their faces.
This makes their faces feel fresher as the aftershave evaporates.

(i) Which of the statements in part (a) explains why the aftershave, even though it is at
room temperature, cools the skin.

statement ................

(ii) Suggest why the aftershave cools the skin better than water at room temperature.

...................................................................................................................................

...................................................................................................................................
[2]

0625/2 W00
7 For
Examiner’s
Use
5 (a) Complete the following sentence.
topicsound
topicspeedofsoundandlight
topicspeed
topicthermal
topicthermalcapacity
topiclight

“The temperature of a body rises when the .............................................. energy of its


molecules is increased.” [1]

(b) Fig. 5.1 gives details about an empty beaker and the same beaker with different
substances in it.

empty beaker beaker


beaker + water + sand

mass 250 g 500 g 500 g

energy needed
to raise 125 J 1175 J 325 J
temperature
by 1°C

Fig. 5.1

(i) Which of the arrangements has the highest thermal capacity?

...................................................................................................................................

(ii) 1. What is the mass of the water? ..................... g

2. What is the mass of the sand? ..................... g

3. How much energy is needed to raise the temperature of

the water by 1°C? ..................... J

4. How much energy is needed to raise the temperature of

the sand by 1°C? ..................... J

5. Use your answers above to suggest why, on a sunny day, the temperature of
the sand on a beach rises faster than the temperature of the sea.

...................................................................................................................................

...................................................................................................................................

...............................................................................................................................[6]

0625/2 W00 [Turn over


8 For
Examiner’s
Use
6 Fig. 6.1 shows a view from above of a vertical mirror. A small lamp is placed at the point
topicbrownian
topicmotion
marked L.
topicbrownianmotion

mirror
P

Fig. 6.1

(a) One ray, LP, from the lamp has been drawn.

(i) At P, draw and label the normal to the mirror.

(ii) At P, draw and label the reflected ray.

(iii) Mark, using an X for each, two angles which are equal.
[3]

(b) Carefully mark, using a clear dot, the position of the image of the lamp. [1]

(c) If you were looking into the mirror from point L, you might see something like Fig. 6.2
“looking back at you”. (Apologies if you are better-looking than this!)

Fig. 6.2

(i) Mark clearly with the letter R, the image of your right ear.

(ii) Your nose is 30 cm from the mirror.


How far from your nose is its image? ........................................................................
[2]

0625/2 W00
9 For
Examiner’s
Use
7 (a) A ray of red light passes through a glass prism, as shown in Fig. 7.1.
topicforceonaconductorinamagneticfield
topicforce
topicconductor

red red
light light

Fig. 7.1

What name do we use for the change of direction of the ray as it enters the glass?

......................................................................................................................................[1]

(b) Fig. 7.2 shows the same prism, with white light passing through it.
The path of red light is shown.

white red
light light

Fig. 7.2

(i) On Fig. 7.2, draw a possible path for blue light.


(ii) Something else is happening to the white light, in addition to what is shown in
Fig. 7.1.
What name do we use for this? ................................................................................
[5]

(c) Light from the Sun is now passed through the prism. The path of red light is shown in
Fig. 7.3.

Sun's red
light light

Fig. 7.3

We can detect infra-red rays using a thermocouple. On Fig. 7.3, mark with the letter T a
position where the thermocouple could detect the infra-red rays after they have passed
through the prism. [1]
0625/2 W00 [Turn over
10 For
Examiner’s
Use
8 Here are the approximate densities of some metals.
topicradioactivity

platinum 21 000 kg/m3 (21 g/cm3)


gold 19 000 kg/m3 (19 g/cm3)
lead 11 000 kg/m3 (11 g/cm3)
brass 9 000 kg/m3 ( 9 g/cm3)
iron 8 000 kg/m3 ( 8 g/cm3)
aluminium 3 000 kg/m3 ( 3 g/cm3)

A person sees a coin offered for sale in an antiques market.

Fig. 8.1

The market trader says that the coin is made of gold. After buying the coin, the person finds
that its volume is 1.4 cm3 and its mass is 12.6 g.

(a) Write down the equation which enables you to calculate density.

[1]

(b) Calculate the density of the metal from which the coin is made.

density = ....................... g/cm3 [2]

(c) Is the coin made of gold? YES/NO [1]

(d) If not, use the list above to suggest what it might be made from.

The coin might be made from ................................ .[1]

(e) If a country wanted to keep its coinage the same but of as low a mass as possible,
which of the metals in the list should it choose?

......................................................................................................................................[1]

0625/2 W00
11 For
Examiner’s
Use
9 The circuit in Fig. 9.1 is connected up.
topiclens
topicvolume
topicpressure
topicdensity

20 Ω

6V

40 Ω

Fig. 9.1

(a) Calculate the combined resistance of the two resistors in Fig. 9.1.

combined resistance = ..................... Ω [2]

(b) (i) State the relationship between resistance, p.d. and current by completing the
following equation.

resistance = ––––––––––––––––––––

(ii) Calculate the current, I, in Fig. 9.1. State the unit in your answer.

current = ..............................
[5]

0625/2 W00 [Turn over


12 For
Examiner’s
Use
(c) Use your answer to (b)(ii) to calculate the p.d. across the 40 Ω resistor.
State the unit in your answer.

p.d. = ................................ [3]

(d) The circuit is now used as a potential divider, as shown in Fig. 9.2.

20 Ω

6V

40 Ω V out

Fig. 9.2

Use your answer to (c) to state the value of Vout , the output voltage of the potential
divider.
Vout = ............................. V [1]

0625/2 W00
13 For
Examiner’s
Use
10 (a) Fig. 10.1 shows a view from above of a person standing at the edge of a pond, dipping
topicripples
topicwaves
the end of a stick up and down in the water.
Some of the wavefronts that spread out are shown.

edge
small piece
water of
of wood,
pond
floating on
water

X
cm
90
Y

Fig. 10.1

(i) How many wavelengths are there between X and Y? ..........................................[1]

(ii) The distance from X to Y is 90 cm. Calculate the wavelength of the waves.

wavelength = .......................... cm [2]

(iii) The speed of the waves is affected by the depth of the water.

1. Describe the shape of the wavefronts, as seen from above.

...........................................................................................................................

2. What does the shape of the wavefronts tell you about the depth of the pond?

Give a reason for your answer.

...........................................................................................................................

...........................................................................................................................
[3]

0625/2 W00 [Turn over


14 For
Examiner’s
Use
(iv) Fig. 10.2 shows a sideways view of the water surface just before the first wave
reaches the floating piece of wood.

small piece stick


of wood

direction of
wave travel

Fig. 10.2

Describe how the piece of wood moves after the waves reach it.
You may draw on Fig. 10.2 if it helps you to answer the question.

...................................................................................................................................

...............................................................................................................................[2]

(b) An underwater loudspeaker, placed in the pond in part (a), sends out sound waves
through the water, as shown in Fig. 10.3.

underwater
loudspeaker
L

Fig. 10.3

(i) What is the difference between the nature of these sound waves and the water
waves in (a)? Write the appropriate words in the gaps in the following sentences.

“Water waves are ..................................... waves.”

“Sound waves are ..................................... waves.” [2]

(ii) Fig. 10.4 shows a sideways view along the line KL.

underwater
loudspeaker
M
K L

Fig. 10.4

The dot labelled M represents a water molecule on the line KL.


Describe how the molecule moves when the loudspeaker is working.
You may draw on Fig. 10.4 if it helps you to answer the question.

...................................................................................................................................

...............................................................................................................................[2]
0625/2 W00
15 For
Examiner’s
Use
11 (a) (i) Copper is an electrical conductor. What is meant by a conductor ?
topicmercury
topicmercurybarometer
topicbarometer

...................................................................................................................................

(ii) Ebonite, glass and polythene are electrical insulators. What is meant by an
insulator ?

...................................................................................................................................
[2]

(b) Polythene is easily given a negative charge by rubbing it with a dry woollen cloth.

(i) Fig. 11.1 shows a charged nylon thread


polythene rod being held
close to a suspended charged
polythene rod.
moves
Complete the phrase, away
“like charges .....................”.
negatively
charged polythene

Fig. 11.1

(ii) Fig.11.2 shows rod X being nylon thread


held near the suspended
charged polythene rod.

moves
negatively towards
Tick any of the following which charged
might correctly describe rod X. polythene
rod X
positively charged glass

negatively charged ebonite


Fig. 11.2
uncharged copper

negatively charged polythene


[3]

0625/2 W00 [Turn over


16 For
Examiner’s
Use
210
12 (a) One nuclide is written as 84 Po.
topicradioactivity

(i) Which figure is the proton number (atomic number)? ...............................................

(ii) Which figure is the nucleon number (mass number)? ..............................................

(iii) Which figure gives the number of protons in the nucleus? .......................................

(iv) How can you find the number of neutrons in the nucleus?

...................................................................................................................................
[4]
4
(b) An α-particle can be written as 2 α.
210
Polonium 84 Po decays into lead (Pb) by emitting an α-particle.

Complete the nuclear equation below, by writing the correct numbers in the boxes.

210 4
84 Po → Pb + 2 α
[2]

0625/2 W00
Candidate
Centre Number Number

Candidate Name

International General Certificate of Secondary Education


UNIVERSITY OF CAMBRIDGE LOCAL EXAMINATIONS SYNDICATE

PHYSICS 0625/2
PAPER 2
MAY/JUNE SESSION 2001 1 hour

Candidates answer on the question paper.


No additional materials required.

TIME 1 hour

INSTRUCTIONS TO CANDIDATES
Write your name, Centre number and candidate number in the spaces at the top of this page.
Answer all questions.
Write your answers in the spaces provided on the question paper.

INFORMATION FOR CANDIDATES


The number of marks is given in brackets [ ] at the end of each question or part question.
You may lose marks if you do not show your working or if you do not use appropriate units.
Take the weight of 1 kg to be 10 N (i.e. acceleration of free fall = 10 m/s2).

FOR EXAMINER’S USE

This question paper consists of 13 printed pages and 3 blank pages.


SB (SC/CG) QF10309/4
© UCLES 2001 [Turn over
2 For
Examiner’s
Use
1 A machine operator is making metal cylinders. The factory inspector wants to check whether
topicspeed
topicdensity
topicweight
the machine operator is working fast enough.

(a) He tells the operator to start working when the clock on the wall of the factory shows the
time in Fig. 1.1.

What time is this? Tick one box.


11 12 1
10 2 3.01
9 3
4
1.03
8
7 5
6 3.05

5.03

Fig. 1.1 [1]

(b) The operator is told to stop when the clock shows the time in Fig. 1.2.

What time is this? Tick one box.


11 12 1
10 2 3.07
9 3
4
7.03
8
7 5
6 3.35

4.35

Fig. 1.2 [1]

(c) How long did the test take?

length of test = ................... minutes [1]

(d) During this time, the operator makes 5 cylinders. What is the average time to make one
cylinder?

time to make one cylinder = ................... minutes [2]

0625/2/M/J/01
3 For
Examiner’s
Use
2 (a) What is meant by the term moment of a force?
topictransfer topicthermal topicenergy

..........................................................................................................................................

......................................................................................................................................[2]

(b) The sawn-off branch of a tree is laid across a log.

Fig. 2.1

The branch balances when point A is in contact with the log.

(i) How does the moment of the part of the branch to the left of A compare with the
moment of the part to the right of A?

...................................................................................................................................

(ii) On Fig. 2.1, mark clearly, using the letter X, the centre of mass of the whole
branch.
[2]

3 A rubber balloon is filled with air.


topicmodels
topicmolecular
topicmolecularmodels

(a) Describe how the pressure in the balloon is caused by the air molecules.

..........................................................................................................................................

......................................................................................................................................[3]

(b) The temperature of the air in the balloon increases.

(i) What happens to the air molecules?

...................................................................................................................................

...................................................................................................................................

(ii) What happens to the pressure in the balloon, and why?

what happens ...........................................................................................................

...................................................................................................................................

why ...........................................................................................................................

...................................................................................................................................
[3]

0625/2/M/J/01 [Turn over


4 For
Examiner’s
Use
4 Fig. 4.1 shows the view from above of a triangular object on one side of a vertical mirror.
topictransformations
topicenergytransformations
topicenergy

object

mirror

Fig. 4.1

On Fig. 4.1, carefully draw the image formed by the mirror. [3]

0625/2/M/J/01
5 For
Examiner’s
Use
5 Fig. 5.1 represents a wave.
topicresistance

displacement
B H

C I
0
A D E G distance
from source

Fig. 5.1

(a) Making use of the letters on Fig. 5.1, state which distances you would measure to find

(i) the wavelength of the wave: measure between ……… and ……… .

(ii) the amplitude of the wave: measure between ……… and ……… .
[2]

(b) What is meant by the frequency of the wave?

..........................................................................................................................................

......................................................................................................................................[2]

(c) One complete wave takes 0.2 s to generate.

Calculate the frequency of the wave.

frequency of wave = ...................Hz [2]

0625/2/M/J/01 [Turn over


6 For
Examiner’s
Use
6 A charged ebonite rod has negative charges all over its surface. It is held above three small
topicmotion
topicbrownian
topicbrownianmotion
pieces of aluminium foil, one positively charged, one negatively charged and one uncharged.
This is shown in Fig. 6.1.

ebonite
rod

insulating
+ sheet

positive negative no charge

Fig. 6.1

(a) Put a circle around any of the pieces of aluminium which are attracted by the ebonite
rod. [2]

(b) Ebonite is an insulator. What is meant by the term insulator?

..........................................................................................................................................

......................................................................................................................................[2]

(c) Write down the name of another insulating material.

......................................................................................................................................[1]

0625/2/M/J/01
7 For
Examiner’s
Use
7 Three resistors are connected in series between X and Y. When a cell is connected across
topicconvection
XY, the current at X is 0.1 A, as shown in Fig. 7.1.

X 12 Ω 5Ω 3Ω Y
0.1A
V1

Fig. 7.1

(a) What is the value of the current at Y? ................... A [1]

(b) Calculate V1, the p.d. across the 12 Ω resistor.

p.d. across the 12 Ω resistor = ...................V [2]

(c) What instrument would you use to measure the p.d. V1? ............................................[1]

(d) How does the e.m.f. of the cell compare with your answer to part (b)? Tick one box.

e.m.f. of cell is larger than V1

e.m.f. of cell is smaller than V1

e.m.f. of cell is the same as V1 [1]

(e) Calculate the combined resistance of the three resistors.

combined resistance = ................... Ω [2]

0625/2/M/J/01 [Turn over


8 For
Examiner’s
Use
8 (a) Fig. 8.1 shows a simple transformer.
topicradioactivity

core

Vp Vs

coil coil
N p turns N s turns

Fig. 8.1

Complete the following sentences about the transformer.

‘The transformer only works using ………………………………………… current.

It steps the potential difference up or down according to the equation

Vp _____ .
__ =
Vs

The core of the transformer is made of …………………………. .’ [3]

(b) In each of the following examples, state the potential difference Vs.

(i)

12 v~ Vs

100 turns 50 turns


Vs = ................... V

(ii)

6 v~ Vs

80 turns 80 turns
Vs = ................... V

(iii)

6v Vs

80 turns 80 turns
Vs = ................... V
[4]
0625/2/M/J/01
9 For
Examiner’s
Use
9 The table below gives some data about an accelerating car.
topicpressure

time/s 0 1 2 3 4 6 8 10
speed
m/s 0 5 10 15 19 24 25 25

(a) On Fig. 9.1, plot the speed/time graph for the motion.

30
speed
m/s

20

10

0
0 2 4 6 8 10
time/s

Fig. 9.1 [4]

(b) How far did the car travel during the first 3 s?

distance travelled = ...................m [3]

(c) What was the top speed of the car? .....................................................................m/s [1]

(d) How far would the car travel in 3 s if travelling at its top speed?

distance travelled = ...................m [3]


0625/2/M/J/01 [Turn over
10 For
Examiner’s
Use
10 Two workers, A and B, are lifting boxes of food in a store-room. The boxes all weigh the
topicwaves
topicripples
topicenergy
same and are lifted from the floor on to the same shelf.

A is able to lift 10 boxes in 2 minutes.

B takes longer than 2 minutes to lift 10 boxes.

(a) How does the total work done by A compare with the total work done by B?

......................................................................................................................................[1]

(b) How does the power of A compare with the power of B?

......................................................................................................................................[1]

(c) (i) Which form of energy in their bodies do the workers transform in order to do the
work lifting the boxes?

...................................................................................................................................

(ii) From what did they obtain this supply of energy?

...................................................................................................................................
[2]

(d) The boxes have more energy when they are on the shelf than when they were on the
floor.

Which form of energy has increased?...........................................................................[1]

(e) One of the boxes falls off the shelf and crashes to the ground.

Describe the energy changes as the box falls and hits the ground.

..........................................................................................................................................

..........................................................................................................................................

......................................................................................................................................[4]

0625/2/M/J/01
11 For
Examiner’s
Use
11 (a) A builder is building a brick wall. He has 500 bricks delivered, all neatly stacked together.
topicmercurybarometer
topicmercury
topicbarometer

Each brick measures 0.2 m x 0.1 m x 0.06 m and is a solid block, as shown in Fig. 11.1.

Fig. 11.1

(i) Calculate the volume of one brick.

volume of one brick = ………...................

(ii) The brick has a density of 2400 kg/m3.


Show that the mass of one brick is 2.88 kg.

(iii) What is the mass of the stack of bricks?

mass of stack = .................. kg


[6]

(b) Some other bricks have the same size and are made of the same material, but they
have a hollow in one face, as shown in Fig. 11.2.

Fig. 11.2

(i) How does the mass of one of these bricks compare with the mass of one of the
bricks in (a)(ii)?

...................................................................................................................................
0625/2/M/J/01 [Turn over
12 For
Examiner’s
Use
(ii) The hollow of one brick is filled level with wet cement, as shown in Fig. 11.3.

wet cement

Fig. 11.3

The brick now has a mass of 2.91 kg.

Compare this with the mass given in (a)(ii). What does it tell you about the density
of the wet cement?

...................................................................................................................................

...............................................................................................................................[3]

12 Fig. 12.1 shows a simplified diagram of the front of a cathode-ray oscilloscope (c.r.o.).
topicray
topictube
topiccathoderaytube
topiccathode

IGCSE OSCILLOSCOPE CO.

brightness focus

time-base y-gain
ms/cm volts/cm

x-shift y-shift
on
y input off

Fig. 12.1

(a) When the oscilloscope is switched on, a bright line is seen across the centre of the
screen.

(i) What causes the bright line?

...................................................................................................................................

...................................................................................................................................
0625/2/M/J/01
13 For
Examiner’s
Use
(ii) When the brightness control is turned up, the line gets brighter.

What happens inside the oscilloscope to cause this increase in brightness?

...................................................................................................................................

...............................................................................................................................[4]

(b) You have an alternating p.d. whose waveform you wish to display on the screen.

(i) Where would you connect this alternating p.d. to the oscilloscope? ........................

(ii) Fig. 12.2 shows what the trace on the screen might look like.

Fig. 12.2

1. Which oscilloscope control would you adjust to vary the amplitude, h, of the trace

on the screen? ..................

2. Which control would you adjust to vary the number of waves visible on the screen?

..................

3. What would you see on the screen if you switched the time-base setting to zero?

...................................................................................................................................
[4]

0625/2/M/J/01
Candidate
Centre Number Number

Candidate Name

International General Certificate of Secondary Education


UNIVERSITY OF CAMBRIDGE LOCAL EXAMINATIONS SYNDICATE

PHYSICS 0625/2
PAPER 2
OCTOBER/NOVEMBER SESSION 2001 1 hour

Candidates answer on the question paper.


No additional materials are required.

TIME 1 hour

INSTRUCTIONS TO CANDIDATES
Write your name, Centre number and candidate number in the spaces at the top of this page.
Answer all questions.
Write your answers in the spaces provided on the question paper.

INFORMATION FOR CANDIDATES


The number of marks is given in brackets [ ] at the end of each question or part question.
You may lose marks if you do not show your working or if you do not use appropriate units.
Take the weight of 1 kg to be 10 N (i.e. acceleration of free fall = 10 m/s2).

FOR EXAMINER’S USE

This question paper consists of 15 printed pages and 1 blank page.


SB (NH/CG) S10760/3
© UCLES 2001 [Turn over
2 For
Examiner’s
Use
1 A piece of string wraps around a cylinder 8 times.
topicmass
topicweight
topicdensity

string

cylinder

Fig. 1.1

Fig. 1.2 shows the string laid along a 30 cm rule.

string

0 5 10 15 20 25 cm 30

Fig. 1.2

(a) How long is the string?

length of string = ......................... cm [1]

(b) Calculate the circumference (distance once round) the cylinder.

circumference of cylinder = ......................... cm [2]

0625/2 O/N01
3 For
Examiner’s
Use
2 A trainee designer makes a jug for holding drinks. The jug is shown in Fig. 2.1.
topiceffect
topicturningeffect
topicturning

Fig. 2.1

Unfortunately, it is very easy to knock the jug over, so the trainee designer has to change the
shape of the jug.

State two things he could do to improve the stability of the jug.

1. .............................................................................................................................................

2. .........................................................................................................................................[2]

0625/2 O/N01 [Turn over


4 For
Examiner’s
Use
3 (a) How can a metal bar be made to expand in all directions?
topicmodels
topicmolecularmodels
topicmolecular

......................................................................................................................................[1]

(b) What happens to the spacing between the atoms in an iron bar when it expands?

.........................................................................................................................................[1]

(c) State one example where expansion of a solid can be useful.

.........................................................................................................................................[1]

(d) State one example where expansion of a solid causes problems which need to be
overcome.

.........................................................................................................................................[1]

0625/2 O/N01
5 For
Examiner’s
Use
4 (a) A ray of light passes through a rectangular glass block, as shown in Fig. 4.1. It emerges
topicevaporation
at point X.

ray

Fig. 4.1

On Fig. 4.1, draw the ray which emerges from the block at X. [2]

(b) The glass of which the block is made has a critical angle of 42°.

Another ray passes into the block as shown in Fig. 4.2.

ray

50°

Fig. 4.2

(i) On Fig. 4.2, show what happens to the ray at Y.

(ii) Why does this happen?

...................................................................................................................................

...............................................................................................................................[3]

(c) A third ray enters the block perpendicularly, as shown in Fig. 4.3.

ray

Fig. 4.3

On Fig. 4.3, draw the ray as it passes through the block and out into the air again. [2]

0625/2 O/N01 [Turn over


6 For
Examiner’s
Use
5 (a) Fig. 5.1 shows a student’s attempt at drawing a diagram to illustrate the formation of a
topicresistance
spectrum by using a glass prism.

screen

Sun’s
rays
red
orange
yellow
green
blue
spectrum
indigo
violet

Fig. 5.1

The student labelled the colours in the correct positions but made two mistakes in
showing the passage of the rays through the prism. What are these mistakes?

1. .......................................................................................................................................

2. ...................................................................................................................................[2]

(b) The prism in Fig. 5.1 can transmit infra-red radiation.

(i) What instrument could the student use to detect the presence of the infra-red
radiation?

...................................................................................................................................

(ii) Use the letters I.R. to show where the infra-red radiation would strike the screen.

...............................................................................................................................[2]

(c) Infra-red radiation is one example of an invisible part of the electromagnetic spectrum.

Give the names of two other invisible parts of the electromagnetic spectrum.

1. .......................................................................................................................................

2. ...................................................................................................................................[2]

0625/2 O/N01
7 For
Examiner’s
Use
6 (a) Three metal rods look the same.
topicbrownianmotion
topicbrownian

One is made of aluminium.


One is made of unmagnetised iron.
One is a permanent magnet.

Describe how a small bar magnet could show which rod is

(i) made of aluminium,

...................................................................................................................................

...................................................................................................................................

(ii) made of unmagnetised iron,

...................................................................................................................................

...................................................................................................................................

(iii) a permanent magnet.

...................................................................................................................................

...............................................................................................................................[4]

(b) In an experiment, a steel ball-bearing is held up by an electromagnet, as shown in


Fig. 6.1.

electromagnet

steel ball-bearing

Fig. 6.1

The electromagnet then releases the ball-bearing, which falls to the floor.

(i) From what metal is the core of the electromagnet made?

...................................................................................................................................

(ii) How is the electromagnet made to work?

...................................................................................................................................

(iii) What is done to make the electromagnet release the ball-bearing?

...............................................................................................................................[3]

0625/2 O/N01 [Turn over


8 For
Examiner’s
Use
7 This question is about the dangers of electricity.
topicforceonaconductorinamagneticfield
topicforce
topicconductor

(a) A builder tries to save money by not fitting fuses or circuit-breakers to the electric
circuits in a house. Why might this be dangerous?

..........................................................................................................................................

..........................................................................................................................................

......................................................................................................................................[2]

(b) Another builder only has switches like the one shown in Fig. 7.1.

Fig. 7.1

The builder decides it might be dangerous to fit one of these switches in a washroom,
where there would be water and steam.

Why might it be dangerous?

..........................................................................................................................................

..........................................................................................................................................

......................................................................................................................................[2]

(c) A cleaning operator is trained to check the cable of a vacuum cleaner for damage
before it is used.

Why might it be dangerous to use equipment with a damaged cable?

..........................................................................................................................................

......................................................................................................................................[1]

0625/2 O/N01
9 For
Examiner’s
Use
8 (a) Describe what happens to an atom when it undergoes radioactive decay.
topicradioactivity

..........................................................................................................................................

..........................................................................................................................................

......................................................................................................................................[3]

(b) A scientist measures the count rate from a radioactive substance over a period of 20
minutes. The table shows the results of his measurements.

time/minutes 0 5 10 15 20
count rate
800 400 205 105 50
counts/s

(i) From the table, estimate the half-life of the substance.

half-life = ......................... minutes

(ii) How many half-lives elapsed during the 20 minute experiment?

number of half-lives = ......................... minutes

(iii) If the scientist had taken readings for 25 minutes, what might the count rate have
been at the end of his experiment?

count rate after 25 minutes = ......................... counts/s


[3]

(c) In the following table, write in the correct numbers of electrons, protons and neutrons
contained in an α-particle and in a β-particle.

electrons protons neutrons

α-particle

β-particle
[4]

0625/2 O/N01 [Turn over


10 For
Examiner’s
Use
9 A student draws the circuit shown in Fig. 9.1. The circuit is intended to be used to measure
topicpressure
the resistance of R, a length of nichrome resistance wire.

X
R
Y

Fig. 9.1

(a) The student cannot remember which meters he should use at X and at Y.

(i) Which meter should he show at X? ..........................................................................

(ii) Which letter should he put in the circle at X? ............................................................

(iii) Which meter should he show at Y? ..........................................................................

(iv) Which letter should he put in the circle at Y? ............................................................


[4]

(b) (i) What is the component labelled Z? ...........................................................................

(ii) What is the purpose of Z in the circuit? .....................................................................


[3]

(c) The student connects the circuit and obtains the graph shown in Fig. 9.2.

4.0
p.d. / V
3.0

2.0

1.0

0
0 0.1 0.2 0.3 0.4 0.5 0.6 0.7 0.8
current / A

Fig. 9.2

0625/2 O/N01
11 For
Examiner’s
Use
(i) How can the student use the graph to find the resistance of R?

...................................................................................................................................

...................................................................................................................................

...................................................................................................................................

(ii) Calculate the value of R.

resistance R = ......................... [4]

(d) The student removes the length of nichrome wire R and replaces it with the same length
of thinner nichrome wire.

(i) Will the resistance of this wire be greater than, smaller than or the same as the wire
in the rest of the question?

...................................................................................................................................

(ii) On Fig. 9.2, sketch a line which the student might obtain when he repeats the
experiment with the thinner wire. [3]

0625/2 O/N01 [Turn over


12 For
Examiner’s
Use
10 The densities of three solids are
topictension

aluminium 2 700 kg/m3,


concrete 2 200 kg/m3,
wood 600 kg/m3,

Blocks having identical dimensions are made of these three substances. The blocks are
stood on a horizontal surface, as shown in Fig. 10.1.

alu
min
con wo
ium cre od
te

aluminium concrete wood

Fig. 10.1

(a) (i) Which block has the greatest mass? .........................................................................

How do you know this?..............................................................................................

...................................................................................................................................

(ii) Which block has the greatest weight? .......................................................................

(iii) Which block exerts the greatest pressure on the horizontal surface? .......................
[4]

(b) The wood block can be placed on the horizontal surface in any one of three ways, A, B
or C (see Fig. 10.2).

A B C
od
wo

wo
wood

od

Fig. 10.2

In which position does the wood block exert the greatest pressure?................................

How do you know this? .....................................................................................................

......................................................................................................................................[2]
0625/2 O/N01
13 For
Examiner’s
Use
(c) A person spends some time relaxing on a sandy beach. He sits on a chair with four
legs, as shown in Fig. 10.3.

sand

Fig. 10.3

Unfortunately, the chair quickly sinks into the sand.

(i) Why does this happen?

...................................................................................................................................

...................................................................................................................................

(ii) Suggest how the person might stop this happening.

...................................................................................................................................

...............................................................................................................................[3]

0625/2 O/N01 [Turn over


14 For
Examiner’s
Use
11 A laboratory thermometer is put into some heated pure water, as shown in Fig. 11.1. After
some time the water boils.
topicthermometer

thermometer

pure water

heat

Fig. 11.1

(a) What quantity does the thermometer measure? ...........................................................[1]

(b) Suggest what liquid might be contained by the thermometer. ......................................[1]

(c) The atmospheric pressure is normal.

What value should the thermometer show when the water is boiling? .........................[2]

(d) The heating is continued and the water carries on boiling.

What happens to the reading on the thermometer?

......................................................................................................................................[1]

(e) How could you check that the zero mark on the thermometer had been correctly
positioned? You may use a diagram if it helps you to answer clearly.

...............................................................

...............................................................

...............................................................

...............................................................

...............................................................

............................................................... [3]

0625/2 O/N01
15 For
Examiner’s
Use
12 Fig. 12.1 shows the plan of the dining room in a house in a cold country. Details of the rest of
topictube
topicray
topiccathoderaytube
topiccathode
the rooms are not shown. The air outside the house is much colder than that inside the house.

interior walls

COLD
WARM
plan (view
door from above)
WARM
external
walls dining room

window
(single glass)

Fig. 12.1

Heat is lost from the dining room by conduction at the rates shown below.

through external walls 3 000 000 J/hour


through internal walls 100 000 J/hour
through door 50 000 J/hour
through window 1 000 000 J/hour

(a) Suggest why

(i) much more heat is lost through the external walls than through the internal walls,

...................................................................................................................................

...................................................................................................................................

(ii) such a lot of heat is lost through the window.

...................................................................................................................................

...................................................................................................................................
[2]

(b) If the figures above relate to the only sources of heat loss, how many J/hour would the
heater in the dining room need to supply in order to keep the temperature in the room
constant?

rate of heat supply needed = ......................... J/hour [2]

(c) Suggest one other way heat might be lost from the room.

......................................................................................................................................[1]

0625/2 O/N01
Candidate
Centre Number Number

Candidate Name

International General Certificate of Secondary Education


CAMBRIDGE INTERNATIONAL EXAMINATIONS

PHYSICS 0625/2
PAPER 2
MAY/JUNE SESSION 2002
1 hour
Candidates answer on the question paper.
No additional materials required.

TIME 1 hour

INSTRUCTIONS TO CANDIDATES
Write your name, Centre number and candidate number in the spaces at the top of this page.
Answer all questions.
Write your answers in the spaces provided on the question paper.

INFORMATION FOR CANDIDATES


The number of marks is given in brackets [ ] at the end of each question or part question.
You may lose marks if you do not show your working or if you do not use appropriate units.
Take the weight of 1 kg to be 10 N (i.e. acceleration of free fall = 10 m/s2).

FOR EXAMINER’S USE

This question paper consists of 19 printed pages and 1 blank page.


SP (NF/JB) S22944/3
© CIE 2002 [Turn over
2 For
Examiner’s
Use
1 Here are two statements made by people who haven’t learned their Physics very well.
Each statement is incorrect. In the space alongside each statement, say what is wrong with
the statement.

statement what is wrong with this statement

“The weight of this bag


of peas is 1 kg.”

“The mass of this object


is another name for its
weight.”

[2]

0625/2/M/J/02
3 For
Examiner’s
Use
2 (a) Which of the following statements describe the property of a substance that would be
suitable for measuring temperature? Tick the box alongside any acceptable statement.

(i) a property that increases regularly with temperature

(ii) a property that decreases regularly with temperature

(iii) a property that remains constant as the temperature changes


[2]

(b) Fig. 2.1 shows how the length of the thread in a liquid-in-glass thermometer varies with
temperature.

20

length / cm

15

10

0
0 25 50 75 100
temperature / °C

Fig. 2.1

(i) What temperature is indicated by a thread length of 14.5 cm?

temperature = ........................ °C

(ii) What happens to the thread of the thermometer if the temperature drops below the
ice point?

...................................................................................................................................
[2]

0625/2/M/J/02 [Turn over


4 For
Examiner’s
Use
3 (a) Fig. 3.1 shows a machine for making loud sounds. It is called a siren. This consists of a
rotating disc with 25 holes. As each hole passes the jet, a puff of air passes through the
hole.

disc with
25 holes

jet of air

Fig. 3.1

(i) How many puffs of air will there be during one revolution of the disc?

number of puffs = ....................


(ii) The disc rotates 40 times per second.

Show that the frequency of the note produced by the siren is 1000 Hz.
[3]

0625/2/M/J/02
5 For
Examiner’s
Use
(b) The siren described in (a) is located some distance from a large building, as shown in
Fig. 3.2.

large
building
siren

Fig. 3.2

The siren is briefly sounded once. A short time later, the sound is heard again.

(i) Why is this second sound heard?

...................................................................................................................................

(ii) What is the frequency of this second sound? Tick one box.

less than 1000 Hz

1000 Hz

more than 1000 Hz

(iii) What is the amplitude of this second sound? Tick one box.

less than the original sound

the same as the original sound

more than the original sound


[3]

0625/2/M/J/02 [Turn over


6 For
Examiner’s
Use
4 (a) In the following table, tick the appropriate boxes to indicate where protons, neutrons and
electrons are found in an atom.

particle found in the nucleus found in an orbit

proton

neutron

electron
[3]

(b) A neutral atom of phosphorus (P) contains 15 protons, 16 neutrons and 15 electrons.

(i) Write down the proton number (atomic number) of phosphorus. ...............

(ii) Write down the nucleon number (mass number) of phosphorus. ..............

(iii) The nuclide notation for oxygen is 168O.

Write down the nuclide notation for phosphorus.


[3]

0625/2/M/J/02
7 For
Examiner’s
Use
5 (a) Describe one method of magnetising a steel rod.

..........................................................................................................................................

..........................................................................................................................................

......................................................................................................................................[2]

(b) How would you check that the steel rod had been magnetised?

what would show that the steel


what I would do rod had been magnetised

[2]
(c) A glass tube is supported vertically on a bench. A strong bar magnet A is placed in the
bottom of the tube, as shown in Fig. 5.1.

hand

N magnet B

glass tube

N
bench magnet A
S

Fig. 5.1
A second magnet B is now dropped into the tube.
Describe what happens to magnet B, and explain why.

what happens to magnet B explanation

[3]
0625/2/M/J/02 [Turn over
8 For
Examiner’s
Use
6 Fig. 6.1 shows a vacuum tube in which it is hoped to generate cathode rays.

vacuum
filament

anode y-plates

Fig. 6.1

(a) (i) On Fig. 6.1, draw the symbol for a battery connected so that thermionic emission
occurs.

(ii) What particles are emitted in thermionic emission? .................................................


[2]

(b)

– +

Fig. 6.2

(i) On Fig. 6.2, show the + and – terminals suitably connected so that cathode rays
may pass along the tube.
(ii) On Fig. 6.2, draw a line showing the path of the cathode rays.
(iii) How are the cathode rays detected in the tube?

...................................................................................................................................

...................................................................................................................................
[4]

0625/2/M/J/02
9 For
Examiner’s
Use
(c) Fig. 6.3 shows the same vacuum tube, with the terminals labelled.

P
Q

R S

Fig. 6.3

Between which two points should a battery be connected if the cathode rays are to be
deflected upwards?

between point ........ and point ........ [1]

0625/2/M/J/02 [Turn over


10 For
Examiner’s
Use
7 Fig. 7.1 illustrates one country’s system for transmitting electricity around the country, from
the power station to people’s houses.

voltage here
= voltage here voltage here
= =

power station step-up grid system step-down step-down house


transformer transformer transformer

Fig. 7.1

(a) At different points in this system, the voltage is 220 V, 11 000 V or 132 000 V. At the three
places marked on Fig. 7.1, write in the appropriate value of the voltage. [3]

(b) State one advantage of using high voltages for the transmission of electricity.

..........................................................................................................................................

......................................................................................................................................[1]

0625/2/M/J/02
11 For
Examiner’s
Use
8 In a research laboratory using radioactive materials, safety precautions have to be observed.
Some of the safety precautions adopted by the laboratory are listed below.

On the lines after each precaution, state reasons why it is a wise precaution.

(a) Radioactive materials should only be picked up using long-handled tools.

REASON 1 .......................................................................................................................

..........................................................................................................................................

REASON 2 .......................................................................................................................

......................................................................................................................................[2]

(b) Food must not be taken where radioactive materials are being used.

REASON ..........................................................................................................................

......................................................................................................................................[1]

(c) The researchers must wash their hands after the source has been put away safely.

REASON ..........................................................................................................................

......................................................................................................................................[1]

(d) Radioactive materials must be stored in a locked drawer or cabinet.

REASON ..........................................................................................................................

......................................................................................................................................[1]

0625/2/M/J/02 [Turn over


12 For
Examiner’s
Use
9 (a) Fig. 9.1 shows a circuit that includes three ammeters. The resistance of the ammeters
and the battery can be ignored.

A1 A3

20 X A2 20 X

Fig. 9.1

(i) What is the total resistance of the circuit?

resistance = ............ Ω

(ii) Ammeter A1 reads 0.2 A. What do the other two ammeters read?

A2 reads ............ A

A3 reads ............ A
[3]

0625/2/M/J/02
13 For
Examiner’s
Use
(b) Fig. 9.2 shows the same components as in (a), but connected differently.

A1 20 X A2 A3

20 X

Fig. 9.2

(i) What is the total resistance of the circuit? Tick one box.

40 Ω

30 Ω

20 Ω

10 Ω

(ii) Ammeter A1 reads 0.8 A. What do the other two ammeters read? Tick the
appropriate boxes.

more than 0.8 A 0.8 A less than 0.8 A

A2 reads

A3 reads

[3]

0625/2/M/J/02 [Turn over


14 For
Examiner’s
Use
10 (a) A manufacturer of car tyres estimates that the area of a car tyre in contact with the road
is about the same as the area of a person’s shoe in contact with the ground.

(i) A car weighs 10 000 N and a person weighs 500 N.

Calculate the ratio


pressure of car on ground
________________________ .
pressure of person on ground

Remember that the car has 4 tyres and a person has 2 feet.

(ii) Suggest why it might be a good idea to reduce the pressure of the air in car tyres if
the car is to be driven over soft sand or over snow.

...................................................................................................................................

...................................................................................................................................

...................................................................................................................................
[5]

(b) A U-tube manometer is used to measure lung pressure by blowing at A, as shown in


Fig. 10.1.

X Y
liquid

Fig. 10.1

(i) Before the person blows at A, the liquid levels X and Y are the same.

State the reason for this.

...................................................................................................................................

0625/2/M/J/02
15 For
Examiner’s
Use
(ii) Which way do the liquid levels move when the person blows at A?

level X moves ..........................

level Y moves ..........................

(iii) What would you measure in order to find the person’s lung pressure?

...................................................................................................................................

...................................................................................................................................
[4]

0625/2/M/J/02 [Turn over


16 For
Examiner’s
Use
11 In an experiment, different weights are hung on the end of a spring, and the length of the
spring is measured. The results are as follows.

weight/N 0 1 2 3 4 5 6 7

length/mm 40 48 60 64 72 80 88 96

extension/mm 0 8 20

(a) What is the length of the unstretched spring?

length of unstretched spring = ............ mm [1]

(b) Some of the extensions have been calculated for you.

Complete the table by writing in the remaining extensions. [2]

(c) (i) On the graph grid of Fig 11.1, plot the values from your table.

60
extension / mm

50

40

30

20

10

0
0 1 2 3 4 5 6 7
weight / N

Fig. 11.1

0625/2/M/J/02
17 For
Examiner’s
Use
(ii) Draw the best straight line through your points.
(iii) The experimenter has read one of the lengths incorrectly.

1. Which one is it? ...................................................................................................

2. What do you think the length reading should have been? ........... mm
[5]

(d) The spring is now attached to a block resting on a rough surface, as shown in Fig. 11.2.

block

rough pulling
surface spring force

friction
force

Fig. 11.2

As the pulling force is increased, the block just starts to move to the right when the
spring is 68 mm long.

(i) What is the extension of the spring when it is 68 mm long?

extension = .......... mm

(ii) Use your graph to find the force that causes this extension.

force = .......... N

(iii) What is the value of the friction force as the block starts to move?

friction force = .......... N


[4]

0625/2/M/J/02 [Turn over


18 For
Examiner’s
Use
12 (a) A beam of light is travelling parallel to the axis of a thin lens, as shown in Fig. 12.1.

X A P C F Y

Fig. 12.1

After passing through the lens, the rays all pass through the point F.

(i) Which word best describes what happens to the rays? Tick one box.

The rays converge

diffract

disperse

reflect

(ii) On Fig. 12.1, complete the paths of the two rays.


(iii) Complete the following sentence, referring to Fig. 12.1.

“The focal length of the lens is the distance between point ....... and point ...... .”
[3]

0625/2/M/J/02
19 For
Examiner’s
Use
(b) In this part of the question, you are required to draw an accurate ray diagram on
Fig. 12.2 for the lens in part (a).

object

X P F Y

Fig. 12.2

(i) From the top of the object, draw a ray that, after leaving the lens, passes through F.
(ii) From the top of the object, draw a ray that passes through P.
(iii) Mark in the image, and write “image” alongside it.
(iv) From your diagram, state two things that are the same (or approximately the same)
about the image and the object.

1. ...............................................................................................................................

2. ...............................................................................................................................

(v) From your diagram, state one thing that is definitely different about the image and
the object.

...................................................................................................................................
[9]

0625/2/M/J/02
Candidate
Centre Number Number

Candidate Name

International General Certificate of Secondary Education


CAMBRIDGE INTERNATIONAL EXAMINATIONS

PHYSICS 0625/2
PAPER 2
OCTOBER/NOVEMBER SESSION 2002
1 hour
Candidates answer on the question paper.
No additional materials are required.

TIME 1 hour

INSTRUCTIONS TO CANDIDATES
Write your name, Centre number and candidate number in the spaces at the top of this page.
Answer all questions.
Write your answers in the spaces provided on the question paper.

INFORMATION FOR CANDIDATES


The number of marks is given in brackets [ ] at the end of each question or part question.
You may lose marks if you do not show your working or if you do not use appropriate units.
Take the weight of 1 kg to be 10 N (i.e. acceleration of free fall = 10 m/s2).

FOR EXAMINER’S USE

This question paper consists of 17 printed pages and 3 blank pages.


SP (CW/SLC) S23398/3
© CIE 2002 [Turn over
2 For
Examiner’s
Use
1 A baggage handler at an airport lifts a suitcase from the ground and places it on a moving
belt. The moving belt then transfers the suitcase to the owner.

moving
belt

A suitcase on ground

B suitcase held
level with belt

C suitcase moving
on belt

Fig. 1.1

(a) (i) How does the total energy of the suitcase in B compare with its energy in A?
Tick one box.

The total energy is greater in B than in A.

The total energy is the same in B and in A.

The total energy is less in B than in A.

(ii) Explain your answer.

...................................................................................................................................

...................................................................................................................................
[2]

0625/2/O/N/02
3 For
Examiner’s
Use
(b) (i) How does the total energy of the suitcase in C compare with its energy in B? Tick
one box.

The total energy is greater in C than in B.

The total energy is the same in C and B.

The total energy is less in C than in B.

(ii) Explain your answer.

...................................................................................................................................

...................................................................................................................................
[2]

2 Here are five words relating to the transfer of thermal energy.

conductor, convection, evaporation, insulator, radiation

Complete each of the following sentences by writing in the appropriate word from the list.

(a) Cooking pots often have a wooden handle, because wood is a good thermal

...................................... . [1]

(b) Thermal energy reaches Earth from the Sun by means of ....................................... [1]

(c) Copper is a good ...................................... of thermal energy. [1]

(d) The heating element is put at the bottom of an electric kettle, so that

...................................... can rapidly transfer thermal energy throughout the water. [1]

0625/2/O/N/02 [Turn over


4 For
Examiner’s
Use
3 Fig. 3.1 shows a vertical wire through a horizontal piece of card. There is a current down the
wire.

large
current

card

Fig. 3.1

Fig. 3.2 shows the wire and card, viewed from above.

card

Fig. 3.2

The large circle is one of the magnetic field lines caused by the current.

On Fig. 3.2,

(a) show the direction of the magnetic field, [1]

(b) carefully draw three more magnetic field lines. [2]

0625/2/O/N/02
5 For
Examiner’s
Use
4 In an experiment to find the density of some oil, a student takes the following readings.

cm3
500
400 mass of empty measuring jug = 610 g
300
200
100

Fig. 4.1

cm3
500
oil 400 mass of jug containing 500 cm3 of oil = 1020 g
300
200
100

Fig. 4.2

(a) (i) Calculate the mass of oil in the jug.

mass of oil =........................ g

(ii) Calculate the density of the oil.

density of oil =......................................

(iii) How could the volume of the oil be more accurately measured than with the
measuring jug?

...................................................................................................................................
[7]

(b) Water is more dense than oil.

On Fig. 4.3, mark approximately where the surface of the same mass of water would be
if it replaced the oil. [1]

level
of oil cm3
500
400
300
200
100

Fig. 4.3

0625/2/O/N/02 [Turn over


6 For
Examiner’s
Use
5 (a) What happens to the nucleus of an atom that undergoes radioactive decay?

..........................................................................................................................................

......................................................................................................................................[1]

(b) Strontium-90 has a half-life of 28 years. How much time must pass before its activity
falls to  of its original value?

time = ........................ years [2]

6 (a) State what is meant by the melting point of a solid.

The melting point is ..........................................................................................................

......................................................................................................................................[2]

(b) Which two of the following quantities are the same? Tick two boxes.

boiling point of iron

freezing (solidifying) point of iron

melting point of iron [1]

0625/2/O/N/02
7 For
Examiner’s
Use
(c) Some liquid in a beaker is kept boiling by heating the beaker, as shown in Fig. 6.1.

boiling
liquid

heat

Fig. 6.1

(i) On the axes of Fig. 6.2, sketch a graph to show what happens to the temperature
of the liquid whilst it is boiling.

temperature

0
time
time at which
liquid starts
to boil

Fig. 6.2

(ii) On your graph, mark the boiling point of the liquid.


[2]

0625/2/O/N/02 [Turn over


8 For
Examiner’s
Use
7 A student holds a polythene rod in one hand and a dry cotton cloth in the other.

dry cotton polythene


cloth rod

Fig. 7.1

(a) How can the student cause the rod to become charged with static electricity?

......................................................................................................................................[1]

(b) How can the student detect that the rod has become charged?

......................................................................................................................................[1]

(c) Around the charged rod will be an electric field.

What is meant by an electric field ?

..........................................................................................................................................

......................................................................................................................................[2]

0625/2/O/N/02
9 For
Examiner’s
Use
(d) The charged polythene rod is brought close to another charged polythene rod that has
been suspended from a nylon thread, as shown in Fig. 7.2.

nylon charged
thread polythene
rod (–)

charged
polythene
rod (–)

Fig. 7.2

(i) What is seen to happen to the suspended rod?

...................................................................................................................................

(ii) Explain why this happens.

...................................................................................................................................
[2]

(e) If the student used a copper rod in (a), instead of the polythene rod, why would he not
be able to charge the rod?

..........................................................................................................................................

......................................................................................................................................[1]

0625/2/O/N/02 [Turn over


10 For
Examiner’s
Use
8 Fig. 8.1 shows an electric circuit containing a battery, a 4.7 Ω resistor, an ammeter and a
variable resistor with a sliding contact. The variable resistor is set at zero. The ammeter and
battery have such a small resistance that it can be ignored.

X
sliding contact
variable
resistor A
Y

4.7 Ω

Fig. 8.1

(a) In what unit do we measure the e.m.f. of the battery? .................................................[1]

(b) Write down the equation that links resistance, potential difference (p.d.) and current.

[2]

(c) The ammeter shows that the current through the 4.7 Ω resistor is 0.5 A.

Calculate the p.d. across the resistor.

p.d. across resistor = ........................ [2]

(d) The sliding contact of the variable resistor is moved from X to Y.

(i) What happens to the resistance of the variable resistor?

...................................................................................................................................

(ii) What happens to the reading on the ammeter?

...................................................................................................................................
[2]

(e) The variable resistor is now adjusted to make the total resistance of the circuit 10.0 Ω.
What is the resistance of the variable resistor now?

resistance of variable resistor = ........................ Ω [2]

0625/2/O/N/02
11 For
Examiner’s
Use
9 A student wrote the following report about an experiment to measure the speed of sound in
air.

My friend and I went into a field a long way from any buildings and measured
the distance across the field. My friend stood at one side of the field and I
stood at the other. My friend banged two pieces of wood together, and as I
saw him do this, I started a stopwatch. I stopped the stopwatch when I heard
the sound. We obtained the following readings.

distance across field = 238 m


time for sound to cross field = 0.7 s

(a) Why was it necessary to be a long way from any buildings?

......................................................................................................................................[1]

(b) Why was it necessary to use such a large distance?

......................................................................................................................................[1]

(c) Suggest what the students could have used to measure the distance across the field.

......................................................................................................................................[1]

(d) Why was there a delay between the student seeing the pieces of wood come together
and hearing the bang?

..........................................................................................................................................

......................................................................................................................................[2]

(e) Use the readings to calculate the speed of sound.

speed of sound = ...................................... [4]

(f) Suggest why it might have been a good idea for the students to repeat the experiment
after they had exchanged positions.

..........................................................................................................................................

......................................................................................................................................[1]

0625/2/O/N/02 [Turn over


12 For
Examiner’s
Use
10 (a) The apparatus shown in Fig. 10.1 can be used to indicate when there is a force on the
copper rod.

hanging copper rod

N
S

mercury

Fig. 10.1

(i) Suggest what is seen to happen to the hanging copper rod when the switch is
closed.

...................................................................................................................................

...................................................................................................................................

(ii) Explain your answer.

...................................................................................................................................

...................................................................................................................................

(iii) The cell is reversed and the switch closed.

How does what is seen now differ from what you described in (a)(i)?

...................................................................................................................................
[4]

0625/2/O/N/02
13 For
Examiner’s
Use
(b) Fig. 10.2 represents a d.c. motor.

coil

S
N

Fig. 10.2

(i) In the boxes, label the various parts of the motor. One part has been labelled as an
example.
(ii) Which part of the motor ensures that the coil keeps rotating when the battery is
connected?

...................................................................................................................................

(iii) The battery is reversed. What difference does this make to the motor?

...................................................................................................................................
[5]

0625/2/O/N/02 [Turn over


14 For
Examiner’s
Use
11 (a) Fig. 11.1 shows a reed relay in a simple circuit.

flexible
lamp iron reeds

Fig. 11.1

Explain why the iron reeds touch each other when switch S is closed.

..........................................................................................................................................

..........................................................................................................................................

......................................................................................................................................[4]

0625/2/O/N/02
15 For
Examiner’s
Use
(b) Fig. 11.2 shows a simple temperature-operated alarm.

warning
lamp

thermistor

Fig. 11.2

When the thermistor is cold, its resistance is too high to allow sufficient current to
operate the reed relay. The resistance of the thermistor decreases as the temperature
increases.

Describe what happens to make the warning lamp light as the air temperature changes.

..........................................................................................................................................

..........................................................................................................................................

..........................................................................................................................................

..........................................................................................................................................

......................................................................................................................................[4]

0625/2/O/N/02 [Turn over


16 For
Examiner’s
Use
12 (a) A narrow beam of red light strikes one face of a triangular prism at A, as shown in
Fig. 12.1.

screen

red light

Fig. 12.1

(i) On Fig. 12.1, show the path of the beam until it reaches the screen.
(ii) What name do we use to describe what happens to the beam at A?

...................................................................................................................................
[4]

0625/2/O/N/02
17 For
Examiner’s
Use
(b) The red light is replaced by a narrow beam of mixed red and violet light, as shown in
Fig. 12.2.

screen

red +
violet light

Fig. 12.2

Complete Fig. 12.2 to show what happens to the red and the violet light. [2]

(c) The beam of red and violet light is replaced by a narrow beam of white light from the
Sun. State what is now seen on the screen.

......................................................................................................................................[1]

(d) The rays from the Sun also include infra-red rays. Infra-red radiation can pass through
glass.

(i) On Fig. 12.2, mark with the letter X the place where infra-red radiation from the
Sun might strike the screen after passing through the prism.
(ii) Why can infra-red not be seen on the screen?

...................................................................................................................................

(iii) What could be used to detect the infra-red radiation?

...................................................................................................................................
[3]

0625/2/O/N/02
Centre Number Candidate Number Name

CAMBRIDGE INTERNATIONAL EXAMINATIONS


International General Certificate of Secondary Education
PHYSICS 0625/02
Paper 2
May/June 2003

1 hour
Candidates answer on the Question Paper.
No Additional Materials are required.

READ THESE INSTRUCTIONS FIRST

Write your Centre number, candidate number and name on all the work you hand in.
Write in dark blue or black pen in the spaces provided on the Question Paper.
You may use a soft pencil for any diagrams, graphs or rough working.
Do not use staples, paper clips, highlighters, glue or correction fluid.

Answer all questions.


At the end of the examination, fasten all your work securely together.
The number of marks is given in brackets [ ] at the end of each question or part question.
You may lose marks if you do not show your working or if you do not use appropriate units.
Take the weight of 1 kg to be 10 N (i.e. acceleration of free fall = 10 m/s2).

For Examiner’s Use

If you have been given a label, look at the


details. If any details are incorrect or
missing, please fill in your correct details
in the space given at the top of this page.

Stick your personal label here, if


provided.

This document consists of 16 printed pages.


SP (AT/KN) S46409/2
© CIE 2003 [Turn over
2 For
Examiner’s
Use
1 A person winds some thread tightly 4 times round the length of a metre rule and cuts the
ends off level with the left-hand end of the rule, as shown in Fig. 1.1.

ends cut
off here
thread 1 m rule

Fig. 1.1

(a) To the nearest metre, what is the length of the thread?

.................. m [1]

(b) Is the actual length of thread slightly greater or slightly less than your answer to (a)?
Tick one box and give your reason.

slightly greater slightly less

reason .......................................................................................................................

...............................................................................................................................[1]

0625/2/M/J/03
3 For
Examiner’s
Use
2 (a) Two horizontal strings are attached to a soft rubber ball, as shown in Fig. 2.1.

10 N F

Fig. 2.1

A force of 10 N pulls on one string.

(i) The ball does not move. What is the value of the force F on the other string?

F = .............................. N

(ii) What change to the rubber ball do the two forces cause?

...................................................................................................................................
[2]

(b) A garden pot containing soil weighs a total of 360 N. The pot rests on three equally-
spaced blocks, so that surplus water can drain out of the holes in the base of the pot.
The soil is uniformly distributed in the pot. The pot is shown in Fig. 2.2.

Fig. 2.2

(i) What is the force exerted by each block on the pot? ...............N
(ii) State the direction of these forces.
...................................................................................................................................

(iii) The gardener finds that the blocks sink into the ground, but he must have the pot
up on blocks to allow the drainage. What can he do to reduce the sinking of the
pot?

...................................................................................................................................
[3]

0625/2/M/J/03 [Turn over


4 For
Examiner’s
Use
3 (a) An unopened bottle of olive oil has
a mass of 0.97 kg. The empty bottle
has a mass of 0.51 kg.
Calculate the mass of the olive oil.
OLI V OLI V
OIL OIL

0.97 kg 0.51 kg

Fig. 3.1

mass of olive oil = .................................. kg [2]

(b) The olive oil is poured into three 250 cm3 measuring cylinders. The first two cylinders
are filled to the 250 cm3 mark. The third is shown in Fig. 3.2.

cm3
250

200
50
150

100

50

Fig. 3.2

(i) What is the volume of the olive oil in the third measuring cylinder?

volume = .................................. cm3

(ii) Calculate the volume of the olive oil in the unopened bottle.

volume = .................................. cm3

(iii) Calculate the density of the olive oil. Express your answer to 2 significant figures.

density = ..................................
[7]

0625/2/M/J/03
5 For
Examiner’s
Use
4 The air trapped in a cylinder by a piston is kept under pressure by a load, as shown in
Fig. 4.1.

fixed
pivot
load

piston

cylinder

air

Fig. 4.1

(a) Describe how the pressure in the cylinder is caused by the air molecules.

..........................................................................................................................................

..........................................................................................................................................

......................................................................................................................................[3]

(b) The load is increased.

(i) State what happens to the piston.

...................................................................................................................................

(ii) State what happens to the pressure in the cylinder, and give a reason.

what happens ...........................................................................................................

...................................................................................................................................

reason .......................................................................................................................

...................................................................................................................................
[3]

0625/2/M/J/03 [Turn over


6 For
Examiner’s
Use
5 An immersion heater is put into some crushed ice at 0 °C. The immersion heater is switched
on.

insulating immersion heater


beaker
crushed ice

Fig. 5.1

(a) On Fig. 5.2, sketch the graph of temperature against time, up to the time when all the
ice has melted. [3]

100
temperature / °C

0 time

time when
all ice has
melted

Fig. 5.2

(b) The heater is left switched on after all the ice has melted, and the temperature rises.
After some time, the temperature stops rising, even though the heater is still on.

(i) Suggest why the temperature stops rising, even though the heater is still on.

...................................................................................................................................

...................................................................................................................................

(ii) State what happens to the energy received by the water whilst this is happening.

...................................................................................................................................

...................................................................................................................................
[2]

0625/2/M/J/03
7 For
Examiner’s
Use
6 In this question, drawing should be done carefully.

Fig. 6.1 shows a ray of light striking mirror 1 at point X.

ray of
light mirror 2

mirror 1 X

Fig. 6.1

(a) On Fig. 6.1,

(i) draw the normal at X,


(ii) draw the ray reflected from mirror 1,
(iii) mark the angle of incidence using the letter i and the angle of reflection using the
letter r.
[3]

(b) Mirror 2 is parallel to mirror 1. The reflected ray from mirror 1 strikes mirror 2.

Compare the direction of the ray reflected from mirror 2 with the incident ray at X. You
may do a further construction if you wish. Complete the sentence below.

The reflected ray from mirror 2 is .....................................................................................

......................................................................................................................................[1]

0625/2/M/J/03 [Turn over


8 For
Examiner’s
Use
7 The speed of sound in air is 340 m/s.

(a) Complete Fig. 7.1 to show how far a sound wave has travelled 2, 3, 4 and 5 seconds
after the sound was made. [1]

time elapsed/s 0 1 2 3 4 5

distance travelled/m 0 340

Fig. 7.1
(b) On Fig. 7.2, draw the graph of distance travelled against time for the sound wave. [3]

distance
travelled
/m
2000

1500

1000

500

0
0 1 2 3 4 5

time/s

Fig. 7.2

0625/2/M/J/03
9 For
Examiner’s
Use
(c) A ship is sinking in the dark as shown in Fig. 7.3.

distress
flare

lifeboat

sinking ship

Fig. 7.3

The sailors on the ship fire a distress flare into the air. It explodes with a bang and a
bright flash of light.

(i) A lifeboat crew hear the bang and see the flash, but not at the same time.
State which reaches the lifeboat first, the bang or the flash, and give a reason.

...................................................................................................................................

...................................................................................................................................

...................................................................................................................................

(ii) The time interval in (c)(i) is 4.2 s. Use your graph in (b) to find how far away the
lifeboat is from the flare. Show clearly on your graph how you got your answer.

distance of lifeboat = .................................. m


[6]

0625/2/M/J/03 [Turn over


10 For
Examiner’s
Use
8 (a) In an electronic circuit, what is a capacitor designed to store? ................................ [1]

(b) The circuit in Fig. 8.1 contains a large-value resistor and a capacitor.

S1
large-value
resistor

6 V d.c.

V capacitor –
S2

Fig. 8.1

(i) Switch S1 is open. Switch S2 is closed and then opened again.

What reading now shows on the voltmeter? ................................ V

(ii) S2 is left open and S1 is closed and left closed.

Describe what happens to the reading on the voltmeter.

...................................................................................................................................

...................................................................................................................................

(iii) The circuit in Fig. 8.1 is an example of a simple time-delay circuit.

State one use of a time-delay circuit.

...................................................................................................................................

...................................................................................................................................
[4]

0625/2/M/J/03
11 For
Examiner’s
Use
9 A length of flexible, slack wire is fixed at A and B so that part of it is held vertically
in the field of a horseshoe magnet, as shown in Fig. 9.1.

S
N

Fig. 9.1

Figs. 9.2 and 9.3 each show the same section through the apparatus. The wire between A
and B is not shown.

A A

magnet magnet
pole pole

B B

Fig. 9.2 Fig. 9.3

(a) (i) On Fig. 9.2, draw what the wire might look like when a large current passes
through it. [2]
(ii) Explain why the wire looks like this.

...................................................................................................................................

...............................................................................................................................[3]

(b) On Fig. 9.3, draw what the wire might look like if the current in (a) is reversed. [1]

0625/2/M/J/03 [Turn over


12 For
Examiner’s
Use
10 Fig. 10.1 shows a simplified diagram of the front of a cathode-ray oscilloscope (c.r.o.).

IGCSE OSCILLOSCOPE CO.

brightness focus

time-base y-gain
ms/cm volts/cm

x-shift y-shift
on
y input off

Fig. 10.1

(a) When the oscilloscope is switched on, a bright spot is seen at the centre of the screen.

(i) Describe what causes this bright spot.

...................................................................................................................................

...................................................................................................................................

...................................................................................................................................

...............................................................................................................................[3]

(ii) The spot is rather blurred. Which control should be adjusted to make it sharper?

...............................................................................................................................[1]

(iii) Which control would be switched on to turn the spot into a horizontal line?

...............................................................................................................................[1]

(iv) Describe what happens inside the oscilloscope to turn the spot into a horizontal
line.

...................................................................................................................................

...................................................................................................................................

...................................................................................................................................

...............................................................................................................................[3]

0625/2/M/J/03
13 For
Examiner’s
Use
(b) You have an alternating p.d. whose waveform you wish to display on the screen.

(i) Where would you connect this alternating p.d. to the oscilloscope?

...................................................................................................................................

(ii) Fig. 10.2 shows what the trace on the screen might look like.

Fig. 10.2

1. What change would you see on the screen if you adjusted the x-shift control?

...................................................................................................................................

2. What change would you see on the screen if you adjusted the y-shift control?

...................................................................................................................................
[3]

0625/2/M/J/03 [Turn over


14 For
Examiner’s
Use
11 (a) Fig. 11.1 shows a circuit containing a lamp and a variable resistor.

Fig. 11.1

The circuit does not work. The lamp does not light and altering the setting on the
variable resistor makes no difference.

In the space below, re-draw the diagram, showing a circuit in which the variable resistor
may be used to change the brightness of the lamp. [2]

0625/2/M/J/03
15 For
Examiner’s
Use
(b) Fig. 11.2 shows two resistors and an ammeter connected in series to a 6 V d.c. supply.
The resistance of the ammeter is so small that it can be ignored.

8Ω
Q
P
+
6V A

R
S
4Ω

Fig. 11.2

(i) Calculate the combined resistance of the 8 Ω and 4 Ω resistors in series.

combined resistance = .................................. Ω [2]

(ii) 1. Calculate the current supplied by the 6 V d.c. supply.

current = ..................................

2. State the value of the current

in section PQ of the circuit ..................................

recorded by the ammeter ..................................

in section SR of the circuit ..................................


[5]

(iii) On Fig. 11.2, show a voltmeter connected to measure the potential difference
across the 4 Ω resistor. [1]

0625/2/M/J/03 [Turn over


16 For
Examiner’s
Use
12 A rock climber climbs up a rock face, as shown in Fig. 12.1.

safety rope

climber

Fig. 12.1

(a) To climb the rock face, the climber must do work.


Which force must the climber work against as he climbs? Tick one box.

air resistance

friction on the rock

his weight

tension in the safety rope [1]

(b) What other quantity, as well as the force ticked in (a), must be known in order to find the
work done by the climber?

......................................................................................................................................[1]

(c) One climber weighs 1000 N and another weighs 800 N. They both take the same time to
climb the cliff.

(i) Which one has done the most work? ........................................................................

(ii) Which one has the greater power rating? .................................................................


[2]

(d) When the first climber reaches the top, he has more gravitational potential
energy than he had at the bottom.

(i) What form of energy, stored in his body, was used to give him this extra

gravitational potential energy? ..................................................................................

(ii) Where did he get this energy from? ..........................................................................

(iii) Other than increasing gravitational potential energy on the way up, how else was
energy in his body used? State one way.

...................................................................................................................................

...................................................................................................................................
[3]
0625/2/M/J/03
Centre Number Candidate Number Name

CAMBRIDGE INTERNATIONAL EXAMINATIONS


International General Certificate of Secondary Education

PHYSICS 0625/02
Paper 2 Theory
October/November 2003

1 hour
Candidates answer on the Question Paper.
No Additional Materials are required.

READ THESE INSTRUCTIONS FIRST

Write your Centre number, candidate number and name on all the work you hand in.
Write in dark blue or black pen in the spaces provided on the Question Paper.
You may use a soft pencil for any diagrams, graphs or rough working.
Do not use staples, paper clips, highlighters, glue or correction fluid.

Answer all questions.


At the end of the examination, fasten all your work securely together.
The number of marks is given in brackets [ ] at the end of each question or part question.
You may lose marks if you do not show your working or if you do not use appropriate units.
Take the weight of 1 kg to be 10 N (i.e. acceleration of free fall = 10 m / s2).

If you have been given a label, look at the For Examiner’s Use
details. If any details are incorrect or
missing, please fill in your correct details
in the space given at the top of this page.

Stick your personal label here, if


provided.

This document consists of 14 printed pages and 2 blank pages.


MML 4505 11/02 S53489/2 UNIVERSITY of CAMBRIDGE
© UCLES 2003 Local Examinations Syndicate [Turn over
2 For
Examiner’s
Use
1 (a) The block of wood in Fig. 1.1 will balance on a horizontal table.

Q R
P
S

B
A

Fig. 1.1

If the block is tilted slightly clockwise about AB, it returns to its original position when
released.
(i) On Fig. 1.1, mark with the letter G where the centre of mass of the block might be.
(ii) Small masses are added to the top PQRS of the block until it is just about to
topple (fall over). Fig. 1.2 shows a side view of the block.

P S

Fig. 1.2

On Fig. 1.2, draw a line along which the centre of mass of the arrangement must
now lie.
[2]

0625/02/O/N/03
3 For
Examiner’s
Use
(b) Fig. 1.3 shows two ways of using a laboratory retort stand.

A B

Fig. 1.3

Which diagram shows the safer arrangement? .....................

Give a reason for your choice.

..........................................................................................................................................

.................................................................................................................................... [2]

(c) Some office filing cabinets have a mechanism that allows only one drawer to be
opened at a time, as shown in Fig. 1.4.

Fig. 1.4

Suggest why they have this mechanism.

..........................................................................................................................................

.................................................................................................................................... [1]

0625/02/O/N/03 [Turn over


4 For
Examiner’s
Use
2 A wheel is rotating at approximately 2 revolutions per second.

Describe how you would use a stopwatch to measure as accurately as possible the time for
one revolution of the wheel. Make sure you include all the relevant information.

.................................................................................................................................................

.................................................................................................................................................

.................................................................................................................................................

.................................................................................................................................................

........................................................................................................................................... [5]

3 Fig. 3.1 shows a rock-fall down a mountain-side.

loose
rocks

mountain

Fig. 3.1

The rocks higher up the mountain were disturbed by something and they roll down the
mountain until they stop at the bottom. In the boxes below, write the name of the type of
energy being described.

Before they fall, the rocks have this


energy because of their position.

As the rocks are falling,


their energy is changing
to these other types of energy.

At the bottom, the only energy


retained by the rocks is this type
of energy.
[5]

0625/02/O/N/03
5 For
Examiner’s
Use
4 (a) A farmer has two vehicles with the same weight and the same number of wheels.
Fig. 4.1 shows what the wheels on these two vehicles look like.

vehicle 1 vehicle 2

Fig. 4.1

Which vehicle should the farmer use when driving across his fields when the ground is
very soft? Give your reasons.

vehicle ................................................................

reasons ............................................................................................................................

..........................................................................................................................................

.................................................................................................................................... [4]

(b) (i) If you stepped on the point of a sharp nail with your bare foot, it would be
extremely painful. Explain, in terms of pressure, why this is so.

..................................................................................................................................

..................................................................................................................................
(ii) A person can lie on a bed of nail-points if there is a large number of nails. Explain
why this is not extremely painful.

..................................................................................................................................

..................................................................................................................................
[5]

0625/02/O/N/03 [Turn over


6 For
Examiner’s
Use
5 Fig. 5.1 shows two parallel rays of light, AB and CD, in a symmetrical glass block.

glass block

A B

C D

Fig. 5.1

(a) On Fig. 5.1,


(i) draw the ray AB before it entered the block at A,
(ii) draw the normal at B,
(iii) draw a ray which could emerge into the air at B,
(iv) draw the normal at D,
(v) draw a ray which could emerge into the air at D.
[5]

(b) State two things about the directions of the two rays emerging from the block.

1. ......................................................................................................................................

2. ................................................................................................................................ [2]

(c) What might happen to a third ray in the block, parallel to the others, which reached the
point P?

.................................................................................................................................... [1]

0625/02/O/N/03
7 For
Examiner’s
Use
6 (a) Here are three quantities that are associated with waves in the electromagnetic spectrum.

speed, wavelength, frequency


(i) Which of these is the same in a vacuum for both X-rays and radio waves?

..................................................................................................................................
(ii) Which of these determines the colour of a ray of light?

..................................................................................................................................
(iii) Which of these is the distance between two successive wavecrests?

..................................................................................................................................
[3]

(b) Which region of the electromagnetic spectrum has the shortest wavelength?

.................................................................................................................................... [1]

0625/02/O/N/03 [Turn over


8 For
Examiner’s
Use
7 (a) The resistance of a piece of wire of constant diameter depends on its length.

On the axes of Fig. 7.1, sketch a graph to show how the resistance of the wire varies
with length. [2]

resistance

0
0 length

Fig. 7.1

(b) A reel contains 50 m of insulated wire. The ends of the wire are connected to the
circuit shown in Fig. 7.2, so that the resistance of the wire can be found.

+
50 m
meter 1


meter 2

Fig. 7.2

(i) What sort of meter is meter 1? ........................................................................... [1]

(ii) Complete the sentence: “Meter 1 is measuring the .................................................


across the coil of wire.” [1]

(iii) What sort of meter is meter 2? ........................................................................... [1]

(iv) Complete the sentence: “Meter 2 is measuring the .................................................


in the coil of wire.” [1]

0625/02/O/N/03
9 For
Examiner’s
Use
(v) The dials of the meters are shown in Figs. 7.3(a) and 7.3(b).

5 6 0.2 0.3
3 4 7
8 0.4
2
9 0.1
1

0.5
10
0

0
V A

Fig. 7.3(a) Fig. 7.3(b)

From the readings, calculate the resistance of the coil of wire.

resistance of coil = ................................ [5]


(vi) What is the resistance per metre of the wire?

resistance per metre = .......................... [2]

0625/02/O/N/03 [Turn over


10 For
Examiner’s
Use
8 Fig. 8.1 shows the view, from above, of a sheet of cardboard on top of a bar magnet. The
dotted line is the outline of the bar magnet.

Fig. 8.1

(a) Describe how the pattern of the magnetic field round the bar magnet can be shown
experimentally. If you wish, you may draw on Fig. 8.1 as well as write on the lines
below.

..........................................................................................................................................

..........................................................................................................................................

..........................................................................................................................................

..........................................................................................................................................

..........................................................................................................................................

..........................................................................................................................................

..........................................................................................................................................

.................................................................................................................................... [4]

(b) State how you would identify the north pole of the magnet.

..........................................................................................................................................

.................................................................................................................................... [1]

0625/02/O/N/03
11 For
Examiner’s
Use
9 (a) When a nucleus decays by emitting an -particle, what happens to
(i) the number of neutrons in the nucleus,

..................................................................................................................................
(ii) the number of protons in the nucleus,

..................................................................................................................................
(iii) the charge on the nucleus?

..................................................................................................................................
[5]

(b) On 1st January 1900, a sample of a particular radioactive nuclide had an activity of
3200 count / min. The nuclide has a half-life of 22 years. Calculate the activity of the
nuclide remaining in the sample on 1st January 1966.

activity = ......................... count / min [4]

0625/02/O/N/03 [Turn over


12 For
Examiner’s
Use
10 In a spring-stretching experiment, the following values were found for the extension of the
spring.

load / N 0 1 2 3 4 5 6 7

extension / mm 0 16 45 48 64 80 104 150

(a) Plot these values on the axes of Fig. 10.1. [3]

200
extension / mm

150

100

50

0
0 2 4 6 8 10
load / N

Fig. 10.1

(b) A mistake was made with measuring one of the extensions. On the table above, put a
circle around the incorrect extension. [1]

(c) After a load of about 5 N, the graph begins to curve upwards.

Suggest what is happening to the spring when the load is greater than 5 N.

..........................................................................................................................................

.................................................................................................................................... [1]

(d) Describe how you would measure the extensions if you were doing this experiment.

..........................................................................................................................................

..........................................................................................................................................

..........................................................................................................................................

..........................................................................................................................................

..........................................................................................................................................

.................................................................................................................................... [3]
0625/02/O/N/03
13 For
Examiner’s
Use
11 Fig. 11.1 shows a mercury-in-glass thermometer, calibrated in °C.

–10 0 10 20 30 40 50 60 70 80 90 100 110

ºC

Fig. 11.1

(a) (i) What temperature would the thermometer read if put in steam, just above boiling
water at standard pressure?

........................... °C
(ii) What temperature would the thermometer read if put in pure melting ice?

........................... °C
(iii) On Fig. 11.1, mark where the end of the mercury thread might be when the
thermometer is in a freezer, where the temperature is well below the freezing
point of water.
[3]

(b) The mercury-in-glass thermometer uses the expansion of a liquid to measure


temperature. State another physical property that can be used to measure temperature.

.................................................................................................................................... [1]

0625/02/O/N/03 [Turn over


14 For
Examiner’s
Use
12 In a country where the mains electricity supply is 240 V, the transformer in Fig. 12.1 is used
to enable a 6 V lamp to be lit.

A
240 V 8000 6V
mains turns lamp

Fig. 12.1

(a) Calculate the number of turns needed on the secondary coil if the lamp is to be lit at
normal brightness.

number of turns = ........................ [3]

(b) Without further calculation, state the number of secondary turns needed to light, at
normal brightness,
A
(i) two identical 6 V lamps in parallel i.e.

number of turns = ....................


B

A
(ii) two identical 6 V lamps in series i.e.

number of turns = ....................


B

[2]

0625/02/O/N/03
Centre Number Candidate Number Name

UNIVERSITY OF CAMBRIDGE INTERNATIONAL EXAMINATIONS


International General Certificate of Secondary Education
PHYSICS 0625/02
Paper 2
May/June 2004
1 hour 15 minutes
Candidates answer on the Question Paper.
No Additional Materials are required.

READ THESE INSTRUCTIONS FIRST

Write your Centre number, candidate number and name on all the work you hand in.
Write in dark blue or black pen in the spaces provided on the Question Paper.
You may use a soft pencil for any diagrams, graphs or rough working.
Do not use staples, paper clips, highlighters, glue or correction fluid.

Answer all questions.


At the end of the examination, fasten all your work securely together.
The number of marks is given in brackets [ ] at the end of each question or part question.
You may lose marks if you do not show your working or if you do not use appropriate units.
Take the weight of 1 kg to be 10 N (i.e. acceleration of free fall = 10 m/s2).

For Examiner’s Use

If you have been given a label, look at the


details. If any details are incorrect or
missing, please fill in your correct details
in the space given at the top of this page.

Stick your personal label here, if


provided.

This document consists of 17 printed pages and 3 blank pages.


SPA (NH/BI) S61205/2
© UCLES 2004 [Turn over
2 For
Examiner’s
Use
1 The clock on a public building has a bell that strikes each hour so that people who cannot
see the clock can know what hour of the day it is.
At precisely 6 o’clock, the clock starts to strike. It strikes 6 times.
At the first strike of the bell, a man’s wrist-watch is as shown in Fig. 1.1.

12
11 1
10 2

9 3

8 4
7 5
6

Fig. 1.1

When the bell strikes for the sixth time, the wrist-watch is as shown in Fig. 1.2.

12
11 1
10 2

9 3

8 4
7 5
6

Fig. 1.2

(a) Calculate the time interval between the 1st strike and the 6th strike.

time interval = ……………… s [1]

(b) Calculate the time interval between one strike and the next.

time interval = ………………s [2]

(c) At precisely 11 o’clock, the clock starts to strike.


Calculate the time interval between the 1st strike and the 11th strike.

time interval = ………………s [2]


© UCLES 2004 0625/02 M/J/04
3 For
Examiner’s
Use
2 Fig. 2.1 shows a hinged rail in a fence. The rail has to be lifted vertically in order to let people
through.

hinge rail

Fig. 2.1

(a) On Fig. 2.1, draw an arrow to show the position and direction of the smallest force that
would be needed to begin to raise the rail. [3]

(b) What is the correct Physics term for the turning effect of a force?

Tick one box.

force

work

moment

movement [1]

(c) Suggest one way the designer of the fence could have reduced the force needed to lift
the rail.

..........................................................................................................................................

......................................................................................................................................[1]

© UCLES 2004 0625/02 M/J/04 [Turn over


4 For
Examiner’s
Use
3 Fig. 3.1 shows the speed-time graph of part of a short journey made by a cyclist.

25

speed P Q
20
m/s
15

10

5
R
0
0 10 20 30 40 50 60 70 80 90 100
time/s

Fig. 3.1

(a) Which part of Fig. 3.1 shows when the cyclist is travelling at constant speed?

......................................................................................................................................[1]

(b) State what is happening during the rest of the journey shown in Fig. 3.1.

......................................................................................................................................[1]

(c) (i) Calculate the distance travelled during the first 50 s.

distance travelled = …………………... m

(ii) Calculate the distance travelled between 50 s and 100 s.

distance travelled = …………………... m

© UCLES 2004 0625/02 M/J/04


5 For
Examiner’s
Use
(iii) Calculate the total distance travelled.

total distance travelled = …………………... m

(iv) Calculate the average speed during the 100 s.

average speed = …………………... m/s


[8]

© UCLES 2004 0625/02 M/J/04 [Turn over


6 For
Examiner’s
Use
4 (a) Fig. 4.1 shows a person pulling a loaded barrow along a path from A to B at a steady
speed.

A B

Fig. 4.1

State the two quantities you need to know in order to be able to calculate the work done
by the person.

1. ......................................................................................................................................

2. ..................................................................................................................................[2]

(b) Another person pulls an identical barrow and load from A to B, but this person pulls
much harder than the person in (a).

Describe what happens to the second person’s barrow.

..........................................................................................................................................

......................................................................................................................................[2]

(c) (i) State which person has the greater power between A and B.

...................................................................................................................................

(ii) Give two reasons for your answer to (c)(i).

reason 1 ....................................................................................................................

reason 2 ...................................................................................................................
[3]

© UCLES 2004 0625/02 M/J/04


7 For
Examiner’s
Use
5 Fig. 5.1 shows a shallow dish containing a liquid that evaporates easily. The bulb of a
thermometer is held in the liquid. A jet of air is blown over the surface of the liquid, so that the
liquid evaporates rapidly.

thermometer
jet of air

liquid

shallow dish

Fig. 5.1

(a) State what happens to the reading shown on the thermometer.

......................................................................................................................................[1]

(b) Explain your answer to (a) in terms of the behaviour of the molecules of the liquid.

..........................................................................................................................................

..........................................................................................................................................

......................................................................................................................................[2]

(c) State one example in everyday life where the effect demonstrated by this experiment
occurs.

......................................................................................................................................[1]

© UCLES 2004 0625/02 M/J/04 [Turn over


8 For
Examiner’s
Use
6 (a) When a certain amount of heat is supplied to 1 kg of insulated aluminium, the temperature
of the aluminium rises by 1 °C.

1 kg aluminium inside
a layer of insulation

heat supplied
to aluminium

Fig. 6.1

In what form does the aluminium store the energy that has been supplied?

......................................................................................................................................[1]

(b) The same amount of heat is supplied to 1 kg of insulated copper, as shown in Fig. 6.2.

1 kg copper inside
a layer of insulation

heat supplied
to copper

Fig. 6.2

The temperature rise of the 1 kg copper block is greater than the temperature rise of the
1 kg aluminium block in (a).

Explain, in terms of thermal capacity, why this is so.

..........................................................................................................................................

......................................................................................................................................[2]

© UCLES 2004 0625/02 M/J/04


9 For
Examiner’s
Use
7 Boy A throws a large stone into a large still pond, as illustrated in Fig. 7.1.

B
A

Fig. 7.1

Fig. 7.1

(a) Girl B hears the ‘plop’ sound of the stone entering the water a very short time after she
sees the splash, but it is many seconds before the water wave reaches the edge of the
pond where she is sitting.
Use this information to decide which wave travels fastest and which travels slowest.

Write ‘fastest’ in one box and ‘slowest’ in another box. Leave one box empty.

sound wave

light wave

water wave [2]

(b) In the boxes below, state whether each type of wave is a transverse or a longitudinal
wave.

sound wave

light wave

water wave [3]

(c) In the boxes below, put a tick alongside any of the types of wave that do not need a
substance in which to travel.

sound wave

light wave

water wave [1]

© UCLES 2004 0625/02 M/J/04 [Turn over


10 For
Examiner’s
Use
8 A, B, C and D are an aluminium bar, an unmagnetised iron bar and two bar magnets. Tests
are performed to find out which bar is which.
Each row of Fig. 8.1 shows what happens when two of the bars are placed end to end.

A B repel

A C attract

B D no effect

Fig. 8.1

Which bar is which? Complete the lines below.

Bar A is ........................................................................

Bar B is ........................................................................

Bar C is ........................................................................

Bar D is ........................................................................ [4]

© UCLES 2004 0625/02 M/J/04


11 For
Examiner’s
Use
9 Some pond water becomes contaminated by the release of radioactive waste.
The radioactivity of a sample of the contaminated water is tested every week for 5 weeks.
The results are shown in the table below.

time/weeks 0 1 2 3 4 5

activity
800 440 240 130 70 40
count/s

(a) Plot these values on Fig. 9.1. [3]

activity 800
count/s
700

600

500

400

300

200

100

0
0 1 2 3 4 5 6
time/weeks

Fig. 9.1

(b) Draw the best curve through your points. [1]

(c) Use your graph to find the half-life of the radioactive material in the sample. Show clearly
on the graph how you obtained your answer.

half-life of radioactive material = ………………… weeks [2]

(d) If the sample of contaminated water used in the test had been smaller, state how this
would have affected, if at all,

(i) the activity readings, .................................................................................................

(ii) the value of the half-life. ............................................................................................


[2]

© UCLES 2004 0625/02 M/J/04 [Turn over


12 For
Examiner’s
Use
10 (a) One coil of a transformer is connected to a toy train set. The other coil is connected to a
240 V a.c. mains supply, as shown in Fig. 10.1.

240 V

primary coil secondary coil


4800 turns 200 turns

to
train
set

Fig. 10.1

(i) How can you tell from Fig. 10.1 that the transformer is a step-down transformer?

...................................................................................................................................

...............................................................................................................................[1]

(ii) Calculate the voltage at which the toy train operates.

toy train operates at ………………… V [3]

(iii) 1. The voltage of the mains supply is reduced. What happens to the voltage
supplied to the train set? Tick one box.

increases
decreases
stays the same

2. An attempt is made to use the train set in a country where the mains supply is
110 V. Suggest one difference that might be noticed in the way the toy train
operates.

...........................................................................................................................

...........................................................................................................................
[2]
© UCLES 2004 0625/02 M/J/04
13 For
Examiner’s
Use
(b) Fig. 10.2 shows an electromagnetic relay being used to operate an electric motor.

pivoted iron
armature power supply
for motor

M motor
switch

contacts
relay core

Fig. 10.2

Below are sentences that describe stages of the process by which the circuit works.

A The armature pivots and the contacts close.


B The core of the relay is magnetised.
C The switch is closed and the current flows through the coil.
D A current flows through the motor, making it work.
E The core attracts the top part of the armature.

Put the sentences so that the stages are in the correct order. Put the appropriate letters
in the boxes below. One box has been filled in as an example.

Stage 1 is sentence C

Stage 2 is sentence

Stage 3 is sentence

Stage 4 is sentence

Stage 5 is sentence [3]

© UCLES 2004 0625/02 M/J/04 [Turn over


14 For
Examiner’s
Use
11 (a) The list below contains the names of five different components that might be found in an
electric circuit.

capacitor light-dependent resistor resistor thermistor variable resistor

Which of these has

(i) a resistance that falls rapidly when the temperature rises,

...................................................................................................................................

(ii) a resistance that changes when a sliding contact is moved,

...................................................................................................................................

(iii) a high resistance in the dark but a low resistance in daylight?

...................................................................................................................................
[3]

(b) A lamp shines with full brightness when connected to a 12 V battery, as shown in
Fig. 11.1.

12 V
0.50 A

lamp

Fig. 11.1

(i) 1. Write down the equation that links resistance with p.d. and current.

2. The current in the lamp is 0.50 A. Calculate the resistance of the lamp.

resistance of lamp = …………………


[4]

© UCLES 2004 0625/02 M/J/04


15 For
Examiner’s
Use
(ii) A resistor is now connected in series with the lamp, as shown in Fig. 11.2.

12 V

lamp

Fig. 11.2

1. State what happens to the current in the lamp when the resistor is added.

...........................................................................................................................

2. Explain your answer.

...........................................................................................................................

...........................................................................................................................

3. Suggest what change might be seen in the lamp.

...........................................................................................................................
[3]

© UCLES 2004 0625/02 M/J/04 [Turn over


16 For
Examiner’s
Use
12 (a) A man looks at his reflection in a vertical mirror. This is shown from the side in Fig. 12.1.

mirror

Fig. 12.1

(i) On Fig. 12.1, accurately mark with a clear dot labelled B where the image of the tip
A of the man’s beard will be.

(ii) On Fig. 12.1, accurately draw a ray from the tip of the man’s beard that reflects from
the mirror and goes into his eye. You may use faint construction lines if you wish.
Use arrows to show the direction of the ray.

(iii) The man can see the image, but it cannot be formed on a screen. What name is
given to this type of image?

...................................................................................................................................

(iv) Write down the equation that links the angles of incidence and reflection that the
ray makes with the mirror.

[7]

© UCLES 2004 0625/02 M/J/04


17 For
Examiner’s
Use
(b) A girl looks into a bathroom mirror to brush her hair. Fig. 12.2 shows what she sees in
the mirror.

reflection seen
in mirror

Fig. 12.2

(i) In which hand is she holding the brush? Tick one box.

left hand

right hand

(ii) She has a spot on her skin just below her left eye.
Mark clearly on Fig. 12.2 where this will appear on the reflection.
[2]

© UCLES 2004 0625/02 M/J/04


Centre Number Candidate Number Name

UNIVERSITY OF CAMBRIDGE INTERNATIONAL EXAMINATIONS


International General Certificate of Secondary Education
PHYSICS 0625/02
Paper 2 Theory
October/November 2004
1 hour 15 minutes
Candidates answer on the Question Paper.
No Additional Materials are required.

READ THESE INSTRUCTIONS FIRST

Write your Centre number, candidate number and name on all the work you hand in.
Write in dark blue or black pen in the spaces provided on the Question Paper.
You may use a soft pencil for any diagrams, graphs or rough working.
Do not use staples, paper clips, highlighters, glue or correction fluid.

Answer all questions.


At the end of the examination, fasten all your work securely together.
The number of marks is given in brackets [ ] at the end of each question or part question.
You may lose marks if you do not show your working or if you do not use appropriate units.
Take the weight of 1 kg to be 10 N (i.e. acceleration of free fall = 10 m/s2).

For Examiner’s Use

If you have been given a label, look at the


details. If any details are incorrect or
missing, please fill in your correct details
in the space given at the top of this page.

Stick your personal label here, if


provided.

This document consists of 15 printed pages and 1 blank page.


SPA (NH/BI) S66584/2
© UCLES 2004 [Turn over
2 For
Examiner’s
Use
1 Fig. 1.1 shows the top part of a measuring cylinder containing some liquid.

cm3

100

90

80 liquid

Fig. 1.1

(a) What is the volume of liquid in the measuring cylinder?

volume = .............................cm3 [1]

(b) Fig. 1.1 indicates four ways the observer’s eye could look when taking the reading from
the measuring cylinder. Put a circle around the eye position that gives the correct
reading. [1]

(c) In order to fill the measuring cylinder up to the 100 cm3 mark, 80 drops of the liquid are
added to the liquid already in the measuring cylinder.
Calculate the average volume of one drop.

average volume of a drop = ............................cm3 [4]

© UCLES 2004 0625/02 O/N/04


3 For
Examiner’s
Use
2 (a) In Fig. 2.1, the sealed drum containing gas has a mercury manometer connected to it in
order to indicate the gas pressure.

air pressure
760 mm of mercury

B
gas pressure
781 mm of mercury A

mercury

Fig. 2.1

For convenience, gas pressure is often expressed in mm of mercury.


The gas pressure is 781 mm of mercury and air pressure is 760 mm of mercury.

(i) State the difference in height between levels A and B on the manometer.

difference in height = ………….………..mm

(ii) The temperature of the gas rises.

State what happens to

1. the gas pressure, ..................................................................................................

2. the level A, ............................................................................................................

3. the level B. ...........................................................................................................


[3]

(b) The air in part (a) is also pressing on a large window pane in the wall of the room where
the drum is situated.

(i) State how the air pressure on the window pane compares with the air pressure on
the mercury surface at B in Fig. 2.1.

...................................................................................................................................

(ii) State how the force exerted by the air on the window pane compares with the force
exerted by the air on the mercury surface at B.

...................................................................................................................................
[2]
© UCLES 2004 0625/02 O/N/04 [Turn over
4 For
Examiner’s
Use
3 A packaging company purchases corrugated cardboard boxes in which to pack its goods.
The boxes are not made up when they are delivered, but are flat, as shown in Fig. 3.1.

0.20 m

0.60 m
0.50 m

Fig. 3.1

(a) A bundle of these boxes measures 0.60 m x 0.50 m x 0.20 m and has a mass of 7.2 kg.

(i) Calculate the volume of the bundle of boxes.

volume = .....................................[3]

(ii) Calculate the density of the corrugated cardboard.

density = .....................................[4]

(b) Corrugated cardboard is made up of 3 sheets of thick paper stuck together. Fig. 3.2
shows an enlarged view of the edge of a sheet of corrugated cardboard.

thick paper air

Fig. 3.2

(i) Here is an incomplete sentence about the paper.

The density of the paper is ................................... that of the corrugated cardboard.

Which of the words below correctly complete the sentence? Tick one box.

greater than the same as less than

(ii) Explain your answer to (b) (i).

...................................................................................................................................

...................................................................................................................................
[2]
© UCLES 2004 0625/02 O/N/04
5 For
Examiner’s
Use
4 Logs of wood are burning in a camp-fire on the ground. A person is sitting nearby.

Fig. 4.1

(a) (i) State two types of energy that the burning logs possess.

1. ...............................................................................................................................

2. ...............................................................................................................................

(ii) State the main method of heat transfer by which energy from the fire reaches the
person sitting nearby.

...................................................................................................................................
[3]

(b) A spark jumps out of the fire.

(i) State the name of the type of energy that the spark possesses due to its
movement.

...................................................................................................................................

(ii) The spark lands on the person’s hand.


State which method of heat transfer causes the person to feel the spark.

...................................................................................................................................

(iii) The pain caused by the spark makes the person stand up.

1. State the type of energy that has increased, now that he is standing.

...................................................................................................................................

2. State the type of energy stored in his body that enabled him to stand.

...................................................................................................................................
[4]

© UCLES 2004 0625/02 O/N/04 [Turn over


6 For
Examiner’s
Use
5 In order to observe Brownian motion, some smoke is mixed with air trapped in a small glass
box. The box is strongly illuminated from the side, and the smoke is viewed from above
through a microscope. This is illustrated in Fig. 5.1.

Fig. 5.1

(a) Describe what is seen when the microscope is focussed on the smoke particles.

..........................................................................................................................................

..........................................................................................................................................

......................................................................................................................................[3]

(b) State what causes the effect described in (a).

..........................................................................................................................................

..........................................................................................................................................

..........................................................................................................................................

......................................................................................................................................[4]

(c) The temperature of the glass box and its contents is increased. Suggest what change is
seen through the microscope.

..........................................................................................................................................

......................................................................................................................................[1]

© UCLES 2004 0625/02 O/N/04


7 For
Examiner’s
Use
6 (a) Here is a list of solid materials. Put a tick in the box alongside those materials which are
good conductors of heat.

aluminium
copper
cork
gold
polystyrene
wood
[3]

(b) State the word we use to describe materials that are poor conductors of heat.

......................................................................................................................................[1]

(c) From the list of materials given in (a), state which would be suitable to use for

(i) the base of a cooking pot,..........................................................................................

(ii) the covering on the handle of a kettle. .......................................................................


[2]

(d) Water is a poor conductor of heat. A beaker of water is heated as shown in Fig. 6.1.

water

heat

Fig. 6.1

All the water heats up quite quickly.


State the process by which heat spreads most rapidly through the water.

......................................................................................................................................[1]

(e) State why the heating element is always placed near the bottom of an electric kettle.

..........................................................................................................................................

......................................................................................................................................[1]

© UCLES 2004 0625/02 O/N/04 [Turn over


8 For
Examiner’s
Use
7 A girl is walking along a path 1600 m from the rock-face of a quarry (a place where stone is
obtained).

girl

explosion

rock face

quarry

1600 m

Fig. 7.1

(a) The quarry workers set off an explosion at X to break up some rock. The girl measures
the time interval between seeing the flash and hearing the bang.
The time is 5.0 s.

(i) Calculate the speed of the sound.

speed of sound = ………….………..m/s [3]

(ii) State what assumption you have made in your working in (i).

...............................................................................................................................[1]

(b) Suppose the explosion had taken place at Y instead of X.


State two ways in which the girl’s observations would have been different.

1. ......................................................................................................................................

2. ..................................................................................................................................[2]

© UCLES 2004 0625/02 O/N/04


9 For
Examiner’s
Use
8 Two light conducting balls A and B are hanging side by side, as shown in Fig. 8.1.

Fig. 8.1

A and B are given a series of different charges, as indicated in the table below.
In the third column of the table, write what is seen to happen in each case.
Use the words repulsion or attraction or nothing, as appropriate.

charge on A charge on B what is seen to happen

positive positive

negative negative

positive negative

zero positive

negative zero

[5]

© UCLES 2004 0625/02 O/N/04 [Turn over


10 For
Examiner’s
Use
9 (a) Two 10 Ω resistors are connected in series in a circuit, as shown in Fig. 9.1.

3.0 A 10 Ω 10 Ω

Fig. 9.1

The current at point X is 3.0 A.

(i) State the current at

1. point Y, ....................A

2. point Z. ....................A

(ii) Calculate the combined resistance of the two 10 Ω resistors.

combined resistance = ………….………..Ω


[3]

© UCLES 2004 0625/02 O/N/04


11 For
Examiner’s
Use
(b) The 10 Ω resistors are now arranged in parallel, as shown in Fig. 9.2.

3.0 A 1.5 A 10 Ω

10 Ω

Fig. 9.2

(i) The current at X is adjusted to 3.0 A. The current through one resistor is 1.5 A.

1. What is the current at Y? Tick one box.

0A
1.5 A
3.0 A
4.5 A

2. State the value of the current at Z. ....................A

(ii) What is the combined resistance of the two 10 Ω resistors? Tick one box.

0Ω
5Ω
10 Ω
20 Ω
[3]

© UCLES 2004 0625/02 O/N/04 [Turn over


12 For
Examiner’s
Use
10 Atoms contain protons, neutrons and electrons.

State which of these three

(a) has a negative charge, ..................................................................................................[1]

(b) is uncharged,.................................................................................................................[1]

(c) has a much smaller mass than the others, ...................................................................[1]

(d) is outside the nucleus,...................................................................................................[1]

(e) are nucleons,.................................................................................................................[2]

(f) are lost from the nucleus during α-particle emission. ...................................................[2]

© UCLES 2004 0625/02 O/N/04


13 For
Examiner’s
Use
11 The ray diagram in Fig. 11.1 shows a lens forming an image II′ of an object OO′.

O′

D E I
O A BC

I′

Fig. 11.1

(a) State

(i) which of the marked points is a principal focus of the lens, .......................................

(ii) which distance is the focal length of the lens.............................................................


[2]

(b) Which of the following words describe the image?

Tick all that are correct.

real
virtual
inverted
upright
enlarged
diminished
[3]

(c) On Fig. 11.1, draw one other ray that goes from O′, the top of the object, to the image.
[1]

© UCLES 2004 0625/02 O/N/04 [Turn over


14 For
Examiner’s
Use
12 (a) A small pin is fixed to the edge of a bench. A triangular piece of card with a small hole in
each corner is hung on the pin from corner A and allowed to settle, as shown in
Fig. 12.1. A plumb-line is then hung from the pin and the vertical line AP is marked on
the card.

P
plumb-line

Fig.

Fig. 12.1

This procedure is then repeated with the card hanging from C and the vertical line CQ is
marked.
After this, the card is as shown in Fig. 12.2.

Q C

Fig. 12
B

Fig. 12.2

On Fig. 12.2,

(i) draw the vertical line that would be obtained if the card were hung from B,
(ii) clearly mark the centre of mass of the card using a dot labelled G. [2]

© UCLES 2004 0625/02 O/N/04


15 For
Examiner’s
Use
(b) Fig. 12.3a shows a glass ornament standing on a shelf. Fig. 12.3b shows an identical
ornament filled with coloured glass beads.

State which ornament is more stable, and why.

..........................................................................................................................................

......................................................................................................................................[2]

(c) Fig. 12.4 shows two wooden blocks standing on a hinged board with a rough surface.

A
rough
surface

Fig. 12.4

The board is slowly tilted. The blocks do not slip. State which block falls over first, and
explain why.

..........................................................................................................................................

......................................................................................................................................[2]

© UCLES 2004 0625/02 O/N/04


Centre Number Candidate Number Name

UNIVERSITY OF CAMBRIDGE INTERNATIONAL EXAMINATIONS


International General Certificate of Secondary Education
PHYSICS 0625/02
Paper 2
May/June 2005

1 hour 15 minutes
Candidates answer on the Question Paper.
No Additional Materials are required.

READ THESE INSTRUCTIONS FIRST

Write your Centre number, candidate number and name on all the work you hand in.
Write in dark blue or black pen in the spaces provided on the Question Paper.
You may use a soft pencil for any diagrams, graphs or rough working.
Do not use staples, paper clips, highlighters, glue or correction fluid.

Answer all questions.


At the end of the examination, fasten all your work securely together.
The number of marks is given in brackets [ ] at the end of each question or part question.
You may lose marks if you do not show your working or if you do not use appropriate units.
Take the weight of 1 kg to be 10 N (i.e. acceleration of free fall = 10 m/s2).

For Examiner’s Use

If you have been given a label, look at the


details. If any details are incorrect or
missing, please fill in your correct details
in the space given at the top of this page.

Stick your personal label here, if


provided.

This document consists of 16 printed pages.


SPA (MML 8112 3/04) S80361/3
© UCLES 2005 [Turn over
2 For
Examiner's
Use
1 (a) A measuring cylinder contains 100 cm3 of water. 20 cm3 of the water is poured into a
beaker.

On Fig. 1.1, mark the level of the water left in the cylinder. [2]

cm3
100

50

Fig. 1.1

(b) A rule, calibrated in cm, is placed alongside the measuring cylinder, as shown in Fig. 1.2.

cm3
100
15

10
50

Fig. 1.2

(i) What is the length of the measuring cylinder, from zero up to the 100 cm3 mark?

..................................................................................................................................
(ii) The volume of a cylinder is found using the equation
volume = cross-sectional area × length.
Calculate the cross-sectional area of the measuring cylinder.

cross-sectional area = .................................


[5]
© UCLES 2005 0625/02/M/J/05
3 For
Examiner's
Use
2 A boat sails along a river, stopping at various places along the way. Fig. 2.1 shows how the
speed of the boat changes during the day, starting at 0900 hrs and reaching its final
destination at 2100 hrs.

speed

0
0900 1100 1300 1500 1700 1900 2100
time of day (24 -hour clock)

Fig. 2.1

(a) Calculate how long the whole journey takes.

time taken = ....................... hours [2]

(b) State the time of day at which the boat reaches its greatest speed.

time of day = ................................. [1]

(c) State the longest time for which the boat was stationary at one place.

longest time = ....................... hours [1]

(d) If the speed axis had values marked on it, state


(i) how the graph could be used to find the distance travelled between 0900 hrs and
1130 hrs,

..................................................................................................................................

..................................................................................................................................
(ii) how the average speed for the whole journey could be found.

..................................................................................................................................

..................................................................................................................................
[3]
© UCLES 2005 0625/02/M/J/05 [Turn over
4 For
Examiner's
Use
3 (a) A light vertical triangular piece of rigid plastic PQR is pivoted at corner P.

A horizontal 5 N force acts at Q, as shown in Fig. 3.1.

Q
5N

pivot

Fig. 3.1

Describe what, if anything, will happen to the piece of plastic.

..........................................................................................................................................

.................................................................................................................................... [2]

(b) On another occasion, two horizontal 5 N forces act on the piece of plastic, as shown in
Fig. 3.2.

Q
5N

pivot

5N
R

Fig. 3.2

(i) Describe what, if anything, will happen to the piece of plastic.

..................................................................................................................................

(ii) On Fig. 3.2, mark the force that the pivot exerts on the piece of plastic. Show the
direction of the force by means of an arrow and write the magnitude of the force
next to the arrow. [4]

© UCLES 2005 0625/02/M/J/05


5 For
Examiner's
Use
4 Fig. 4.1 represents a hydroelectric system for generating electricity.

reservoir

power
station water flows down

water flows out

Fig. 4.1

Answer the following questions, using words from this list.

chemical electrical gravitational internal (heat)

kinetic light nuclear sound strain

(a) What sort of energy, possessed by the water in the reservoir, is the main source of
energy for this system?

.................................................................................................................................... [1]

(b) When the water flows down the pipe, it is moving. What sort of energy does it possess
because of this movement?

.................................................................................................................................... [1]

(c) The water makes the turbines in the power station rotate. What sort of energy do the
turbines possess because of their rotation?

.................................................................................................................................... [1]

(d) What sort of energy does the power station generate?

.................................................................................................................................... [1]

(e) None of the energy transfer processes is perfect. In what form is most of the wasted
energy released?

.................................................................................................................................... [1]

© UCLES 2005 0625/02/M/J/05 [Turn over


6 For
Examiner's
Use
5 (a) On a hot day, a child drinks all the water in a plastic bottle. She then screws the cap
back tightly on the bottle, so that the bottle contains only air.

cap screwed on tightly

air

Fig. 5.1

She throws the bottle into a waste basket, where the Sun shines on it.

After a while in the Sun’s rays, the air in the bottle is much hotter than before.
(i) State what has happened to the pressure of the air in the bottle.

..................................................................................................................................
(ii) In terms of the behaviour of the air molecules, explain your answer to (a)(i).

..................................................................................................................................

..................................................................................................................................

..................................................................................................................................

..................................................................................................................................
[5]

© UCLES 2005 0625/02/M/J/05


7 For
Examiner's
Use
(b) Also in the waste basket is a broken glass bottle containing a small quantity of water, as
shown in Fig. 5.2.

water

Fig. 5.2

As the Sun shines on it, the volume of water slowly decreases.


(i) State the name of the process causing this decrease.

..................................................................................................................................
(ii) In terms of the effect of the Sun’s rays on the water molecules, explain your answer
to (b)(i).

..................................................................................................................................

..................................................................................................................................

..................................................................................................................................

..................................................................................................................................
[4]

© UCLES 2005 0625/02/M/J/05 [Turn over


8 For
Examiner's
Use
6 The table below shows the potential difference (p.d.) needed at different times during a day
to cause a current of 0.03 A in a particular thermistor.

time of day (24-hour clock) 0900 1200 1500 1800

p.d. / V 15.0 9.9 7.5

resistance /  500 210 250

(a) Calculate the two values missing from the table. You may use the space below for your
working. Write your answers in the table.

[3]

(b) On Fig. 6.1, plot the four resistance values.

600

resistance / 

400

200

0
0600 0900 1200 1500 1800 2100
time of day

Fig. 6.1
[2]
(c) (i) Draw a smooth curve through your points.
(ii) Why do we draw a smooth curve rather than a series of straight lines joining the
points?

..................................................................................................................................
[2]

© UCLES 2005 0625/02/M/J/05


9 For
Examiner's
Use
(d) The thermistor is a circuit component with a resistance that decreases as the temperature
increases.
(i) From your graph, estimate the time of day when the temperature was greatest.

time of day = .............................


(ii) State the reason for your answer to (d)(i).

..................................................................................................................................

..................................................................................................................................
[2]

© UCLES 2005 0625/02/M/J/05 [Turn over


10 For
Examiner's
Use
7 Fig. 7.1 shows the various regions of the electromagnetic spectrum.

radio visible

Fig. 7.1

Two of the regions have been labelled.

(a) In the boxes provided, write the names of the other regions. [4]

(b) Only one of the following types of wave is not an electromagnetic wave.

Tick one box to show which type of wave is not electromagnetic.

microwave

radar

sound [1]

© UCLES 2005 0625/02/M/J/05


11 For
Examiner's
Use
8 An inventor is trying to make a device to enable him to see objects behind him. He cuts a
square box in half diagonally and sticks two plane mirrors on the inside of the box.

A side view of the arrangement is shown in Fig. 8.1.

mirror

box cut
in half

mirror

Fig. 8.1

Fig. 8.2 shows the arrangement, drawn larger.

ray 1 from object

ray 2 from object


45°

90°

Fig. 8.2

Fig. 8.2 shows parallel rays from two different points on a distant object behind the man.

(a) Carefully continue the two rays until they reach the place where the inventor’s head will
be. [3]

(b) Look at what has happened to the two rays.

What can be said about the image the inventor sees?

.................................................................................................................................... [1]
© UCLES 2005 0625/02/M/J/05 [Turn over
12 For
Examiner's
Use
9 The speed of sound in air is 332 m/s. A man stands 249 m from a large flat wall, as shown in
Fig. 9.1, and claps his hands once.

woman man

249 m 249 m

Fig. 9.1

(a) Calculate the interval between the time when the man claps his hands and the time
when he hears the echo from the wall.

time interval = ........................... s [3]

(b) A woman is standing 249 m further away from the wall than the man. She hears the
clap twice, once directly and once after reflection from the wall.

How long after the man claps does she hear these two sounds? Tick two boxes.

0.75 s

1.50 s

2.25 s

3.00 s [2]

© UCLES 2005 0625/02/M/J/05


13 For
Examiner's
Use
10 (a) (i) What name do we give to the type of material that allows electrical charges to pass
through it?

..................................................................................................................................
(ii) Give an example of such a material.

..................................................................................................................................
(iii) What must be done to this type of material in order to make electrical charges pass
through it?

..................................................................................................................................

..................................................................................................................................
[3]

(b) (i) What name do we give to the type of material that does not allow electrical
charges to pass through it?

..................................................................................................................................
(ii) Give an example of such a material.

..................................................................................................................................
[2]

(c) Which of the two types of material in (a)(i) and (b)(i) may be held in the hand and
charged by friction (e.g. by rubbing with a soft cloth)?

.................................................................................................................................... [1]

© UCLES 2005 0625/02/M/J/05 [Turn over


14 For
Examiner's
Use
11 The circuit in Fig. 11.1 is connected up.

2Ω

resistance wire

Fig. 11.1

(a) How does the current in the resistance wire compare with the current in the 2 
resistor? Tick one box.

smaller same greater [1]

(b) A voltmeter connected across the resistance wire shows the same reading as a
voltmeter connected across the 2  resistor.

State the value of the resistance of the resistance wire.

........................................  [1]

(c) Calculate the combined resistance of the wire and the resistor.

combined resistance = ........................................  [2]

(d) The wire and resistor are disconnected and then reconnected in parallel, as shown in
Fig. 11.2.

A
resistance wire

2Ω

Fig. 11.2

© UCLES 2005 0625/02/M/J/05


15 For
Examiner's
Use
(i) What is the combined resistance of the wire and resistor in Fig.11.2? Tick one box.

zero

1

2

3

(ii) The ammeter in Fig. 11.1 reads 0.3 A. What is the reading on the ammeter in
Fig. 11.2? Tick one box.

zero

less than 0.3 A

0.3 A

more than 0.3 A


[2]

(e) Walls in buildings sometimes develop cracks. The width of a crack can be monitored by
measuring the resistance of a thin wire stretched across the crack and firmly fixed on
either side of the crack, as illustrated in Fig. 11.3.

thin wire

fixing pin
fixing pin
crack

Fig. 11.3

The wall moves and the crack widens slightly.

State what happens to

(i) the length of the wire, ...............................................................................................

(ii) the resistance of the wire. ........................................................................................


[2]

© UCLES 2005 0625/02/M/J/05 [Turn over


16 For
Examiner's
Use
12 (a) Complete the following table about the particles in an atom. The first row has been filled
in as an example.

particle mass charge location

proton 1 unit +1 unit in the nucleus

neutron

electron
[6]

(b) (i) Which of the particles in the table make up an -particle?

..................................................................................................................................
(ii) On the same scale as indicated by the table, state

1. the mass of an -particle, .................................................................................

2. the charge of an -particle. ...............................................................................


[3]

Permission to reproduce items where third-party owned material protected by copyright is included has been sought and cleared where possible. Every
reasonable effort has been made by the publisher (UCLES) to trace copyright holders, but if any items requiring clearance have unwittingly been included, the
publisher will be pleased to make amends at the earliest possible opportunity.

University of Cambridge International Examinations is part of the University of Cambridge Local Examinations Syndicate (UCLES), which is itself a department of
the University of Cambridge.

© UCLES 2005 0625/02/M/J/05


Centre Number Candidate Number Name

UNIVERSITY OF CAMBRIDGE INTERNATIONAL EXAMINATIONS


International General Certificate of Secondary Education
PHYSICS 0625/02
Paper 2 Core
October/November 2005

1 hour 15 minutes
Candidates answer on the Question Paper.
No Additional Materials are required.

READ THESE INSTRUCTIONS FIRST

Write your Centre number, candidate number and name on all the work you hand in.
Write in dark blue or black pen in the spaces provided on the Question Paper.
You may use a soft pencil for any diagrams, graphs or rough working.
Do not use staples, paper clips, highlighters, glue or correction fluid.

Answer all questions.


At the end of the examination, fasten all your work securely together.
The number of marks is given in brackets [ ] at the end of each question or part question.
You may lose marks if you do not show your working or if you do not use appropriate units.
Take the weight of 1 kg to be 10 N (i.e. acceleration of free fall = 10 m/s2).

For Examiner’s Use

If you have been given a label, look at the


details. If any details are incorrect or
missing, please fill in your correct details
in the space given at the top of this page.

Stick your personal label here, if


provided.

This document consists of 15 printed pages and 1 blank page.


MML 8113 3/04 S80925/2
© UCLES 2005 [Turn over
2 For
Examiner's
Use
1 The length of a spring is measured when various loads from 1.0 N to 6.0 N are hanging
from it. Fig. 1.1 gives a graph of the results.

25

20
length / cm

15

10

0
0 1 2 3 4 5 6
load / N

Fig. 1.1

Use the graph to find

(a) the length of the spring with no load attached,

length = ......................... cm [1]

(b) the length of the spring with 4.5 N attached,

length = ......................... cm [1]

(c) the extension caused by a 4.5 N load.

extension = ................... cm [2]

© UCLES 2005 0625/02/O/N/05


3 For
Examiner's
Use
2 Palm trees are growing every 25 m alongside the highway in a holiday resort.

1 2 3 4

IGCSE

Fig. 2.1

The IGCSE school bus drives along the highway.

(a) It takes 2 s for the bus to travel between palm tree 1 and palm tree 2.

Calculate the average speed of the bus between tree 1 and tree 2.

average speed = .......................... [4]

(b) It takes more than 2 s for the bus to travel from tree 2 to tree 3.

State what this information indicates about the speed of the bus.

.................................................................................................................................... [1]

(c) The speed of the bus continues to do what you have said in (b). State how the time
taken to go from tree 3 to tree 4 compares with the time in (b).

The time taken to travel from tree 3 to tree 4 is ............................................. the time to
travel from tree 2 to tree 3. [1]

© UCLES 2005 0625/02/O/N/05 [Turn over


4 For
Examiner's
Use
3 (a) Fig. 3.1 shows two examples of footwear being worn by people of equal weight at a
Winter Olympics competition.

skate
ski

Fig. 3.1

Which footwear creates the greatest pressure below it, and why?

Which? .............................................................................................................................

Why? .......................................................................................................................... [2]

(b) Drivers of high-sided vehicles, like the one in Fig. 3.2, are sometimes warned not to
drive when it is very windy.

Fig. 3.2

Suggest why they receive this warning.

..........................................................................................................................................

..........................................................................................................................................

.................................................................................................................................... [2]

© UCLES 2005 0625/02/O/N/05


5 For
Examiner's
Use
4 A man is delivering a cupboard to a house.

house

wheels step

Fig. 4.1

(a) The man rolls the cupboard at a steady speed from the lorry to the house. The friction
force in the wheels is 40 N. State the force with which the man has to push.

force = .................... N [1]

(b) The cupboard weighs 720 N. State the smallest force needed to lift the cupboard.

force = .................... N [1]

(c) The step is 0.20 m high. Calculate the work required to lift the cupboard onto the step.

work required = ..................................... [4]

(d) The man has to ask his assistant to help him lift the cupboard onto the step. Together,
they lift it onto the step in 1.2 s. The men work equally hard.

Calculate the power developed by each man.

power developed = ................................ [4]

© UCLES 2005 0625/02/O/N/05 [Turn over


6 For
Examiner's
Use
5 The apparatus shown in Fig. 5.1 is set up in a laboratory during a morning science lesson.

flask

air

clamp

tube

water

Fig. 5.1

Later in the day, the room temperature is higher than in the morning.

(a) What change is observed in the apparatus?

.................................................................................................................................... [1]

(b) Explain why this change happens.

.................................................................................................................................... [1]

(c) Suggest one disadvantage of using this apparatus to measure temperature.

.................................................................................................................................... [1]

© UCLES 2005 0625/02/O/N/05


7 For
Examiner's
Use
6 Fig. 6.1 shows a reed relay being used to switch on an electric motor when a variable
resistor is adjusted.

M +
power
– supply

reed
S
relay
variable resistor

Fig. 6.1

(a) The variable resistor is set at its greatest resistance and then switch S is closed. The
reeds in the reed relay do not close when this is done.
(i) State two things that happen in the coil of the reed relay.

1. ..............................................................................................................................

2. ........................................................................................................................ [2]
(ii) State what happens to the two reeds in the reed relay.

............................................................................................................................ [1]

(b) The resistance of the variable resistor is slowly decreased. The reeds in the reed relay
close. Fig. 6.2(a) shows how the current in the coil changes with time.

current
in
coil Fig. 6.2(a)

0
time
0

current
in
motor Fig. 6.2(b)

0
time
0

On Fig. 6.2(b), draw a line that might show how the current in the motor changes with
time as the variable resistor is adjusted. [4]
© UCLES 2005 0625/02/O/N/05 [Turn over
8 For
Examiner's
Use
7 (a) The filament of a lamp is placed at the principal focus of a lens, as shown in Fig. 7.1.

Fig. 7.1

On Fig. 7.1, continue the three rays through the lens and out into the air on the right of
the lens. [1]

(b) The lens in Fig. 7.2 has a focal length of 2.0 cm.

object

1 cm

1 cm

Fig. 7.2

On Fig. 7.2,
(i) mark and label the positions of the principal focus on the left of the lens and the
principal focus on the right of the lens, [1]
(ii) carefully draw a ray from the top of the object, parallel to the axis, through the lens
and continue it until it reaches the edge of the squared area, [1]
(iii) carefully draw a ray from the top of the object, which travels parallel to the axis
after it has passed through the lens, [1]
(iv) draw and label the image. [2]

© UCLES 2005 0625/02/O/N/05


9 For
Examiner's
Use
8 (a) Two magnets are laid on a bench. End A of an unidentified rod is held in turn above
one end of each magnet, with the results shown in Fig. 8.1.

A A
S N
N S

bench south pole north pole


lifted off lifted off
bench bench

Fig. 8.1

(i) Suggest what the unidentified rod is made from.

............................................................................................................................ [1]
(ii) State what, if anything, happens when the end A is held over one end of

1. an unmagnetised iron bar, ................................................................................

2. an uncharged plastic rod. ........................................................................... [2]

(b) Fig. 8.2 shows four identical plotting compasses placed around a bar magnet where
the magnetic field of the surroundings can be ignored. The pointer has only been
drawn on one plotting compass.

S N

Fig. 8.2

On Fig. 8.2, draw the pointers on the other three plotting compasses to indicate the
directions of the magnetic field of the bar magnet in those three places. [3]

© UCLES 2005 0625/02/O/N/05 [Turn over


10 For
Examiner's
Use
9 (a) Fig. 9.1 shows five circuit symbols and their names. The names are in the wrong order.

Draw a straight line from each symbol to its name. One line has been drawn as an
example.

V cell

lamp

ammeter

voltmeter

A switch

Fig. 9.1
[3]

(b) Fig. 9.2 shows a circuit.

ammeter 1
switch

lamp 2

lamp 1

V
1.5 ammeter 2
cell

Fig. 9.2

© UCLES 2005 0625/02/O/N/05


11 For
Examiner's
Use
(i) In the space below, draw the circuit using circuit symbols. [1]

(ii) On your diagram in (b)(i), add a voltmeter connected to measure the potential
difference across the cell. [1]
(iii) When the switch is pressed so that the contacts join, which of the lamps light up?

............................................................................................................................ [1]
(iv) When there is a current in the circuit, ammeter 1 reads 0.5 A.

What current does ammeter 2 read?

current = .................... A [1]


(v) One lamp “blows”, so that its filament breaks.
What happens in the circuit?

..................................................................................................................................

............................................................................................................................ [1]

© UCLES 2005 0625/02/O/N/05 [Turn over


12 For
Examiner's
Use
10 Some fat purchased from a shop is supplied as the block shown in Fig. 10.1.

10 cm

4 cm

6.5 cm

Fig. 10.1

Use the information in Fig. 10.1 to calculate

(a) the volume of the block,

volume = ...................... cm3 [2]

(b) the density of the fat. Give your answer to 2 significant figures.

density = ............................. [5]

© UCLES 2005 0625/02/O/N/05


13 For
Examiner's
Use
11 Fig. 11.1 shows a tube for producing cathode rays. The tube contains various parts.

B C

Fig. 11.1

A spot is formed on the screen by the cathode rays.

(a) What do cathode rays consist of? .............................................................................. [1]

(b) Which part, A, B, C or D, must be heated to create the cathode rays? ..................... [1]

(c) (i) Which part, A, B, C or D, is coated with fluorescent material? ........................... [1]
(ii) What is the purpose of the fluorescent material?

............................................................................................................................ [1]

(d) A potential difference is applied between the two halves of part C.

What effect does this have on the cathode rays?

.................................................................................................................................... [1]

(e) Explain why there needs to be a vacuum inside the tube.

..........................................................................................................................................

..........................................................................................................................................

.................................................................................................................................... [2]

© UCLES 2005 0625/02/O/N/05 [Turn over


14 For
Examiner's
Use
12 (a) State what is meant by
(i) the half-life of a radioactive substance,

..................................................................................................................................

............................................................................................................................ [3]
(ii) background radiation.

..................................................................................................................................

............................................................................................................................ [1]

(b) In a certain laboratory, the background radiation level is 25 counts/minute.

Fig. 12.1 is a graph of the count-rate measured by a detector placed a short distance
from a radioactive source in the laboratory.
80

70

count-rate
counts / min
60

50

40

30

20

10

0
0 20 40 60 80 100 120 140
time / min

Fig. 12.1
© UCLES 2005 0625/02/O/N/05
15 For
Examiner's
Use
(i) At zero time, the measured count-rate of the source and background together is
80 counts/minute.
Calculate the count-rate due to the source alone.

count-rate due to source = .................. counts/min [2]


(ii) After one half-life has elapsed, what is the count-rate
1. due to the source alone,

count-rate due to source = ................. counts/min


2. measured by the detector?

count-rate measured by detector = ........................ counts/min


[2]
(iii) Use the graph to find the half-life of the source.

half-life of source = ......................... min [1]


(iv) Why does the graph not drop below the 25 counts/minute line?

..................................................................................................................................

............................................................................................................................ [1]
(v) On Fig. 12.1, sketch the curve that might be obtained for a source with a shorter
half-life. [2]

© UCLES 2005 0625/02/O/N/05


16

BLANK PAGE

Permission to reproduce items where third-party owned material protected by copyright is included has been sought and cleared where possible. Every
reasonable effort has been made by the publisher (UCLES) to trace copyright holders, but if any items requiring clearance have unwittingly been included, the
publisher will be pleased to make amends at the earliest possible opportunity.

University of Cambridge International Examinations is part of the University of Cambridge Local Examinations Syndicate (UCLES), which is itself a department
of the University of Cambridge.

0625/02/O/N/05
Centre Number Candidate Number Name

UNIVERSITY OF CAMBRIDGE INTERNATIONAL EXAMINATIONS


International General Certificate of Secondary Education
PHYSICS 0625/02
Paper 2 Core
May/June 2006

1 hour 15 minutes
Candidates answer on the Question Paper.
No Additional Materials are required.

READ THESE INSTRUCTIONS FIRST

Write your Centre number, candidate number and name on all the work you hand in.
Write in dark blue or black pen.
You may use a soft pencil for any diagrams, graphs or rough working.
Do not use staples, paper clips, highlighters, glue or correction fluid.

Answer all questions.


You may lose marks if you do not show your working or if you do not use appropriate units.
Take the weight of 1 kg to be 10 N (i.e. acceleration of free fall = 10 m/s2).
At the end of the examination, fasten all your work securely together.
The number of marks is given in brackets [ ] at the end of each question or part question.

For Examiner’s Use

This document consists of 16 printed pages.


MML 10757 3/05 S99216/2
© UCLES 2006 [Turn over
2 For
Examiner's
Use
1 (a) For a special parade, the guest of honour is to sit on a chair whilst the parade passes
by. Unfortunately the ground beneath the chair is soft, so the parade organisers put the
chair on a large flat board, as shown in Fig. 1.1.

chair

board

soft
ground

Fig. 1.1

Explain why the board prevents the chair from sinking into the ground.

..........................................................................................................................................

.................................................................................................................................... [2]

(b) At the parade, some air-filled balloons are used as decorations, as shown in Fig. 1.2.

Fig. 1.2

(i) State what happens to the balloons when the Sun makes them hotter.

............................................................................................................................ [1]
(ii) In terms of molecules, explain your answer to (b)(i).

..................................................................................................................................

............................................................................................................................ [2]

© UCLES 2006 0625/02/M/J/06


3 For
Examiner's
Use
(c) A pump is used to pump up the balloons in (b). A valve in the pump becomes blocked,
as shown in Fig. 1.3.

air
blocked piston
valve

direction of motion of piston

Fig. 1.3

(i) The piston of the pump is pushed in. State what happens to the pressure of the air
trapped in the pump.

............................................................................................................................ [1]
(ii) In terms of molecules, explain your answer to (c)(i).

..................................................................................................................................

............................................................................................................................ [3]

© UCLES 2006 0625/02/M/J/06 [Turn over


4 For
Examiner's
Use
2 Fig. 2.1 is a full-size diagram of a rectangular block.

C
B

Fig. 2.1

(a) Use your rule to measure the lengths of the three sides AB, BC and CD. Write your
values below, in cm, to 2 significant figures.

length of AB = ......................... cm

length of BC = ......................... cm

length of CD = ........................ cm [2]

(b) Write down the equation you would use to calculate the volume of the block. Do not
attempt a calculation.

[1]

(c) If you used your values from (a), what would be the unit for the volume of the block?

unit of volume = ........................... [1]

© UCLES 2006 0625/02/M/J/06


5 For
Examiner's
Use
3 Fig. 3.1(a) shows a measuring cylinder, containing some water, on a balance.

Fig. 3.1(b) shows the same arrangement with a stone added to the water.

measuring
cylinder reading Q

reading P

water
stone

balance

reading R reading S
(a) (b)

Fig. 3.1

(a) Which two readings should be subtracted to give the volume of the stone?

reading ...................... and reading ...................... [1]

(b) Which two readings should be subtracted to give the mass of the stone?

reading ...................... and reading ...................... [1]

(c) In a certain experiment,

mass of stone = 57.5 g,

volume of stone = 25 cm3.


(i) Write down the equation linking density, mass and volume.

[1]
(ii) Calculate the density of the stone.

density of stone = ........................ [3]

© UCLES 2006 0625/02/M/J/06 [Turn over


6 For
Examiner's
Use
4 A piece of fruit is falling from a tree.

Fig. 4.1

(a) The list below contains the names of some different forms of energy.

Put a tick in the box alongside four that are possessed by the falling fruit.

chemical

electrical

gravitational (PE)

internal (thermal)

kinetic (KE)

light

sound

strain [4]

(b) Which form of energy increases as the fruit falls?

.................................................................... [1]

(c) Which form of energy decreases as the fruit falls?

.................................................................... [1]

(d) Which form of energy is stored in the body of a person as a result of eating the fruit?

.................................................................... [1]
© UCLES 2006 0625/02/M/J/06
7 For
Examiner's
Use
5 (a) State two changes that usually happen to the molecules of a solid when the solid is
heated.

1. ......................................................................................................................................

2. ................................................................................................................................ [2]

(b) Most substances expand when they are heated.


(i) State one example where such expansion is useful.

............................................................................................................................ [1]
(ii) State one example where such expansion is a nuisance, and has to be allowed for.

............................................................................................................................ [1]

6 Fig. 6.1 shows a section through a series of waves on water.

Fig. 6.1

(a) On Fig. 6.1, carefully mark and label


(i) the wavelength of the waves, [2]
(ii) the level of the flat, still water surface after the waves have passed. [2]

(b) Describe how, using a stopwatch, the frequency of the waves could be found.

..........................................................................................................................................

..........................................................................................................................................

.................................................................................................................................... [2]

© UCLES 2006 0625/02/M/J/06 [Turn over


8 For
Examiner's
Use
7 (a) Fig. 7.1 shows a ray of blue light shining onto a glass prism.

screen
air air

no
rm
al

of
ray ht
lig
blue

Fig. 7.1

With the aid of a straight edge, draw a possible path of the ray through the prism and
into the air until it reaches the screen. [3]

(b) When a ray of white light passes through the prism, it spreads into a spectrum of
colours that can be seen on the screen.
(i) What is the name of this spreading effect? Tick one box.

convergence

diffraction

dispersion

reflection [1]

(ii) Which colour is deviated least by the prism? ..................................................... [1]

(iii) Which colour is deviated most by the prism? ..................................................... [1]

© UCLES 2006 0625/02/M/J/06


9 For
Examiner's
Use
8 (a) State what is meant by the north pole of a magnet.

..........................................................................................................................................

.................................................................................................................................... [2]

(b) The north poles of two magnets are brought close together.

What sort of force, if any, is there between the poles? Tick one box.

attractive

repulsive

no force [1]

(c) Fig. 8.1 shows the north pole of a magnet close to an iron bar.

magnet

iron
bar

Fig. 8.1

(i) The iron bar is attracted to the north pole because of induced magnetism in the
iron bar.
On Fig. 8.1, mark clearly the induced north pole and the induced south pole of the
iron bar. [1]
(ii) State what happens to the induced magnetism in the iron bar when the magnet is
taken away.

............................................................................................................................ [1]

© UCLES 2006 0625/02/M/J/06 [Turn over


10 For
Examiner's
Use
9 (a) The table below gives the half-lives of three radioactive substances.

substance half-life

iodine-128 25 minutes
radon-222 3.8 days
strontium-90 28 years

Samples of each of the three substances have the same activity today. Which sample
will have the greatest activity in 1 year’s time? Explain your answer.

substance with greatest activity after 1 year ....................................................................

explanation ......................................................................................................................

.................................................................................................................................... [2]

(b) In 1986, an explosion at the Chernobyl nuclear power station released radioactive
substances into the air. One of the radioactive substances released was iodine-131.
Some of the iodine-131 found its way into cow’s milk.

The activity of a sample of this contaminated milk was measured each week for 4 weeks.
The results are shown below.

time / days 0 7 14 21 28
activity
––––––– 1000 547 294 162 88
counts / s

© UCLES 2006 0625/02/M/J/06


11 For
Examiner's
Use
(i) On Fig. 9.1, plot the values given in the table.

1000

800
activity
_______
counts / s

600

400

200

0
0 5 10 15 20 25 30
time / days

Fig. 9.1

(ii) Draw the best-fit curve through your points.


(iii) Use your graph to find the half-life of iodine-131, showing clearly on your graph
how you obtained your value.

half-life of iodine-131 = ....................... days


[6]

© UCLES 2006 0625/02/M/J/06 [Turn over


12 For
Examiner's
Use
10 (a) Fig. 10.1 shows a type of tube in which cathode rays can be produced.

fluorescent
filament anode screen

C D

Fig. 10.1

(i) A p.d. is connected between two terminals in order to cause thermionic emission.
Between which two of the four labelled terminals is the p.d. connected?

between point .................. and point .................... [1]


(ii) Where does the thermionic emission occur?

............................................................................................................................ [1]
(iii) What particles are emitted during thermionic emission? Tick one box.

-particles

electrons

neutrons

protons [1]

(iv) On Fig. 10.1, draw the path of the cathode rays that are created when all the
electrical connections are correctly made. [1]
(v) State what is seen when the cathode rays strike the fluorescent screen.

............................................................................................................................ [1]

© UCLES 2006 0625/02/M/J/06


13 For
Examiner's
Use
(b) Fig. 10.2 shows the same tube as in Fig. 10.1, with two metal plates alongside the tube.
A high p.d. is connected between the plates.

+V

–V

Fig. 10.2

On Fig. 10.2, draw the path of the cathode rays. [3]

(c) The tube in Fig. 10.1 and Fig. 10.2 has a vacuum inside it.

State why this vacuum is necessary.

..........................................................................................................................................

.................................................................................................................................... [1]

© UCLES 2006 0625/02/M/J/06 [Turn over


14 For
Examiner's
Use
11 Fig. 11.1 illustrates part of the journey of a car.

500 m 1000 m 1500 m

start end
of oil drops tree pylon
of
town town on road

Fig. 11.1

The car engine is leaking oil. Regularly, every 2.5 s, a drop of oil hits the road.

(a) The car is driven at a steady speed of 10 m/s through the town.
(i) Calculate the distance on the road between one oil drop and the next oil drop.

distance between oil drops = ................... m [2]


(ii) The town is 500 m across. Show that it takes the car 50 s to travel through the
town.

[3]

(b) At a distance of 1000 m outside the town, the car passes a tree. At a further distance of
1500 m, the car passes a pylon. Between the tree and the pylon the oil drops are all
75 m apart.

Calculate the speed of the car between the tree and the pylon.

speed of car = ................ m/s [2]

© UCLES 2006 0625/02/M/J/06


15 For
Examiner's
Use
(c) What has happened to the car between the end of the town and the tree?

Tick one box.

The car has accelerated.

The car has decelerated.

The car has travelled at constant speed. [1]

(d) Each of the three parts of the journey takes 50 s.

Calculate the average speed of the car for the whole journey between the beginning of
the town and the pylon.

average speed = ................ m/s [5]

© UCLES 2006 0625/02/M/J/06 [Turn over


16 For
Examiner's
Use
12 In the boxes of the left column below are some electrical hazards. In the boxes of the right
column are means of protecting against those hazards.

From each hazard, draw a line to the appropriate protection. One line has been drawn as an
example.

electrical hazard means of protection

loose live wire touches fuse or circuit-breaker


metal case of appliance in the circuit

worn insulation on cable use of switches with


to an appliance a nylon pull-cord

steam in a washroom earth wire connected to the


condenses inside a switch metal case of the appliance

wires get hot because visual check of cables


current is too high before connecting appliance

[3]

Permission to reproduce items where third-party owned material protected by copyright is included has been sought and cleared where possible. Every
reasonable effort has been made by the publisher (UCLES) to trace copyright holders, but if any items requiring clearance have unwittingly been
included, the publisher will be pleased to make amends at the earliest possible opportunity.

University of Cambridge International Examinations is part of the University of Cambridge Local Examinations Syndicate (UCLES), which is itself a
department of the University of Cambridge.

© UCLES 2006 0625/02/M/J/06


Centre Number Candidate Number Name

UNIVERSITY OF CAMBRIDGE INTERNATIONAL EXAMINATIONS


International General Certificate of Secondary Education
PHYSICS 0625/02
Paper 2 Core
October/November 2006

1 hour 15 minutes
Candidates answer on the Question Paper.
No Additional Materials are required.

READ THESE INSTRUCTIONS FIRST

Write your Centre number, candidate number and name on all the work you hand in.
Write in dark blue or black pen.
You may use a soft pencil for any diagrams, graphs or rough working.
Do not use staples, paper clips, highlighters, glue or correction fluid.

Answer all questions.


You may lose marks if you do not show your working or if you do not use appropriate units.
Take the weight of 1 kg to be 10 N (i.e. acceleration of free fall = 10 m/s2).

At the end of the examination, fasten all your work securely together.
The number of marks is given in brackets [ ] at the end of each question or part question.

For Examiner’s Use

This document consists of 15 printed pages and 1 blank page.


MML 10757 3/05 S99316/2
© UCLES 2006 [Turn over
2 For
Examiner's
Use
1 A worker on the production line in a factory is making brackets. An inspector times the
worker whilst he makes 5 brackets. To start, the stopwatch is set to zero.

After 5 brackets have been made, the stopwatch is as shown in Fig. 1.1.

60 5
55
50 10
60
50 10
45 40 20
15
30

40 20
35 25
30

Fig. 1.1

(a) State the reading on the stopwatch.

reading = .......................... s [1]

(b) Calculate the time taken to make 1 bracket.

time taken = .......................... s [2]

(c) The worker has a target of making 300 brackets per hour.

Does the worker meet his target? Tick one box and show the working which led you to
your answer.

Does the worker meet his target? Yes

No [4]

© UCLES 2006 0625/02/O/N/06


3 For
Examiner's
Use
2 Some IGCSE students were asked to write statements about mass and weight.

Their statements are printed below. Put a tick in the box alongside each of the two correct
statements.

Mass and weight are the same thing.

Mass is measured in kilograms.

Weight is a type of force.

Weight is the acceleration caused by gravity. [2]

© UCLES 2006 0625/02/O/N/06 [Turn over


4 For
Examiner's
Use
3 Fig. 3.1 shows the speed/time graph for a journey travelled by a tractor.

Q R
speed 6
m/s

2 P
O
S
0
0 50 100 170 200
time / s

Fig. 3.1

(a) Use the graph to describe the motion of the tractor during each of the marked sections.

OP ....................................................................................................................................

PQ ....................................................................................................................................

QR ...................................................................................................................................

RS .............................................................................................................................. [4]

(b) Which two points on the graph show when the tractor is stationary?

point ................. and point ................. [1]

(c) State the greatest speed reached by the tractor.

greatest speed = ................ m/s [1]

(d) For how long was the tractor travelling at constant speed?

at constant speed for ................... s [1]

(e) State how the graph may be used to find the total distance travelled during the 200 s
journey. Do not attempt a calculation.

.................................................................................................................................... [1]

© UCLES 2006 0625/02/O/N/06


5 For
Examiner's
Use
4 (a) Name the process by which thermal energy is transferred

(i) from the Sun to the Earth, ........................................................................................

(ii) through the metal of a cooking pot. ..........................................................................


[2]

(b) A child is sitting on an oscillating swing, as shown in Fig. 4.1. At the top of the
oscillation, the child and swing are momentarily at rest.

Fig. 4.1

(i) Use the names of appropriate types of energy to complete the following word
equation. Write on the lines in the boxes.

gravitational
......................... .........................
potential energy
at the top of the
= energy at the + energy at the + energy losses
bottom of the bottom of the
oscillation
oscillation oscillation

(ii) The child continues to sit still on the swing. The amplitude of the oscillations slowly
decreases.
Explain why this happens.

..................................................................................................................................

..................................................................................................................................
[3]
© UCLES 2006 0625/02/O/N/06 [Turn over
6 For
Examiner's
Use
5 (a) State the two factors on which the turning effect of a force depends.

1. ......................................................................................................................................

2. ................................................................................................................................ [2]

(b) Forces F1 and F2 are applied vertically downwards at the ends of a beam resting on a
pivot P. The beam has weight W. The beam is shown in Fig. 5.1.

F1 P W F2

Fig. 5.1

(i) Complete the statements about the two requirements for the beam to be in
equilibrium.

1. There must be no resultant ..................................................

2. There must be no resultant ..................................................

(ii) The beam in Fig. 5.1 is in equilibrium. F is the force exerted on the beam by the
pivot P.
Complete the following equation about the forces on the beam.

F = ....................................................
(iii) Which one of the four forces on the beam does not exert a moment about P?

...................................................
[4]

© UCLES 2006 0625/02/O/N/06


7 For
Examiner's
Use
6 A woman stands so that she is 1.0 m from a mirror mounted on a wall, as shown in Fig. 6.1.

mirror

1.0 m

Fig. 6.1

(a) On Fig. 6.1, carefully draw


(i) a clear dot to show the position of the image of her eye,
(ii) the normal to the mirror at the bottom edge of the mirror,
(iii) a ray from her toes to the bottom edge of the mirror and then reflected from the
mirror.
[5]

(b) Explain why the woman cannot see the reflection of her toes.

..........................................................................................................................................

.................................................................................................................................... [1]

(c) (i) How far is the woman from her image?

............................ m
(ii) How far must the woman walk, and in what direction, before the distance between
her and her image is 6.0 m?

distance walked = ............................ m

direction = ............................................
[4]
© UCLES 2006 0625/02/O/N/06 [Turn over
8 For
Examiner's
Use
7 A man is using an axe to chop down a tree, as shown in Fig. 7.1.

Fig. 7.1

(a) A short time after the axe hits the tree, the man hears a clear echo.

He estimates that the echo is heard 3 seconds after the axe hits the tree.
(i) Suggest what type of obstacle might have caused such a clear echo.

..................................................................................................................................
(ii) The speed of sound in air is 320 m/s.
Calculate the distance of the obstacle from the tree.

obstacle distance = ............................ m


[4]

© UCLES 2006 0625/02/O/N/06


9 For
Examiner's
Use
(b) A branch from the tree falls into some shallow water in a pond nearby. The branch sets
up a wave. The wave moves to the left a distance of 3.0 m before hitting the side of a
moored boat and reflecting back again.

water
wave
B

3.0 m
side of
boat
A

Fig. 7.2

The wave takes 5.0 s to travel from AB to the boat and back to AB.

Calculate the speed of the water-wave.

speed of wave = ......................... m/s [2]

© UCLES 2006 0625/02/O/N/06 [Turn over


10 For
Examiner's
Use
8 (a) Fig. 8.1 shows two groups of materials.

GROUP 1 GROUP 2
copper plastics
iron silk
gold glass
aluminium ebonite

Fig. 8.1

(i) Which group contains metals?

...................................................

(ii) Which group contains insulators?

...................................................

(iii) Write down the name of one of the eight materials above that may be charged by
rubbing it with a suitable dry cloth.

...................................................
[3]

(b) Two charged metal balls are placed close to a positively-charged metal plate.

One is attracted to the plate and one is repelled.

attracted to plate

charged
plate
repelled by plate

Fig. 8.2

Write a + sign on the ball that is positively charged and a – sign on the one that is
negatively charged. [1]

(c) State what is meant by an electric field.

..........................................................................................................................................

.................................................................................................................................... [3]
© UCLES 2006 0625/02/O/N/06
11 For
Examiner's
Use
9 The points plotted on the grid shown in Fig. 9.1 were obtained from a spring-stretching
experiment.

100

length / mm

80

60

40

20

0
0 1 2 3 4 5
load / N

Fig. 9.1

(a) Using a straight edge, draw a straight line through the first 5 points. Extend your line to
the edge of the grid. [1]

(b) Suggest a reason why the sixth point does not lie on the line you have drawn.

.................................................................................................................................... [1]

(c) Calculate the extension caused by the 3 N load.

extension = ...................... mm [2]

(d) A small object is hung on the unloaded spring, and the length of the spring becomes
62 mm.

Use the graph to find the weight of the object.

weight of object = ......................... N [1]

© UCLES 2006 0625/02/O/N/06 [Turn over


12 For
Examiner's
Use
10 A person has a 6 V bell. He hopes to operate the bell from a 240 V a.c. mains supply, with
the help of the transformer shown in Fig. 10.1.

240 V
a.c. output
input

primary coil secondary coil


5000 turns 250 turns

Fig. 10.1

(a) State how you can tell from Fig. 10.1 that the transformer is a step-down transformer.

.................................................................................................................................... [1]

(b) State how the output voltage compares with the input voltage in a step-down transformer.

.................................................................................................................................... [1]

(c) Calculate the output voltage of the transformer when connected to the 240 V mains
supply.

output voltage = .................... V [3]

(d) Why would it not be wise for the person to connect the 6 V bell to this output?

..........................................................................................................................................

.................................................................................................................................... [1]

© UCLES 2006 0625/02/O/N/06


13 For
Examiner's
Use
11 The table below contains some information about uranium-238.

proton number Z = 92

nucleon number A = 238

decays by emitting -particle

(a) State how many electrons there are in a neutral atom of uranium-238.

........................................ [1]

(b) State where in the atom the electrons are to be found.

........................................ [1]

(c) State how many neutrons there are in an atom of uranium-238.

........................................ [1]

(d) State where in the atom the neutrons are to be found.

........................................ [1]

(e) State what happens to the number of protons in an atom of uranium-238 when an
-particle is emitted.

.................................................................................................................................... [2]

© UCLES 2006 0625/02/O/N/06 [Turn over


14 For
Examiner's
Use
12 Fig. 12.1 shows an electric circuit.

ammeter

battery

lamp

15  resistor

Fig. 12.1

(a) The lamp lights, but the ammeter needle moves the wrong way.

What change should be made so that the ammeter works correctly?

.................................................................................................................................... [1]

(b) What does an ammeter measure?

............................................................................. [1]

(c) In the space below, draw a circuit diagram of the circuit in Fig. 12.1, using correct circuit
symbols.

[2]

© UCLES 2006 0625/02/O/N/06


15 For
Examiner's
Use
(d) (i) Name the instrument that would be needed to measure the potential difference
(p.d.) across the 15  resistor.

.............................................................
(ii) Using the correct symbol, add this instrument to your circuit diagram in (c), in a
position to measure the p.d. across the 15  resistor.
[2]

(e) The potential difference across the 15  resistor is 6 V.

Calculate the current in the resistor.

current = .................... A [3]

(f) Without any further calculation, state the value of the current in the lamp.

lamp current = .................... A [1]

(g) Another 15  resistor is connected in parallel with the 15  resistor that is already in the
circuit.
(i) What is the combined resistance of the two 15  resistors in parallel?
Tick one box.

30 

15 

7.5 

zero

(ii) State what effect, if any, adding this extra resistor has on the current in the lamp.

..................................................................................................................................
[2]

© UCLES 2006 0625/02/O/N/06


16

BLANK PAGE

Permission to reproduce items where third-party owned material protected by copyright is included has been sought and cleared where possible. Every
reasonable effort has been made by the publisher (UCLES) to trace copyright holders, but if any items requiring clearance have unwittingly been included, the
publisher will be pleased to make amends at the earliest possible opportunity.

University of Cambridge International Examinations is part of the University of Cambridge Local Examinations Syndicate (UCLES), which is itself a department of
the University of Cambridge.

0625/02/O/N/06
UNIVERSITY OF CAMBRIDGE INTERNATIONAL EXAMINATIONS
International General Certificate of Secondary Education
*8187295232*

PHYSICS 0625/02
Paper 2 Core May/June 2007
1 hour 15 minutes
Candidates answer on the Question Paper.
No Additional Materials are required.

READ THESE INSTRUCTIONS FIRST

Write your Centre number, candidate number and name on all the work you hand in.
Write in dark blue or black pen.
You may use a soft pencil for any diagrams, graphs or rough working.
Do not use staples, paper clips, highlighters, glue or correction fluid.
DO NOT WRITE IN ANY BARCODES. For Examiner’s Use
Answer all questions. 1
You may lose marks if you do not show your working or if you do not use
appropriate units. 2
Take the weight of 1 kg to be 10 N (i.e. acceleration of free fall = 10 m/s2).
3
At the end of the examination, fasten all your work securely together.
The number of marks is given in brackets [ ] at the end of each question or 4
part question.
5

10

11

12

Total

This document consists of 19 printed pages and 1 blank page.

SPA (MML 13116 3/06) T25803/4


© UCLES 2007 [Turn over
2

1 The mechanical stop-clock shown in Fig. 1.1 has For


Examiner’s
a seconds hand, which rotates once every minute Use

and a minutes hand, which rotates once every hour.

start reset stop


60
55 5

50 10
minutes hand

45 15
seconds hand

40 20

35 25
30

Fig. 1.1

(a) A student uses the clock to time the intervals between trains travelling along the railway
past his school.

He sets the clock to zero (both hands vertical).

As train 1 passes, he starts the clock and leaves it running.

After 35 s, train 2 passes.

On the blank face of Fig. 1.2, show the positions of the two hands of the clock as train 2
passes. Make sure it is clear which hand is which. [2]

60
55 5

50 10

45 15

40 20

35 25
30

Fig. 1.2

© UCLES 2007 0625/02/M/J/07


3

(b) Train 3 passes the school 4 minutes and 55 s after the clock was started. For
Examiner’s
On the blank face of Fig. 1.3, show the positions of the hands of the clock as train 3 Use

passes. [2]

60
55 5

50 10

45 15

40 20

35 25
30

Fig. 1.3

(c) Calculate the time interval between train 2 and train 3.

time interval = ............... min ............... s [1]

[Total: 5]

© UCLES 2007 0625/02/M/J/07 [Turn over


4

2 In a training session, a racing cyclist’s journey is in three stages. For


Examiner’s
Stage 1 He accelerates uniformly from rest to 12 m/s in 20 s. Use

Stage 2 He cycles at 12 m/s for a distance of 4800 m.

Stage 3 He decelerates uniformly to rest.

The whole journey takes 500 s.

(a) Calculate the time taken for stage 2.

time = ............... s [2]

(b) On the grid of Fig. 2.1, draw a speed/time graph of the cyclist’s ride. [3]

14

12
speed /
m/s 10

0
0 100 200 300 400 500
time / s

Fig. 2.1

© UCLES 2007 0625/02/M/J/07


5

(c) Show that the total distance travelled by the cyclist is 5400 m. For
Examiner’s
Use

[4]

(d) Calculate the average speed of the cyclist.

average speed = ............... m/s [2]

[Total: 11]

© UCLES 2007 0625/02/M/J/07 [Turn over


6

3 A piece of stiff cardboard is stuck to a plank of wood by means of two sticky-tape “hinges”. For
This is shown in Fig. 3.1. Examiner’s
Use

stiff
cardboard

sticky-tape “hinge”

A plank of
B wood
C

Fig. 3.1

(a) The cardboard is lifted as shown, using a force applied either at A or B or C.

(i) On Fig. 3.1, draw the force in the position where its value will be as small as
possible.
[2]

(ii) Explain why the position you have chosen in (a)(i) results in the smallest force.

............................................................................................................................ [1]

(b) Initially, the cardboard is flat on the plank of wood. A box of matches is placed on it. The
cardboard is then slowly raised at the left hand edge, as shown in Fig. 3.2.

stiff
cardboard

sticky-tape “hinge”

plank of
wood

Fig. 3.2

State the condition for the box of matches to fall over.

..........................................................................................................................................

.................................................................................................................................... [2]

© UCLES 2007 0625/02/M/J/07


7

(c) The box of matches is opened, as shown in Fig. 3.3. The procedure in (b) is repeated. For
Examiner’s
Use

stiff
cardboard

sticky-tape “hinge”

plank of
wood

Fig. 3.3

(i) Complete the sentence below, using either the words “greater than” or “the same
as” or “less than”.

In Fig. 3.3, the angle through which the cardboard can be lifted before

the box of matches falls is …………………………………………… the angle

before the box of matches falls in Fig. 3.2. [1]

(ii) Give a reason for your answer to (c)(i).

..................................................................................................................................

............................................................................................................................ [1]

[Total: 7]

© UCLES 2007 0625/02/M/J/07 [Turn over


8

4 In Fig. 4.1, a small bird, a large bird and a squirrel are on the ground under a tree. For
Examiner’s
Use

Fig. 4.1

A loud noise scares the two birds. They both fly up to the top of the tree.

(a) (i) Which bird does the most work raising itself to the top of the tree? ................... [1]

(ii) Explain your answer to (a)(i).

............................................................................................................................ [1]

(b) A squirrel has the same weight as the large bird. It climbs the tree, to the same height
as the birds.

How does the increase in the squirrel’s gravitational potential energy compare with that
of each of the two birds? Answer the question by completing the sentences below.

Compared with that of the small bird, the increase of the squirrel’s potential

energy is ................................................................. .

Compared with that of the large bird, the increase of the squirrel’s potential

energy is ................................................................. . [2]

(c) Which creature has the least gravitational potential energy when they are at the top of
the tree?

.................................................................................................................................... [1]

(d) The small bird flies back down to the ground.

What happens to the gravitational potential energy it had at the top of the tree?

.................................................................................................................................... [2]

[Total: 7]
© UCLES 2007 0625/02/M/J/07
9

5 (a) Here is a list of descriptions of molecules in matter. For


Examiner’s
Use
description solid gas

free to move around from place to place

can only vibrate about a fixed position

closely packed

relatively far apart

almost no force between molecules

strong forces are involved between molecules

In the columns alongside the descriptions, put ticks next to those which apply to the
molecules in

(i) a solid,

(ii) a gas. [4]

(b) The water in a puddle of rainwater is evaporating.

Describe what happens to the molecules when the water evaporates.

..........................................................................................................................................

.................................................................................................................................... [2]

[Total: 6]

© UCLES 2007 0625/02/M/J/07 [Turn over


10

6 (a) Fig. 6.1 shows how the pressure of the gas sealed in a container varies during a period For
of time. Examiner’s
Use

pressure

time

Fig. 6.1

Which of the following statements could explain this variation of pressure?

Tick two statements.

The temperature of the gas is increasing.

The temperature of the gas is decreasing.

The volume of the container is increasing.

The volume of the container is decreasing. [2]

© UCLES 2007 0625/02/M/J/07


11

(b) Fig. 6.2 shows some gas trapped in a cylinder with a movable piston. For
Examiner’s
Use
cylinder piston

gas

Fig. 6.2

The temperature of the gas is raised.

(i) State what must happen to the piston, if anything, in order to keep the pressure of
the gas constant.

............................................................................................................................ [1]

(ii) State your reasons for your answer to (b)(i).

..................................................................................................................................

............................................................................................................................ [1]

[Total: 4]

© UCLES 2007 0625/02/M/J/07 [Turn over


12

7 An electric soldering iron is used to melt solder, for joining wires in an electric circuit. A For
soldering iron is shown in Fig. 7.1. Examiner’s
Use

metal cylinder plastic handle


with heater coil
copper tip
inside
lead to

electricity
supply

Fig. 7.1

Solder is a metal which melts easily. The heater coil inside the metal cylinder heats the
copper tip.

(a) (i) Suggest why the tip is made of copper.

............................................................................................................................ [1]

(ii) Suggest why the handle is made of plastic.

............................................................................................................................ [1]

(b) The heater coil is switched on. When the tip is put in contact with the solder, some of the
heat is used to melt the solder.

(i) State the process by which the heat is transferred from the copper tip to the solder.

............................................................................................................................ [1]

(ii) By which process or processes is the rest of the heat transferred to the surroundings?
Tick the boxes alongside any of the following (you may tick as many as you think
are correct).

conduction

convection

evaporation

radiation [2]

© UCLES 2007 0625/02/M/J/07


13

(c) A short time after switching on the soldering iron, it reaches a steady temperature, even For
though the heater coil is constantly generating heat. Examiner’s
Use

The soldering iron is rated at 40 W.

What is the rate at which heat is being lost from the soldering iron? Tick one box.

greater than 40 W

equal to 40 W

less than 40 W [1]

[Total: 6]

© UCLES 2007 0625/02/M/J/07 [Turn over


14

8 A square wooden block is made to rotate 3000 times per minute. A springy metal strip presses For
against the block, as shown in Fig. 8.1. A person nearby observes what is happening. Examiner’s
Use

3000 rotations / minute

springy
metal strip

Fig. 8.1

(a) Calculate how many times per second the block rotates.

number of rotations per second = ....................... [1]

(b) Calculate the frequency of the sound caused by this arrangement.

frequency = ................ Hz [2]

(c) State whether or not this sound could be heard by the person nearby, and give a reason
for your answer.

..........................................................................................................................................

.................................................................................................................................... [1]

[Total: 4]

© UCLES 2007 0625/02/M/J/07


15

9 (a) Fig. 9.1 shows two resistors connected to a 6 V battery. For


Examiner’s
Use

6V X

10 Ω

Fig. 9.1

(i) What name do we use to describe this way of connecting resistors?

............................................................................................................................ [1]

(ii) Calculate the combined resistance of the two resistors.

combined resistance = ..................  [1]

(iii) Calculate the current in the circuit.

current = ...................... [4]

(iv) Use your answer to (a)(iii) to calculate the potential difference across the 10 
resistor.

potential difference = .................. V [2]

(v) State the potential difference between terminals X and Y.

.................. V [1]

© UCLES 2007 0625/02/M/J/07 [Turn over


16

(b) The circuit in Fig. 9.2 is similar to the circuit in Fig. 9.1, but it uses a resistor AB with a For
sliding contact. Examiner’s
Use

A
sliding
contact

6V X

B
Y

Fig. 9.2

(i) State the potential difference between X and Y when the sliding contact is at

1. end A of the resistor, .............. V

2. end B of the resistor. .............. V [2]

(ii) The sliding contact of the resistor AB is moved so that the potential difference
between X and Y is 5 V.

On Fig. 9.2, mark with the letter C the position of the sliding contact. [1]

[Total: 12]

© UCLES 2007 0625/02/M/J/07


17

10 Your teacher gives you a length of wire, a sensitive millivoltmeter and a powerful magnet. You For
are asked to demonstrate the induction of an e.m.f. in the wire. Examiner’s
Use

(a) Describe what you would do.

..........................................................................................................................................

..........................................................................................................................................

..........................................................................................................................................

..........................................................................................................................................

.................................................................................................................................... [2]

(b) How would you know that an e.m.f. has been induced?

.................................................................................................................................... [1]

(c) Name a device which makes use of electromagnetic induction.

.................................................................................................................................... [1]

[Total: 4]

© UCLES 2007 0625/02/M/J/07 [Turn over


18

11 Fig. 11.1 shows a bar magnet on a board in a region where the magnetic field of the For
surroundings is so weak it can be ignored. The letters N and S show the positions of the Examiner’s
north and south poles of the magnet. Also on the diagram are marked four dots. Use

N S

Fig. 11.1

(a) On Fig. 11.1, carefully draw four magnetic field lines, one passing through each of the
four dots. The lines you draw should begin and end either on the magnet or at the edge
of the board. [5]

(b) On one of your lines, put an arrow to show the direction of the magnetic field. [1]

[Total: 6]

© UCLES 2007 0625/02/M/J/07


19

12 Three particles you have learned about are For


Examiner’s
protons, neutrons and electrons. Use

(a) How many of each of these particles

(i) are found in an -particle,

number of protons = ...................

number of neutrons = ...................

number of electrons = ................... [1]

(ii) are found in a -particle?

number of protons = ...................

number of neutrons = ...................

number of electrons = ................... [1]

(b) Sodium-24 can be represented as 24


11Na.

How many of each of these particles are there in a neutral atom of 24


11Na?

number of protons = .....................

number of neutrons = .....................

number of electrons = ..................... [3]

(c) A nucleus of sodium-24 decays to become magnesium-24, by the emission of one


particle. The equation below describes this change.

The symbol yx  represents the emitted particle.

24Na 24 Mg + yx 
11 12

(i) State the value of x. .................... [1]

(ii) State the value of y. .................... [1]

(iii) What type of particle is ? .................... [1]

[Total: 8]

© UCLES 2007 0625/02/M/J/07


20

BLANK PAGE

Permission to reproduce items where third-party owned material protected by copyright is included has been sought and cleared where possible. Every
reasonable effort has been made by the publisher (UCLES) to trace copyright holders, but if any items requiring clearance have unwittingly been included, the
publisher will be pleased to make amends at the earliest possible opportunity.

University of Cambridge International Examinations is part of the Cambridge Assessment Group. Cambridge Assessment is the brand name of University of
Cambridge Local Examinations Syndicate (UCLES), which is itself a department of the University of Cambridge.

0625/02/M/J/07
UNIVERSITY OF CAMBRIDGE INTERNATIONAL EXAMINATIONS
International General Certificate of Secondary Education
*8902325113*

PHYSICS 0625/02
Paper 2 Core October/November 2007
1 hour 15 minutes
Candidates answer on the Question Paper.
No Additional Materials are required.

READ THESE INSTRUCTIONS FIRST

Write your Centre number, candidate number and name on all the work you hand in.
Write in dark blue or black pen.
You may use a soft pencil for any diagrams, graphs or rough working.
Do not use staples, paper clips, highlighters, glue or correction fluid.
DO NOT WRITE IN ANY BARCODES.
For Examiner’s Use
Answer all questions.
You may lose marks if you do not show your working or if you do not use 1
appropriate units.
Take the weight of 1 kg to be 10 N (i.e. acceleration of free fall = 10 m/s2). 2
At the end of the examination, fasten all your work securely together. 3
The number of marks is given in brackets [ ] at the end of each question or
part question. 4

10

11

12

Total

This document consists of 16 printed pages.

SP (MML 13331 4/06) T25940/5


© UCLES 2007 [Turn over
2 For
Examiner’s
Use
1 Fig. 1.1 shows some liquid in a measuring cylinder. The metal can next to it has a cross-
sectional area which is four times that of the measuring cylinder.

cm3

100
measuring
cylinder 90

80

70

60

50 metal
can
40
liquid 30

20

10

Fig. 1.1

(a) State the volume of the liquid.

volume = ........................................ cm3 [1]

(b) The liquid is poured into the metal can.

On Fig. 1.1, draw the surface of the liquid in the can. [2]

(c) Complete the following sentence.

The pressure of the water at the bottom of the can is ……………………………… than
the pressure that the water had at the bottom of the measuring cylinder. [1]

[Total: 4]

© UCLES 2007 0625/02/O/N/07


3 For
Examiner’s
Use
2 A theatre measures 100 m × 80 m × 25 m. The air inside it has a density of 1.3 kg / m3 when it
is cool.

(a) Calculate the volume of the air in the theatre.

volume of air = .......................................... m3 [1]

(b) Calculate the mass of the air. State the equation you are using.

mass of air = ................................................ [4]

(c) Some time after the doors are opened, the heating in the theatre is switched on.

State and explain what happens to the mass of the air in the theatre as it warms up.

statement .........................................................................................................................

..........................................................................................................................................

explanation ......................................................................................................................

.................................................................................................................................... [2]

(d) Suggest why the temperature of the air in the balcony of the theatre (nearer the ceiling)
is likely to be greater than that lower down in the theatre.

.................................................................................................................................... [1]

[Total: 8]

© UCLES 2007 0625/02/O/N/07 [Turn over


4 For
Examiner’s
Use
3 Fig. 3.1 shows a simple mercury barometer, drawn 1/10 full size.

glass tube

mercury

reservoir

Fig. 3.1

(a) On Fig. 3.1, use your rule to make an appropriate measurement, and then use it to
calculate the atmospheric pressure.

atmospheric pressure = ..................................... cm Hg [2]

(b) State what occupies the space in the tube above the mercury.

..........................................................................................................................................

.................................................................................................................................... [1]

© UCLES 2007 0625/02/O/N/07


5 For
Examiner’s
Use
(c) On another occasion, the atmospheric pressure is much less than that shown in
Fig. 3.1.

On Fig. 3.1, mark where the mercury surfaces in the tube and in the reservoir might
be. [2]

(d) The tube above the mercury gets broken and allows air to move in to and out of the
tube.

Explain why the barometer no longer functions.

..........................................................................................................................................

.................................................................................................................................... [2]

[Total: 7]

© UCLES 2007 0625/02/O/N/07 [Turn over


6 For
Examiner’s
Use
4 A diesel engine is used to drag a boat up a slipway (see Fig. 4.1).

diesel
engine

slipway
s

Fig. 4.1

The boat finishes in the position shown by the broken outline.

(a) On Fig. 4.1, carefully mark

(i) the weight W of the boat, using an arrow labelled W, [1]

(ii) the friction force F on the boat, using an arrow labelled F. [1]

(b) State, in terms of W, F, h and s, how you could calculate

(i) the work done lifting the weight of the boat,

..................................................................................................................................

............................................................................................................................ [1]

(ii) the work done against the friction force,

..................................................................................................................................

............................................................................................................................ [1]

(iii) the total work done pulling the boat up the slipway.

............................................................................................................................ [1]

(c) What other measurement would you need to make if you wanted to calculate the useful
power output of the diesel engine?

.................................................................................................................................... [1]

[Total: 6]

© UCLES 2007 0625/02/O/N/07


7 For
Examiner’s
Use
5 Fig. 5.1 shows a liquid-in-glass thermometer.

capillary tube

–10 0 10 20 30 40 50 60 70 80 90 100 110 120 130 140 150

liquid

Fig. 5.1

(a) The thermometer is used for measuring temperatures in school laboratory experiments.

State the units in which the temperatures are measured. .......................................... [1]

(b) On Fig. 5.1, mark where the liquid thread will reach when the thermometer is placed in

(i) pure melting ice (label this point ICE), [1]

(ii) steam above boiling water (label this point STEAM). [1]

(c) A liquid-in-glass thermometer makes use of the expansion of a liquid to measure


temperature. Other thermometers make use of other properties that vary with
temperature.

In the table below, write in two properties, other than expansion of a liquid, that can be
used to measure temperature.

example expansion OF a liquid


1. OF

2. OF

[2]

[Total: 5]

© UCLES 2007 0625/02/O/N/07 [Turn over


8 For
Examiner’s
Use
6 A motorcyclist drives along a straight road. Fig. 6.1 gives information about the first 10 s of
his ride.

start after 5 s after 10 s


0m/s 9m/s 18 m / s

Fig. 6.1

(a) From the information on Fig. 6.1,

(i) describe the motion of the motorcyclist by ticking one of the following boxes,

constant speed

uniform acceleration

uniform deceleration
[1]

(ii) estimate the average speed of the motorcyclist during the 10 s,

average speed = ........................................ m / s [1]

(iii) calculate the distance travelled during the 10 s.

distance travelled = ............................................ m [2]

(b) State why the distance travelled in the first 5 s is less than half of the distance travelled
in the first 10 s of the journey.

..........................................................................................................................................

.................................................................................................................................... [1]

[Total: 5]

© UCLES 2007 0625/02/O/N/07


9 For
Examiner’s
Use
7 A girl drops a small stone from a bridge into a pond.

Fig. 7.1

(a) The stone hits the water surface at point X. Fig. 7.2, which is drawn full-size, shows the
wavefront a fraction of a second after the stone hits the water.

(i) The wave travels at 5 cm / s.


Calculate how far the wave travels in 0.3 s.

distance travelled = .......................................... cm [1]

(ii) On Fig. 7.2, draw the position of the wavefront 0.3 s after that already shown. [2]

Fig. 7.2

(b) A ringing bell also sends out waves in all directions.

State two ways in which these waves are different from the waves in part (a), other than
the fact that one is created in air and the other in water.

1. ......................................................................................................................................

2. ................................................................................................................................ [2]

[Total: 5]
© UCLES 2007 0625/02/O/N/07 [Turn over
10 For
Examiner’s
Use
8 An object OX is placed in front of a converging lens. The lens forms an image IY.

Fig. 8.1 shows two rays from the object to the image.

I
O

Fig. 8.1

(a) On Fig. 8.1,

(i) clearly mark and label the principal focus and the focal length of the lens, [3]

(ii) draw a third ray from X to Y. [1]

© UCLES 2007 0625/02/O/N/07


11 For
Examiner’s
Use
(b) The following list contains descriptions that can be applied to images.

Tick any which apply to the image shown in Fig. 8.1.

real

virtual

enlarged

diminished

inverted

upright

image distance less than object distance

image distance more than object distance


[4]

(c) State two things that happen to the image in Fig. 8.1 when the object is moved further
away from the lens.

1. ......................................................................................................................................

2. ................................................................................................................................ [2]

[Total: 10]

© UCLES 2007 0625/02/O/N/07 [Turn over


12 For
Examiner’s
Use
9 A thermistor connected to a variable voltage supply is immersed in a beaker of water.

The beaker of water is heated slowly, using a Bunsen burner, as shown in Fig. 9.1.

variable voltage supply

thermometer

water

Bunsen
burner

Fig. 9.1

At different temperatures, the voltage is adjusted until the current is 25 mA and the value of
the voltage is noted.

The results are shown below.

temperature / °C 15 30 45 60 75 90

voltage / V 18.8 8.8 4.7 2.6 1.5 1.2

(a) On the axes of Fig. 9.2, shown on page 13, plot a graph of voltage against temperature.
[4]

© UCLES 2007 0625/02/O/N/07


13 For
Examiner’s
Use
25

voltage / V

20

15

10

0
0 20 40 60 80 100
temperature / oC
Fig. 9.2

(b) From the graph, find the voltage needed to give a current of 25 mA
(i) at 40 °C, voltage needed = ............................................. V [1]

(ii) at 80 °C. voltage needed = ............................................. V [1]

(c) Use your results in (b) to calculate the resistance of the thermistor
(i) at 40 °C,

resistance at 40 °C = .....................................................

(ii) at 80 °C.

resistance at 80 °C = .....................................................
[5]

(d) Use your results in (c) to complete the following sentence about thermistors of the sort
used in this experiment.

The thermistor in this experiment is a device whose resistance …………………………


as the temperature increases. [1]

[Total: 12]
© UCLES 2007 0625/02/O/N/07 [Turn over
14 For
Examiner’s
Use
10 A coil of insulated wire is connected in series with a battery, a resistor and a switch.

Fig. 10.1

(a) The switch is closed and the current in the coil creates a magnetic field.

(i) On Fig. 10.1, draw the shape of the magnetic field, both inside and outside the coil.
[4]

(ii) A glass bar, an iron bar and a perspex bar are placed in turn inside the coil.

Which one makes the field stronger? ................................................................. [1]

(b) Two thin iron rods are placed inside the coil as shown in Fig. 10.2. The switch is then
closed.

Fig. 10.2

The iron rods move apart. Suggest why this happens.

..........................................................................................................................................

.................................................................................................................................... [3]

[Total: 8]
© UCLES 2007 0625/02/O/N/07
15 For
Examiner’s
Use
11 The activity of a sample of radioactive material is determined every 10 minutes for an hour.
The results are shown in the table.

time / minutes 0 10 20 30 40 50 60

activity
461 332 229 162 106 81 51
count / s

(a) From the figures in the table, estimate the half-life of the radioactive material.

half-life = .................................. minutes [1]

(b) A second experiment is carried out with another sample of the same material. At the
start of the experiment, this sample has twice the number of atoms as the first sample.

Suggest what values might be obtained for

(i) the activity at the start of the second experiment,

................................... count / s [1]

(ii) the half-life of the material in the second experiment.

................................... minutes [1]

(c) Name one type of particle that the material might be emitting in order to cause this
activity.

.................................................................................................................................... [1]

[Total: 4]

© UCLES 2007 0625/02/O/N/07 [Turn over


16 For
Examiner’s
Use
12 A beam of cathode rays is travelling in a direction perpendicularly out of the page. The beam
is surrounded by four metal plates P1, P2, P3 and P4 as shown in Fig. 12.1.

On Fig. 12.1, the beam is shown as the dot at the centre.

P2

P1

P3

P4
Q

Fig. 12.1

(a) Cathode rays are produced by thermionic emission.

What is the name of the particles which make up cathode rays?

.................................................................................................................................... [1]

(b) A potential difference is applied between P1 and P3, with P1 positive with respect to P3.

State what happens to the beam of cathode rays.

.................................................................................................................................... [2]

(c) The potential difference in (b) is removed. Suggest how the beam of cathode rays can
now be deflected down the page towards Q.

..........................................................................................................................................

.................................................................................................................................... [2]

(d) Cathode rays are invisible. State one way to detect them.

.................................................................................................................................... [1]

[Total: 6]

Permission to reproduce items where third-party owned material protected by copyright is included has been sought and cleared where possible.
Every reasonable effort has been made by the publisher (UCLES) to trace copyright holders, but if any items requiring clearance have unwittingly been
included, the publisher will be pleased to make amends at the earliest possible opportunity.

University of Cambridge International Examinations is part of the Cambridge Assessment Group. Cambridge Assessment is the brand name of University
of Cambridge Local Examinations Syndicate (UCLES), which is itself a department of the University of Cambridge.

© UCLES 2007 0625/02/O/N/07


w
w
w
.X
tr
me
eP
ap
UNIVERSITY OF CAMBRIDGE INTERNATIONAL EXAMINATIONS

er
s
International General Certificate of Secondary Education

.c
om
*3498617057*

PHYSICS 0625/02
Paper 2 Core May/June 2008
1 hour 15 minutes
Candidates answer on the Question Paper.
No Additional Materials are required.

READ THESE INSTRUCTIONS FIRST

Write your Centre number, candidate number and name on all the work you hand in.
Write in dark blue or black pen.
You may use a soft pencil for any diagrams, graphs or rough working.
Do not use staples, paper clips, highlighters, glue or correction fluid.
DO NOT WRITE IN ANY BARCODES.
For Examiner’s Use
Answer all questions.
You may lose marks if you do not show your working or if you do not use 1
appropriate units.
Take the weight of 1 kg to be 10 N (i.e. acceleration of free fall = 10 m/s2). 2

At the end of the examination, fasten all your work securely together. 3
The number of marks is given in brackets [ ] at the end of each question or part
question. 4

10

11

12

Total

This document consists of 14 printed pages and 2 blank pages.

SP (SLM/CGW) T41975/6
© UCLES 2008 [Turn over
2

1 (a) Fig. 1.1 shows a uniform rod. For


Examiner’s
Use

Fig. 1.1

(i) Use your rule to find the length of the rod.

length = ...........................................cm [1]

(ii) On Fig. 1.1, show the position of the centre of mass of the rod using the letter C.
[1]

(b) Fig. 1.2 shows another rod, of the same length as the previous one, but this rod is thicker
at one end.

Fig. 1.2

Use your judgement to mark with the letter M approximately where the centre of mass
of this rod will be.
[2]

[Total: 4]

© UCLES 2008 0625/02/M/J/08


3

2 A motorcyclist is travelling along a country road, as shown in Fig. 2.1. For


Examiner’s
Use

1375 m

Fig. 2.1

The statements below describe the motion of the motorcycle from point A to point B.

1. The motorcycle accelerates uniformly from rest at point A, increasing its speed to
25 m/s in 10 s.
2. It then travels at a constant speed of 25 m/s for 40 s.
3. It then decelerates uniformly to rest at point B, 70 s after leaving point A.

35

30
speed
m/s 25

20

15

10

0
0
time/s

Fig. 2.2

(a) For the motorcycle moving from point A to point B, draw on Fig. 2.2,

(i) a suitable time scale, [1]


(ii) the graph of the motion of the motorcycle. [5]

(b) The distance from A to B is 1375 m.

Calculate the average speed of the motorcycle between A and B. Give your answer to
the nearest m/s.

average speed = ..........................................m/s [4]


[Total: 10]
© UCLES 2008 0625/02/M/J/08 [Turn over
4

3 A beam is pivoted at its centre. Three forces, F1, F2 and F3, act on the beam as shown in For
Fig. 3.1. Examiner’s
Use
b c

a
F1 F2 F3
Fig. 3.1

(a) Which of the forces exert(s)

a clockwise moment, ...................

an anticlockwise moment? ................... [3]

(b) When the beam is released, the right-hand side of the beam starts to go down.
Which of the three distances, a, b or c, should be decreased in order to balance the
beam? Explain your answer.

Which distance? ..............................................................................................................

Explanation

..........................................................................................................................................

..........................................................................................................................................

...................................................................................................................................... [3]

(c) Fig. 3.2 represents a simple beam-balance with the pivot accurately at its centre.

?
Fig. 3.2

The person using the beam-balance puts the object to be weighed in the left-hand pan.
He has a selection of standard masses to put in the right-hand pan, but he finds he
cannot exactly balance the beam.

His best attempts are

masses used effect


10 g, 10 g, 5 g, 2 g, 2 g beam tips down slightly on the left-hand side
20 g, 10 g beam tips down slightly on the right-hand side

Estimate the mass of the object.


mass = ............................................. g [1]
[Total: 7]
© UCLES 2008 0625/02/M/J/08
5

4 For
(a) An electrician climbs up to the platform Examiner’s
of a special tower, in order to reach a Use
high-level spotlight, as shown in Fig. 4.1.

Which form of energy, possessed by the


electrician’s body,

(i) is greater at the top of the tower than


it was at the bottom,

..................................................... [1]

(ii) is less at the top of the tower than it


was at the bottom?

..................................................... [1]

(b) One of the electrician’s assistants also


climbs up to the platform. The assistant
weighs less than the electrician.

Which of the two people does the most


work climbing up to the platform, and
why?

Which person? ........................................

Why? .......................................................

.................................................................

.................................................................

............................................................. [1]

Fig. 4.1

(c) The electrician wishes to know what power he develops as he climbs the tower.

Which quantities does he need to know in order to do this?

..........................................................................................................................................

...................................................................................................................................... [1]

[Total: 4]

© UCLES 2008 0625/02/M/J/08 [Turn over


6

5 In the atomic model, the atom has a central mass. Much smaller particles orbit this central For
mass, as shown in Fig. 5.1. Examiner’s
Use
central
mass

orbiting
particles

Fig. 5.1

(a) State the name given to the central mass.

...................................................................................................................................... [1]

(b) State the name given to the orbiting particles.

...................................................................................................................................... [1]

(c) State the names of the particles from which the central mass is made.

........................................................... and .................................................................... [2]

(d) The central mass of the helium atom is identical to one of the particles emitted in
radioactive decay.

Which particle is this? ................................................................................................... [1]

(e) State the name of the particles that form cathode rays.

...................................................................................................................................... [1]

[Total: 6]

© UCLES 2008 0625/02/M/J/08


7

6 Fig. 6.1 and Fig. 6.2 show two of the rays from the top of an object, passing through a lens. For
Examiner’s
Use

object F2 F1

Fig. 6.1

object F2 F1

Fig. 6.2

(a) On Fig. 6.1, draw the third ray whose path from the top of the object through the lens is
known. [1]

(b) On Fig. 6.2,

(i) copy the ray shown on Fig. 6.1 and complete the diagram to locate the image
formed by the lens, [1]
(ii) mark and label the image. [2]

(c) On Fig. 6.2, indicate clearly where you would position a screen on which to see the
focused image. [1]

[Total: 5]

© UCLES 2008 0625/02/M/J/08 [Turn over


8

7 (a) The table below describes the conditions of the molecules of a substance in each of the For
three states of matter, solid, liquid and gas. Examiner’s
Use

In the right-hand column, write the state of the substance that is described in the left-
hand column.

state in which the


condition of the molecules
substance exists
The molecules are a great distance apart, moving
very rapidly, with negligible interaction. The
substance occupies all the space available.
The molecules are only able to vibrate rapidly about
fixed positions. The substance does not need a
container to maintain its shape.
The molecules move about amongst each other,
with attractive forces between them. The substance
does not necessarily fill its container.
[2]

(b) (i) What is the state of matter just before a substance boils?

.............................................................................................................................. [1]

(ii) Describe what happens to the molecules during boiling.

..................................................................................................................................

.............................................................................................................................. [2]

(iii) State two differences between boiling and evaporating.

1. ..............................................................................................................................

2. .......................................................................................................................... [2]

(c) (i) What is the state of matter just before a substance melts?

.............................................................................................................................. [1]

(ii) Aluminium melts at 660 °C. At what temperature does it freeze?

.............................................................................................................................. [1]

[Total: 9]

© UCLES 2008 0625/02/M/J/08


9

8 (a) The thermometer in Fig. 8.1 is calibrated at two fixed points, and the space between For
these is divided into equal divisions. Examiner’s
Use
-10 0 10 20 30 40 50 60 70 80 90 100 110

Fig. 8.1

A thermometer is being calibrated with the Celsius scale.

(i) 1. Write down another name for the lower fixed point.

.............................................................................................................................. [1]

2. How is this temperature achieved?

..................................................................................................................................

..................................................................................................................................

.............................................................................................................................. [2]

3. What is the temperature of this fixed point?................................................... [1]

(ii) 1. Write down another name for the upper fixed point.

.............................................................................................................................. [1]

2. How is this temperature achieved?

..................................................................................................................................

..................................................................................................................................

.............................................................................................................................. [2]

3. What is the temperature of this fixed point?................................................... [2]

(b) A block of copper and a block of aluminium have identical masses. They both start at
room temperature and are given equal quantities of heat. When the heating is stopped,
the aluminium has a lower temperature than the copper.
Fill in the missing words in the sentence below, to explain this temperature difference.

The aluminium block has a smaller temperature rise than the copper block because the

aluminium block has a larger ................................................ than the copper block. [1]

[Total: 10]

© UCLES 2008 0625/02/M/J/08 [Turn over


10

9 Fuses are often included in circuits. For


Examiner’s
(a) In the space below, draw the circuit symbol for a fuse. Use

[1]

(b) When the statements in the boxes below are put in the correct order, they describe how
a fuse protects a circuit.

A
fuse wire heats up
B
circuit is broken,
so current stops

C
fuse wire melts
D
current becomes
too high

On the line below, list the letters of the four boxes in the correct order.

...................................................................................................................................... [2]

(c) By mistake, a fuse with too high a rated value is put in the fuse-holder in a circuit.

State two possible outcomes of this mistake.

1. .....................................................................................................................................

2. ................................................................................................................................. [2]

[Total: 5]

© UCLES 2008 0625/02/M/J/08


11

10 Fig. 10.1 shows a series circuit. For


Examiner’s
Use

R1 R2
X Y

Fig. 10.1

Resistance R1 = 25 Ω and resistance R2 = 35 Ω. The cell has zero resistance.

(a) Calculate the combined resistance of R1 and R2.

resistance = ........................................... Ω [2]

(b) On Fig. 10.1, use the correct circuit symbol to draw a voltmeter connected to measure
the potential difference between X and Y. [1]

(c) The variable resistor is set to zero resistance. The voltmeter reads 1.5 V.

(i) Calculate the current in the circuit.

current = .................................................. [4]

(ii) State the value of the potential difference across the cell.

potential difference = ............................................ V [1]

© UCLES 2008 0625/02/M/J/08 [Turn over


12

(d) The resistance of the variable resistor is increased. For


Examiner’s
(i) What happens to the current in the circuit? Tick one box. Use

increases

stays the same

decreases [1]

(ii) What happens to the voltmeter reading? Tick one box.

increases

stays the same

decreases [1]

(iii) State the resistance of the variable resistor when the voltmeter reads 0.75 V.

resistance = ............................................Ω [1]

[Total: 11]

© UCLES 2008 0625/02/M/J/08


13

11 (a) An experimenter uses a length of wire ABC in an attempt to demonstrate For


electromagnetic induction. The wire is connected to a sensitive millivoltmeter G. Examiner’s
Use

B
N
A
S G
C

Fig. 11.1

Using the arrangement in Fig. 11.1, the experimenter finds that she does not obtain the
expected deflection on G when she moves the wire ABC down through the magnetic
field.

(i) Explain why there is no deflection shown on G.

..................................................................................................................................

..................................................................................................................................

.............................................................................................................................. [2]

(ii) What change should be made in order to observe a deflection on G?

..................................................................................................................................

.............................................................................................................................. [1]

(b) Name one device that makes use of electromagnetic induction.

...................................................................................................................................... [1]

[Total: 4]

© UCLES 2008 0625/02/M/J/08 [Turn over


14

12 (a) The table below shows how the activity of a sample of a radioactive substance changes For
with time. Examiner’s
Use

activity
time/minutes
counts/s
0 128
30 58
60 25
90 11
120 5

Use the data in the table to estimate the half-life of the radioactive substance.

half-life = .........................................min [2]

(b) The half-lives of various substances are given below.

radon-220 55 seconds
iodine-128 25 minutes
radon-222 3.8 days
strontium-90 28 years

(i) If the radioactive substance in (a) is one of these four, which one is it?

.............................................................................................................................. [1]

(ii) A sample of each of these substances is obtained.

Which sample will have the greatest proportion of decayed nuclei by the end of one
year, and why?

Which? .....................................................................................................................

Why? ........................................................................................................................

.............................................................................................................................. [2]

[Total: 5]

© UCLES 2008 0625/02/M/J/08


15

BLANK PAGE

0625/02/M/J/08
16

BLANK PAGE

Permission to reproduce items where third-party owned material protected by copyright is included has been sought and cleared where possible. Every
reasonable effort has been made by the publisher (UCLES) to trace copyright holders, but if any items requiring clearance have unwittingly been included, the
publisher will be pleased to make amends at the earliest possible opportunity.

University of Cambridge International Examinations is part of the Cambridge Assessment Group. Cambridge Assessment is the brand name of University of
Cambridge Local Examinations Syndicate (UCLES), which is itself a department of the University of Cambridge.

0625/02/M/J/08
UNIVERSITY OF CAMBRIDGE INTERNATIONAL EXAMINATIONS
International General Certificate of Secondary Education
*5545678956*

PHYSICS 0625/02
Paper 2 Core October/November 2008
1 hour 15 minutes
Candidates answer on the Question Paper.
No Additional Materials are required.

READ THESE INSTRUCTIONS FIRST

Write your Centre number, candidate number and name on all the work you hand in.
Write in dark blue or black pen.
You may use a soft pencil for any diagrams, graphs or rough working.
Do not use staples, paper clips, highlighters, glue or correction fluid.
DO NOT WRITE IN ANY BARCODES.
For Examiner’s Use
Answer all questions.
You may lose marks if you do not show your working or if you do not use 1
appropriate units.
Take the weight of 1 kg to be 10 N (i.e. acceleration of free fall = 10 m/s2). 2

At the end of the examination, fasten all your work securely together. 3
The number of marks is given in brackets [ ] at the end of each question or part
question. 4

10

11

12

Total

This document consists of 16 printed pages.

SP (NF/CGW) T41976/4
© UCLES 2008 [Turn over
2

1 A boy is fishing in a river. Nearby, a ferry-boat is taking passengers backwards and forwards For
across the river. Examiner’s
Use

12 12
11 1 11 1
10 2 10 2

9 3 9 3

8 4 8 4
7 5 7 5
6 6

Fig. 1.1 Fig. 1.2

When he starts fishing one morning, the boy’s wristwatch is as shown in Fig. 1.1.
When he finishes his morning’s fishing, the wristwatch is as shown in Fig. 1.2.

(a) For how many minutes was the boy fishing?

number of minutes = ................................................ [3]

(b) It takes the ferry-boat 20 minutes to load up passengers, take them across the river,
load up with passengers at the other side, and return, to start all over again.
Calculate how many journeys, across the river and back, the ferry made whilst the boy
was fishing.

number of journeys = ................................................ [2]

[Total: 5]

© UCLES 2008 0625/02/O/N/08


3

2 The tank in Fig. 2.1 measures 2.0 m × 1.5 m × 1.0 m. It is full of water. For
Examiner’s
Use
water

Fig. 2.1

(a) Calculate the volume of the tank.

volume = ................................................ [3]

(b) Water has a density of 1000 kg/m3.


Calculate the mass of water in the tank.

mass = ................................................ [3]

[Total: 6]

© UCLES 2008 0625/02/O/N/08 [Turn over


4

3 A spring is hung from a beam, and a load is suspended from it. For
Fig. 3.1 shows the spring before and after suspending the load. Examiner’s
Use

before after
W
X

load

Fig. 3.1

(a) Using the letters on Fig. 3.1, state which distance you would need to measure in order
to find the extension of the spring.

distance = ................................................ [1]

(b) An experimenter measures the extension of the spring with different loads, and obtains
the graph shown in Fig. 3.2.

extension / cm

0
0 1 2 3 4 load / N 5

Fig. 3.2

(i) When the experimenter hangs an unknown load on the spring, the extension
of the spring is 1.7 cm.
Use Fig. 3.2 to find the weight of the unknown load.

weight = ............................................ N [1]

© UCLES 2008 0625/02/O/N/08


5

(ii) State the value of the tension in the spring when this load is hanging from it. For
Examiner’s
tension = ................................................ [2] Use

(iii) The load is pulled down further.


State what happens to the tension in the spring.

............................................................................................................................ [1]

(iv) After being pulled down, the load is released.


State what happens to the load immediately after it is released.

..................................................................................................................................

............................................................................................................................ [1]

[Total: 6]

© UCLES 2008 0625/02/O/N/08 [Turn over


6

4 Fig. 4.1 represents the regions of the electromagnetic spectrum, with an enlargement of the For
visible light part of the spectrum. Examiner’s
Use

micro- infra visible X-rays γ -rays


waves -red

visible

M N

Fig. 4.1

(a) Two of the regions have not been named in Fig. 4.1.
In the two boxes below the spectrum, write the names of these regions. [2]

(b) Write “long wavelength” next to the long wavelength end of the electromagnetic
spectrum. [1]

(c) State one use for the radiation of each of the following regions.

microwaves ......................................................................................................................

infra-red ...........................................................................................................................

γ-rays ...............................................................................................................................
[3]

(d) State the colour you would expect to find

(i) at end M of the visible spectrum, ..............................................................................

(ii) at end N of the visible spectrum. ..............................................................................


[2]

[Total: 8]

© UCLES 2008 0625/02/O/N/08


7

5 (a) Make the following sentences about sound echoes correct by crossing out the incorrect For
alternatives. An example has been given to help you. Examiner’s
Use

large
(example) An echo is caused when a sound wave hits a small obstacle.
thin

reflected
(i) An echo is a sound wave which is refracted by an obstacle. [1]
dispersed

(ii) An echo from a stationary obstacle has


a greater frequency than
the same frequency as
a smaller frequency than
the sound hitting the obstacle. [1]

(b) Two students, A and B, are trying to use echoes to enable them to measure the speed
of sound. Student A has two blocks of wood that make a loud sound when banged
together. Student B has a stopwatch. They stand 240 m from the school wall, as shown
in Fig. 5.1.

school
wall

B A
240 m

Fig. 5.1

Student A bangs the blocks together and, at the same time, B starts the stopwatch.
B stops the stopwatch when he hears the echo. The watch then reads 1.6 s.

(i) How far did the sound travel during the 1.6 s? ................................ m [1]

(ii) Calculate the speed of sound.

speed of sound = ......................................... m/s [3]

© UCLES 2008 0625/02/O/N/08 [Turn over


8

(iii) The students expected that the value for the speed of sound would be closer to For
330 m/s. Examiner’s
Use

Suggest two reasons that might explain why the students’ value was different from
what was expected.

1. ...............................................................................................................................

2. ......................................................................................................................... [2]

[Total: 8]

6 Fig. 6.1 shows two mercury barometers standing side by side. The right-hand diagram is
incomplete. The space labelled X is a vacuum.

glass
tube

mercury

dish

Fig. 6.1

(a) On the left-hand barometer, carefully mark the distance that would have to be measured
in order to find the value of the atmospheric pressure. [2]

(b) A small quantity of air is introduced into X.

(i) State what happens to the mercury level in the tube.

............................................................................................................................ [1]

(ii) In terms of the behaviour of the air molecules, explain your answer to (b)(i).

..................................................................................................................................

............................................................................................................................ [2]

(c) The space above the mercury in the right-hand barometer is a vacuum.
On Fig. 6.1, mark the level of the mercury surface in the tube. [1]
© UCLES 2008 0625/02/O/N/08
9

(d) The left-hand tube now has air above the mercury; the right-hand tube has a vacuum. For
Complete the table below, using words chosen from the following list, to indicate the Examiner’s
effect of changing the external conditions. Use

rises falls stays the same

effect on the level of the effect on the level of the


change
mercury in the left-hand tube mercury in the right-hand tube

atmospheric pressure
rises

temperature
rises
[4]

[Total: 10]

7 This question requires the use of a ruler or other straight edge. Fig. 7.1 shows a ray of light
striking a plane mirror.

ray of
light

mirror

Fig. 7.1

(a) On Fig. 7.1,

(i) draw the normal to the mirror at A, [1]


(ii) draw the path of the reflected ray, [1]
(iii) mark and label the angle of incidence, i, [1]
(iv) mark and label the angle of reflection, r. [1]

(b) State the equation that links i and r.

.................................................................................................................................... [1]

(c) On Fig. 7.1

(i) draw another mirror which is perpendicular to the one in the diagram and 2 cm to
the right of A, [1]
(ii) draw the path of the ray of light after reflection from this second mirror. [1]

[Total: 7]
© UCLES 2008 0625/02/O/N/08 [Turn over
10

8 (a) An iron rod is placed next to a bar magnet, as shown in Fig. 8.1. For
Examiner’s
Use
N S
iron rod

Fig. 8.1

(i) On Fig. 8.1, mark clearly the north pole and the south pole that are induced in the
iron rod. [1]
(ii) What happens to the magnet and the rod? Tick one box.

nothing

they attract

they repel [1]

(b) A second bar magnet is now placed next to the iron rod, as shown in Fig. 8.2.

N S N S
iron rod

Fig. 8.2

(i) On Fig. 8.2, mark clearly the magnetic poles induced in the iron rod. [1]
(ii) What happens to the iron rod and the second magnet? Tick one box.

nothing

they attract

they repel [1]

(c) The iron rod is removed, leaving the two magnets, as shown in Fig. 8.3.

N S N S

Fig. 8.3

What happens to the two magnets? Tick one box.

nothing

they attract

they repel [1]

© UCLES 2008 0625/02/O/N/08


11

(d) The second magnet is removed and replaced by a charged plastic rod, as shown in For
Fig. 8.4. Examiner’s
Use

N S + –
charged
plastic rod

Fig. 8.4

What happens to the magnet and the plastic rod? Tick one box.

nothing

they attract

they repel [1]

[Total: 6]

© UCLES 2008 0625/02/O/N/08 [Turn over


12

9 A toymaker wires a circuit in a toy house, so that a 6 V lamp can be switched on using either For
switch 1 at the bottom of the stairs or switch 2 at the top of the stairs. Examiner’s
The circuit is shown in Fig. 9.1. Use

X Y

6V switch 2
lamp
a

6 V supply

switch 1
A

Fig. 9.1

(a) When switch 1 is in position A, what is the position of switch 2 so that the lamp is lit?

.................................................................................................................................... [1]

(b) When switch 2 is in position b, what is the position of switch 1 so that the lamp is lit?

.................................................................................................................................... [1]

(c) The lamp has a resistance of 4.0 Ω.

Calculate the current in the circuit.

current = ................................................ [4]

© UCLES 2008 0625/02/O/N/08


13

(d) The toymaker decides he wants to have two 6 V lamps in the circuit with the 6 V supply. For
Examiner’s
Which of the following means of connecting the lamps between X and Y, shown in Use

Fig. 9.2, is the best? Give two reasons for your answer.

X Y X Y

connection 1 connection 2

Fig. 9.2

Which is best? .................................................................................................................

Reason 1 .........................................................................................................................

Reason 2 ................................................................................................................... [3]

[Total: 9]

© UCLES 2008 0625/02/O/N/08 [Turn over


14

10 The circuit in Fig. 10.1 shows an electromagnetic relay being used to switch an electric motor For
on and off. The relay coil has a much greater resistance than the potential divider. Examiner’s
Use

pivoted iron
armature
power supply
6V for motor

M motor
switch

relay core contacts

Fig. 10.1

(a) The relay operates when there is a potential difference of 3 V across the coil.
On Fig. 10.1, mark the position of the slider of the potential divider when the relay just
operates. [1]

(b) Describe how the relay closes the contacts in the motor circuit.

..........................................................................................................................................

..........................................................................................................................................

..........................................................................................................................................

..........................................................................................................................................

.................................................................................................................................... [3]

[Total: 4]

© UCLES 2008 0625/02/O/N/08


15

11 Fig. 11.1 shows a transformer being used to operate a 6 V lamp from a 240 V mains supply. For
Examiner’s
Use
A

240V 6V
mains lamp

8000 turns iron

Fig. 11.1

(a) What name is given to the piece of iron that links the two coils?

.................................................................................................................................... [1]

(b) For the transformer to operate, must the mains supply be a.c., d.c., or doesn’t it
matter?

.................................................................................................................................... [1]

(c) Using the information on Fig. 11.1, calculate the number of turns on the secondary coil.

number of turns = ................................................ [3]

(d) State why it would not be wise to connect a 1.5 V lamp between A and B.

.................................................................................................................................... [1]

[Total: 6]

© UCLES 2008 0625/02/O/N/08 [Turn over


16

12 The nucleus of one of the different nuclides of polonium can be represented by For
the symbol Examiner’s
Use
218
84 Po

(a) State the proton number of this nuclide. ........................ [1]

(b) State the nucleon number of this nuclide. ........................ [1]

(c) The nucleus decays according to the following equation.

218 214
84 Po 82 Pb + emitted particle

(i) State the proton number of the emitted particle. .......................... [1]

(ii) State the nucleon number of the emitted particle. .......................... [1]

(iii) Name the emitted particle. Tick one box.

α-particle

β-particle

neutron

proton [1]

[Total: 5]

Permission to reproduce items where third-party owned material protected by copyright is included has been sought and cleared where possible. Every
reasonable effort has been made by the publisher (UCLES) to trace copyright holders, but if any items requiring clearance have unwittingly been included, the
publisher will be pleased to make amends at the earliest possible opportunity.

University of Cambridge International Examinations is part of the Cambridge Assessment Group. Cambridge Assessment is the brand name of University of
Cambridge Local Examinations Syndicate (UCLES), which is itself a department of the University of Cambridge.

© UCLES 2008 0625/02/O/N/08


UNIVERSITY OF CAMBRIDGE INTERNATIONAL EXAMINATIONS
International General Certificate of Secondary Education
*4743961177*

PHYSICS 0625/02
Paper 2 Core May/June 2009
1 hour 15 minutes
Candidates answer on the Question Paper.
No Additional Materials are required.

READ THESE INSTRUCTIONS FIRST

Write your Centre number, candidate number and name on all the work you hand in.
Write in dark blue or black pen.
You may use a soft pencil for any diagrams, graphs or rough working.
Do not use staples, paper clips, highlighters, glue or correction fluid. For Examiner’s Use
DO NOT WRITE IN ANY BARCODES.
1
Answer all questions.
You may lose marks if you do not show your working or if you do not use 2
appropriate units.
Take the weight of 1 kg to be 10 N (i.e. acceleration of free fall = 10 m/s2). 3
At the end of the examination, fasten all your work securely together. 4
The number of marks is given in brackets [ ] at the end of each question or
part question. 5

10

11

12

Total

This document consists of 19 printed pages and 1 blank page.

SPA SHW 00151 2/08 T76303/3


© UCLES 2009 [Turn over
2

1 A schoolteacher is concerned about the large number of vehicles passing along the busy For
road next to the school. He decides to make a measurement to find the number of vehicles Examiner’s
per minute. Using the school clock he notes the following readings. Use

12
11 1
10 2

appearance of clock at beginning of investigation 9 3

8 4
7 5
6

12
11 1
10 2

appearance of clock at end of investigation 9 3

8 4
7 5
6

vehicles counted travelling left to right = 472

vehicles counted travelling right to left = 228

(a) Calculate the time for which the schoolteacher was counting vehicles.

counting time = ......................................... min [1]

(b) Calculate the total number of vehicles passing the school per minute.

vehicles per minute = ................................................ [3]

[Total: 4]

© UCLES 2009 0625/02/M/J/09


3

2 In the left-hand column below are four physical quantities that might be measured in the For
Physics laboratory. In the right-hand column are eight statements which might be definitions Examiner’s
of the quantities in the left-hand column. Use

Use a line to join each quantity with its definition. An example has been given to help you.
There is only one definition for each quantity.

work force of gravity on a body

how big the body is

mass power of a given force

weight ÷ mass

weight amount of matter in a body

force × distance moved

density mass ÷ volume

the acceleration due to gravity


[3]

[Total: 3]

© UCLES 2009 0625/02/M/J/09 [Turn over


4

3 The car in Fig. 3.1 is on a level road. For


Examiner’s
Use
500 N 2000 N force
frictional forces propelling car forwards

Fig. 3.1

(a) Calculate the magnitude of the resultant force on the car.

resultant force = ............................................. N [1]

(b) Tick the box below that describes the motion of the car.

travels forward at constant speed

travels forward with increasing speed

travels forward with decreasing speed

travels backward at constant speed

travels backward with increasing speed

travels backward with decreasing speed

remains at rest
[1]

© UCLES 2009 0625/02/M/J/09


5

(c) The frictional forces increase to 2000 N when the car is moving. What happens to the For
car? Examiner’s
Use

.................................................................................................................................... [1]

(d) Suggest two things that might have caused the frictional forces in (c) to increase.

1. ......................................................................................................................................

2. ................................................................................................................................ [2]

[Total: 5]

© UCLES 2009 0625/02/M/J/09 [Turn over


6

4 Fig. 4.1 shows a manometer, containing mercury, being used to monitor the pressure of a For
gas supply. Examiner’s
Use

mm
300

250

from a
gas supply 200

150

100

50

mercury
0

Fig. 4.1

(a) Using the scale on Fig. 4.1, find the vertical difference between the two mercury levels.

difference = ......................................... mm [1]

(b) What is the value of the excess pressure of the gas supply, measured in millimetres of
mercury?

excess pressure = ....................... mm of mercury [1]

(c) The atmospheric pressure is 750 mm of mercury.

Calculate the actual pressure of the gas supply.

actual pressure = ....................... mm of mercury [1]

(d) The gas pressure now decreases by 20 mm of mercury.

On Fig. 4.1, mark the new positions of the two mercury levels. [2]

[Total: 5]

© UCLES 2009 0625/02/M/J/09


7

5 Fig. 5.1 shows a typical laboratory thermometer. For


Examiner’s
Use
°C –10 0 10 20 30 40 50 60 70 80 90 100 110

Fig. 5.1

(a) Explain why the thermometer has

(i) thin walls on its bulb,

..................................................................................................................................

............................................................................................................................ [1]

(ii) thick walls on its stem,

..................................................................................................................................

............................................................................................................................ [1]

(iii) a narrow capillary tube along which the liquid expands.

..................................................................................................................................

............................................................................................................................ [1]

(b) Suggest a liquid which the thermometer might contain.

.................................................................................................................................... [1]

(c) Such a thermometer is calibrated at the ice point and the steam point. The scale is put
on between these two marks.

State the values of

(i) the ice point, ......................................

(ii) the steam point. .................................


[2]

[Total: 6]

© UCLES 2009 0625/02/M/J/09 [Turn over


8

6 Fig. 6.1 shows the waveform of the note from a bell. A grid is given to help you take For
measurements. Examiner’s
Use

time

Fig. 6.1

(a) (i) State what, if anything, is happening to the loudness of the note.

............................................................................................................................ [1]

(ii) State how you deduced your answer to (a)(i).

............................................................................................................................ [1]

(b) (i) State what, if anything, is happening to the frequency of the note.

............................................................................................................................ [1]

(ii) State how you deduced your answer to (b)(i).

............................................................................................................................ [1]

© UCLES 2009 0625/02/M/J/09


9

(c) (i) How many oscillations does it take for the amplitude of the wave to decrease to half For
its initial value? Examiner’s
Use

............................................................................................................................ [1]

(ii) The wave has a frequency of 300 Hz.

1. What is meant by a frequency of 300 Hz ?

...........................................................................................................................

..................................................................................................................... [1]

2. How long does 1 cycle of the wave take?

..................................................................................................................... [1]

3. How long does it take for the amplitude to decrease to half its initial value?

..................................................................................................................... [2]

(d) A student says that the sound waves, which travelled through the air from the bell, were
longitudinal waves, and that the air molecules moved repeatedly closer together and
then further apart.

(i) Is the student correct in saying that the sound waves are longitudinal? .................

(ii) Is the student correct about the movement of the air molecules? .................

(iii) The student gives light as another example of longitudinal waves.

Is this correct? .................


[2]

[Total: 11]

© UCLES 2009 0625/02/M/J/09 [Turn over


10

7 Two apartment blocks are one each side of a road, as shown in Fig. 7.1. A beam of light from For
a police helicopter is hitting the top window H of the left-hand apartment block. Examiner’s
Use

beam of light

H Z

G Y

F X

E W

D V

C U

B T

apartment apartment
A S
block block

Fig. 7.1

(a) (i) On Fig. 7.1,

1. draw the normal at the point where the beam hits window H, [1]

2. label the angle of incidence of the beam of light on window H. [1]

© UCLES 2009 0625/02/M/J/09


11

(ii) State the equation that links the angle of incidence with the angle of reflection. For
Examiner’s
................................................. [1] Use

(iii) Which window does the beam hit next, after reflection from H?

................................................. [1]

(iv) Which other windows, if any, receive light from the helicopter?

................................................. [1]

(b) Fig. 7.2 shows another example of reflection. The drawing is incomplete.

mirror

card

Fig. 7.2

The horizontal card with the letter P on it is being reflected in the vertical mirror.

On Fig. 7.2, draw the reflection of the letter P. [2]

[Total: 7]

© UCLES 2009 0625/02/M/J/09 [Turn over


12

8 (a) Four rods are shown in Fig. 8.1. For


Examiner’s
Use

plastic rod iron rod wooden rod brass rod

Fig. 8.1

State which of these could be held in the hand at one end and be

(i) magnetised by stroking it with a magnet,

................................................. [1]

(ii) charged by stroking it with a dry cloth.

................................................. [1]

(b) Magnets A and B in Fig. 8.2 are repelling each other.

N
magnet A magnet B

Fig. 8.2

The north pole has been labelled on magnet A.

On Fig. 8.2, label the other three poles. [1]

© UCLES 2009 0625/02/M/J/09


13

(c) Charged rods C and D in Fig. 8.3 are attracting each other. For
Examiner’s
Use

+
rod C rod D

Fig. 8.3

On Fig. 8.3, show the charge on rod D. [1]

(d) Fig. 8.4 shows a plotting compass with its needle pointing north.

Fig. 8.4

A brass rod is positioned in an east-west direction. A plotting compass is put at each


end of the brass rod, as shown in Fig. 8.5.

brass rod N
plotting
compass

Fig. 8.5

On Fig. 8.5, mark the position of the pointer on each of the two plotting compasses. [2]

[Total: 6]

© UCLES 2009 0625/02/M/J/09 [Turn over


14

9 Fig. 9.1 shows a simple circuit. For


Examiner’s
Use
6V

reading
A
50 mA

Fig. 9.1

(a) What is the value of

(i) the e.m.f. of the battery,

................................................. [1]

(ii) the current in the circuit?

................................................. [1]

(b) Calculate the resistance R of the resistor.

R = ................................................ [3]

(c) State how the circuit could be changed to

(i) halve the current in the circuit,

............................................................................................................................ [2]

(ii) reduce the current to zero.

............................................................................................................................ [1]

© UCLES 2009 0625/02/M/J/09


15

(d) A student wishes to include a switch in the circuit, but mistakenly connects it as shown For
in Fig. 9.2. Examiner’s
Use

6V

student’s
A incorrect
connection
R

Fig. 9.2

(i) Comment on the size of the current in the circuit if the student closes the switch.

............................................................................................................................ [1]

(ii) What effect would this current have on the circuit?

..................................................................................................................................

..................................................................................................................................

............................................................................................................................ [2]

[Total: 11]

© UCLES 2009 0625/02/M/J/09 [Turn over


16

10 The apparatus in Fig. 10.1 is called a force-on-conductor balance. When there is an electric For
current I as shown in XY, there is a force on XY that pulls it down. This force is measured by Examiner’s
putting weights in the pan until XY is brought back to its original position. Use

pan

current
out

current
in
Y

N I
X

Fig. 10.1

(a) State what would happen if the current direction were from Y to X.

.................................................................................................................................... [1]

(b) An experimenter uses the balance to determine the force F on XY for different currents I.
His results are given below.

I/A 0 0.5 1.0 1.5 2.0

F/N 0 0.012 0.023 0.035 0.047

(i) On the grid of Fig. 10.2,

1. mark suitable scales to plot a graph of F / N against I / A for these values, [2]

2. plot the points on your grid, [2]

3. draw the best straight line through your points. [1]

© UCLES 2009 0625/02/M/J/09


17

For
F/N Examiner’s
Use

0
0 I/A

Fig. 10.2

(ii) From your graph, find the force on XY when the current is 1.6 A.

force = ............................................. N [1]

(c) Name one common device that uses the effect demonstrated by the force-on-conductor
balance.

.................................................................................................................................... [1]

[Total: 8]

© UCLES 2009 0625/02/M/J/09 [Turn over


18

11 Fig. 11.1 represents a cathode-ray tube containing a number of different parts. There are For
empty boxes connected to four of the parts. These boxes are for some of the answers to this Examiner’s
question. Not all of the boxes will be used. Use

heater filament

cathode rays

anode

grid

Fig. 11.1

(a) On Fig. 11.1, write the word CATHODE in the appropriate box. [1]

(b) One part shown in Fig. 11.1 is used to deflect the cathode rays up and down.

Write UP AND DOWN in the appropriate box. [1]

(c) One part glows when the cathode rays strike it.

Write GLOWS in the appropriate box. [1]

(d) On Fig. 11.1, draw a battery connected so that the cathode is heated. [1]

(e) Name the particles that make up cathode rays. ................................................ [1]

(f) What fills the rest of the space in the cathode-ray tube? Tick one box.

air

alpha particles

hydrogen

steam

vacuum
[1]

[Total: 6]
© UCLES 2009 0625/02/M/J/09
19

12 The following table includes some of the properties of alpha, beta and gamma radiations. For
Examiner’s
Complete the table by filling in the missing properties. Use

alpha beta gamma

2 protons +
nature [2]
2 neutrons

approximate mass 1 unit [2]

charge positive [2]

ability to penetrate
very penetrating [2]
solids

[Total: 8]

© UCLES 2009 0625/02/M/J/09


20

BLANK PAGE

Permission to reproduce items where third-party owned material protected by copyright is included has been sought and cleared where possible. Every
reasonable effort has been made by the publisher (UCLES) to trace copyright holders, but if any items requiring clearance have unwittingly been included, the
publisher will be pleased to make amends at the earliest possible opportunity.

University of Cambridge International Examinations is part of the Cambridge Assessment Group. Cambridge Assessment is the brand name of University of
Cambridge Local Examinations Syndicate (UCLES), which is itself a department of the University of Cambridge.

0625/02/M/J/09
UNIVERSITY OF CAMBRIDGE INTERNATIONAL EXAMINATIONS
International General Certificate of Secondary Education
*3735851861*

PHYSICS 0625/02
Paper 2 Core October/November 2009
1 hour 15 minutes
Candidates answer on the Question Paper.
No Additional Materials are required.

READ THESE INSTRUCTIONS FIRST

Write your Centre number, candidate number and name on all the work you hand in.
Write in dark blue or black pen.
You may use a soft pencil for any diagrams, graphs or rough working.
Do not use staples, paper clips, highlighters, glue or correction fluid.
DO NOT WRITE IN ANY BARCODES.

Answer all questions.


You may lose marks if you do not show your working or if you do not use appropriate units.
Take the weight of 1 kg to be 10 N (i.e. acceleration of free fall = 10 m/s2).

At the end of the examination, fasten all your work securely together.
The number of marks is given in brackets [ ] at the end of each question or
part question.

This document consists of 19 printed pages and 1 blank page.

DCA (SHW 00151 2/08) 12741/5


© UCLES 2009 [Turn over
2

1 Fig. 1.1 shows a measuring cylinder containing some water. For


Examiner’s
A student allows 200 drops of water to fall into the water in the measuring cylinder. Fig. 1.2 Use

shows the measuring cylinder after the addition of the drops.

cm3 cm3
50 50

40 40

30 30
final level
20 original level 20 of water
of water
10 10

Fig. 1.1 Fig. 1.2

(a) State

(i) the original volume of water in the cylinder,

................................................ cm3

(ii) the final volume of water in the cylinder.

................................................ cm3
[1]

(b) Calculate the volume of water added.

volume added = ......................................... cm3 [1]

(c) Calculate the average volume of one of the drops of water.

average volume = ......................................... cm3 [2]

[Total: 4]

© UCLES 2009 0625/02/O/N/09


3

2 As a pole vaulter runs towards the jump, his pole is straight. When he is jumping, the pole is For
bent, as illustrated in Fig. 2.1. Examiner’s
Use

pole

Fig. 2.1

(a) Which form of energy of the athlete increases as he accelerates towards the jump?

.................................................................................................................................... [1]

(b) Which form of energy is stored in the pole because it is bent?

.................................................................................................................................... [1]

(c) Which form of energy of the athlete increases because he is rising towards the bar?

.................................................................................................................................... [1]

(d) Which two quantities need to be known in order to calculate how much work is done
lifting the athlete up from the ground to the bar?

............................................................. and .............................................................. [1]

[Total: 4]

© UCLES 2009 0625/02/O/N/09 [Turn over


4

3 Fig. 3.1 shows the distance/time graph for a girl’s bicycle ride and the axes for the For
corresponding speed/time graph. Examiner’s
Use

distance
from
starting
point

0A B C D time

speed

0
A B C D time

Fig. 3.1

(a) Look at the distance/time graph that has been drawn for you.

(i) Answer the following questions for the time interval AB.

1. What is happening to the distance from the starting point?

...........................................................................................................................

..................................................................................................................... [2]

2. What can you say about the speed of the bicycle?

..................................................................................................................... [1]

© UCLES 2009 0625/02/O/N/09


5

(ii) On the speed/time axes in the lower part of Fig. 3.1, draw a thick line that could For
show the speed during AB. [1] Examiner’s
Use

(b) On the speed/time axes of Fig. 3.1

(i) draw a thick line that could show the speed during BC, [1]

(ii) draw a thick line that could show the speed during CD. [2]

(c) How far from her starting point is the girl when she has finished her ride?

distance from starting point = ................................................ [1]

[Total: 8]

© UCLES 2009 0625/02/O/N/09 [Turn over


6

4 (a) The object illustrated in Fig. 4.1 is not in equilibrium. It has a weight of 12 N. For
Examiner’s
Use
12 N

15 N 6N

weight = 12 N

Fig. 4.1

(i) State what happens to the object.

..................................................................................................................................

............................................................................................................................ [2]

(ii) On Fig. 4.1, draw an arrow to show the extra force necessary to bring the object to
a state of equilibrium. Label the arrow with the size of the force. [2]

(iii) On Fig. 4.1, show where the centre of mass of the object is situated, using the
letter G. [1]

© UCLES 2009 0625/02/O/N/09


7

(b) Fig. 4.2 shows a car transporter, first unloaded and then loaded with two cars on the For
upper deck. Examiner’s
Use

Fig. 4.2

(i) What happens to the centre of mass of the transporter and its load when the cars
are loaded?

Tick one box.

rises

stays at the same height

falls
[1]

(ii) How do the two loaded cars affect the stability of the transporter?

Tick one box.

more stable

no effect

less stable
[1]

[Total: 7]

© UCLES 2009 0625/02/O/N/09 [Turn over


8

5 (a) Fig. 5.1 shows the cooling curve for a pure substance. The substance is liquid at A. For
Examiner’s
Use
temperature
A

C
B

time

Fig. 5.1

(i) Describe what is happening to the substance between

1. A and B, . ..........................................................................................................

2. B and C, ...........................................................................................................

3. C and D. ...........................................................................................................
[3]

(ii) During which of the sections of the graph is energy being lost from the substance to
the surroundings? Put a tick in any of the boxes to indicate where this is happening.

between A and B

between B and C

between C and D
[2]

(iii) What is the state of the substance at D?

................................................. [1]

© UCLES 2009 0625/02/O/N/09


9

(b) When the temperature reaches that at D, the substance is then heated steadily until it For
again reaches the temperature it had at A. Examiner’s
Use

On the axes of Fig. 5.2, sketch a graph of temperature against time for the heating of the
substance. [2]

temperature

time

Fig. 5.2

[Total: 8]

© UCLES 2009 0625/02/O/N/09 [Turn over


10

6 A bicycle pump has its outlet sealed with a tight rubber bung, as shown in Fig. 6.1. For
Examiner’s
Use
handle

bung B A
air piston

Fig. 6.1

(a) The temperature of the air in the pump is kept constant. The handle of the pump is
pushed in so that the piston moves from A to B.

Tick the correct box in each row of the table below to show how each quantity varies as
the piston is moved.

greater with greater with same with


piston at A piston at B piston at A or B

the average speed of the air molecules

the frequency with which the air


molecules hit the walls and the piston

the pressure of the air in the pump

[3]

(b) When the handle is pushed in even further, the rubber bung pops out of the hole.

Which two forces are equal just before the bung pops out? Tick two boxes.

friction force between bung and hole

resultant due to forces of air on each side of piston

resultant due to forces of air on each side of bung

gravitational force on bung

force of air on walls


[2]

[Total: 5]

© UCLES 2009 0625/02/O/N/09


11

7 Fig. 7.1 shows an experiment in which an image is being formed on a card by a lens and a For
plane mirror. Examiner’s
Use

lens image

torch
p q
plane
mirror
hole cut
in card

Fig. 7.1

The card and the mirror are shown angled, so that you can see what is happening. In a real
experiment they are each roughly perpendicular to the line joining the torch bulb and the
centre of the lens.

(a) State which of the three marked distances, p, q and r, is the focal length of the lens.

................................................. [1]

(b) On Fig. 7.1 clearly mark a principal focus of the lens, using the letter F. [1]

(c) Tick the boxes alongside two features that describe the image formed on the card.

erect

inverted

real

virtual [2]

(d) What can be said about the size of the image, compared with the size of the object?

.................................................................................................................................... [1]

(e) In the experiment, the plane mirror is perpendicular to the beam of light.

State what, if anything, happens to the image on the card if

(i) the plane mirror is moved slightly to the left,

............................................................................................................................ [1]

(ii) the lens is moved slightly to the left.

............................................................................................................................ [1]

[Total: 7]
© UCLES 2009 0625/02/O/N/09 [Turn over
12

8 Fig. 8.1 shows a student standing midway between a bell tower and a steep mountainside. For
Examiner’s
Use

steep
mountainside
student

bell tower
and bell
990 m 990 m

Fig. 8.1

The bell rings once, but the student hears two rings separated by a short time interval.

(a) Explain why the student hears two rings.

..........................................................................................................................................

..........................................................................................................................................

.................................................................................................................................... [2]

(b) State which of the sounds is loudest, and why.

..........................................................................................................................................

..........................................................................................................................................

.................................................................................................................................... [2]

© UCLES 2009 0625/02/O/N/09


13

(c) Sound in that region travels at 330 m / s. For


Examiner’s
(i) Calculate the time interval between the bell ringing and the student hearing it for Use

the first time.

time interval = ............................................. s [2]

(ii) Calculate the time interval between the bell ringing and the student hearing it for
the second time.

time interval = ............................................. s [1]

(iii) Calculate the time interval between the two sounds.

time interval = ............................................. s [1]

[Total: 8]

© UCLES 2009 0625/02/O/N/09 [Turn over


14

9 (a) The two circuits shown in Fig. 9.1 each contain two resistors and a battery. For
Examiner’s
Complete the statement under each circuit. Use

R1 R3

R2 R4

Resistors R1 and R2 are Resistors R3 and R4 are

connected in ………………………… connected in ………………………… [1]

Fig. 9.1

(b) A student connects the circuit shown in Fig. 9.2. The resistance of the 12 V battery is so
low that it can be ignored.

280 Ω
12 V

200 Ω

Fig. 9.2

(i) Calculate the total resistance in the circuit.

resistance = ............................................ Ω [2]

© UCLES 2009 0625/02/O/N/09


15

(ii) Calculate the current delivered by the battery. For


Examiner’s
Use

current = ................................................ [4]

(iii) Calculate the potential difference (p.d.) across the 200 Ω resistor.

p.d. = ............................................. V [2]

(iv) Describe how the student could check whether the p.d. across the 200 Ω resistor is
the same as you have calculated. Include the name of the instrument he would use
for this.

..................................................................................................................................

..................................................................................................................................

............................................................................................................................ [2]

[Total: 11]

© UCLES 2009 0625/02/O/N/09 [Turn over


16

10 Fig. 10.1 is a diagram of a transformer. For


Examiner’s
Use

30 turns 300 turns

12 V a.c. voltmeter
V
a.c.

Fig. 10.1

(a) (i) On Fig. 10.1, clearly label the core of the transformer. [1]

(ii) Name a suitable material from which the core could be made.

............................................................................................................................ [1]

(iii) State the purpose of the core.

............................................................................................................................ [1]

(b) Calculate the reading on the voltmeter.

voltmeter reading = ............................................. V [3]

[Total: 6]

© UCLES 2009 0625/02/O/N/09


17

11 A woodworker buys an old electric drill that has been used before. The drill is illustrated in For
Fig. 11.1. Examiner’s
Use

Fig. 11.1

The drill mechanism itself is guaranteed to be electrically safe.

Suggest three checks the woodworker could make in order to ensure that the rest of his
purchase is electrically safe.

1. .............................................................................................................................................

.................................................................................................................................................

2. .............................................................................................................................................

.................................................................................................................................................

3. .............................................................................................................................................

........................................................................................................................................... [3]

[Total: 3]

© UCLES 2009 0625/02/O/N/09 [Turn over


18

12 A radioactive material, X, has a half-life of 2 minutes. For


Examiner’s
At the beginning of an experiment, a sample of the material produces a count-rate of Use

800 counts / s.

Fig. 12.1 shows the count-rate readings at the start of the experiment and after one half-life
has elapsed.

900

800

700

600

500
count-rate
counts / s
400

300

200

100

0
0 2 4 6 8 10 12 14
time t / minutes

Fig. 12.1

© UCLES 2009 0625/02/O/N/09


19

(a) Complete the plot of the decay curve, for times up to t = 12 minutes. Draw the best curve For
through your points. [3] Examiner’s
Use

(b) From your graph find

(i) the count-rate from the sample after 9 minutes,

count-rate = ................................. counts / s [1]

(ii) the time it takes for the count-rate to fall from 700 counts / s to 350 counts / s.

time = ................................... minutes [1]

(c) A second sample of this radioactive material X gives an initial count-rate of


400 counts / s.

What will be the half-life of this sample?

half-life = ................................... minutes [1]

(d) A river contains fine particles of mud. The mud builds up on the bottom of the river and
prevents the movement of large ships. To solve this problem, the authorities remove the
mud and dump it at sea. To check where the mud goes after it is dumped, a radioactive
material is mixed with it and tracked with radioactivity detectors.

(i) Explain why the radioactive material X is not suitable to trace the movement of the
dumped mud.

..................................................................................................................................

............................................................................................................................ [1]

(ii) State two other properties that a radioactive material must have to make it suitable
for tracking the mud.

1. ...............................................................................................................................

2. ......................................................................................................................... [2]

[Total: 9]

© UCLES 2009 0625/02/O/N/09


20

BLANK PAGE

Permission to reproduce items where third-party owned material protected by copyright is included has been sought and cleared where possible. Every
reasonable effort has been made by the publisher (UCLES) to trace copyright holders, but if any items requiring clearance have unwittingly been included, the
publisher will be pleased to make amends at the earliest possible opportunity.

University of Cambridge International Examinations is part of the Cambridge Assessment Group. Cambridge Assessment is the brand name of University of
Cambridge Local Examinations Syndicate (UCLES), which is itself a department of the University of Cambridge.

0625/02/O/N/09
UNIVERSITY OF CAMBRIDGE INTERNATIONAL EXAMINATIONS
International General Certificate of Secondary Education
*2425863101*

PHYSICS 0625/21
Paper 2 Core May/June 2010
1 hour 15 minutes
Candidates answer on the Question Paper.
No Additional Materials are required.

READ THESE INSTRUCTIONS FIRST

Write your Centre number, candidate number and name on all the work you hand in.
Write in dark blue or black pen.
You may use a soft pencil for any diagrams, graphs or rough working.
Do not use staples, paper clips, highlighters, glue or correction fluid.
DO NOT WRITE IN ANY BARCODES.

Answer all questions.


You may lose marks if you do not show your working or if you do not use appropriate units.
Take the weight of 1 kg to be 10 N (i.e. acceleration of free fall = 10 m / s2).

At the end of the examination, fasten all your work securely together.
The number of marks is given in brackets [ ] at the end of each question or part question.

This document consists of 18 printed pages and 2 blank pages.

DCA (SHW 00380 1/09) 20403/6


© UCLES 2010 [Turn over
2

1 Imagine that you live beside a busy road. One of your neighbours thinks that many of the For
vehicles are travelling faster than the speed limit for the road. Examiner’s
Use

You decide to check this by measuring the speeds of some of the vehicles.

(a) Which two quantities will you need to measure in order to find the speed of a vehicle,
and which instruments would you use to measure them?

quantity measured instrument used

[4]

(b) State the equation you would use to calculate the speed of the vehicle. If you use
symbols, state what your symbols mean.

[1]

(c) One lorry travels from your town to another town. The lorry reaches a top speed of
90 km / h, but its average speed between the towns is only 66 km / h.

(i) Why is the average speed less than the top speed?

..................................................................................................................................

..................................................................................................................................

............................................................................................................................ [1]

(ii) The journey between the towns takes 20 minutes.

Calculate the distance between the towns.

distance = .......................................... km [3]

[Total: 9]

© UCLES 2010 0625/21/M/J/10


3

2 A helical spring is hung from the edge of a bench top, as shown in Fig. 2.1. For
Examiner’s
Use
bench top
0 0

29.8
cm pin spring

62.8
cm
metre rule

load = 5.5 N
100 100

Fig. 2.1

Before the load is hung on the spring, the pin points to the 29.8 cm mark on the metre rule.

When a load of 5.5 N is hung on the spring, the pin points to 62.8 cm.

(a) Calculate the extension of the spring.

extension = .......................................... cm [2]

(b) The law relating extension to load is given by the equation

load = constant × extension.

(i) Calculate the numerical value of the constant.

constant = ................................................ [2]

(ii) Suggest a suitable unit for the constant.

............................................................................................................................ [1]

[Total: 5]

© UCLES 2010 0625/21/M/J/10 [Turn over


4

3 (a) Fig. 3.1 represents the energy into and out of a machine. For
Examiner’s
Use

useful output
input energy I energy U

wasted energy W

Fig. 3.1

Write down the equation linking I, U and W.

.................................................................................................................................... [1]

(b) An electric motor and a pulley in a warehouse are being used to lift a packing case of
goods from the ground up to a higher level. This is shown in Fig. 3.2.

electric
motor pulley

cable

chains
packing case
pallet

ground

Fig. 3.2

The packing case of goods, the chains and the pallet together weigh 850 N.

(i) State the value of the tension force in the cable when the load is being lifted at a
steady speed.

tension force = ............................................. N [1]

© UCLES 2010 0625/21/M/J/10


5

(ii) When the load is just leaving the floor, why is the force larger than your answer to For
(b)(i)? Examiner’s
Use

..................................................................................................................................

............................................................................................................................ [1]

(iii) The warehouse manager wishes to calculate the useful work done when the load
is lifted from the ground to the higher level. Which quantity, other than the weight,
does he need to measure?

............................................................................................................................ [1]

(iv) Which further quantity does the manager need to know, in order to calculate the
power required to lift the load?

............................................................................................................................ [1]

(c) How does the electrical energy supplied to the electric motor compare with the increase
in energy of the load? Answer by completing the sentence below.

The electrical energy supplied to the motor is ……………………………… the

increase in energy of the load. [1]

[Total: 6]

© UCLES 2010 0625/21/M/J/10 [Turn over


6

4 (a) A musical note is being produced by a loudspeaker connected to a signal generator. For
Examiner’s
A person is listening to the note, as shown in Fig. 4.1. Use

signal generator

amplitude frequency

Fig. 4.1

By adjusting the controls on the signal generator, the amplitude and the frequency of
the note from the loudspeaker can each be changed.

The person moves to a position further away from the loudspeaker.

(i) State what, if anything, happens to

1. the pitch of the sound heard,

...........................................................................................................................

2. the loudness of the sound heard.

..................................................................................................................... [2]

(ii) What adjustment, if any, should be made to the two controls so that the sound
heard in the new position is the same as in the original position?

frequency control ......................................................................................................

amplitude control ................................................................................................ [2]

© UCLES 2010 0625/21/M/J/10


7

(b) Fig. 4.2 shows a girl standing some distance from a rock face. She has a bell in her For
hand. Examiner’s
Use

rock face

Fig. 4.2

The girl rings the bell once. After a short time the sound of the bell reaches her again.

(i) Why did the sound return to her?

............................................................................................................................ [1]

(ii) Why was there a short time delay before the girl heard the second sound?

............................................................................................................................ [1]

[Total: 6]

© UCLES 2010 0625/21/M/J/10 [Turn over


8

5 Fig. 5.1 shows a child’s toy. It is made out of wood, in the shape of a bird. The toy includes a For
metal weight stuck to the tail. When placed on a metal rod, the toy balances in equilibrium. Examiner’s
Use

metal rod

metal weight

Fig. 5.1

(a) On Fig. 5.1, mark with the letter X a possible position for the centre of mass of the toy.
[1]

(b) The metal weight falls off the tail.

(i) On Fig. 5.1, mark with the letter Y a possible new position for the centre of mass.
[1]

(ii) What happens to the toy immediately after the metal weight falls off?

..................................................................................................................................

..................................................................................................................................

............................................................................................................................ [2]

[Total: 4]

© UCLES 2010 0625/21/M/J/10


9

6 (a) Fig. 6.1 shows a beaker in which coffee is served at an airport kiosk. For
Examiner’s
Use

section through
beaker

layer of corrugated
cardboard stuck to
a layer of smooth
cardboard, with air
trapped between them

Fig. 6.1 Fig. 6.2

The beaker itself is made of two layers of cardboard, as shown in section in Fig. 6.2. It
has a thin plastic lid.

(i) State two sources of heat loss that are reduced by the lid.

1. ...............................................................................................................................

2. ......................................................................................................................... [2]

(ii) State two reasons why the layer of corrugated cardboard stops the fingers of the
person holding the beaker from becoming uncomfortably hot.

1. ...............................................................................................................................

2. ......................................................................................................................... [2]

(b) (i) State the meaning of the term thermal capacity.

..................................................................................................................................

............................................................................................................................ [2]

(ii) Another airport kiosk serves coffee in pottery mugs. The mugs all have the same
internal dimensions but some have a high thermal capacity and some have a low
thermal capacity.

When hot drinks are poured into the mugs, the temperature of the drink always
drops because of the thermal energy absorbed by the mug.

State which mug, high thermal capacity or low thermal capacity, causes the least
fall in temperature of the hot drink, and explain why.

mug ..........................................................................................................................

explanation ......................................................................................................... [2]

[Total: 8]
© UCLES 2010 0625/21/M/J/10 [Turn over
10

7 The electric lamp in Fig. 7.1 has “240 V, 100 W” marked on it. Its filament is a coiled coil of For
fine tungsten wire, as shown in Fig. 7.2. Examiner’s
Use

Fig. 7.1 Fig. 7.2

(a) (i) To give out white light, the filament has to reach a very high temperature.

Suggest why having the filament as a coiled coil helps to achieve this high
temperature.

..................................................................................................................................

..................................................................................................................................

............................................................................................................................ [1]

(ii) If your hand is close to the lamp when it is switched on for a few seconds, you can
feel warmth from the lamp but the glass will remain cool.

By what method has the thermal energy reached your hand? Tick one box.

conduction

convection

evaporation

radiation
[1]

© UCLES 2010 0625/21/M/J/10


11

(b) (i) The markings on the lamp give electrical information about the lamp when it is For
operating normally. Examiner’s
Use

State the value of

1. the normal operating potential difference across the lamp,

...........................................................................................................................

2. the power of the lamp.

..................................................................................................................... [2]

(ii) When the lamp is working normally, its resistance is 576 .

Calculate the current in the lamp.

current = ................................................ [4]

[Total: 8]

© UCLES 2010 0625/21/M/J/10 [Turn over


12

8 Fig. 8.1 shows how an image is formed by a converging lens. For


Examiner’s
Use
24 cm 10 cm 8cm

I
O F2 F1

Fig. 8.1

(a) State the value of the focal length of the lens.

focal length = .......................................... cm [1]

(b) The object O is moved a small distance to the left.

State two things that happen to the image I.

1. ......................................................................................................................................

2. ................................................................................................................................ [2]

(c) Points F1 and F2 are marked on Fig. 8.1.

(i) State the name we give to these two points.

............................................................................................................................ [1]

(ii) On Fig. 8.1, draw the ray from the top of the object which passes through F2.

Continue your ray until it meets the image. [4]

[Total: 8]

© UCLES 2010 0625/21/M/J/10


13

9 (a) A warning on the packaging of a light switch purchased from an electrical store reads For
Examiner’s
Use
SAFETY WARNING

This push-button switch is not suitable for use in a washroom. Lights in washrooms should be
operated by pull-cord switches.

(i) Explain why it might be dangerous to use a push-button switch in a washroom.

..................................................................................................................................

..................................................................................................................................

............................................................................................................................ [2]

(ii) Why is it safe to use a pull-cord switch in a washroom?

..................................................................................................................................

............................................................................................................................ [1]

(b) An electric heater, sold in the electrical store, has a current of 8 A when it is working
normally.

The cable fitted to the heater has a maximum safe current of 12 A.

Which of the following fuses would be most suitable to use in the plug fitted to the cable
of the heater? Tick one box.

5A

10 A

13 A

20 A
[1]

(c) The cable for connecting an electric cooker is much thicker than the cable on a table
lamp.

(i) Why do cookers need a much thicker cable?

..................................................................................................................................

............................................................................................................................ [1]

(ii) What would happen if a thin cable were used for wiring a cooker to the supply?

..................................................................................................................................

............................................................................................................................ [1]

[Total: 6]
© UCLES 2010 0625/21/M/J/10 [Turn over
14

10 A transformer has 500 turns in both its primary and its secondary coils. It is connected to a For
240 V mains supply. There are 4 possible connections to the secondary, labelled W, X, Y and Examiner’s
Z on Fig. 10.1. Use

W
240 V X
Y
Z

primary coil secondary coil


500 turns 500 turns

Fig. 10.1

Between W and X there are 300 turns.

Between X and Y there are 175 turns.

Between Y and Z there are 25 turns.

(a) A person wishes to run a 12 V electric bell, using the transformer.

Between which two of the labelled terminals should the bell be connected?

Show your working.

bell connected between terminal ............. and terminal ............ [4]

(b) State the voltage between terminals W and Z.

.................................................................................................................................... [1]

© UCLES 2010 0625/21/M/J/10


15

(c) State the name given to the part labelled P and the material from which it is made. For
Examiner’s
name ................................................................................................................................ Use

material ...................................................................................................................... [2]

(d) Why are the coils normally made from copper wire?

..........................................................................................................................................

.................................................................................................................................... [1]

[Total: 8]

© UCLES 2010 0625/21/M/J/10 [Turn over


16

11 Fig. 11.1 shows apparatus being used to project a visible spectrum onto a screen, using a For
lamp with a white-hot filament. Examiner’s
Use

glass screen
prism
glass
lens P

visible spe
ctrum
B

filament lamp
(white light source)

Fig. 11.1

(a) State two things that happen to the white light as it passes through surface PQ of the
prism.

1. ......................................................................................................................................

2. ................................................................................................................................ [2]

(b) What colour light will be seen at

(i) edge A of the spectrum, ...........................................................................................

(ii) edge B of the spectrum? ..........................................................................................


[2]

(c) A thermometer with a blackened bulb is moved very slowly across the screen.

(i) On Fig. 11.1, mark using a cross (×) the position where the thermometer will show
its largest reading. [2]

(ii) What type of radiation would cause this high reading?

............................................................................................................................ [1]

[Total: 7]

© UCLES 2010 0625/21/M/J/10


17

12 (a) A scientist, who is also an antiques collector, buys an old watch at an antiques market. For
The figures on the dial of the watch are painted with a type of luminous paint that is Examiner’s
radioactive. Use

In his laboratory, he puts the watch close to a radiation detector and then places sheets
of different materials in the gap between them, as shown in Fig. 12.1.

radiation
detector
watch
counter

sheet

Fig. 12.1

The results of his investigation are given in the following table.

material effect

sheet of paper no observable change in count rate

1 mm thick sheet of aluminium a noticeable decrease in the count rate

considerable decrease in the count rate


1 mm thick sheet of lead
but still above background

(i) From this information, deduce the type or types of radiation escaping from the
watch.

..................................................................................................................................

..................................................................................................................................

............................................................................................................................ [2]

(ii) The back of the watch is made of steel 1 mm thick.

State one reason why there would be a health hazard when wearing this watch.

..................................................................................................................................

..................................................................................................................................

............................................................................................................................ [1]

© UCLES 2010 0625/21/M/J/10 [Turn over


18

(b) Radioactive materials are stored in a cupboard. For


Examiner’s
(i) Which of the symbols shown in Fig. 12.2 is used on the door of the cupboard to Use

warn of the radiation hazard? Tick one box.

Fig. 12.2
[1]

(ii) State one other safety precaution that should be taken when storing the radioactive
substances in the cupboard.

..................................................................................................................................

............................................................................................................................ [1]

[Total: 5]

© UCLES 2010 0625/21/M/J/10


19

BLANK PAGE

© UCLES 2010 0625/21/M/J/10


20

BLANK PAGE

Permission to reproduce items where third-party owned material protected by copyright is included has been sought and cleared where possible. Every
reasonable effort has been made by the publisher (UCLES) to trace copyright holders, but if any items requiring clearance have unwittingly been included, the
publisher will be pleased to make amends at the earliest possible opportunity.

University of Cambridge International Examinations is part of the Cambridge Assessment Group. Cambridge Assessment is the brand name of University of
Cambridge Local Examinations Syndicate (UCLES), which is itself a department of the University of Cambridge.

© UCLES 2010 0625/21/M/J/10


w
w
w
.X
tr
me
eP
ap
UNIVERSITY OF CAMBRIDGE INTERNATIONAL EXAMINATIONS

er
s
International General Certificate of Secondary Education

.c
om
*2512877189*

PHYSICS 0625/21
Paper 2 Core October/November 2010
1 hour 15 minutes
Candidates answer on the Question Paper.
No Additional Materials are required.

READ THESE INSTRUCTIONS FIRST

Write your Centre number, candidate number and name on all the work you hand in.
Write in dark blue or black pen.
You may use a soft pencil for any diagrams, graphs or rough working.
Do not use staples, paper clips, highlighters, glue or correction fluid.
DO NOT WRITE IN ANY BARCODES.

Answer all questions.


You may lose marks if you do not show your working or if you do not use appropriate units.
Take the weight of 1 kg to be 10 N (i.e. acceleration of free fall = 10 m / s2).

At the end of the examination, fasten all your work securely together.
The number of marks is given in brackets [ ] at the end of each question or part question.

This document consists of 16 printed pages and 4 blank pages.

DC (NF/SW) 24995/3
© UCLES 2010 [Turn over
2

1 (a) (i) Figs. 1.1 and 1.2 show the dimensions of a rectangular block being measured using a
ruler. They are not shown full size.

Use the scales shown to find the length and the width of the block, giving your answers
in cm.

length of block ................. cm [1]

140 150 160 170 180 190 200 210 220 230 240 250
millimetres
50 60 70 80 90 100 110 120 130 140 150 160

Fig. 1.1
210 220 230 240 250 260 270 280 290 300
10 20 30 40 50 60 70 80 90
millimetres

width of block ...................... cm [1]

Fig. 1.2

(ii) When the block was made, it was cut from a piece of metal 2.0 cm thick.

Calculate the volume of the block.

volume = ................................................. cm3 [2]


© UCLES 2010 0625/21/O/N/10
3

(b) Another block has a volume of 20 cm3.

Fig. 1.3 shows the reading when the block is placed on a balance.

block

40 50 60 70
grams

Fig. 1.3

Find the density of this block.

density = ......................................................... [4]

[Total: 8]

© UCLES 2010 0625/21/O/N/10 [Turn over


4

2 A boy cycles a distance of 960 m from home to school in 8.0 minutes.

(a) Calculate his average speed for the journey.

average speed = ......................................................... [4]

(b) The journey is all along a horizontal road. At the end of the journey the boy is tired because of
the work he has done.

Against which force has this work been done?

............................................................................................................................................. [1]

[Total: 5]

3 (a) Name three different energy resources used to obtain energy directly from water (not steam).

1. ...............................................................................................................................................

2. ...............................................................................................................................................

3. ......................................................................................................................................... [3]

(b) Choose one of the energy resources you have named in (a) and write a brief description of
how the energy is converted to electrical energy.

Which energy resource are you describing? ............................................................................

description ................................................................................................................................

...................................................................................................................................................

...................................................................................................................................................

...................................................................................................................................................

...................................................................................................................................................

............................................................................................................................................. [3]

[Total: 6]

© UCLES 2010 0625/21/O/N/10


5

4 Fig. 4.1 shows four parallel rays of light reaching a thin converging lens. Point F is a principal focus
of the lens.

P F

Fig. 4.1

(a) What name do we give to the distance PF? ....................................................................... [1]

(b) On Fig. 4.1, carefully draw the paths of the rays through the lens and into the air as far as the
broken line. [2]

(c) A flat white screen is placed at F, parallel to the broken line.

Describe what is seen on the screen.

...................................................................................................................................................

............................................................................................................................................. [1]

(d) The screen is moved so that it is along the broken line.

Describe what is now seen on the screen.

...................................................................................................................................................

............................................................................................................................................. [1]

[Total: 5]

© UCLES 2010 0625/21/O/N/10 [Turn over


6

5 Here is a list of different types of radiation.

alpha (α), beta (β), gamma (γ), infra-red, radio, ultra-violet, visible, X-rays

(a) Underline all those radiations in the list which are not electromagnetic radiations. [2]

(b) Which radiation is the most penetrating? ............................................................................ [1]

(c) Which radiation has the longest wavelength? ..................................................................... [1]

(d) Which radiation consists of particles that are the same as 4He nuclei? ............................. [1]

[Total: 5]

6 Fig. 6.1 shows two experiments to investigate energy transfer in water.

water water
ice

gentle
heating

ice trapped by
small piece of
gentle wire gauze
heating

Experiment 1 Experiment 2
Cold water is gently heated Cold water is gently heated
at the bottom. The ice at the at the top. The ice trapped
top melts before the water at the bottom remains solid,
boils. even when the water at the
top begins to boil.

Fig. 6.1

(a) Name the process by which thermal (heat) energy travels through the glass.

............................................................................................................................................. [1]

(b) (i) Name the principal process in Experiment 1 which takes the energy from the water at the
bottom to the ice at the top.

..................................................................................................................................... [1]

© UCLES 2010 0625/21/O/N/10


7

(ii) Describe how the process in (b)(i) occurs.

...........................................................................................................................................

...........................................................................................................................................

...........................................................................................................................................

...........................................................................................................................................

..................................................................................................................................... [2]

(c) Suggest two reasons why the ice in Experiment 2 does not melt, even when the water at the
top begins to boil.

1. ...............................................................................................................................................

...................................................................................................................................................

2. ...............................................................................................................................................

............................................................................................................................................. [2]

[Total: 6]

© UCLES 2010 0625/21/O/N/10 [Turn over


8

BLANK PAGE

© UCLES 2010 0625/21/O/N/10


9

7 (a) In Fig. 7.1, a ray of light is shown passing into water from air. The angle of the refracted ray to
the normal is 40°.

On Fig. 7.1, mark clearly the angle of incidence i. [1]

ray of
light
air

40° water

Fig. 7.1

(b) In Fig. 7.2, a ray of light is shown in water and reaching the surface with the air at an angle of
40° to the normal.

air

water
40°

Fig. 7.2

(i) On Fig. 7.2, draw accurately the path of the ray in the air. [2]

(ii) The angle in the water in Fig. 7.2 is increased from 40° to 70°, and the ray no longer
emerges into the air.

State what happens to the ray at the surface and explain why this happens.

...........................................................................................................................................

...........................................................................................................................................

...........................................................................................................................................

..................................................................................................................................... [2]

[Total: 5]

© UCLES 2010 0625/21/O/N/10 [Turn over


10

8 Fig. 8.1 shows a workman hammering a metal post into the ground. Some distance away is a
vertical cliff.

cliff
workman
boy girl

Fig. 8.1

(a) A boy is standing at the foot of the cliff. The speed of sound in air is 330 m / s. It takes 1.5 s for
the sound of the hammer hitting the post to reach the boy.

(i) What does the boy hear after he sees each strike of the hammer on the post?

..................................................................................................................................... [1]

(ii) Calculate the distance between the post and the boy.

distance = ..................................................... m [3]

(b) A girl is also watching the workman. She is standing the same distance behind the post as
the boy is in front of it. She hears two separate sounds after each strike of the hammer on the
post.

(i) Why does she hear two sounds?

...........................................................................................................................................

...........................................................................................................................................

..................................................................................................................................... [2]

© UCLES 2010 0625/21/O/N/10


11

(ii) How long after the hammer strike does the girl hear each of these sounds?

girl hears first sound after ............... s

girl hears second sound after ............... s [2]

[Total: 8]

© UCLES 2010 0625/21/O/N/10 [Turn over


12

9 (a) Fig. 9.1 shows the magnetic field pattern around a single bar magnet.

A B

Fig. 9.1

(i) On Fig. 9.1, mark the north and south poles of the magnet, using the letters N and S.
[2]
(ii) A small piece of unmagnetised iron is placed at A.

What, if anything, happens to it?

..................................................................................................................................... [1]

(iii) A small piece of positively charged plastic is placed at B.

What, if anything, happens to it?

..................................................................................................................................... [1]

(b) Fig. 9.2 shows an electromagnet.

coil

core

Fig. 9.2

(i) What must be done to magnetise the core?

..................................................................................................................................... [1]

(ii) Suggest the material from which the core should be made.

..................................................................................................................................... [1]

(iii) State one advantage of an electromagnet, compared with a magnet such as that in (a).

..................................................................................................................................... [1]
[Total: 7]

© UCLES 2010 0625/21/O/N/10


13

10 A cruise ship is anchored in a harbour. The crew holds a party for the guests on board, and the
ship’s electrical department decorates the decks with strings of coloured lamps.

Fig. 10.1

Each string of lamps contains thirty 100 V lamps. The strings of lamps are run from a 100 V
generator. The resistance of each lamp is 250 Ω. Ignore the resistance of the generator.

(a) State whether the lamps on a particular string are connected in series or in parallel.

............................................................................................................................................. [1]

(b) Calculate the current in each lamp when it is at normal brightness.

current = ..................................................... A [3]

(c) What current does the generator supply to each string of lamps?

current = ..................................................... A [1]

(d) The generator supplies current to several strings of lamps.

State whether the strings are connected to the generator in series or in parallel.

............................................................................................................................................. [1]

(e) One of the lamps “blows” and forms an open circuit.

What effect, if any, does this have on

(i) the other lamps in the same string, ...................................................................................

(ii) the lamps in the other strings? .................................................................................... [2]

[Total: 8]

© UCLES 2010 0625/21/O/N/10 [Turn over


14

BLANK PAGE

© UCLES 2010 0625/21/O/N/10


15

11 The reed switch (reed relay) shown in Fig. 11.1 is a normally-closed one.

Fig. 11.1

When a magnet is held close to the reed switch, the contacts open, as shown in Fig. 11.2.
Fig. 11.2 also includes the circuit symbol for a bell.

N S

Fig. 11.2

(a) Complete Fig. 11.2 so that it shows a circuit that will cause the bell to ring when the magnet is
taken away. [2]

(b) Fig. 11.3 shows a door in a wall.

(i) On Fig 11.3, show where


you would fix the reed
switch and the magnet of
Fig 11.2, so that the bell
rings when the door opens.
Use the letter S for the
switch and the letter M for
the magnet. [2]

(ii) Suggest one application of


this arrangement.

............................................

............................................

............................................

...................................... [1]

Fig. 11.3

[Total: 5]
© UCLES 2010 0625/21/O/N/10 [Turn over
16

12 The table below lists the three types of emission which can occur during radioactive decay.

(a) Complete the table to indicate whether each of the emissions has mass and whether it has
charge. Three answers have been given to help you.

mass charge

alpha (α) YES

beta (β) YES

gamma (γ) NO
[3]

(b) From which part of the atom do all of these emissions come?

............................................................................................................................................. [1]

(c) The values in the table below were obtained during the decay of a radioactive substance.

elapsed time / minutes count rate


counts / min

0 909

20 689

40 522

60 400

80 300

100 230

120 170

140 125

160 99

(i) On Fig. 12.1, three points have been plotted for you.

Plot the remaining points, using dots in circles as shown, and draw the best-fit curve for
these points. [3]

© UCLES 2010 0625/21/O/N/10


17

1000

900
count rate
counts / min

800

time to decrease from 800 counts / min to


700 200 counts / min

= .................................. minutes
600
half-life of substance

= .................................. minutes
500

400

300

200

100

0
0 20 40 60 80 100 120 140 160
elapsed time / minutes

Fig. 12.1

© UCLES 2010 0625/21/O/N/10 [Turn over


18

(ii) From the graph, find the time taken for the count rate to decrease from 800 counts / min
to 200 counts / min. Write your answer and any working in the space on the graph. [3]
(iii) Use your value from (c)(ii) to determine the half-life of the radioactive substance.

Write your answer in the space on the graph. [1]

(d) A different sample of the same radioactive substance as in (c) has an initial count rate of
4000 counts / min.

Write down the time taken for the count rate to decrease to 1000 counts / min.

............................................................................................................................................. [1]

[Total: 12]

© UCLES 2010 0625/21/O/N/10


19

BLANK PAGE

© UCLES 2010 0625/21/O/N/10


20

BLANK PAGE

Permission to reproduce items where third-party owned material protected by copyright is included has been sought and cleared where possible. Every
reasonable effort has been made by the publisher (UCLES) to trace copyright holders, but if any items requiring clearance have unwittingly been included, the
publisher will be pleased to make amends at the earliest possible opportunity.

University of Cambridge International Examinations is part of the Cambridge Assessment Group. Cambridge Assessment is the brand name of University of
Cambridge Local Examinations Syndicate (UCLES), which is itself a department of the University of Cambridge.

© UCLES 2010 0625/21/O/N/10


w
w
w
.X
tr
me
eP
ap
UNIVERSITY OF CAMBRIDGE INTERNATIONAL EXAMINATIONS

er
s
International General Certificate of Secondary Education

.c
om
* 6 9 9 3 4 3 1 9 7 2 *

PHYSICS 0625/21
Paper 2 Core May/June 2011
1 hour 15 minutes
Candidates answer on the Question Paper.
No Additional Materials are required.

READ THESE INSTRUCTIONS FIRST

Write your Centre number, candidate number and name on all the work you hand in.
Write in dark blue or black pen.
You may use a pencil for any diagrams, graphs or rough working.
Do not use staples, paper clips, highlighters, glue or correction fluid.
DO NOT WRITE IN ANY BARCODES.

Answer all questions.


You may lose marks if you do not show your working or if you do not use appropriate units.
Take the weight of 1 kg to be 10 N (i.e. acceleration of free fall = 10 m / s2).

At the end of the examination, fasten all your work securely together.
The number of marks is given in brackets [ ] at the end of each question or part question.

This document consists of 19 printed pages and 1 blank page.

DC (NH/DJ) 28876/6
© UCLES 2011 [Turn over
2

1 Fig. 1.1 shows a measuring cylinder containing water.

Fig. 1.2 shows the same measuring cylinder after stone A has been lowered into it on a fine
string.

60.4 cm3

44.2 cm3

water A

Fig. 1.1 Fig. 1.2

(a) Calculate the volume of stone A.

volume = .......................... cm3 [2]

(b) The mass of stone A is measured as 40.5 g.

Calculate the density of the rock from which stone A was formed.

density = ................................. [4]

© UCLES 2011 0625/21/M/J/11


3

(c) Stone B, from the same rock as stone A, has a larger volume.

Stone A is removed from the water and replaced by stone B. The measurements are then
repeated.

Which of the values in the experiment will be different when using stone B?

Tick boxes alongside any of the quantities that will have changed.

60.4 cm3

40.5 g

density
[2]

[Total: 8]

© UCLES 2011 0625/21/M/J/11 [Turn over


4

2 A fixed amount of dry gas is exerting a pressure on its container.

(a) In terms of molecules, explain what causes the pressure.

...................................................................................................................................................

...................................................................................................................................................

...................................................................................................................................................

...............................................................................................................................................[3]

(b) One of the sketch graphs in Fig. 2.1 shows how the pressure of the gas varies with volume
at constant temperature, and the other shows how the pressure varies with temperature at
constant volume.

The pressure axis has been labelled in each case.

pressure pressure
/ Pa / Pa

0 0

Fig. 2.1

(i) On the appropriate graphs, label one horizontal axis “ volume / m3 ” and the other horizontal
axis “ temperature / °C ”.

(ii) On one of the graphs, mark, with the letter X, the pressure of the gas at the ice point.
[2]

[Total: 5]

© UCLES 2011 0625/21/M/J/11


5

3 Electricity can be generated from either renewable or non-renewable energy sources.

(a) Describe the difference between a renewable energy source and a non-renewable energy
source.

...................................................................................................................................................

...................................................................................................................................................

...................................................................................................................................................

...............................................................................................................................................[1]

(b) (i) Name one renewable energy source.

...........................................................................................................................................

(ii) Suggest one reason why your choice in (b)(i) is not, so far, widely used for generating
electricity.

...........................................................................................................................................

.......................................................................................................................................[2]

(c) (i) Name one non-renewable energy source.

...........................................................................................................................................

(ii) Suggest one reason why non-renewable sources have been widely used for generating
electricity up until the present time.

...........................................................................................................................................

.......................................................................................................................................[2]

[Total: 5]

© UCLES 2011 0625/21/M/J/11 [Turn over


6

4 The main parts of a cold store are shown in Fig. 4.1.

cooling tubes cold store

refrigeration
unit

Fig. 4.1

(a) Explain why the cooling tubes are positioned at the top of the store.

...................................................................................................................................................

...................................................................................................................................................

...................................................................................................................................................

...............................................................................................................................................[1]

(b) Suggest why the refrigeration unit is outside the cold store.

...................................................................................................................................................

...................................................................................................................................................

...................................................................................................................................................

...............................................................................................................................................[2]

(c) The walls are made of thick thermally-insulating material.

Why is it important to have the walls made like this?

...................................................................................................................................................

...................................................................................................................................................

...................................................................................................................................................

...............................................................................................................................................[2]

© UCLES 2011 0625/21/M/J/11


7

(d) Even when the refrigeration unit is running continuously, there comes a time when the
temperature in the store stops falling, and remains constant.

Explain why this happens.

...................................................................................................................................................

...................................................................................................................................................

...................................................................................................................................................

...............................................................................................................................................[2]

[Total: 7]

© UCLES 2011 0625/21/M/J/11 [Turn over


8

5 (a) The four words below can each be used to describe waves.

Put a tick in each of the boxes alongside any words that can be applied to sound waves.

longitudinal

transverse

electromagnetic

mechanical
[2]

(b) How does an echo occur?

...................................................................................................................................................

...............................................................................................................................................[1]

(c) Fig. 5.1 shows a boat steaming along a river. The river is in a wide gorge and there are high
cliffs on each side.

cliff cliff
A B

Fig. 5.1

The boat sounds its hooter once. Two clear echoes are heard by a person on the boat.

The first echo is 1.5 s after the hooter sounds. The second echo is 2.5 s after the hooter sounds.

(i) Which cliff caused the first echo? ...................................................................................[1]


(ii) Sound travels at 330 m / s in air.

Calculate the distance between the two cliffs.

distance = ....................................................... m [3]


© UCLES 2011 0625/21/M/J/11
9

(iii) Further along the river, the cliffs are the same distance apart but the river is midway
between the cliffs. The boat sounds its hooter again.

Without further calculation, describe what is now heard and approximately when it is
heard.

what is heard .....................................................................................................................

...........................................................................................................................................

when ..................................................................................................................................

.......................................................................................................................................[2]

[Total: 9]

© UCLES 2011 0625/21/M/J/11 [Turn over


10

6 A laser is a device that gives a narrow parallel beam of monochromatic (single-colour)


light.

Fig. 6.1 shows the light from a laser shining on a triangular glass prism.

screen

prism

light from laser

Fig. 6.1

(a) On Fig. 6.1, complete the path of the light until it reaches the screen. [2]

(b) What will be seen on the screen?

...............................................................................................................................................[1]

(c) The beam of light from the laser is replaced by a beam of white light from a lamp.

State what is now seen on the screen.

...................................................................................................................................................

...............................................................................................................................................[2]

[Total: 5]

© UCLES 2011 0625/21/M/J/11


11

7 (a) Two light, identical spheres, A and B, are suspended alongside each other on thin nylon
threads, as shown in Fig. 7.1.

nylon threads

A B

Fig. 7.1

A is given a positive charge and B is given a negative charge.

On Fig. 7.1, draw how the threads and spheres might look after the spheres have been
charged. [1]

(b) A cleaner is attempting to remove dust from some plastic-covered furniture, using a dry cloth.
Unfortunately, this seems to make the dust cling more firmly to the plastic covering.

(i) Suggest why this happens.

...........................................................................................................................................

...........................................................................................................................................

...........................................................................................................................................

.......................................................................................................................................[3]

(ii) Suggest why this would be less likely to happen if the cleaner used a cloth which was
very slightly damp.

...........................................................................................................................................

...........................................................................................................................................

...........................................................................................................................................

.......................................................................................................................................[2]

[Total: 6]

© UCLES 2011 0625/21/M/J/11 [Turn over


12

8 Fig. 8.1 shows two resistors forming part of a circuit.

A B C D

4.2 V

Fig. 8.1

(a) (i) Complete the following sentence about the two resistors connected between A and D.

The two resistors are connected in ..................................................... . [1]

(ii) The potential difference between A and D is 4.2 V.

State the value of the potential difference between B and C.

potential difference = ........................................................V [1]

(iii) Calculate the current in the 3 Ω resistor.

current = ............................................................[4]

(iv) State how the current in AB compares with

1. the current in each of the two resistors,

...........................................................................................................................................

2. the current in CD.

.......................................................................................................................................[2]

© UCLES 2011 0625/21/M/J/11


13

(b) In the boxes below, draw two circuit diagrams, each containing a battery and three identical
resistors. Arrange each circuit so that all the resistors in the circuit carry the same current.

Draw one circuit in each box.

[2]

[Total: 10]

© UCLES 2011 0625/21/M/J/11 [Turn over


14

9 (a) A car headlamp circuit consists of a 12 V battery, a switch, two 12 V headlamps and a 12 V
indicator lamp (inside the car).

Fig. 9.1 shows the symbols of these components.

left headlamp

right headlamp

indicator lamp
(inside car)

switch 12 V battery

Fig. 9.1

Complete Fig. 9.1 to show the wiring necessary so that all three lamps light up at full brightness
when the switch is closed. [2]

(b) (i) Explain briefly, in terms of molecules, why solids expand when heated.

...........................................................................................................................................

...........................................................................................................................................

.......................................................................................................................................[1]

© UCLES 2011 0625/21/M/J/11


15

(ii) Brass expands when heated.

Invar is a metal that expands very little when heated.

The flasher lamp shown in Fig. 9.2 is used as a warning lamp in a car.

lamp
contact
filament

brass strip invar strip

Fig. 9.2

The flasher lamp is designed to flash on and off repeatedly.

It contains a bimetallic strip made of a brass strip and an invar strip joined together along
their lengths.

1. State what happens to the bimetallic strip when it is heated.

.......................................................................................................................................[2]

2. Explain why the lamp flashes on and off repeatedly when it is connected to an
appropriate battery.

...........................................................................................................................................

...........................................................................................................................................

...........................................................................................................................................

...........................................................................................................................................

.......................................................................................................................................[4]

[Total: 9]

© UCLES 2011 0625/21/M/J/11 [Turn over


16

10 A rotating-coil generator consists of a rectangular coil of wire that rotates at constant speed
between the poles of a permanent magnet.

Figs. 10.1, 10.2 and 10.3 show views from one end of the coil, when the coil is in different
positions.

N S N S N S

Fig. 10.1 Fig. 10.2 Fig. 10.3

(a) Which diagram shows the coil in the position where

(i) the induced e.m.f. in the coil is at its maximum, .................................................

(ii) the induced e.m.f. in the coil is zero? .................................................................. [2]

(b) On Fig. 10.4, sketch the graph of e.m.f. against time, for two complete rotations of the coil.

e.m.f.

0
time

Fig. 10.4
[4]

[Total: 6]

© UCLES 2011 0625/21/M/J/11


17

11 Fig. 11.1 shows a cathode-ray tube connected to external voltage supplies and switches. The
terminals labelled H.T. are connected to a high voltage source.

S1 – +
H.T. horizontal metal
plates
screen

V1
C D

S2
cathode anode S3

V2

Fig. 11.1

(a) When the cathode becomes hot it releases electrons.

What name is used to describe the release of electrons by heating?

...............................................................................................................................................[1]

(b) (i) Which switch should be closed to make the cathode become hot?

switch = ...............................................................

(ii) Which switch should then be closed to obtain a beam of electrons along CD?

switch = ...............................................................

(iii) Which switch should then be closed to deflect the beam of electrons downwards?

switch = ...........................................................[2]

(c) What must be done in order to deflect the beam of electrons upwards, instead of
downwards?

...................................................................................................................................................

...................................................................................................................................................

...............................................................................................................................................[1]

[Total: 4]

© UCLES 2011 0625/21/M/J/11 [Turn over


18

12 (a) Complete the following definition of radioactive half-life.

The half-life of a radioactive substance is the time taken for .....................................................

...................................................................................................................................................

...................................................................................................................................................

...............................................................................................................................................[2]

(b) Fig. 12.1 shows how the count rate from a sample of radioactive material changes with time.

30
count rate
counts / s

25

20

15

10

0
0 20 40 60 80 100
time / s

Fig. 12.1

© UCLES 2011 0625/21/M/J/11


19

(i) Use Fig. 12.1 to find the half-life of the radioactive material.

half-life = ....................................................... s [1]

(ii) Use the graph to find how long it takes for the count rate to decrease from 30 counts / s to
10 counts / s.

time taken = ....................................................... s [1]

(iii) Use your answers to (i) and (ii) to find how long it takes for the count rate to decrease
from 30 counts / s to 5 counts / s.

time taken = ....................................................... s [2]

[Total: 6]

© UCLES 2011 0625/21/M/J/11


20

BLANK PAGE

Permission to reproduce items where third-party owned material protected by copyright is included has been sought and cleared where possible. Every
reasonable effort has been made by the publisher (UCLES) to trace copyright holders, but if any items requiring clearance have unwittingly been included, the
publisher will be pleased to make amends at the earliest possible opportunity.

University of Cambridge International Examinations is part of the Cambridge Assessment Group. Cambridge Assessment is the brand name of University of
Cambridge Local Examinations Syndicate (UCLES), which is itself a department of the University of Cambridge.

© UCLES 2011 0625/21/M/J/11


w
w
w
.X
tr
me
eP
ap
UNIVERSITY OF CAMBRIDGE INTERNATIONAL EXAMINATIONS

er
s
International General Certificate of Secondary Education

.c
om
* 1 2 6 4 5 3 7 1 3 4 *

PHYSICS 0625/21
Paper 2 Core October/November 2011
1 hour 15 minutes
Candidates answer on the Question Paper.
No Additional Materials are required.

READ THESE INSTRUCTIONS FIRST

Write your Centre number, candidate number and name on all the work you hand in.
Write in dark blue or black pen.
You may use a soft pencil for any diagrams, graphs or rough working.
Do not use staples, paper clips, highlighters, glue or correction fluid.
DO NOT WRITE IN ANY BARCODES.

Answer all questions.


You may lose marks if you do not show your working or if you do not use appropriate units.
Take the weight of 1 kg to be 10 N (i.e. acceleration of free fall = 10 m / s2).

At the end of the examination, fasten all your work securely together.
The number of marks is given in brackets [ ] at the end of each question or part question.

This document consists of 15 printed pages and 1 blank page.

DC (CW/CGW) 34004/4
© UCLES 2011 [Turn over
2

1 Fig. 1.1 shows the speed-time graph of a moving object.

speed B C
m/s 5

A D
0
0 10 20 time / s 35

Fig. 1.1

(a) Which part, or parts, of Fig. 1.1 indicate when the object is

(i) travelling at uniform speed, ...........................................................................................[1]

(ii) accelerating? .................................................................................................................[1]

(b) Calculate the distance travelled in the last 15 s.

distance = ...................................................... m [3]

[Total: 5]

© UCLES 2011 0625/21/O/N/11


3

2 Two girls attempt to measure the speed of the water in a river, as shown in Fig. 2.1.

bridge 1

50
m

bridge 2

Fig. 2.1

(a) The distance between the two bridges is measured as 50 m.

Suggest an appropriate instrument that they might use to measure this distance.

...............................................................................................................................................[1]

(b) The girl on bridge 1 drops a piece of wood into the water. The girl on bridge 2 measures how
long it takes for the piece of wood to reach bridge 2. It takes 400 s to travel between the two
bridges.

(i) Name an appropriate instrument that could be used to measure this time.

.......................................................................................................................................[1]

(ii) Describe the procedure for measuring this time.

...........................................................................................................................................

...........................................................................................................................................

...........................................................................................................................................

...........................................................................................................................................

.......................................................................................................................................[3]

(iii) Calculate the speed of the water.

speed = ...........................................................[4]

[Total: 9]

© UCLES 2011 0625/21/O/N/11 [Turn over


4

3 (a) In a laboratory experiment to find the centre of mass of a triangular piece of card, the card is
suspended first from point A and then from point B, as shown in Figs. 3.1 and 3.2.

B B
A
E

D D
E

C
C

Fig. 3.1 Fig. 3.2

When suspended from A, point D is found to be vertically below A.

When suspended from B, point E is vertically below B.

(i) What piece of apparatus might be used to determine the vertical lines through A and B?

.......................................................................................................................................[1]

(ii) On Fig. 3.3 below, draw construction lines to find the position of the centre of mass of
the triangular card. Label this point clearly with the letter G.

C
B D
[3]
Fig. 3.3

© UCLES 2011 0625/21/O/N/11


5

(b) Fig. 3.4 illustrates a toy that always returns to the upright position, whatever position it is put
in to start with.

Fig. 3.4

On one of the three positions of the toy shown in Fig. 4.3, clearly mark X at a possible position
for the centre of mass of the toy. [2]

[Total: 6]

© UCLES 2011 0625/21/O/N/11 [Turn over


6

4 In Fig. 4.1, the smoke cell consists of an illuminated glass box into which some smoke has been
injected.

microscope

smoke
cell light

Fig. 4.1

(a) Describe briefly what is seen when the contents of the smoke cell are viewed through the
focused microscope.

...................................................................................................................................................

...................................................................................................................................................

...................................................................................................................................................

...............................................................................................................................................[3]

(b) State the name we normally give to what is seen.

...............................................................................................................................................[1]

(c) What deductions about the properties and behaviour of air molecules can be made from
these observations?

...................................................................................................................................................

...................................................................................................................................................

...................................................................................................................................................

...............................................................................................................................................[3]

[Total: 7]

© UCLES 2011 0625/21/O/N/11


7

5 A card is held against a rotating toothed wheel, as shown in Fig. 5.1.

card

Fig. 5.1

The wheel has 150 teeth and rotates 3 times per second.

(a) Calculate the frequency of the sound produced.

frequency = .................................................... Hz [2]

(b) State the approximate range of frequencies that can be heard by a healthy human ear.

lowest frequency = ........................... Hz

highest frequency = ........................... Hz [1]

(c) The speed of rotation of the wheel is increased.

What happens, if anything, to the pitch of the sound that is heard?

...............................................................................................................................................[1]

[Total: 4]

© UCLES 2011 0625/21/O/N/11 [Turn over


8

6 The list below contains the names of the various regions of the electromagnetic spectrum, but they
are in the wrong order and an extra, non-electromagnetic type of wave motion has been included.

infra-red radiation

X-rays

gamma rays

visible light

radio waves

ultrasound waves

ultra-violet radiation

(a) Which one of these is not electromagnetic?

...............................................................................................................................................[1]

(b) (i) In the boxes below, list the various regions of the electromagnetic spectrum in order of
wavelength. The first and last boxes have been filled in for you.

radio waves

gamma rays
[2]

(ii) Which of these has the longest wavelength?

.......................................................................................................................................[1]

(iii) Which of these produces a significant heating effect?

.......................................................................................................................................[1]

(iv) Which of these may be used to obtain a picture of a broken bone inside the body?

.......................................................................................................................................[1]

[Total: 6]

© UCLES 2011 0625/21/O/N/11


9

7 The pivoted steel arrow from a small compass has become demagnetised.

The coil shown in Fig. 7.1 is to be used to magnetise it again.

coil
steel
arrow

Fig. 7.1

(a) Describe carefully

(i) how the coil is used to magnetise the steel arrow,

...........................................................................................................................................

...........................................................................................................................................

...........................................................................................................................................

...........................................................................................................................................

.......................................................................................................................................[3]

(ii) how the polarity of the magnetised needle may be checked.

...........................................................................................................................................

...........................................................................................................................................

...........................................................................................................................................

.......................................................................................................................................[1]

(b) On Fig. 7.1, draw the magnetic field pattern of the magnetised needle. [2]

[Total: 6]

© UCLES 2011 0625/21/O/N/11 [Turn over


10

8 A student learning about electric circuits connects up the circuit shown in Fig. 8.1.

battery
ammeter
01 23
4
AMP 5
S

coil of
wire

variable resistor
(rheostat)

Fig. 8.1

(a) The student has already made one mistake with his connections.

What is his mistake?

...............................................................................................................................................[1]

(b) In the space below, draw the circuit diagram of the arrangement shown in Fig. 8.1, using
standard symbols. [2]

(c) The student now connects a voltmeter to the circuit, so that readings can be taken to find
the resistance of the coil of wire. On your circuit diagram, draw the voltmeter, connected to
measure the potential difference across the coil. [1]

© UCLES 2011 0625/21/O/N/11


11

(d) Having obtained a correctly-working circuit, the readings on the two meters for one setting of
the variable resistor are as shown in Figs. 8.2 and 8.3.

2 3 4 5 10
1
0 5 0 15

AMPS VOLTS

Fig. 8.2 Fig. 8.3

(i) Record these two readings.

ammeter reading = ...................................................... A

voltmeter reading = ....................................................... V [1]

(ii) The slider on the variable resistor (see Fig. 8.1) is moved a small distance to the left,
reducing its resistance.

State what happens to the readings on the two meters.

ammeter reading ...............................................................................................................

voltmeter reading ...........................................................................................................[2]

(iii) The slider is adjusted so that the total resistance in the circuit is double that which gave
the readings in Figs. 8.2 and 8.3.

Calculate the new values of the readings on the meters.

ammeter reading = ...................................................... A

voltmeter reading = ....................................................... V [2]

[Total: 9]

© UCLES 2011 0625/21/O/N/11 [Turn over


12

9 (a) The circuit symbol shown in Fig. 9.1 represents a device often used in electrical equipment.

Fig. 9.1

State the name of this device.

...............................................................................................................................................[1]

(b) Fig. 9.2 shows, in simplified form, the essential parts of a grid system for distributing electrical
energy from a power station to domestic consumers. The device in part (a) is used both at X
and at Y.

132 000 V

power transmission
22 000 V cables and 240 V domestic
station
X pylons Y consumer

Fig. 9.2

(i) Using information from Fig. 9.2, deduce the ratio secondary turns necessary at X and
at Y. primary turns

turns ratio at X = ...............................................................

turns ratio at Y = ...........................................................[3]

(ii) State two reasons why power transmission is cheaper if the voltage across the cables is
very high.

1. ........................................................................................................................................

...........................................................................................................................................

2. ........................................................................................................................................

.......................................................................................................................................[2]

[Total: 6]
© UCLES 2011 0625/21/O/N/11
13

10 (a) Complete the following statement about an electric field.

An electric field is a region in which ........................................................

experiences a ....................................................... [2]

(b) Fig. 10.1 shows two identical light uncharged balls suspended on thin nylon threads.

nylon thread

ball

A B

Fig. 10.1

Ball A is given a positive charge and ball B is given a negative charge.

In the space below, draw a diagram showing the positions that the balls and threads will take,
now that the balls are charged.

[2]

(c) On ball B in your diagram, use labelled arrows to show the directions of

(i) the electrostatic force on the ball (label it E),

(ii) the weight of the ball (label it W),

(iii) the tension force of the thread on the ball (label it T). [2]

(d) Ball B is in equilibrium. State the value of the resultant of forces E, W and T.

resultant = ...........................................................[1]

[Total: 7]

© UCLES 2011 0625/21/O/N/11 [Turn over


14

11 Fig. 11.1 shows a tube that can be used to produce cathode rays. Metal plates above and below
the tube have zero potential difference between them.

Fig. 11.1

(a) On Fig. 11.1, clearly label

(i) the filament cathode (label it C),


(ii) the anode (label it A). [2]

(b) On Fig. 11.1, draw

(i) a battery (label it B), connected to cause thermionic emission of electrons,


(ii) a 1000 V power supply (label it P), connected to cause a beam of cathode rays along the
tube,
(iii) the path of the beam of cathode rays along the tube.
[3]

(c) What is seen when the beam of cathode rays hits the screen?

...................................................................................................................................................

...............................................................................................................................................[1]

(d) A high potential difference is now connected between the two metal plates, so that the lower
plate is positive and the upper plate is negative.

What change is seen on the screen?

...................................................................................................................................................

...............................................................................................................................................[1]

[Total: 7]

© UCLES 2011 0625/21/O/N/11


15

12 The count-rate from a sample of radioactive material is investigated by a team of scientists in a


laboratory.

(a) With the radioactive sample in place, the count-rate from the sample is determined every
7 days, with the results shown below. The background count-rate has already been subtracted.

time / days 0 7 14 21 28
count-rate
1000 550 300 160 90
counts / s

On Fig. 12.1, plot a graph of count-rate from the sample against time. [3]

1000

800

600

count-rate
counts / s

400

200

0
0 5 10 15 20 25 30
time / days

Fig. 12.1

(b) (i) From your graph, find the time at which the count-rate is

1. 800 counts / s, .................................................. days

2. 200 counts / s. .................................................. days [2]

(ii) From the figures in (b)(i), find the value of the half-life of the substance.

half-life = ................................................. days [3]

[Total: 8]
© UCLES 2011 0625/21/O/N/11
16

BLANK PAGE

Permission to reproduce items where third-party owned material protected by copyright is included has been sought and cleared where possible. Every
reasonable effort has been made by the publisher (UCLES) to trace copyright holders, but if any items requiring clearance have unwittingly been included, the
publisher will be pleased to make amends at the earliest possible opportunity.

University of Cambridge International Examinations is part of the Cambridge Assessment Group. Cambridge Assessment is the brand name of University of
Cambridge Local Examinations Syndicate (UCLES), which is itself a department of the University of Cambridge.

© UCLES 2011 0625/21/O/N/11


w
w
w
.X
tr
me
eP
ap
UNIVERSITY OF CAMBRIDGE INTERNATIONAL EXAMINATIONS

er
s
International General Certificate of Secondary Education

.c
om
* 4 8 0 8 1 1 0 5 3 7 *

PHYSICS 0625/21
Paper 2 Core May/June 2012
1 hour 15 minutes
Candidates answer on the Question Paper.
No Additional Materials are required.

READ THESE INSTRUCTIONS FIRST

Write your Centre number, candidate number and name on all the work you hand in.
Write in dark blue or black pen.
You may use a pencil for any diagrams or graphs.
Do not use staples, paper clips, highlighters, glue or correction fluid.
DO NOT WRITE IN ANY BARCODES.
For Examiner’s Use
Answer all questions.
You may lose marks if you do not show your working or if you do not use 1
appropriate units.
Take the weight of 1 kg to be 10 N (i.e. acceleration of free fall = 10 m / s2). 2

At the end of the examination, fasten all your work securely together. 3
The number of marks is given in brackets [ ] at the end of each question or
part question. 4

10

11

12

Total

This document consists of 19 printed pages and 1 blank page.

DC (NF/JG) 43201/2
© UCLES 2012 [Turn over
2

1 A car is travelling along a level road at a steady speed. Fig. 1.1 shows the speedometer in For
the car. A speedometer registers how fast the car is going. Examiner’s
Use

60 80
40 100

20 120
km / hour
0 140

Fig. 1.1

(a) How far, in km, does the car travel in ½ hour at the speed shown in Fig. 1.1?

distance = .......................................... km [3]

© UCLES 2012 0625/21/M/J/12


3

(b) (i) On the axes shown in Fig. 1.2, draw a line representing the motion of the car for the For
½ hour mentioned in (a). Do not go beyond ½ hour. [3] Examiner’s
Use

100
speed
km / hour
80

60

40

20

0
0 0.2 0.4 0.6 0.8 1.0 1.2
time / hours

Fig. 1.2

(ii) At the end of the ½ hour, the car reaches a region where the road begins to rise up
into some mountains. The car climbs the mountains for a further ½ hour.

During the climb, its speed steadily decreases to 30 km / hour. The driver then stops
the car so that he can admire the view.

On Fig. 1.2, draw a line representing the climb and the stopping of the car. [4]

[Total: 10]

© UCLES 2012 0625/21/M/J/12 [Turn over


4

2 A student carries out an experiment to find the density of water, using a method that is For
slightly different from normal. In his method, he starts with a measuring cylinder containing Examiner’s
some water, and then adds more water to that already in the measuring cylinder. Use

His experiment is illustrated in Fig. 2.1.

measuring
cylinder reading 2

reading 1

water

balance

reading 3 reading 4

Fig. 2.1

The readings he obtains are as follows:

reading 1 53 cm3

reading 2 84 cm3

reading 3 205 g

reading 4 238 g

Calculate

(a) the volume of the added water,

volume = ......................................... cm3 [2]

(b) the mass of the added water,

mass = ............................................. g [2]

© UCLES 2012 0625/21/M/J/12


5

(c) the density of water, stating clearly the equation you are using. For
Examiner’s
Use

density = ................................................. [4]

[Total: 8]

© UCLES 2012 0625/21/M/J/12 [Turn over


6

3 A train is passing through a station at constant speed, as shown in Fig. 3.1. The track is For
horizontal. Examiner’s
Use

Fig. 3.1

The engine produces a forward thrust of 70 000 N. There is a 25 000 N force opposing the
motion, due to friction in the wheels.

(a) Mark these forces on Fig. 3.1, using an arrow labelled 70 000 N and an arrow labelled
25 000 N. [2]

(b) The train is travelling at constant speed, so there must be another horizontal force acting
on it.

(i) State the direction of this force.

..................................................................................................................................

(ii) Calculate the size of this force.

size of force = ................................................... N

(iii) Suggest what might be causing this force.

..................................................................................................................................
[3]

(c) Once the train has passed the station, the driver increases the engine’s forward thrust.

All other forces stay the same.

(i) What happens to the train? ......................................................................................

(ii) Why does this happen? ............................................................................................

..................................................................................................................................

..................................................................................................................................
[2]

[Total: 7]

© UCLES 2012 0625/21/M/J/12


7

4 (a) Explain, in terms of molecules, how a gas causes a pressure on the walls of its container. For
Examiner’s
.......................................................................................................................................... Use

..........................................................................................................................................

..........................................................................................................................................

..........................................................................................................................................

..........................................................................................................................................

..................................................................................................................................... [4]

(b) Complete the following two sentences.

(i) At constant temperature, the pressure of a gas increases as its volume

.............................................. .

(ii) At constant volume, the pressure of a gas increases as its temperature

.............................................. .
[2]

[Total: 6]

© UCLES 2012 0625/21/M/J/12 [Turn over


8

5 (a) The principle of conservation of energy states that energy can neither be created nor For
destroyed. Examiner’s
Use

What, then, does happen to the energy supplied to a device such as a motor or a
television?

..........................................................................................................................................

..........................................................................................................................................

..................................................................................................................................... [1]

(b) The television in Fig. 5.1 is switched on to watch a programme. During this time, 720 kJ
of electrical energy is supplied.

electrical energy
input = 720 kJ

light energy
output = 4 kJ sound energy
output = 20 kJ

Fig. 5.1

(i) From the information on Fig. 5.1, find the total energy provided for the viewer to see
and hear the television during this programme.

energy = ............................................ kJ [1]

(ii) Suggest what happens to the rest of the energy supplied.

..................................................................................................................................

..................................................................................................................................

............................................................................................................................. [2]

© UCLES 2012 0625/21/M/J/12


9

(iii) Calculate how much energy is involved in (b)(ii). For


Examiner’s
Use

energy = ............................................ kJ [1]

(iv) Comment on the efficiency of the television.

..................................................................................................................................

..................................................................................................................................

............................................................................................................................. [1]

[Total: 6]

© UCLES 2012 0625/21/M/J/12 [Turn over


10

6 The ray diagram in Fig. 6.1 shows one ray from the top of an object placed to the left of a For
converging lens. Examiner’s
Use

object F2 F1

Fig. 6.1

(a) On Fig. 6.1, use your ruler to draw another ray from the top of the object until it crosses
the ray printed on the diagram. [2]

(b) On Fig. 6.1, draw the image of the object. [1]

(c) Which of the following descriptions fit the image formed by the lens? Tick 3 boxes.

much larger than the object

much smaller than the object

same size as the object

upright

inverted

real [3]

© UCLES 2012 0625/21/M/J/12


11

(d) The object is moved to a position further from the lens. For
Examiner’s
What differences are seen in the image, compared with the previous image? Use

..........................................................................................................................................

..................................................................................................................................... [2]

[Total: 8]

© UCLES 2012 0625/21/M/J/12 [Turn over


12

7 (a) Remote controllers for television sets send a beam of electromagnetic radiation to the For
television. Examiner’s
Use

Which region of the electromagnetic spectrum is used? Tick one box.

microwaves

infra-red

visible

ultra-violet

X-rays [1]

(b) Modern warfare often uses heat-seeking missiles.

Which region of the electromagnetic spectrum is used? Tick one box.

microwaves

infra-red

visible

ultra-violet

X-rays [1]

(c) Injured legs may be checked for possible fractures using electromagnetic radiation.

Which region of the electromagnetic spectrum is used? Tick one box.

microwaves

infra-red

visible

ultra-violet

X-rays [1]

© UCLES 2012 0625/21/M/J/12


13

(d) Mobile phones communicate using electromagnetic radiation. For


Examiner’s
Which region of the electromagnetic spectrum is used? Tick one box. Use

microwaves

infra-red

visible

ultra-violet

X-rays [1]

[Total: 4]

© UCLES 2012 0625/21/M/J/12 [Turn over


14

8 (a) Complete the following sentences. For


Examiner’s
(i) An electric current exists in a wire when .................................................. are made Use

to flow in the wire. [1]

(ii) The current in a wire may be measured using an instrument called

.................................................... . [1]

(iii) The potential difference across a wire may be measured by connecting

................................................... across the wire. [1]

(b) A length of resistance wire is connected in a simple series circuit.

The current in it is 0.8 A. The potential difference across it is 9.6 V.

Calculate the resistance of the wire.

resistance = ................................................. [4]

(c) The resistance wire in (b) is replaced by a greater length of wire from the same reel.

Without further calculation, state the effect this has on

(i) the resistance in the circuit,

..................................................................................................................................

(ii) the current in the new wire when there is a potential difference of 9.6 V across it, as
before.

..................................................................................................................................
[2]

[Total: 9]

© UCLES 2012 0625/21/M/J/12


15

9 The coil in the d.c. motor in Fig. 9.1 is rotating as shown. For
Examiner’s
Use

rotation
magnet

– +
y
batter

Fig. 9.1

(a) On Fig. 9.1, clearly label the coil. [1]

(b) State two things that could be done to the apparatus shown in Fig. 9.1 in order to make
the coil rotate more rapidly.

1. ......................................................................................................................................

2. ......................................................................................................................................
[2]

(c) Suggest how the coil could be made to rotate in the opposite direction.

..........................................................................................................................................

..................................................................................................................................... [1]

[Total: 4]

© UCLES 2012 0625/21/M/J/12 [Turn over


16

10 Fig. 10.1 shows four different types of switch. For


Examiner’s
Use

wall-mounted
switch

ceiling-mounted
pull-cord
switch flush
wall-mounted
switch

nylon cord
plug

plug switch

metal
sections

Fig. 10.1

(a) In the space below, draw the circuit symbol for a switch.

[1]

(b) (i) Which one of the switches is definitely dangerous to use with mains voltages?

..................................................................................................................................

(ii) State the reason for your answer to (b)(i).

..................................................................................................................................

..................................................................................................................................
[2]

© UCLES 2012 0625/21/M/J/12


17

(c) A laundry, where clothes are washed, is likely to have lots of steam and condensation. For
Examiner’s
(i) Which switch is the most suitable for turning the lights on or off from within the Use
laundry?

..................................................................................................................................

(ii) State the reason for your answer to (c)(i).

..................................................................................................................................

..................................................................................................................................

..................................................................................................................................
[3]

(d) The laundry is lit by three mains-voltage lamps. Fig. 10.2 shows the mains supply and
the three lamps.

mains
supply

Fig. 10.2

Complete Fig. 10.2 by adding the switch and the wiring that will allow all three lamps to
light at full brightness when the switch is on. [2]

[Total: 8]

© UCLES 2012 0625/21/M/J/12 [Turn over


18

11 Fig. 11.1 shows an electron beam about to enter, at point A, the electric field between two For
charged metal plates. Examiner’s
Use

electron
beam A

Fig. 11.1

(a) On Fig. 11.1, carefully draw the path of the electron beam between A and the line BC.
[3]

(b) The voltage across the plates is reversed. State what difference this makes to the path
of the electron beam.

..........................................................................................................................................

..................................................................................................................................... [1]

[Total: 4]

© UCLES 2012 0625/21/M/J/12


19

12 The table below gives details about some radioactive substances. For
Examiner’s
Use
substance symbol type of radiation emitted half-life

barium-139 139 Ba beta (β) 85 minutes


56

silver-110 110 Ag beta (β) 24 seconds


47

technetium-99m 99 Tc gamma (γ) 6.0 hours


43

thorium-232 232 Th
90
alpha (α) 1.4 × 1010 years

(a) Which of these substances has the greatest number of particles in the nucleus of its
atoms?

..................................................................................................................................... [1]

(b) Which of these substances has the least number of electrons in the orbits of a neutral
atom?

..................................................................................................................................... [1]

(c) Which of these substances are emitting particles?

..........................................................................................................................................

..................................................................................................................................... [2]

(d) Samples of each of these substances are decaying. Each sample starts with the same
number of atoms.

Which sample decays the most in one hour?

..................................................................................................................................... [1]

(e) In the investigation of a blood circulation problem, a patient is given an injection


containing one of these substances. The radiation needs to be detectable from outside
the body.

Which of the substances might be suitable for this use?

..................................................................................................................................... [1]

[Total: 6]

© UCLES 2012 0625/21/M/J/12


20

BLANK PAGE

Permission to reproduce items where third-party owned material protected by copyright is included has been sought and cleared where possible. Every
reasonable effort has been made by the publisher (UCLES) to trace copyright holders, but if any items requiring clearance have unwittingly been included, the
publisher will be pleased to make amends at the earliest possible opportunity.

University of Cambridge International Examinations is part of the Cambridge Assessment Group. Cambridge Assessment is the brand name of University of
Cambridge Local Examinations Syndicate (UCLES), which is itself a department of the University of Cambridge.

© UCLES 2012 0625/21/M/J/12


w
w
w
.X
tr
me
eP
ap
UNIVERSITY OF CAMBRIDGE INTERNATIONAL EXAMINATIONS

er
s
International General Certificate of Secondary Education

.c
om
* 0 2 9 5 0 5 8 2 3 2 *

PHYSICS 0625/21
Paper 2 Core October/November 2012
1 hour 15 minutes
Candidates answer on the Question Paper.
No Additional Materials are required.

READ THESE INSTRUCTIONS FIRST


For Examiner’s Use
Write your Centre number, candidate number and name on all the work you
hand in. 1
Write in dark blue or black pen.
You may use a pencil for any diagrams or graphs. 2
Do not use staples, paper clips, highlighters, glue or correction fluid.
DO NOT WRITE IN ANY BARCODES.
3
Answer all questions.
You may lose marks if you do not show your working or if you do not use 4
appropriate units.
Take the weight of 1 kg to be 10 N (i.e. acceleration of free fall = 10 m / s2). 5
At the end of the examination, fasten all your work securely together.
The number of marks is given in brackets [ ] at the end of each question or part 6
question.
7

10

11

12

Total

This document consists of 16 printed pages.

DC (CW/SW) 49779/4
© UCLES 2012 [Turn over
2

For
Examiner’s
1 (a) State the name that is given to the turning effect of a force. Use

...................................................................................................................................... [1]

(b) A gate has rusty hinges that are very stiff. A man opens the gate by pulling on it, as
shown in Fig. 1.1.

Fig. 1.1

After he has passed through the opening, he closes the gate behind him.

When closing the gate, the man pulls it at a point halfway along its length.

State two differences between his force when closing the gate and his force when he
opened the gate.

1. ......................................................................................................................................

2. ......................................................................................................................................
[2]

(c) Suggest one way of reducing the force needed to open the gate.

...................................................................................................................................... [1]

[Total: 4]

© UCLES 2012 0625/21/O/N/12


3

2 (a) State the equation linking the density of a substance with its mass and volume. For
Examiner’s
.......................................................................................................................................... Use

...................................................................................................................................... [1]

(b) When oil leaks out of a damaged oil-tanker, it forms a very thin layer of oil, known as an
oil slick, on the water.

One such oil slick covers an approximately rectangular area measuring 2.5 × 104 m by
6.0 × 103 m.

The oil slick is 3.0 × 10–6 m (0.0000030 m) thick.

(i) Calculate the volume of the oil slick.

volume = ........................................... m3 [3]

(ii) The density of the oil is 900 kg / m3.

Calculate the mass of oil in the slick.

mass = ............................................ kg [2]

[Total: 6]

© UCLES 2012 0625/21/O/N/12 [Turn over


4

3 Fig. 3.1 shows four runners at the start of an 80 m race on a school sports day. For
Examiner’s
Use
starting pistol timekeeper

80 m

Fig. 3.1 (not to scale)

(a) Sound travels at 320 m / s.

Calculate the time taken for the sound from the starting pistol to reach the timekeeper.

time = .............................................. s [3]

(b) The timekeeper takes 0.20 s to react after hearing the sound and then starts the
stopwatch.

He makes no other experimental inaccuracies.

(i) By how much will his time for the race be in error?

time error = .............................................. s [2]

(ii) Suggest how he can reduce this error, whilst still using the same stopwatch.

..................................................................................................................................

..................................................................................................................................

.............................................................................................................................. [1]

© UCLES 2012 0625/21/O/N/12


5

(c) When he stops the stopwatch as the winner crosses the finishing line, the appearance For
of the stopwatch is as shown in Fig. 3.2. Examiner’s
Use

55 0 5
50 min 10
5

45 15
seconds
40 20
35
30 25

Fig. 3.2

How long did the winner actually take to run the race?

time = .............................................. s [2]

[Total: 8]

© UCLES 2012 0625/21/O/N/12 [Turn over


6

4 An archer pulls the string of his bow, and moves the arrow to the position shown in Fig. 4.1. For
He then releases the string so that the arrow is fired towards a target. Examiner’s
Use

bow

string

arrow

Fig. 4.1

(a) The archer does work on the bow.

When is this work done? Tick one box.

as the string is pulled back to the position shown in Fig. 4.1

whilst holding the string in the position shown in Fig. 4.1

after releasing the string to fire the arrow [1]

(b) What type of energy is stored in the bow because it is bent?

...................................................................................................................................... [1]

(c) What type of energy does the arrow have because it is moving?

...................................................................................................................................... [1]

(d) On another occasion, the archer fires the arrow so that it rises up to a maximum height
before falling back down to the ground.

Use words from the following list to complete the sentences below.

gravitational potential, kinetic, thermal, maximum, minimum, zero

As the arrow rises, its ..................................................... energy increases. At the top of

the flight, this energy is at a ..................................................... . As the arrow falls, this

energy is converted into ..................................................... energy. When it hits the ground,

the energy of the arrow is converted into ..................................................... energy.


[4]

[Total: 7]
© UCLES 2012 0625/21/O/N/12
7

5 (a) A metal ruler with a rectangular cross-section is heated in an oven. For


Examiner’s
(i) State two things that happen to the atoms of the metal. Use

1. ...............................................................................................................................

2. ...............................................................................................................................
[2]

(ii) State what happens to

1. the length of the ruler, ..........................................................................................

2. the width of the ruler, ............................................................................................

3. the thickness of the ruler. ......................................................................................


[2]

(b) The nut in Fig. 5.1 has become jammed on the bolt, so that it will not rotate.

nut

bolt

very hot
flame

Fig. 5.1

Explain why heating the nut with a very hot flame is likely to free the jammed nut.

..........................................................................................................................................

..........................................................................................................................................

...................................................................................................................................... [2]

[Total: 6]

© UCLES 2012 0625/21/O/N/12 [Turn over


8

6 (a) In Fig. 6.1, a ray of red light is shown passing through a triangular glass prism and on to For
another prism that is identical but upside down. Examiner’s
Use

t
l igh
red

Fig. 6.1

(i) The angle of incidence of the red light at the first surface is shown on Fig. 6.1 as i.

On Fig. 6.1, use the letter r to mark clearly the angle of refraction at the first surface.
[1]

(ii) On Fig. 6.1, complete the path of the ray through the right-hand prism and out into
the air again. Label the emergent ray “line R”. [3]

(iii) The beam of red light is moved so that it shines into the right-hand prism along
line R.

Using the letter P, mark clearly the point where this ray will emerge from the left-
hand prism. [1]

(b) On another occasion, a beam containing a mixture of red and blue light is shone into a
prism, as shown in Fig. 6.2.

i red lig
ht
t
igh
l
red + ht
ig
el
blu

Fig. 6.2

(i) On Fig. 6.2, draw the path of the blue light through the prism and out into the air
again. [3]

© UCLES 2012 0625/21/O/N/12


9

(ii) Refraction is occurring at the first surface. For


Examiner’s
Which of the following is also occurring? Tick one box. Use

diffraction

dispersion

focusing

total internal reflection [1]

[Total: 9]

© UCLES 2012 0625/21/O/N/12 [Turn over


10

7 Fig. 7.1 shows a compass needle that has come to rest in the Earth’s magnetic field. For
Examiner’s
Use
compass
needle

N S

Fig. 7.1

(a) On Fig. 7.1, draw an arrow pointing towards the north pole of the Earth. [1]

(b) The S pole of a bar magnet is brought towards the S pole of the compass needle, as
shown in Fig. 7.2.

N S S

Fig. 7.2

Describe what is seen happening to the compass needle as this is done.

..........................................................................................................................................

..........................................................................................................................................

...................................................................................................................................... [2]

© UCLES 2012 0625/21/O/N/12


11

(c) The magnet in (b) is removed and a horizontal wire is positioned above the compass For
needle, as shown in Fig. 7.3. Examiner’s
Use

wire

N S

Fig. 7.3

When there is a current in the wire, the compass needle rotates through a small angle.

Suggest why this rotation occurs.

..........................................................................................................................................

..........................................................................................................................................

...................................................................................................................................... [2]

[Total: 5]

© UCLES 2012 0625/21/O/N/12 [Turn over


12

8 Fig. 8.1 shows a cell. For


Examiner’s
Use

Fig. 8.1

(a) What does the 1.5 V indicate about the cell?

...................................................................................................................................... [2]

(b) Three cells identical to the cell in Fig. 8.1 make up a 4.5 V battery. The battery is
connected in series with a 180 Ω resistor.

Calculate the current in the circuit.

current = .................................................. [4]

(c) A second 180 Ω resistor is connected in parallel with the 180 Ω resistor from (b).

(i) In the space below, draw the circuit diagram of the two resistors in parallel,
connected to the battery. Use standard symbols.

[3]

(ii) State the value of

1. the potential difference across the second 180 Ω resistor, ...................................

2. the current in the second 180 Ω resistor. ..............................................................


[2]

[Total: 11]
© UCLES 2012 0625/21/O/N/12
13

9 Fig. 9.1 shows a time-delay circuit that includes a capacitor C and a resistor of very high For
resistance. Examiner’s
Use

C V

Fig. 9.1

(a) On Fig. 9.1, use the letter S to label the switch. [1]

(b) When the switch is open, the voltmeter in the circuit registers zero.

After the switch has been closed, what happens, if anything, to

(i) the charge in the circuit,

..................................................................................................................................

.............................................................................................................................. [1]

(ii) the reading on the voltmeter?

..................................................................................................................................

.............................................................................................................................. [2]

(c) The switch is now opened again.

State what happens, if anything, to the reading on the voltmeter.

..........................................................................................................................................

...................................................................................................................................... [1]

[Total: 5]

© UCLES 2012 0625/21/O/N/12 [Turn over


14

10 A 240 V a.c. mains supply is connected to the primary coil of the transformer shown in For
Fig. 10.1. A lamp that gives full brightness with a 6 V supply is connected to the secondary Examiner’s
coil. Use

240 V 6V
mains lamp

8000 turns soft iron

Fig. 10.1

(a) Name a suitable material from which the coils may be made.

...................................................................................................................................... [1]

(b) State the name given to the part of the transformer that is made of soft iron (see
Fig. 10.1).

...................................................................................................................................... [1]

(c) Calculate the number of turns of wire in the secondary coil that will enable the lamp to
light at full brightness.

number of turns = .................................................. [3]

(d) State what would happen to the lamp if the number of turns in the secondary coil was

(i) much less than that calculated in (c),

.............................................................................................................................. [1]

(ii) much more than that calculated in (c).

.............................................................................................................................. [1]

[Total: 7]

© UCLES 2012 0625/21/O/N/12


15

11 The apparatus for investigating the absorption of the emissions from a radioactive source is For
shown in Fig. 11.1. Examiner’s
Use

radioactive absorbing detector


source material

Fig. 11.1

The source and detector are about 2 cm apart. The detector is connected to a scaler, which
measures the count rate.

Different absorbing materials are placed between the source and the detector.

The table below shows the count rate obtained with each of five absorbers.

count rate
absorbing material
counts / s
air 523
sheet of paper 523
0.5 mm of aluminium 391
10 mm of aluminium 214
10 mm of lead 122

(a) How can you tell that the source is not emitting any α-particles?

..........................................................................................................................................

..........................................................................................................................................

...................................................................................................................................... [2]

(b) What is the evidence that β-particles are being emitted?

..........................................................................................................................................

..........................................................................................................................................

...................................................................................................................................... [2]

(c) What is the evidence that γ-rays are being emitted?

..........................................................................................................................................

..........................................................................................................................................

...................................................................................................................................... [2]

[Total: 6]

© UCLES 2012 0625/21/O/N/12 [Turn over


16

4 4
12 (a) The symbol for an α-particle is either 2 α or 2 He. For
Examiner’s
Use
(i) What does the 4 indicate about an α-particle?

.............................................................................................................................. [1]

(ii) What does the 2 indicate about an α-particle?

.............................................................................................................................. [1]
0 0
(b) The symbol for a β-particle is either –1β or –1e.

(i) What does the 0 indicate about a β-particle?

.............................................................................................................................. [1]

(ii) What does the –1 indicate about a β-particle?

.............................................................................................................................. [1]

(c) The list below gives, in nuclide notation, the symbols of five radioactive nuclides.
240 244 248 250 250
94Pu 96Cm 98Cf 97Bk 98Cf

244
(i) 96Cm decays by emitting an α-particle.

Into which of the other nuclides in the list does it decay? .................................... [1]
250
(ii) 97Bk decays by emitting a β-particle.

Into which of the other nuclides in the list does it decay? .................................... [1]

[Total: 6]

Permission to reproduce items where third-party owned material protected by copyright is included has been sought and cleared where possible. Every
reasonable effort has been made by the publisher (UCLES) to trace copyright holders, but if any items requiring clearance have unwittingly been included, the
publisher will be pleased to make amends at the earliest possible opportunity.

University of Cambridge International Examinations is part of the Cambridge Assessment Group. Cambridge Assessment is the brand name of University of
Cambridge Local Examinations Syndicate (UCLES), which is itself a department of the University of Cambridge.

© UCLES 2012 0625/21/O/N/12


EXTENSION
PAPER THREE

3
Candidate
Centre Number Number

Candidate Name

International General Certificate of Secondary Education


UNIVERSITY OF CAMBRIDGE LOCAL EXAMINATIONS SYNDICATE

PHYSICS 0625/3
PAPER 3
Tuesday 25 MAY 1999 Morning 1 hour 15 minutes

Candidates answer on the question paper.


Additional materials:
Electronic calculator and/or Mathematical tables
Protractor
Ruler (30 cm)

TIME 1 hour 15 minutes

INSTRUCTIONS TO CANDIDATES
Write your name, Centre number and candidate number in the spaces at the top of this page.
Answer all questions.
Write your answers in the spaces provided on the question paper.

INFORMATION FOR CANDIDATES


The number of marks is given in brackets [ ] at the end of each question or part question.

FOR EXAMINER’S USE

TOTAL

This question paper consists of 13 printed pages and 3 blank pages.


SB (SLC) QF91700/5
© UCLES 1999 [Turn over
2 For
Examiner’s
Use
1 Fig. 1.1 shows a plan view of a rotating sprayer used for the watering of crops.
topicmass
topicweight
topicdensity
topicvolume
topicprojectiles

W E

S O

holes through which


water sprays out horizontally

P direction of rotation

Fig. 1.1

(a) The device rotates about O at a constant rate of 0.2 revolutions per second. OP is 10 m
long.
Calculate the speed of the point P. (The circumference of a circle is 2π x radius.)

speed = ........................................... [4]

(b) (i) Use your answer to (a) to write down the velocity of the point P when P is at the
point shown in Fig. 1.1.

...................................................................................................................................

...................................................................................................................................

(ii) Explain why the speed of point P is constant but its velocity changes as the
sprayer rotates.

...................................................................................................................................

...................................................................................................................................

...................................................................................................................................

...................................................................................................................................
[4]

0625/3 S99
3 For
Examiner’s
Use
(c) Explain how you know that there is a net force at the end of the arm P, acting towards O.

..........................................................................................................................................

..........................................................................................................................................

..........................................................................................................................................

..........................................................................................................................................

......................................................................................................................................[3]

Question 1 is continued on page 4

0625/3 S99 [Turn over


4 For
Examiner’s
Use
(d) Water is forced out of the sprayer horizontally. Fig. 1.2 shows how the horizontal speed
of the water changes between leaving the sprayer and hitting the ground.

12

10
horizontal speed
m/s 8

0
0 0.1 0.2 0.3 0.4 0.5 0.6 0.7
time/s

Fig. 1.2

(i) Explain

1. why the horizontal speed decreases slightly over the 0.6 s of the motion,

...................................................................................................................................

...................................................................................................................................

2. why the line is approximately vertical at 0.6 s.

...................................................................................................................................

...................................................................................................................................

(ii) The acceleration of free fall is 10 m/s2. Calculate

1. the height above the ground at which the water leaves the sprayer,

height = ...........................................

2. the horizontal distance travelled by the water.

distance = ...........................................
[7]
0625/3 S99
5 For
Examiner’s
Use
(e) Fig. 1.3 shows the path of the water at one instant. The pressure of the water on an
object placed at point Q is much greater than the pressure of the water on the same
object when placed at point R.

Q
sprayer

ground
R

Fig. 1.3

The pressure exerted by the water on the object at point Q is 5 x 105 N/m2 and the
pressure on the object at R is 1 x 105 N/m2.
Explain this decrease in pressure.

..........................................................................................................................................

..........................................................................................................................................

..........................................................................................................................................

..........................................................................................................................................

......................................................................................................................................[4]

0625/3 S99 [Turn over


6 For
Examiner’s
Use
2 A student attempted to find the specific heat capacity of water using the following data
topicturningeffect
topicturning
topiceffect
obtained from the heating system of a small swimming pool:

mass of water in the pool, heating system and circulation pipes, 54 000 kg;
power of the heating system, 30 kW;
rise in temperature, 2 °C in 5 hours (18 000 s).

(a) Assuming no energy loss, use these data to calculate a value for the specific heat
capacity of water. Show your working.

specific heat capacity .............................................. [6]

0625/3 S99
7 For
Examiner’s
Use
(b) The student found that the value for the specific heat capacity of water, worked out by
this method, was higher than the accepted value.

The average temperature of the water in the pool during the test period was 24 °C,
whilst the average temperature of the air was 19 °C.

(i) Describe, in molecular terms, ways in which the water loses heat from its surface.

...................................................................................................................................

...................................................................................................................................

...................................................................................................................................

...................................................................................................................................

...................................................................................................................................

...................................................................................................................................

...................................................................................................................................

...................................................................................................................................

(ii) Explain why the loss of heat from the water led to the student’s higher value.

...................................................................................................................................

...................................................................................................................................

...................................................................................................................................

...................................................................................................................................
[7]

0625/3 S99 [Turn over


8 For
Examiner’s
Use
3 (a) A converging lens of focal length 4.0 cm is used to produce a virtual image which is
topicflow
topicmodels
topicrefraction
3 times the height of the object.
topicmolecularmodels
topicmolecular

Fig. 3.1 shows the lens position and the focal length PF. The length PF is to scale.
The object and the image are both on the left-hand side of the lens but their positions
are not shown.

line which passes through the top of the image


lens position

line which passes through the top of the object

line which passes through the bottom of the image and the object
P F

Fig. 3.1

Draw rays on Fig. 3.1 and determine

(i) the scale used, ..........................................................................................................

(ii) the distance of the image from the lens, ...................................................................

(iii) the distance of the object from the lens. ...................................................................


[6]

0625/3 S99
9 For
Examiner’s
Use
(b) Fig. 3.2 shows a wide parallel beam of monochromatic light incident on a block of glass
at an angle of 37°.

37°

90° air
glass

Fig. 3.2

(i) What is meant by the word monochromatic ?

...................................................................................................................................

...................................................................................................................................

(ii) State the approximate speed of light in air.

...................................................................................................................................

(iii) The angle of refraction in the glass is 22°. Calculate the refractive index of this
glass.

refractive index = .....................................

(iv) On Fig. 3.2, use your protractor to draw in the path of the beam of light in the glass.
[7]

0625/3 S99 [Turn over


10 For
Examiner’s
Use
4 Fig. 4.1 shows part of a cathode-ray tube.
topicenergy
An electron beam PQ is entering the region between two horizontal, charged metal plates.
topicenergytransformations
topictransformations
topicthermometers
topicapparentdepth
topiccharge
topicelectrons
topicelectronsinamagneticfield
topicfield
topicmagnetic

tube wall S
+V
positively
charged plate
vacuum
R

P Q

negatively
0V charged plate

Fig. 4.1

(a) (i) On Fig. 4.1, draw the electron beam from Q to show its path between the charged
plates.

(ii) Explain any change of direction of the electron beam when it is between the
charged plates.

...................................................................................................................................

...................................................................................................................................

...................................................................................................................................

(iii) On Fig. 4.1, show the direction of the conventional current in the electron beam by
drawing an arrow and labelling it D.
[5]

(b) The voltage across the plates is increased so that one of the plates collects 1014
electrons in 10 s. Each electron carries a charge of 1.6 x 10–19 C.

(i) Calculate the total charge collected by the plate in 10 s.

charge = ..........................................

(ii) State an equation linking charge and current. Hence calculate the current in wire RS.

...................................................................................................................................

current = ..........................................
[4]
0625/3 S99
11 For
Examiner’s
Use
(c) Air containing charged dust particles flows between two metal plates. A high potential
difference is connected across the plates as illustrated in Fig. 4.2.

A +

direction of air flow


containing charged large p.d.
dust particles

Fig. 4.2

The charged particles are attracted to the upper plate and move through a potential
difference of 10 000 V. The ammeter records a current of 2.1 x 10 –6 A.

Calculate

(i) the energy supplied by the voltage source in 10 minutes (600 s),

energy = ...........................................

(ii) the power supplied.

power = ...........................................
[6]

0625/3 S99 [Turn over


12 For
Examiner’s
Use
(d) Fig. 4.3 shows a beam of electrons entering the magnetic field of a coil. This magnetic
field is directed into the paper.

beam of coil
electrons vacuum

+
12 V

Fig. 4.3

(i) On Fig. 4.3, sketch the path of the electron beam until it hits the end of the tube.
Explain your choice of path.

...................................................................................................................................

...................................................................................................................................

...................................................................................................................................

...................................................................................................................................

(ii) The resistance of the coil producing the magnetic field is 100 Ω. Calculate the
current in the coil.

current = ..........................................

(iii) State the effect on the electron deflection of increasing and reversing the potential
difference connected across the coil.

...................................................................................................................................

...................................................................................................................................
[7]

0625/3 S99
13 For
Examiner’s
Use
5 (a) A laboratory needs to find a radioactive isotope which will produce very intense
topicspeed
topicspeedofsoundandlight
topicsound
ionisation of air.

The apparatus is shown in Fig. 5.1.

thick copper wire


+
insulator
high potential difference
5000 V
copper gauze

radioactive isotope

Fig. 5.1

(i) Explain why sparks jump between the gauze and the wire when a radioactive
isotope with high ionising properties is brought near the gauze.

...................................................................................................................................

...................................................................................................................................

...................................................................................................................................

...................................................................................................................................

(ii) An α-emitting source, a β-emitting source and a γ-emitting source, each of the
same activity, are tested. One source gives no sparks at all, the second gives only
a few sparks per second and the third many sparks per second. State the relative
quantities of ionisation produced by each type of emitter.

...................................................................................................................................

...................................................................................................................................

...................................................................................................................................
[6]

(b) Some of the results of a comparison between α-particles, β-particles and γ-rays are
shown in the table below.

α β γ

mass 4 units

constitution 2 protons + 2 neutrons

charge +2 units

Complete the table by filling in the blank boxes. [4]

0625/3 S99
Candidate
Centre Number Number

Candidate Name

International General Certificate of Secondary Education


UNIVERSITY OF CAMBRIDGE LOCAL EXAMINATIONS SYNDICATE

PHYSICS 0625/3
PAPER 3
Thursday 16 NOVEMBER 1999 Morning 1 hour 15 minutes
Candidates answer on the question paper.
Additional materials:
Electronic calculator and/or Mathematical tables

TIME 1 hour 15 minutes

INSTRUCTIONS TO CANDIDATES
Write your name, Centre number and candidate number in the spaces at the top of this page.
Answer all questions.
Write your answers in the spaces provided on the question paper.

INFORMATION FOR CANDIDATES


The number of marks is given in brackets [ ] at the end of each question or part question.

FOR EXAMINER’S USE

TOTAL

This question paper consists of 12 printed pages.


SB (CW/DJ) QK93706/2
© UCLES 1999 [Turn over
2 For
Examiner’s
Use
1 Fig. 1.1 shows the outline of a machine for driving steel pillars (called piles) into the ground.
topicmass
topicvolume
topicweight
topickineticenergy
topickinetic
topicprojectiles
topicdensity

suspension
steel mass

tube

steel pile

ground

Fig. 1.1

The steel mass is raised by an electric motor and then falls under gravity.
The falling steel has a mass of 200 kg and falls a distance of 6.0 m.

(a) The acceleration of free fall is 10 m/s2. Calculate

(i) the potential energy gained by the mass each time it is raised,

potential energy gained = .....................................

(ii) the maximum speed at which the mass hits the pile.

speed = .....................................
[7]

0625/3 W99
3 For
Examiner’s
Use
(b) When the mass hits the pile, it has kinetic energy. This energy is transformed into other
forms of energy as the speed of the falling mass rapidly reduces to zero. As this
happens, the pile is forced a small distance into the ground.

(i) State the energy conversions which take place, starting from the kinetic energy of
the falling mass.

..................................................................................................................................

..................................................................................................................................

..................................................................................................................................

..................................................................................................................................

(ii) Explain how a large force is produced when the pile is driven a short distance into
the ground.

..................................................................................................................................

..................................................................................................................................

..................................................................................................................................

..................................................................................................................................
[8]

(c) In raising the steel mass 6.0 m, the electric motor uses more energy than that
calculated in (a) (i).

Write down and explain two causes of this higher energy requirement.

1. .....................................................................................................................................

..........................................................................................................................................

..........................................................................................................................................

..........................................................................................................................................

2. .....................................................................................................................................

..........................................................................................................................................

..........................................................................................................................................

......................................................................................................................................[4]

0625/3 W99 [Turn over


4 For
Examiner’s
Use
(d) The equipment design is changed so that when the mass falls once, the pile is driven
further into the ground than before the design was changed.

Suggest three changes that could be made to do this.

1. .....................................................................................................................................

..........................................................................................................................................

2. .....................................................................................................................................

..........................................................................................................................................

3. .....................................................................................................................................

......................................................................................................................................[3]

;;;
2 Fig. 2.1 shows a piece of apparatus which could be used to find the specific heat capacity of
topicenergy
topiceffect
a metal at high temperatures.
topicturningeffect
topicturning
topicthermocouple

;;;
electric heater inserted
lagging centrally in the block

;;;
metal thermocouple inserted
block in the block

Fig. 2.1

Results from an experiment using the apparatus are recorded as follows:


mass of the metal block, 1.0 kg;
power of the heater, 200 W;
time for which the heater is switched on, 2.5 minutes (150 s);
rise in temperature during this time, from 160 °C to 210 °C.

(a) Describe the experimental steps which were taken to obtain these results.

..........................................................................................................................................

..........................................................................................................................................

..........................................................................................................................................

..........................................................................................................................................

..........................................................................................................................................

......................................................................................................................................[3]

0625/3 W99
5 For
Examiner’s
Use
(b) Use the results to calculate an average value for the specific heat capacity of the metal
over this temperature range.

specific heat capacity = ..................................... [4]

(c) The temperature of the metal was measured by using a thermocouple.

(i) Draw a labelled diagram of a thermocouple being used as a thermometer.

(ii) Describe the action of a thermocouple when measuring a temperature change.

..................................................................................................................................

..................................................................................................................................

..................................................................................................................................

..................................................................................................................................

(iii) Suggest two reasons why use of a thermocouple might have an advantage over a
mercury-in-glass thermometer.

1. ..............................................................................................................................

..................................................................................................................................

2. ..............................................................................................................................

..................................................................................................................................
[6]

0625/3 W99 [Turn over


6 For
Examiner’s
Use
3 Fig. 3.1 shows a ray of light, PQRS, passing along a simple optical fibre.
topicmolecularmodels
topiclens
topicmolecular
topicmodels
topicrefraction
topicrefraction
topicreflection
topictotal
topicinternal
topicangle
topictotalinternalreflection

air
Q
P 85° R
85°
S
T N
M

glass
air

Fig. 3.1

(a) Calculate the angle between the ray PQ and the ray RS.

angle = ..................................... [2]

(b) Explain why the ray PQ does not leave the fibre at Q.

..........................................................................................................................................

..........................................................................................................................................

......................................................................................................................................[2]

(c) Another ray TQ also strikes the surface at Q.


The refractive index of the glass is 1.50.

(i) Calculate the critical angle for this glass.

critical angle = .....................................

(ii) Explain why the ray TQ leaves the fibre.

..................................................................................................................................

..................................................................................................................................

..................................................................................................................................
[4]

0625/3 W99
7 For
Examiner’s
Use
(d) The light waves travelling towards Q are monochromatic and have a frequency of
4 x 1014 Hz and a wavelength of 5 x 10–7 m.

(i) What is meant by monochromatic?

..................................................................................................................................

..................................................................................................................................

(ii) Calculate the speed of these waves in the glass.

speed = .....................................

(iii) Waves travelling along TQ pass into the air. The refractive index of the glass
is 1.50.

Write down an expression from which the speed of the light waves in air could be
found.

..............................................................................................................................[5]

0625/3 W99 [Turn over


8 For
Examiner’s
Use
4 Fig. 4.1 is a block diagram of an electrical generating and distribution system.
topictransformations
topicenergy
topicenergytransformations
topicapparentdepth
topicelectronsinamagneticfield
topicelectrons
topicelectric
topiccharge

long supply cables


(several km)

turbine generator transformer transformer consumer


No. 1 No. 2 circuits

Fig. 4.1

(a) The generator produces an e.m.f. by a process called electromagnetic induction.

(i) Name two factors and state how they are changed in order to increase the output
e.m.f. of the generator.

1. .............................................................................................................................

..................................................................................................................................

2. .............................................................................................................................

..................................................................................................................................

(ii) Explain what is meant by the statement ‘the induced e.m.f. acts in such a direction
as to produce effects to oppose the change causing it’.

..................................................................................................................................

..................................................................................................................................

..................................................................................................................................

..................................................................................................................................
[6]

0625/3 W99
9 For
Examiner’s
Use
(b) (i) Fig. 4.2 shows the basic parts of transformer No. 1 which is 100% efficient.

input output to
from generator supply cables
400 V, 80 A 30 000 V

Fig. 4.2

Using the information on Fig 4.2, calculate the current in the supply cables.

current = .....................................

(ii) Describe the function of transformer No. 2.

..................................................................................................................................

..................................................................................................................................

(iii) Explain why the use of the two transformers results in a big reduction in power loss
in the supply cables.

..................................................................................................................................

..................................................................................................................................

..................................................................................................................................

..................................................................................................................................
[6]

0625/3 W99 [Turn over


10 For
Examiner’s
Use
(c) Fig. 4.3 shows one of the consumer circuits with three electrical appliances R, S and T,
connected into the circuit.

7.7 A X Y
4.6 A 2.3 A

110 V 24 Ω R 48 Ω S T

Fig. 4.3

Using the current, voltage and resistance values shown on Fig. 4.3, calculate

(i) the current at point X and at point Y,

current at X = .....................................

current at Y = .....................................

(ii) the resistance of appliance T,

resistance = .....................................

(iii) the combined resistance of appliances R and S,

resistance = .....................................

(iv) the power developed in appliance R,

power = .....................................

(v) the energy converted by the appliance S in 2 minutes (120 s).

energy converted = .....................................


[10]

0625/3 W99
11 For
Examiner’s
Use
5 Lengths of steel may be joined by welding them together, as iillustrated in Fig 5.1.
topicresistance

steel weld

steel

Fig. 5.1

A liquid radioactive source is to be used to test that the welds joining lengths of steel pipe
are of equal thickness.

The diameter of the pipes is 120 mm and the pipe wall thickness is 5 mm.

The liquid runs through the pipes whilst a suitable detector moves around the outside of the
joints.

(a) With the aid of a labelled diagram, explain how this method detects places where the
welds are thinner than 5 mm.

..........................................................................................................................................

..........................................................................................................................................

..........................................................................................................................................

......................................................................................................................................[3]

0625/3 W99 [Turn over


12 For
Examiner’s
Use
(b) In order to find out the most suitable type of isotope for this purpose, tests were carried
out on the ability of the radiations from an α-emitter, a β-emitter and a γ-emitter to
penetrate steel.

(i) Write down what you would expect to be the results of these tests.

α-emitter ..................................................................................................................

..................................................................................................................................

β-emitter ..................................................................................................................

..................................................................................................................................

γ-emitter ..................................................................................................................

..................................................................................................................................

(ii) State and explain which type of emitter would be most useful for testing these
welds.

..................................................................................................................................

..................................................................................................................................

..................................................................................................................................
[4]

(c) Describe three precautions which should be taken to ensure the safety of the operator
who is making these tests.

1. ......................................................................................................................................

..........................................................................................................................................

2. ......................................................................................................................................

..........................................................................................................................................

3. ......................................................................................................................................

......................................................................................................................................[3]

0625/3 W99
Candidate
Centre Number Number

Candidate Name

International General Certificate of Secondary Education


UNIVERSITY OF CAMBRIDGE LOCAL EXAMINATIONS SYNDICATE

PHYSICS 0625/3
PAPER 3
MAY/JUNE SESSION 2000 1 hour 15 minutes

Candidates answer on the question paper.


Additional materials:
Electronic calculator and/or Mathematical tables
Protractor
Ruler (30 cm)

TIME 1 hour 15 minutes

INSTRUCTIONS TO CANDIDATES
Write your name, Centre number and candidate number in the spaces at the top of this page.
Answer all questions.
Write your answers in the spaces provided on the question paper.

INFORMATION FOR CANDIDATES


The number of marks is given in brackets [ ] at the end of each
question or part question.
FOR EXAMINER’S USE

10

TOTAL

This question paper consists of 14 printed pages and 2 blank pages.


(NH) QF05709/2
© UCLES 2000 [Turn over
2 For
Examiner’s
Use
1 A firework leaves the ground with an initial velocity of 45 m/s, travelling vertically upwards. It
topicmass
topicweight
topicdensity
reaches a maximum height of 100 m.

At this point the firework fails to explode and falls back down the same vertical path to the
ground.

At any point on its path, the firework has both a velocity and a speed.

(a) Using the terms vector and scalar, explain the difference between velocity and speed.

..........................................................................................................................................

..........................................................................................................................................

..........................................................................................................................................

......................................................................................................................................[3]

(b) Fig. 1.1 is a graph which shows the height of the firework above the ground during the
first 5 s of its journey.

120

height 100
above ground / m
80

60

40

20

0
0 1 2 3 4 time / s 5

Fig. 1.1

(i) Use the information on the graph to

1. find the time taken for the firework to reach its maximum height above the
ground,

...................................................................................................................................

2. describe how the motion of the firework changes over the first 5 s of its journey.

...................................................................................................................................

...................................................................................................................................

...................................................................................................................................

...................................................................................................................................
0625/3 S00
3 For
Examiner’s
Use
(ii) The acceleration of free fall is 10 m/s2 and air resistance on the firework is
negligible.

State

1. the deceleration of the firework as it is rising,

deceleration = ........................................................

2. the total time taken for the firework to rise 100 m and then to fall back to the
ground.

time taken = ...........................................................

(iii) State the velocity with which the falling firework hits the ground.

velocity =................................................................
[8]

0625/3 S00 [Turn over


4 For
Examiner’s
Use
2 In an experiment to find the specific latent heat of fusion of ice, an electric heater, of power
topicturningeffect
topicturning
topiceffect
topicenergy
200 W, is used.
topicthermocouple

The following readings are taken.

mass of ice at 0 °C, before heating started, 0.54 kg


mass of ice at 0 °C, after 300 s of heating, 0.36 kg

(a) Calculate a value of the specific latent heat of fusion of ice.

specific latent heat of fusion of ice = ....................... [4]

(b) Explain, in molecular terms, how heat is transferred from the surface of a block of ice to
its centre.

..........................................................................................................................................

..........................................................................................................................................

..........................................................................................................................................

..........................................................................................................................................

......................................................................................................................................[2]

0625/3 S00
5 For
Examiner’s
Use
3 Fig. 3.1 shows a simple beam balance made from a pivot and a metre rule.
topicmolecularmodels
topicmodels
topicmolecular
topiclens
topictotalinternalreflection
topicangle
topicrefraction

2 cm markings 50 98 metre rule

cotton
bag of 0.25 kg
sand mass 0.50 kg
mass
pivot

bench top

Fig. 3.1

(a) Find

(i) the mass of the bag of sand,

mass =.................................................

(ii) the weight of the bag of sand. (The acceleration of freefall is 10 m/s2.)

weight =............................................... [3]

(b) Explain, in terms of moments of forces, why the beam balances.

..........................................................................................................................................

..........................................................................................................................................

..........................................................................................................................................

......................................................................................................................................[3]

(c) The cotton holding the 0.50 kg mass snaps and the mass falls to the bench.
It strikes the bench at a speed of 1.2 m/s.

Calculate its kinetic energy just before it hits the bench.

kinetic energy of the mass = ................................... [3]

(d) On impact with the bench, the mass bounces up a small distance. Some transformation
of energy occurs during the impact. State the forms of the energy just before and just
after the impact.

before: ..............................................................................................................................

after: .............................................................................................................................[2]
0625/3 S00 [Turn over
6 For
Examiner’s
Use
4 Fig. 4.1 shows a sealed box containing only dry air. At a particular instant, one of the air
topictransformations
topicenergy
topicenergytransformations
molecules in the box is situated at P and it is moving towards face ABCD along the direction
shown by the arrow.

C G

B F

D H

A E

Fig. 4.1

(a) Describe and explain a possible path of the molecule within the box.

..........................................................................................................................................

..........................................................................................................................................

..........................................................................................................................................

..........................................................................................................................................

......................................................................................................................................[2]

(b) Explain how this molecule

(i) helps to cause a pressure on the side ABCD,

...................................................................................................................................

...................................................................................................................................

...................................................................................................................................

(ii) helps to cause an equal pressure on all the sides.

...................................................................................................................................

...................................................................................................................................

...............................................................................................................................[2]

0625/3 S00
7 For
Examiner’s
Use
(c) The box is squashed but no air leaks out. By calculation, complete the table below.

volume of box pressure temperature


/m3 /Pa /°C
before squashing 0.09 1.0 x 105 20
after squashing 0.04 20
[2]

5 Fig. 5.1 shows how a right-angled prism may be used to change the direction of a ray of
topicresistance
light.

D E

90° F C
B

Fig. 5.1

(a) Explain why the ray of light does not change direction at D and at F.

......................................................................................................................................[1]

(b) State one property of the light which does change at D and at F. At each point say
whether it increases or decreases.

..........................................................................................................................................

......................................................................................................................................[2]

(c) At E the light splits, with one ray along the surface of the prism and one ray along EF.
Draw the normal at E. Label the critical angle with the letter X and state its value.

critical angle = ....................................... [2]

(d) The refractive index of this glass may be calculated using the formula
refractive index of glass = 1/sin c,
where c is the critical angle.
Use your value of the critical angle of this glass to calculate its refractive index.

refractive index = ................................... [2]


0625/3 S00 [Turn over
8 For
Examiner’s
Use
6 (a) A sound wave in air is made up of compressions and rarefactions.
topicbrownianmotion
topicbrownian
topicmotion

(i) State what is meant by a compression.

...................................................................................................................................

(ii) State what is meant by a rarefaction.

...................................................................................................................................
[2]

(b) The distance between two consecutive rarefactions in a sound wave is 2.5 m. The
speed of sound in air is 330 m/s.

Calculate the frequency of this sound wave.

frequency = ............................................................. [2]

(c) A person makes a loud sound and hears the echo of this sound 1.2 s later.

Calculate how far the person is from the object causing the echo. Assume that the
speed of sound is 330 m / s.

distance =................................................................ [2]

0625/3 S00
9 For
Examiner’s
Use
7 A student is given a battery, a switch, two insulated thick copper leads and a coil of
topicforceonaconductorinamagneticfield
topicforce
topicconductor
topicmagnetic
resistance wire. On Fig. 7.1 only the coil is drawn in.

coil of
resistance
wire

cardboard
tube

Fig. 7.1

(a) The student set up the apparatus to make a current flow through the coil.
Using standard symbols for components, complete a circuit diagram on Fig. 7.1. Also on
Fig. 7.1, draw the magnetic field lines in and around the coil, with arrows to indicate the
direction of the lines. [4]

(b) A charge of 16 C flows through the coil in 40 s.

Calculate the current in the coil.

current =.................................................................. [2]

0625/3 S00 [Turn over


10 For
Examiner’s
Use
(c) The potential difference across the coil is 1.2 V.

(i) Calculate the energy released as heat in the coil in 40 s.

energy = .................................................................. [4]

(ii) Calculate the resistance of the coil.

resistance =............................................................. [4]

(d) The battery supplies 24 J of energy to drive 16 C of charge around the circuit.
Define the e.m.f. of this battery.

..........................................................................................................................................

......................................................................................................................................[2]

0625/3 S00
11 For
Examiner’s
Use
8 Fig. 8.1 shows a transformer.
topicradioactivity

240 V a.c. 12 V a.c.

primary secondary
coil coil

Fig. 8.1

(a) Explain why there is an e.m.f. across the secondary coil even though there is no
electrical connection between the primary and secondary coils.

..........................................................................................................................................

..........................................................................................................................................

..........................................................................................................................................

..........................................................................................................................................

......................................................................................................................................[3]

(b) When the transformer is in use, the current in the secondary circuit is 3.2 A. The
transformer may be considered 100% efficient.

Calculate the current in the primary coil.

current = ..................................................... [3]

0625/3 S00 [Turn over


12 For
Examiner’s
Use
9 (a) Fig. 9.1 shows a beam of electrons about to enter the region between two charged
topiclens
topicvolume
topicdensity
topicpressure
metal plates.

Fig. 9.1

On Fig. 9.1 continue the path of the electron beam between the plates

(i) for plates with a very small charge (label this path P),

(ii) for plates with the opposite charges to those shown on Fig. 9.1 (label this path R).
[3]

(b) Fig. 9.2 shows another arrangement, similar to the first, but in this case the electron
beam continues in a straight line because a magnet (which is not shown) has been
placed near the plates.

Fig. 9.2

Explain where you would place the N-pole of the magnet in order to achieve this effect.
You may draw on the diagram if you feel that it will make your answer clearer.

..........................................................................................................................................

..........................................................................................................................................

..........................................................................................................................................

......................................................................................................................................[3]

0625/3 S00
13 For
Examiner’s
Use
10 (a) A radioactive source contains an isotope of thorium.
Thorium (22980 Th) decays by -particle emission to radium (Ra).
topictension

Write an equation to show this decay.

[2]

(b) The radium produced is also radioactive. Fig. 10.1 shows a laboratory experiment to
test for the presence of the radioactive emissions from the thorium source, using a
radiation detector.
In the laboratory there is a background count of 20 counts/minute.

5 mm thick
aluminium

P Q

radioactive 3 cm
source

Fig. 10.1

The readings are given in the table.

position reading in counts/minute

P 2372

Q 361

State and explain

(i) which radiation could be causing the count at Q,

...................................................................................................................................

...................................................................................................................................

...................................................................................................................................

(ii) which radiations could be causing the count at P.

...................................................................................................................................

...................................................................................................................................

...................................................................................................................................
[4]

0625/3 S00 [Turn over


14 For
Examiner’s
Use
(c) All three types of radioactive emission cause some ionisation of gases.

(i) Explain what is meant by the term ionisation of gases.

...................................................................................................................................

...................................................................................................................................

...................................................................................................................................

(ii) Suggest a reason why -radiation produces very little ionisation.

...................................................................................................................................

...................................................................................................................................

...................................................................................................................................
[3]

0625/3 S00
Candidate
Centre Number Number

Candidate Name

International General Certificate of Secondary Education


UNIVERSITY OF CAMBRIDGE LOCAL EXAMINATIONS SYNDICATE

PHYSICS 0625/3
PAPER 3
Tuesday 14 NOVEMBER 2000 Morning 1 hour 15 minutes

Candidates answer on the question paper.


Additional materials:
Electronic calculator and/or Mathematical tables
Protractor
Ruler (30 cm)

TIME 1 hour 15 minutes

INSTRUCTIONS TO CANDIDATES
Write your name, Centre number and candidate number in the spaces at the top of this page.
Answer all questions.
Write your answers in the spaces provided on the question paper.

INFORMATION FOR CANDIDATES


The number of marks is given in brackets [ ] at the end of each
question or part question.
FOR EXAMINER’S USE

10

TOTAL

This question paper consists of 14 printed pages and 2 blank pages.


(NH) QK07575/2
© UCLES 2000 [Turn over
2 For
Examiner’s
Use
1 Fig. 1.1 shows a 0.5 kg mass hanging freely on a length of steel wire.
topicspeed

length of steel
wire

0.5 kg mass

Fig. 1.1

(a) On Fig. 1.1 use labelled arrows to indicate the direction and line of action of each of the
two forces acting on the 0.5 kg mass.

The acceleration of free fall is 10 m/s2. Calculate the values of the two forces which you
have indicated.

first force = .............................. second force = .............................. [4]

(b) Suggest what causes the two forces to act on the mass.

..........................................................................................................................................

..........................................................................................................................................

......................................................................................................................................[2]

0625/3 W00
3 For
Examiner’s
Use
(c) The 0.5 kg mass is increased by steps of 0.5 kg up to 10 kg. The corresponding
extensions of the steel wire are measured. When the mass on the wire is 10 kg, the wire
snaps. Fig. 1.2 shows part of the graph of extension against load for the wire.

15

extension
/mm Z
10

5
Y

X
0 2.0 4.0 6.0 8.0 10.0
mass/kg

Fig. 1.2

(i) On Fig. 1.2, sketch a possible graph line between Y and Z.

(ii) Determine the mass needed to produce an extension of 3 mm.

mass =..........................................................

(iii) Determine the extension of the wire just before it snaps.

extension = ...................................................
[4]

0625/3 W00 [Turn over


4 For
Examiner’s
Use
2 Fig. 2.1 shows a student’s design for a thermometer. The student stated that the material
topictransfer topicthermal topicenergy
labelled M could be a copper rod, alcohol or nitrogen gas.

M free pivot
fixed pivot
cylinder piston

scale

Fig. 2.1

(a) Explain what is meant by the term sensitivity of the thermometer.

..........................................................................................................................................

......................................................................................................................................[1]

(b) (i) State which of the three suggested materials would give a thermometer of greatest
sensitivity.

...................................................................................................................................

(ii) Explain your answer.

...................................................................................................................................

...................................................................................................................................
[2]

(c) (i) State which of the three materials would allow the thermometer to measure the
largest range of temperature.

...................................................................................................................................

(ii) Explain your answer.

...................................................................................................................................

...................................................................................................................................
[2]

(d) The student found that the temperature scale of this thermometer was non-linear.
Explain what this means.

..........................................................................................................................................

......................................................................................................................................[2]

0625/3 W00
5 For
Examiner’s
Use
3 Fig. 3.1 shows a person raising a concrete block from a river bed by using two pulleys.
topicmolecularmodels
topicmolecular
topicmodels
topiclens

pulley
pulley

6m

block

Fig. 3.1

(a) As shown in Fig. 3.1, the top of the block is 6.0 m below the water surface. The density
of water is 1000 kg/m3 and the acceleration of free fall is 10 m/s2.

Calculate the water pressure acting on the top of the block.

pressure = ............................................ [3]

(b) The block is raised through the water. At one point, the water pressure acting on the top
of the block is 4.5 x 104 Pa. The area of the top of the block is 0.015 m2. Calculate the
downward force exerted by the water on the top of the block.

force =................................................... [2]

(c) When the block is clear of the water, it is raised a further 4.0 m. The weight of the block
is 550 N. Calculate the work done on the block as it is raised the 4.0 m through the air.

work =................................................... [2]

(d) Some of the energy the person uses to raise the block is converted into heat energy.
Indicate on the Fig. 3.1, using an arrow and the letter H, two places where heat is
released. For each place, explain why heat is released there.

..........................................................................................................................................

..........................................................................................................................................

..........................................................................................................................................

......................................................................................................................................[4]

0625/3 W00 [Turn over


6 For
Examiner’s
Use
4 Fig. 4.1 shows water wavefronts which are approaching a small gap in a wall which divides
two stretches of water of the same depth. The diagram is drawn to scale.
topicevaporation

wavefronts

gap in wall

Fig. 4.1

(a) The waves moving towards the wall have a wavelength of 1.6 m and a frequency of
0.80 Hz.

Calculate the speed of these water waves.

speed of waves =.................................. [2]

(b) State the wavelength and frequency of the waves after they have passed through the
gap in the wall.

wavelength = ................................................

frequency =........................................... [2]

(c) On Fig. 4.1, complete the pattern of wavefronts to the right of the wall. [3]

0625/3 W00
7 For
Examiner’s
Use
5 (a) A student determines the specific heat capacity of water. It is found that 15.5 kJ of
topicspeedofsoundandlight
topiclight
topicspeed
topicsound
energy supplied raise the temperature of 0.45 kg of water by 8.2 °C.

Calculate the specific heat capacity of water.

specific heat capacity of water = ............................. [4]

(b) A cylinder, which is closed by a gas-tight moveable piston, contains 0.0060 m3 of gas.
The gas has its pressure raised from 2.0 x 105 Pa to 3.5 x 105 Pa, without any change in
temperature.

(i) Describe how this could be achieved.

...................................................................................................................................

...................................................................................................................................

...................................................................................................................................

(ii) Calculate the volume when the pressure is 3.5 x 105 Pa.

volume = .......................................................
[4]

0625/3 W00 [Turn over


8 For
Examiner’s
Use
6 Fig. 6.1 shows an object placed 2.0 cm from a thin lens, which is to be used as a magnifying
topicbrownianmotion
topicbrownian
topiclens
topicmotion
glass.

The focal length of the lens is 3.0 cm. The diagram is drawn to full scale.

lens object

Fig. 6.1

(a) On Fig. 6.1, draw any two rays from the tip of the object which enable you to locate the
tip of the image. Draw in the image and label it I. [3]

(b) On Fig. 6.1, draw in an eye position which would enable image I to be seen. [1]

(c) By taking measurements from Fig. 6.1, work out how many times bigger the image is
than the object.

The image is .................... times bigger than the object. [2]

0625/3 W00
9 For
Examiner’s
Use
7 Fig. 7.1 shows how a student set up a circuit using three identical lamps. Assume that the
topicconvection
resistance of each lamp does not change with the brightness of the lamp.

Each lamp is labelled 12 V, 2.0 A.

12 V

A
P R

Fig. 7.1

(a) Calculate the resistance of one of the lamps.

resistance = .......................................... [2]

(b) Calculate the combined resistance of the three lamps as connected in Fig. 7.1.

combined resistance = ......................... [2]

(c) Calculate the current which would be shown on the ammeter in Fig. 7.1.

current = ............................................... [2]

(d) Explain why lamp R is less bright than normal and why lamps P and Q are both equally
very dim.

..........................................................................................................................................

..........................................................................................................................................

..........................................................................................................................................

......................................................................................................................................[3]

0625/3 W00 [Turn over


10 For
Examiner’s
Use
(e) In the space below draw a circuit diagram which shows P, Q and R connected so that
they will all work at normal brightness.

[1]

8 Fig. 8.1 shows a simple electrical generator. By turning the handle, the single coil may be
topicatomic
topicatomicstructure
topicstructure
spun between the poles of the magnet.
topicelectromagneticinduction
topicelectromagnetic
topicinduction

rotation

S
N

bulb

Fig. 8.1

0625/3 W00
11 For
Examiner’s
Use
(a) The handle is turned so that the coil makes two complete revolutions per second. The
maximum output is 7 V. On Fig. 8.2, sketch this output over a period of 1 s.

8
e.m.f.
/V 6

0
0 0.25 0.50 0.75 1.00
2 time / s

[3]
Fig. 8.2

(b) Explain

(i) how an e.m.f. is induced,

...................................................................................................................................

...................................................................................................................................

...................................................................................................................................

...................................................................................................................................

(ii) why the e.m.f. varies in magnitude and direction.

...................................................................................................................................

...................................................................................................................................

...................................................................................................................................

...................................................................................................................................
[4]

0625/3 W00 [Turn over


12 For
Examiner’s
Use
9 Fig. 9.1 shows an uncharged metal plate held in a wooden clamp and stand.
topicvolume
topicdensity

topicpressure

metal plate
wooden clamp
and stand

Fig. 9.1

(a) A polythene rod is charged negatively by rubbing it with a duster.

Suggest, in terms of the movement of electrons,

(i) how the polythene becomes negatively charged,

...................................................................................................................................

...................................................................................................................................

...................................................................................................................................

(ii) how the metal plate can be positively charged without the polythene touching the
plate.

...................................................................................................................................

...................................................................................................................................

...................................................................................................................................

...................................................................................................................................
[4]

(b) A strong α-particle emitting source is brought close to, but not touching, the positively
charged metal plate.

Explain why the plate rapidly loses its charge.

..........................................................................................................................................

..........................................................................................................................................

..........................................................................................................................................

......................................................................................................................................[2]

0625/3 W00
13 For
Examiner’s
Use
10 (a) A nuclide, symbol AZ X, decays by β-particle emission to a nuclide, symbol Y.
topictension
A β-particle has the symbol –o1 e.

Write an equation for this decay.

[2]

(b) Fig. 10.1 shows how a β-particle source may be used to measure the thickness of paper
as it is being produced.

rollers

β- particle
source
paper
wound
on roll

detector

Fig. 10.1

(i) Explain why the reading of the detector changes with the thickness of the paper.

...................................................................................................................................

...................................................................................................................................

...................................................................................................................................

(ii) Write down two reasons why β-particles are more useful than γ -rays for this
purpose.

reason 1. ...................................................................................................................

...................................................................................................................................

...................................................................................................................................

reason 2. ...................................................................................................................

...................................................................................................................................

...............................................................................................................................[4]

0625/3 W00 [Turn over


14 For
Examiner’s
Use
(c) Fig. 10.2 shows a beam of β -particles entering a magnetic field, the direction of which is
into the paper.

beam of magnetic
β -particles field into
paper

Fig. 10.2

On Fig. 10.2 continue the path of the beam of β -particles as they pass through the magnetic
field. [2]

0625/3 W00
Candidate
Centre Number Number

Candidate Name

International General Certificate of Secondary Education


UNIVERSITY OF CAMBRIDGE LOCAL EXAMINATIONS SYNDICATE

PHYSICS 0625/3
PAPER 3
MAY/JUNE SESSION 2001 1 hour 15 minutes

Candidates answer on the question paper.


No additional materials required.

TIME 1 hour 15 minutes

INSTRUCTIONS TO CANDIDATES
Write your name, Centre number and candidate number in the spaces at the top of this page.
Answer all questions.
Write your answers in the spaces provided on the question paper.

INFORMATION FOR CANDIDATES


The number of marks is given in brackets [ ] at the end of each
question or part question.
FOR EXAMINER’S USE

10

TOTAL

This question paper consists of 14 printed pages and 2 blank pages.


SB (CW/KN) QF10310/2
© UCLES 2001 [Turn over
2 For
Examiner’s
Use
1 Fig. 1.1 shows the speed of a small, very dense object which is falling vertically from an
topicspeed
aeroplane, up to the point at which it hits the ground. The air resistance on the object is
negligibly small for the first 5 s of its fall. The object is fitted with a parachute which springs
open after a certain time of fall.

150

100
vertical speed
–––––––––––
m/s

50

0
0 5 10 15 20 25 30 35 40 45 50
time/s

Fig. 1.1

(a) State the type of motion

(i) between 0 and 5 s,

...................................................................................................................................

(ii) between 42 s and 47 s.

...................................................................................................................................
[2]

(b) Estimate the time at which the parachute opens.

......................................................................................................................................[1]

0625/3/M/J/01
3 For
Examiner’s
Use
(c) On Figs 1.2 and 1.3, indicate by labelled arrows the vertical forces acting on the falling
object

(i) after 3 s of fall,

falling object

Fig. 1.2

(ii) after 45 s of fall.

parachute

falling object

Fig. 1.3 [3]

(d) State whether or not there is a resultant vertical force acting on the falling object

(i) after 3 s of fall,

.....................................

(ii) after 45 s of fall.

.....................................
[1]

(e) Calculate the distance fallen in the first 5 s of fall.

distance fallen = .....................................[2]

0625/3/M/J/01 [Turn over


4 For
Examiner’s
Use
2 Fig. 2.1 shows a moving car on a level road.
topicturningeffect
topicturning

uniform speed of
20 m/s in the
direction shown

mass 800 kg

Fig. 2.1

(a) Calculate the momentum of the car.

momentum of car = ..................................... [2]

(b) The brakes of the car are applied for 4 s, which reduces the speed of the car to 5 m/s.

(i) Calculate the average force of the brakes.

average force = .....................................

(ii) Calculate the average deceleration of the car.

average deceleration = .....................................


[6]

0625/3/M/J/01
5 For
Examiner’s
Use
3 Describe an experiment to find the average density of a small rock sample of approximately
topicmagnet
100 g mass.

(a) In the space below draw a labelled diagram of the apparatus.

[2]

(b) List all the measurements which must be taken.

[2]

(c) Explain how to work out the average density from the measurements taken.

..........................................................................................................................................

..........................................................................................................................................

......................................................................................................................................[2]

0625/3/M/J/01 [Turn over


6 For
Examiner’s
Use
4 Fig. 4.1 shows a very magnified view of tiny dust particles suspended in still air, as seen
topicenergy
under a microscope.
topicenergytransformations
topictransformations

air-tight container

Fig. 4.1

(a) In the space below, draw a diagram to show how the particle labelled P would move
when it is observed for a short time.

[1]

(b) With reference to dust particles and air molecules, explain the movement which you
have drawn.

..........................................................................................................................................

..........................................................................................................................................

..........................................................................................................................................

......................................................................................................................................[2]

(c) Describe and explain how the movement would change if the temperature of the air in
the container increased.

..........................................................................................................................................

..........................................................................................................................................

..........................................................................................................................................

......................................................................................................................................[2]

0625/3/M/J/01
7 For
Examiner’s
Use
5 Fig. 5.1 shows apparatus which may be used to find the specific heat capacity of a liquid.
topiclight
topicsound
topicspeedofsoundandlight
topicspeed

electric
immersion thermometer
heater

glass beaker
liquid

Fig. 5.1

The readings taken are:


power of the heater, 50 W
time heater is switched on, 600 s
initial temperature of the liquid, 20 °C
final temperature of the liquid, 65 °C
mass of the liquid heated, 200 g

(a) Use the data to calculate the specific heat capacity of the liquid.

specific heat capacity = ..................................... [5]

(b) (i) Explain why the value obtained from this data will be higher than the actual value.

...................................................................................................................................

...................................................................................................................................

...................................................................................................................................

...................................................................................................................................

(ii) Describe one addition to the apparatus which would make the calculated
experimental value nearer to the actual value.

...................................................................................................................................

...................................................................................................................................
[3]

0625/3/M/J/01 [Turn over


8 For
Examiner’s
Use
6 Fig. 6.1 shows some apparatus in use in an experiment to find the critical angle for blue light.
topicbrownianmotion
topicbrownian
topictotal
topicmotion
topicinternal
topiclens
topictotalinternalreflection
topicreflection

60°

incident ray Q
of blue light
glass
P prism

R
emergent
ray

Fig. 6.1

The ray hits the prism at point P, then crosses the prism to point Q. Part of the ray emerges
along the surface QR as shown.

(a) (i) By using measurements taken from the diagram, find the critical angle of the glass
for blue light.

critical angle = .....................................

(ii) Use your value to explain how total internal reflection of blue light could be made to
occur at point Q.

...................................................................................................................................

...................................................................................................................................

...................................................................................................................................
[4]

(b) Using measured angles on the diagram, calculate the refractive index of the glass for
blue light.

refractive index = ..................................... [4]

0625/3/M/J/01
9 For
Examiner’s
Use
7 Fig. 7.1 shows an unlabelled diagram which a teacher draws to represent a sound wave in
topicconvection
air.

}
P Q

Fig. 7.1

(a) What label should be put on the line with the arrow?

......................................................................................................................................[1]

(b) (i) What does the uneven spacing of the lines show?

...................................................................................................................................

(ii) What is being shown at P?

...................................................................................................................................

(iii) What is being shown at Q?

...................................................................................................................................
[2]

(c) Describe the motion of an air particle at R.

..........................................................................................................................................

..........................................................................................................................................

......................................................................................................................................[2]

(d) From Fig. 7.1, measure the wavelength of the sound wave.

wavelength = ..................................... [1]

0625/3/M/J/01 [Turn over


10 For
Examiner’s
Use
8 (a) Fig. 8.1 shows a coil of thin wire and a lamp connected to a 4 V supply.
topicradioactivity

4V

Fig. 8.1

The lamp is marked 1.5 V, 0.6 W. The lamp lights at normal brightness.

Calculate

(i) the current in the lamp,

current = .....................................

(ii) the resistance of the lamp,

resistance = .....................................

(iii) the charge flowing through the lamp in 20 s.

charge = ...............................
[5]

0625/3/M/J/01
11 For
Examiner’s
Use
(b) The resistance of the coil of wire shown in Fig. 8.1 is 6.2 Ω and its length is 1.0 m.
Using only 1.0 m lengths from the same reel of wire, and without cutting any of them,
state how you would produce a resistance of

(i) 3.1 Ω,

...................................................................................................................................

(ii) 12.4 Ω.

...................................................................................................................................

Complete the circuits in Fig. 8.2 and in Fig. 8.3 to show how the lengths of wire are
connected in each case. [3]

4V 4V

resistance resistance
3.1Ω 12.4Ω

Fig. 8.2 Fig. 8.3

(c) In a similar circuit to that shown in Fig. 8.1, the resistance of the coil is 5.0 Ω and the
current through it is 0.6 A.
Calculate the heat energy produced in the coil in 20 s.

energy = ..................................... [3]

0625/3/M/J/01 [Turn over


12 For
Examiner’s
Use
9 Fig. 9.1 shows a transformer.
topicpressure

240 V a.c. 12 V a.c.

Fig. 9.1

(a) Explain

(i) why a secondary output is obtained even though there is no electrical connection
between the primary and secondary coils,

...................................................................................................................................

...................................................................................................................................

...................................................................................................................................

...................................................................................................................................

...................................................................................................................................

(ii) why there would be no output voltage if the primary coil were connected to a 240 V
d.c. supply.

...................................................................................................................................

...................................................................................................................................

...................................................................................................................................

...................................................................................................................................
[5]

(b) The transformer is assumed to be 100% efficient.

(i) There are 100 turns on the secondary coil. How many turns are there on the
primary coil?

turns on the primary = .....................................

(ii) The output current is 4.0 A. Calculate the input current.

input current = .....................................


[4]

0625/3/M/J/01
13 For
Examiner’s
Use
10 (a) Complete the following table for α-particles. The first answer has been given.
topictension

property/nature complete this column

symbol 4 He (hint: it is a helium nucleus)


2

mass
number

(hint: write down the number of


charge
(hint: proton charges)

(hint: choose from: strong, weak


ionisation of gases
(hint: or almost none)

(hint: choose from: towards N,


deflection in a
(hint: towards S or at right angles
magnetic field
(hint: to the magnetic field lines)

deflection in an (hint: choose from: towards +ve,


electric field (hint: towards -ve or no deflection)

[5]

0625/3/M/J/01 [Turn over


14

(b) Fig.10.1 shows the paths of α-particles scattered by the nuclei of metal atoms in thin foils.

key:
α-particle
nuclei of metal atoms

Fig. 10.1

Explain what can be deduced from the paths shown in Fig. 10.1 about

(i) the mass of the nucleus of a metal atom compared to the mass of an α-particle,

............................................................................................................................................

............................................................................................................................................

............................................................................................................................................

(ii) the charge on the nucleus of a metal atom,

............................................................................................................................................

............................................................................................................................................

............................................................................................................................................

(iii) the volume occupied by a metal atom compared to its nucleus.

............................................................................................................................................

............................................................................................................................................

............................................................................................................................................
[5]

0625/3/M/J/01
Candidate
Centre Number Number

Candidate Name

International General Certificate of Secondary Education


UNIVERSITY OF CAMBRIDGE LOCAL EXAMINATIONS SYNDICATE

PHYSICS 0625/3
PAPER 3
OCTOBER/NOVEMBER SESSION 2001 1 hour 15 minutes

Candidates answer on the question paper.


No additional materials are required.

TIME 1 hour 15 minutes

INSTRUCTIONS TO CANDIDATES
Write your name, Centre number and candidate number in the spaces at the top of this page.
Answer all questions.
Write your answers in the spaces provided on the question paper.

INFORMATION FOR CANDIDATES


The number of marks is given in brackets [ ] at the end of each
question or part question.
FOR EXAMINER’S USE

10

TOTAL

This question paper consists of 16 printed pages.


SB (CW/KN) S10764/2
© UCLES 2001 [Turn over
2 For
Examiner’s
Use
1 Fig. 1.1 shows the motion of a train over a section of track which includes a sharp bend.
topicmass
topicweight
topicdensity
topicvolume
topicprojectiles

30

25

20
speed
––––– 15
m/s

10

0
0 10 20 30
time/s

Fig. 1.1

(a) The section of the track with the sharp bend has a maximum speed restriction. The train
decelerates approaching the bend so that at the start of the bend it has just reached the
maximum speed allowed. The train is driven around the bend at the maximum speed
allowed and accelerates immediately on leaving the bend.

(i) What is the maximum speed allowed round the bend in the track?

maximum speed = .....................................

(ii) How long does the train take to travel the bend of the track?

time taken = .....................................

(iii) Calculate the length of the bend.

length of bend = .....................................


[3]

(b) The train has to slow down to go round the bend. Calculate the deceleration.

deceleration = .....................................[2]

0625/3/O/N/01
3 For
Examiner’s
Use
(c) As the train is driven round the bend, there is an extra force acting, called the centripetal
force.

(i) On Fig. 1.2, draw an arrow to show the direction of this force.

train

curved
track

direction
of motion

Fig. 1.2

(ii) State the effect that this force has on the motion.

...................................................................................................................................

...................................................................................................................................

(iii) State how this force is provided.

...................................................................................................................................

...................................................................................................................................
[3]

0625/3/O/N/01 [Turn over


4 For
Examiner’s
Use
2 Fig. 2.1 shows a car with a dummy driver before and after a collision test.
topictransfer topicthermal topicenergy

just before impact just after impact

V = 0 m/s
V = 45 m/s

Fig. 2.1

The mass of the dummy driver is 90 kg. The impact time to reduce the dummy’s speed from
45 m/s to zero is 1.2 s.

(a) Calculate

(i) the momentum of the dummy just before impact,

momentum = .....................................

(ii) the average force on the dummy during impact.

force = .....................................
[4]

(b) State the main energy transformation during the collision.

......................................................................................................................................[1]

(c) Calculate how much of the dummy’s energy is transformed during the collision.

energy = ..................................... [3]

0625/3/O/N/01
5 For
Examiner’s
Use
3 A body is in equilibrium and is acted upon by two vertical downward forces in such a way that
topicmagnet
there is no net moment about a pivot. A student is asked to show this experimentally.
The student is provided with a suitable pivot, a metre rule with a hole drilled in the centre,
two sets of masses and strong cotton.

(a) In the space below, draw a labelled diagram of the apparatus set up ready for use.

[2]

(b) Describe how two sets of readings are taken, explaining how equilibrium is achieved in
each case.

..........................................................................................................................................

..........................................................................................................................................

..........................................................................................................................................

......................................................................................................................................[2]

(c) Write down, in table form, two possible sets of values and use them to show that there
is no net moment. [3]

0625/3/O/N/01 [Turn over


6 For
Examiner’s
Use
4 Fig. 4.1 shows the path that one molecule, M, in a gas might take.
topictransformations
topicenergy
topicenergytransformations

start of
movement
M

position after
a short time
interval

Fig. 4.1

(a) Explain why, in Fig. 4.1, the path of M has sudden, sharp changes of direction.

..........................................................................................................................................

..........................................................................................................................................

..........................................................................................................................................

......................................................................................................................................[2]

(b) At the end of the short time interval, M is a short distance from its starting point. Use
this observation to explain why a gas spreads slowly through air.

..........................................................................................................................................

..........................................................................................................................................

......................................................................................................................................[2]

0625/3/O/N/01
7 For
Examiner’s
Use
5 (a) A student concludes that the results of his experiments show that it requires more
topicspeed
topicspeedofsoundandlight
topicthermal
topicsound
topiclight
energy to convert 1 g of water into steam at 100 °C than it does to raise the temperature
topicthermalcapacity
topicthermometer

of 1 g of water from 0 °C to 100 °C.


Use the student’s data to confirm that this conclusion is correct and calculate the
difference between the two amounts of energy.

experiment 1 experiment 2
mass of water used 250 g mass of water evaporated at 100 °C 15 g
heat energy supplied 10 500 J heat energy supplied 33 900 J
rise in temperature 10 °C

energy difference = .....................................[5]

(b) Explain, in molecular terms, why considerable heat energy is needed to convert 1 g of
water into 1 g of steam at 100 °C, without any change in temperature taking place.

..........................................................................................................................................

..........................................................................................................................................

......................................................................................................................................[2]

(c) The mercury-in-glass thermometer used in experiment 1 above is said to have


– moderate sensitivity,
– a good range,
– a linear scale.

By reference to this thermometer, explain what is meant by

(i) sensitivity,

...................................................................................................................................

...................................................................................................................................

(ii) range,

...................................................................................................................................

...................................................................................................................................

(iii) linear scale.

...................................................................................................................................

...................................................................................................................................
[3]
0625/3/O/N/01 [Turn over
8 For
Examiner’s
Use
6 (a) Figs 6.1 and 6.2 show what happens to waves at two different types of boundary.
topicdispersion

Fig. 6.1 Fig. 6.2

Complete the table below.

Fig. 6.1 Fig. 6.2

name of the effect shown

wavelength change, if any

frequency change, if any


[4]

0625/3/O/N/01
9 For
Examiner’s
Use
(b) Fig. 6.3 is drawn to full scale. The distance CF is the focal length of the lens.

object

F C F

Fig. 6.3

(i) By drawing rays from the tip of the object, locate the position of the image.
Hence work out how many times bigger the image is than the object.

number of times bigger = .....................................

(ii) 1. Draw an eye on Fig. 6.3 to show a suitable place to view the image.
2. Suggest a use for this lens arrangement.

...................................................................................................................................
[6]

0625/3/O/N/01 [Turn over


10 For
Examiner’s
Use
7 (a) Fig. 7.1 shows a current-carrying solenoid and the position of a plotting compass.
topicconductor
topicforce
topicforceonaconductorinamagneticfield

cardboard

B
A
direction of current
in the solenoid
indicated by
arrows

compass
originally
placed at A

Fig. 7.1

The plotting compass is used to follow magnetic field lines.


On Fig. 7.1 draw in one magnetic field line which links A and B, both through the inside
of the solenoid and round the outside of the solenoid.
Mark the direction of each part of the field line with an arrow. [3]

0625/3/O/N/01
11 For
Examiner’s
Use
(b) Fig. 7.2 shows the result of a similar experiment with a current-carrying, straight wire.

direction of
current vertically
upwards through
the cardboard

C A
B

horizontal
cardboard

Fig. 7.2

(i) On Fig. 7.2, draw another magnetic field line starting at C. Mark its direction with an
arrow.
(ii) Explain why the line from C could never pass through B.

...................................................................................................................................

...................................................................................................................................

(iii) What would be the effect on the strength and on the direction of the magnetic
field of

1. reversing the current without changing its value,

strength....................................... direction...........................................

2. increasing the value of the current without changing its direction?

strength....................................... direction...........................................
[5]

0625/3/O/N/01 [Turn over


12 For
Examiner’s
Use
8 Fig. 8.1 shows how two security lamps are connected to a mains supply.
topicradioactivity

240 V
a.c.
Z X A

Y B

Fig. 8.1

Lamp A is labelled 240 V, 600 W and lamp B is labelled 240 V, 300 W.

(a) Calculate the currents at points X, Y and Z in Fig. 8.1.

current at X = .....................................

current at Y = .....................................

current at Z = ..................................... [2]

(b) The resistance of lamp A is 96 Ω and the resistance of lamp B 192 Ω. Using these
values, or by an alternative method, calculate the total circuit resistance. (Ignore the
resistance of the circuit wiring.)

resistance = ..................................... [3]

(c) Fig. 8.2 shows the same lamps connected differently.

240 V
a.c.
P
A B
96 Ω 192 Ω

Fig. 8.2

(i) Calculate the current at P.

current at P = .....................................

(ii) Calculate the potential difference across A and across B.

potential difference across A = .....................................

potential difference across B = .....................................


[3]

0625/3/O/N/01
13 For
Examiner’s
Use
(d) (i) With reference to values already worked out, explain why the lamps should be
connected as in Fig. 8.1 and not as in Fig. 8.2.

...................................................................................................................................

...................................................................................................................................

...................................................................................................................................

(ii) The two lamps are to be switched on and off independently. State and explain
which circuit is better for this purpose when suitably placed switches are included.

...................................................................................................................................

...................................................................................................................................

...................................................................................................................................

...................................................................................................................................
[4]

0625/3/O/N/01 [Turn over


14 For
Examiner’s
Use
9 Fig. 9.1 shows a circuit and a cathode-ray oscilloscope (c.r.o.).
topicdensity
topicpressure
topicvolume

d.c. supply

Fig. 9.1

(a) Complete the connections to show how you would use the c.r.o. to measure the
potential difference across the lamp. [1]

(b) Fig. 9.2 shows the screen of the c.r.o. when measuring this potential difference.

trace when
p.d. connected

trace before p.d. connected

Fig. 9.2

When calibrated, each vertical division corresponds to a potential difference of 0.4 V.


What is the potential difference across the lamp?

potential difference = ..................................... [2]

(c) Suggest one advantage of using this method of measuring potential difference rather
than using a standard voltmeter.

..........................................................................................................................................

......................................................................................................................................[1]

0625/3/O/N/01
15 For
Examiner’s
Use
10 (a) Radioactive sodium has a nuclide represented by the symbol 24
11 Na.
topictension
Complete the equation below to show how this nuclide decays with the emission of
a β-particle.

24
11 Na → Mg + e [2]

(b) Fig. 10.1 shows a narrow beam of β-particles entering an electric field created by two
charged plates.

narrow beam P
of β-particles

Fig 10.1

(i) Complete the path of the β-particles, starting from the point P.
(ii) Explain any change of direction which you have shown.

...................................................................................................................................

...................................................................................................................................

...................................................................................................................................
[3]

0625/3/O/N/01 [Turn over


16 For
Examiner’s
Use
(c) (i) In the space below, draw a labelled diagram of an arrangement, using a radioactive
source which emits β-particles, for finding the variation in thickness of a sheet of
paper.

(ii) State the readings which need to be taken and how they would be used to decide
whether or not the thickness of the paper varies.

...................................................................................................................................

...................................................................................................................................

...................................................................................................................................

...................................................................................................................................

...............................................................................................................................[4]

0625/3/O/N/01
Candidate
Centre Number Number

Candidate Name

International General Certificate of Secondary Education


CAMBRIDGE INTERNATIONAL EXAMINATIONS
PHYSICS 0625/3
PAPER 3
MAY/JUNE SESSION 2002
1 hour 15 minutes
Candidates answer on the question paper.
No additional materials required.

TIME 1 hour 15 minutes

INSTRUCTIONS TO CANDIDATES
Write your name, Centre number and candidate number in the spaces at the top of this page.
Answer all questions.
Write your answers in the spaces provided on the question paper.

INFORMATION FOR CANDIDATES


The number of marks is given in brackets [ ] at the end of each
question or part question.
FOR EXAMINER’S USE

10

TOTAL

This question paper consists of 12 printed pages.


SP (AT/JB) S22941/2
© CIE 2002 [Turn over
2 For
Examiner’s
Use
1 A group of students attempts to find out how much power each student can generate. The
students work in pairs in order to find the time taken for each student to run up a flight of
stairs.
The stairs used are shown in Fig. 1.1.

finishing point

starting point

Fig. 1.1

(a) Make a list of all the readings that would be needed. Where possible, indicate how the
accuracy of the readings could be improved.

..........................................................................................................................................

..........................................................................................................................................

..........................................................................................................................................

..........................................................................................................................................

..........................................................................................................................................

..................................................................................................................................... [4]

(b) Using words, not symbols, write down all equations that would be needed to work out
the power of a student.

..........................................................................................................................................

..........................................................................................................................................

..................................................................................................................................... [2]

(c) (i) When the student has reached the finishing point and is standing at the top of the
stairs, what form of energy has increased to its maximum?

...................................................................................................................................

(ii) Suggest why the total power of the student is greater than the power calculated by
this method.

...................................................................................................................................

...................................................................................................................................
[3]

0625/3/M/J/02
3 For
Examiner’s
Use
2 A small rubber ball falls vertically, hits the ground and rebounds vertically upwards.
Fig. 2.1 is the speed-time graph for the ball.

10 B
speed D
8
m/s
6

2
A
0 C E
0 0.5 1.0 1.5 2.0
time / s

Fig. 2.1

(a) Using information from the graph, describe the following parts of the motion of the ball.

(i) part AB

...................................................................................................................................

...................................................................................................................................

...................................................................................................................................

(ii) part DE

...................................................................................................................................

...................................................................................................................................

...................................................................................................................................
[3]

(b) Explain what is happening to the ball along the part of the graph from B through C to D.

..........................................................................................................................................

..........................................................................................................................................

..................................................................................................................................... [2]

(c) Whilst the ball is in contact with the ground, what is the

(i) overall change in speed,

change in speed = ........................................

(ii) overall change in velocity?

change in velocity = ......................................


[2]
0625/3/M/J/02 [Turn over
4 For
Examiner’s
Use
(d) Use your answer to (c) to explain the difference between speed and velocity.

..........................................................................................................................................

..........................................................................................................................................

..................................................................................................................................... [2]

(e) Use the graph to calculate the distance travelled by the ball between D and E.

distance travelled = ..................................[2]

(f) Use the graph to calculate the deceleration of the ball between D and E.

deceleration = ..................................[2]

0625/3/M/J/02
5 For
Examiner’s
Use
3 Fig. 3.1 is an attempt to show the molecules in water and the water vapour molecules over
the water surface.

water vapour
molecules

water molecules

Fig. 3.1

(a) Explain, in terms of the energies of the molecules, why only a few water molecules have
escaped from the water surface.

..........................................................................................................................................

..........................................................................................................................................

..................................................................................................................................... [2]

(b) State two ways of increasing the number of water molecules escaping from the surface.

1 .......................................................................................................................................

2 .................................................................................................................................. [2]

(c) Energy is required to evaporate water.

Explain, in molecular terms, why this energy is needed.

..........................................................................................................................................

..........................................................................................................................................

..................................................................................................................................... [2]

0625/3/M/J/02 [Turn over


6 For
Examiner’s
Use
4 (a) Fig. 4.1 shows a cylinder containing air at a pressure of 1.0 × 105 Pa. The length of the
air column in the cylinder is 80 mm.

80 mm

air

piston
cylinder

Fig. 4.1

The piston is pushed in until the pressure in the cylinder rises to 3.8 × 105 Pa.

Calculate the new length of the air column in the cylinder, assuming that the
temperature of the air has not changed.

new length = .................................. [3]

(b) Fig. 4.2 shows the same cylinder containing air.

air

Fig. 4.2

The volume of the air in the cylinder changes as the temperature of the air changes.

(i) The apparatus is to be used as a thermometer. Describe how two fixed points, 0 °C
and 100 °C, and a temperature scale could be marked on the apparatus.

...................................................................................................................................

...................................................................................................................................

...................................................................................................................................

...................................................................................................................................

(ii) Describe how this apparatus could be used to indicate the temperature of a large
beaker of water.

...................................................................................................................................

...................................................................................................................................

...................................................................................................................................

...................................................................................................................................
[5]
0625/3/M/J/02
7 For
Examiner’s
Use
5 Fig. 5.1 shows an arrangement where a plane mirror is used in a shop to watch a display
counter. The arrangement is drawn to a scale of 1 cm : 1 m.

plane mirror

P wall
display counter

Fig. 5.1

(a) (i) State the law of reflection.

...................................................................................................................................

(ii) On Fig. 5.1, draw rays to show how much of the display cannot be seen from P.
Indicate this by shading in the part that cannot be seen.
[3]

(b) By construction on Fig. 5.1 and by using the scale, calculate how far the mirror must be
moved so that all of the display counter can be seen from P.

distance moved = .................................... [2]

(c) State the characteristics of an image seen in a plane mirror.

..........................................................................................................................................

..........................................................................................................................................

..................................................................................................................................... [2]

0625/3/M/J/02 [Turn over


8 For
Examiner’s
Use
6 Observations of a distant thunderstorm are made.

(a) During a lightning flash, the average wavelength of the light emitted is 5 × 10–7 m. This
light travels at 3 × 108 m/s.

Calculate the average frequency of this light.

frequency = ...................................... [2]

(b) The interval between the lightning flash being seen and the thunder being heard is
3.6 s. The speed of sound in air is 340 m/s.

(i) Calculate the distance between the thunderstorm and the observer.

distance = ............................................

(ii) Explain why the speed of light is not taken into account in this calculation.

...................................................................................................................................

...................................................................................................................................
[3]

(c) A single ray of white light from the lightning is incident on a prism as shown in Fig. 6.1.

prism

screen
ray of
light

Fig. 6.1

Complete the path of the ray to show how a spectrum is formed on the screen. Label the
colours. [2]

0625/3/M/J/02
9 For
Examiner’s
Use
7 (a) Two non-conducting spheres, made of different materials, are initially uncharged. They
are rubbed together. This causes one of the spheres to become positively charged and
one negatively charged.

Describe, in terms of electron movement, why the spheres become charged.

..........................................................................................................................................

..........................................................................................................................................

..................................................................................................................................... [2]

(b) Once charged, the two spheres are separated, as shown in Fig. 7.1.

+ + – –
+ + + – – –
+ + – –

Fig. 7.1

On Fig. 7.1, draw the electric field between the two spheres. Indicate by arrows the
direction of the electric field lines. [2]

(c) A conducting wire attached to a negatively charged metal object is connected to earth.
This allows 2.0 × 1010 electrons, each carrying a charge of 1.6 × 10–19 C, to flow to
earth in 1.0 × 10–3 s.

Calculate

(i) the total charge that flows,

charge .....................................

(ii) the average current in the wire.

current .....................................
[3]

0625/3/M/J/02 [Turn over


10 For
Examiner’s
Use
8 Fig. 8.1 shows a transformer and a rectifier used in a battery charging circuit for a 12 V
battery.

T1

240 V a.c.
T2

primary secondary

Fig. 8.1

(a) The transformer produces an output of 15 V across the secondary coil.

Calculate a suitable turns ratio for the transformer.

turns ratio = ................................ [2]

(b) Fig. 8.2 shows the 15 V output across the secondary coil.

potential
difference

time

Fig. 8.2

On the same axes, sketch the graph of the potential difference across the terminals T1
and T2 before the battery is connected. [2]

(c) Explain how the circuit converts an a.c. supply into a d.c. output.

..........................................................................................................................................

..........................................................................................................................................

..................................................................................................................................... [2]

(d) On Fig. 8.1, draw in a battery connected so that it may be charged. [1]

0625/3/M/J/02
11 For
Examiner’s
Use
(e) When fully charged, the 12V battery can supply a current of 2.0 A for 30 hours (1.08 ×
105 s).

Calculate

(i) the battery power when supplying a current of 2.0 A,

power = ......................................

(ii) the total energy that the battery will supply during the 30 hours.

energy = ......................................
[4]

9 Fig. 9.1 shows three resistors connected across a low voltage d.c. supply, and a c.r.o.

A d.c. F
supply

B C D E

Y input

Fig. 9.1

(a) Explain how you would use a 1 V d.c. supply to calibrate the c.r.o.

..........................................................................................................................................

..........................................................................................................................................

..................................................................................................................................... [2]

(b) On Fig. 9.1, draw in the connections between the c.r.o. and the circuit so that the
potential difference between points C and D may be measured. [2]

(c) The potential differences between A and F, B and C, C and D, and D and E are
measured.

State the relationship between them.

..........................................................................................................................................

......................................................................................................................................[2]

0625/3/M/J/02 [Turn over


12 For
Examiner’s
Use
10 Some liquid from an atomic power station is known to be radioactive. A sample of this liquid
is tested in a laboratory.

(a) In the space below, draw a labelled diagram of the test apparatus used to verify that
α-particles are emitted from the liquid. [2]

(b) Explain how the apparatus may be used to estimate the quantity of α-radiation being
emitted from the sample.

..........................................................................................................................................

..........................................................................................................................................

..........................................................................................................................................

..........................................................................................................................................

..................................................................................................................................... [2]

(c) State any two safety precautions that the technician might take whilst making the test.

precaution 1 .....................................................................................................................

..........................................................................................................................................

precaution 2 .....................................................................................................................

..................................................................................................................................... [2]

0625/3/M/J/02
Candidate
Centre Number Number

Candidate Name

International General Certificate of Secondary Education


CAMBRIDGE INTERNATIONAL EXAMINATIONS

PHYSICS 0625/3
PAPER 3
OCTOBER/NOVEMBER SESSION 2002
1 hour 15 minutes
Candidates answer on the question paper.
No additional materials are required.

TIME 1 hour 15 minutes

INSTRUCTIONS TO CANDIDATES
Write your name, Centre number and candidate number in the spaces at the top of this page.
Answer all questions.
Write your answers in the spaces provided on the question paper.

INFORMATION FOR CANDIDATES


The number of marks is given in brackets [ ] at the end of each
question or part question.
FOR EXAMINER’S USE

10

TOTAL

This question paper consists of 14 printed pages and 2 blank pages.


SP (NF/KS) S23400/3
© CIE 2002 [Turn over
2 For
Examiner’s
Use
1 Fig. 1.1 shows a smooth metal block about to slide down BD, along DE and up EF.
BD and DE are friction-free surfaces, but EF is rough. The block stops at F.

B
metal
block
C
F

D A
E

Fig. 1.1

(a) On Fig. 1.2, sketch the speed-time graph for the journey from B to F.
Label D, E and F on your graph. [3]

speed

B
time

Fig. 1.2

(b) The mass of the block is 0.2 kg. The vertical height of B above A is 0.6 m.
The acceleration due to gravity is 10 m/s2.

(i) Calculate the work done in lifting the block from A to B.

work done = ............................

(ii) At C, the block is moving at a speed of 2.5 m/s. Calculate its kinetic energy at C.

kinetic energy = ...............................


[5]

0625/3/O/N/02
3 For
Examiner’s
Use
(c) As it passes D, the speed of the block remains almost constant but the velocity
changes. Using the terms vector and scalar, explain this statement.

..........................................................................................................................................

..........................................................................................................................................

......................................................................................................................................[2]

(d) F is the point where the kinetic energy of the block is zero. In terms of energy changes,
explain why F is lower than B.

..........................................................................................................................................

..........................................................................................................................................

..........................................................................................................................................

......................................................................................................................................[3]

0625/3/O/N/02 [Turn over


4 For
Examiner’s
Use
2 A student is given the following apparatus in order to find the density of a piece of rock.

100 g mass
metre rule
suitable pivot on which the rule will balance
measuring cylinder that is big enough for the piece of rock to fit inside
cotton
water

The rock has a mass of approximately 90 g.

(a) (i) In the space below, draw a labelled diagram of apparatus from this list set up so
that the student is able to find the mass of the piece of rock.

(ii) State the readings the student should take and how these would be used to find the
mass of the rock.

...................................................................................................................................

...................................................................................................................................

...................................................................................................................................
[5]

(b) Describe how the volume of the rock could be found.

..........................................................................................................................................

..........................................................................................................................................

......................................................................................................................................[2]

(c) The mass of the rock is 88 g and its volume is 24 cm3.


Calculate the density of the rock.

density of rock = .............................. [2]

0625/3/O/N/02
5 For
Examiner’s
Use
3 A thermocouple is used to measure the temperature of the inner wall of a pottery kiln.

(a) In the space below, draw a labelled diagram of a thermocouple that could be used for
this purpose. [2]

(b) Describe

(i) how you would read the temperature of the wall from the thermocouple,

...................................................................................................................................

...................................................................................................................................

(ii) how the thermocouple works.

...................................................................................................................................

...................................................................................................................................

...................................................................................................................................
[2]

(c) State two conditions in which a thermocouple is very suitable for temperature
measurement.

..........................................................................................................................................

......................................................................................................................................[2]

0625/3/O/N/02 [Turn over


6 For
Examiner’s
Use
4 (a) In an experiment to find the specific latent heat of water, the following readings were
taken.

m1 mass of water at 100 °C, before boiling starts 120 g


m2 mass of water at 100 °C, after boiling finishes 80 g
V voltage across the heater 12 V
I current through the heater 2.0 A
t time that the heater was supplying energy 3750 s

(i) Using the symbols above, write down the equation that must be used to find the
value of the specific latent heat L of water.

(ii) Use the equation to calculate the specific latent heat of water from the readings
above.

specific latent heat = ......................................


[4]

(b) Explain, in terms of the energy of molecules, why the specific latent heat of water has a
high value.

..........................................................................................................................................

..........................................................................................................................................

......................................................................................................................................[2]

0625/3/O/N/02
7 For
Examiner’s
Use
5 (a) Fig. 5.1 shows the air pressure variation along a sound wave.

air pressure

above normal

normal
A B

below normal distance along


wave

Fig. 5.1

(i) On AB in Fig. 5.1, mark one point of compression with a dot and the letter C and
the next point of rarefaction with a dot and the letter R.
(ii) In terms of the wavelength, what is the distance along the wave between a
compression and the next rarefaction?

...................................................................................................................................
[3]

(b) A sound wave travels through air at a speed of 340 m/s. Calculate the frequency of a
sound wave of wavelength 1.3 m.

frequency = ......................... [2]

0625/3/O/N/02 [Turn over


8 For
Examiner’s
Use
6 (a) Fig. 6.1 shows the results of an experiment to find the critical angle for light in a semi-
circular glass block.

air Q
glass

Fig. 6.1

The ray of light PO hits the glass at O at an angle of incidence of 0°.


Q is the centre of the straight side of the block.

(i) Measure the critical angle of the glass from Fig. 6.1.

critical angle = .........................

(ii) Explain what is meant by the critical angle of the light in the glass.

...................................................................................................................................

...................................................................................................................................

...................................................................................................................................
[3]

0625/3/O/N/02
9 For
Examiner’s
Use
(b) Fig. 6.2 shows another ray passing through the same block.

Fig. 6.2

The speed of the light between W and Q is 3.0 × 108 m/s. The speed of the light
between Q and Y is 2.0 × 108 m/s.

(i) State the speed of the light between Y and Z.

speed = ......................................

(ii) Write down an expression, in terms of the speeds of the light, that may be used to
find the refractive index of the glass. Determine the value of the refractive index.

refractive index = ........................

(iii) Explain why there is no change of direction of ray QY as it passes out of the glass.

...................................................................................................................................

(iv) What happens to the wavelength of the light as it passes out of the glass?

...................................................................................................................................
[5]

0625/3/O/N/02 [Turn over


10 For
Examiner’s
Use
7 Fig. 7.1 shows an arrangement that could be used for making an electromagnet or a
permanent magnet.

12 V d.c.
supply
A

turns of thick
copper wire

cardboard tube

Fig. 7.1

Two bars of the same size are also available, one made of iron and the other of steel.

(a) (i) State which bar should be used to make a permanent magnet.

...................................................................................................................................

(ii) Describe how the apparatus would be used to make a permanent magnet.

...................................................................................................................................

...................................................................................................................................

...................................................................................................................................

(iii) Suggest one reason why the circuit contains an ammeter and a variable resistor.

...................................................................................................................................

...................................................................................................................................
[3]

0625/3/O/N/02
11 For
Examiner’s
Use
(b) During the making of a permanent magnet, the ammeter reads a steady current of 4.0 A
throughout the 5.0 s that the current is switched on. The voltage of the supply is 12 V.

Calculate

(i) the total circuit resistance,

resistance = ............................

(ii) the power of the supply,

power = .................................

(iii) the energy supplied during the 5.0 s.

energy = .................................
[6]

(c) The potential difference across the variable resistor is 7.0 V and that across the
ammeter is zero.

(i) Calculate the potential difference across the magnetising coil.

potential difference = .................................

(ii) State the general principle used in making this calculation.

...................................................................................................................................

...................................................................................................................................
[3]

0625/3/O/N/02 [Turn over


12 For
Examiner’s
Use
8 Fig. 8.1 shows a long straight wire between the poles of a permanent magnet.
It is connected through a switch to a battery so that, when the switch is closed, there is a
steady current in the wire.

N S

wire in a direction
at right angles to the page

Fig. 8.1

(a) State the direction of the magnetic field between the poles of the magnet.

......................................................................................................................................[1]

(b) The wire is free to move. The current is switched on so that its direction is into the page.

(i) State the direction of movement of the wire.

...................................................................................................................................

...................................................................................................................................

(ii) Explain how you reached your answer to (b)(i).

...................................................................................................................................

...................................................................................................................................

...................................................................................................................................
[4]

(c) This experiment is the basis of an electric motor.


Describe two changes to the arrangement shown in Fig. 8.1 that would enable
continuous rotation to take place.

change 1 ..........................................................................................................................

..........................................................................................................................................

change 2 ..........................................................................................................................

......................................................................................................................................[2]

0625/3/O/N/02
13 For
Examiner’s
Use
9 Fig. 9.1 shows a beam of electrons, two charged plates and a screen. These components
are inside an electron tube, the outline of which is not shown.

charged plate
beam of P
electrons
screen
X Y

charged plate

Fig. 9.1

The beam of electrons hits the screen at the point P.

(a) On Fig. 9.1,

(i) complete the path of the electron beam,


(ii) mark the charges on both plates,
(iii) mark with an arrow and the letter C the direction of the conventional current in the
electron beam.
[4]

(b) In this electron tube, the electrons are produced at X and are accelerated towards Y.
In the space below, draw a labelled diagram of the components needed to produce and
accelerate the electrons. [4]

0625/3/O/N/02 [Turn over


14 For
Examiner’s
Use
10 Fig. 10.1 is part of the decay curve for a sample of a β-emitting isotope.

100
% activity
75

50

25

0
0 5 10 15 20
time / days

Fig. 10.1

(a) Use Fig. 10.1 to find the half-life of the isotope.

half-life = .......................... [1]

(b) Complete Fig. 10.1 as far as time = 20 days, by working out the values of a number of
points and plotting them. Show your working. [2]

(c) The decay product of the β-emitting isotope is not radioactive.


Explain why the sample of the radioactive isotope will be safer after 20 days than after
1 day. Support your answer by reference to the graph.

..........................................................................................................................................

......................................................................................................................................[1]

(d) The isotope used for this decay curve may be represented by the symbol AZX.
Write down an equation, by filling in the gaps below, to show the β-decay of this isotope
to a decay product that has the symbol Y.

AX
Z → + Y [2]

0625/3/O/N/02
Centre Number Candidate Number Name

CAMBRIDGE INTERNATIONAL EXAMINATIONS


International General Certificate of Secondary Education
PHYSICS 0625/03
Paper 3
May/June 2003

1 hour 15 minutes
Candidates answer on the Question Paper.
No Additional Materials are required.

READ THESE INSTRUCTIONS FIRST

Write your Centre number, candidate number and name on all the work you hand in.
Write in dark blue or black pen in the spaces provided on the Question Paper.
You may use a soft pencil for any diagrams, graphs or rough working.
Do not use staples, paper clips, highlighters, glue or correction fluid.

Answer all questions.


The number of marks is given in brackets [ ] at the end of each question or part question.

For Examiner’s Use


1

8
If you have been given a label, look at the
details. If any details are incorrect or 9
missing, please fill in your correct details
in the space given at the top of this page. 10

Stick your personal label here, if 11


provided.
Total

This document consists of 12 printed pages.


SP (AT/KN) S46413/2
© CIE 2003 [Turn over
2 For
Examiner’s
Use
1 Fig. 1.1 shows apparatus that may be used to compare the strengths of two springs of the
same size, but made from different materials.

spring

scale

masses

Fig. 1.1

(a) (i) Explain how the masses produce a force to stretch the spring.

...................................................................................................................................

(ii) Explain why this force, like all forces, is a vector quantity.

...................................................................................................................................

...................................................................................................................................
[2]

(b) Fig. 1.2 shows the graphs obtained when the two springs are stretched.

20
force/N spring 1
15
spring 2
10
5
0
0 10 20 30 40
extension/mm

Fig. 1.2

0625/3/M/J/03
3 For
Examiner’s
Use
(i) State which spring is more difficult to extend. Quote values from the graphs to
support your answer.

...................................................................................................................................

...................................................................................................................................

...................................................................................................................................

...................................................................................................................................

(ii) On the graph of spring 2, mark a point P at the limit of proportionality. Explain your
choice of point P.

...................................................................................................................................

...................................................................................................................................

...................................................................................................................................

(iii) Use the graphs to find the difference in the extensions of the two springs when a
force of 15 N is applied to each one.

difference in extensions = ..................................


[6]

2 The speed of a cyclist reduces uniformly from 2.5 m/s to 1.0 m/s in 12 s.

(a) Calculate the deceleration of the cyclist.

deceleration = ..................................[3]

(b) Calculate the distance travelled by the cyclist in this time.

distance = ..................................[2]

0625/3/M/J/03 [Turn over


4 For
Examiner’s
Use
3 Fig. 3.1 shows the arm of a crane when it is lifting a heavy box.

1220 N
950 N

40° 30°

box

Fig. 3.1

(a) By the use of a scale diagram (not calculation) of the forces acting at P, find the weight
of the box. [5]

0625/3/M/J/03
5 For
Examiner’s
Use
(b) Another box of weight 1500 N is raised vertically by 3.0 m.

(i) Calculate the work done on the box.

work done = ..................................

(ii) The crane takes 2.5 s to raise this box 3.0 m. Calculate the power output of the
crane.

power = ..................................
[4]

4 Fig. 4.1 shows a sealed glass syringe that contains air and many very tiny suspended dust
particles.

syringe
seal

piston
dust particles

Fig. 4.1

(a) Explain why the dust particles are suspended in the air and do not settle to the bottom.

..........................................................................................................................................

..........................................................................................................................................

..........................................................................................................................................

......................................................................................................................................[3]

(b) The air in the syringe is at a pressure of 2.0 × 105 Pa. The piston is slowly moved into the
syringe, keeping the temperature constant, until the volume of the air is reduced from
80 cm3 to 25 cm3. Calculate the final pressure of the air.

pressure = ..................................[3]

0625/3/M/J/03 [Turn over


6 For
Examiner’s
Use
5 Fig. 5.1 shows a thermocouple set up to measure the temperature at a point on a solar
panel.

Sun's rays

surface
Z
of solar
panel X cold junction

Y
hot junction

Fig. 5.1

(a) X is a copper wire.

(i) Suggest a material for Y.

...................................................................................................................................

(ii) Name the component Z.

...................................................................................................................................
[2]

(b) Explain how a thermocouple is used to measure temperature.

..........................................................................................................................................

..........................................................................................................................................

......................................................................................................................................[3]

(c) Experiment shows that the temperature of the surface depends upon the type of
surface used.

Describe the nature of the surface that will cause the temperature to rise most.

..........................................................................................................................................

......................................................................................................................................[1]

0625/3/M/J/03
7 For
Examiner’s
Use
6 Fig. 6.1 shows wavefronts of light crossing the edge of a glass block from air into glass.

air
direction in which
wavefronts
are moving

glass

edge of glass

Fig. 6.1

(a) On Fig. 6.1

(i) draw in an incident ray, a normal and a refracted ray that meet at the same point on
the edge of the glass block,
(ii) label the angle of incidence and the angle of refraction,
(iii) measure the two angles and record their values.

angle of incidence = ..................................

angle of refraction = ..................................


[4]

(b) Calculate the refractive index of the glass.

refractive index = ..................................[3]

0625/3/M/J/03 [Turn over


8 For
Examiner’s
Use
7 In a thunderstorm, both light and sound waves are generated at the same time.

(a) How fast does the light travel towards an observer?

speed = .................................. [1]

(b) Explain why the sound waves always reach the observer after the light waves.

......................................................................................................................................[1]

(c) The speed of sound waves in air may be determined by experiment using a source that
generates light waves and sound waves at the same time.

(i) Draw a labelled diagram of the arrangement of suitable apparatus for the
experiment.

(ii) State the readings you would take.

...................................................................................................................................

...................................................................................................................................

...................................................................................................................................

(iii) Explain how you would calculate the speed of sound in air from your readings.

...................................................................................................................................

...................................................................................................................................
[4]

0625/3/M/J/03
9 For
Examiner’s
Use
8 Fig. 8.1 shows a battery with a resistor connected across its terminals. The e.m.f. of the
battery is 6.0 V.

6.0 V

Fig. 8.1

The battery causes 90 C of charge to flow through the circuit in 45 s.

(a) Calculate

(i) the current in the circuit,

current = ..................................

(ii) the resistance of the circuit,

resistance = ..................................

(iii) the electrical energy transformed in the circuit in 45 s.

energy = ..................................
[6]

(b) Explain what is meant by the term e.m.f. of the battery.

..........................................................................................................................................

..........................................................................................................................................

......................................................................................................................................[2]

0625/3/M/J/03 [Turn over


10 For
Examiner’s
Use
9 A transformer has an output of 24 V when supplying a current of 2.0 A. The current in the
primary coil is 0.40 A and the transformer is 100% efficient.

(a) Calculate

(i) the power output of the transformer,

power = ..................................

(ii) the voltage applied across the primary coil.

voltage = ..................................
[4]

(b) Explain

(i) what is meant by the statement that the transformer is 100% efficient,

...................................................................................................................................

...................................................................................................................................

...................................................................................................................................

(ii) how the transformer changes an input voltage into a different output voltage.

...................................................................................................................................

...................................................................................................................................

...................................................................................................................................

...................................................................................................................................
[4]

0625/3/M/J/03
11 For
Examiner’s
Use
10 Fig. 10.1 and Fig. 10.2 show two views of a vertical wire carrying a current up through a
horizontal card. Points P and Q are marked on the card.

vertical
P Q
wire

view from above the card

Fig. 10.1 Fig. 10.2

(a) On Fig. 10.2,

(i) draw a complete magnetic field line (line of force) through P and indicate its
direction with an arrow,
(ii) draw an arrow through Q to indicate the direction in which a compass placed at Q
would point.
[3]
(b) State the effect on the direction in which compass Q points of

(i) increasing the current in the wire,

...................................................................................................................................

(ii) reversing the direction of the current in the wire.

...................................................................................................................................
[2]

(c) Fig. 10.3 shows the view from above of another vertical wire carrying a current up
through a horizontal card. A cm grid is marked on the card. Point W is 1 cm vertically
above the top surface of the card.

T vertical
wire carrying
R S current

Fig. 10.3

State the magnetic field strength at S, T and W in terms of the magnetic field strength
at R. Use one of the alternatives, weaker, same strength or stronger for each answer.

at S ........................................................................

at T ........................................................................

at W........................................................................ [3]
0625/3/M/J/03 [Turn over
12 For
Examiner’s
Use
11 (a) A radioactive isotope emits only α-particles.

(i) In the space below, draw a labelled diagram of the apparatus you would use to
prove that no β-particles or γ-radiation are emitted from the isotope.

(ii) Describe the test you would carry out.

...................................................................................................................................

...................................................................................................................................

...................................................................................................................................

...................................................................................................................................

(iii) Explain how your results would show that only α-particles are emitted.

...................................................................................................................................

...................................................................................................................................

...................................................................................................................................
[6]

(b) Fig. 11.1 shows a stream of α-particles about to enter the space between the poles of a
very strong magnet.

α-particles
S

Fig. 11.1

Describe the path of the α-particles in the space between the magnetic poles.

..........................................................................................................................................

..........................................................................................................................................

......................................................................................................................................[3]
0625/3/M/J/03
Centre Number Candidate Number Name

CAMBRIDGE INTERNATIONAL EXAMINATIONS


International General Certificate of Secondary Education
PHYSICS 0625/03
Paper 3
October/November 2003

1 hour 15 minutes
Candidates answer on the Question Paper.
No Additional Materials are required.

READ THESE INSTRUCTIONS FIRST

Write your Centre number, candidate number and name on all the work you hand in.
Write in dark blue or black pen in the spaces provided on the Question Paper.
You may use a soft pencil for any diagrams, graphs or rough working.
Do not use staples, paper clips, highlighters, glue or correction fluid.

Answer all questions.


At the end of the examination, fasten all your work securely together.
The number of marks is given in brackets [ ] at the end of each question or part question.
You may lose marks if you do not show your working or if you do not use appropriate units.
Take the weight of 1 kg to be 10 N (i.e. acceleration of free fall = 10 m / s2).

For Examiner’s Use


1

8
If you have been given a label, look at the
details. If any details are incorrect or 9
missing, please fill in your correct details
in the space given at the top of this page. 10

Stick your personal label here, if 11


provided.
Total

This document consists of 15 printed pages and 1 blank page.


MML 4504 11/02 S53411/2 UNIVERSITY of CAMBRIDGE
© UCLES 2003 Local Examinations Syndicate [Turn over
2 For
Examiner’s
Use
1 Fig. 1.1 shows the speed-time graph for a bus during tests.

At time t = 0, the driver starts to brake.

speed 20
m/s
P
15
test 2 graph
10
Q
5
test 1 graph
R
0
0 1 2 3 4 5 6 7 8 9 10
time t / s

Fig. 1.1

(a) For test 1,


(i) determine how long the bus takes to stop,

..................................................................................................................................
(ii) state which part of the graph shows the greatest deceleration,

..................................................................................................................................
(iii) use the graph to determine how far the bus travels in the first 2 seconds.

distance = ..........................................
[4]

(b) For test 2, a device was fitted to the bus. The device changed the deceleration.
(i) State two ways in which the deceleration during test 2 is different from that during
test 1.

1 ...............................................................................................................................

2 ...............................................................................................................................
(ii) Calculate the value of the deceleration in test 2.

deceleration = ....................................
[4]
0625/03/O/N/03
3 For
Examiner’s
Use
(c) Fig. 1.2 shows a sketch graph of the magnitude of the acceleration for the bus when it
is travelling around a circular track at constant speed.

magnitude
of acceleration

0
0 time

Fig. 1.2

(i) Use the graph to show that there is a force of constant magnitude acting on the
bus.

..................................................................................................................................

..................................................................................................................................
(ii) State the direction of this force.

..................................................................................................................................
[3]

0625/03/O/N/03 [Turn over


4 For
Examiner’s
Use
2 Fig. 2.1 shows a diver 50 m below the surface of the water.

water surface

50 m

Fig. 2.1

(a) The density of water is 1000 kg/m 3 and the acceleration of free fall is 10 m/s 2 .
Calculate the pressure that the water exerts on the diver.

pressure = .................................... [3]

(b) The window in the diver’s helmet is 150 mm wide and 70 mm from top to bottom.

Calculate the force that the water exerts on this window.

force = .......................................... [3]

0625/03/O/N/03
5 For
Examiner’s
Use
3 Fig. 3.1 shows a simple see-saw. One child A sits near to end X and another child B sits
near to end Y. The feet of the children do not touch the ground when the see-saw is
balanced.

X Y

pivot

Fig. 3.1

(a) Child A has a mass of 18.0 kg and child B has a mass of 20.0 kg.

Without calculation, indicate where the children could sit so that the see-saw balances
horizontally. You may draw on Fig. 3.1 if you wish.

..........................................................................................................................................

..........................................................................................................................................

.................................................................................................................................... [2]

(b) State the relationship between the moment caused by child A and that caused by
child B.

..........................................................................................................................................

.................................................................................................................................... [1]

(c) Child A is 2.50 m from the pivot. Calculate the distance of child B from the pivot.

distance = .................................... [2]

0625/03/O/N/03 [Turn over


6 For
Examiner’s
Use
4 Fig. 4.1 shows water being heated by an electrical heater. The water in the can is not
boiling, but some is evaporating.

can

heater

water

Fig. 4.1

(a) Describe, in terms of the movement and energies of the water molecules, how
evaporation takes place.

..........................................................................................................................................

..........................................................................................................................................

..........................................................................................................................................

.................................................................................................................................... [2]

(b) State two differences between evaporation and boiling.

1 .......................................................................................................................................

..........................................................................................................................................

2 .......................................................................................................................................

.................................................................................................................................... [2]

(c) After the water has reached its boiling point, the mass of water in the can is reduced
by 3.2 g in 120 s. The heater supplies energy to the water at a rate of 60 W. Use this
information to calculate the specific latent heat of vaporisation of water.

specific latent heat = ............................. [3]

0625/03/O/N/03
7 For
Examiner’s
Use
5 (a) Equal volumes of nitrogen, water and copper at 20 °C are heated to 50 °C.
(i) Which one of the three will have a much greater expansion than the other two?

..................................................................................................................................
(ii) Explain your answer in terms of the way the molecules are arranged in the three
substances.

..................................................................................................................................

..................................................................................................................................

..................................................................................................................................
[3]

(b) Fig. 5.1 shows a thermometer with a range of –10 °C to 50 °C.

–10 °C 50 °C

Fig. 5.1

Explain what is meant by


(i) the sensitivity of a thermometer,

..................................................................................................................................

..................................................................................................................................
(ii) the linearity of a thermometer.

..................................................................................................................................

..................................................................................................................................
[2]

0625/03/O/N/03 [Turn over


8 For
Examiner’s
Use
6 Fig. 6.1 shows the diffraction of waves by a narrow gap.

P is a wavefront that has passed through the gap.

Fig. 6.1

(a) On Fig. 6.1, draw three more wavefronts to the right of the gap. [3]

(b) The waves travel towards the gap at a speed of 3 x 108 m/s and have a frequency of
5 x 1014 Hz. Calculate the wavelength of these waves.

wavelength = ................................ [3]

0625/03/O/N/03
9 For
Examiner’s
Use
7 Fig. 7.1 is drawn to full scale. The focal length of the lens is 5.0 cm.

axis

Fig. 7.1

(a) On Fig. 7.1, mark each principal focus of the lens with a dot and the letter F. [2]

(b) On Fig. 7.1, draw two rays from the tip of the object O that appear to pass through the
tip of the image. [2]

(c) On Fig. 7.1, draw the image and label it with the letter I. [1]

(d) Explain why the base of the image lies on the axis.

..........................................................................................................................................

.................................................................................................................................... [1]

(e) State a practical use of a convex lens when used as shown in Fig. 7.1.

.................................................................................................................................... [1]

0625/03/O/N/03 [Turn over


10 For
Examiner’s
Use
8 Fig. 8.1 shows the outline of an a.c. generator. The peak output voltage of the generator is
6.0 V and the output has a frequency of 10 Hz.

output N coil
iron core

bearing

rings
connected
one to each
end of coil S
Fig. 8.1
handle to
turn coil

(a) Fig. 8.2 shows the axes of a voltage-time graph for the generator output.

voltage / V

0
0
0.1 0.2 0.3
time / s

Fig. 8.2

On Fig. 8.2,
(i) mark suitable voltage values on the voltage axis,
(ii) draw a graph of the generator output.
[3]
0625/03/O/N/03
11 For
Examiner’s
Use
(b) The generator shown in Fig. 8.1 works by electromagnetic induction.

Explain how this effect produces the output voltage.

..........................................................................................................................................

..........................................................................................................................................

..........................................................................................................................................

.................................................................................................................................... [3]

(c) State the energy changes that occur in the generator when it is producing output.

.................................................................................................................................... [2]

0625/03/O/N/03 [Turn over


12 For
Examiner’s
Use
9 (a) Fig. 9.1 shows the screen of a c.r.o. (cathode-ray oscilloscope).

The c.r.o. is being used to display the output from a microphone.

The vertical scale on the screen is in volts.

voltage / V
2

—1

—2

Fig. 9.1

(i) Describe the output from the microphone.

..................................................................................................................................

..................................................................................................................................
(ii) Use the graph to determine the peak voltage of the output.

..................................................................................................................................
(iii) Describe how you could check that the voltage calibration on the screen is
correct.

..................................................................................................................................

..................................................................................................................................
[4]

0625/03/O/N/03
13 For
Examiner’s
Use
(b) Fig. 9.2 shows the screen of the c.r.o. when it is being used to measure a small time
interval between two voltage pulses.

0 1 2 3 4 5 6 7 8 9 10
cm scale

Fig. 9.2

(i) What is the distance on the screen between the two voltage pulses?

..................................................................................................................................
(ii) The time-base control of the c.r.o. is set at 5.0 ms / cm.
Calculate the time interval between the voltage pulses.

time = ........................................
(iii) Suggest one example where a c.r.o. can be used to measure a small time
interval.

..................................................................................................................................
[4]

0625/03/O/N/03 [Turn over


14 For
Examiner’s
Use
10 Fig. 10.1 shows a battery with an e.m.f of 12 V supplying power to two lamps.

The total power supplied is 150 W when both lamps are on.

L1

L2

Fig. 10.1

(a) Calculate the current supplied by the battery when both lamps are on.

current = ...................................... [2]

(b) The current in lamp L2 is 5.0 A.

Calculate
(i) the current in lamp L1,

current = ............................................
(ii) the power of lamp L1,

power = ..............................................
(iii) the resistance of lamp L1.

resistance = .......................................
[6]
0625/03/O/N/03
15 For
Examiner’s
Use
11 (a) A sodium nucleus decays by the emission of a -particle to form magnesium.
(i) Complete the decay equation below.

24 Na
11 → Mg +

(ii) Fig. 11.1 shows -particles from sodium nuclei moving into the space between the
poles of a magnet.

-particles

Fig. 11.1

Describe the path of the -particles between the magnetic poles.

..................................................................................................................................

..................................................................................................................................

..................................................................................................................................
[5]

(b) Very small quantities of a radioactive isotope are used to check the circulation of blood
by injecting the isotope into the bloodstream.
(i) Describe how the results are obtained.

..................................................................................................................................

..................................................................................................................................

..................................................................................................................................

..................................................................................................................................
(ii) Explain why a -emitting isotope is used for this purpose rather than one that
emits either -particles or -particles.

..................................................................................................................................

..................................................................................................................................

..................................................................................................................................
[4]

0625/03/O/N/03
Centre Number Candidate Number Name

UNIVERSITY OF CAMBRIDGE INTERNATIONAL EXAMINATIONS


International General Certificate of Secondary Education
PHYSICS 0625/03
Paper 3
May/June 2004
1 hour 15 minutes
Candidates answer on the Question Paper.
No Additional Materials are required.

READ THESE INSTRUCTIONS FIRST

Write your Centre number, candidate number and name on all the work you hand in.
Write in dark blue or black pen in the spaces provided on the Question Paper.
You may use a soft pencil for any diagrams, graphs or rough working.
Do not use staples, paper clips, highlighters, glue or correction fluid.

Answer all questions.


At the end of the examination, fasten all your work securely together.
The number of marks is given in brackets [ ] at the end of each question or part question.
You may lose marks if you do not show your working or if you do not use appropriate units.

For Examiner’s Use

8
If you have been given a label, look at the
details. If any details are incorrect or 9
missing, please fill in your correct details
in the space given at the top of this page. 10

Stick your personal label here, if 11


provided.
Total

This document consists of 13 printed pages and 3 blank pages.


SPA (NH/BI) S61207/2
© UCLES 2004 [Turn over
2 For
Examiner’s
Use
1 Fig. 1.1 shows a cycle track.

A B

E C

v = 6 m/s

Fig. 1.1

A cyclist starts at A and follows the path ABCDEB.

The speed-time graph is shown in Fig. 1.2.

B C D E B
6
speed
m/s 5
4

3
2
1

0A
0 10 20 30 40 50 60 70 80 90 100
time / s

Fig. 1.2

(a) Use information from Fig. 1.1 and Fig. 1.2 to describe the motion of the cyclist

(i) along AB,

...................................................................................................................................

(ii) along BCDEB.

...................................................................................................................................

...................................................................................................................................
[4]
© UCLES 2004 0625/03 M/J/04
3 For
Examiner’s
Use
(b) The velocity v of the cyclist at C is shown in Fig. 1.1.

State one similarity and one difference between the velocity at C and the velocity at E.

similarity ...........................................................................................................................

difference ......................................................................................................................[2]

(c) Calculate

(i) the distance along the cycle track from A to B,

distance = …………………

(ii) the circumference of the circular part of the track.

circumference = …………………
[4]

© UCLES 2004 0625/03 M/J/04 [Turn over


4 For
Examiner’s
Use
2 Fig. 2.1 shows a rock that is falling from the top of a cliff into the river below.

cliff

falling
rock

river

Fig. 2.1

(a) The mass of the rock is 75 kg. The acceleration of free fall is 10 m/s2.
Calculate the weight of the rock.

weight = …………………[1]

(b) The rock falls from rest through a distance of 15 m before it hits the water.
Calculate its kinetic energy just before hitting the water. Show your working.

kinetic energy = …………………[3]

(c) The rock hits the water. Suggest what happens to the kinetic energy of the rock during
the impact.

..........................................................................................................................................

..........................................................................................................................................

......................................................................................................................................[3]

© UCLES 2004 0625/03 M/J/04


5 For
Examiner’s
Use
3 A large spring is repeatedly stretched by an athlete to increase the strength of his arms.
Fig. 3.1 is a table showing the force required to stretch the spring.

extension of spring / m 0.096 0.192 0.288 0.384

force exerted to produce extension / N 250 500 750 1000

Fig. 3.1

(a) (i) State Hooke’s law.

...................................................................................................................................

...............................................................................................................................[1]

(ii) Use the results in Fig. 3.1 to show that the spring obeys Hooke’s law.

[1]

(b) Another athlete using a different spring exerts an average force of 400 N to enable her
to extend the spring by 0.210 m.

(i) Calculate the work done by this athlete in extending the spring once.

work done = …………………

(ii) She is able to extend the spring by this amount and to release it 24 times in 60 s.
Calculate the power used by this athlete while doing this exercise.

power = …………………
[4]

© UCLES 2004 0625/03 M/J/04 [Turn over


6 For
Examiner’s
Use
4 (a) Two identical open boxes originally contain the same volume of water.
One is kept at 15 °C and the other at 85 °C for the same length of time.

Fig. 4.1 shows the final water levels.

15 °C
85 °C

Fig. 4.1

With reference to the energies of the water molecules, explain why the levels are
different.

..........................................................................................................................................

..........................................................................................................................................

..........................................................................................................................................

......................................................................................................................................[3]

(b) In an experiment to find the specific latent heat of vaporisation of water, it took 34 500 J
of energy to evaporate 15 g of water that was originally at 100 °C.

A second experiment showed that 600 J of energy was lost to the atmosphere from the
apparatus during the time it took to evaporate 15 g of water.

Calculate the specific latent heat of vaporisation of water that would be obtained from
this experiment.

specific latent heat = …………………[3]

© UCLES 2004 0625/03 M/J/04


7 For
Examiner’s
Use
5 (a) Fig. 5.1 shows two identical metal plates. The front surface of one is dull black and the
front surface of the other is shiny silver.
The plates are fitted with heaters that keep the surfaces of the plates at the same
temperature.

dull black shiny silver

Fig. 5.1

(i) State the additional apparatus needed to test which surface is the best emitter of
heat radiation.

...................................................................................................................................

(ii) State one precaution that is needed to ensure a fair comparison.

...................................................................................................................................

...................................................................................................................................

(iii) State the result that you expect.

...................................................................................................................................

(iv) Write down another name for heat radiation.

...................................................................................................................................
[4]

(b) In the space below, draw a labelled diagram of an everyday situation in which a
convection current occurs.

Mark the path of the current with a line and show its direction with arrows. [3]

© UCLES 2004 0625/03 M/J/04 [Turn over


8 For
Examiner’s
Use
6 Fig. 6.1 shows a ray PQ of blue light incident on the side of a rectangular glass block.

A B

glass
C D
Q air

P Fig. 6.1

Fig. 6.1

(a) (i) By drawing on Fig. 6.1, continue the ray PQ through and beyond the block.

(ii) Mark the angle of incidence at CD with the letter i and the angle of refraction at CD
with the letter r.
[3]

(b) The speed of light in air is 3.0 x 108 m/s and the speed of light in glass is 2.0 x 108 m/s.

(i) Write down a formula that gives the refractive index of glass in terms of the
speeds of light in air and glass.

refractive index =

(ii) Use this formula to calculate the refractive index of glass.

refractive index = …………………


[2]

(c) The frequency of the blue light in ray PQ is 6.0 x 1014 Hz.
Calculate the wavelength of this light in air.

wavelength = ……………..……[2]

© UCLES 2004 0625/03 M/J/04


9 For
Examiner’s
Use
7 Fig. 7.1 shows the cone of a loudspeaker that is producing sound waves in air.
At any given moment, a series of compressions and rarefactions exist along the line XY.

cone

X Y
wires

air

Fig. 7.1

(a) On Fig. 7.1, use the letter C to mark three compressions and the letter R to mark three
rarefactions along XY. [1]

(b) Explain what is meant by

(i) a compression,

...................................................................................................................................

...................................................................................................................................

(ii) a rarefaction.

...................................................................................................................................

...................................................................................................................................
[2]

(c) A sound wave is a longitudinal wave. With reference to the sound wave travelling along
XY in Fig. 7.1, explain what is meant by a longitudinal wave.

..........................................................................................................................................

......................................................................................................................................[2]

(d) There is a large vertical wall 50 m in front of the loudspeaker. The wall reflects the
sound waves.
The speed of sound in air is 340 m/s.
Calculate the time taken for the sound waves to travel from X to the wall and to return
to X.

time = …………………[2]

© UCLES 2004 0625/03 M/J/04 [Turn over


10 For
Examiner’s
Use
8 Fig. 8.1 shows a 240 V a.c. mains circuit to which a number of appliances are connected and
switched on.

240 V a.c. refrigerator


fan
60 W 60 W
1.2 kW 200 W

Fig. 8.1

(a) Calculate the power supplied to the circuit.

power = …………..[1]

(b) The appliances are connected in parallel.

(i) Explain what connected in parallel means.

...................................................................................................................................

...................................................................................................................................

(ii) State two advantages of connecting the appliances in parallel rather than in series.

advantage 1 ...............................................................................................................

advantage 2 ...............................................................................................................
[3]

(c) Calculate

(i) the current in the refrigerator,

current = …………..

(ii) the energy used by the fan in 3 hours,

energy = …………..

(iii) the resistance of the filament of one lamp.

resistance = …………..
[7]
© UCLES 2004 0625/03 M/J/04
11 For
Examiner’s
Use
9 Electromagnetic induction can be demonstrated using a solenoid, a magnet, a sensitive
ammeter and connecting wire.

(a) In the space below, draw a labelled diagram of the apparatus set up to demonstrate
electromagnetic induction. [2]

(b) State one way of using the apparatus to produce an induced current.

..........................................................................................................................................

......................................................................................................................................[1]

(c) Explain why your method produces an induced current.

..........................................................................................................................................

..........................................................................................................................................

......................................................................................................................................[2]

(d) Without changing the apparatus, state what must be done to produce

(i) an induced current in the opposite direction to the original current,

...................................................................................................................................

...................................................................................................................................

(ii) a larger induced current.

...................................................................................................................................

...................................................................................................................................
[2]

© UCLES 2004 0625/03 M/J/04 [Turn over


12 For
Examiner’s
Use
10 (a) Fig. 10.1 shows the faces of two ammeters. One has an analogue display and the other
a digital display.

2 3
1 A 4
A
0

5
Fig. 10.1

State what is meant by the terms analogue and digital.

..........................................................................................................................................

..........................................................................................................................................

......................................................................................................................................[2]

(b) (i) Name the components from which logic gates are made.

...............................................................................................................................[1]

(ii) In the space below, draw the symbol for an AND gate.
Label the inputs and the output. [1]

(iii) Describe the action of an AND gate with two inputs. [2]

© UCLES 2004 0625/03 M/J/04


13 For
Examiner’s
Use
11 (a) α-particles can be scattered by thin gold foils.

Fig. 11.1 shows part of the paths of three α-particles.


Complete the paths of the three α-particles. [3]

α-particle 1

α-particle 2

α-particle 3

gold nuclei

Fig. 11.1

(b) What does the scattering of α-particles show about atomic structure?

..........................................................................................................................................

..........................................................................................................................................

......................................................................................................................................[2]

(c) State the nucleon number (mass number) of an α-particle.

nucleon number = …………………[1]

© UCLES 2004 0625/03 M/J/04


UNIVERSITY OF CAMBRIDGE INTERNATIONAL EXAMINATIONS
International General Certificate of Secondary Education
PHYSICS
Paper 3
0625/03
October/November 2004

1 hour 15 minutes
Candidates answer on the Question Paper.
No Additional Materials are required.

Candidate
Name

Centre Candidate
Number Number

READ THESE INSTRUCTIONS FIRST

Write your Centre number, candidate number and name on all the work you hand in.
Write in dark blue or black pen in the spaces provided on the Question Paper. For Examiner’s Use
You may use a soft pencil for any diagrams, graphs or rough working.
Do not use staples, paper clips, highlighters, glue or correction fluid. 1

Answer all questions. 2


At the end of the examination, fasten all your work securely together.
The number of marks is given in brackets [ ] at the end of each question or part 3
question.
You may lose marks if you do not show your working or if you do not use appropriate 4
units.
5
DO NOT WRITE IN THE BARCODE. 6
DO NOT WRITE IN THE GREY AREAS BETWEEN THE PAGES. 7

8
If you have been given a label, look at the
details. If any details are incorrect or 9
missing, please fill in your correct details
in the space given on this page. 10

Stick your personal label here, if 11


provided.
Total

This document consists of 15 printed pages and 1 blank page.


SPA (NH/BI) S80765/4
© UCLES 2004 [Turn over
2

1 Fig. 1.1 shows the path of one drop of water in the jet from a powerful hose.
For
Examiner’s
Use

vertical path
of a single
water drop

hose set in ground

Fig. 1.1

Fig. 1.2 is a graph of speed against time for the water drop shown in Fig. 1.1.

speed 40
m/s
30

20

10

0
0 2 4 6 8
time after leaving the hose / s

Fig. 1.2

(a) Describe the movement of the water drop in the first 4 s after leaving the hose.

..........................................................................................................................................

..........................................................................................................................................

......................................................................................................................................[2]

© UCLES 2004 0625/03 O/N/04


3

(b) Use Fig. 1.2 to find


For
Examiner’s
(i) the speed of the water leaving the hose, Use

speed = …………………...

(ii) the time when the speed of the water is least.

time = ………….………..
[2]

(c) Use values from Fig. 1.2 to calculate the acceleration of the drop as it falls back towards
the ground. Show your working.

acceleration = ……………………...[3]

(d) Calculate the greatest distance above the ground reached by the drop.

distance = ……………………...[3]

© UCLES 2004 0625/03 O/N/04 [Turn over


4

2 Fig. 2.1 shows a reservoir that stores water.


For
Examiner’s
Use

20 m
valve
water

dam

exit pipe

Fig. 2.1

(a) The valve in the exit pipe is closed. The density of water is 1000 kg/m3 and the
acceleration of free fall is 10 m/s2.
Calculate the pressure of the water acting on the closed valve in the exit pipe.

pressure = ……………………...[2]

(b) The cross-sectional area of the pipe is 0.5 m2.


Calculate the force exerted by the water on the closed valve.

force = ……………………...[2]

(c) The valve is then opened and water, originally at the surface of the reservoir, finally
flows out of the exit pipe. State the energy transformation of this water between the
surface of the reservoir and the open end of the pipe.

..........................................................................................................................................

......................................................................................................................................[2]

© UCLES 2004 0625/03 O/N/04


5

3 A scientist needs to find the density of a sample of rock whilst down a mine. He has only a
For
spring balance, a measuring cylinder, some water and some thread. Examiner’s
Use
(a) In the space below, draw two labelled diagrams, one to show the spring balance being
used and the other to show the measuring cylinder being used with a suitable rock
sample. [2]

(b) The spring balance is calibrated in newtons. State how the mass of the rock sample
may be found from the reading of the spring balance.

......................................................................................................................................[1]

(c) State the readings that would be taken from the measuring cylinder.

..........................................................................................................................................

......................................................................................................................................[1]

(d) State how the volume of the rock would be found from the readings.

......................................................................................................................................[1]

(e) State in words the formula that would be used to find the density of the sample.

density =
[1]

© UCLES 2004 0625/03 O/N/04 [Turn over


6

4 (a) Fig. 4.1 shows a simple type of thermocouple that has been calibrated to measure
For
temperature. Examiner’s
Use

copper wire

sensitive voltmeter
V
hot junction
iron wire cold junction

Fig. 4.1

(i) Describe how the thermocouple could be used to measure the temperature of a
beaker of hot water.

...................................................................................................................................

...................................................................................................................................

...................................................................................................................................

(ii) State two situations where a thermocouple would be a good choice of thermometer
to measure temperature.

1. ..............................................................................................................................

...................................................................................................................................

2. ..............................................................................................................................

...................................................................................................................................
[4]

© UCLES 2004 0625/03 O/N/04


7

(b) A mercury-in-glass thermometer is placed in an insulated beaker of water at 60 °C. The


For
water is heated at a constant rate. The temperature of the water is measured and Examiner’s
recorded on the graph shown in Fig. 4.2. Use

100

80
temperature/°C
60

40

20
0
0 5 10 15 20
time/min

Fig. 4.2

State the effect of the heat supplied

(i) during the period 0 to 5 minutes,

...................................................................................................................................

......................................................................................................................................

(ii) during the period 10 to 15 minutes.

...................................................................................................................................

...................................................................................................................................
[2]

© UCLES 2004 0625/03 O/N/04 [Turn over


8

5 (a) Fig. 5.1 shows a sealed box.


For
Examiner’s
Use

Fig. 5.1

(i) The box contains a large number of air molecules. On Fig. 5.1, draw a possible
path of one of the air molecules, as it moves inside the box.

(ii) Explain

1 how air molecules in the box create a pressure on the inside walls,

...........................................................................................................................

...........................................................................................................................

...........................................................................................................................

2 why this pressure rises as the temperature of the air in the box increases.

...........................................................................................................................

...........................................................................................................................

...........................................................................................................................
[5]

(b) Air in a cylinder is compressed slowly, so that the temperature does not rise. The
pressure changes from 2.0 x 105 Pa to 5.0 x 105 Pa. The original volume was 0.35 m3.
Calculate the new volume.

volume = …………………...[3]

© UCLES 2004 0625/03 O/N/04


9

6 Fig. 6.1 shows an optical fibre. XY is a ray of light passing along the fibre.
For
Examiner’s
Use

Y fibre

Fig. 6.1

(a) On Fig. 6.1, continue the ray XY until it passes Z. [1]

(b) Explain why the ray does not leave the fibre at Y.

..........................................................................................................................................

..........................................................................................................................................

......................................................................................................................................[2]

(c) The light in the optical fibre has a wavelength of 3.2 x 10–7 m and is travelling at a speed
of 1.9 x 108 m/s.

(i) Calculate the frequency of the light.

frequency = …………………...

(ii) The speed of light in air is 3.0 x 108 m/s.


Calculate the refractive index of the material from which the fibre is made.

refractive index = …………………...


[4]

© UCLES 2004 0625/03 O/N/04 [Turn over


10

7 Fig. 7.1 shows a 12 V battery connected to a number of resistors.


For
Examiner’s
Use
12 V

4Ω

4Ω A

8Ω

5Ω 5Ω

Fig. 7.1

(a) Calculate the current in the 8 Ω resistor.

current = ………………..[2]

(b) Calculate, for the resistors connected in the circuit, the combined resistance of

(i) the two 5 Ω resistors,

resistance = ………………..

(ii) the two 4 Ω resistors.

resistance = ………………..
[2]

(c) The total current in the two 4 Ω resistors is 6 A.


Calculate the total power dissipated in the two resistors.

power = ………………..[2]

© UCLES 2004 0625/03 O/N/04


11

(d) What will be the reading on a voltmeter connected across


For
Examiner’s
(i) the two 4 Ω resistors, Use

reading = ………………..

(ii) one 5 Ω resistor?

reading = ………………..
[2]

(e) The 8 Ω resistor is made from a length of resistance wire of uniform cross-sectional area.
State the effect on the resistance of the wire of using

(i) the same length of the same material with a greater cross-sectional area,

...................................................................................................................................

(ii) a smaller length of the same material with the same cross-sectional area.

...................................................................................................................................
[2]

© UCLES 2004 0625/03 O/N/04 [Turn over


12

8 Fig. 8.1 shows plane waves passing through a gap in a barrier that is approximately equal to
For
the wavelength of the waves. Examiner’s
Use

barrier

Fig. 8.1

(a) What is the name given to the wave property shown in Fig. 8.1?

......................................................................................................................................[1]

(b) In the space below, carefully draw the pattern that would be obtained if the gap were
increased to six times the wavelength of the waves. [4]

(c) The effect in Fig. 8.1 is often shown using water waves on the surface of a tank of water.
These are transverse waves. Explain what is meant by a transverse wave.

..........................................................................................................................................

..........................................................................................................................................

......................................................................................................................................[2]

© UCLES 2004 0625/03 O/N/04


13

9 (a) An engine on a model railway needs a 6 V a.c. supply. A mains supply of 240 V a.c. is
For
available. Examiner’s
Use
(i) In the space below, draw a labelled diagram of a transformer suitable for producing
the required supply voltage.

(ii) Suggest suitable numbers of turns for the coils.

...................................................................................................................................

...................................................................................................................................
[4]

(b) The power needed for this model engine is 12 W. Calculate the current taken from the
mains when just this engine is in use, assuming that the transformer is 100% efficient.

current = ………………..[2]

(c) Explain why transformers will only work when connected to an a.c. supply.

..........................................................................................................................................

..........................................................................................................................................

......................................................................................................................................[2]

© UCLES 2004 0625/03 O/N/04 [Turn over


14

10 (a) (i) What is the function of a transistor when placed in an electrical circuit?
For
Examiner’s
................................................................................................................................... Use

(ii) Describe the action of a transistor.

...................................................................................................................................

..................................................................................................................................

..................................................................................................................................
[3]

(b) (i) In the space below, draw the symbol for an OR gate. Label the inputs and the output.
[1]

(ii) Describe the action of an OR gate that has two inputs.

...................................................................................................................................

...................................................................................................................................

...............................................................................................................................[2]

© UCLES 2004 0625/03 O/N/04


15

11 (a) The decay of a nucleus of radium 226 88


Ra leads to the emission of an α-particle and For
leaves behind a nucleus of radon (Rn). Examiner’s
Use
In the space below, write an equation to show this decay. [2]

(b) In an experiment to find the range of α-particles in air, the apparatus in Fig. 11.1 was
used.

α−particle source detector

ruler

Fig. 11.1

The results of this experiment are shown below.

count rate / (counts/minute) 681 562 441 382 317 20 19 21 19

distance from source to detector/cm 1 2 3 4 5 6 7 8 9

(i) State what causes the count rate 9 cm from the source.

...................................................................................................................................

(ii) Estimate the count rate that is due to the source at a distance of 2 cm.

...................................................................................................................................

(iii) Suggest a value for the maximum distance that α-particles can travel from the
source.

...................................................................................................................................

(iv) Justify your answer to (iii).

...................................................................................................................................

...................................................................................................................................
[4]

© UCLES 2004 0625/03 O/N/04


UNIVERSITY OF CAMBRIDGE INTERNATIONAL EXAMINATIONS
International General Certificate of Secondary Education
PHYSICS
Paper 3
0625/03
May/June 2005

1 hour 15 minutes
Candidates answer on the Question Paper.
No Additional Materials are required.

Candidate
Name

Centre Candidate
Number Number

READ THESE INSTRUCTIONS FIRST

Write your Centre number, candidate number and name on all the work you
hand in.
Write in dark blue or black pen in the spaces provided on the Question Paper. For Examiner’s Use
You may use a soft pencil for any diagrams, graphs or rough working. 1
Do not use staples, paper clips, highlighters, glue or correction fluid.
2
Answer all questions.
At the end of the examination, fasten all your work securely together. 3
The number of marks is given in brackets [ ] at the end of each question or
part question. 4
You may lose marks if you do not show your working or if you do not use
appropriate units. 5
Take the weight of 1 kg to be 10 N (i.e. acceleration of free fall = 10 m/s2).
6
DO NOT WRITE IN THE BARCODE.
DO NOT WRITE IN THE GREY AREAS BETWEEN THE PAGES. 7

10

11

Total

This document consists of 15 printed pages and 1 blank page.


SPA (SJF3442/CG) S92054/2.1
© UCLES 2005 [Turn over
2

1 A solid plastic sphere falls towards the Earth. For


Examiner’s
Fig. 1.1 is the speed-time graph of the fall up to the point where the sphere hits the Earth’s Use
surface.

140
R S T
120
speed
m/s
100

80

60
Q
40

20
P
0
0 10 20 30 40 50 60 70 80 90 100 110
time / s

Fig. 1.1

(a) Describe in detail the motion of the sphere shown by the graph.

..........................................................................................................................................

..........................................................................................................................................

..........................................................................................................................................

..........................................................................................................................................

..................................................................................................................................... [3]

© UCLES 2005 0625/03/M/J/05


3

(b) On Fig. 1.2, draw arrows to show the directions of the forces acting on the sphere when For
it is at the position shown by point S on the graph. Label your arrows with the names of Examiner’s
the forces. [2] Use

Fig. 1.2

(c) Explain why the sphere is moving with constant speed at S.

..........................................................................................................................................

..........................................................................................................................................

..................................................................................................................................... [2]

(d) Use the graph to calculate the approximate distance that the sphere falls

(i) between R and T,

distance = ………………. [2]


(ii) between P and Q.

distance = ………………. [2]

© UCLES 2005 0625/03/M/J/05 [Turn over


4

2 Fig. 2.1 shows a simple pendulum that swings backwards and forwards between P and Q. For
Examiner’s
Use

support

string

P Q
R pendulum bob

Fig. 2.1

(a) The time taken for the pendulum to swing from P to Q is approximately 0.5 s.

Describe how you would determine this time as accurately as possible.

..........................................................................................................................................

..........................................................................................................................................

..................................................................................................................................... [2]

(b) (i) State the two vertical forces acting on the pendulum bob when it is at position R.

1. ...............................................................................................................................

2. .......................................................................................................................... [1]

(ii) The pendulum bob moves along the arc of a circle. State the direction of the
resultant of the two forces in (i).

.............................................................................................................................. [1]

(c) The mass of the bob is 0.2 kg. During the swing it moves so that P is 0.05 m higher
than R.

Calculate the increase in potential energy of the pendulum bob between R and P.

potential energy = ………………. [2]

© UCLES 2005 0625/03/M/J/05


5

3 A mass of 3.0 kg accelerates at 2.0 m/s2 in a straight line. For


Examiner’s
Use
(a) State why the velocity and the acceleration are both described as vector quantities.

..........................................................................................................................................

..................................................................................................................................... [1]

(b) Calculate the force required to accelerate the mass.

force = ………………. [2]

(c) The mass hits a wall.


The average force exerted on the wall during the impact is 120 N.
The area of the mass in contact with the wall at impact is 0.050 m2.
Calculate the average pressure that the mass exerts on the wall during the impact.

pressure = ………………. [2]

© UCLES 2005 0625/03/M/J/05 [Turn over


6

4 Fig. 4.1 shows apparatus that a student uses to make an estimate of the specific heat For
capacity of iron. Examiner’s
Use

thermometer electrical heater

iron block

Fig. 4.1

(a) The power of the heater is known. State the four readings the student must take to find
the specific heat capacity of iron.

1. ......................................................................................................................................

2. ......................................................................................................................................

3. ......................................................................................................................................

4. ................................................................................................................................. [3]

(b) Write down an equation, in words or in symbols, that could be used to work out the
specific heat capacity of iron from the readings in (a).

[2]

© UCLES 2005 0625/03/M/J/05


7

(c) (i) Explain why the value obtained with this apparatus is higher than the actual value. For
Examiner’s
................................................................................................................................... Use

.............................................................................................................................. [1]

(ii) State one addition to the apparatus that would help to improve the accuracy of the
value obtained.

...................................................................................................................................

.............................................................................................................................. [1]

© UCLES 2005 0625/03/M/J/05 [Turn over


8

5 (a) Fig. 5.1 shows the paths of a few air molecules and a single dust particle. The actual air For
molecules are too small to show on the diagram. Examiner’s
Use

paths of
air molecules

dust particle

Fig. 5.1

Explain why the dust particle undergoes small random movements.

..........................................................................................................................................

..........................................................................................................................................

..........................................................................................................................................

..................................................................................................................................... [4]

(b) Fig. 5.2 shows the paths of a few molecules leaving the surface of a liquid. The liquid is
below its boiling point.

air and vapour


liquid

Fig. 5.2

(i) State which liquid molecules are most likely to leave the surface.

...................................................................................................................................

.............................................................................................................................. [1]

(ii) Explain your answer to (i).

...................................................................................................................................

...................................................................................................................................

.............................................................................................................................. [2]

© UCLES 2005 0625/03/M/J/05


9

6 Fig. 6.1 shows a ray of light OPQ passing through a semi-circular glass block. For
Examiner’s
Use
O

30°

Fig. 6.1

(a) Explain why there is no change in the direction of the ray at P.

..........................................................................................................................................

..................................................................................................................................... [1]

(b) State the changes, if any, that occur to the speed, wavelength and frequency of the light
as it enters the glass block.

..........................................................................................................................................

..........................................................................................................................................

..................................................................................................................................... [2]

(c) At Q some of the light in ray OPQ is reflected and some is refracted.

On Fig. 6.1, draw in the approximate positions of the reflected ray and the refracted ray.
Label these rays. [2]

(d) The refractive index for light passing from glass to air is 0.67.

Calculate the angle of refraction of the ray that is refracted at Q into air.

angle = ………………. [3]

© UCLES 2005 0625/03/M/J/05 [Turn over


10

7 Fig. 7.1 shows the parts of the electromagnetic spectrum. For


Examiner’s
Use
v
i
γ - rays and X - rays ultra- s infra- radio
violet i red waves
b
l
e

Fig. 7.1

(a) Name one type of radiation that has

(i) a higher frequency than ultra-violet,

.............................................................................................................................. [1]

(ii) a longer wavelength than visible light.

.............................................................................................................................. [1]

(b) Some γ-rays emitted from a radioactive source have a speed in air of 3.0 x 108 m/s and
a wavelength of 1.0 x 10–12 m.

Calculate the frequency of the γ-rays.

frequency = ………………. [2]

(c) State the approximate speed of infra-red waves in air.

..................................................................................................................................... [1]

© UCLES 2005 0625/03/M/J/05


11

8 A student has a power supply, a resistor, a voltmeter, an ammeter and a variable resistor.
For
Examiner’s
(a) The student obtains five sets of readings from which he determines an average value Use
for the resistance of the resistor.

In the space below, draw a labelled diagram of a circuit that he could use.

[3]

(b) Describe how the circuit should be used to obtain the five sets of readings.

..........................................................................................................................................

..........................................................................................................................................

..................................................................................................................................... [2]

(c) Fig. 8.1 shows another circuit.

6.0 V

A
resistor resistor of
3.0 Ω unknown value

Fig. 8.1

When the circuit is switched on, the ammeter reads 0.50 A.

(i) Calculate the value of the unknown resistor.

resistance = ………………. [2]

(ii) Calculate the charge passing through the 3.0 Ω resistor in 120 s.

charge = ………………. [1]

(iii) Calculate the power dissipated in the 3.0 Ω resistor.

power = ………………. [2]

© UCLES 2005 0625/03/M/J/05 [Turn over


12

9 (a) Fig. 9.1 shows an a.c. supply connected to a resistor and a diode. For
Examiner’s
Use

resistor
a.c. supply output

Fig. 9.1

(i) State the effect of fitting the diode in the circuit.

...................................................................................................................................

.............................................................................................................................. [1]

(ii) On Fig. 9.2, sketch graphs to show the variation of the a.c. supply voltage and the
output voltage with time.

a.c. supply
voltage
0
time

output
voltage
0
time

Fig. 9.2
[2]

(b) (i) In the space below, draw the symbol for a NOT gate.

[1]

(ii) State the action of a NOT gate.

...................................................................................................................................

...................................................................................................................................

.............................................................................................................................. [2]

© UCLES 2005 0625/03/M/J/05


13

10 (a) Fig. 10.1 is the decay curve for a radioactive isotope that emits only β-particles. For
Examiner’s
Use
400
count rate
300
counts / min

200

100

0
0 10 20 30 40
time / min

Fig. 10.1

Use the graph to find the value of the half-life of the isotope.

Indicate, on the graph, how you arrived at your value.

half-life …………………………. [2]

(b) A student determines the percentage of β-particles absorbed by a thick aluminium


sheet. He uses a source that is emitting only β-particles and that has a long half-life.

(i) In the space below, draw a labelled diagram of the apparatus required, set up to
make the determination.

[2]

(ii) List the readings that the student needs to take.

...................................................................................................................................

...................................................................................................................................

...................................................................................................................................

.............................................................................................................................. [3]

© UCLES 2005 0625/03/M/J/05 [Turn over


14

11 Fig. 11.1 shows a flexible wire hanging between two magnetic poles. The flexible wire is For
connected to a 12 V d.c. supply that is switched off. Examiner’s
Use

wire fixed here

+
N S 12 V d.c.

flexible wire hanging


between magnetic poles

wire fixed here

Fig. 11.1

(a) Explain why the wire moves when the supply is switched on.

..........................................................................................................................................

..........................................................................................................................................

..................................................................................................................................... [2]

(b) State the direction of the deflection of the wire.

..........................................................................................................................................

..................................................................................................................................... [2]

(c) When the wire first moves, energy is changed from one form to another. State these two
forms of energy.

from ........................................................... to ............................................................ [1]

© UCLES 2005 0625/03/M/J/05


15

(d) Fig. 11.2 shows the flexible wire made into a rigid rectangular coil and mounted on an For
axle. Examiner’s
Use

magnetic pole

axle
N N
coil

S S
magnetic pole

axle

Fig. 11.2

(i) Add to the diagram an arrangement that will allow current to be fed into the coil
whilst allowing the coil to turn continuously. Label the parts you have added. [1]

(ii) Briefly explain how your arrangement works.

...................................................................................................................................

.............................................................................................................................. [2]

© UCLES 2005 0625/03/M/J/05


16

BLANK PAGE

Permission to reproduce items where third-party owned material protected by copyright is included has been sought and cleared where possible. Every
reasonable effort has been made by the publisher (UCLES) to trace copyright holders, but if any items requiring clearance have unwittingly been included, the
publisher will be pleased to make amends at the earliest possible opportunity.

University of Cambridge International Examinations is part of the University of Cambridge Local Examinations Syndicate (UCLES), which is itself a department of
the University of Cambridge.

0625/03/M/J/05
UNIVERSITY OF CAMBRIDGE INTERNATIONAL EXAMINATIONS
International General Certificate of Secondary Education
PHYSICS
Paper 3 Extended
0625/03
October/November 2005

1 hour 15 minutes
Candidates answer on the Question Paper.
No Additional Materials are required.

READ THESE INSTRUCTIONS FIRST

Write your Centre number, candidate number and name on all the work you
hand in. For Examiner’s Use
Write in dark blue or black pen in the spaces provided on the Question Paper.
You may use a soft pencil for any diagrams, graphs, music or rough working. 1
Do not use staples, paper clips, highlighters, glue or correction fluid.
2
Answer all questions.
At the end of the examination, fasten all your work securely together. 3
Take the weight of 1 kg to be 10 N (i.e. acceleration of free fall = 10 m/s2).
The number of marks is given in brackets [ ] at the end of each question or 4
part question.
5
You may lose marks if you do not show your working or if you do not use
appropriate units. 6
DO NOT WRITE IN THE BARCODE. 7

DO NOT WRITE IN THE GREY AREAS BETWEEN THE PAGES. 8

10

11

Total

This document consists of 14 printed pages and 2 blank pages.


SP (SJF3441/CG) T03202/4
© UCLES 2005 [Turn over
2

1 (a) State what is meant by the terms


For
Examiner’s
(i) weight, ...................................................................................................................... Use

.............................................................................................................................. [1]

(ii) density. ......................................................................................................................

.............................................................................................................................. [1]

(b) A student is given a spring balance that has a scale in newtons. The student is told that
the acceleration of free-fall is 10 m/s2.

(i) Describe how the student could find the mass of an irregular solid object.

...................................................................................................................................

...................................................................................................................................

.............................................................................................................................. [2]

(ii) Describe how the student could go on to find the density of the object.

...................................................................................................................................

...................................................................................................................................

...................................................................................................................................

.............................................................................................................................. [2]

(c) Fig. 1.1 shows three forces acting on an object of mass 0.5 kg. All three forces act
through the centre of mass of the object.

centre of mass

9.0 N 3.0 N
4.0 N

Fig. 1.1

Calculate

(i) the magnitude and direction of the resultant force on the object,

magnitude = ……………… direction .............................................................. [2]

(ii) the magnitude of the acceleration of the object.

acceleration =…………………… [2]

© UCLES 2005 0625/03/O/N/05


3

2 Fig. 2.1 shows apparatus for investigating moments of forces.


For
Examiner’s
Use

spring
balance horizontally balanced
metre rule

0 10 20 30 40 50 60 70 80 90 100

horizontal pivot
6.0 N
weight

Fig. 2.1

The uniform metre rule shown in Fig. 2.1 is in equilibrium.

(a) Write down two conditions for the metre rule to be in equilibrium.

condition 1 ........................................................................................................................

..........................................................................................................................................

..........................................................................................................................................

condition 2 ........................................................................................................................

..........................................................................................................................................

..................................................................................................................................... [2]

(b) Show that the value of the reading on the spring balance is 8.0 N. [2]

(c) The weight of the uniform metre rule is 1.5 N.

Calculate the force exerted by the pivot on the metre rule.

magnitude of force = …………………………………

direction of force ………………………………… [2]

© UCLES 2005 0625/03/O/N/05 [Turn over


4

3 Fig. 3.1 shows a pond that is kept at a constant depth by a pressure-operated valve in the
For
base. Examiner’s
Use

water

pressure-operated
outlet valve
spring

Fig. 3.1

(a) The pond is kept at a depth of 2.0 m. The density of water is 1000 kg/m3.

Calculate the water pressure on the valve.

pressure =…………………….. [2]

(b) The force required to open the valve is 50 N. The valve will open when the water depth
reaches 2.0 m.

Calculate the area of the valve.

area = ……………………….. [2]

(c) The water supply is turned off and the valve is held open so that water drains out
through the valve.

State the energy changes of the water that occur as the depth of the water drops from
2.0 m to zero.

..........................................................................................................................................

..................................................................................................................................... [2]

© UCLES 2005 0625/03/O/N/05


5

4 Fig. 4.1 shows apparatus that could be used to measure the specific latent heat of ice.
For
Examiner’s
Use

60 W heater
power supply

melting ice at 0 °C

beaker

Fig. 4.1

(a) Describe how you would use the apparatus. You may assume that ice at 0 °C and a
stopwatch are available. State all the readings that would be needed at each stage.

..........................................................................................................................................

..........................................................................................................................................

..........................................................................................................................................

..........................................................................................................................................

..................................................................................................................................... [4]

(b) In an experiment, 120 g of ice at 0 °C is to be melted. The specific latent heat of ice is
340 J/g. Assume that all the energy from the heater will be used to melt the ice.

Calculate the expected time for which the 60 W heater is switched on.

expected time = …………………… [2]

(c) When the experiment is carried out, the ice melts in slightly less time than the expected
time.
(i) State one reason why this happens.

...................................................................................................................................

.............................................................................................................................. [1]

(ii) Suggest one modification to the experiment that would reduce the difference
between the experimental time and the expected time.

...................................................................................................................................

.............................................................................................................................. [1]
© UCLES 2005 0625/03/O/N/05 [Turn over
6

5 Fig. 5.1 shows a way of indicating the positions and direction of movement of some
For
molecules in a gas at one instant. Examiner’s
Use

piston
cylinder

Fig. 5.1

(a) (i) Describe the movement of the molecules.

.............................................................................................................................. [1]

(ii) Explain how the molecules exert a pressure on the container walls.

...................................................................................................................................

.............................................................................................................................. [1]

(b) When the gas in the cylinder is heated, it pushes the piston further out of the cylinder.

State what happens to

(i) the average spacing of the molecules,

.............................................................................................................................. [1]

(ii) the average speed of the molecules.

.............................................................................................................................. [1]

(c) The gas shown in Fig. 5.1 is changed into a liquid and then into a solid by cooling.

Compare the gaseous and solid states in terms of

(i) the movement of the molecules,

...................................................................................................................................

.............................................................................................................................. [1]

(ii) the average separation of the molecules.

...................................................................................................................................

.............................................................................................................................. [1]

© UCLES 2005 0625/03/O/N/05


7

6 Fig. 6.1 shows the path of a sound wave from a source X.


For
Examiner’s
Use

X
path of sound wave

Y wall

Fig. 6.1

(a) State why a person standing at point Y hears an echo.

..................................................................................................................................... [1]

(b) The frequency of the sound wave leaving X is 400 Hz. State the frequency of the sound
wave reaching Y.

frequency = ……………….. [1]

(c) The speed of the sound wave leaving X is 330 m/s. Calculate the wavelength of these
sound waves.

wavelength = ………………………. [2]

(d) Sound waves are longitudinal waves.

State what is meant by the term longitudinal.

..........................................................................................................................................

..................................................................................................................................... [1]

© UCLES 2005 0625/03/O/N/05 [Turn over


8

7 (a) Fig. 7.1 shows two rays of light from a point O on an object. These rays are incident on
For
a plane mirror. Examiner’s
Use

Fig. 7.1

(i) On Fig. 7.1, continue the paths of the two rays after they reach the mirror. Hence
locate the image of the object O. Label the image I. [2]

(ii) Describe the nature of the image I.

...................................................................................................................................

.............................................................................................................................. [2]

(b) Fig. 7.2 is drawn to scale. It shows an object PQ and a convex lens.

position of
P convex lens

F F principal
Q axis
principal focus principal focus

Fig. 7.2

© UCLES 2005 0625/03/O/N/05


9

(i) On Fig. 7.2, draw two rays from the top of the object P that pass through the lens.
For
Use these rays to locate the top of the image. Label this point T. [3] Examiner’s
Use
(ii) On Fig. 7.2, draw an eye symbol to show the position from which the image T
should be viewed. [1]

© UCLES 2005 0625/03/O/N/05 [Turn over


10

8 Fig. 8.1 shows a high-voltage supply connected across two metal plates.
For
Examiner’s
+ – Use

high-voltage
supply
A

metal plates

Fig. 8.1

When the supply is switched on, an electric field is present between the plates.

(a) Explain what is meant by an electric field .

..................................................................................................................................... [2]

(b) On Fig. 8.1, draw the electric field lines between the plates and indicate their direction
by arrows. [2]

(c) The metal plates are now joined by a high-resistance wire. A charge of 0.060 C passes
along the wire in 30 s.
Calculate the reading on the ammeter.

ammeter reading = ……………………… [2]

(d) The potential difference of the supply is re-set to 1500 V and the ammeter reading
changes to 0.0080 A. Calculate the energy supplied in 10 s. Show your working.

energy = ………………. [3]

© UCLES 2005 0625/03/O/N/05


11

9 (a) In the space provided, draw the symbol for a NOR gate. Label the inputs and the output.
For
Examiner’s
Use

[2]

(b) State whether the output of a NOR gate will be high (ON) or low (OFF) when

(i) one input is high and one input is low,

...................................................................................................................................

(ii) both inputs are high.

...................................................................................................................................
[1]

(c) Fig. 9.1 shows a digital circuit made from three NOT gates and one NAND gate.

HIGH

LOW

Fig. 9.1

(i) Write HIGH or LOW in each of the boxes on Fig. 9.1. [2]

(ii) State the effect on the output of changing both of the inputs.

...................................................................................................................................

.............................................................................................................................. [1]

© UCLES 2005 0625/03/O/N/05 [Turn over


12

10 Fig. 10.1 shows the basic parts of a transformer.


For
Examiner’s
Use
X
primary coil secondary coil

input output

Fig. 10.1

(a) Use ideas of electromagnetic induction to explain how the input voltage is transformed
into an output voltage. Use the three questions below to help you with your answer.

What happens in the primary coil?

..........................................................................................................................................

..........................................................................................................................................

..........................................................................................................................................

..........................................................................................................................................

What happens in the core?

..........................................................................................................................................

..........................................................................................................................................

What happens in the secondary coil?

..........................................................................................................................................

..........................................................................................................................................

..................................................................................................................................... [5]

(b) State what is needed to make the output voltage higher than the input voltage.

..................................................................................................................................... [1]

© UCLES 2005 0625/03/O/N/05


13

(c) The core of this transformer splits along XX and YY. Explain why the transformer would
For
not work if the two halves of the core were separated by about 30 cm. Examiner’s
Use
..........................................................................................................................................

..................................................................................................................................... [1]

(d) A 100% efficient transformer is used to step up the voltage of a supply from 100 V to
200 V. A resistor is connected to the output. The current in the primary coil is 0.4 A.

Calculate the current in the secondary coil.

current = …………………… [2]

© UCLES 2005 0625/03/O/N/05 [Turn over


14

11 A radioactive source emits only β-particles.


For
Examiner’s
(a) A scientist wishes to investigate the deflection of β-particles by an electric field. Draw a Use
labelled diagram to suggest a suitable experimental arrangement.

[3]

(b) State how the apparatus would be used to show the deflection of the β-particles by the
electric field.

..........................................................................................................................................

..........................................................................................................................................

..................................................................................................................................... [2]

(c) State how the results would show the deflection of the β-particles.

..........................................................................................................................................

..................................................................................................................................... [1]

(d) Explain the direction of the deflection obtained.

..........................................................................................................................................

..................................................................................................................................... [1]

© UCLES 2005 0625/03/O/N/05


15

BLANK PAGE

0625/03/O/N/05
16

BLANK PAGE

Permission to reproduce items where third-party owned material protected by copyright is included has been sought and cleared where possible. Every
reasonable effort has been made by the publisher (UCLES) to trace copyright holders, but if any items requiring clearance have unwittingly been included, the
publisher will be pleased to make amends at the earliest possible opportunity.

University of Cambridge International Examinations is part of the University of Cambridge Local Examinations Syndicate (UCLES), which is itself a department of
the University of Cambridge.

0625/03/O/N/05
UNIVERSITY OF CAMBRIDGE INTERNATIONAL EXAMINATIONS
International General Certificate of Secondary Education
PHYSICS
Paper 3 Extended
0625/03
May/June 2006

1 hour 15 minutes
Candidates answer on the Question Paper.
No Additional Materials are required.

READ THESE INSTRUCTIONS FIRST

Write your Centre number, candidate number and name on all the work you
hand in. For Examiner’s Use
Write in dark blue or black pen.
1
You may use a soft pencil for any diagrams, graphs or rough working.
Do not use staples, paper clips, highlighters, glue or correction fluid.
2
Answer all questions. 3
You may lose marks if you do not show your working or if you do not use
appropriate units. 4
Take the weight of 1 kg to be 10 N (i.e. acceleration of free fall = 10 m/s2).
DO NOT WRITE IN THE BARCODE. 5

DO NOT WRITE IN THE GREY AREAS BETWEEN THE PAGES. 6

At the end of the examination, fasten all your work securely together. 7
The number of marks is given in brackets [ ] at the end of each question or
part question. 8

10

11

Total

This document consists of 12 printed pages.

SP (SLM/KS) T04130/2
© UCLES 2006 [Turn over
2

1 A bus travels from one bus stop to the next. The journey has three distinct parts. Stated in For
order they are Examiner’s
uniform acceleration from rest for 8.0 s, Use

uniform speed for 12 s,


non-uniform deceleration for 5.0 s.
Fig. 1.1 shows only the deceleration of the bus.

15
speed
m/s

10

0
0 5 10 15 20 25
time/s

Fig. 1.1

(a) On Fig. 1.1, complete the graph to show the first two parts of the journey. [3]

(b) Calculate the acceleration of the bus 4.0 s after leaving the first bus stop.

acceleration = ........................[2]

(c) Use the graph to estimate the distance the bus travels between 20 s and 25 s.

estimated distance = ........................[2]

(d) On leaving the second bus stop, the uniform acceleration of the bus is 1.2 m / s2. The
mass of the bus and passengers is 4000 kg.
Calculate the accelerating force that acts on the bus.

force = ........................[2]

(e) The acceleration of the bus from the second bus stop is less than that from the first bus
stop.
Suggest two reasons for this.

1. ......................................................................................................................................

..........................................................................................................................................

2. ......................................................................................................................................

......................................................................................................................................[2]
© UCLES 2006 0625/03/M/J/06
3

2 A student sets up the apparatus shown in Fig. 2.1 in order to find the resultant of the two For
tensions T1 and T2 acting at P. When the tensions T1, T2 and T3 are balanced, the angles Examiner’s
between T1 and the vertical and T2 and the vertical are as marked on Fig. 2.1. Use

pulley

vertical
pulley T1 = 6.0 N T2 = 8.0 N board
44°
69°

T3

Fig. 2.1

In the space below, draw a scale diagram of the forces T1 and T2. Use the diagram to find the
resultant of the two forces.

State

(a) the scale used, scale = ........................................

(b) the value of the resultant, value = ........................................

(c) the direction of the resultant. direction = ........................................


[6]

© UCLES 2006 0625/03/M/J/06 [Turn over


4

3 An electric pump is used to raise water from a well, as shown in Fig. 3.1. For
Examiner’s
Use
pump

ground

well

Fig. 3.1

(a) The pump does work in raising the water. State an equation that could be used to
calculate the work done in raising the water.

......................................................................................................................................[2]

(b) The water is raised through a vertical distance of 8.0 m. The weight of water raised in
5.0 s is 100 N.

(i) Calculate the work done in raising the water in this time.

work done = .......................[1]

(ii) Calculate the power the pump uses to raise the water.

power = ........................[1]

(iii) The energy transferred by the pump to the water is greater than your answer to (i).
Suggest what the additional energy is used for.

..............................................................................................................................[1]

© UCLES 2006 0625/03/M/J/06


5

4 (a) State two differences between evaporation of water and boiling of water. For
Examiner’s
1. ...................................................................................................................................... Use

2. ..................................................................................................................................[2]

(b) The specific latent heat of vaporisation of water is 2260 kJ / kg.


Explain why this energy is needed to boil water and why the temperature of the water
does not change during the boiling.

..........................................................................................................................................

..........................................................................................................................................

..........................................................................................................................................

......................................................................................................................................[3]

(c) A laboratory determination of the specific latent heat of vaporisation of water uses a
120 W heater to keep water boiling at its boiling point. Water is turned into steam at the
rate of 0.050 g / s.
Calculate the value of the specific latent heat of vaporisation obtained from this
experiment. Show your working.

specific latent heat of vaporisation = ........................[3]

© UCLES 2006 0625/03/M/J/06 [Turn over


6

5 (a) Fig. 5.1 shows a tank used for evaporating salt solution to produce crystals. For
Examiner’s
Use
evaporating tank

steam in
salt solution

steam out

Fig. 5.1

Suggest two ways of increasing the rate of evaporation of the water from the solution.
Changes may be made to the apparatus, but the rate of steam supply must stay constant.
You may assume the temperature of the salt solution remains constant.

1. ......................................................................................................................................

..........................................................................................................................................

2. ......................................................................................................................................

......................................................................................................................................[2]

(b) A manufacturer of liquid-in-glass thermometers changes the design in order to meet


new requirements.

Describe the changes that could be made to

(i) give the thermometer a greater range,

..............................................................................................................................[1]

(ii) make the thermometer more sensitive.

..............................................................................................................................[1]

(c) A toilet flush is operated by the compression of air. The air inside the flush has a
pressure of 1.0 × 105 Pa and a volume of 150 cm3. When the flush is operated the
volume is reduced to 50 cm3. The temperature of the air remains constant during this
process.
Calculate the new pressure of the air inside the flush.

pressure = .......................[2]

© UCLES 2006 0625/03/M/J/06


7

6 Fig. 6.1 shows white light incident at P on a glass prism. Only the refracted red ray PQ is For
shown in the prism. Examiner’s
Use

P red ray
Q
t
white ligh
screen

Fig. 6.1

(a) On Fig. 6.1, draw rays to complete the path of the red ray and the whole path of the
violet ray up to the point where they hit the screen. Label the violet ray. [3]

(b) The angle of incidence of the white light is increased to 40°. The refractive index of the
glass for the red light is 1.52.
Calculate the angle of refraction at P for the red light.

angle of refraction = ........................[3]

(c) State the approximate speed of

(i) the white light incident at P, speed = ........................ [1]

(ii) the red light after it leaves the prism at Q. speed = ........................ [1]

© UCLES 2006 0625/03/M/J/06 [Turn over


8

7 Fig. 7.1 shows how the air pressure at one instant varies with distance along the path of a For
continuous sound wave. Examiner’s
Use

air pressure

normal P X Y
air pressure distance in direction
of travel of the wave

Fig. 7.1

(a) What type of waves are sound waves?

......................................................................................................................................[1]

(b) On Fig. 7.1, mark on the axis PY

(i) one point C where there is a compression in the wave, [1]

(ii) one point R where there is a rarefaction in the wave. [1]

(c) Describe the motion of a group of air particles situated on the path of the wave shown in
Fig. 7.1.

..........................................................................................................................................

..........................................................................................................................................

......................................................................................................................................[2]

(d) The sound wave shown has speed of 340 m / s and a frequency of 200 Hz.
Calculate the distance represented by PX on Fig. 7.1.

distance = ........................[2]

© UCLES 2006 0625/03/M/J/06


9

8 Fig. 8.1 shows an electrical circuit. For


Examiner’s
12.0 V d.c. Use

A C B R

4.0 Ω
one metre resistance wire
sliding
contact

Fig. 8.1

The resistance of the lamp is 4.0 Ω when it is at its normal brightness.

(a) The lamp is rated at 6.0 V, 9.0 W.


Calculate the current in the lamp when it is at its normal brightness.

current = ........................[2]

(b) The sliding contact C is moved to A. The lamp lights at its normal brightness.
Calculate

(i) the total circuit resistance,

resistance = ........................[1]

(ii) the potential difference across the 4.0 Ω resistor R.

potential difference = ........................[1]

(c) The sliding contact C is moved from A to B.

(i) Describe any change that occurs in the brightness of the lamp.

..............................................................................................................................[1]

(ii) Explain your answer to (i).

..................................................................................................................................

..............................................................................................................................[2]

(d) The 1 m wire between A and B, as shown in Fig. 8.1, has a resistance of 2.0 Ω.
Calculate the resistance between A and B when

(i) the 1 m length is replaced by a 2 m length of the same wire,

resistance = ........................[1]

(ii) the 1 m length is replaced by a 1 m length of a wire of the same material but of only
half the cross-sectional area.

resistance = ........................[1]

© UCLES 2006 0625/03/M/J/06 [Turn over


10

9 A transformer is needed to step down a 240 V a.c. supply to a 12 V a.c. output. For
Examiner’s
(a) In the space below, draw a labelled diagram of a suitable transformer. [3] Use

(b) Explain

(i) why the transformer only works on a.c.,

..................................................................................................................................

..............................................................................................................................[1]

(ii) how the input voltage is changed to an output voltage.

..................................................................................................................................

..................................................................................................................................

..............................................................................................................................[2]

(c) The output current is 1.5 A.

Calculate

(i) the power output,

power = ........................[1]

(ii) the energy output in 30 s.

energy = ........................[1]

© UCLES 2006 0625/03/M/J/06


11

10 (a) Fig. 10.1 shows a positively charged plastic rod, a metal plate resting on an insulator, For
and a lead connected to earth. Examiner’s
Use

positively charged
plastic rod

metal plate
lead connected
insulator
to earth

Fig. 10.1

Describe how the metal plate may be charged by induction.

..........................................................................................................................................

..........................................................................................................................................

......................................................................................................................................[3]

(b) An electrostatic generator sets up a current of 20 mA in a circuit.

Calculate

(i) the charge flowing through the circuit in 15 s,

charge = ............................

(ii) the potential difference across a 10 kΩ resistor in the circuit.

potential difference = ............................


[3]

© UCLES 2006 0625/03/M/J/06 [Turn over


12

11 Fig. 11.1 shows a beam of radiation that contains α-particles, β-particles and γ-rays. The For
beam enters a very strong magnetic field shown in symbol form by N and S poles. Examiner’s
Use

beam of
radiation

Fig. 11.1

Complete the table below.

direction of deflection, charge carried by


radiation
if any radiation, if any
α-particles

β-particles

γ-rays

[6]

Permission to reproduce items where third-party owned material protected by copyright is included has been sought and cleared where possible. Every reasonable
effort has been made by the publisher (UCLES) to trace copyright holders, but if any items requiring clearance have unwittingly been included, the publisher will
be pleased to make amends at the earliest possible opportunity.

University of Cambridge International Examinations is part of the University of Cambridge Local Examinations Syndicate (UCLES), which is itself a department
of the University of Cambridge.

© UCLES 2006 0625/03/M/J/06


UNIVERSITY OF CAMBRIDGE INTERNATIONAL EXAMINATIONS
International General Certificate of Secondary Education
PHYSICS
Paper 3 Extended
0625/03
October/November 2006

1 hour 15 minutes
Candidates answer on the Question Paper.
No Additional Materials are required.

READ THESE INSTRUCTIONS FIRST

Write your Centre number, candidate number and name on all the work you
hand in. For Examiner’s Use
Write in dark blue or black pen.
1
You may use a soft pencil for any diagrams, graphs or rough working.
Do not use staples, paper clips, highlighters, glue or correction fluid.
2
Answer all questions. 3
You may lose marks if you do not show your working or if you do not use
appropriate units. 4
Take the weight of 1 kg to be 10 N (i.e. acceleration of free fall = 10 m/s2).
DO NOT WRITE IN THE BARCODE. 5

DO NOT WRITE IN THE GREY AREAS BETWEEN THE PAGES. 6

At the end of the examination, fasten all your work securely together. 7
The number of marks is given in brackets [ ] at the end of each question or
part question. 8

10

11

Total

This document consists of 15 printed pages and 1 blank page.

SP (SLM/KS) T04132/2
© UCLES 2006 [Turn over
2

1 (a) A stone falls from the top of a building and hits the ground at a speed of 32 m/s. For
The air resistance-force on the stone is very small and may be neglected. Examiner’s
Use

(i) Calculate the time of fall.

time = ............................

(ii) On Fig. 1.1, draw the speed-time graph for the falling stone.

40

30
speed
m/s

20

10

0
0 1 2 3 4
time/s

Fig. 1.1

(iii) The weight of the stone is 24 N.


Calculate the mass of the stone.

mass = ............................
[5]

© UCLES 2006 0625/03/O/N/06


3

(b) A student used a suitable measuring cylinder and a spring balance to find the density of For
a sample of the stone. Examiner’s
Use

(i) Describe how the measuring cylinder is used, and state the readings that are
taken.

..................................................................................................................................

..................................................................................................................................

..................................................................................................................................

..................................................................................................................................

(ii) Describe how the spring balance is used, and state the reading that is taken.

..................................................................................................................................

..................................................................................................................................

(iii) Write down an equation from which the density of the stone is calculated.

..................................................................................................................................

(iv) The student then wishes to find the density of cork. Suggest how the apparatus and
the method would need to be changed.

..................................................................................................................................

..................................................................................................................................

..................................................................................................................................
[6]

© UCLES 2006 0625/03/O/N/06 [Turn over


4

2 In an experiment, forces are applied to a spring as shown in Fig. 2.1a. The results of this For
experiment are shown in Fig. 2.1b. Examiner’s
Use

16
R
Q

12
ruler force/N
spring
8.0 P

4.0

weights

0
0 2.0 4.0 6.0
extension/mm

Fig. 2.1a Fig. 2.1b

(a) What is the name given to the point marked Q on Fig. 2.1b?

......................................................................................................................................[1]

(b) For the part OP of the graph, the spring obeys Hooke’s Law.
State what this means.

..........................................................................................................................................

......................................................................................................................................[1]

(c) The spring is stretched until the force and extension are shown by the point R on the
graph. Compare how the spring stretches, as shown by the part of the graph OQ, with
that shown by QR.

..........................................................................................................................................

......................................................................................................................................[1]

(d) The part OP of the graph shows the spring stretching according to the expression

F = kx.

Use values from the graph to calculate the value of k.

k =..................................[2]

© UCLES 2006 0625/03/O/N/06


5

3 Fig. 3.1 shows water falling over a dam. For


Examiner’s
Use
dam falling water

7.0 m

Fig. 3.1

(a) The vertical height that the water falls is 7.0 m.


Calculate the potential energy lost by 1.0 kg of water during the fall.

potential energy = ........................[2]

(b) Assuming all this potential energy loss is changed to kinetic energy of the water,
calculate the speed of the water, in the vertical direction, at the end of the fall.

speed = ........................[3]

(c) The vertical speed of the water is less than that calculated in (b). Suggest one reason
for this.

..........................................................................................................................................

......................................................................................................................................[1]

© UCLES 2006 0625/03/O/N/06 [Turn over


6

4 Some water is heated electrically in a glass beaker in an experiment to find the specific heat For
capacity of water. The temperature of the water is taken at regular intervals. Examiner’s
Use

The temperature-time graph for this heating is shown in Fig. 4.1.

60
temperature / °C
50

40

30

20

10

0
0 60 120 180 240 300
time / s

Fig. 4.1

(a) (i) Use the graph to find

1. the temperature rise in the first 120 s,


............................

2. the temperature rise in the second 120 s interval.


............................

(ii) Explain why these values are different.

..................................................................................................................................

..................................................................................................................................
[2]

(b) The experiment is repeated in an insulated beaker. This time, the temperature of the
water increases from 20 °C to 60 °C in 210 s. The beaker contains 75 g of water. The
power of the heater is 60 W. Calculate the specific heat capacity of water.

specific heat capacity = ........................[4]

© UCLES 2006 0625/03/O/N/06


7

(c) In order to measure the temperature during the heating, a thermocouple is used. For
Draw a labelled diagram of a thermocouple connected to measure temperature. Examiner’s
Use

[2]

© UCLES 2006 0625/03/O/N/06 [Turn over


8

5 (a) Fig. 5.1 shows a copper rod AB being heated at one end. For
Examiner’s
Use
copper rod

B A

Bunsen
burner

Fig. 5.1

(i) Name the process by which heat moves from A to B.

..................................................................................................................................

(ii) By reference to the behaviour of the particles of copper along AB, state how this
process happens.

..................................................................................................................................

..................................................................................................................................
[3]

(b) Give an account of an experiment that is designed to show which of four surfaces will
absorb most heat radiation.
The four surfaces are all the same metal, but one is a polished black surface, one is a
polished silver surface, one is a dull black surface and the fourth one is painted white.
Give your answer under the headings below.

labelled diagram of the apparatus

readings to be taken

one precaution to try to achieve a fair comparison between the various surfaces

..........................................................................................................................................

......................................................................................................................................[3]

© UCLES 2006 0625/03/O/N/06


9

6 Fig. 6.1 shows a ray of light, from the top of an object PQ, passing through two glass For
prisms. Examiner’s
Use

P A B

Q
C

F E

Fig. 6.1

(a) Complete the path through the two prisms of the ray shown leaving Q. [1]

(b) A person looking into the lower prism, at the position indicated by the eye symbol, sees
an image of PQ.
State the properties of this image.

......................................................................................................................................[2]

(c) Explain why there is no change in direction of the ray from P at points A, C, D and F.

..........................................................................................................................................

......................................................................................................................................[1]

(d) The speed of light as it travels from P to A is 3 × 108 m/s and the refractive index of the
prism glass is 1.5.
Calculate the speed of light in the prism.

speed = ........................[2]

(e) Explain why the ray AB reflects through 90° at B and does not pass out of the prism
at B.

..........................................................................................................................................

..........................................................................................................................................

......................................................................................................................................[2]

© UCLES 2006 0625/03/O/N/06 [Turn over


10

7 Fig. 7.1 is a drawing of a student’s attempt to show the diffraction pattern of water waves that For
have passed through a narrow gap in a barrier. Examiner’s
Use

barrier with
narrow gap

direction of
water waves

Fig. 7.1

(a) State two things that are wrong with the wave pattern shown to the right of the barrier.

1. ......................................................................................................................................

2. ..................................................................................................................................[2]

(b) In the space below, sketch the wave pattern when the gap in the barrier is made five
times wider.

[2]

(c) The waves approaching the barrier have a wavelength of 1.2 cm and a frequency of
8.0 Hz.
Calculate the speed of the water waves.

speed =..................................[2]

© UCLES 2006 0625/03/O/N/06


11

8 Fig. 8.1 shows a low-voltage lighting circuit. For


Examiner’s
12 V d.c. Use
supply

Fig. 8.1

(a) On Fig. 8.1, indicate with a dot and the letter S, a point in the circuit where a switch
could be placed that would turn off lamps Y and Z at the same time but would leave
lamp X still lit. [1]

(b) (i) In the space below, draw the circuit symbol for a component that would vary the
brightness of lamp X.

(ii) On Fig. 8.1, mark with a dot and the letter R where this component should be
placed.
[2]

(c) Calculate the current in lamp Y.

current = ........................[2]

(d) The current in lamp Z is 3.0 A. Calculate the resistance of this lamp.

resistance = ........................[2]

(e) The lamp Y is removed.

(i) Why do lamps X and Z still work normally?

..................................................................................................................................

..................................................................................................................................

(ii) The current in lamp X is 1.0 A. Calculate the current supplied by the battery with
lamp Y removed.

current = ............................
[2]
© UCLES 2006 0625/03/O/N/06 [Turn over
12

9 (a) Fig. 9.1 shows how a beam of electrons would be deflected by an electric field produced For
between two metal plates. Examiner’s
The connections of the source of high potential difference are not shown. Use

+ high –
potential
difference
metal
plate
beam of electrons

metal
plate

Fig. 9.1

(i) On Fig. 9.1, draw in the missing connections.

(ii) Explain why the beam of electrons is deflected in the direction shown. In your
answer, consider all the charges involved and their effect on each other.

..................................................................................................................................

..................................................................................................................................

..................................................................................................................................

..................................................................................................................................
[5]

(b) The deflection of a beam of electrons by an electric field is used in cathode-ray


oscilloscopes.

(i) What makes the electron beam move backwards and forwards across the screen?

..................................................................................................................................

..................................................................................................................................

(ii) What makes the electron beam move up and down the screen?

..................................................................................................................................

..................................................................................................................................
[2]

© UCLES 2006 0625/03/O/N/06


13

(c) An a.c. waveform is displayed so that two full waves appear on the screen of a cathode- For
ray oscilloscope. Examiner’s
Use

Fig. 9.2 shows the face of the oscilloscope. On Fig. 9.2, draw in the waveform.

Fig. 9.2
[1]

© UCLES 2006 0625/03/O/N/06 [Turn over


14

10 Fig. 10.1 shows a circuit that is used to switch on a lamp automatically when it starts to go For
dark. Examiner’s
Use

D
A

+
V

C
B

Fig. 10.1

(a) Write down the names of the components labelled A, B, C and D.

A ........................................... B ...........................................

C ........................................... D ........................................... [2]

(b) Which of the four components A, B, C or D acts as a switch?

........................[1]

(c) Explain why the lamp comes on as it goes dark.

..........................................................................................................................................

..........................................................................................................................................

..........................................................................................................................................

......................................................................................................................................[3]

© UCLES 2006 0625/03/O/N/06


15

11 (a) α-particles, β-particles and γ-rays are known as ionising radiations. For
Examiner’s
(i) Describe what happens when gases are ionised by ionising radiations. Use

..................................................................................................................................

..................................................................................................................................

..................................................................................................................................

(ii) Suggest why α-particles are considered better ionisers of gas than β-particles.

..................................................................................................................................

..................................................................................................................................
[3]

(b) (i) Suggest two practical applications of radioactive isotopes.

1. ..............................................................................................................................

2. ..............................................................................................................................

(ii) For one of the applications that you have suggested, describe how it works, or draw
a labelled diagram to illustrate it in use.

..................................................................................................................................

..................................................................................................................................

..................................................................................................................................

[4]

© UCLES 2006 0625/03/O/N/06


16

BLANK PAGE

Permission to reproduce items where third-party owned material protected by copyright is included has been sought and cleared where possible. Every reasonable
effort has been made by the publisher (UCLES) to trace copyright holders, but if any items requiring clearance have unwittingly been included, the publisher will
be pleased to make amends at the earliest possible opportunity.

University of Cambridge International Examinations is part of the University of Cambridge Local Examinations Syndicate (UCLES), which is itself a department
of the University of Cambridge.

0625/03/O/N/06
UNIVERSITY OF CAMBRIDGE INTERNATIONAL EXAMINATIONS
International General Certificate of Secondary Education
*9716875438*

PHYSICS 0625/03
Paper 3 Extended May/June 2007
1 hour 15 minutes
Candidates answer on the Question Paper.
No Additional Materials are required.

READ THESE INSTRUCTIONS FIRST

Write your Centre number, candidate number and name on all the work you hand in.
Write in dark blue or black pen.
You may use a soft pencil for any diagrams, graphs or rough working.
Do not use staples, paper clips, highlighters, glue or correction fluid.
DO NOT WRITE IN ANY BARCODES.

Answer all questions.


For Examiner’s Use
You may lose marks if you do not show your working or if you do not use
appropriate units. 1
Take the weight of 1 kg to be 10 N (i.e. acceleration of free fall = 10 m/s2).
2
At the end of the examination, fasten all your work securely together.
The number of marks is given in brackets [ ] at the end of each question or 3
part question.
4

10

11

Total

This document consists of 15 printed pages and 1 blank page.

SPA (MML 13116 3/06) T25815/6


© UCLES 2007 [Turn over
2 For
Examiner’s
Use
1 Fig. 1.1 shows a model car moving clockwise around a horizontal circular track.

direction of
movement
P
model circular
car track

Fig. 1.1

(a) A force acts on the car to keep it moving in a circle.

(i) Draw an arrow on Fig. 1.1 to show the direction of this force. [1]

(ii) The speed of the car increases. State what happens to the magnitude of this force.

............................................................................................................................ [1]

(b) (i) The car travels too quickly and leaves the track at P. On Fig. 1.1, draw an arrow to
show the direction of travel after it has left the track. [1]

(ii) In terms of the forces acting on the car, suggest why it left the track at P.

..................................................................................................................................

..................................................................................................................................

............................................................................................................................ [2]

© UCLES 2007 0625/03/M/J/07


3 For
Examiner’s
Use
(c) The car, starting from rest, completes one lap of the track in 10 s. Its motion is shown
graphically in Fig. 1.2.

30

25

speed / 20
cm / s
15

10

0
0 1 2 3 4 5 6 7 8 9 10
time / s

Fig. 1.2

(i) Describe the motion between 3.0 s and 10.0 s after the car has started.

............................................................................................................................ [1]

(ii) Use Fig. 1.2 to calculate the circumference of the track.

circumference = ................................................ [2]

(iii) Calculate the increase in speed per second during the time 0 to 3.0 s.

increase in speed per second = ................................................ [2]

[Total: 10]

© UCLES 2007 0625/03/M/J/07 [Turn over


4 For
Examiner’s
Use
2 Fig. 2.1 shows a steam safety valve. When the pressure gets too high, the steam lifts the
weight W and allows steam to escape.

0.2 m

pivot
W

force of
steam

Fig. 2.1

(a) Explain, in terms of moments of forces, how the valve works.

..........................................................................................................................................

..........................................................................................................................................

..........................................................................................................................................

.................................................................................................................................... [2]

(b) The moment of weight W about the pivot is 12 N m. The perpendicular distance of the
line of action of the force of the steam on the valve from the pivot is 0.2 m.

The area of the piston is 0.0003 m2.

Calculate

(i) the minimum steam force needed for the steam to escape,

force = ................................................ [2]

(ii) the minimum steam pressure for the steam to escape.

pressure = ................................................ [2]

[Total: 6]
© UCLES 2007 0625/03/M/J/07
5 For
Examiner’s
Use
3 A student wishes to work out how much power she uses to lift her body when climbing a
flight of stairs.

Her body mass is 60 kg and the vertical height of the stairs is 3.0 m. She takes 12 s to walk
up the stairs.

(a) Calculate

(i) the work done in raising her body mass as she climbs the stairs,

work = ................................................ [2]

(ii) the output power she develops when raising her body mass.

power = ................................................ [2]

(b) At the top of the stairs she has gravitational potential energy.

Describe the energy transformations taking place as she walks back down the stairs
and stops at the bottom.

..........................................................................................................................................

..........................................................................................................................................

..........................................................................................................................................

.................................................................................................................................... [2]

[Total: 6]

© UCLES 2007 0625/03/M/J/07 [Turn over


6 For
Examiner’s
Use
4 Fig. 4.1 shows a student’s attempt to estimate the specific latent heat of fusion of ice by
adding ice at 0 °C to water at 20 °C. The water is stirred continuously as ice is slowly added
until the temperature of the water is 0 °C and all the added ice has melted.

glass rod
thermometer stirrer

ice
glass beaker

water

top-pan balance

Fig. 4.1

(a) Three mass readings are taken. A description of the first reading is given.

Write down descriptions of the other two.

reading 1 the mass of the beaker + stirrer + thermometer

reading 2 .........................................................................................................................

reading 3 ................................................................................................................... [2]

(b) Write down word equations which the student could use to find

(i) the heat lost by the water as it cools from 20 °C to 0 °C,

............................................................................................................................ [1]

(ii) the heat gained by the melting ice.

............................................................................................................................ [1]

© UCLES 2007 0625/03/M/J/07


7 For
Examiner’s
Use
(c) The student calculates that the water loses 12 800 J and that the mass of ice melted is
30 g.

Calculate a value for the specific latent heat of fusion of ice.

specific latent heat of fusion = ................................................ [2]

(d) Suggest two reasons why this value is only an approximate value.

Reason 1 .........................................................................................................................

..........................................................................................................................................

Reason 2 .........................................................................................................................

.................................................................................................................................... [2]

[Total: 8]

© UCLES 2007 0625/03/M/J/07 [Turn over


8 For
Examiner’s
Use
5 Fig. 5.1 shows some apparatus designed to compare the ability of two surfaces to absorb
infra-red radiation.

surface
painted surface
dull painted
black shiny white

Bunsen burner
Fig. 5.1

The containers, which are identical, are painted on the outside. One is dull black, the other is
shiny white. Both are filled with water, initially at the same temperature.

(a) (i) Describe how you would use the apparatus to compare the abilities of the two
surfaces to absorb infra-red radiation.

..................................................................................................................................

..................................................................................................................................

..................................................................................................................................

............................................................................................................................ [2]

(ii) State the result that you would expect.

............................................................................................................................ [1]

(b) The thermometers used have high sensitivity and linear scales.

(i) State what is meant by high sensitivity.

..................................................................................................................................

............................................................................................................................ [1]

(ii) Explain why a high sensitivity is important for this experiment.

..................................................................................................................................

............................................................................................................................ [1]

(iii) State what is meant by a linear scale.

..................................................................................................................................

............................................................................................................................ [1]

[Total: 6]
© UCLES 2007 0625/03/M/J/07
9 For
Examiner’s
Use
6 Fig. 6.1 shows a rectangular glass block ABCD.

o
60
A B
E

D C

Fig. 6.1

(a) The ray FE is partly reflected and partly refracted at E.

(i) On Fig. 6.1, draw in the approximate path of the refracted ray, within and beyond
the block. Label the ray refracted ray. [1]

(ii) On Fig. 6.1, draw in the path of the reflected ray. Label the ray reflected ray. [1]

(b) A second ray, almost parallel to AE, strikes the block at E and is partly refracted at an
angle of refraction of 43°.

(i) State an approximate value for the angle of incidence at E.

................................................. [1]

(ii) State an approximate value for the critical angle for the light in the glass block.

................................................. [1]

(iii) Calculate an approximate value for the refractive index of the glass of the block.

refractive index = ................................................ [2]

(c) The speed of the light along ray FE is 3.0 x 108 m/s. Calculate the speed of the refracted
light in the glass block.

speed = ................................................ [2]

[Total: 8]

© UCLES 2007 0625/03/M/J/07 [Turn over


10 For
Examiner’s
Use
7 Two students are asked to determine the speed of sound in air on the school playing fields.

(a) List the apparatus they need.

..........................................................................................................................................

..........................................................................................................................................

.................................................................................................................................... [1]

(b) List the readings that the students need to take.

..........................................................................................................................................

..........................................................................................................................................

.................................................................................................................................... [1]

(c) State how the speed of sound is calculated from the readings.

.................................................................................................................................... [1]

(d) State one precaution that could be taken to improve the accuracy of the value obtained.

..........................................................................................................................................

.................................................................................................................................... [1]

(e) The table gives some speeds.

speed/ speed of sound speed of sound


m/s in air in water

10

100

1000

10 000

Place a tick in the table to show the speed which is closest to

(i) the speed of sound in air,

(ii) the speed of sound in water.


[2]

[Total: 6]

© UCLES 2007 0625/03/M/J/07


11 For
Examiner’s
Use
8 Fig. 8.1 shows part of a low-voltage lighting circuit containing five identical lamps.

12 V d.c.
supply

A B

Fig. 8.1

(a) Complete the circuit, by the addition of components as necessary, so that

(i) the total current from the supply can be measured,


(ii) the brightness of lamp E only can be varied,
(iii) lamps C and D may be switched on and off together whilst lamps A, B and E remain
on. [4]

(b) All five lamps are marked 12 V, 36 W. Assume that the resistance of each lamp is the
same fixed value regardless of how it is connected in the circuit.

Calculate

(i) the current in one lamp when operating at normal brightness,

current = ................................................ [1]

(ii) the resistance of one lamp when operating at normal brightness,

resistance = ................................................ [1]

(iii) the combined resistance of two lamps connected in parallel with the 12 V supply,

resistance = ................................................ [1]

(iv) the energy used by one lamp in 30 s when operating at normal brightness.

energy = ................................................ [1]

© UCLES 2007 0625/03/M/J/07 [Turn over


12 For
Examiner’s
Use
(c) The whole circuit is switched on. Explain why the brightness of lamps A and B is much
less than that of one lamp operating at normal brightness.

..........................................................................................................................................

..........................................................................................................................................

.................................................................................................................................... [2]

[Total: 10]

9 Fig. 9.1 is a sketch of some apparatus, found in a Science museum, which was once used to
show how electrical energy can be converted into kinetic energy.

When the switch is closed the wheel starts to turn.

switch

+
metal d.c. supply
supports –
metal
magnet spoked
N
wheel
S
N
S small dish of
mercury

magnet
wood base

Fig. 9.1

(a) Explain why the wheel turns when the switch is closed.

..........................................................................................................................................

..........................................................................................................................................

..........................................................................................................................................

.................................................................................................................................... [2]

(b) On Fig. 9.1, draw an arrow to show the direction of rotation of the wheel. [1]

© UCLES 2007 0625/03/M/J/07


13 For
Examiner’s
Use
(c) The d.c. motor is another way to convert electrical energy into kinetic energy.

In the space below, draw a labelled diagram of a d.c. motor.

[3]

(d) Describe how the split-ring commutator on an electric motor works.

..........................................................................................................................................

..........................................................................................................................................

..........................................................................................................................................

.................................................................................................................................... [2]

[Total: 8]

© UCLES 2007 0625/03/M/J/07 [Turn over


14 For
Examiner’s
Use
10 Fig. 10.1 shows a circuit based on a transistor and a thermistor.

power
R2 supply

R1

Fig. 10.1

(a) Describe the action of the thermistor in this circuit.

..........................................................................................................................................

..........................................................................................................................................

..........................................................................................................................................

.................................................................................................................................... [3]

(b) State and explain how the circuit may be modified so that the lamp switches on at a
different temperature.

..........................................................................................................................................

..........................................................................................................................................

.................................................................................................................................... [2]

(c) State one practical use of this circuit.

.................................................................................................................................... [1]

© UCLES 2007 0625/03/M/J/07


15 For
Examiner’s
Use
11 Fig. 11.1 shows the paths of three α-particles moving towards a thin gold foil.

gold foil

Fig. 11.1

Particle A is moving directly towards a gold nucleus.


Particle B is moving along a line which passes close to a gold nucleus.
Particle C is moving along a line which does not pass close to a gold nucleus.

(a) On Fig. 11.1, complete the paths of the α-particles A, B and C. [3]

(b) State how the results of such an experiment, using large numbers of α-particles,
provides evidence for the existence of nuclei in gold atoms.

..........................................................................................................................................

..........................................................................................................................................

..........................................................................................................................................

.................................................................................................................................... [3]

[Total: 12]

© UCLES 2007 0625/03/M/J/07


16

BLANK PAGE

Permission to reproduce items where third-party owned material protected by copyright is included has been sought and cleared where possible. Every
reasonable effort has been made by the publisher (UCLES) to trace copyright holders, but if any items requiring clearance have unwittingly been included, the
publisher will be pleased to make amends at the earliest possible opportunity.

University of Cambridge International Examinations is part of the Cambridge Assessment Group. Cambridge Assessment is the brand name of University of
Cambridge Local Examinations Syndicate (UCLES), which is itself a department of the University of Cambridge.

0625/03/M/J/07
UNIVERSITY OF CAMBRIDGE INTERNATIONAL EXAMINATIONS
International General Certificate of Secondary Education
*8019038925*

PHYSICS 0625/03
Paper 3 Extended October/November 2007
1 hour 15 minutes
Candidates answer on the Question Paper.
No Additional Materials are required.

READ THESE INSTRUCTIONS FIRST

Write your Centre number, candidate number and name on all the work you hand in.
Write in dark blue or black pen.
You may use a soft pencil for any diagrams, graphs or rough working.
Do not use staples, paper clips, highlighters, glue or correction fluid.
DO NOT WRITE IN ANY BARCODES
For Examiner’s Use
Answer all questions.
You may lose marks if you do not show your working or if you do not use 1
appropriate units.
Take the weight of 1 kg to be 10 N (i.e. acceleration of free fall = 10 m/s2). 2
At the end of the examination, fasten all your work securely together.
3
The number of marks is given in brackets [ ] at the end of each question
or part question. 4

10

11

Total

This document consists of 14 printed pages and 2 blank pages.

SPA (MML 13331 4/06) T25941/7


© UCLES 2007 [Turn over
2 For
Examiner’s
Use
1 A large plastic ball is dropped from the top of a tall building.

Fig. 1.1 shows the speed-time graph for the falling ball until it hits the ground.

20

15
speed
m / s 10

0
0 1 2 3 4 5 6
time / s

Fig. 1.1

(a) From the graph estimate,

(i) the time during which the ball is travelling with terminal velocity,

time = ................................................ [1]

(ii) the time during which the ball is accelerating,

time = ................................................ [1]

(iii) the distance fallen while the ball is travelling with terminal velocity,

distance = ................................................ [2]

(iv) the height of the building.

height = ................................................ [2]

© UCLES 2007 0625/03/O/N/07


3 For
Examiner’s
Use
(b) Explain, in terms of the forces acting on the ball, why

(i) the acceleration of the ball decreases,

..................................................................................................................................

..................................................................................................................................

..................................................................................................................................

............................................................................................................................ [3]

(ii) the ball reaches terminal velocity.

..................................................................................................................................

............................................................................................................................ [2]

[Total: 11]

© UCLES 2007 0625/03/O/N/07 [Turn over


4 For
Examiner’s
Use
2 Fig. 2.1 shows a track for a model car.

P
S
Q

0.5 m

0.4 m 0.4 m

T R

Fig. 2.1

The car has no power supply, but can run down a sloping track due to its weight.

(a) The car is released at Q. It comes to rest just before it reaches S and rolls back.

(i) Describe the motion of the car after it starts rolling back and until it eventually
comes to rest.

..................................................................................................................................

..................................................................................................................................

............................................................................................................................ [2]

(ii) Explain in terms of energy transformations why the car, starting at Q, cannot
pass S.

..................................................................................................................................

..................................................................................................................................

............................................................................................................................ [1]

(b) A second car, of mass 0.12 kg, is released from P. It continues until it runs off the track at
T.

Calculate the maximum speed that the car could have at T assuming friction in the car is
negligible.

speed = ................................................ [3]

[Total: 6]

© UCLES 2007 0625/03/O/N/07


5 For
Examiner’s
Use
3 (a) A spring of original length 3.0 cm is extended to a total length of 5.0 cm by a force of
8.0 N.

Assuming the limit of proportionality of the spring has not been reached, calculate the
force needed to extend it to a total length of 6.0 cm.

force = ................................................ [3]

(b) Fig. 3.1 shows the arrangement for an experiment on moments.

spring
pivot
F

metre rule

Fig. 3.1

The spring exerts a force F on the metre rule.

(i) On Fig. 3.1, mark another quantity which must be measured to find the moment of
the force F. [1]

(ii) State how the moment of the force F is calculated.

..................................................................................................................................

............................................................................................................................ [1]

[Total: 5]

© UCLES 2007 0625/03/O/N/07 [Turn over


6 For
Examiner’s
Use
4 Fig. 4.1 shows a sealed steel cylinder filled with high pressure steam.

steam

Fig. 4.1

Fig. 4.2 shows the same cylinder much later when all the steam has condensed.

water

Fig. 4.2

(a) (i) Describe the movement of the molecules in the high pressure steam.

..................................................................................................................................

..................................................................................................................................

............................................................................................................................ [2]

(ii) Explain how the molecules in the steam exert a high pressure on the inside walls of
the cylinder.

..................................................................................................................................

..................................................................................................................................

............................................................................................................................ [2]

(b) Describe, in terms of particles, the process by which heat is transferred through the
cylinder wall.

..........................................................................................................................................

..........................................................................................................................................

.................................................................................................................................... [2]

(c) When all the steam has condensed, 75 g of water is in the cylinder.

Under these high pressure conditions, the specific latent heat of vaporisation of steam
is 3200 J / g.

Calculate the heat lost by the steam as it condenses.

heat = ................................................ [2]

[Total: 8]
© UCLES 2007 0625/03/O/N/07
7 For
Examiner’s
Use
5 Fig. 5.1 shows some apparatus which is to be used to compare the emission of infra-red
radiation from four differently painted surfaces.

this side
this side painted dull
painted shiny white
white
metal box water inlet

this side
this side painted shiny
painted dull black
black

Fig. 5.1

Near the centre of each side is an infra-red detector. The four detectors are identical.

A supply of very hot water is available.

(a) Describe how you would use this apparatus to compare the infra-red radiation from the
four surfaces.

..........................................................................................................................................

..........................................................................................................................................

.................................................................................................................................... [3]

(b) Suggest which surface will be the best emitter and which will be the worst emitter.

best emitter ...........................................

worst emitter ......................................... [1]

(c) The infra-red detectors are made from thermocouples soldered to blackened metal
plates. These are connected to galvanometers.

In the space below, draw a labelled diagram of a thermocouple. [2]

[Total: 6]

© UCLES 2007 0625/03/O/N/07 [Turn over


8 For
Examiner’s
Use
6 Virtual images may be formed by both plane mirrors and by convex lenses.

Fig. 6.1 shows a plane mirror and a convex lens.

F F

Fig. 6.1

(a) On Fig. 6.1, draw rays to locate the approximate positions of the images of the tops of
the two arrow objects O and P.
Label the images. [5]

(b) Both images are virtual.

(i) What is meant by a virtual image?

............................................................................................................................ [1]

(ii) State one other similarity between the two images.

............................................................................................................................ [1]

(iii) State one difference between the two images.

............................................................................................................................ [1]

[Total: 8]

© UCLES 2007 0625/03/O/N/07


9 For
Examiner’s
Use
7 (a) In the space below, draw a diagram to represent a sound wave.

On your diagram, mark and label

(i) two consecutive compressions and two consecutive rarefactions,

(ii) the wavelength of the wave.


[3]

(b) Fig. 7.1 shows part of the electromagnetic spectrum.

INFRA–
X-RAYS
RED

Fig. 7.1

(i) On Fig. 7.1, label the positions of γ-rays, visible light waves and radio waves. [1]

(ii) State which of the three types of wave in (i) has the lowest frequency.

............................................................................................................................ [1]

(iii) State the approximate value of the speed in air of radio waves.

............................................................................................................................ [1]

[Total: 6]

© UCLES 2007 0625/03/O/N/07 [Turn over


10 For
Examiner’s
Use
8 Fig. 8.1 shows two electrical circuits.

ammeter
A 1
4.0 Ω

ammeter P
4.0 Ω 6.0 Ω A
A 2 6.0 Ω
P Q
Q
circuit 1 circuit 2

The batteries in circuit 1 and circuit 2 are identical.

Fig. 8.1

(a) Put ticks in the table below to describe the connections of the two resistors P and Q.

series parallel

circuit 1

circuit 2

[1]
(b) The resistors P and Q are used as small electrical heaters.

State two advantages of connecting them as shown in circuit 2.

advantage 1 .....................................................................................................................

advantage 2 ............................................................................................................... [2]

(c) In circuit 1, the ammeter reads 1.2 A when the switch is closed.

Calculate the reading of the voltmeter in this circuit.

voltmeter reading = ................................................ [2]

(d) The two switches in circuit 2 are closed. Calculate the combined resistance of the two
resistors in this circuit.

combined resistance = ................................................ [2]

© UCLES 2007 0625/03/O/N/07


11 For
Examiner’s
Use
(e) When the switches are closed in circuit 2, ammeter 1 reads 5 A and ammeter 2 reads
2 A.

Calculate

(i) the current in resistor P,

current = ................................................ [1]

(ii) the power supplied to resistor Q,

power = ................................................ [1]

(iii) the energy transformed in resistor Q in 300 s.

energy = ................................................ [1]

[Total: 10]

© UCLES 2007 0625/03/O/N/07 [Turn over


12 For
Examiner’s
Use
9 Electromagnetic induction may be demonstrated using a magnet, a solenoid and other
necessary apparatus.

(a) Explain what is meant by electromagnetic induction.

..........................................................................................................................................

..........................................................................................................................................

..........................................................................................................................................

.................................................................................................................................... [2]

(b) In the space below, draw a labelled diagram of the apparatus set up so that
electromagnetic induction may be demonstrated. [2]

(c) Describe how you would use the apparatus to demonstrate electromagnetic induction.

..........................................................................................................................................

..........................................................................................................................................

..........................................................................................................................................

.................................................................................................................................... [2]

(d) State two ways of increasing the magnitude of the induced e.m.f. in this experiment.

1. ......................................................................................................................................

..........................................................................................................................................

2. ......................................................................................................................................

.................................................................................................................................... [2]

[Total: 8]
© UCLES 2007 0625/03/O/N/07
13 For
Examiner’s
Use
10 (a) Fig. 10.1 shows an AND gate with two inputs A and B and one output.

A
output

Fig. 10.1

State the output when

(i) A is high and B is low,

............................................................................................................................ [1]

(ii) both A and B are low.

............................................................................................................................ [1]

(b) An electrical thermometer in a greenhouse gives a low output if the temperature is too
low.

A humidity sensor in the same greenhouse gives a high output if the humidity in the
greenhouse is too high.

An alarm sounds when both the temperature is too low and the humidity is too high.

(i) Complete the diagram below to show how a NOT gate and an AND gate may be
used to provide the required output to the alarm. [2]

electrical thermometer

alarm

humidity sensor

(ii) On your diagram, use either ‘high’ or ‘low’ to indicate the level of the inputs and
outputs of both gates when the alarm sounds. [2]

[Total: 6]

© UCLES 2007 0625/03/O/N/07 [Turn over


14 For
Examiner’s
Use
11 Fig. 11.1 shows an experiment to test the absorption of β-particles by thin sheets of
aluminium. Ten sheets are available, each 0.5 mm thick.

β-particle source

detector counter
sheets of
aluminium

Fig. 11.1

(a) Describe how the experiment is carried out, stating the readings that should be taken.

..........................................................................................................................................

..........................................................................................................................................

..........................................................................................................................................

..........................................................................................................................................

.................................................................................................................................... [4]

(b) State the results that you would expect to obtain.

..........................................................................................................................................

..........................................................................................................................................

..........................................................................................................................................

.................................................................................................................................... [2]

[Total: 6]

© UCLES 2007 0625/03/O/N/07


15

BLANK PAGE

0625/03/O/N/07
16

BLANK PAGE

Permission to reproduce items where third-party owned material protected by copyright is included has been sought and cleared where possible. Every
reasonable effort has been made by the publisher (UCLES) to trace copyright holders, but if any items requiring clearance have unwittingly been included, the
publisher will be pleased to make amends at the earliest possible opportunity.

University of Cambridge International Examinations is part of the Cambridge Assessment Group. Cambridge Assessment is the brand name of University of
Cambridge Local Examinations Syndicate (UCLES), which is itself a department of the University of Cambridge.

0625/03/O/N/07
First Variant Question Paper

UNIVERSITY OF CAMBRIDGE INTERNATIONAL EXAMINATIONS


International General Certificate of Secondary Education
*5079794953*

PHYSICS 0625/31
Paper 3 Extended May/June 2008
1 hour 15 minutes
Candidates answer on the Question Paper.
No Additional Materials are required.

READ THESE INSTRUCTIONS FIRST

Write your Centre number, candidate number and name on all the work you hand in.
Write in dark blue or black pen.
You may use a soft pencil for any diagrams, graphs or rough working.
Do not use staples, paper clips, highlighters, glue or correction fluid.
DO NOT WRITE IN ANY BARCODES.

Answer all questions.


You may lose marks if you do not show your working or if you do not use
appropriate units.
Take the weight of 1 kg to be 10 N (i.e. acceleration of free fall = 10 m/s2).

At the end of the examination, fasten all your work securely together.
The number of marks is given in brackets [ ] at the end of each question or
part question.

This document consists of 15 printed pages and 1 blank page.

SPA (SHW 00013 3/07) T49721/6


© UCLES 2008 [Turn over
2

1 Fig. 1.1 shows the speed-time graphs for two falling balls. For
Examiner’s
Use
30
rubber ball
speed
m/s
20

plastic ball
10

0
0 1 2 3 4 5
time / s

Fig. 1.1

Both balls fall from the same height above the ground.

(a) Use the graphs to find

(i) the average acceleration of the falling rubber ball during the first 3.0 s,

acceleration = ................................................ [2]

(ii) the distance fallen by the rubber ball during the first 3.0 s,

distance = ................................................ [2]

(iii) the terminal velocity of the plastic ball.

terminal velocity = ................................................ [1]

© UCLES 2008 0625/31/M/J/08


3

(b) Both balls have the same mass but the volume of the plastic ball is much greater than For
that of the rubber ball. Explain, in terms of the forces acting on each ball, why the plastic Examiner’s
ball reaches a terminal velocity but the rubber ball does not. Use

..........................................................................................................................................

..........................................................................................................................................

..........................................................................................................................................

..........................................................................................................................................

..........................................................................................................................................

.................................................................................................................................... [3]

(c) The rubber ball has a mass of 50 g. Calculate the gravitational force acting on the rubber
ball.

force = ................................................ [2]

[Total: 10]

© UCLES 2008 0625/31/M/J/08 [Turn over


4

2 (a) Name the process by which energy is released in the core of the Sun. For
Examiner’s
.................................................................................................................................... [1] Use

(b) Describe how energy from the Sun becomes stored energy in water behind a dam.

..........................................................................................................................................

..........................................................................................................................................

..........................................................................................................................................

.................................................................................................................................... [3]

(c) Data for two small power stations is given in Table 2.1.

input to power station output of power station

gas-fired 100 MW 25 MW

hydroelectric 90 MW 30 MW

Table 2.1

(i) State what is meant by the efficiency of a power station.

..................................................................................................................................

..................................................................................................................................

..................................................................................................................................

............................................................................................................................ [1]

(ii) Use the data in Table 2.1 to explain that the hydroelectric station is more efficient
than the gas-fired power station.

..................................................................................................................................

..................................................................................................................................

............................................................................................................................ [1]

[Total: 6]

© UCLES 2008 0625/31/M/J/08


5

3 A cyclist rides up and then back down the hill shown in Fig. 3.1. For
Examiner’s
Use
top of hill

14 m

starting and
finishing point

Fig. 3.1

The cyclist and her bicycle have a combined mass of 90 kg. She pedals up to the top and
then stops. She turns around and rides back to the bottom without pedalling or using her
brakes.

(a) Calculate the potential energy gained by the cyclist and her bicycle when she has
reached the top of the hill.

potential energy = ................................................ [2]

(b) Calculate the maximum speed she could have when she arrives back at the starting
point.

speed = ................................................ [3]

(c) Explain why her actual speed will be less than that calculated in (b).

..........................................................................................................................................

..........................................................................................................................................

.................................................................................................................................... [1]

[Total: 6]

© UCLES 2008 0625/31/M/J/08 [Turn over


6

4 Fig. 4.1 is a design for remotely operating an electrical switch using air pressure. For
Examiner’s
Use
electrical switch flexible rubber
operated by box cover
air pressure connecting pipe

metal
box

Fig. 4.1

The metal box and the pipe contain air at normal atmospheric pressure and the switch is off.
When the pressure in the metal box and pipe is raised to 1.5 times atmospheric pressure by
pressing down on the flexible rubber box cover, the switch comes on.

(a) Explain in terms of pressure and volume how the switch is made to come on.

..........................................................................................................................................

..........................................................................................................................................

..........................................................................................................................................

.................................................................................................................................... [2]

(b) Normal atmospheric pressure is 1.0 × 105 Pa. At this pressure, the volume of the box
and pipe is 60 cm3.

Calculate the reduction in volume that must occur for the switch to be on.

reduction in volume = ................................................ [3]

(c) Explain, in terms of air particles, why the switch may operate, without the rubber cover
being squashed, when there is a large rise in temperature.

..........................................................................................................................................

..........................................................................................................................................

..........................................................................................................................................

.................................................................................................................................... [2]

[Total: 7]

© UCLES 2008 0625/31/M/J/08


7

5 (a) Explain, in terms of molecules, how thermal expansion takes place in a solid and in a For
gas. Examiner’s
Use

solid .................................................................................................................................

..........................................................................................................................................

..........................................................................................................................................

..........................................................................................................................................

..........................................................................................................................................

gas ...................................................................................................................................

..........................................................................................................................................

..........................................................................................................................................

..........................................................................................................................................

.................................................................................................................................... [4]

(b) Complete Table 5.1 to show the relative expansion of equal volumes of liquids, gases
and solids.

Choose words from

much less, slightly less, slightly more and much more. [2]

state of matter expansion compared to solids, for the


same temperature rise

liquids

gases

Table 5.1

(c) Alcohol is often used in thermometers.

State one property of alcohol that makes it suitable for use in thermometers.

..........................................................................................................................................

.................................................................................................................................... [1]

[Total: 7]

© UCLES 2008 0625/31/M/J/08 [Turn over


8

6 Fig. 6.1 shows an object, the tip of which is labelled O, placed near a lens L. For
Examiner’s
The two principal foci of the lens are F1 and F2. Use

L O

F2 F1

Fig. 6.1

(a) On Fig. 6.1, draw the paths of two rays from the tip of the object so that they pass
through the lens and continue beyond.

Complete the diagram to locate the image of the tip of the object. Draw in the whole
image and label it I. [3]

(b) Describe image I.

..........................................................................................................................................

..........................................................................................................................................

..........................................................................................................................................

.................................................................................................................................... [3]

[Total: 6]

© UCLES 2008 0625/31/M/J/08


9

7 Fig. 7.1 and Fig. 7.2 show wavefronts of light approaching a plane mirror and a rectangular For
glass block, respectively. Examiner’s
Use

direction of travel
of wavefronts

mirror

Fig. 7.1

direction of travel
of wavefronts

glass block

Fig. 7.2

(a) On Fig. 7.1 and on Fig. 7.2 draw wavefronts to show what happens after the waves
strike the surface. [4]

(b) In Fig. 7.2, the waves approaching the block have a speed of 3.0 × 108 m/s and an angle
of incidence of 70°. The refractive index of the glass of the block is 1.5.

(i) Calculate the speed of light waves in the block.

speed = ................................................ [2]

(ii) Calculate the angle of refraction in the block.

angle = ................................................ [2]

[Total: 8]
© UCLES 2008 0625/31/M/J/08 [Turn over
10

8 Fig. 8.1 is the plan of a small apartment that has four lamps as shown. For
Examiner’s
Use

100 W
kitchen
2 × 60 W
living room
60 W
bathroom

Fig. 8.1

Power for the lamps is supplied at 200 V a.c. and the lamps are all in parallel.

(a) In the space below, draw a lighting circuit diagram so that there is one switch for each
room and one master switch that will turn off all the lamps. Label the lamps as 60 W or
100 W.

[3]

(b) The 100 W lamp is switched on. Calculate

(i) the current in the lamp,

current = ................................................ [2]

(ii) the charge passing through the lamp in one minute.

charge = ................................................ [2]

© UCLES 2008 0625/31/M/J/08


11

(c) The three 60 W lamps are replaced by three energy-saving ones, that give the same For
light output but are rated at only 15 W each. Examiner’s
Use

Calculate

(i) the total reduction in power,

reduction in power = ................................................ [1]

(ii) the energy saved when the lamps are lit for one hour.

energy saved = ................................................. [2]

[Total: 10]

© UCLES 2008 0625/31/M/J/08 [Turn over


12

9 Fig. 9.1 shows apparatus used to investigate electromagnetic effects around straight wires. For
Examiner’s
Use
T3
thin flexible wire T1 thick rigid
vertical wire

large circular
hole in card

small circular
hole in card

T4
T2

Fig. 9.1

Fig. 9.2 is a view looking down on the apparatus shown in Fig. 9.1.

Fig. 9.2

(a) A battery is connected to T1 and T2 so that there is a current vertically down the thick
wire.

On Fig. 9.2, draw three magnetic field lines and indicate, with arrows, the direction of all
three. [2]

(b) Using a variable resistor, the p.d. between terminals T1 and T2 is gradually reduced.

State the effect, if any, that this will have on

(i) the strength of the magnetic field, ...................................................................... [1]

(ii) the direction of the magnetic field. ...................................................................... [1]

© UCLES 2008 0625/31/M/J/08


13

(c) The battery is now connected to terminals T3 and T4, as well as to terminals T1 and T2, For
so that there is a current down both wires. This causes the flexible wire to move. Examiner’s
Use

(i) Explain why the flexible wire moves.

..................................................................................................................................

..................................................................................................................................

..................................................................................................................................

............................................................................................................................ [2]

(ii) State the direction of the movement of the flexible wire.

............................................................................................................................ [1]

(iii) The battery is replaced by one that delivers a smaller current.

State the effect that this will have on the force acting on the flexible wire.

..................................................................................................................................

............................................................................................................................ [1]

[Total: 8]

© UCLES 2008 0625/31/M/J/08 [Turn over


14

10 (a) In the space below, draw the symbol for a NOR gate. For
Examiner’s
Use

[1]

(b) Describe the action of a NOR gate in terms of its inputs and output.

..........................................................................................................................................

..........................................................................................................................................

..........................................................................................................................................

..........................................................................................................................................

.................................................................................................................................... [2]

(c) A chemical process requires heating at low pressure to work correctly.

When the heater is working, the output of a temperature sensor is high.

When the pressure is low enough, a pressure sensor has a low output.

Both outputs are fed into a NOR gate. A high output from the gate switches on an
indicator lamp.

(i) Explain why the indicator lamp is off when the process is working correctly.

..................................................................................................................................

..................................................................................................................................

............................................................................................................................ [1]

(ii) State whether the lamp is on or off in the following situations.

1. The pressure is low enough, but the heater stops working. .............................

2. The heater is working, but the pressure rises too high. .............................. [2]

[Total: 6]

© UCLES 2008 0625/31/M/J/08


15

11 (a) Chlorine has two isotopes, one of nucleon number 35 and one of nucleon number 37. For
The proton number of chlorine is 17. Examiner’s
Use

Table 11.1 refers to neutral atoms of chlorine.

Complete Table 11.1.

nucleon number 35 nucleon number 37

number of protons

number of neutrons

number of electrons

[3]
Table 11.1

(b) Some isotopes are radioactive.

State the three types of radiation that may be emitted from radioactive isotopes.

1. .......................................................

2. .......................................................

3. ....................................................... [1]

(c) (i) State one practical use of a radioactive isotope.

..................................................................................................................................

............................................................................................................................ [1]

(ii) Outline how it is used.

..................................................................................................................................

..................................................................................................................................

..................................................................................................................................

..................................................................................................................................

............................................................................................................................ [1]

[Total: 6]

© UCLES 2008 0625/31/M/J/08


16

BLANK PAGE

Permission to reproduce items where third-party owned material protected by copyright is included has been sought and cleared where possible. Every
reasonable effort has been made by the publisher (UCLES) to trace copyright holders, but if any items requiring clearance have unwittingly been included, the
publisher will be pleased to make amends at the earliest possible opportunity.

University of Cambridge International Examinations is part of the Cambridge Assessment Group. Cambridge Assessment is the brand name of University of
Cambridge Local Examinations Syndicate (UCLES), which is itself a department of the University of Cambridge.

0625/31/M/J/08
First Variant Question Paper

UNIVERSITY OF CAMBRIDGE INTERNATIONAL EXAMINATIONS


International General Certificate of Secondary Education
*0631463082*

PHYSICS 0625/31
Paper 3 Extended October/November 2008
1 hour 15 minutes
Candidates answer on the Question Paper.
No Additional Materials are required.

READ THESE INSTRUCTIONS FIRST

Write your Centre number, candidate number and name on all the work you hand in.
Write in dark blue or black pen.
You may use a soft pencil for any diagrams, graphs or rough working.
Do not use staples, paper clips, highlighters, glue or correction fluid.
DO NOT WRITE IN ANY BARCODES.

Answer all questions.


You may lose marks if you do not show your working or if you do not use appropriate units.
Take the weight of 1 kg to be 10 N (i.e. acceleration of free fall = 10 m/s2).

At the end of the examination, fasten all your work securely together.
The number of marks is given in brackets [ ] at the end of each question or part question.

This document consists of 16 printed pages.

SPA (SHW 00014 3/07) T50468/5


© UCLES 2008 [Turn over
2

1 Fig. 1.1 shows apparatus used to find a relationship between the force applied to a trolley For
and the acceleration caused by the force. Examiner’s
Use

ticker- ticker-tape
trolley tape timer roll of tape
string

hanging mass runway

Fig. 1.1

For each mass, hung as shown, the acceleration of the trolley is determined from the tape.
Some of the results are given in the table below.

acceleration of the trolley


weight of the hanging mass / N
m/s2
0.20 0.25

0.40 0.50

0.70

0.80 1.0

(a) (i) Explain why the trolley accelerates.

..................................................................................................................................

............................................................................................................................ [2]

(ii) Suggest why the runway has a slight slope as shown.

..................................................................................................................................

............................................................................................................................ [1]

(b) Calculate the mass of the trolley, assuming that the accelerating force is equal to the
weight of the hanging mass.

mass = ................................................ [2]

© UCLES 2008 0625/31/O/N/08


3

(c) Calculate the value missing from the table. Show your working. For
Examiner’s
Use

value = ................................................ [2]

(d) In one experiment, the hanging mass has a weight of 0.4 N and the trolley starts from
rest.

Use data from the table to calculate

(i) the speed of the trolley after 1.2 s,

speed = ................................................ [2]

(ii) the distance travelled by the trolley in 1.2 s.

distance = ................................................ [2]

[Total: 11]

© UCLES 2008 0625/31/O/N/08 [Turn over


4

2 Fig. 2.1 shows a circular metal disc of mass 200 g, freely pivoted at its centre. For
Examiner’s
Use

pivot

Fig. 2.1

Masses of 100 g, 200 g, 300 g, 400 g, 500 g and 600 g are available, but only one of each
value. These may be hung with string from any of the holes. There are three small holes on
each side of the centre, one at 4.0 cm from the pivot, one at 8.0 cm from the pivot and one at
12.0 cm from the pivot.

The apparatus is to be used to show that there is no net moment of force acting on a body
when it is in equilibrium.

(a) On Fig. 2.1, draw in two different value masses hanging from appropriate holes. The
values of the masses should be chosen so that there is no net moment. Alongside the
masses chosen, write down their values. [2]

(b) Explain how you would test that your chosen masses give no net moment to the disc.

..........................................................................................................................................

..........................................................................................................................................

..........................................................................................................................................

.................................................................................................................................... [1]

(c) Calculate the moments about the pivot due to the two masses chosen.

moment due to first mass = .....................................................

moment due to second mass = ......................................................


[2]

© UCLES 2008 0625/31/O/N/08


5

(d) Calculate the force on the pivot when the two masses chosen are hanging from the For
disc. Examiner’s
Use

force = ................................................ [2]

[Total: 7]

© UCLES 2008 0625/31/O/N/08 [Turn over


6

3 (a) A submarine descends to a depth of 70 m below the surface of water. For


Examiner’s
The density of the water is 1050 kg/m3. Atmospheric pressure is 1.0 × 105 Pa. Use

Calculate

(i) the increase in pressure as it descends from the surface to a depth of 70 m,

increase in pressure = ................................................ [2]

(ii) the total pressure on the submarine at a depth of 70 m.

total pressure = ................................................ [1]

(b) On another dive, the submarine experiences a total pressure of 6.5 × 105 Pa. A hatch
cover on the submarine has an area of 2.5 m2.

Calculate the force on the outside of the cover.

force = ................................................ [2]

(c) The submarine undergoes tests in fresh water of density 1000 kg/m3.

Explain why the pressure on the submarine is less at the same depth.

..........................................................................................................................................

.................................................................................................................................... [1]

[Total: 6]

© UCLES 2008 0625/31/O/N/08


7

4 The whole of a sealed, empty, dusty room is kept at a constant temperature of 15 °C. Light For
shines into the room through a small outside window. Examiner’s
Use
An observer points a TV camera with a magnifying lens into the room through a second
small window, set in an inside wall at right angles to the outside wall.

Dust particles in the room show up on the TV monitor screen as tiny specks of light.

(a) In the space below draw a diagram to show the motion of one of the specks of light over
a short period of time.

[1]

(b) After a period of one hour the specks are still observed, showing that the dust particles
have not fallen to the floor.

Explain why the dust particles have not fallen to the floor. You may draw a labelled
diagram to help your explanation.

..........................................................................................................................................

..........................................................................................................................................

..........................................................................................................................................

.................................................................................................................................... [2]

(c) On another day, the temperature of the room is only 5 °C. All other conditions are the
same and the specks of light are again observed.

Suggest any differences that you would expect in the movement of the specks when the
temperature is 5 °C, compared to before.

..........................................................................................................................................

..........................................................................................................................................

.................................................................................................................................... [1]

[Total: 4]

© UCLES 2008 0625/31/O/N/08 [Turn over


8

5 Fig. 5.1 shows apparatus that could be used to determine the specific latent heat of fusion of For
ice. Examiner’s
Use

finely crushed ice

40 W electric heater glass funnel

stand with clamps to hold


funnel and heater

Fig. 5.1

(a) In order to obtain as accurate a result as possible, state why it is necessary to

(i) wait until water is dripping into the beaker at a constant rate before taking
readings,

..................................................................................................................................

............................................................................................................................ [1]

(ii) use finely crushed ice rather than large pieces.

..................................................................................................................................

............................................................................................................................ [1]

(b) The power of the heater and the time for which water is collected are known. Write down
all the other readings that are needed to obtain a value for the specific latent heat of
fusion of ice.

..........................................................................................................................................

.................................................................................................................................... [2]

© UCLES 2008 0625/31/O/N/08


9

(c) Using a 40 W heater, 16.3 g of ice is melted in 2.0 minutes. The heater is then switched For
off. In a further 2.0 minutes, 2.1 g of ice is melted. Examiner’s
Use

Calculate the value of the specific latent heat of fusion of ice from these results.

specific latent heat of fusion of ice = ................................................ [4]

[Total: 8]

© UCLES 2008 0625/31/O/N/08 [Turn over


10

6 Fig. 6.1 shows two rays of monochromatic light, one entering the prism along the normal DE For
and the second one along PQ. Examiner’s
Use

E
F
P 30°

R
42°

A C

Fig. 6.1

(a) State what is meant by monochromatic light.

.................................................................................................................................... [1]

(b) The refractive index of the glass of the prism is 1.49. The ray EF is refracted at F. Use
information from Fig. 6.1 to calculate the angle of refraction at F.

angle of refraction = ................................................ [3]

(c) On Fig. 6.1, draw in the refracted ray, starting from F. [1]

(d) State how the refraction, starting at F, would be different if the monochromatic ray were
replaced by a ray of white light.

.................................................................................................................................... [1]

(e) The critical angle for the glass of the prism is just over 42°. State the approximate angle
of refraction for the ray striking BC at R.

.................................................................................................................................... [1]

(f) Another monochromatic ray, not shown in Fig. 6.1, passes through the prism and strikes
BC at an angle of incidence of 50°. State what happens to this ray at the point where it
strikes BC.

.................................................................................................................................... [1]

[Total: 8]
© UCLES 2008 0625/31/O/N/08
11

7 Fig. 7.1 shows a scale drawing of plane waves approaching a gap in a barrier. For
Examiner’s
Use
direction of travel
of plane waves

barrier

Fig. 7.1

(a) On Fig. 7.1, draw in the pattern of the waves after they have passed the gap. [3]

(b) The waves approaching the barrier have a wavelength of 2.5 cm and a speed of 20 cm/s.
Calculate the frequency of the waves.

frequency = ................................................ [2]

(c) State the frequency of the diffracted waves.

.................................................................................................................................... [1]

[Total: 6]

© UCLES 2008 0625/31/O/N/08 [Turn over


12

8 Fig. 8.1 shows a car battery being charged from a 200 V a.c. mains supply. For
Examiner’s
Use

car
200 V ~ battery

Fig. 8.1

(a) State the function of the diode.

..........................................................................................................................................

.................................................................................................................................... [1]

(b) The average charging current is 2.0 A and the battery takes 12 hours to charge fully.

Calculate the charge that the battery stores when fully charged.

charge stored ................................................ [2]

(c) The battery has an electromotive force (e.m.f.) of 12 V and, when connected to a circuit,
supplies energy to the circuit components.

State what is meant by an electromotive force of 12 V.

..........................................................................................................................................

..........................................................................................................................................

.................................................................................................................................... [2]

© UCLES 2008 0625/31/O/N/08


13

(d) (i) In the space below, draw a circuit diagram to show how two 6.0 V lamps should be For
connected to a 12 V battery so that both lamps glow with normal brightness. [1] Examiner’s
Use

(ii) The power of each lamp is 8.0 W. Calculate the current in the circuit.

current = ................................................. [2]

(iii) Calculate the energy used by the two lamps when both are lit for one hour.

energy = ................................................ [2]

[Total: 10]

© UCLES 2008 0625/31/O/N/08 [Turn over


14

9 Fig. 9.1 is a block diagram of an electrical energy supply system, using the output of a coal- For
fired power station. Examiner’s
Use

transmission
power output step-up output step-down output
consumer
station at 1100 V transformer at 32 000 V transformer at 240 V

Fig. 9.1

(a) Suggest one possible way of storing surplus energy when the demand from the
consumers falls below the output of the power station.

..........................................................................................................................................

.................................................................................................................................... [1]

(b) State why electrical energy is transmitted at high voltage.

.................................................................................................................................... [1]

(c) A transmission cable of resistance R carries a current I. Write down a formula that gives
the power loss in the cable in terms of R and I.

.................................................................................................................................... [1]

(d) The step-up transformer has 1200 turns on the primary coil. Using the values in Fig. 9.1,
calculate the number of turns on its secondary coil. Assume that the transformer has no
energy losses.

number of turns = ................................................. [2]

(e) The input to the step-up transformer is 800 kW.

Using the values in Fig. 9.1, calculate the current in the transmission cables, assuming
that the transformer is 100% efficient.

current = ................................................ [3]

[Total: 8]

© UCLES 2008 0625/31/O/N/08


15

10 Fig. 10.1 shows a circuit for a warning lamp that comes on when the external light intensity For
falls below a pre-set level. Examiner’s
Use

+
low voltage
supply

Fig. 10.1

(a) On Fig. 10.1, label

(i) with the letter X the component that detects the change in external light intensity,

(ii) with the letter Y the lamp,

(iii) with the letter Z the component that switches the lamp on and off.
[3]

(b) Describe how the circuit works as the external light intensity decreases and the lamp
comes on.

..........................................................................................................................................

..........................................................................................................................................

..........................................................................................................................................

..........................................................................................................................................

..........................................................................................................................................

..........................................................................................................................................

.................................................................................................................................... [3]

[Total: 6]

© UCLES 2008 0625/31/O/N/08 [Turn over


16

11 Fig. 11.1 shows the basic design of the tube of a cathode ray oscilloscope (CRO). For
Examiner’s
Use

heater filament B C

cathode rays

A anode D

Fig. 11.1

(a) On Fig. 11.1, write the names of parts A, B, C and D in the boxes provided. [2]

(b) State the function of:

part A, ..............................................................................................................................

..........................................................................................................................................

part B. ..............................................................................................................................

.................................................................................................................................... [2]

(c) A varying p.d. from a 12 V supply is connected to a CRO, so that the waveform of the
supply is shown on the screen.

To which of the components in Fig. 11.1

(i) is the 12 V supply connected,

............................................................................................................................ [1]

(ii) is the time-base connected?

............................................................................................................................ [1]

[Total: 6]

Permission to reproduce items where third-party owned material protected by copyright is included has been sought and cleared where possible.
Every reasonable effort has been made by the publisher (UCLES) to trace copyright holders, but if any items requiring clearance have unwittingly been
included, the publisher will be pleased to make amends at the earliest possible opportunity.

University of Cambridge International Examinations is part of the Cambridge Assessment Group. Cambridge Assessment is the brand name of University
of Cambridge Local Examinations Syndicate (UCLES), which is itself a department of the University of Cambridge.
© UCLES 2008 0625/31/O/N/08
UNIVERSITY OF CAMBRIDGE INTERNATIONAL EXAMINATIONS
International General Certificate of Secondary Education
*6024898353*

PHYSICS 0625/31
Paper 3 Extended October/November 2009
1 hour 15 minutes
Candidates answer on the Question Paper.
No Additional Materials are required.

READ THESE INSTRUCTIONS FIRST

Write your Centre number, candidate number and name on all the work you hand in.
Write in dark blue or black pen.
You may use a soft pencil for any diagrams, graphs or rough working.
Do not use staples, paper clips, highlighters, glue or correction fluid.
DO NOT WRITE IN ANY BARCODES.

Answer all questions.


You may lose marks if you do not show your working or if you do not use appropriate units.
Take the weight of 1 kg to be 10 N (i.e. acceleration of free fall = 10 m/s2).

At the end of the examination, fasten all your work securely together.
The number of marks is given in brackets [ ] at the end of each question or part question.

This document consists of 16 printed pages.

DC (SHW 00151 2/08) 14866/7


© UCLES 2009 [Turn over
2

1 Fig 1.1 shows part of a measuring instrument. For


Examiner’s
Use

0
mm 25
20

Fig. 1.1

(a) State the name of this instrument.

................................................. [1]

(b) Record the reading shown in Fig. 1.1.

................................................. [1]

(c) Describe how you would find the thickness of a sheet of paper used in a magazine.

..........................................................................................................................................

..........................................................................................................................................

..........................................................................................................................................

..........................................................................................................................................

..........................................................................................................................................

.................................................................................................................................... [3]

[Total: 5]

© UCLES 2009 0625/31/O/N/09


3

2 The list below gives the approximate densities of various metals. For
Examiner’s
gold 19 g / cm3 Use

lead 11 g / cm3

copper 9 g / cm3

iron 8 g / cm3

At an antiques market, a collector buys what is advertised as a small ancient gold statue.
When the collector tests it in the laboratory, he finds its mass is 600 g and its volume is
65 cm3.

(a) In the space below, describe how the volume of the statue could be measured. You may
draw diagrams if you wish.

[3]

(b) Use the figures given above to decide whether the statue was really made of gold. Show
your working.

Was the statue made of gold? (Tick one box.)

yes

no

[3]

[Total: 6]
© UCLES 2009 0625/31/O/N/09 [Turn over
4

3 A student investigated the stretching of a spring by hanging various weights from it and For
measuring the corresponding extensions. The results are shown below. Examiner’s
Use

weight / N 0 1 2 3 4 5

extension / mm 0 21 40 51 82 103

(a) On Fig. 3.1, plot the points from these results. Do not draw a line through the points yet.
[2]

120

100

extension / mm

80

60

40

20

0
0 1 2 3 4 5 6
weight / N

Fig. 3.1

© UCLES 2009 0625/31/O/N/09


5

(b) The student appears to have made an error in recording one of the results. For
Examiner’s
Which result is this? Use

.................................................................................................................................... [1]

(c) Ignoring the incorrect result, draw the best straight line through the remaining points.
[1]

(d) State and explain whether this spring is obeying Hooke’s Law.

..........................................................................................................................................

..........................................................................................................................................

..........................................................................................................................................

..........................................................................................................................................

.................................................................................................................................... [2]

(e) Describe how the graph might be shaped if the student continued to add several more
weights to the spring.

..........................................................................................................................................

..........................................................................................................................................

..........................................................................................................................................

.................................................................................................................................... [1]

(f) The student estimates that if he hangs a 45 N load on the spring, the extension will be
920 mm.

Explain why this estimate may be unrealistic.

..........................................................................................................................................

..........................................................................................................................................

..........................................................................................................................................

.................................................................................................................................... [1]

[Total: 8]

© UCLES 2009 0625/31/O/N/09 [Turn over


6

4 (a) A force acting on an object causes the object to accelerate. For


Examiner’s
In which direction is the acceleration? Use

................................................. [1]

(b) Any object moving in a circle has a force acting on it towards the centre of the circle.

What does this force do to the object?

.................................................................................................................................... [1]

(c) A woman of mass 60 kg is standing in a lift at a shopping centre.

(i) The lift is at rest.

1. State the value of the weight of the woman.

................................................. [1]

2. State the value of the force exerted on the woman by the floor of the lift.

............................................................................................................................ [1]

(ii) Calculate the force required to accelerate a mass of 60 kg at 2.5 m / s2.

force = ................................................ [2]

(iii) The lift accelerates upwards at 2.5 m / s2.

Calculate the force exerted on the woman by the floor when the lift is accelerating.

force = ................................................ [1]

(iv) The lift reaches a steady upward speed.

State the value of the force exerted on the woman by the floor at this steady
speed.

............................................................................................................................ [1]

[Total: 8]

© UCLES 2009 0625/31/O/N/09


7

5 A farmer uses an electric pump to raise water from a river in order to fill the irrigation channels For
that keep the soil in his fields moist. Examiner’s
Use

water pours electric


into channel pump
field

water rises 3m
up tube

irrigation
channel
river

Fig. 5.1

Every minute, the pump raises 12 kg of water through a vertical height of 3 m.

(a) Calculate the increase in the gravitational potential energy of 12 kg of water when it is
raised 3 m.

increase in gravitational potential energy = ................................................ [3]

(b) Calculate the useful power output of the pump as it raises the water.

power = ................................................ [3]

[Total: 6]

© UCLES 2009 0625/31/O/N/09 [Turn over


8

6 A vertical cylinder has a smooth well-fitting piston in it. Weights can be added to or removed For
from a tray on the top of the piston. Examiner’s
Use

(a) Weights are added to the tray, as shown in Fig. 6.1.

weights

piston

air

cylinder

Fig. 6.1

(i) State what happens to the pressure of the air in the cylinder as a result of adding
these weights.

............................................................................................................................ [1]

(ii) The initial pressure of the trapped air is 1.05 × 105 Pa. When the weights are added,
the volume of the air decreases from 860 cm3 to 645 cm3.

The temperature of the air does not change.

Calculate the final pressure of the trapped air.

pressure = ................................................ [3]

(iii) The area of the piston is 5.0 × 10–3 m2.

Calculate the weight that is added to the piston.

weight added = ................................................ [4]

© UCLES 2009 0625/31/O/N/09


9

(b) The weights are kept as shown in Fig. 6.1. The temperature of the air in the cylinder is For
increased. Examiner’s
Use

(i) State what happens to the volume of the air in the cylinder as a result of this
temperature rise.

............................................................................................................................ [1]

(ii) State how, if at all, the pressure of the air changes as the temperature changes.

............................................................................................................................ [1]

(iii) State what must be done to prevent the volume change in (b)(i).

............................................................................................................................ [1]

(iv) The volume change in (b)(i) is prevented. State what happens to the pressure of
the air in the cylinder.

............................................................................................................................ [1]

[Total: 12]

© UCLES 2009 0625/31/O/N/09 [Turn over


10

7 Three wires and a meter are used to construct a thermocouple for measuring the surface For
temperature of a pipe carrying hot liquid, as shown in Fig. 7.1. Examiner’s
Use

meter

wire 1 wire 2

cold junction

wire 3
hot junction

hot liquid in pipe

Fig. 7.1

(a) Copper wire and constantan wire are used in the construction of the thermocouple.

State which metal might be used for

wire 1 ......................................................

wire 2 ......................................................

wire 3 ......................................................
[1]

(b) State what type of meter is used.

.................................................................................................................................... [1]

(c) State one particular advantage of thermocouples for measuring temperature.

.................................................................................................................................... [1]

[Total: 3]

© UCLES 2009 0625/31/O/N/09


11

8 Fig. 8.1 shows a thin converging lens. The two principal foci are shown. For
Examiner’s
Use

principal
F2 F1 axis

Fig. 8.1

A vertical object, 2 cm tall, is to be positioned to the left of the lens, with one end on the
principal axis.

On Fig. 8.1,

(a) draw the object in a position which will produce a virtual image, labelling the object with
the letter O, [1]

(b) draw two rays showing how the virtual image is formed, [2]

(c) draw in the image, labelling it with the letter I. [1]

[Total: 4]

© UCLES 2009 0625/31/O/N/09 [Turn over


12

9 (a) State what is meant by specific heat capacity. For


Examiner’s
.......................................................................................................................................... Use

.................................................................................................................................... [2]

(b) Water has a very high specific heat capacity.

Suggest why this might be a disadvantage when using water for cooking.

..........................................................................................................................................

.................................................................................................................................... [1]

(c) Fig. 9.1 illustrates an experiment to measure the specific heat capacity of some metal.

stirrer
thermometer

lid

thread
cup
boiling water
insulation
metal water

heater

Fig. 9.1

The piece of metal is heated in boiling water until it has reached the temperature of
the water. It is then transferred rapidly to some water in a well-insulated cup. A very
sensitive thermometer is used to measure the initial and final temperatures of the water
in the cup.

specific heat capacity of water = 4200 J / (kg K)

The readings from the experiment are as follows.

mass of metal = 0.050 kg


mass of water in cup = 0.200 kg
initial temperature of water in cup = 21.1 °C
final temperature of water in cup = 22.9 °C

(i) Calculate the temperature rise of the water in the cup and the temperature fall of
the piece of metal.

temperature rise of water = ......................................................

temperature fall of metal = ......................................................


[1]

© UCLES 2009 0625/31/O/N/09


13

(ii) Calculate the thermal energy gained by the water in the cup. State the equation For
that you use. Examiner’s
Use

thermal energy gained = ................................................ [3]

(iii) Assume that only the water gained thermal energy from the piece of metal.

Making use of your answers to (c)(i) and (c)(ii), calculate the value of the specific
heat capacity of the metal. Give your answer to 3 significant figures.

specific heat capacity = ................................................ [2]

(iv) Suggest one reason why the experiment might not have given a correct value for
the specific heat capacity of the metal.

..................................................................................................................................

............................................................................................................................ [1]

[Total: 10]

© UCLES 2009 0625/31/O/N/09 [Turn over


14

10 Alternating current electricity is delivered at 22 000 V to a pair of transmission lines. For


The transmission lines carry the electricity to the customer at the receiving end, where Examiner’s
the potential difference is V. This is shown in Fig. 10.1. Each transmission line has a Use

resistance of 3 Ω.

22 000 V 3Ω
V

Fig. 10.1

(a) The a.c. generator actually generates at a much lower voltage than 22 000 V.

(i) Suggest how the voltage is increased to 22 000 V.

............................................................................................................................ [1]

(ii) State one advantage of delivering electrical energy at high voltage.

............................................................................................................................ [1]

(b) The power delivered by the generator is 55 kW. Calculate the current in the transmission
lines.

current = ................................................ [2]

(c) Calculate the rate of loss of energy from one of the 3 Ω transmission lines.

rate of energy loss = ................................................ [2]

© UCLES 2009 0625/31/O/N/09


15

(d) Calculate the voltage drop across one of the transmission lines. For
Examiner’s
Use

voltage drop = ................................................ [2]

(e) Calculate the potential difference V at the receiving end of the transmission lines.

V = ................................................ [2]

[Total: 10]

Question 11 is on the next page.

© UCLES 2009 0625/31/O/N/09 [Turn over


16

11 Fig. 11.1 is a schematic diagram of an electronic circuit controlling a lamp. For


Examiner’s
Use
temperature
sensor

relay lamp

B
A
light
sensor

Fig. 11.1

(a) State the names of the logic gates A and B.

A ........................................................ B ........................................................ [2]

(b) The output of the temperature sensor is high (logic 1) when it detects raised temperature.
The output of the light sensor is high (logic 1) when it detects raised light levels.

State the outputs of A and B when the surroundings are

(i) dark and cold, output of A = ....................................

output of B = .................................... [1]

(ii) dark and warm, output of A = ....................................

output of B = .................................... [1]

(iii) bright and warm. output of A = ....................................

output of B = .................................... [1]

(c) (i) Suggest why B is connected to a relay, rather than directly to the lamp.

............................................................................................................................ [1]

(ii) The relay switches on when its input is high. In which of the three combinations in
(b) will the lamp light up?

............................................................................................................................ [1]

(iii) Suggest a practical use for this circuit.

............................................................................................................................ [1]

[Total: 8]
Permission to reproduce items where third-party owned material protected by copyright is included has been sought and cleared where possible. Every
reasonable effort has been made by the publisher (UCLES) to trace copyright holders, but if any items requiring clearance have unwittingly been included, the
publisher will be pleased to make amends at the earliest possible opportunity.

University of Cambridge International Examinations is part of the Cambridge Assessment Group. Cambridge Assessment is the brand name of University of
Cambridge Local Examinations Syndicate (UCLES), which is itself a department of the University of Cambridge.
© UCLES 2009 0625/31/O/N/09
UNIVERSITY OF CAMBRIDGE INTERNATIONAL EXAMINATIONS
International General Certificate of Secondary Education
*7776753992*

PHYSICS 0625/31
Paper 3 Extended May/June 2010
1 hour 15 minutes
Candidates answer on the Question Paper.
No Additional Materials are required.

READ THESE INSTRUCTIONS FIRST

Write your Centre number, candidate number and name on all the work you hand in.
Write in dark blue or black pen.
You may use a soft pencil for any diagrams, graphs or rough working.
Do not use staples, paper clips, highlighters, glue or correction fluid.
DO NOT WRITE IN ANY BARCODES.

Answer all questions.


You may lose marks if you do not show your working or if you do not use appropriate units.
Take the weight of 1 kg to be 10 N (i.e. acceleration of free fall = 10 m / s2).

At the end of the examination, fasten all your work securely together.
The number of marks is given in brackets [ ] at the end of each question or part question.

This document consists of 19 printed pages and 1 blank page.

DC (SHW 00380 1/09) 20284/4


© UCLES 2010 [Turn over
2

1 Fig. 1.1 shows the speed/time graph for a car travelling along a straight road. For
Examiner’s
The graph shows how the speed of the car changes as the car passes through a small Use

town.

35

D
30
speed
m/s
A
25

enters leaves
20 town town
here here

15

B C

10

0
0 10 20 30 40 50 60 70
time / s

Fig. 1.1

(a) Describe what happens to the speed of the car

(i) between A and B, .....................................................................................................

(ii) between B and C, .....................................................................................................

(iii) between C and D. .....................................................................................................


[1]

© UCLES 2010 0625/31/M/J/10


3

(b) Calculate the distance between the start of the town and the end of the town. For
Examiner’s
Use

distance = ................................................ [3]

(c) Calculate the acceleration of the car between C and D.

acceleration = ................................................ [3]

(d) State how the graph shows that the deceleration of the car has the same numerical
value as its acceleration.

..........................................................................................................................................

.................................................................................................................................... [1]

[Total: 8]

© UCLES 2010 0625/31/M/J/10 [Turn over


4

2 A car of mass 900 kg is travelling at a steady speed of 30 m / s against a resistive force of For
2000 N, as illustrated in Fig. 2.1. Examiner’s
Use

30 m / s

2000 N
resistive
force

Fig. 2.1

(a) Calculate the kinetic energy of the car.

kinetic energy = ................................................ [2]

(b) Calculate the energy used in 1.0 s against the resistive force.

energy = ................................................ [2]

(c) What is the minimum power that the car engine has to deliver to the wheels?

minimum power = ................................................ [1]

© UCLES 2010 0625/31/M/J/10


5

(d) What form of energy is in the fuel, used by the engine to drive the car? For
Examiner’s
.................................................................................................................................... [1] Use

(e) State why the energy in the fuel is converted at a greater rate than you have calculated
in (c).

..........................................................................................................................................

.................................................................................................................................... [1]

[Total: 7]

© UCLES 2010 0625/31/M/J/10 [Turn over


6

3 Two students make the statements about acceleration that are given below. For
Examiner’s
Student A: For a given mass the acceleration of an object is proportional to the resultant Use

force applied to the object.

Student B: For a given force the acceleration of an object is proportional to the mass of
the object.

(a) One statement is correct and one is incorrect.

Re-write the incorrect statement, making changes so that it is now correct.

For a given ..................... the acceleration of an object is ...............................................

.................................................................................................................................... [1]

(b) State the equation which links acceleration a, resultant force F and mass m.

[1]

(c) Describe what happens to the motion of a moving object when

(i) there is no resultant force acting on it,

............................................................................................................................ [1]

(ii) a resultant force is applied to it in the opposite direction to the motion,

............................................................................................................................ [1]

(iii) a resultant force is applied to it in a perpendicular direction to the motion.

............................................................................................................................ [1]

[Total: 5]

© UCLES 2010 0625/31/M/J/10


7

4 (a) Four identical metal plates, at the same temperature, are laid side by side on the ground. For
The rays from the Sun fall on the plates. Examiner’s
Use

One plate has a matt black surface.

One plate has a shiny black surface.

One plate has a matt silver surface.

One plate has a shiny silver surface.

State which plate has the fastest-rising temperature when the sunlight first falls on the
plates.

.................................................................................................................................... [1]

(b) The apparatus shown in Fig. 4.1 is known as Leslie’s Differential Air Thermometer.

glass bulb
radiant heater
painted shiny
matt black glass bulb

air
T

liquid

Fig. 4.1

The heater is switched off. Tap T is opened so that the air on the two sides of T has the
same pressure. Tap T is then closed.

(i) The heater is switched on. On Fig. 4.1, mark clearly where the two liquid levels
might be a short time later. [1]

(ii) Explain your answer to (b)(i).

..................................................................................................................................

..................................................................................................................................

............................................................................................................................ [2]

[Total: 4]

© UCLES 2010 0625/31/M/J/10 [Turn over


8

5 A certain substance is in the solid state at a temperature of –36 °C. It is heated at a constant For
rate for 32 minutes. The record of its temperature is given in Fig. 5.1. Examiner’s
Use

time / min 0 1 2 6 10 14 18 22 24 26 28 30 32

temperature / °C –36 –16 –9 –9 –9 –9 32 75 101 121 121 121 121

Fig. 5.1

(a) State what is meant by the term latent heat.

..........................................................................................................................................

.................................................................................................................................... [2]

(b) State a time at which the energy is being supplied as latent heat of fusion.

.................................................................................................................................... [1]

(c) Explain the energy changes undergone by the molecules of a substance during the
period when latent heat of vaporisation is being supplied.

..........................................................................................................................................

..........................................................................................................................................

.................................................................................................................................... [2]

(d) (i) The rate of heating is 2.0 kW.

Calculate how much energy is supplied to the substance during the period
18 – 22 minutes.

energy supplied = ................................................ [2]

© UCLES 2010 0625/31/M/J/10


9

(ii) The specific heat capacity of the substance is 1760 J / (kg °C). For
Examiner’s
Use the information in the table for the period 18 – 22 minutes to calculate the Use

mass of the substance being heated.

mass heated = ................................................ [3]

[Total: 10]

© UCLES 2010 0625/31/M/J/10 [Turn over


10

6 Some plane waves travel on the surface of water in a tank. They pass from a region of deep For
water into a region of shallow water. Fig. 6.1 shows what the waves look like from above. Examiner’s
Use

boundary

waves move
this way

deep shallow
water water

Fig. 6.1

(a) State what happens at the boundary, if anything, to

(i) the frequency of the waves,

............................................................................................................................ [1]

(ii) the speed of the waves,

............................................................................................................................ [1]

(iii) the wavelength of the waves.

............................................................................................................................ [1]

(b) The waves have a speed of 0.12 m / s in the deep water. Wave crests are 0.08 m apart in
the deep water.

Calculate the frequency of the source producing the waves. State the equation that you
use.

frequency = ................................................ [3]

© UCLES 2010 0625/31/M/J/10


11

(c) Fig. 6.2 shows identical waves moving towards the boundary at an angle. For
Examiner’s
Use
boundary

A B

waves move
this way

deep shallow
water water

Fig. 6.2

On Fig. 6.2, draw carefully the remainder of waves A and B, plus the two previous waves
which reached the shallow water. You will need to use your ruler to do this. [3]

[Total: 9]

© UCLES 2010 0625/31/M/J/10 [Turn over


12

7 During a thunderstorm, thunder and lightning are produced at the same time. For
Examiner’s
(a) A person is some distance away from the storm. Use

Explain why the person sees the lightning before hearing the thunder.

..........................................................................................................................................

..........................................................................................................................................

.................................................................................................................................... [1]

(b) A scientist in a laboratory made the following measurements during a thunderstorm.

time from start of storm / minutes 0.0 2.0 4.0 6.0 8.0 10.0

time between seeing lightning and hearing thunder / s 3.6 2.4 1.6 2.4 3.5 4.4

Fig. 7.1

(i) How many minutes after the storm started did it reach its closest point to the
laboratory?

............................................................................................................................ [1]

(ii) How can you tell that the storm was never immediately over the laboratory?

............................................................................................................................ [1]

(iii) When the storm started, it was immediately above a village 1200 m from the
laboratory.

Using this information and information from Fig. 7.1, calculate the speed of sound.

speed of sound = ................................................ [2]

(iv) State the assumption you made when you calculated your answer to (b)(iii).

............................................................................................................................ [1]

© UCLES 2010 0625/31/M/J/10


13

(c) Some waves are longitudinal; some waves are transverse. For
Examiner’s
Some waves are electromagnetic; some waves are mechanical. Use

Put ticks (✓) in the table below to indicate which of these descriptions apply to the light
waves of the lightning and the sound waves of the thunder.

light waves sound waves

longitudinal

transverse

electromagnetic

mechanical

[3]

[Total: 9]

© UCLES 2010 0625/31/M/J/10 [Turn over


14

8 (a) The transformer in Fig. 8.1 is used to convert 240 V a.c. to 6 V a.c. For
Examiner’s
Use
iron core

A
C

D
B

primary secondary
coil coil
(480 turns)

Fig. 8.1

(i) Using the information above, calculate the number of turns on the secondary coil.

number of turns = ................................................ [2]

(ii) Describe how the transformer works.

..................................................................................................................................

..................................................................................................................................

..................................................................................................................................

..................................................................................................................................

............................................................................................................................ [3]

(iii) State one way in which energy is lost from the transformer, and from which part it is
lost.

............................................................................................................................ [1]

© UCLES 2010 0625/31/M/J/10


15

(b) Fig. 8.2 shows a device labelled “IGCSE Transformer”. For


Examiner’s
Use

Fig. 8.2

Study the label on the case of the IGCSE Transformer.

(i) What is the output of the device? ....................................................................... [1]

(ii) From the information on the case, deduce what other electrical component must be
included within the case of the IGCSE Transformer, apart from a transformer.

............................................................................................................................ [1]

(c) A transformer supplying electrical energy to a factory changes the 11 000 V a.c. supply to
440 V a.c. for use in the factory. The current in the secondary coil is 200 A.

Calculate the current in the primary coil, assuming no losses from the transformer.

current = ................................................ [2]

[Total: 10]

© UCLES 2010 0625/31/M/J/10 [Turn over


16

9 (a) Fig. 9.1 illustrates the left hand rule, which helps when describing the force on a current- For
carrying conductor in a magnetic field. Examiner’s
Use

thumb

motion / force

first finger

second finger

Fig. 9.1

One direction has been labelled for you.

In each of the other two boxes, write the name of the quantity that direction represents.
[1]

(b) Fig. 9.2 shows a simple d.c. motor connected to a battery and a switch.

N
S

switch

battery

Fig. 9.2

© UCLES 2010 0625/31/M/J/10


17

(i) On Fig. 9.2, write in each of the boxes the name of the part of the motor to which For
the arrow is pointing. [2] Examiner’s
Use

(ii) State which way the coil of the motor will rotate when the switch is closed, when
viewed from the position X.

............................................................................................................................ [1]

(iii) State two things which could be done to increase the speed of rotation of the coil.

1. ...............................................................................................................................

2. ......................................................................................................................... [2]

[Total: 6]

© UCLES 2010 0625/31/M/J/10 [Turn over


18

10 A certain element is known to exist as two different isotopes. For


Examiner’s
(a) State one thing that is the same for atoms of both isotopes. Use

.................................................................................................................................... [1]

(b) State one thing that is different between atoms of these two isotopes.

.................................................................................................................................... [1]

(c) An atom of one of these isotopes is unstable and decays into a different element by
emitting a -particle.

(i) State one thing about the atom that remains the same during this decay.

............................................................................................................................ [1]

(ii) State one thing about the atom that changes as a result of this decay.

............................................................................................................................ [1]

[Total: 4]

© UCLES 2010 0625/31/M/J/10


19

11 (a) A coil of wire is connected into a circuit containing a variable resistor and a battery. For
Examiner’s
The variable resistor is adjusted until the potential difference across the coil is 1.8 V. Use

In this condition, the current in the circuit is 0.45 A.

Calculate

(i) the resistance of the coil,

resistance = ................................................ [1]

(ii) the thermal energy released from this coil in 9 minutes.

energy released = ................................................ [3]

(b) The coil in part (a) is replaced by one made of wire which has half the diameter of that
in (a).

When the potential difference across the coil is again adjusted to 1.8 V, the current is
only 0.30 A.

Calculate how the length of wire in the second coil compares with the length of wire in
the first coil.

length of wire in second coil is ………………………… the length of wire in first coil [4]

[Total: 8]
© UCLES 2010 0625/31/M/J/10
20

BLANK PAGE

Permission to reproduce items where third-party owned material protected by copyright is included has been sought and cleared where possible. Every
reasonable effort has been made by the publisher (UCLES) to trace copyright holders, but if any items requiring clearance have unwittingly been included, the
publisher will be pleased to make amends at the earliest possible opportunity.

University of Cambridge International Examinations is part of the Cambridge Assessment Group. Cambridge Assessment is the brand name of University of
Cambridge Local Examinations Syndicate (UCLES), which is itself a department of the University of Cambridge.

© UCLES 2010 0625/31/M/J/10


w
w
w
.X
tr
me
eP
ap
UNIVERSITY OF CAMBRIDGE INTERNATIONAL EXAMINATIONS

er
s
International General Certificate of Secondary Education

.c
om
*3428909673*

PHYSICS 0625/31
Paper 3 Extended October/November 2010
1 hour 15 minutes
Candidates answer on the Question Paper.
No Additional Materials are required.

READ THESE INSTRUCTIONS FIRST

Write your Centre number, candidate number and name on all the work you hand in.
Write in dark blue or black pen.
You may use a pencil for any diagrams, graphs or rough working.
Do not use staples, paper clips, highlighters, glue or correction fluid.
DO NOT WRITE IN ANY BARCODES.

Answer all questions.


You may lose marks if you do not show your working or if you do not use appropriate units.
Take the weight of 1 kg to be 10 N (i.e. acceleration of free fall = 10 m / s2).

At the end of the examination, fasten all your work securely together.
The number of marks is given in brackets [ ] at the end of each question or part question.

This document consists of 17 printed pages and 3 blank pages.

DC (NF/SW) 24990/4
© UCLES 2010 [Turn over
2

1 An object of weight W is suspended by two ropes from a beam, as shown in Fig. 1.1.

86.6 N
30°
50.0 N 60°

Fig. 1.1

The tensions in the ropes are 50.0 N and 86.6 N, as shown.

(a) In the space below, draw a scale diagram to find the resultant of the two tensions.

Use a scale of 1.0 cm = 10 N.

Clearly label the resultant. [3]

© UCLES 2010 0625/31/O/N/10


3

(b) From your diagram, find the value of the resultant.

resultant = ......................................................... [1]

(c) State the direction in which the resultant is acting.

............................................................................................................................................. [1]

(d) State the value of W. W = ......................................................... [1]

[Total: 6]

2 A car travels around a circular track at constant speed.

(a) Why is it incorrect to describe the circular motion as having constant velocity?

............................................................................................................................................. [1]

(b) A force is required to maintain the circular motion.

(i) Explain why a force is required.

...........................................................................................................................................

...........................................................................................................................................

..................................................................................................................................... [2]

(ii) In which direction does this force act?

..................................................................................................................................... [1]

(iii) Suggest what provides this force.

..................................................................................................................................... [1]

[Total: 5]

© UCLES 2010 0625/31/O/N/10 [Turn over


4

3 Fig. 3.1 shows a hydraulic lift in a car repair workshop.

car support

hydraulic fluid

4 pistons, each
of area 0.02 m2

piston A, area 0.01 m2

Fig. 3.1

The hydraulic fluid transmits the pressure, caused by piston A, equally to each of the four pistons
holding up the car supports. The pressure throughout the fluid is the same.

A force of 1000 N on piston A is just enough to raise the car.

(a) Using values from Fig. 3.1, find

(i) the pressure caused by piston A on the fluid,

pressure = ......................................................... [2]

(ii) the total upward force caused by the fluid.

force = ......................................................... [3]


© UCLES 2010 0625/31/O/N/10
5

(b) The weight of each of the two car supports is 1000 N.

Calculate the mass of the car.

mass = ......................................................... [2]

[Total: 7]

© UCLES 2010 0625/31/O/N/10 [Turn over


6

4 A student in a laboratory uses the apparatus shown in Fig. 4.1 to determine the specific heat
capacity of aluminium.

to low voltage supply


and measuring instruments

thermometer

electrical heater

aluminium block

Fig. 4.1

The readings obtained in the experiment are given below.

mass of aluminium block = 0.930 kg


initial temperature of block = 13.1 °C
final temperature of block = 41.3 °C
electrical energy supplied = 23 800 J

(a) Define specific heat capacity.

...................................................................................................................................................

............................................................................................................................................. [2]

(b) Use the readings above to calculate the specific heat capacity of aluminium.

State the equation you use.

specific heat capacity = ......................................................... [3]

© UCLES 2010 0625/31/O/N/10


7

(c) Because the student knows it is good scientific practice to repeat readings, after a short time
he carries out the experiment again, supplying the same quantity of electrical energy.

This time the temperature readings are:

initial temperature of block = 41.0 °C


final temperature of block = 62.1 °C

(i) Use these figures to calculate a second value for the specific heat capacity of
aluminium.

specific heat capacity = ......................................................... [1]

(ii) The student did not make any mistakes when taking the readings.

Suggest why the second value for the specific heat capacity of the aluminium is greater
than the first.

...........................................................................................................................................

..................................................................................................................................... [2]

(d) Suggest two ways of improving the experiment in order to give as accurate a result as
possible.

1. ...............................................................................................................................................

...................................................................................................................................................

2. ...............................................................................................................................................

.............................................................................................................................................. [2]

[Total: 10]

© UCLES 2010 0625/31/O/N/10 [Turn over


8

5 Fig. 5.1 shows a model cable-car system. It is driven by an electric motor coupled to a gear
system.

gears

m
6.0

2.0 m electric
motor

model
smooth cable-car
pulley

Fig. 5.1

The model cable-car has a mass of 5.0 kg and is lifted from the bottom pulley to the top pulley in
40 s. It stops automatically at the top.

(a) Calculate

(i) the average speed of the cable-car,

average speed = .......................................................... [2]

(ii) the gravitational potential energy gained by the cable-car,

gravitational potential energy gained = .......................................................... [2]

© UCLES 2010 0625/31/O/N/10


9

(iii) the useful output power of the driving mechanism.

power = ......................................................... [2]

(b) How would the electrical power input to the motor compare with your answer to (a)(iii)?

............................................................................................................................................. [1]

[Total: 7]

© UCLES 2010 0625/31/O/N/10 [Turn over


10

6 Fig. 6.1 shows part of the path of a ray of light PQ travelling in an optical fibre.

glass

Fig. 6.1

PQ undergoes total internal reflection at Q.

(a) Explain what is meant by total internal reflection, and state the conditions under which it
occurs.

...................................................................................................................................................

...................................................................................................................................................

...................................................................................................................................................

...................................................................................................................................................

............................................................................................................................................. [3]

(b) Carefully complete the path of the ray of light, until it reaches the end R of the optical fibre.
[2]

[Total: 5]

© UCLES 2010 0625/31/O/N/10


11

7 (a) The following list contains the names of types of energy transfer by means of waves.

γ-rays, infra-red, radio/TV/microwaves, sound, visible light, X-rays

(i) Which one of these is not a type of electromagnetic wave?

..................................................................................................................................... [1]

(ii) State the nature of the wave you have named in (a)(i).

..................................................................................................................................... [1]

(iii) The remaining names in the list are all regions of the electromagnetic spectrum, but one
region is missing.

Name the missing region.

..................................................................................................................................... [1]

(b) A television station emits waves with a frequency of 2.5 × 108 Hz. Electromagnetic waves
travel at a speed of 3.0 × 108 m / s.

Calculate the wavelength of the waves emitted by this television station. State the equation
you use.

wavelength = ......................................................... [3]

[Total: 6]

© UCLES 2010 0625/31/O/N/10 [Turn over


12

8 The circuit in Fig. 8.1 contains a 2.0 V cell, whose resistance you should ignore.
There are also three resistors, a 3-position switch, an ammeter and another component, P.

5.0

A
S B
20.0
C

A
P

2.0 V

Fig. 8.1

(a) State the name of component P. .......................................................... [1]

(b) Deduce the resistance of the circuit when switch S is

(i) in position A,

resistance = ......................................................... [1]

(ii) in position B.

resistance = ......................................................... [3]

© UCLES 2010 0625/31/O/N/10


13

(c) Describe and explain what is seen on the ammeter when S is moved to position C.

...................................................................................................................................................

...................................................................................................................................................

...................................................................................................................................................

............................................................................................................................................. [2]

(d) With S in position A, calculate how long it takes for the circuit to transfer 320 J of electrical
energy to other forms.

time taken = ......................................................... [3]

[Total: 10]

© UCLES 2010 0625/31/O/N/10 [Turn over


14

BLANK PAGE

© UCLES 2010 0625/31/O/N/10


15

9 In Fig. 9.1, A and B are two conductors on insulating stands. Both A and B were initially
uncharged.

X Y

A B

Fig. 9.1

(a) Conductor A is given the positive charge shown on Fig. 9.1.

(i) On Fig. 9.1, mark the signs of the charges induced at end X and at end Y of conductor B.
[1]

(ii) Explain how these charges are induced.

...........................................................................................................................................

...........................................................................................................................................

..................................................................................................................................... [3]

(iii) Explain why the charges at X and at Y are equal in magnitude.

...........................................................................................................................................

...........................................................................................................................................

..................................................................................................................................... [1]

(b) B is now connected to earth by a length of wire.

Explain what happens, if anything, to

(i) the charge at X,

...........................................................................................................................................

..................................................................................................................................... [1]

(ii) the charge at Y.

...........................................................................................................................................

..................................................................................................................................... [2]

[Total: 8]
© UCLES 2010 0625/31/O/N/10 [Turn over
16

10 Emissions from a radioactive source pass through a hole in a lead screen and into a magnetic
field, as shown in Fig. 10.1.

radioactive A magnetic field


source into paper

lead
screen

3 cm

Fig. 10.1

Radiation detectors are placed at A, B and C. They give the following readings:

A B C

32 counts / min 543 counts / min 396 counts / min

The radioactive source is then completely removed, and the readings become:

A B C

33 counts / min 30 counts / min 31 counts / min

(a) Explain why there are still counts being recorded at A, B and C, even when the radioactive
source has been removed, and give the reason for them being slightly different.

...................................................................................................................................................

...................................................................................................................................................

...................................................................................................................................................

............................................................................................................................................. [2]

© UCLES 2010 0625/31/O/N/10


17

(b) From the data given, deduce the type of emission being detected, if any, at A, at B and at C
when the radiation source is present.

State the reasons for your answers.

detector at A .............................................................................................................................

...................................................................................................................................................

............................................................................................................................................. [2]

detector at B .............................................................................................................................

...................................................................................................................................................

............................................................................................................................................. [3]

detector at C .............................................................................................................................

...................................................................................................................................................

............................................................................................................................................. [3]

[Total: 10]

© UCLES 2010 0625/31/O/N/10 [Turn over


18

11 When no circuit is connected to the input of a cathode-ray oscilloscope (CRO), there is a horizontal
trace across the middle of the screen.

Fig. 11.1 shows three circuits, each connected to a CRO.

On the grid alongside each circuit, draw the trace that might be seen on the screen of the CRO.

+
battery CRO

a.c. supply CRO

a.c. supply CRO

[6]
Fig. 11.1

[Total: 6]

© UCLES 2010 0625/31/O/N/10


19

BLANK PAGE

© UCLES 2010 0625/31/O/N/10


20

BLANK PAGE

Permission to reproduce items where third-party owned material protected by copyright is included has been sought and cleared where possible. Every
reasonable effort has been made by the publisher (UCLES) to trace copyright holders, but if any items requiring clearance have unwittingly been included, the
publisher will be pleased to make amends at the earliest possible opportunity.

University of Cambridge International Examinations is part of the Cambridge Assessment Group. Cambridge Assessment is the brand name of University of
Cambridge Local Examinations Syndicate (UCLES), which is itself a department of the University of Cambridge.

© UCLES 2010 0625/31/O/N/10


w
w
w
.X
tr
me
eP
ap
UNIVERSITY OF CAMBRIDGE INTERNATIONAL EXAMINATIONS

er
s
International General Certificate of Secondary Education

.c
om
* 9 9 7 8 9 4 1 3 2 5 *

PHYSICS 0625/31
Paper 3 Extended May/June 2011
1 hour 15 minutes
Candidates answer on the Question Paper.
No Additional Materials are required.

READ THESE INSTRUCTIONS FIRST

Write your Centre number, candidate number and name on all the work you hand in.
Write in dark blue or black pen.
You may use a pencil for any diagrams, graphs or rough working.
Do not use staples, paper clips, highlighters, glue or correction fluid.
DO NOT WRITE IN ANY BARCODES.

Answer all questions.


You may lose marks if you do not show your working or if you do not use appropriate units.
Take the weight of 1 kg to be 10 N (i.e. acceleration of free fall = 10 m / s2).

At the end of the examination, fasten all your work securely together.
The number of marks is given in brackets [ ] at the end of each question or part question.

This document consists of 20 printed pages.

DC (LEO/DJ) 28843/5
© UCLES 2011 [Turn over
2

1 In a laboratory, an experiment is carried out to measure the acceleration of a trolley on a horizontal


table, when pulled by a horizontal force.

trolley
force

Fig. 1.1

The measurements are repeated for a series of different forces, with the results shown in the table
below.

force / N 4.0 6.0 10.0 14.0

acceleration 0.50 0.85 1.55 2.25


m / s2

(a) On Fig. 1.2, plot these points and draw the best straight line for your points. [2]

16

force / N

12

0
0 0.5 1.0 1.5 2.0 2.5
acceleration
m / s2

Fig. 1.2

© UCLES 2011 0625/31/M/J/11


3

(b) The graph shows that below a certain force there is no acceleration.

(i) Find the value of this force. ............................................................................................ [1]

(ii) A force smaller than that in (b)(i) is applied to the stationary trolley. Suggest what happens
to the trolley, if anything.

.......................................................................................................................................[1]

(c) Show that the gradient of your graph is about 5.7.

gradient = ...........................................................[1]

(d) (i) State the equation that links resultant force F, mass m and acceleration a.

[1]

(ii) Use your gradient from (c) to find the mass of the trolley.

mass = ...........................................................[2]

(e) On Fig. 1.3, sketch a speed / time graph for a trolley with constant acceleration.

speed

0
0
time

Fig. 1.3
[1]

[Total: 9]

© UCLES 2011 0625/31/M/J/11 [Turn over


4

2 Some builders decide to measure their personal power ratings using apparatus they already have
on site. Fig. 2.1 shows the arrangement they use.

pulley

rope

load

pulley

Fig. 2.1

(a) In the table below, list the three quantities they must measure in order to calculate one man’s
power, and the instrument they would use for each measurement.

quantity to be measured instrument used for measurement

1.

2.

3.

[3]

(b) One workman is measured as having a power of 528 W. His weight is 800 N.

He can develop the same power climbing a ladder, whose rungs are 30 cm apart.

How many rungs can he climb in 5 s?

number of rungs = ...........................................................[3]


© UCLES 2011 0625/31/M/J/11
5

(c) The human body is only about 15% efficient when climbing ladders.

Calculate the actual energy used from the body of the workman in (b) when he climbs 20
rungs.

energy used = ...........................................................[2]

[Total: 8]

© UCLES 2011 0625/31/M/J/11 [Turn over


6

3 During a period of hot weather, the atmospheric pressure on the pond in Fig. 3.1 remains constant.
Water evaporates from the pond, so that the depth h decreases.

force due to
air pressure

Fig. 3.1

(a) Study the diagram and state, giving your reason, what happens during this hot period to

(i) the force of the air on the surface of the pond,

...........................................................................................................................................

.......................................................................................................................................[1]

(ii) the pressure at the bottom of the pond.

...........................................................................................................................................

.......................................................................................................................................[1]

(b) On a certain day, the pond is 12 m deep.

(i) Water has a density of 1000 kg / m3.

Calculate the pressure at the bottom of the pond due to the water.

pressure due to the water = ...........................................................[2]

© UCLES 2011 0625/31/M/J/11


7

(ii) Atmospheric pressure on that day is 1.0 × 105 Pa.

Calculate the total pressure at the bottom of the pond.

total pressure = ...........................................................[1]

(iii) A bubble of gas is released from the mud at the bottom of the pond. Its initial volume is
0.5 cm3.

Ignoring any temperature differences in the water, calculate the volume of the bubble as
it reaches the surface.

volume = ...........................................................[2]

(iv) In fact, the temperature of the water is greater at the top than at the bottom of the pond.

Comment on the bubble volume you have calculated in (b)(iii).

...........................................................................................................................................

...........................................................................................................................................

.......................................................................................................................................[1]

[Total: 8]

© UCLES 2011 0625/31/M/J/11 [Turn over


8

4 The circuit of Fig. 4.1 is set up to run a small immersion heater from a 6.0 V battery.

6.0 V

X A

heater

Fig. 4.1

(a) State the name and purpose of component X.

name .........................................................................................................................................

purpose .................................................................................................................................[1]

(b) The heater is designed to work from a 3.6 V supply. It has a power rating of 4.5 W at this
voltage.

(i) Calculate the current in the heater when it has the correct potential difference across it.

current = ...........................................................[2]

(ii) Calculate the resistance of component X if there is to be the correct potential difference
across the heater. The battery and the ammeter both have zero resistance.

resistance = ...........................................................[3]

(c) Some time after the heater is switched on, the ammeter reading is seen to have decreased.

Suggest why this happens.

...................................................................................................................................................

...............................................................................................................................................[1]

© UCLES 2011 0625/31/M/J/11


9

(d) As an alternative to running the heater from a battery, it is decided to construct a circuit to
enable it to be operated from the a.c. mains supply.

Name the electrical component needed to

(i) reduce the potential difference from that of the mains supply down to a potential difference
suitable for the heater,

.......................................................................................................................................[1]

(ii) change the current from a.c. to a current which has only one direction.

.......................................................................................................................................[1]

[Total: 9]

© UCLES 2011 0625/31/M/J/11 [Turn over


10

5 The manufacturer’s label on an electric heater is as shown in Fig. 5.1.

C.I.E. Electrical Company


Suitable for use on 110 V, 60 Hz supply
1 kW/ 2 kW
This appliance must be earthed when in use

Fig. 5.1

(a) State what electrical quantity is represented by

(i) 110 V, ............................................................................................................

(ii) 60 Hz, ............................................................................................................

(iii) 1 kW. ............................................................................................................ [1]

(b) (i) Which part of the electric heater must be earthed?

.......................................................................................................................................[1]

(ii) Explain what the hazard might be if the heater is not earthed.

...........................................................................................................................................

...........................................................................................................................................

.......................................................................................................................................[2]

(c) The heater has two 110 V heating elements, with two switches, so that either one or both
elements may be switched on.

In the space below, draw a circuit diagram showing how the heating elements and switches
are connected to the mains supply.
Use the symbol for each heating element.

[2]

[Total: 6]
© UCLES 2011 0625/31/M/J/11
11

6 (a) Six different nuclides have nucleon and proton numbers as follows:

nuclide nucleon number proton number

A 214 84

B 214 85

C 211 84

D 211 86

E 210 82

F 210 83

State which two nuclides are isotopes of the same element. .................. and ................. [1]

(b) Thorium-232 has a half-life of 1.4 × 1010 years.

At a particular instant, the activity of a sample of thorium-232 is 120 Bq.

(i) Calculate the time taken for the activity of this sample to fall to 15 Bq.

time taken ...........................................................[1]

(ii) Explain why, when the activity has become 15 Bq, much of the sample will no longer be
thorium-232.

...........................................................................................................................................

...........................................................................................................................................

.......................................................................................................................................[1]

(iii) The sample of thorium-232 is used in an experiment in a laboratory.

Explain why its activity may be regarded as constant.

...........................................................................................................................................

...........................................................................................................................................

.......................................................................................................................................[1]

[Total: 4]

© UCLES 2011 0625/31/M/J/11 [Turn over


12

7 (a) The speed of light in air is known to be 3.0 × 108 m / s.

Outline how you would use a refraction experiment to deduce the speed of light in glass. You
may draw a diagram if it helps to clarify your answer.

...................................................................................................................................................

...................................................................................................................................................

...................................................................................................................................................

...................................................................................................................................................

...................................................................................................................................................

...............................................................................................................................................[4]

(b) A tsunami is a giant water wave. It may be caused by an earthquake below the ocean.

Waves from a certain tsunami have a wavelength of 1.9 × 105 m and a speed of 240 m / s.

(i) Calculate the frequency of the tsunami waves.

frequency = ...........................................................[2]

© UCLES 2011 0625/31/M/J/11


13

(ii) The shock wave from the earthquake travels at 2.5 × 103 m / s.

The centre of the earthquake is 6.0 × 105 m from the coast of a country.

Calculate how much warning of the arrival of the tsunami at the coast is given by the
earth tremor felt at the coast.

warning time = ...........................................................[4]

[Total: 10]

© UCLES 2011 0625/31/M/J/11 [Turn over


14

8 (a) Fig. 8.1 shows a section of an optical fibre. It consists of a fibre of denser transparent material,
coated with a layer of a less dense transparent material.

less dense
material

ray
denser material

Fig. 8.1

One ray within the fibre has been started for you on Fig. 8.1.

(i) State and explain what happens to the ray already drawn, after it reaches the boundary
between the materials.

...........................................................................................................................................

...........................................................................................................................................

.......................................................................................................................................[2]

(ii) On Fig. 8.1, carefully continue the ray until it reaches the end of the section of optical
fibre. [1]

(b) Fibre-optic cables are sometimes used to carry out internal examinations on the human
stomach.

(i) Suggest one reason why the cable is made of thousands of very thin optical fibres.

...........................................................................................................................................

...........................................................................................................................................

.......................................................................................................................................[1]

(ii) Describe briefly how the inside of the stomach is illuminated.

...........................................................................................................................................

...........................................................................................................................................

.......................................................................................................................................[1]

© UCLES 2011 0625/31/M/J/11


15

(iii) Describe briefly how the light from the stomach is transferred to the detecting equipment
outside the body.

...........................................................................................................................................

...........................................................................................................................................

.......................................................................................................................................[1]

[Total: 6]

© UCLES 2011 0625/31/M/J/11 [Turn over


16

9 A simple motor is made in a school laboratory. A coil of wire is mounted on an axle between the
poles of a horseshoe magnet, as illustrated in Fig. 9.1.

coil

B C

N S

A D

springy contacts (brushes)

+ –

battery

Fig. 9.1

(a) At the instant illustrated in Fig. 9.1, the coil ABCD is horizontal and the battery is connected
as shown.

(i) For this position, state the direction of the force on AB and the direction of the motion of
AB.

force on AB ........................................................................................................................

direction of motion of AB ...............................................................................................[1]

(ii) Explain why BC does not contribute to the turning force on the coil.

...........................................................................................................................................

.......................................................................................................................................[1]

© UCLES 2011 0625/31/M/J/11


17

(b) At the instant when the coil is vertical, the springy contacts do not, in fact, make contact with
the ends of the coil.

Describe and explain what happens to the coil.

...................................................................................................................................................

...................................................................................................................................................

...................................................................................................................................................

...............................................................................................................................................[2]

(c) The motor in Fig. 9.1 does not rotate very quickly. The designer of a commercial motor is
required to produce a faster-rotating motor.

Suggest one change that could be made to increase the speed of the motor.

...................................................................................................................................................

...............................................................................................................................................[1]

[Total: 5]

© UCLES 2011 0625/31/M/J/11 [Turn over


18

10 (a) A cathode-ray oscilloscope makes use of the process known as thermionic emission.

Describe what happens during this process.

...................................................................................................................................................

...................................................................................................................................................

...............................................................................................................................................[1]

(b) In the space below, draw a labelled diagram of a cathode-ray oscilloscope.

Include in your diagram the tube, the cathode, the accelerating anode, the focusing anode
and both X- and Y-plates. Do not attempt to show any external circuits.

[3]

(c) A cathode ray is a beam of electrons.

Suggest one way of controlling the number of electrons in the beam.

...................................................................................................................................................

...............................................................................................................................................[1]

© UCLES 2011 0625/31/M/J/11


19

(d) One cathode-ray tube has 5000 V between the accelerating anode and the cathode.

The beam of electrons carries a total charge of 0.0095 C in 5.0 s.

Calculate

(i) the current caused by the beam,

current = ...........................................................[2]

(ii) the energy transferred by the beam in 20 s.

energy = ...........................................................[2]

[Total: 9]

Question 11 is on the next page.

© UCLES 2011 0625/31/M/J/11 [Turn over


20

11 A mass of 0.36 kg of a certain substance is in the solid state in a well-insulated container. The
substance is heated at the rate of 1.2 × 104 J / minute.

2.0 minutes after starting the heating, the substance is all at the same temperature, and it starts to
melt.

11.0 minutes after starting the heating, the substance finishes melting and the temperature starts
to rise again.

(a) Calculate the specific latent heat of the substance.

specific latent heat = ...........................................................[3]

(b) (i) After 11 minutes of heating, when the temperature starts rising again, in which state is
the substance?

.......................................................................................................................................[1]

(ii) Describe what happens to the molecules as thermal energy is supplied to them in this
state.

...........................................................................................................................................

...........................................................................................................................................

...........................................................................................................................................

.......................................................................................................................................[2]

[Total: 6]

Permission to reproduce items where third-party owned material protected by copyright is included has been sought and cleared where possible. Every
reasonable effort has been made by the publisher (UCLES) to trace copyright holders, but if any items requiring clearance have unwittingly been included, the
publisher will be pleased to make amends at the earliest possible opportunity.

University of Cambridge International Examinations is part of the Cambridge Assessment Group. Cambridge Assessment is the brand name of University of
Cambridge Local Examinations Syndicate (UCLES), which is itself a department of the University of Cambridge.

© UCLES 2011 0625/31/M/J/11


w
w
w
.X
tr
me
eP
ap
UNIVERSITY OF CAMBRIDGE INTERNATIONAL EXAMINATIONS

er
s
International General Certificate of Secondary Education

.c
om
* 4 2 0 3 7 8 0 5 5 2 *

PHYSICS 0625/31
Paper 3 Extended October/November 2011
1 hour 15 minutes
Candidates answer on the Question Paper.
No Additional Materials are required.

READ THESE INSTRUCTIONS FIRST

Write your Centre number, candidate number and name on all the work you hand in.
Write in dark blue or black pen.
You may use a pencil for any diagrams, graphs or rough working.
Do not use staples, paper clips, highlighters, glue or correction fluid.
DO NOT WRITE IN ANY BARCODES.

Answer all questions.


You may lose marks if you do not show your working or if you do not use appropriate units.
Take the weight of 1 kg to be 10 N (i.e. acceleration of free fall = 10 m / s2).

At the end of the examination, fasten all your work securely together.
The number of marks is given in brackets [ ] at the end of each question or part question.

This document consists of 15 printed pages and 1 blank page.

DC (NH/DJ) 33051/8
© UCLES 2011 [Turn over
2

1 (a) Define acceleration. Explain any symbols in your definition.

...................................................................................................................................................

...............................................................................................................................................[1]

(b) Fig. 1.1 shows a graph of speed against time for a train. After 100 s the train stops at a
station.

30

speed 25
m/s
20

15

10

0
0 20 40 60 80 100 120 140 160 180 200 220 240 260 280
time / s

Fig. 1.1

(i) For the time interval between 40 s and 100 s, calculate the distance travelled by the train.

distance = ...........................................................[2]

(ii) The train stops for 80 s, then accelerates to 30 m / s with an acceleration of 0.60 m / s2. It
then travels at constant speed.

Complete the graph for the interval 100 s to 280 s, showing your calculations in the space
below.

[5]

[Total: 8]

© UCLES 2011 0625/31/O/N/11


3

2 (a) Energy from the Sun evaporates water from the sea. Some of this water eventually drives a
hydroelectric power station. Give an account of the processes and energy changes involved.

...................................................................................................................................................

...................................................................................................................................................

...................................................................................................................................................

...................................................................................................................................................

...................................................................................................................................................

...................................................................................................................................................

...................................................................................................................................................

...................................................................................................................................................

...................................................................................................................................................

...............................................................................................................................................[4]

(b) In a hydroelectric power station, 200 000 kg of water per second fall through a vertical distance
of 120 m. The water passes through turbines to generate electricity, and leaves the turbines
with a speed of 14 m / s.

(i) Calculate the gravitational potential energy lost by the water in 1 second. Use g = 10 m / s2.

potential energy lost = ............................................................[2]

(ii) Calculate the kinetic energy of the water leaving the turbines in 1 second.

kinetic energy = ............................................................[2]

[Total: 8]

© UCLES 2011 0625/31/O/N/11 [Turn over


4

3 (a) State the two conditions required for the equilibrium of a body acted upon by a number of
forces.

1. ...............................................................................................................................................

...................................................................................................................................................

2. ...............................................................................................................................................

...............................................................................................................................................[2]

(b) Fig. 3.1 shows a diagram of an arm with the hand holding a weight of 120 N.

2 cm

15 cm
20 N
120 N
33 cm

Fig. 3.1

The 20 N force is the weight of the forearm, acting at its centre of mass. F is the force in the
muscle of the upper arm. P is the point in the elbow about which the arm pivots. The distances
of the forces from point P are shown.

(i) By taking moments about point P, calculate the force F.

force F = ...........................................................[3]

(ii) A force acts on the forearm at point P. Calculate this force and state its direction.

force = ...............................................................

direction = ...........................................................[2]

[Total: 7]
© UCLES 2011 0625/31/O/N/11
5

4 (a) Complete Fig. 4.1 to show a simple mercury barometer. Insert the correct labels in the
boxes. Label with the letter h the measurement required to calculate the pressure of the
atmosphere.

[3]
Fig. 4.1

(b) The value of h taken using this barometer is 0.73 m. The density of mercury is 13 600 kg / m3.
Calculate the value of the atmospheric pressure suggested by this measurement.
Use g = 10 m / s2.

atmospheric pressure = ...........................................................[2]

(c) Standard atmospheric pressure is 0.76 m of mercury. Suggest a reason why the value of h in
(b) is lower than this.

...................................................................................................................................................

...............................................................................................................................................[1]

[Total: 6]

© UCLES 2011 0625/31/O/N/11 [Turn over


6

5 (a) Equal volumes of a gas held at constant pressure, a liquid and a solid undergo the same
temperature rise.

(i) State which of the three, solid, liquid or gas,

1. expands the most, ..............................

2. expands the least. ..............................

(ii) Explain why the pressure of the gas must be kept constant for this comparison.

...........................................................................................................................................

.......................................................................................................................................[2]

(b) Fig. 5.1 shows an alcohol thermometer.

–10 0 10 20 30 40 50 60 70 °C

Fig. 5.1

(i) State two properties of alcohol which make it suitable for use in a thermometer.

1. ........................................................................................................................................

...........................................................................................................................................

2. ........................................................................................................................................

.......................................................................................................................................[2]

(ii) State two changes to the design of this thermometer which would make it more
sensitive.

1. ........................................................................................................................................

...........................................................................................................................................

2. ........................................................................................................................................

.......................................................................................................................................[2]

(c) Explain why it is an advantage for the glass surrounding the alcohol in the bulb of the
thermometer to be very thin.

...................................................................................................................................................

...............................................................................................................................................[1]

[Total: 7]

© UCLES 2011 0625/31/O/N/11


7

6 (a) Fig. 6.1 shows the position of layers of air, at one moment, as a sound wave of constant
frequency passes through the air. Compressions are labelled C. Rarefactions are labelled R.

C R C R C R C R C R C

Fig. 6.1

(i) State how Fig. 6.1 would change if

1. the sound had a higher frequency,

................................................................................................................................[1]

2. the sound were louder.

....................................................................................................................................

....................................................................................................................................

................................................................................................................................[2]

(ii) On Fig. 6.1, draw a line marked with arrows at each end to show the wavelength of the
sound. [1]

(b) In an experiment to measure the speed of sound in steel, a steel pipe of length 200 m is
struck at one end with a hammer. A microphone at the other end of the pipe is connected
to an accurate timer. The timer records a delay of 0.544 s between the arrival of the sound
transmitted by the steel pipe and the sound transmitted by the air in the pipe.

The speed of sound in air is 343 m / s. Calculate the speed of sound in steel.

speed of sound in steel = ...........................................................[3]

[Total: 7]

© UCLES 2011 0625/31/O/N/11 [Turn over


8

7 (a) Fig. 7.1 shows a ray of monochromatic red light, in air, incident on a glass block at an angle of
incidence of 50°.

50°

Fig. 7.1

(i) State what is meant by monochromatic light.

.......................................................................................................................................[1]

(ii) For this red ray the refractive index of the glass is 1.52. Calculate the angle of refraction
for the ray.

angle of refraction = ...........................................................[2]

(iii) Without measuring angles, use a ruler to draw the approximate path of the ray in the
glass block and emerging from the block. [2]

© UCLES 2011 0625/31/O/N/11


9

(b) The red ray in Fig. 7.1 is replaced by a ray of monochromatic violet light. For this violet ray the
refractive index of the glass is 1.54. The speed of light in air is 3.00 × 108 m / s.

(i) Calculate the speed of the violet light in the glass block.

speed = ...........................................................[2]

(ii) Use a ruler to draw the approximate path of this violet ray in the glass block and emerging
from the block. Make sure this path is separated from the path drawn for the red light in
(a)(iii). Mark both parts of this path with the letter V. [2]

[Total: 9]

© UCLES 2011 0625/31/O/N/11 [Turn over


10

8 (a) In Fig. 8.1, a magnet is moving towards one end of a solenoid connected to a sensitive centre-
zero meter. During this movement a current is induced in the solenoid.

S N

Fig. 8.1

Suggest three possible changes to the system in Fig. 8.1 that would increase the induced
current.

1. ...............................................................................................................................................

2. ...............................................................................................................................................

3. ...........................................................................................................................................[3]

(b) Fig. 8.2 shows a transformer. P is the primary coil. S is the secondary coil. The coils are
wound on an iron core.

P S

Fig. 8.2

P has 200 turns and S has 800 turns. The e.m.f. induced across S is 24 V. The current in S is
0.50 A. The transformer operates with 100% efficiency.

© UCLES 2011 0625/31/O/N/11


11

Calculate

(i) the voltage of the supply to P,

voltage = ...........................................................[2]

(ii) the current in P.

current = ...........................................................[2]

[Total: 7]

© UCLES 2011 0625/31/O/N/11 [Turn over


12

9 The graphs in Fig. 9.1 show the relation between the current I and the potential difference V for a
resistor and a lamp.

5
I/A
4 lamp

2 resistor

0
0 1 2 3 4 5 6 7 8 9
V/V

Fig. 9.1

(a) (i) Describe how, if at all, the resistance varies as the current increases in

1. the resistor, ...................................................................................................................

2. the lamp. ...................................................................................................................[2]

(ii) State the value of the potential difference when the resistor and the lamp have the same
resistance.

potential difference = ...........................................................[1]

(b) The two components are connected in parallel to a supply of e.m.f. 4.0 V. Calculate the total
resistance of the circuit.

total resistance = ...........................................................[4]

[Total: 7]

© UCLES 2011 0625/31/O/N/11


13

10 (a) In Fig. 10.1, A is a 1000  resistor, C is a transistor, and D is a lamp. S is a 9 V supply.

D
A

+
S

C
B

Fig. 10.1

(i) Name the component labelled B.

.......................................................................................................................................[1]

(ii) At 20 °C the resistance of B is 800 .


At 100 °C the resistance of B is 25 .

In terms of the p.d. across B, explain what happens in the circuit as the temperature
varies from 20 °C to 100 °C.

...........................................................................................................................................

...........................................................................................................................................

...........................................................................................................................................

...........................................................................................................................................

...........................................................................................................................................

.......................................................................................................................................[4]

(b) Suggest a practical use for this circuit.

...................................................................................................................................................

...............................................................................................................................................[1]

[Total: 6]

© UCLES 2011 0625/31/O/N/11 [Turn over


14

11 Fig. 11.1 shows the main components of a cathode-ray oscilloscope.

fluorescent
screen

electron
heater
beam

cathode anode Y-plates X-plates


system vacuum

Fig. 11.1

(a) State the purpose of

(i) the heater,

...........................................................................................................................................

...........................................................................................................................................

(ii) the cathode,

...........................................................................................................................................

...........................................................................................................................................

(iii) the anode system.

...........................................................................................................................................

...........................................................................................................................................

...........................................................................................................................................
[4]

© UCLES 2011 0625/31/O/N/11


15

(b) Without deflection, the electron beam produces a spot at the centre of the fluorescent screen.
A deflection of the spot towards the top of the screen is required.

(i) Describe how the Y-plates can be used to bring about this deflection.

...........................................................................................................................................

...........................................................................................................................................

(ii) Fig. 11.2 shows the Y-plates.

Fig. 11.2

On Fig. 11.2, sketch the pattern of the electric field produced between the plates.
[4]

[Total: 8]

© UCLES 2011 0625/31/O/N/11


16

BLANK PAGE

Permission to reproduce items where third-party owned material protected by copyright is included has been sought and cleared where possible. Every
reasonable effort has been made by the publisher (UCLES) to trace copyright holders, but if any items requiring clearance have unwittingly been included, the
publisher will be pleased to make amends at the earliest possible opportunity.

University of Cambridge International Examinations is part of the Cambridge Assessment Group. Cambridge Assessment is the brand name of University of
Cambridge Local Examinations Syndicate (UCLES), which is itself a department of the University of Cambridge.

© UCLES 2011 0625/31/O/N/11


w
w
w
.X
tr
me
eP
ap
UNIVERSITY OF CAMBRIDGE INTERNATIONAL EXAMINATIONS

er
s
International General Certificate of Secondary Education

.c
om
* 9 3 4 4 1 2 2 4 5 9 *

PHYSICS 0625/31
Paper 3 Extended May/June 2012
1 hour 15 minutes
Candidates answer on the Question Paper.
No Additional Materials are required.

READ THESE INSTRUCTIONS FIRST

Write your Centre number, candidate number and name on all the work you hand in.
Write in dark blue or black pen.
You may use a pencil for any diagrams or graphs.
Do not use staples, paper clips, highlighters, glue or correction fluid.
DO NOT WRITE IN ANY BARCODES.

Answer all questions. For Examiner’s Use


You may lose marks if you do not show your working or if you do not use
appropriate units. 1
Take the weight of 1 kg to be 10 N (i.e. acceleration of free fall = 10 m / s2).
2
At the end of the examination, fasten all your work securely together.
The number of marks is given in brackets [ ] at the end of each question or part 3
question.
4

10

11

Total

This document consists of 12 printed pages.

DC (NF/JG) 43223/5
© UCLES 2012 [Turn over
2

1 The period of the vertical oscillations of a mass hanging from a spring is known to be For
constant. Examiner’s
Use

(a) A student times single oscillations with a stopwatch. In 10 separate measurements, the
stopwatch readings were:

1.8 s, 1.9 s, 1.7 s, 1.9 s, 1.8 s, 1.8 s, 1.9 s, 1.7 s, 1.8 s, 1.8 s.

What is the best value obtainable from these readings for the time of one oscillation?
Explain how you arrive at your answer.

best value = .....................................................................................................................

explanation ......................................................................................................................

..........................................................................................................................................

..................................................................................................................................... [1]

(b) Describe how, using the same stopwatch, the student can find the period of oscillation
more accurately.

..........................................................................................................................................

..........................................................................................................................................

..........................................................................................................................................

..........................................................................................................................................

..........................................................................................................................................

..........................................................................................................................................

..................................................................................................................................... [4]

[Total: 5]

© UCLES 2012 0625/31/M/J/12


3

2 A girl rides her bicycle along a straight level road. Fig. 2.1 shows a graph of her distance For
moved against time. Examiner’s
Use

400 D

300

distance / m

200

100
B

A
0
0 10 20 30 40 50 60
time / s

Fig. 2.1

(a) Describe her motion

(i) from A to B, ..............................................................................................................

(ii) from B to C, ..............................................................................................................

(iii) from C to D. ..............................................................................................................


[3]

(b) Calculate

(i) her average speed from A to D,

average speed = ................................................. [2]

(ii) her maximum speed.

maximum speed = ................................................. [3]

[Total: 8]
© UCLES 2012 0625/31/M/J/12 [Turn over
4

3 (a) State an example of the conversion of chemical energy to another form of energy. For
Examiner’s
example ........................................................................................................................... Use

energy conversion ....................................................................................................... [1]

(b) The electrical output of a solar panel powers a pump. The pump operates a water
fountain. The output of the solar panel is 17 V and the current supplied to the pump is
0.27 A.

(i) Calculate the electrical power generated by the solar panel.

power = ................................................. [2]

(ii) The pump converts electrical energy to kinetic energy of water with an efficiency
of 35%.

Calculate the kinetic energy of the water delivered by the pump in 1 second.

kinetic energy = ................................................. [2]

(iii) The pump propels 0.00014 m3 of water per second. This water rises vertically as a
jet. The density of water is 1000 kg / m3.

Calculate

1. the mass of water propelled by the pump in 1 second,

mass = ................................................. [2]

2. the maximum height of the jet of water.

maximum height = ................................................. [2]

[Total: 9]
© UCLES 2012 0625/31/M/J/12
5

4 Fig. 4.1 represents part of the hydraulic braking system of a car. For
Examiner’s
Use
piston X piston Y

F1
F2

Fig. 4.1

The force F1 of the driver’s foot on the brake pedal moves piston X. The space between
pistons X and Y is filled with oil which cannot be compressed. The force F2 exerted by the oil
moves piston Y. This force is applied to the brake mechanism in the wheels of the car.

The area of cross-section of piston X is 4.8 cm2.

(a) The force F1 is 90 N. Calculate the pressure exerted on the oil by piston X.

pressure = ................................................. [2]

(b) The pressure on piston Y is the same as the pressure applied by piston X. Explain why
the force F2 is greater than the force F1.

..........................................................................................................................................

..................................................................................................................................... [1]

(c) Piston Y moves a smaller distance than piston X. Explain why.

..........................................................................................................................................

..........................................................................................................................................

..................................................................................................................................... [2]

(d) Suggest why the braking system does not work properly if the oil contains bubbles of air.

..........................................................................................................................................

..........................................................................................................................................

..................................................................................................................................... [2]

[Total: 7]

© UCLES 2012 0625/31/M/J/12 [Turn over


6

5 (a) Suggest For


Examiner’s
(i) an example of a change of state resulting from the removal of thermal energy from Use

a quantity of material,

.............................................................................................................................. [1]

(ii) the effect of this change of state on the temperature of the material.

............................................................................................................................. [1]

(b) Define the thermal capacity of a body.

..........................................................................................................................................

..........................................................................................................................................

..................................................................................................................................... [2]

(c) A polystyrene cup holds 250 g of water at 20 °C. In order to cool the water to make a
cold drink, small pieces of ice at 0 °C are added until the water reaches 0 °C and no
unmelted ice is present.

[specific heat capacity of water = 4.2 J / (g °C), specific latent heat of fusion of
ice = 330 J / g]

Assume no thermal energy is lost or gained by the cup.

(i) Calculate the thermal energy lost by the water in cooling to 0 °C.

thermal energy lost = ................................................. [2]

(ii) State the thermal energy gained by the ice in melting.

thermal energy gained = ................................................. [1]

(iii) Calculate the mass of ice added.

mass of ice = ................................................. [2]

[Total: 9]

© UCLES 2012 0625/31/M/J/12


7

6 Fig. 6.1 shows a glass flask full of water at 10 °C and sealed with a bung. A long glass tube For
passes through the bung into the water. The water level in the tube is at X. Examiner’s
Use

bung X

glass flask

water

Fig. 6.1

When the flask is placed in hot water, the water level initially falls a little below X, and then
rises some way above X.

(a) Suggest why

(i) the water level initially falls,

..................................................................................................................................

............................................................................................................................. [2]

(ii) the water level then rises,

..................................................................................................................................

............................................................................................................................. [2]

(iii) the rise is greater than the fall.

..................................................................................................................................

............................................................................................................................. [1]

(b) Suggest a change to the apparatus that would make the fall and rise of the water level
greater.

..........................................................................................................................................

..................................................................................................................................... [1]

[Total: 6]

© UCLES 2012 0625/31/M/J/12 [Turn over


8

7 (a) A wave passes along the surface of the water in a ripple tank. Describe the motion of a For
molecule on the surface as the wave passes. Examiner’s
Use

..........................................................................................................................................

..................................................................................................................................... [1]

(b) Fig. 7.1 shows a view from above of water waves approaching a narrow gap in a barrier.
The water on both sides of the barrier has the same depth.

barrier with
narrow gap

direction of
water waves

Fig. 7.1

(i) On Fig. 7.1, sketch the pattern of waves in the region to the right of the barrier. [2]
(ii) State the process by which waves arrive at point P to the right of the barrier.

.............................................................................................................................. [1]

(c) The waves approaching the barrier in Fig. 7.1 have a wavelength of 1.4 cm and travel at
a speed of 12 cm / s.

Calculate the frequency of the waves.

frequency = ................................................. [2]

[Total: 6]

© UCLES 2012 0625/31/M/J/12


9

8 (a) In Fig. 8.1, S is a metal sphere standing on an insulating base. R is a negatively charged For
rod placed close to S. Examiner’s
Use



R –
S


– insulating
base

Fig. 8.1

(i) Name the particles in S that move when R is brought close to S.

............................................................................................................................. [1]

(ii) On Fig. 8.1, add + signs and – signs to suggest the result of this movement. [1]

(iii) Describe the actions which now need to take place so that S becomes positively
charged with the charge distributed evenly over its surface. A positively charged
object is not available.

..................................................................................................................................

..................................................................................................................................

............................................................................................................................. [3]

(b) During a thunderstorm, the potential difference between thunderclouds and the ground
builds up to 1.5 × 106 V. In each stroke of lightning, 30 C of charge passes between the
thunderclouds and the ground. Lightning strokes to the ground occur, on average, at 2
minute intervals.

Calculate

(i) the average current between the thunderclouds and the ground,

average current = ................................................. [2]

(ii) the energy transferred in each stroke of lightning.

energy = ................................................. [2]


[Total: 9]
© UCLES 2012 0625/31/M/J/12 [Turn over
10

9 This question refers to quantities and data shown on the circuit diagram of Fig. 9.1. For
Examiner’s
6.0 V Use

I4

I1 3.0 1
A R
I2
X Y

I3

2.0 1

Fig. 9.1

(a) State the relationship between

(i) the currents I1, I2 and I3 , ..................................................................................... [1]

(ii) the currents I1 and I4 . ......................................................................................... [1]

(b) The ammeter reads 0.80 A. Assume it has zero resistance.

Calculate

(i) the potential difference between X and Y,

p.d. = ................................................. [1]

(ii) the current I3,

current = ................................................. [2]

(iii) the resistance of R.

resistance = ................................................. [4]


[Total: 9]
© UCLES 2012 0625/31/M/J/12
11

10 (a) Fig. 10.1 shows a wire PQ placed between the poles of a magnet. There is a current in For
wire PQ. Examiner’s
Use

P Q
S

Fig. 10.1

(i) On Fig. 10.1, sketch lines with arrows to show the direction of the magnetic field
between the poles of the magnet. [1]
(ii) The force on PQ is into the paper.

Draw an arrow on PQ to show the direction of the current. [1]

(b) The wire PQ in Fig. 10.1 is replaced by a narrow beam of β-particles travelling from left
to right.

(i) Suggest a suitable detector for the β-particles.

............................................................................................................................. [1]

(ii) State the direction of the force on the β-particles.

............................................................................................................................. [1]

(iii) Describe the path of the β-particles in the space between the poles of the magnet.

..................................................................................................................................

............................................................................................................................. [1]

(iv) State what happens to the air molecules along the path of the β-particles.

............................................................................................................................. [1]

[Total: 6]

Question 11 is on the next page.

© UCLES 2012 0625/31/M/J/12 [Turn over


12

11 Fig. 11.1 shows part of a circuit designed to switch on a security lamp when it gets dark. For
Examiner’s
Use

relay coil S L
A

+

X

Fig. 11.1

When there is a current in the relay coil, switch S closes and the lamp L comes on.

(a) Write down the name of the component X. .................................................. [1]

(b) The circuit has gaps at A and at B.

State the components that need to be connected into these gaps for the circuit to
perform its required function.

gap A ...............................................................................................................................

gap B ...............................................................................................................................
[3]

(c) The circuit in Fig. 11.1 is modified. The function of lamp L is now to give a warning when
the temperature becomes too high.

State any necessary changes of components in the circuit.

..........................................................................................................................................

..........................................................................................................................................

..................................................................................................................................... [2]

[Total: 6]

Permission to reproduce items where third-party owned material protected by copyright is included has been sought and cleared where possible. Every
reasonable effort has been made by the publisher (UCLES) to trace copyright holders, but if any items requiring clearance have unwittingly been included, the
publisher will be pleased to make amends at the earliest possible opportunity.

University of Cambridge International Examinations is part of the Cambridge Assessment Group. Cambridge Assessment is the brand name of University of
Cambridge Local Examinations Syndicate (UCLES), which is itself a department of the University of Cambridge.

© UCLES 2012 0625/31/M/J/12


w
w
w
.X
tr
me
eP
ap
UNIVERSITY OF CAMBRIDGE INTERNATIONAL EXAMINATIONS

er
s
International General Certificate of Secondary Education

.c
om
* 4 1 5 8 3 5 2 4 2 9 *

PHYSICS 0625/31
Paper 3 Extended October/November 2012
1 hour 15 minutes
Candidates answer on the Question Paper.
No Additional Materials are required.

READ THESE INSTRUCTIONS FIRST

Write your Centre number, candidate number and name on all the work you hand in.
Write in dark blue or black pen.
You may use a pencil for any diagrams or graphs.
Do not use staples, paper clips, highlighters, glue or correction fluid. For Examiner’s Use
DO NOT WRITE IN ANY BARCODES.
1
Answer all questions.
You may lose marks if you do not show your working or if you do not use 2
appropriate units.
Take the weight of 1 kg to be 10 N (i.e. acceleration of free fall = 10 m / s2). 3
At the end of the examination, fasten all your work securely together.
4
The number of marks is given in brackets [ ] at the end of each question or part
question.
5

10

11

Total

This document consists of 16 printed pages.

DC (AC/SW) 49515/3
© UCLES 2012 [Turn over
2

1 Fig. 1.1 shows the graph of speed v against time t for a train as it travels from one station to For
the next. Examiner’s
Use
20

v
m/s

10

0
0 20 40 60 80 100 120 140 160
t /s
Fig. 1.1
(a) Use Fig. 1.1 to calculate

(i) the distance between the two stations,

distance = ................................................. [4]

(ii) the acceleration of the train in the first 10 s.

acceleration = ................................................. [2]

© UCLES 2012 0625/31/O/N/12


3

(b) The mass of the train is 1.1 × 105 kg. For


Examiner’s
Calculate the resultant force acting on the train in the first 10 s. Use

resultant force = ................................................. [2]

(c) The force generated by the engine of the train is called the driving force.

Write down, in words, an equation relating the driving force to any other forces acting on
the train during the period t = 10 s to t = 130 s.

.................................................................................................................................... [1]

[Total: 9]

© UCLES 2012 0625/31/O/N/12 [Turn over


4

2 (a) State the factors which completely describe a vector quantity. For
Examiner’s
.......................................................................................................................................... Use

.................................................................................................................................... [1]

(b) An aeroplane is flying towards the east in still air at 92 m / s. A wind starts to blow at
24 m / s towards the north.

Draw a vector diagram to find the resultant velocity of the aeroplane. Use a scale of
1.0 cm = 10 m / s.

resultant speed = ......................................................

angle between resultant and easterly direction = ......................................................


[5]

[Total: 6]

© UCLES 2012 0625/31/O/N/12


5

3 (a) A stationary body is acted upon by a number of forces. State the two conditions which For
must apply for the body to remain at rest. Examiner’s
Use

1. ......................................................................................................................................

2. ......................................................................................................................................
[2]

(b) Fig. 3.1 shows a device used for compressing crushed material.

380 mm 120 mm

lever arm
plunger
20 N

cylinder

cross-sectional crushed material


F
area A

Fig. 3.1

The lever arm rotates about the hinge H at its right-hand end. A force of 20 N acts
downwards on the left-hand end of the lever arm. The force F of the crushed material on
the plunger acts upwards. Ignore the weight of the lever arm.

(i) Use the clockwise and anticlockwise moments about H to calculate the upward
force F which the crushed material exerts on the plunger. The distances are shown
on Fig. 3.1.

force F = ................................................. [3]

(ii) The cross-sectional area A of the plunger in contact with the crushed material is
0.0036 m2. Calculate the pressure exerted on the crushed material by the plunger.

pressure = ................................................. [2]


[Total: 7]
© UCLES 2012 0625/31/O/N/12 [Turn over
6

4 (a) State what is meant by the centre of mass of a body. For


Examiner’s
.......................................................................................................................................... Use

..........................................................................................................................................

.................................................................................................................................... [1]

(b) Fig. 4.1 shows an athlete successfully performing a high jump.

Fig. 4.1

The height of the bar above the ground is 2.0 m. The maximum increase in gravitational
potential energy (g.p.e.) of the athlete during the jump is calculated using the expression
g.p.e. = mgh.

Explain why the value of h used in the calculation is much less than 2.0 m.

..........................................................................................................................................

..........................................................................................................................................

.................................................................................................................................... [1]

© UCLES 2012 0625/31/O/N/12


7

(c) Fig. 4.2 shows, in order, five stages of an athlete successfully performing a pole-vault. For
Examiner’s
Use

Fig. 4.2

Describe the energy changes which take place during the performance of the pole-
vault, from the original stationary position of the pole-vaulter before the run-up, to the
final stationary position after the vault.

..........................................................................................................................................

..........................................................................................................................................

..........................................................................................................................................

..........................................................................................................................................

..........................................................................................................................................

..........................................................................................................................................

..........................................................................................................................................

..........................................................................................................................................

..........................................................................................................................................

..........................................................................................................................................

.................................................................................................................................... [6]

[Total: 8]

© UCLES 2012 0625/31/O/N/12 [Turn over


8

5 (a) Explain For


Examiner’s
(i) how gas molecules exert a force on a solid surface, Use

..................................................................................................................................

..................................................................................................................................

............................................................................................................................ [1]

(ii) the increase in pressure of a gas when its volume is decreased at constant
temperature.

..................................................................................................................................

..................................................................................................................................

..................................................................................................................................

............................................................................................................................ [3]

(b) A cylinder of volume 5.0 × 103 cm3 contains air at a pressure of 8.0 × 105 Pa.

A leak develops so that air gradually escapes from the cylinder until the air in the cylinder
is at atmospheric pressure. The pressure of the atmosphere is 1.0 × 105 Pa.

Calculate the volume of the escaped air, now at atmospheric pressure. Assume that the
temperature stays constant.

volume = ............................................cm3 [4]

[Total: 8]

© UCLES 2012 0625/31/O/N/12


9

6 (a) Define specific latent heat of fusion. For


Examiner’s
.......................................................................................................................................... Use

..........................................................................................................................................

.................................................................................................................................... [1]

(b) (i) A tray of area 0.25 m2, filled with ice to a depth of 12 mm, is removed from a
refrigerator.

Calculate the mass of ice on the tray. The density of ice is 920 kg / m3.

mass = ................................................. [2]

(ii) Thermal energy from the Sun is falling on the ice at a rate of 250 W / m2. The ice
absorbs 60 % of this energy.

Calculate the energy absorbed in 1.0 s by the 0.25 m2 area of ice on the tray.

energy = ................................................. [2]

(iii) The ice is at its melting temperature.

Calculate the time taken for all the ice to melt. The specific latent heat of fusion of
ice is 3.3 × 105 J / kg.

time = ................................................. [3]

[Total: 8]

© UCLES 2012 0625/31/O/N/12 [Turn over


10

7 (a) Explain why a liquid cools when evaporation takes place from its surface. For
Examiner’s
.......................................................................................................................................... Use

..........................................................................................................................................

.................................................................................................................................... [2]

(b) Fig. 7.1 shows five vessels each made of the same metal and containing water.

Vessels A, B, C and D are identical in size and shape. Vessel E is shallower and wider.
The temperature of the air surrounding each vessel is 20 °C.

A B C D E

Fig. 7.1

The table shows details about each vessel and their contents.

volume of initial temperature


vessel outer surface
water / cm3 of water / °C
A dull 200 80
B shiny 200 80
C dull 200 95
D dull 100 80
E dull 200 80

The following questions are about the time taken for the temperature of the water in the
vessels to fall by 10 °C from the initial temperature.

(i) Explain why the water in B takes longer to cool than the water in A.

..................................................................................................................................

............................................................................................................................ [1]

(ii) Explain why the water in C cools more quickly than the water in A.

..................................................................................................................................

............................................................................................................................ [1]

(iii) Explain why the water in D cools more quickly than the water in A.

..................................................................................................................................

............................................................................................................................ [1]

© UCLES 2012 0625/31/O/N/12


11

(iv) Suggest two reasons why the water in E cools more quickly than the water in A. For
Examiner’s
1. ............................................................................................................................... Use

..................................................................................................................................

2. ...............................................................................................................................

..................................................................................................................................
[2]

[Total: 7]

© UCLES 2012 0625/31/O/N/12 [Turn over


12

8 (a) A ray of light in air travels across a flat boundary into glass. The angle of incidence is For
51°. The angle of refraction is 29°. Examiner’s
Use

(i) In the space below, draw a labelled diagram to illustrate this information. [3]

(ii) Calculate the refractive index of the glass.

refractive index = ................................................. [2]

(b) A ray of light in glass travels towards a flat boundary with air. The angle of incidence is
51°. This ray does not emerge into the air.

State and explain what happens to this ray.

..........................................................................................................................................

..........................................................................................................................................

..........................................................................................................................................

.................................................................................................................................... [2]

[Total: 7]

© UCLES 2012 0625/31/O/N/12


13

9 Fig. 9.1 shows a thin, straight rod XY placed in the magnetic field between the poles of a For
magnet. The wires from the ends of XY are connected to a centre-zero voltmeter. Examiner’s
Use

N S

Fig. 9.1

(a) When XY is moved slowly upwards the needle of the voltmeter shows a small deflection.

(i) State how XY must be moved to produce a larger deflection in the opposite
direction.

..................................................................................................................................

............................................................................................................................ [2]

(ii) XY is now rotated about its central point by raising X and lowering Y. Explain why
no deflection is observed.

..................................................................................................................................

..................................................................................................................................

............................................................................................................................ [2]

(b) The effect of moving XY can be seen if the wires are connected to the terminals of a
cathode-ray oscilloscope instead of the voltmeter.

(i) State the parts inside the oscilloscope tube to which these terminals are connected.

............................................................................................................................ [1]

(ii) The spot on the oscilloscope screen moves up and down repeatedly. State how XY
is being moved.

............................................................................................................................ [1]

(iii) State the setting of the time-base of the oscilloscope during the process described
in (ii).

............................................................................................................................ [1]

[Total: 7]

© UCLES 2012 0625/31/O/N/12 [Turn over


14

10 (a) State the electrical quantity that has the same value for each of two resistors connected For
to a battery Examiner’s
Use

(i) when they are in series, ............................................................................................

(ii) when they are in parallel. ..........................................................................................


[1]

(b) Fig. 10.1 shows a circuit with a 1.2 kΩ resistor and a thermistor in series. There is no
current in the voltmeter.

1.2 k1

9.0 V

Fig. 10.1

Calculate the voltmeter reading when the resistance of the thermistor is 3.6 kΩ.

voltmeter reading = ................................................. [3]

© UCLES 2012 0625/31/O/N/12


15

(c) Fig. 10.2 shows a fire-alarm circuit. The circuit is designed to close switch S and ring For
bell B if there is a fire. Examiner’s
Use

S B
relay coil

9.0 V

Fig. 10.2

Explain the operation of the circuit.

..........................................................................................................................................

..........................................................................................................................................

..........................................................................................................................................

..........................................................................................................................................

..........................................................................................................................................

..........................................................................................................................................

.................................................................................................................................... [3]

[Total: 7]

Question 11 is on the next page.

© UCLES 2012 0625/31/O/N/12 [Turn over


16

11 (a) A radioactive source emits α-, β- and γ-radiation. For


Examiner’s
Which of these radiations Use

(i) has the shortest range in air, ..............................

(ii) has a negative charge, ..............................

(iii) is not deflected in a magnetic field? ..............................


[2]

(b) In a famous experiment, carried out in a vacuum, a very thin sheet of gold was placed in
the path of alpha particles.

It was found that a large number of the alpha particles passed through the sheet with
little or no deflection from their original path. A very small number of the alpha particles
were reflected back towards the source.

(i) Explain, in terms of the force acting, why the direction of motion of an alpha particle
changes when it comes close to the nucleus of a gold atom.

..................................................................................................................................

..................................................................................................................................

............................................................................................................................ [2]

(ii) State two conclusions, about the nuclei of atoms, that were made from the results
of this experiment.

1. ...............................................................................................................................

..................................................................................................................................

2. ...............................................................................................................................

..................................................................................................................................
[2]

[Total: 6]

Permission to reproduce items where third-party owned material protected by copyright is included has been sought and cleared where possible. Every
reasonable effort has been made by the publisher (UCLES) to trace copyright holders, but if any items requiring clearance have unwittingly been included, the
publisher will be pleased to make amends at the earliest possible opportunity.

University of Cambridge International Examinations is part of the Cambridge Assessment Group. Cambridge Assessment is the brand name of University of
Cambridge Local Examinations Syndicate (UCLES), which is itself a department of the University of Cambridge.

© UCLES 2012 0625/31/O/N/12


ALTERNATIVE
TO PRACTICAL
PAPER SIX

6
Candidate
Centre Number Number

Candidate Name

International General Certificate of Secondary Education


UNIVERSITY OF CAMBRIDGE LOCAL EXAMINATIONS SYNDICATE

PHYSICS 0625/6
PAPER 6 Alternative to Practical
Friday 28 MAY 1999 Morning 1 hour
Candidates answer on the question paper.
Additional materials:
Electronic calculator and/or Mathematical tables
Protractor
Ruler (30 cm)

TIME 1 hour

INSTRUCTIONS TO CANDIDATES
Write your name, Centre number and candidate number in the spaces at the top of this page.
Answer all questions.
Write your answers in the spaces provided on the question paper.

INFORMATION FOR CANDIDATES


The number of marks is given in brackets [ ] at the end of each question or part question.
You may use a calculator.

FOR EXAMINER’S USE

TOTAL

This question paper consists of 13 printed pages and 3 blank pages.


MML LOC 898 4/98 QF91703
© UCLES 1999 [Turn over
2 For
Examiner's
Use
1 Rays of light were reflected from the outside surface of a beaker that contained some water
topicspeed
coloured with blue ink.

P4

layout of apparatus as seen from above


beaker
P3

I P2
P1
A B L
lamp

Fig. 1.1

The path of an incident ray LI, from lamp L, was marked by two pins P1 and P2. The
reflected ray was located and marked using pins P3 and P4.

The diagram of Fig. 1.1 represents the beaker, the lamp and the pin marks.

(a) Complete the diagram by drawing lines to represent the incident and reflected rays.
Label each ray. [3]

(b) Measure the angle between the incident and the reflected ray, call this angle d and
record its value in the table. Also measure and record the angle e between the incident
ray and the line AL. [2]

angle
°
d

0625/6/S99
3 For
Examiner's
Use
(c) Use your values of d and e to calculate a value for the ratio d
–.
e

d
– = ............................................................................................ [1]
e
(d) (i) On Fig. 1.1, mark, with the letter E, the position where you would place your eye
so as to see the lamp by the reflected ray.
(ii) Why is it an advantage to fill the beaker with coloured water when viewing the
lamp by a ray reflected from the outside surface of the beaker? You may draw a
diagram if you wish.

..................................................................................................................................

..................................................................................................................................

..................................................................................................................................
[2]

0625/6/S99 [Turn over


4 For
Examiner's
Use
2 The lever balance shown in Fig. 2.1 was constructed from a straw AB fitted with a pointer at
topictransfer topicthermal topicenergy
one end and a piece of card. The balance was supported on a pivot and the straw set
horizontal by adjusting the position of a small counterweight. The height of the pointer was
measured by using a metre rule placed beside the apparatus.

pointer

pivot
straw
A B
metre rule
card
Y
X
counterweight bench

Fig. 2.1

(a) Describe how you would check that the metre rule was perpendicular to the bench.
You may draw on Fig. 2.1.

..........................................................................................................................................

..........................................................................................................................................

.................................................................................................................................... [1]

(b) When the counterweight was placed at the corner X of the card, the straw was found to
be horizontal. In the space below, draw a sketch of the straw to represent its position
when the counterweight was attached at the point labelled Y, not X.

[2]

0625/6/S99
5 For
Examiner's
Use
(c) With the straw horizontal, as shown in Fig. 2.1, a small paper clip was attached to the
end A of the straw. The balance came to rest as shown in Fig. 2.2. Write down the
reading shown for the height of the pointer B.

metre rule

27
26
pointer
pivot
B

25
straw

24
card

paper clip

Fig. 2.2

pointer reading = ........................................................................................................ [1]

(d) In one experiment, additional clips were attached to the end of the straw. The clips
were added one at a time. After each clip had been added, the height h of the pointer
was determined. Draw up a table in which you could record the values of h together
with the corresponding total number of clips used. Your table should be suitable for use
in your laboratory book.

[2]

0625/6/S99 [Turn over


6 For
Examiner's
Use
(e) The distance d moved by the pointer, for each number n of paper clips hung from the
balance, was calculated from the values of h. The graph of Fig. 2.3 represents the
results of one experiment.

d / cm x
10 x
x
9
x
8 x

7 x

6 x

x
5

4 x

x
3

2 x

1 x

0
0 1 2 3 4 5 6 7 8 9 10 11 12
number of paper clips n

Fig. 2.3

The paper clips were removed from the straw and a small mass of plasticine was hung
on the end A of the straw. The following information was obtained when this small
mass of plasticine was in air and when it was totally submerged in water.

with the plasticine in air, the distance moved by the pointer da = 9.1 cm
with the plasticine in water, the distance moved by the pointer dw = 4.6 cm
(i) From Fig. 2.3 determine the corresponding value for the number n of clips in use.

for da = 9.1 cm, na = ........................

for dw = 4.6 cm, nw = ........................


(ii) The values for na and nw are not the same. As well as its weight, another force F
is acting on the plasticine when it is submerged in water.

1. In which direction does F act? ..............................................................................

2. Using the information you gave in (i) what can you say about the magnitude of F?

........................................................................................................................................
[2]
0625/6/S99
7 For
Examiner's
Use
3 An equal volume of water was placed in each of three similar test-tubes. The tubes were
topicmodels
topicmolecular
topicmolecularmodels
topiclens
used in three experiments as described below.

Experiment 1

A small sheet of glass was placed on top of the tube and the apparatus turned upside
down, as shown in Fig. 3.1. A strip of graph paper was held close to the tube and used as a
scale.

graph grid as a scale


water

h1

sheet of glass

Fig. 3.1

The height h1 of the water level above the glass was observed over a period of 5 hours.
The observations are recorded in the table on page 8.

Experiment 2

The apparatus was similar to that for experiment 1. In addition, a piece of paper tissue was
placed between the sheet of glass and the tube, as shown in Fig. 3.2. Once again, the
height h2 of the water level was observed over a period of 5 hours.

graph grid as a scale graph grid as a scale

water
fan
h2 water
h3
tissue tissue
sheet of glass sheet of glass

Fig. 3.2 Fig. 3.3

0625/6/S99 [Turn over


8 For
Examiner's
Use
Experiment 3

The apparatus was similar to that used for experiment 2. In addition, an electric fan was
used to produce a draught over the surface of the tissue, as shown in Fig. 3.3. The area
and shape of the tissue paper was the same as for experiment 2. Once again, the height h3
of the water level was observed over a period of 5 hours.

The three experiments were conducted over the same 5 hour period. The temperature of
the water was the same at the start of the experiments.

The observations are shown in the table.

height of water levels

room glass tissue tissue + fan


time temperature
t / hour  / °C h1 / cm h2 / cm h3 / cm

0 26.8 14.0 14.0 14.0

1 26.8 14.0 13.3 12.5

2 27.0 14.0 12.8 9.3

3 26.9 14.0 12.1 6.5

4 27.0 14.0 11.4 3.4

5 27.0 14.0 10.9 0.1

Fig. 3.4

(a) (i) Calculate an average value for room temperature  during the 5 hour period.

average value for  = ......................................................


(ii) What was the largest difference between the average room temperature and the
actual temperature of the room?

largest temperature difference = ....................................


Explain why it is reasonable to assume that the temperature of the room remained
constant during the three experiments.

..................................................................................................................................

..................................................................................................................................
[3]

0625/6/S99
9 For
Examiner's
Use
(b) (i) The area of cross-section of each tube was 2.0 cm2. During the 5 hour period,
what volume of water flowed out of each tube?
Experiment 1

volume = ...........................
Experiment 2

volume = ...........................
Experiment 3

volume = ...........................
(ii) Calculate the average rate of flow of water, in cm3 per hour, from the tube during
experiment 3.

rate of flow = ..............................


[2]

(c) Write a conclusion to the experiments. Your conclusion should explain why the three
rates of flow are so different.

..........................................................................................................................................

..........................................................................................................................................

..........................................................................................................................................

..........................................................................................................................................

.................................................................................................................................... [3]

0625/6/S99 [Turn over


10 For
Examiner's
Use
4 The circuit shown in Fig. 4.1 was used to determine R, the resistance of a resistor, using
topicevaporation
the equation
R=V
–.
I
power supply

+ –

switch
X

Y
R

Fig. 4.1

The value for R is to be determined for different values of current I.

(a) Name the components labelled X and Y.

X ......................................................................................................................................

Y ................................................................................................................................ [2]

(b) What is the purpose of the component X?

..........................................................................................................................................

.................................................................................................................................... [1]

(c) Explain how you would use the apparatus to determine values of R. Your answer
should include what you would do before you close the switch.

..........................................................................................................................................

..........................................................................................................................................

..........................................................................................................................................

.................................................................................................................................... [4]

(d) The value of R is about 9.5  and the current through it must not exceed 0.10 A. What
would be a good choice for the maximum reading of the component labelled Z?

maximum reading = ...................................................... [1]

0625/6/S99
11

BLANK PAGE

Question 5 starts on page 12

0625/6/S99 [Turn over


12 For
Examiner's
Use
5 A drinking straw was sealed at one end with candle wax. The straw was made to float
topicresistance
upright in water by putting a small amount of ballast into the straw. The height of the top of
the straw above the bench was determined. This height was called h, as shown in Fig. 5.1.

set-square rule

drinking straw

container full of water

h
ball

ballast

wax

Fig. 5.1

A steel ball was put inside the straw and the new value for h was obtained. Additional balls
were put in the straw one at a time. For each new ball the corresponding value of h was
determined. Fig. 5.2 shows the values obtained.

number n of balls 0 1 2 3 4 5 6 7 8

h / cm 29.1 28.2 27.1 26.3 25.0 24.2 23.1 22.0 21.1

Fig. 5.2

(a) (i) Plot the graph of h / cm (y-axis) against n (x-axis). Start your y-axis at h = 16 cm
and make sure that your x-axis is scaled over the values n = 0 to n = 10.
(ii) Using your graph, estimate the value of h you would obtain if 10 balls were added.
Show your working.
Mark the graph to show how you obtained this value for h.

h = .................................................................
[4]
0625/6/S99
13 For
Examiner's
Use

0625/6/S99 [Turn over


14 For
Examiner's
Use
(b) Explain why the set-square is used when taking the reading for h.

..........................................................................................................................................

.................................................................................................................................... [2]

(c) Before taking the readings for h, one student pushed the straw down a little. The straw
then moved up and down before coming to rest. Suggest a reason why this was done.

.................................................................................................................................... [1]

(d) The straw floats in the water so that its top is as high as possible. For this reason, the
container was always full of water. Suggest what would happen if the container was
not quite full so that the water surface was as shown in Fig. 5.3.

Fig. 5.3

.................................................................................................................................... [1]

0625/6/S99
Candidate
Centre Number Number

Candidate Name

International General Certificate of Secondary Education


UNIVERSITY OF CAMBRIDGE LOCAL EXAMINATIONS SYNDICATE

PHYSICS 0625/6
PAPER 6 Alternative to Practical
Monday 22 NOVEMBER 1999 Morning 1 hour

Candidates answer on the question paper.


Additional materials:
Electronic calculator and/or Mathematical tables
Ruler (30 cm)

TIME 1 hour

INSTRUCTIONS TO CANDIDATES
Write your name, Centre number and candidate number in the spaces at the top of this page.
Answer all questions.
Write your answers in the spaces provided on the question paper.

INFORMATION FOR CANDIDATES


The number of marks is given in brackets [ ] at the end of each question or part question.
You may use a calculator.

FOR EXAMINER’S USE

TOTAL

This question paper consists of 11 printed pages and 1 blank page.


SB (SLC/DJ) QK93709/2
© UCLES 1999 [Turn over
2 For
Examiner’s
Use
1 A transparent U-tube is held vertically in a clamp. Some water and kerosene are poured into
topicspeed
the tube and the final levels of the liquids are as shown in Fig. 1.1. This diagram is drawn full
size.

P kerosene

water

bench top

Fig. 1.1

(a) Using your own ruler, make measurements which will enable you to determine h, the
difference between the vertical heights of the water levels above the bench top at P and
at Q. Show all of your measurements and your calculation of h in the space below.

[4]

0625/6 W99
3 For
Examiner’s
Use
(b) When the experiment is carried out in a laboratory, the vertical height of the water level
at P is measured using a half-metre rule.

Explain how you would use the rule when the apparatus is on a bench in the laboratory.
Your answer should include how you would position the rule and what you would do to
obtain an accurate value for the height of the water level. You may draw a diagram or
draw on Fig. 1.1 if you wish.

..........................................................................................................................................

..........................................................................................................................................

..........................................................................................................................................

......................................................................................................................................[4]

0625/6 W99 [Turn over


4 For
Examiner’s
Use
2 (a) The circuit symbol for a diode is shown in Fig. 2.1. The diode conducts when the polarity
topicturningeffect
topicturning
is as shown.

+ –

Fig. 2.1

Draw a circuit diagram showing the following components, all connected in series:
a d.c. power supply, labelled to show its polarity,
a fixed resistor,
a diode,
a switch.

On your circuit diagram, the switch should be shown open and the diode should be able
to conduct when the switch is closed.

[3]

(b) (i) Redraw your circuit diagram, adding an ammeter to measure the current in the
diode. Label the polarity of the ammeter terminals.

(ii) Is there any other position in the circuit where you could put the ammeter to
measure the current through the diode? Tick one box.

yes
no

Give one reason to support your answer.

...................................................................................................................................

...................................................................................................................................

...................................................................................................................................
[3]
0625/6 W99
5 For
Examiner’s
Use
(c) (i) Assuming that the fixed resistor has a resistance of 100 Ω and that the potential
difference of the power supply is 3.0 V, calculate the maximum current Imax in the
circuit.

Imax = ......................................

(ii) In order to calculate the value for Imax in (i) above, what assumption did you make
about the resistance of the circuit?

...................................................................................................................................

...................................................................................................................................
[2]

0625/6 W99 [Turn over


6 For

; ;
Examiner’s
Use
3 A small steel ball is dropped through a hole in a card and falls through a clear plastic tube
topicmolecularmodels
topicmolecular
topicmodels
topiclens
before it hits a glass surface as shown in Fig. 3.1. There is a rubber band on the outside of
the tube. This band can be moved along the tube.

hole

;;
card
plastic tube

clamp
ball

rubber
band

14.7 cm

glass surface

Fig. 3.1

When the ball bounces, it rises up inside the tube to a height shown as h in Fig. 3.1. The top
of the bounce is marked by the position of the rubber band.

(a) Four different balls are used. These are dropped from the same height onto four pieces
of glass, which are 2 mm thick. The areas of the top surface of the glass are 56 cm2,
112 cm2, 224 cm2 and 450 cm2. The results of this experiment are shown in Fig. 3.2.

mass of values of h / cm for each of the four pieces of glass


ball / g 56 cm2 112 cm2 224 cm2 450 cm2
1.08 0 0 0 0
0.40 1.8 1.8 1.8 1.8
0.24 3.5 3.5 3.5 3.5
0.12 5.4 5.4 5.4 5.4

Fig. 3.2

0625/6 W99
7 For
Examiner’s
Use
maximum mass
(i) 1. Calculate the ratio ––––––––––––– for the balls.
minimum mass

ratio of masses = ......................................


maximum area
2. Calculate the ratio –––––––––––– for the glass surfaces.
minimum area

ratio of areas = ......................................

(ii) Use the table to justify the assertion that the value for h does not depend upon the
value for the surface area of the glass.

...................................................................................................................................

...................................................................................................................................

...................................................................................................................................

...................................................................................................................................

(iii) Describe how the height h depends upon the mass of the ball for this experiment.

...................................................................................................................................

...................................................................................................................................
[6]

(b) The height of the hole above the surface of the glass block is 14.7 cm. Each ball is
released from this height. The following relation gives the change in potential energy of
each ball when it falls.

energy = mass x g x height

All of this energy becomes kinetic energy just before the moment of impact with the
glass.

Suggest what happens to this kinetic energy after the impact for the heaviest and for the
lightest balls. Your answer should include information from the table.

heaviest ball .....................................................................................................................

..........................................................................................................................................

lightest ball .......................................................................................................................

......................................................................................................................................[2]

0625/6 W99 [Turn over


8 For
Examiner’s
Use
4 A small mass of ammonium chloride is dissolved in some water, causing the temperature of
topicenergy
topicenergytransformations
topicapparentdepth
topictransformations
topicelectric
the water to fall. The apparatus, which is used to determine the fall in temperature, is shown
in Fig. 4.1.

lid

;;;
hand
stirrer lens

thermometer

;;;
lagging

Fig. 4.1

(a) Give a reason for using each of the following items of apparatus.

(i) the lagging

...................................................................................................................................

...................................................................................................................................

(ii) the stirrer

...................................................................................................................................

...................................................................................................................................

(iii) the hand lens

...................................................................................................................................

...................................................................................................................................
[3]

0625/6 W99
9 For
Examiner’s
Use
(b) Part of the thermometer that is used to determine the fall in temperature is shown in
Fig. 4.2. The diagram shows the thermometer before and after adding the ammonium
chloride.

°C -10 0 10 20 30 40 50

°C -10 0 10 20 30 40 50

Fig. 4.2

(i) Record each of the temperatures and determine the fall in temperature.

temperature before adding the ammonium chloride = .....................................

temperature after adding the ammonium chloride = .....................................

fall in temperature = .....................................

(ii) In Fig. 4.2 the liquid thread is shown along the edge of the scale marks. This is the
recommended way to position the liquid thread before reading a temperature. In
Fig. 4.3 the thread is positioned away from the edge of the scale.

°C -10 0 10 20 30 40 50

Fig. 4.3

Suggest a reason for the recommended way to use a thermometer.

...................................................................................................................................

...................................................................................................................................
[4]

(c) How would you avoid making a parallax error when reading the thermometer shown in
Fig. 4.2? You may draw a labelled diagram if you wish.

......................................................................................................................................[1]

0625/6 W99 [Turn over


10 For
Examiner’s
Use
5 An image of the filament of a lamp is produced on a screen using the apparatus shown in
topicsound
topicspeedofsoundandlight
topicspeed
topiclight
Fig. 5.1. The image distance, shown as x, is measured and recorded.

screen lens lamp

Fig. 5.1

y
w

screen block lens lamp

Fig. 5.2

A glass block is now placed between the lens and the screen as shown in Fig. 5.2. The block
has width w. The image is out of focus. The screen is moved until the image is once again
sharp. The lens to screen distance, shown as y in Fig. 5.2, is now greater than x. The new
value for y is measured and recorded.

The experiment is repeated for different values of the lamp to lens distance. The values
obtained for x and y are given in Fig. 5.3.

y / mm 175 184 212 250 290

x / mm 152 163 189 229 269

(y – x )/mm

Fig. 5.3

0625/6 W99
11 For
Examiner’s
Use
(a) Plot the graph of y / mm (y-axis) against x / mm (x-axis). Start your x-axis at x / mm = 140
and your y-axis at y / mm = 170. Draw the best straight line to fit the plotted points.

[5]

(b) (i) Determine the values of (y – x) and enter them in the table of Fig. 5.3, shown on
page 10. On Fig. 5.2, show what is meant by the distance (y – x).

(ii) State what happens to the value for (y – x) as the value for x increases.

...................................................................................................................................

(iii) A second glass block of width greater than w is used in a repeat experiment.
Suggest how this might affect the values of (y – x).

...................................................................................................................................

...................................................................................................................................

...................................................................................................................................
[3]

0625/6 W99
12

BLANK PAGE

0625/6 W99
Candidate
Centre Number Number

Candidate Name

International General Certificate of Secondary Education


UNIVERSITY OF CAMBRIDGE LOCAL EXAMINATIONS SYNDICATE

PHYSICS 0625/6
PAPER 6 Alternative to Practical
MAY/JUNE SESSION 2000 1 hour
Candidates answer on the question paper.
Additional materials:
Electronic calculator and/or Mathematical tables
Protractor
Ruler (30 cm)

TIME 1 hour

INSTRUCTIONS TO CANDIDATES
Write your name, Centre number and candidate number in the spaces at the top of this page.
Answer all questions.
Write your answers in the spaces provided on the question paper.

INFORMATION FOR CANDIDATES


The number of marks is given in brackets [ ] at the end of each question or part question.
You may use a calculator.

FOR EXAMINER’S USE

TOTAL

This question paper consists of 11 printed pages and 1 blank page.


SB (SM/KS) QF05712/3
© UCLES 2000 [Turn over
2 For
Examiner’s
Use
1 Fig. 1.1 represents the apparatus an IGCSE class is using for an optics experiment, in which
topicspeed
a glass beaker filled with water acts like a lens.

The glass beaker filled with water is placed with C, the centre of its base, on a line labelled
I
LL . An optics pin is placed at the point labelled O, so that the pin is touching the side of the
beaker.

Two points A and A are on the surface of the beaker at equal distances from the line LL . The
I I

pin at point O acts as an optical object. The ray emerging from A is located by using two pins
placed at two points labelled P1 and P2.

(a) Draw a neat, thin and accurate line to show the path of the ray from O to A in the water.
Complete the path, in air, of the emerging ray along AP1P2. [3]

(b) Produce the line P2P1A backwards so as to cut the line LL . Label, with the letter I, the
I

point where the two lines cross. Point I is the position of the image of the pin O when it
is touching the side of the beaker. [2]

(c) Draw the line OA to represent a ray in water from O passing through A . Using the
I I

information you gained in (b), draw a line to show the path of the ray in air after it passes
through the point A . Mark your diagram in such a way as to show how you found the
I

direction of the ray in air. [1]

(d) Take measurements to calculate the following ratio.

IR : OC = …… : 1

Record your measurements and show your working.

IR : OC = …… : 1 [2]

0625/6 S00
3 For
Examiner’s
Use

glass beaker filled with water

P2
P1
A

L pin R L'
O C

A'

Fig. 1.1

0625/6 S00 [Turn over


4 For
Examiner’s
Use
2 When investigating the magnetic field due to a bar magnet, a student places the magnet on
topicturningeffect
topicturning
topiceffect
a sheet of paper as shown in Fig. 2.1, on page 5. The edge of the paper is placed so that it is
parallel to the direction of the Earth’s magnetic field. The bar magnet is then placed as
shown so that it is at right angles to the direction of the Earth’s magnetic field. (In Fig. 2.1,
the lines OX and OY are perpendicular to each other.) A small plotting compass is used to
investigate the magnetic field.

(a) It is found that there are positions where the small magnet in the plotting compass
points so that it is parallel to the line OX. Some of these positions are located and are
labelled A, B, C, D, E, F, G and H, as shown on Fig. 2.1. The positions shown in Fig. 2.1
also lie on straight lines that come from the centre of the bar magnet.
Describe how you would locate the position labelled A. Your answer should explain
(i) what you would do to help you judge when the small magnet in the plotting
compass is parallel to OX,
(ii) how you would ensure that the small magnet of the plotting compass is not sticking,
(iii) what you would do so as to mark the point A on the radial line,
(iv) how you would avoid making a parallax error when locating the point A.

...................................................................................................................................

...................................................................................................................................

...................................................................................................................................

...................................................................................................................................

...................................................................................................................................

...................................................................................................................................

...................................................................................................................................

...............................................................................................................................[5]
0625/6 S00
5 For
Examiner’s
Use
Y
80°
North
(magnetic) 60°
plotting
C compass

B
D

A
40°
E

20°
G

O X
N S

bar magnet

Fig. 2.1

(b) The plotting compass is at point C as shown in Fig. 2.1.


(i) Mark the plotting compass in such a way as to show which end of the small magnet
of the plotting compass is a North pole.
(ii) The compass is at point C. It is then moved along the radial line so that it is closer
to the bar magnet. Describe and explain what happens to the small magnet of the
plotting compass.

...................................................................................................................................

...................................................................................................................................

...................................................................................................................................

...............................................................................................................................[3]

0625/6 S00 [Turn over


6 For
Examiner’s
Use
3 Fig. 3.1 shows the top of a variable resistor that has a scale of resistance, which gives the
topicmagnet
resistance in use.

terminals

y 5
4 6
x

3 7
control knob

2 8

1 9

0 10
OHM

Fig. 3.1

(a) What range of values of resistance are available with this resistor?

range of values of resistance available = .................................. [1]

(b) On Fig. 3.1, draw a line representing the position of the pointer when the value of the
resistance in use is 6.3 Ω. [1]

(c) Between the numbers 3 and 4, there are two letters x and y.

(i) What is the resistance when the pointer is at x?

resistance at x = ..................................

(ii) What is the change in resistance when the pointer moves from x to y?

change in resistance = ..................................


[2]

(d) Draw the circuit symbol for a variable resistor.

[1]

0625/6 S00
7 For
Examiner’s
Use
(e) A student is asked to connect a circuit so that the current through a filament lamp can
be changed by using a variable resistor.

The student makes a mistake when connecting the circuit.

Fig. 3.2 represents the student’s wrongly connected circuit. (In this diagram the circuit
symbol is not used for the variable resistor.)

lamp

5
4 6

3 7

variable resistor
2 8

1 9

0
OHM
10 Ω

Fig. 3.2

When the variable resistor is varied from 10 Ω to 5 Ω, the change in the current is very
small.

What could the student do to obtain a larger change in the current when the variable
resistor is changed from 10 Ω to 5 Ω?

..........................................................................................................................................

......................................................................................................................................[1]

0625/6 S00 [Turn over


8 For
Examiner’s
Use
4 The apparatus shown in Fig. 4.1 is used in a heat experiment.
topicenergytransformations
topicenergy
topicapparentdepth
topictransformations

boiling water

plastic cup
cold water
metal block

HEATER

Fig. 4.1

A piece of metal at the boiling temperature of water is transferred to a mass of cold water.
Initially, the cold water is at a temperature of TC. The hot metal raises the temperature of this
water to TH. The rise in temperature, θ, is determined from the relation θ = TH – TC. The
experiment is repeated so as to obtain five sets of readings for different masses of cold
water.

(a) Draw up a table, for use in your laboratory notebook, in which you can record

m, the mass of cold water used,


TC, the temperature of the cold water,
TH, the maximum temperature reached by the cold water,
θ, the rise in temperature of the cold water.

[3]

(b) Fig. 4.2 on page 9 is a graph showing how θ varies with m, the mass of cold water used.

(i) Why has a smooth line been drawn through the points?

...................................................................................................................................

...................................................................................................................................

...................................................................................................................................

0625/6 S00
9 For
Examiner’s
Use

graph showing how θ varies with m


15
θ
°C 14

13

12 A

11

10

4
20 30 40 50 60 70
m
g

Fig. 4.2

(ii) The graph point that is labelled A does not lie on the graph line. (You can assume
that the graph line is correctly drawn.) Complete the following statements about the
value of θ and of m at the point A.

1. If the value of θ were .................... °C smaller, the point A would lie on the line.

2. If the value of m were .................... g smaller, the point A would lie on the line.

(iii) In (ii) above which is the most likely reason, 1 or 2, for the point A not being on the
line? Give a reason for your choice.

choice: Tick one box.

1.

2.

reason:

...................................................................................................................................

...............................................................................................................................[4]

0625/6 S00 [Turn over


10 For
Examiner’s
Use
5 The class is investigating the use of nichrome (resistance) wire instead of thin thread as part
topicresistance
of a simple pendulum. The apparatus is shown in Fig. 5.1.

clamp

nichrome wire

wire wrapped round a


cylindrical mass and
twisted back on itself

Fig. 5.1

Four tests are carried out.


Test A using very thin cotton thread for the suspension, (this thread is considered to
have a negligible diameter).
Tests B, C and D in which nichrome wires of different diameters, d, are used.

In each test the length of the pendulum is 30.0 cm. The period, T, is determined by obtaining
the total time, t, of a suitable number of oscillations. The period is given by T = t /N, where N
is the number of oscillations.

The table gives the measurements taken by the class.

test suspension d /mm N t /s T /s


A cotton thread negligible 50 54.8

B nichrome wire 0.31 50 53.4

C nichrome wire 0.56 50 50.3

D nichrome wire 0.91 50 43.3

(a) For each test, determine the value T and record it in the table. [1]

(b) Suggest why 50 oscillations are used.

......................................................................................................................................[1]

(c) (i) Plot a graph of T /s ( y-axis) against d /mm (x-axis ). Start the T /s axis at T /s = 0.7.
Draw a neat thin curved line through the four points.

(ii) Label each plotted point with the correct test letter A, B, C or D.

0625/6 S00
11 For
Examiner’s
Use
(iii) Describe how the values of T change when the values of d, the diameter of the
wire, decrease.

...................................................................................................................................

...............................................................................................................................[7]

(d) In the laboratory you have enough time to take another set of measurements for one
other value for the diameter of the nichrome wire. Study the shape of your graph line
and then suggest an approximate value for the diameter that you think should be used.
Give a reason for your choice.

choice for the value of d = ...................... mm

reason for this choice

..........................................................................................................................................

..........................................................................................................................................

......................................................................................................................................[2]

0625/6 S00
12

BLANK PAGE

0625/6 S00
Candidate
Centre Number Number

Candidate Name

International General Certificate of Secondary Education


UNIVERSITY OF CAMBRIDGE LOCAL EXAMINATIONS SYNDICATE

PHYSICS 0625/6
PAPER 6 Alternative to Practical
Monday 20 NOVEMBER 2000 Morning 1 hour

Candidates answer on the question paper.


Additional materials:
Electronic calculator and/or Mathematical tables
Ruler (30 cm)

TIME 1 hour

INSTRUCTIONS TO CANDIDATES
Write your name, Centre number and candidate number in the spaces at the top of this page.
Answer all questions.
Write your answers in the spaces provided on the question paper.

INFORMATION FOR CANDIDATES


The number of marks is given in brackets [ ] at the end of each question or part question.
You may use a calculator.

FOR EXAMINER’S USE

TOTAL

This question paper consists of 12 printed pages.


SB (SM/DJ) QK07578/3
© UCLES 2000 [Turn over
2 For
Examiner’s
Use
1 Fig. 1.1 shows a metre rule clamped to a bench top. A mass of 50 g is attached to the free
topicspeed
end of the rule. The weight of the rule and of the 50 g mass depresses the free end of the
rule by a small distance, x, from the horizontal.

clamp
block of wood weight

x depression
metre rule

Fig. 1.1

In the experiment, the depression is found to be about 4 cm.

(a) Describe how you would determine the value of x to the nearest mm. Your answer should
include the following points.
(i) How you would locate the position of the horizontal line from which the depression
is measured.
(ii) What apparatus you would use to help you measure the depression.
(iii) What practical steps you would take to improve the accuracy of your measurement.

You may draw diagrams if you wish.

..........................................................................................................................................

..........................................................................................................................................

..........................................................................................................................................

..........................................................................................................................................

......................................................................................................................................[4]

0625/6 Nov00
3 For
Examiner’s
Use
(b) The free end of the loaded rule is now pushed down so that the depression is greater
than x, and then the rule is released. The rule performs vertical oscillations. How would
you determine the time, T, for one oscillation?
Your answer should include the following points.
(i) What measurements you would make and how you would use these measurements
to determineT.
(ii) What equipment you would use to make the measurements.
(iii) How you would avoid making errors.
(iv) How you would try to reduce errors.

...................................................................................................................................

...................................................................................................................................

...................................................................................................................................

...................................................................................................................................

...................................................................................................................................

...................................................................................................................................

...................................................................................................................................

[4]

0625/6 Nov00 [Turn over


4 For
Examiner’s
Use
2 The class is using a beaker filled with water to investigate one of the effects produced by a
topictransfer topicthermal topicenergy
cylindrical water lens. Fig. 2.1 shows the apparatus.

patch of light
beaker of water

I
sheet of paper
w width of patch of light

x distance from the


centre of the beaker lamp

Fig. 2.1

A large sheet of paper is placed on the bench. A straight line is drawn along the centre of the
sheet.
A beaker is placed with its centre on the straight line. The beaker of water acts as a
cylindrical lens. A small lamp is placed on the line. The lamp acts as a bright object. The light
that emerges on the other side of the lens produces a patch of light on the sheet of paper.
The broken lines mark the edge of this patch of light. The width of the patch of light
decreases and shows a sharp focus at the point I. At different points along the central line
the width, w, of the patch of light is determined. The investigation is to discover how w
depends upon x, the distance from the centre of the beaker.
Fig. 2.2 is a full size copy of the patch of light obtained in one experiment.

(a) On Fig. 2.2, at the points denoted by X1, X2, X3 and X4, take measurements of the
width, w, of the patch of light and the distance, x, from the centre of the lens. Record the
values for w and x in a table.
Determine the values of the product wx, and include these values in your table.

[5]

(b) Describe how the values of w change as the values of x increase.

..........................................................................................................................................

..........................................................................................................................................

......................................................................................................................................[2]

0625/6 Nov00
5 For
Examiner’s
Use

X4 X3
X2
X1
centre of beaker

Fig. 2.2

0625/6 Nov00 [Turn over


6 For
Examiner’s
Use
3 In this experiment, the class is investigating how the thermal energy lost by a 50 g mass of
topicmolecularmodels
topicmolecular
topicmodels
topiclens
brass, initially at 100 °C, depends upon the time it is cooling. The apparatus is shown in
Fig. 3.1.

boiling water

plastic cup
cold water
50 g brass cooling for t seconds

HEATER

Fig. 3.1

A 50 g mass of brass is heated in boiling water for at least five minutes. The mass is then
removed from the boiling water and given a very quick shake to remove any hot water
remaining on the mass. A time t, after the brass has been removed from the boiling water,
when the temperature of the brass has cooled to θH, the 50 g mass is placed into 50 cm3 of
cold water. The initial temperature of the cold water is θC. The temperature of the cold water
rises and the maximum temperature θM is determined.
θH is then calculated using the following equation.

θH = 11θM – 10θC

The experiment is repeated for different values of cooling time t.


Fig. 3.2 is a graph of a set of typical results for this experiment.
100

90

H
°C
80

Y
X

70

60
0 10 20 30 40 50 60 70 80 90 100 110 120 130 140 150
t /s

Fig. 3.2
0625/6 Nov00
7 For
Examiner’s
Use
(a) Why is it important to remove any water remaining on the brass as soon as it is lifted
from the boiling water?

..........................................................................................................................................

......................................................................................................................................[1]

(b) You do not have a partner helping you with this experiment. How would you measure
the cooling time t, using the clock shown in Fig. 3.3?

.................................................................................... re-set
.................................................................................... 55
60
5
10
start
50
....................................................................................
45 15
55 60 5
.................................................................................... 40
50
45
10
15 20
40 20
35 30 25
stop
35 25
30
....................................................................................

................................................................................[2]
Fig. 3.3

(c) Describe how you would perform the experiment in order to obtain the values that will
enable you to plot the point labelled X on the graph.

..........................................................................................................................................

..........................................................................................................................................

..........................................................................................................................................

......................................................................................................................................[2]

(d) The point labelled Y does not lie close to the graph line and it is to be repeated.

(i) How would you ensure that the initial temperature of the cold water in the plastic
cup is the same as in the previous trial?

...................................................................................................................................

(ii) Measuring cylinders are not very accurate for measuring small volumes of water.
How would you obtain 50 cm3 of water with more accuracy than that obtained by
using a measuring cylinder?

...............................................................................................................................[2]

(e) Why is it a good procedure to stir the cold water after adding the brass mass?

......................................................................................................................................[1]

0625/6 Nov00 [Turn over


8 For
Examiner’s
Use
4 Fig. 4.1 is a series circuit in which a variable resistor is used so as to control the magnitude
topicevaporation
of the current in the circuit. The circuit is designed so as to obtain any value of current from
0.2 A to 2 A.

fixed resistor
V=2V V
R = 1Ω

Fig. 4.1

(a) (i) The variable resistor is marked “0 to 10 Ω”.


What is meant by the phrase “0 to 10 Ω”?

...............................................................................................................................[1]

(ii) Why is it important that the value of the variable resistance may be changed
smoothly?

...............................................................................................................................[1]

(b) (i) A 1 m length of nichrome wire has a resistance of 10.0 Ω.


How would you use 1 m of this wire, and a jockey-slide contact, as the variable
resistor shown in Fig. 4.1?
Your answer should
1. include a diagram showing the wire in use,
2. explain how you would achieve smooth changes in the value of the variable
resistance,
3. explain why the wire must be bare and clean.

Diagram

0625/6 Nov00
9 For
Examiner’s
Use

...................................................................................................................................

...................................................................................................................................

...................................................................................................................................

...................................................................................................................................

...............................................................................................................................[3]

(ii) If the current in the nichrome wire becomes 2.0 A, then the wire becomes very hot
and has a temperature of about 300 °C. The wire is then dangerous to touch.
A safe current to use in the circuit is about 0.6 A. To obtain a current of 0.6 A, the
total resistance in the circuit should be about 3.3 Ω. The length of resistance wire
in use is then 23 cm.
What could you do to the apparatus you have been given in (b)(i) to prevent
anyone using a length of resistance wire that is less than 23 cm?
You may draw a diagram if you wish.

...................................................................................................................................

...................................................................................................................................

...................................................................................................................................

...............................................................................................................................[2]

0625/6 Nov00 [Turn over


10 For
Examiner’s
Use
5 A bar magnet A is placed on a waxed cardboard disc which floats on water as shown in
topicspeed
topicsound
topicspeedofsoundandlight
Figs. 5.1 and 5.2. At first, magnet A points in the North-South direction. A similar magnet, B,
is moved along the EW line as shown. As magnet B approaches magnet A, magnet A is
deflected. The deflection, y, is measured. An experiment is performed to find out how y
depends upon x, the distance between the centre of A and the centre of B.

magnetic N

waxed cardboard
disc floating on water

W E

A B
N
N S
S

cylindrical glass x
vessel
S
Fig. 5.1

disc A
N
B wood
S N
water S
0 5 6
1 2 3 4

scale y

x
S

Fig. 5.2

The data obtained from the experiment are given in the table.

deflection y / mm 3 8 16 40.5 74.5

separation x / mm 600 400 300 200 150

0625/6 Nov00
11 For
Examiner’s
Use
(a) Plot a graph of y /mm (y-axis) against x /mm (x-axis). Draw a smooth curve through the
points. [5]

(b) (i) Use the graph to complete the table below.

x / mm y / mm
x1 170 y1

x2 600 y2

x1/x2 y1/y2

0625/6 Nov00 [Turn over


12 For
Examiner’s
Use
(ii) Given that an increase in y means an increase in the force between the magnets,
describe how the force changes as the separation, x, decreases. Your answer
should include a comment on the magnitude of the two ratios that you have
calculated in part (b) (i).

...................................................................................................................................

...................................................................................................................................

...................................................................................................................................

...............................................................................................................................[5]

0625/6 Nov00
Candidate
Centre Number Number

Candidate Name

International General Certificate of Secondary Education


UNIVERSITY OF CAMBRIDGE LOCAL EXAMINATIONS SYNDICATE

PHYSICS 0625/6
PAPER 6 Alternative to Practical
MAY/JUNE SESSION 2001 1 hour

Candidates answer on the question paper.


No additional materials required.

TIME 1 hour

INSTRUCTIONS TO CANDIDATES

Write your name, Centre number and candidate number in the spaces at the top of this page.
Answer all questions.
Write your answers in the spaces provided on the question paper.

INFORMATION FOR CANDIDATES

The number of marks is given in brackets [ ] at the end of each question or part question.
You may use a calculator.

FOR EXAMINER’S USE

TOTAL

This question paper consists of 12 printed pages.


SB (ND/CG) QF10313/2
© UCLES 2001 [Turn over
2 For
Examiner’s
Use
1 (a) Fig. 1.1 shows the apparatus used for an experiment to investigate the extension of a
topicspeed
steel spring. The laboratory bench to which the pulley and support for the spring are
firmly fixed is not shown.

pointer
spring
0 10
20
180 170 16 30
01
50 40
14
0

5 0 120
13

60
0

70
110
protractor

80 90 100 110
100 80 7
pulley

0 6 0 13
0
12
50
40

0
14 30
0 20
15 10 0
01
60 1
70 180

load

Fig. 1.1

The load is attached to the spring by a length of thread which passes over a pulley. The
protractor is fixed in position. It is assumed that the angle through which the pointer
moves when the load is increased is proportional to the extension of the spring.

A student recorded the angle θ through which the pointer moved for a range of loads, L,
as shown in the table.

L /N θ / °

0 0
1 3
2 14
3 27
4 43
5 57

(i) Plot the graph of θ / ° (y-axis) against L / N (x-axis).

Draw the best fit straight line for loads from 1 N to 5 N. [6]

0625/6/M/J/01
3 For
Examiner’s
Use

(ii) The student expected the graph to show a straight line through the origin. Suggest
a reason why the results were not as expected.

.................................................................................................................................

.............................................................................................................................[1]

0625/6/M/J/01 [Turn over


4 For
Examiner’s
Use
(b) Another student obtained results using the apparatus set up as shown in Figs. 1.2 and
1.3 which show the spring unloaded and the spring with a load of 1 N.

rigid support
rigid
support

load

Fig. 1.2 Fig. 1.3

(i) On Fig. 1.2, show clearly lo, the length of the unloaded spring.

(ii) On Fig. 1.3, show clearly l1, the corresponding length of the spring with a load of
1 N.

(iii) Write down the equation you would use to calculate e, the extension produced by
adding the 1 N load.

e =.........................
[2]

0625/6/M/J/01
5 For
Examiner’s
Use
2 Some students were asked to carry out a simple experiment to compare different heat
insulation materials.
topictransfer topicthermal topicenergy

(a) One student measured the temperature of hot water in insulated beakers (all the same
size), waited for 5 minutes for the water to cool and then measured the temperatures
again. Fig. 2.1 shows how one student recorded the results.

80° C start 85° C start


73° C after 69° C after
200cm³ 5 minutes 170cm³ 5 minutes
water water

A B

79° C start
67° C after
150cm³ 5 minutes
water

Fig. 2.1

(i) Calculate the temperature fall for each beaker.

beaker A ..........................................................

beaker B ..........................................................

beaker C ..........................................................

If you had only these results and no information about the way these results had
been obtained, which beaker would appear to be the best insulated?

.............................................................

(ii) Suggest a simple, practical way to overcome the problem of heat loss, by
evaporation and convection, from the surface of the water in the beaker.

.................................................................................................................................

(iii) Look at Fig. 2.1 again. Suggest one further improvement that you would make to
improve the reliability of the experiment.

.................................................................................................................................

.................................................................................................................................
[5]

0625/6/M/J/01 [Turn over


6 For
Examiner’s
Use

(b) Another student carried out a similar experiment, with proper control of the variables,
and took temperature readings every 4 minutes. Room temperature during the
experiment was 19 °C. He plotted a graph of temperature against time to show the
cooling of the water in each beaker. Fig. 2.2 shows the graph obtained.
100

90

80

70

60

Temp/°C 50
A
40
B
30
C
20

10

0
10 20 30 40 50 60
Time/Min

Fig. 2.2

(i) From the graph, which beaker, A, B or C, was best insulated ? .............................

(ii) The student extended graph line B with a dotted line as shown. Explain why this
does not show a realistic continuation of the cooling of the water.

.................................................................................................................................

(iii) On the graph, extend line A to show a realistic result up to 60 minutes.


[3]

0625/6/M/J/01
7 For
Examiner’s
Use
3 Fig. 3.1 shows a circuit in which three resistors are connected to a d.c. power supply.
topicmagnet

power
source

R1 R2 R3

V1 V2 V3

Fig. 3.1

A 0 - 1 V voltmeter was used to measure the potential differences V1, V2 and V3.

Fig. 3.2 represents the face of the voltmeter when reading these values.

VOLT VOLT VOLT


0.4 0.6 0.8 0.4 0.6 0.8 0.4 0.6 0.8
0.2 0.2 0.2
0 1 0 1 0 1

V1 V2 V3

Fig. 3.2

(a) (i) Record the reading of each potential difference, shown in Fig. 3.2, in the table
below.

potential V/V
difference

V1

V2

V3

(ii) Using the values in your table, predict the voltmeter reading when a 0 - 5 V
voltmeter is connected across all three resistors together.

potential difference = ...............................................................................................


[5]

0625/6/M/J/01 [Turn over


8 For
Examiner’s
Use
(iii) The current, I, in the circuit is 0.35 A. Using the values in your table and the
equation R = V / I, calculate the resistances R1, R2 and R3

R1 = .................................................

R2 = .................................................

R3 = .................................................
[2]

(b) In the space below, draw a circuit diagram showing the same components as in Fig. 3.1
but with

• the three resistors in parallel,

• a voltmeter connected to record the potential difference across all three resistors

• the ammeter connected to record the current through R1 only.

[3]

0625/6/M/J/01
9 For
Examiner’s
Use
4 Fig. 4.1 shows the apparatus used to determine the maximum temperature rise of cold
topictensiontopicdispersion
water when a hot glass stopper is transferred to the cold water.
topicevaporation
topicresistance

string thermometer

boiling
water
at 100 °C
cold
glass water
stopper at 20 °C

heater

Fig. 4.1

(a) On Fig. 4.2, draw the mercury thread of the thermometer when it shows the
temperature of the cold water in the plastic cup shown in Fig. 4.1.

°C
-10 0 10 20 30 40 50

Fig. 4.2
[1]

(b) The glass stopper was heated for a long time in the boiling water. Suggest a reason for
this.

.....................................................................................................................................[1]

(c) Give one reason why the glass stopper should be transferred quickly from the boiling
water to the cold water.

.....................................................................................................................................[1]

(d) The maximum temperature that the cold water reached after the hot glass stopper was
transferred to the cold water was 30.5 °C.

Calculate

(i) the temperature rise of the cold water,

.................................................................................................................................

0625/6/M/J/01 [Turn over


10 For
Examiner’s
Use
(ii) the temperature fall of the glass stopper.

.................................................................................................................................
[2]

(e) When the experiment is repeated with a larger glass stopper, the temperature rise of
the cold water was greater than with the smaller glass stopper. Why was this?

.....................................................................................................................................[1]

0625/6/M/J/01
11 For
Examiner’s
Use
5 Fig. 5.1 shows an illuminated object, a lens and a screen set up for an experiment to
topicthermometer
topicradioactivity
topicpressure
topicconvection

investigate the size of the image produced by the lens. The lens is mounted on a rectangular
wooden block.

lens
u v
stiff card
lamp illuminated screen
object

wooden block

Fig. 5.1

Fig. 5.2 shows the shape and height (x cm) of the object.

stiff card

thin wire

triangular hole in
the card is the illuminated
object

Fig. 5.2

(a) A student carried out the experiment, keeping the lens in the same position throughout.
He set the object at a distance u = 15.0 cm from the centre of the lens, moved the
screen until the image was sharply focused and then measured v, the distance from the
centre of the lens to the screen. Finally, he measured y, the height of the image on the
screen. He repeated the procedure using different values of u. The table below shows
the readings he obtained.

u / cm v / cm x / cm y / cm y / cm (by calculation)

15.0 30.0 1.5 2.9


20.0 19.6 1.5 1.6
25.0 16.8 1.5 1.1

0625/6/M/J/01 [Turn over


12 For
Examiner’s
Use
The height of the image can be found by calculation, using the equation below.

y=vxx
u

Calculate the y values in this way and enter them in the table, giving the values to an
appropriate number of significant figures for comparison with the measured values. [2]

(b) Describe how you would measure as accurately as possible from the centre of the lens
to the screen. You may draw a diagram and assume that a metre rule is available and
other simple laboratory apparatus (e.g. set square and fine line marker pen).

.........................................................................................................................................

.........................................................................................................................................

.....................................................................................................................................[2]

(c) In Fig. 5.3 below, draw a full-size diagram of the image formed on the screen when
u = 15.0 cm. (Use the student’s measured value, not the calculated value.)

[2]
Fig. 5.3

(d) State one precaution you would take when setting up the apparatus in order to obtain a
clear image on the screen.

.........................................................................................................................................

.........................................................................................................................................

.....................................................................................................................................[1]
0625/6/M/J/01
Candidate
Centre Number Number

Candidate Name

International General Certificate of Secondary Education


UNIVERSITY OF CAMBRIDGE LOCAL EXAMINATIONS SYNDICATE
PHYSICS 0625/6
PAPER 6 Alternative to Practical
OCTOBER/NOVEMBER SESSION 2001 1 hour

Candidates answer on the question paper.


No additional materials required.

TIME 1 hour

INSTRUCTIONS TO CANDIDATES
Write your name, Centre number and candidate number in the spaces at the top of this page.
Answer all questions.
Write your answers in the spaces provided on the question paper.

INFORMATION FOR CANDIDATES


The number of marks is given in brackets [ ] at the end of each question or part question.

FOR EXAMINER’S USE

TOTAL

This question paper consists of 11 printed pages and 1 blank page.


SB (SLC/SLC) S11938/5
© UCLES 2001 [Turn over
2 For
Examiner’s
Use
1 Fig. 1.1 shows a strip of wood approximately 1 m long, clamped at each end to rigid supports
topicspeed
so that the strip is horizontal and about 50 cm above the floor. A small loop of string is fixed
at the centre of the strip.

clamp wooden strip clamp

load, L

Fig. 1.1

A student investigates how the bending of the strip of wood depends on the load hung from
the loop of string. The depression d of the strip at the centre is measured for each load L.
The readings obtained are shown in the table below.

L / N d / mm
0.0 0.0
1.0 11.5
2.0 23.0
3.0 34.0
4.0 46.0
5.0 57.5

(a) (i) Plot a graph of d / mm (y-axis) against L / N (x-axis). Draw the line of best fit. [6]

(ii) From the graph, determine the depression that would be produced by a load of
2.5 N. Show clearly on the graph how you obtained the necessary information.

depression produced by a load of 2.5 N = ....................................................... [2]

(b) Draw a diagram in the space below to show how you would measure the depression
produced by a load. You may assume that simple standard laboratory apparatus is
available.

[2]
0625/6/O/N/01
3 For
Examiner’s
Use

0625/6/O/N/01 [Turn over


4 For
Examiner’s
Use
2 Fig. 2.1 shows a circuit in which lamps are connected to a d.c. power supply.
topictransfer topicthermal topicenergy

d.c. power
supply

Fig. 2.1

Three lamps B, C and D are available and are connected in the circuit in turn. Each time the
potential difference V across the lamp and the current I through the lamp are measured. A
0 – 0.5 A ammeter is used to measure the current. Fig. 2.2 represents the face of the
ammeter when reading the current values.

AMP AMP
0.2 0.3 0.4 0.2 0.3 0.4
0 .1 0 .1
0 0.5 0 0.5

reading with lamp B reading with lamp C

AMP
0 2 0.3
. 0.4
0 .1
0 0.5

reading with lamp D

Fig. 2.2

0625/6/O/N/01
5 For
Examiner’s
Use
(a) (i) In the table below, record each ammeter reading shown in Fig. 2.2.

lamp V/V I/A


B 2.0

C 2.1

D 1.9
[3]

(ii) Using the values in the table and the equation R = V / I, calculate the resistance of
each lamp.

resistance of lamp B = ...........................................................

resistance of lamp C = ...........................................................

resistance of lamp D = ........................................................... [2]

(b) In the space below, draw a new circuit diagram showing


the power source and switch,
the three lamps in parallel,
the voltmeter connected to measure the voltage across the lamps,
the ammeter connected to measure the current through lamp C.

[3]

0625/6/O/N/01 [Turn over


6 For
Examiner’s
Use
3 Fig. 3.1 shows a pile of 15 glass microscope slides, drawn actual size.
topicmolecularmodels
topicmolecular

Fig. 3.1

(a) (i) On Fig. 3.1, use your rule to measure h, the height in mm of the pile of slides.

h = .............................................................

(ii) Calculate the thickness t of one slide.

t = ..............................................................
[2]

(iii) The surface area A of each slide is 1200 mm2. Calculate the volume V of one slide
using the equation
V = A x t.

V = .............................................................. [2]

(iv) The mass of one microscope slide is 3.7 g. Calculate the density d of the glass, in
g/mm3, using the equation
d = m /V.
Give your answer to an appropriate number of significant figures.

d = ............................................................. [2]

0625/6/O/N/01
7 For
Examiner’s
Use
(b) Fig. 3.2 shows the pile of microscope slides being used in a balancing experiment. The
metre rule is to be balanced, at the 50.0 cm mark, on the pivot. The pile of slides has a
mass of 55.5 g and is placed with its centre of mass on the rule at a distance 20.0 cm
from the pivot.

On Fig. 3.2, show clearly the approximate position of a 100 g mass placed on the rule
to make it balance. (You are not expected to carry out a calculation.)

microscope slides
metre rule

pivot

Fig. 3.2
[2]

0625/6/O/N/01 [Turn over


8 For
Examiner’s
Use
4 Fig. 4.1 shows three metal cans A, B and C. The cans are all drawn to one quarter actual
topicenergytransformations
topictransformations
topicenergy
topicapparentdepth
size.

A B C

Fig. 4.1

Each can contains water at room temperature. The outer surfaces of the cans are as follows:

A – painted white
B – painted black
C – polished metal (no paint).

(a) A student carries out an experiment to investigate how the colour of the surface affects
the rate of heating of the water when the cans are placed close to a radiant heater. Her
readings are shown below.

can initial water final water heating temperature


temperature/°C temperature/°C time/min change/°C
A 20 55 20
B 20 43 23
C 20 52 18

(i) Calculate the temperature change for each can and record the values in the table
above. [1]

(ii) The student realises that the variables have not been controlled, so that it is not
possible to draw reliable conclusions about the effect of the colour of the surface
on the rate of heating.

Suggest two changes you would make to obtain a more reliable set of readings.

1. ...............................................................................................................................

2. ...........................................................................................................................[2]

0625/6/O/N/01
9 For
Examiner’s
Use
(b) Another student carries out a similar experiment, with proper control of the variables,
and takes temperature readings every 60 s. All the cans reach a steady temperature
within 25 minutes of the start of timing. She plots a graph of temperature against time to
show the heating of the water in each can. Fig. 4.2 shows the graph obtained.

30 Q

R
temperature / °C

20

10
0 5 10 15 20 25
time/min

Fig. 4.2

(i) From the graph, which can, P, Q or R, reached its final temperature most quickly?

.................... [1]

(ii) Using the graph, determine the temperature of the water at the start of the
experiment. Show clearly on the graph how you obtained the necessary information.

temperature = .................... [2]

0625/6/O/N/01 [Turn over


10 For
Examiner’s
Use
5 Fig. 5.1 shows the outline of a transparent perspex block used in an experiment to
topicresistance
determine the value of a quantity called the refractive index of perspex. The block is placed
on a sheet of plain paper that is mounted on a cork mat. Two pins P1 and P2 are viewed
through the block and two additional pins P3 and P4 are stuck into the mat so that P3, P4 and
the images of P1 and P2 appear to be exactly in line, one behind the other.

P1

P2
A E B

D N C

P3

P4

eye

Fig. 5.1
0625/6/O/N/01
11 For
Examiner’s
Use
(a) (i) On Fig. 5.1, draw a line through the positions of P3 and P4 and extend the line until
it meets CD. Mark this point as F.

Show the path of the ray of light through the block by joining point E to point F. [1]

(ii) Measure the angle i.

i = ........................................................... [1]

(b) Give two precautions that you would take to obtain an accurate path for the ray of light.
You should consider the positioning of the pins P3 and P4 and how they are viewed. You
may assume that simple laboratory apparatus is available, if required.

first precaution ..................................................................................................................

..........................................................................................................................................

second precaution ............................................................................................................

......................................................................................................................................[2]

(c) (i) Measure the distance NF.

NF = ..............................

(ii) Measure the distance EF.

EF = ...............................

(iii) Calculate the refractive index n of perspex using the equation


EF
n = 0.5 x ––– .
NF

n = .........................
[4]

0625/6/O/N/01
12

BLANK PAGE

0625/6/O/N/01
Candidate
Centre Number Number

Candidate Name

International General Certificate of Secondary Education


CAMBRIDGE INTERNATIONAL EXAMINATIONS

PHYSICS 0625/6
PAPER 6 Alternative to Practical
MAY/JUNE SESSION 2002
1 hour
Candidates answer on the question paper.
No additional materials required.

TIME 1 hour

INSTRUCTIONS TO CANDIDATES

Write your name, Centre number and candidate number in the spaces at the top of this page.
Answer all questions.
Write your answers in the spaces provided on the question paper.

INFORMATION FOR CANDIDATES

The number of marks is given in brackets [ ] at the end of each question or part question.

FOR EXAMINER’S USE

TOTAL

This question paper consists of 10 printed pages and 2 blank pages.


SP (SC/JB) S25730/5 UNIVERSITY of CAMBRIDGE
© CIE 2002 Local Examinations Syndicate [Turn over
2 For
Examiner’s
Use
1 Fig. 1.1 shows how a student used a rule to measure h, the height of a pile of wooden
blocks. The reading was inaccurate because the student’s practical technique was poor.

cm
30

20

eye
10

h 0

Fig. 1.1

(a) Complete Fig. 1.2 to show the correct method to read the height h using the metre rule.
[2]

Fig. 1.2

(b) (i) Use your rule to measure the height marked h on Fig. 1.2.

h = ................................................

(ii) Fig. 1.2 is drawn to scale. It is a tenth of actual size.

Calculate t, the actual thickness of one block of wood.

t = .................................................
[5]

0625/6/M/J/02
3 For
Examiner’s
Use
2 (a) A student set up a circuit in order to measure the current through a lamp when different
potential differences were applied. Fig. 2.1 below shows the diagram that the student
drew. The diagram is incomplete. A voltmeter and an ammeter are required.

power source

meter

Fig. 2.1

(i) Draw the appropriate symbol in the space labelled ‘meter’.


(ii) Draw the circuit symbol for the other meter and show it connected correctly to the
circuit.
[2]

(b) Fig. 2.2 shows the scale of the ammeter.

0.2 0.3
0.1 0.4

0 0.5
A

Fig. 2.2

(i) On Fig. 2.2, draw the position of the pointer when the ammeter reading is 0.46 A.

(ii) What is the range of the ammeter shown?

..............................................................................................................................[2]

(c) When the current is 0.46 A, the voltmeter reading is 6.0 V. Calculate the resistance of
the lamp filament, using the equation V = IR.

resistance = ..................[3]

0625/6/M/J/02 [Turn over


4 For
Examiner’s
Use
3 The IGCSE class was asked to investigate the solubility of sugar under different conditions.
They were reminded of the importance of carrying out a ‘fair test’ by controlling the possible
variables. One student decided to investigate the effect of temperature on the rate at which
the sugar dissolved.

For each test the student added water to some sugar in a beaker, stirred the mixture briefly,
and recorded ␪, its temperature, and t, the time taken for all the sugar to dissolve. The
readings are shown below.

temperature, ␪ / °C dissolving time, t/s

20 450

25 240

31 145

36 90

39 72

46 45

50 24

(a) List three possible variables that this student should keep constant throughout the
experiment.

.........................................................................................................................................

.........................................................................................................................................

.....................................................................................................................................[3]

0625/6/M/J/02
5 For
Examiner’s
Use
(b) Plot the graph of t/s (y-axis) against ␪ /°C (x-axis). Start both axes at zero. Draw the
best-fit curve.

[5]

(c) (i) When the temperature of the water is increased, what is the effect on the time
taken for sugar to dissolve?

..................................................................................................................................

..................................................................................................................................

(ii) Calculate the ratios dissolving time at 20°C ,


dissolving time at 30°C

ratio = ................................

dissolving time at 40°C .


dissolving time at 50°C

ratio = ................................

(iii) Use your answers from part (ii) to predict a possible value for the ratio

dissolving time at 50°C .


dissolving time at 60°C

ratio = ............................[4]

0625/6/M/J/02 [Turn over


6 For
Examiner’s
Use
4 A student carried out an experiment to find the position of an image in a plane mirror. The
object was an optics pin. The arrangement is shown in Fig. 4.1.

sheet of
plain paper
mirror

cork mat
P
B

Fig. 4.1

The student viewed the image of the object pin P in the mirror. He placed two pins A and B
some distance apart so that the image of P and pins A and B were exactly in line, one
behind the other.

Then, without moving the object pin P, he viewed the image from a different position
and repeated the experiment with pins C and D. The student’s ray trace sheet is shown in
Fig. 4.2.

mirror

C
A

P
D

Fig. 4.2

0625/6/M/J/02
7 For
Examiner’s
Use
(a) On Fig. 4.2

(i) draw in the two reflected rays,


(ii) draw in the two incident rays that produced the reflected rays you have drawn,
(iii) show clearly, on the incident and reflected rays, the direction in which the light is
travelling,
(iv) find the position of the image of the object pin P by using the directions of the
reflected rays. Show clearly on the diagram how you found the image position.
Label the image position I. [6]

(b) State whether the image is real or virtual.

.........................................................................................................................................

Justify your answer by reference to the lines you have drawn in part (a).

.........................................................................................................................................

.....................................................................................................................................[2]

0625/6/M/J/02 [Turn over


8

BLANK PAGE

0625/6/M/J/02
9 For
Examiner’s
Use
5 In an experiment to study the effect of increasing pressure on the volume of air, the IGCSE
class used the apparatus shown in Fig. 5.1.

scale
air

oil pressure gauge

to foot pump

Fig. 5.1

air

tube
20 cm3

30 cm3
oil

Fig. 5.2

(a) What is the volume reading shown in Fig. 5.2?

volume reading = ............................[1]

0625/6/M/J/02 [Turn over


10 For
Examiner’s
Use

220

p / kPa

200

180

160

140

120

100
0 10 20 30 40 50
V / cm3

Fig. 5.3

Fig. 5.3 shows the graph that one student plotted from the readings. She drew a best-fit
curve.
Theory suggests that the relationship between pressure and volume is given by the
equation

p x V = constant.

The student is required to find the value of the constant.

(b) Why is it better to find the value of the constant using the graph than from a single
measurement of p and V?

.........................................................................................................................................

.....................................................................................................................................[1]

0625/6/M/J/02
11

(c) (i) Use these two examples, taken from the graph, to show that the readings from the
experiment support the theory.

Example 1: when p = 200 kPa

V = ………cm3

pV = …………

Example 2: when V = 37 cm3

p = …….. kPa

pV = …………

(ii) Using your answers from (c)(i), predict the pressure required to reduce the volume to
18 cm3.

p = …………………. kPa
[4]

0625/6/M/J/02
Candidate
Centre Number Number

Candidate Name

International General Certificate of Secondary Education


CAMBRIDGE INTERNATIONAL EXAMINATIONS
PHYSICS 0625/6
PAPER 6 Alternative to Practical
OCTOBER/NOVEMBER SESSION 2002
1 hour
Candidates answer on the question paper.
No additional materials required.

TIME 1 hour

INSTRUCTIONS TO CANDIDATES
Write your name, Centre number and candidate number in the spaces at the top of this page.
Answer all questions.
Write your answers in the spaces provided on the question paper.

INFORMATION FOR CANDIDATES


The number of marks is given in brackets [ ] at the end of each question or part question.

FOR EXAMINER’S USE

TOTAL

This question paper consists of 11 printed pages and 1 blank page.


SP (NF/SLC) S37969/1
© CIE 2002 [Turn over
2 For
Examiner’s
Use
1 In an experiment to determine the volume of glass beads, a student used two different
methods.

Method 1

The student measured the combined diameters of some beads and then calculated the
volume of one bead. The end view of the apparatus used is shown in Fig. 1.1.

bead

wooden block

Fig. 1.1

Fig. 1.2 shows the side view of the same apparatus, drawn actual size.

Fig. 1.2

(a) (i) On Fig. 1.2, use your rule to measure the distance x, in cm.

...................................................................................................................................

(ii) Calculate d, the average diameter in cm of one glass bead. Show your working.

d = ............................................... cm

(iii) Calculate V, the volume of one glass bead using the equation
πd 3
V = ___ .
6

V = .................................................[6]

0625/6/O/N/02
3 For
Examiner’s
Use
Method 2

The student used a displacement method to determine the volume of a glass bead. Fig. 1.3
and Fig. 1.4 show how this was done.

measuring measuring
cm3 100 cylinder cm3 100 cylinder

80 80

60 60
water water
40 40

20 20
225 glass beads

Fig. 1.3 Fig. 1.4

(b) (i) Write down the values of the readings taken.

...................................................................................................................................

...................................................................................................................................

(ii) Calculate the volume of 225 glass beads.

volume = .........................................

(iii) Calculate V, the average volume of one glass bead.

V = .........................................
[3]

(c) Suggest which of the two methods will give the more accurate result for the volume of a
glass bead. Give a reason for your answer.

..........................................................................................................................................

..........................................................................................................................................

......................................................................................................................................[1]

0625/6/O/N/02 [Turn over


4 For
Examiner’s
Use
2 The IGCSE class is investigating electromagnets. The electromagnets are made by
wrapping insulated wire around a soft-iron core. The wire is connected to a power pack.
Fig. 2.1 shows the arrangement.

power
pack

soft-iron core

paper clip

Fig. 2.1

Two students studied how the number of paper clips that an electromagnet can hold up
depends on the potential difference across the coil.

(a) Complete Fig. 2.1 by adding a voltmeter, connected to measure the p.d. across the coil.
[2]

(b) Student A used the control on the power pack to obtain set values of p.d. and recorded
the maximum number of paper clips that the electromagnet could hold at each p.d. The
results are shown below.

Student A

p.d. / V number of paper clips

0 0

2 0

4 1

6 2

8 3

10 4

12 5

0625/6/O/N/02
5 For
Examiner’s
Use
Student B connected a variable resistor into the circuit and used it to change the p.d.
across the coil. She recorded the minimum p.d. required to hold 1 paper clip, then 2
paper clips, etc. The results are shown below.

Student B

p.d. / V number of paper clips

0 0

2.2 1

4.5 2

6.6 3

8.7 4

11.0 5

(i) Which set of results gives the more accurate indication of the strength of the
electromagnet at different potential differences? Tick the correct box.

Student A

Student B

(ii) Justify your answer to part (b)(i).

...................................................................................................................................

...................................................................................................................................

...............................................................................................................................[2]

(c) Draw the circuit symbol for a variable resistor.

[1]

(d) On the diagram below, show the position of the pointer on the voltmeter when the
voltmeter reading is 8.7 V.

5 6 7
3 4 8
2
1 9
10

V
0

[1]

0625/6/O/N/02 [Turn over


6 For
Examiner’s
Use
3 A student carried out a ‘principle of moments’ experiment using a metre rule placed on a
pivot at the 50.0 cm mark. The aim was to determine an unknown weight. The arrangement
of the apparatus is shown in Fig. 3.1.

a b
unknown 50 cm 5.0 N weight
weight W mark

metre rule
pivot

Fig. 3.1

The student placed the unknown weight W at a convenient distance a from the pivot. He
found b, the distance from the pivot that the 5.0 N weight must be placed so that the rule
balanced horizontally. He then repeated the experiment using different values of a.
The readings are shown in the table below.

a/m b/m

0.100 0.122

0.200 0.238

0.250 0.302

0.300 0.360

0.350 0.435

0.400 0.470

0625/6/O/N/02
7 For
Examiner’s
Use
(a) (i) Plot the graph of b / m (y-axis) against a / m (x-axis).

(ii) Draw the best-fit straight line.

[6]

(iii) Determine G, the gradient of the line.

G = ....................................

(iv) Determine W, the unknown weight, using the equation

W = XG

where X = 5.0 N.

W = ....................................

0625/6/O/N/02 [Turn over


8 For
Examiner’s
Use
(v) Explain why the student could not choose distance a to have a value of 0.450 m.

...................................................................................................................................

...............................................................................................................................[5]

(b) Another student, who was performing this experiment, found that the unloaded metre
rule balanced on the pivot at the 50.3 cm mark, instead of the 50.0 cm mark.
Suggest what the student should do to obtain the correct value for W from the
experiment.

..........................................................................................................................................

......................................................................................................................................[1]

4 The IGCSE class was performing a heating experiment. The apparatus is shown in Fig. 4.1.
The aim was to determine the rate at which the temperature of 200 cm3 of water increased
when heated with an electric immersion heater.

thermometer

beaker cable to
power supply

electric
water immersion
heater

Fig. 4.1

0625/6/O/N/02
9 For
Examiner’s
Use
The water was heated from room temperature up to 60 °C. The teacher measured the power
of the immersion heater and calculated (correctly) the time required to raise the temperature
of 200 cm3 of water from 21 °C to 60 °C.
The students found that the water must be heated for longer than the calculated time.

(a) (i) What is the most likely cause of the longer time recorded?

Tick the appropriate box.

an inaccurate thermometer

errors in reading the stopwatch

heat loss during the experiment

(ii) Suggest two precautions that could be taken to obtain more accurate results.

...................................................................................................................................

...................................................................................................................................

...................................................................................................................................

...............................................................................................................................[3]

(b) What is the reading on the thermometer shown in Fig. 4.2?

–10 0 10 20 30 40 50 60 70 80 90 100 110 °C

reading = ........................ [1]


Fig. 4.2

(c) The power P of the immersion heater is calculated using the equation P = V I

Calculate the power of an immersion heater in which the current is 5.5 A when the p.d.
across it is 12.0 V.

..........................................................................................................................................

......................................................................................................................................[2]

0625/6/O/N/02 [Turn over


10 For
Examiner’s
Use
5 A student carried out a lens experiment to investigate the magnification of an image. The
apparatus is shown in Fig. 5.1.

u v

illuminated lens screen


object

Fig. 5.1

The object is a triangular hole in a screen. Fig. 5.2 shows this, actual size.

1.5 cm

2.0 cm

Fig. 5.2

The student set the distance u at 35.0 cm and moved the screen to obtain a sharply focused
image. The image distance v was 72.3 cm.

(a) (i) Calculate m, the magnification, using the equation

m = v /u.

m = .................................................

0625/6/O/N/02
11 For
Examiner’s
Use
(ii) Draw a diagram of the image, actual size, for a magnification m = 2.0.

[5]

(b) The image distance v is the distance from the screen to the centre of the lens.
Explain briefly how you would position a metre rule to obtain an accurate value for v.
You may draw a diagram.

..........................................................................................................................................

..........................................................................................................................................

......................................................................................................................................[1]

0625/6/O/N/02
Centre Number Candidate Number Name

CAMBRIDGE INTERNATIONAL EXAMINATIONS


International General Certificate of Secondary Education
PHYSICS 0625/06
Paper 6 Alternative to Practical
May/June 2003

1 hour
Candidates answer on the Question Paper.
No Additional Materials are required.

READ THESE INSTRUCTIONS FIRST

Write your Centre number, candidate number and name on all the work you hand in.
Write in dark blue or black pen in the spaces provided on the Question Paper.
You may use a soft pencil for any diagrams, graphs or rough working.
Do not use staples, paper clips, highlighters, glue or correction fluid.

Answer all questions.


At the end of the examination, fasten all your work securely together.
The number of marks is given in brackets [ ] at the end of each question or part question.

For Examiner’s Use


1

2
If you have been given a label, look at the
details. If any details are incorrect or 3
missing, please fill in your correct details
in the space given at the top of this page. 4

Stick your personal label here, if 5


provided.
Total

This document consists of 9 printed pages and 3 blank pages.


SP (AT/KS) S47471/3
© UCLES 2003 [Turn over
2 For
Examiner’s
Use
1 A student carried out an experiment to find the spring constant of a steel spring. The
apparatus is shown in Fig. 1.1.

l0
l

load W

half-metre half-metre
rule rule

Fig. 1.1

The student recorded the unstretched length l0 of the spring. Then she added loads W to the
spring, recording the new length l each time. The readings are shown in the table below.

W/N l / mm e / mm l0 = 30 mm

0 30

1 32

2 33

3 36

4 39

5 40

6 42

(a) Calculate the extension e of the spring produced by each load, using the equation

e = (l – l0).

Record the values of e in the table. [2]

0625/06/M/J/03
3 For
Examiner’s
Use
(b) Plot the graph of e / mm (y-axis) against W / N (x-axis). [4]

(c) Draw the best-fit straight line for the points you have plotted. Calculate the gradient of
the line. Show clearly on the graph how you obtained the necessary information.

gradient = ………………………………..[4]

0625/06/M/J/03 [Turn over


4 For
Examiner’s
Use
2 An IGCSE student was investigating the passage of red light through a prism. Fig. 2.1
shows the outline of the prism and an incident ray.

A
normal

i D

ray
box

B C

Fig. 2.1

(a) Measure the angle of incidence i shown in Fig. 2.1.

i = …………………………….[1]

(b) The angle of refraction as the ray entered side AB of the prism was 22°.

(i) On Fig. 2.1, draw in the refracted ray from point D as accurately as possible.
(ii) Mark the point E, where the ray meets side AC. Draw the normal at point E.
[4]

(c) At point E the ray came out of the prism with an angle of refraction of 75°. On Fig. 2.1,
draw as accurately as possible the ray coming out of the prism. [1]

(d) Another student used four optics pins to trace the passage of a ray through a prism.
Fig. 2.2 shows the prism, the position of the student’s eye and the directions of the ray.

incident ray

eye

Fig. 2.2

On Fig. 2.2, show positions of the four optics pins, placed to obtain as accurate a result
as possible. Mark each position clearly with a cross (X). [2]

0625/06/M/J/03
5 For
Examiner’s
Use
3 In an electrical experiment, a student set up a circuit to measure current and potential
difference. Part of the circuit is shown in Fig. 3.1.

power
source

Fig. 3.1

(a) (i) Complete the circuit diagram by drawing in a voltmeter connected across the lamp.
(ii) Name the component labelled Y. ………………………………………. [2]

(b) The first reading on the voltmeter was 2.2 V. On the voltmeter face shown in Fig. 3.2,
show the position of the pointer giving the reading 2.2 V.

4 5 6
3 7
2 8
1 9
0 10
V

[1]
Fig. 3.2

(c) The readings of V and I obtained by the student are given in the table below.

V/ I/ R/

2.2 0.36

4.1 0.62

6.0 0.86

7.9 0.98

9.8 1.20

(i) Calculate the resistance R of the lamp filament for each set of V and I readings
and write the values in the table. Use the equation

V
R= .
I
(ii) Complete the column headings in the table. [6]

0625/06/M/J/03 [Turn over


6 For
Examiner’s
Use
4 An IGCSE class watched a demonstration experiment to show that a metal rod expands
when heated. The apparatus is shown in Fig. 4.1.

pointer
metal rod
pin

wooden block
heat

bench

Fig. 4.1

When the rod expands, it rolls the pin which moves the pointer. So a very small expansion
moves the pointer far enough to be seen clearly.

(a) One student wanted to find out how much longer the rod became when heated above
room temperature with a Bunsen burner. The rod was 0.750 m long at room
temperature.
To find the circumference of the pin, the student wrapped a piece of string 10 times
round the pin, marked the string at the beginning and end of the 10 turns, and then
measured the length of the string between the marks. Fig. 4.2 shows the string actual
size.

Fig. 4.2

(i) Use your rule to measure the distance x between the marks on the string on
Fig. 4.2.

x = ………………………………..
(ii) Calculate the circumference c of the pin.

c = ………………………………..
[3]

0625/06/M/J/03
7 For
Examiner’s
Use
(b) A second student measured the diameter d of the pin using a micrometer screw gauge.

The diameter was 1.20 mm. When the rod was heated, the pointer moved through 90°.

(i) Calculate the circumference c using the equation

c = πd.

c = ………………………………..[2]

(ii) Use this value of the circumference to calculate the increase e in the length of the
rod when heated.

e = ………………………………..[1]

(iii) Calculate the length l of the heated rod.

l = ………………………………..[1]

(c) The micrometer screw gauge is a very accurate instrument. Suggest why the string and rule
method of finding the circumference, used by the first student, was inaccurate.

.................................................................................................................................................

.............................................................................................................................................[1]

0625/06/M/J/03 [Turn over


8 For
Examiner’s
Use
5 A student was asked to carry out an experiment to compare the insulating properties of
cotton wool, cardboard and polystyrene. The apparatus provided was hot water, a
thermometer, a stopclock and a copper can with a lid, as shown in Fig. 5.1.

thermometer

copper can lid

water

insulation

Fig. 5.1

The student wrapped one of the insulators around the can, poured hot water into the can,
and then took temperature and time readings as the water cooled. This was then repeated
for each insulator. The graph in Fig. 5.2 shows how the student displayed his readings.

80
temperature / °C polystyrene
60

40

cotton wool
20
cardboard
0
0 5 10 15
time / min

Fig. 5.2

(a) (i) Using the information on the graph, which material appears to be the best
insulator?

..................................................................................................................................

(ii) Justify your answer by referring to the information on the graph.

..................................................................................................................................

..................................................................................................................................
[2]

0625/06/M/J/03
9 For
Examiner’s
Use
(b) In this experiment, it is important to control the variables. Suggest three variables that
the student should keep constant for this experiment.

1. .....................................................................................................................................

2. .....................................................................................................................................

3. .....................................................................................................................................
[3]

0625/06/M/J/03
Centre Number Candidate Number Name

CAMBRIDGE INTERNATIONAL EXAMINATIONS


International General Certificate of Secondary Education
PHYSICS 0625/06
Paper 6 Alternative to Practical
October/November 2003

1 hour
Candidates answer on the Question Paper.
No Additional Materials are required.

READ THESE INSTRUCTIONS FIRST

Write your Centre number, candidate number and name on all the work you hand in.
Write in dark blue or black pen in the spaces provided on the Question Paper.
You may use a soft pencil for any diagrams, graphs or rough working.
Do not use staples, paper clips, highlighters, glue or correction fluid.

Answer all questions.


At the end of the examination, fasten all your work securely together.
The number of marks is given in brackets [ ] at the end of each question or part question.

For Examiner’s Use


1

2
If you have been given a label, look at the
details. If any details are incorrect or 3
missing, please fill in your correct details
in the space given at the top of this page. 4

Stick your personal label here, if 5


provided.
Total

This document consists of 11 printed pages and 1 blank page.


MML 4504 11/02 S53410/3 UNIVERSITY of CAMBRIDGE
© UCLES 2003 Local Examinations Syndicate [Turn over
2 For
Examiner’s
Use
1 (a) A student was asked to make some measurements of the test-tube shown in Fig. 1.1.
He was given a 1 m length of thin string and a metre rule and instructed to determine
as accurately as possible the circumference of the tube. The student used the rule to
measure the diameter d of the tube and then calculated the circumference c using the
equation

c = d.

Describe how the student could have obtained a more accurate result with the
apparatus given. You may draw on Fig. 1.1.

Fig. 1.1

..........................................................................................................................................

..........................................................................................................................................

.................................................................................................................................... [2]

(b) (i) On Fig. 1.2, show where you would place two small rectangular blocks of wood to
help you make an accurate measurement of the overall length of the test-tube.

Fig. 1.2

(ii) The test-tube is shown actual size in Fig. 1.2. Use your rule to measure the length l
of the test-tube.

l = ..............................................
[2]

0625/06/O/N/03
3 For
Examiner’s
Use
(c) Using another test-tube, the student obtained these readings.

l = 14.5 cm

c = 5.3 cm

Calculate the approximate external volume V of the test-tube using the student’s
readings and the equation

c 2l
V = –– .
4

V = .................................................... [2]

(d) The equation used in (c) assumes the test-tube to be a cylinder with flat ends. It does
not allow for the rounded end of the test-tube.
(i) Estimate the volume Vm of the ‘missing’ part of the cylinder shown shaded in
Fig. 1.3.

Fig. 1.3

Vm = ...........................................
(ii) Using your values for V and Vm , calculate the actual external volume Va of the
test-tube.

Va = ...........................................
[2]

0625/06/O/N/03 [Turn over


4 For
Examiner’s
Use
2 Fig. 2.1 shows a ray tracing sheet obtained by a student carrying out a reflection of light
experiment using pins and a plane mirror.

mirror

M M'
B C E

D F
A

Fig. 2.1

The student looks into the mirror MM' and views the images of pins A and B. He then places
pins C and D so that pins C and D and the images of pins A and B appear to be in line.

(a) On Fig. 2.1,


(i) draw the incident ray in this experiment,
(ii) draw the reflected ray,
(iii) draw a normal at the point where the incident ray meets the mirror,
(iv) measure the angle of incidence i.

i = ...............................................
[3]
0625/06/O/N/03
5 For
Examiner’s
Use
(b) The student moves pin B and then repeats the experiment, obtaining the reflected ray
EF.
(i) On Fig. 2.1, continue the lines CD and EF behind the mirror to find the point
where they meet. Label this point X.
(ii) Draw the line AX. Label with the letter Y the point where line AX crosses the
mirror MM'.
(iii) Use your rule to measure the distances AY and YX.

AY = ...........................................

YX = ...........................................
[2]

(c) According to theory, AY = YX. Suggest why, in spite of very careful work, the student’s
values may have been slightly different.

..........................................................................................................................................

.................................................................................................................................... [1]

0625/06/O/N/03 [Turn over


6 For
Examiner’s
Use
3 (a) Fig. 3.1 shows the scale of an ammeter. Draw the position of the pointer when the
ammeter reading is 0.35 A. [1]

0.4 0.6
0.2 0.8

0 A 1.0

Fig. 3.1

(b) The ammeter was used in the circuit shown in Fig. 3.2 to investigate the current in a
lamp.

Fig. 3.2

(i) Name the component labelled Y.

...................................................

0625/06/O/N/03
7 For
Examiner’s
Use
(ii) The table shows the current I in the lamp for different values of the p.d. V across
the lamp.

V/ I/ R/

1.9 0.31

1.5 0.26

0.8 0.20

1. Calculate the values for the resistance R of the lamp, using the equation
V
R= – .
I
Write your answers in the table.
2. Complete the column headings in the table.
(iii) Suggest how the value of V could be varied.

..................................................................................................................................

..................................................................................................................................
[8]

(c) Fig. 3.3 shows a power source connected to three resistors labelled X, Y and Z.

power
source

Fig. 3.3

Complete the diagram to show


(i) a voltmeter connected to measure the voltage across the resistors,
(ii) an ammeter connected to measure the current in resistor X only,
(iii) connecting wires to complete the circuit.
[3]
0625/06/O/N/03 [Turn over
8 For
Examiner’s
Use
4 The IGCSE class was studying the acceleration a of a toy truck that was pulled along a
track by a force F. The arrangement is shown in Fig. 4.1.

truck

Fig. 4.1

The results obtained are shown in the table.

F/N a / (m / s2 )

0.5 0.35

1.0 0.72

1.5 1.02

2.0 1.44

2.5 1.74

(a) Plot a graph of F / N (y-axis) against a / (m / s2 ) (x-axis). Draw the line of best fit through
your points. [6]

0625/06/O/N/03
9 For
Examiner’s
Use
(b) Theory suggests that the relationship between force and acceleration is given by the
equation

F = ma,

where m is the mass of the truck.

The gradient of the graph is equal to the mass of the truck.

From the graph, determine the mass m of the truck. Show clearly how you obtained
the necessary information.

m = .................................................... [4]

0625/06/O/N/03 [Turn over


10 For
Examiner’s
Use
5 A student wants to find out which of the three materials is the best thermal insulator.

The student’s apparatus at the beginning of each test is as shown in Fig. 5.1.

200 cm3 of 100 cm3 of


water at water at 100 cm3 of
80 °C 80 °C water at
70 °C

material 1 material 2 material 3


beaker A beaker B beaker C

Fig. 5.1

Each beaker is surrounded by a different insulating material. The water is allowed to cool
and the temperatures are recorded at different times. The student is unable to write a
correct conclusion because the variables have not been controlled.

(a) Study Fig. 5.1 and then state two ways in which you would improve the control of
variables.

1. ......................................................................................................................................

..........................................................................................................................................

2. ......................................................................................................................................

.................................................................................................................................... [2]

(b) State the reading shown on the thermometer shown in Fig. 5.2.

°C
110 100 90 80 70 60 50 40 30 20 10 0

Fig. 5.2

temperature reading .................................................................................................. [1]

0625/06/O/N/03
11 For
Examiner’s
Use
(c) The graph of Fig. 5.3 shows the results obtained by the student.

The graph lines A, B and C correspond to the beakers A, B and C.

90

80

70
temperature / °C

60

50

40 A

30 B

C
20

10

0
0 20 40 60 80 100 120 140 160 180 200

time / s

Fig. 5.3

Which beaker cools the most quickly in the first 60 s?

........................................................... [1]

0625/06/O/N/03
Centre Number Candidate Number Name

UNIVERSITY OF CAMBRIDGE INTERNATIONAL EXAMINATIONS


International General Certificate of Secondary Education
PHYSICS 0625/06
Paper 6 Alternative to Practical
May/June 2004

1 hour
Candidates answer on the Question Paper.
No Additional Materials are required.

READ THESE INSTRUCTIONS FIRST

Write your Centre number, candidate number and name on all the work you hand in.
Write in dark blue or black pen in the spaces provided on the Question Paper.
You may use a soft pencil for any diagrams, graphs or rough working.
Do not use staples, paper clips, highlighters, glue or correction fluid.

Answer all questions.


At the end of the examination, fasten all your work securely together.
The number of marks is given in brackets [ ] at the end of each question or part question.

For Examiner’s Use


1

2
If you have been given a label, look at the
details. If any details are incorrect or 3
missing, please fill in your correct details
in the space given at the top of this page. 4

Stick your personal label here, if 5


provided.
Total

This document consists of 11 printed pages and 1 blank page.


MML 5677 5/03 S61130/2
© UCLES 2004 [Turn over
2 For
Examiner’s
Use
1 The IGCSE class is investigating the conduction of electric current through copper sulphate
solution. The circuit used is shown in Fig. 1.1.

power source

A
V

electrode

copper sulphate
solution

Fig. 1.1

During the experimental work, the students measure the volume of water, the mass of
copper sulphate that is dissolved in the water, the current in the solution, the potential
difference across the electrodes and the gap between the electrodes.

One set of readings is shown in Figs. 1.2 – 1.6.

(a) Write down the readings shown. Include appropriate units.

1
0.5 1.0
0.5 1.5

A V
0 1.5 0 2

Fig. 1.2 Fig. 1.3

current = ............................................ potential difference = .....................................

copper
plastic sulphate empty plastic
dish dish

6.58 g 3.26 g

Fig. 1.4

mass of copper sulphate = .............................


© UCLES 2004 0625/06/M/J/04
3 For
Examiner’s
Use

cm3

200
180
160
140

water 120
100
80
60
40
20

Fig. 1.5

volume of water = ..........................................

electrode

0 cm 1 2 3 4 5 6 7 8 9 10

Fig. 1.6

gap between electrodes = ..............................


[6]
(b) It is not possible to put the rule inside the beaker to measure the gap between the
electrodes. Explain how you would overcome this problem.

..........................................................................................................................................

..........................................................................................................................................

..........................................................................................................................................

.................................................................................................................................... [2]

(c) Suggest a variable, which is not measured in Figs. 1.2 – 1.6, that might affect the value
of the current.

.................................................................................................................................... [1]
© UCLES 2004 0625/06/M/J/04 [Turn over
4 For
Examiner’s
Use
2 A student is investigating the oscillation of a metre rule that has one end resting on the
laboratory bench. The other end is held above the level of the bench by a spring attached
at the 90.0 cm mark. The arrangement is shown in Fig. 2.1.

clamp

spring

metre rule
d

bench

Fig. 2.1

The period of oscillation is changed by moving a 200 g mass to different positions along the
rule. The student records the time t taken for 10 oscillations of the end of the rule for each
position of the mass. He measures the distance d from the end of the rule to the mark
under the centre of the mass. The readings are shown in the table.

d / cm t/s T/s

20.0 3.4

40.0 4.4

50.0 4.9

60.0 5.3

70.0 6.0

80.0 6.3

(a) Calculate the period T for each set of readings and enter the values in the table. [2]

© UCLES 2004 0625/06/M/J/04


5 For
Examiner’s
Use
(b) Plot a graph of d / cm (x-axis) against T / s (y-axis). The scale on the x-axis has been
started for you. [5]

0 20 40 60 80

(c) Using the graph, determine the period T when the distance d is 55.0 cm.

T = ............................................................ [2]

(d) The student suggests that T should be proportional to d. State with a reason whether
your results support this suggestion.

statement .........................................................................................................................

reason ..............................................................................................................................

.................................................................................................................................... [2]

© UCLES 2004 0625/06/M/J/04 [Turn over


6 For
Examiner’s
Use
3 Fig. 3.1 shows the circuit that a student uses to find the resistance of a combination of three
lamps.

power
source

Fig. 3.1

The voltmeter and the ammeter have not been drawn in.

(a) Complete Fig. 3.1 by drawing in the voltmeter and the ammeter, using conventional
symbols. [2]

(b) The student obtains these readings.

current I = 0.54 A

potential difference V = 1.8 V


V
Calculate the resistance R using the equation R = – .
I

R = ........................................................... [2]

© UCLES 2004 0625/06/M/J/04


7 For
Examiner’s
Use
(c) The three lamps are now connected in parallel with one another. Draw a circuit
diagram of the three lamps connected to the power supply. Include in your circuit
diagram
(i) an ammeter to record the total current through the lamps,
(ii) a variable resistor to vary the brightness of all three lamps,
(iii) a voltmeter to record the potential difference across the lamps.
[3]

© UCLES 2004 0625/06/M/J/04 [Turn over


8 For
Examiner’s
Use
4 The IGCSE class carries out an experiment using a convex lens, an illuminated object and
a screen. Fig. 4.1 shows the apparatus. A sharp image is obtained on the screen.

illuminated
object
lens screen

lamp

card

Fig. 4.1

(a) (i) Use your rule to measure, on Fig. 4.1, the distance x from the illuminated object to
the centre of the lens.

x = ..................................................................
(ii) Use your rule to measure, on Fig. 4.1, the distance y from the centre of the lens to
the screen.

y = ..................................................................
(iii) Fig. 4.1 shows the apparatus drawn to 1/5th of actual size. Calculate the actual
distance u between the object and the lens, and the actual distance v between the
lens and the screen.

u = ..................................................................

v = ..................................................................
v
(iv) Calculate the magnification m using the equation m = – .
u

m = .................................................................
[5]

© UCLES 2004 0625/06/M/J/04


9 For
Examiner’s
Use
(b) The illuminated object is triangular in shape, as shown in Fig. 4.2.

Fig. 4.2

Draw a diagram of the image as it would appear on the screen.

[1]

(c) State two precautions that the IGCSE class should take to obtain experimental readings
that are as accurate as possible.

1. ......................................................................................................................................

..........................................................................................................................................

2. ......................................................................................................................................

.................................................................................................................................... [2]

© UCLES 2004 0625/06/M/J/04 [Turn over


10 For
Examiner’s
Use
5 In a heating experiment, a student takes the temperature of a beaker B containing water at
room temperature. Fig. 5.1 shows the thermometer used.

–10 0 10 20 30 40 50 60 70 80 90 100 110 °C

Fig. 5.1

(a) State the temperature reading shown on the thermometer.

temperature reading = ............................. [1]

(b) The student then transfers a small metal cylinder from beaker A of boiling water to the
beaker B of water at room temperature, as shown in Fig. 5.2.

string

metal water at
boiling cylinder room
water temperature

beaker A beaker B

Fig. 5.2

The student assumes that the metal is at a temperature of 100 °C when it enters the
water in beaker B.

The temperature of the water in beaker B rises to 36 °C.


(i) Calculate the temperature rise of the water in beaker B.

temperature rise = ..........................................


(ii) Calculate the temperature fall of the metal cylinder.

temperature fall = ...........................................


[3]

© UCLES 2004 0625/06/M/J/04


11 For
Examiner’s
Use
(c) The student uses these readings and some other information to calculate the specific
heat capacity of the metal.

Why is it important to transfer the metal between the beakers as quickly as possible?

..........................................................................................................................................

.................................................................................................................................... [1]

© UCLES 2004 0625/06/M/J/04


Centre Number Candidate Number Name

UNIVERSITY OF CAMBRIDGE INTERNATIONAL EXAMINATIONS


International General Certificate of Secondary Education
PHYSICS 0625/06
Paper 6 Alternative to Practical
October/November 2004

1 hour
Candidates answer on the Question Paper.
No Additional Materials are required.

READ THESE INSTRUCTIONS FIRST

Write your Centre number, candidate number and name on all the work you hand in.
Write in dark blue or black pen in the spaces provided on the Question Paper.
You may use a soft pencil for any diagrams, graphs or rough working.
Do not use staples, paper clips, highlighters, glue or correction fluid.

Answer all questions.


At the end of the examination, fasten all your work securely together.
The number of marks is given in brackets [ ] at the end of each question or part question.

For Examiner’s Use


1

2
If you have been given a label, look at the
details. If any details are incorrect or 3
missing, please fill in your correct details
in the space given at the top of this page. 4

Stick your personal label here, if 5


provided.
Total

This document consists of 12 printed pages.


MML 5677 5/03 S67109/2
© UCLES 2004 [Turn over
2 For
Examiner’s
Use
1 The IGCSE class is investigating the rate of cooling of water in a beaker. Some of the
apparatus used is shown in Fig. 1.1.

thermometer

string
beaker

hot water

Fig. 1.1

During the experiment, a student measures the temperature of the water, its volume, the
length of string wrapped round a beaker and the depth of water in the beaker.

(a) Write down the readings shown in Figs. 1.2 and 1.3. Include appropriate units.

–10 0 10 20 30 40 50 60 70 80 90 100 110 °C

Fig. 1.2

temperature = ............................

cm3
100

80

60

40

20

Fig. 1.3

volume of water in the measuring cylinder = ..................................... [3]

© UCLES 2004 0625/06/O/N/04


3 For
Examiner’s
Use
(b) The string is wrapped 5 times round the beaker and marked as shown in Fig. 1.4.

mark

string

mark beaker

Fig. 1.4

The string is held against a metre rule as shown in Fig. 1.5.

string

cm 10 20 30 40 50 60 70 80 90

metre rule

Fig. 1.5

(i) Write down the length of the string between the marks.

length = ................................ cm
(ii) Calculate the circumference c of the beaker.

c = ........................................ cm
(iii) Suggest one source of error in this method of determining the circumference.

..................................................................................................................................

..................................................................................................................................
(iv) Suggest one improvement to this method.

..................................................................................................................................

..................................................................................................................................
[4]

© UCLES 2004 0625/06/O/N/04 [Turn over


4 For
Examiner’s
Use
(c) A rule is placed beside the beaker, as shown in Fig. 1.6.

beaker 3

2
water
1

cm

Fig. 1.6

(i) Write down the depth d of the water in the beaker.

d = ........................................ cm
(ii) Calculate the surface area A of the curved surface of the beaker up to the water
level using the equation A = dc.

A = ..................................
[2]

(d) State the other measurements that need to be taken to determine the rate of cooling of
the water.

..........................................................................................................................................

.................................................................................................................................... [2]

© UCLES 2004 0625/06/O/N/04


5 For
Examiner’s
Use
2 A student carries out an experiment to determine the density of plasticine. She records the
mass m and the volume V of a range of differently-sized samples. These readings are
plotted on a graph as shown in Fig. 2.1.

40

V / cm3

30

20

10

0
0 20 40 60 80 100
m/g

Fig. 2.1

(a) (i) Determine the gradient G of the line. Show clearly how you obtain the necessary
information.

G = ...................................
1.
(ii) Determine the density  of the plasticine using the equation  = —
G

 = ....................................
[5]

(b) The student could calculate the density from one set of readings. Suggest why she
takes more than one set of readings and plots a graph.

..........................................................................................................................................

.................................................................................................................................... [1]

© UCLES 2004 0625/06/O/N/04 [Turn over


6 For
Examiner’s
Use
3 A student carries out an experiment using a simple pendulum. Fig. 3.1 shows the apparatus.

clamp

string
x

pendulum bob

Fig. 3.1

The student records the time t taken for 20 complete oscillations for a range of different
lengths x of the string. The readings are shown in the table.

x / cm l / cm t/s T/s
90.0 38.5

80.0 36.0

70.0 33.4
60.0 31.4

50.0 28.2

40.0 25.5

The length l of the pendulum is given by the equation l = x + r, where r is the radius of the
pendulum bob.

Fig. 3.2 shows the pendulum bob drawn actual size.

pendulum
bob

string

bench

Fig. 3.2

© UCLES 2004 0625/06/O/N/04


7 For
Examiner’s
Use
(a) (i) Use your rule to measure the diameter d of the pendulum bob.

d = ....................................
(ii) Calculate the radius r of the pendulum bob.

r = .....................................
[2]

(b) (i) Complete the column for the length l / cm in the table using the equation l = x + r.
(ii) The period T is the time taken for one complete oscillation. Complete the column
for the period T / s in the table.
[3]

(c) Plot the graph of T / s (y-axis) against l / cm (x-axis). Start the T / s axis at T = 1.0 s.

[5]

(d) Using the graph, find the length la of the pendulum that would have a period of 1.50 s.

la = ............................ cm [1]

© UCLES 2004 0625/06/O/N/04 [Turn over


8 For
Examiner’s
Use
4 A student is investigating the passage of light through a transparent block, as shown in
Fig. 4.1.

incident
ray

A
transparent
block

emergent
ray

eye

Fig. 4.1

© UCLES 2004 0625/06/O/N/04


9 For
Examiner’s
Use
The student looks through the block. He places pins so that two pins marking the incident
ray and two pins marking the emergent ray all appear to be exactly one behind the other.

(a) On Fig. 4.1, mark suitable positions for the four pins, two on the incident ray and two
on the emergent ray. [1]

(b) (i) On Fig. 4.1, draw the normal at point A.


(ii) On Fig. 4.1, draw in the line AB. Measure and record the angle of refraction r
between the line AB and the normal.

r = .....................................
(iii) Measure and record the angle of incidence i between the incident ray and the
normal.

i = .....................................
[4]

© UCLES 2004 0625/06/O/N/04 [Turn over


10 For
Examiner’s
Use
5 The IGCSE class is carrying out investigations of the resistance of bare resistance wires.
Fig. 5.1 shows the circuit used.

power
source

l
A B
bare
V resistance
wire

Fig. 5.1

The students record the current I in the circuit and then record the p.d. V across different
lengths l of the bare resistance wire. The length of wire from A to B is 100.0 cm. The
readings obtained by one student are shown in the table.

I = 0.84 A

V/ l/ R/

0.39 20.0

0.82 40.0
1.22 60.0

1.58 80.0

1.89 100.0

(a) (i) Calculate the resistance of each length l of wire using the equation R = V
—. Write
the resistance values in the table.
I

(ii) Complete the column headings in the table.


[3]

© UCLES 2004 0625/06/O/N/04


11 For
Examiner’s
Use
(b) In a second experiment, the students use wires of the same material but with different
diameters d. The p.d. is measured across the same length of wire each time. Fig. 5.2
shows the circuit used.

power
source

A B
bare
V resistance
wire

Fig. 5.2

These are the readings correctly obtained by one student.

Wire 1 I = 0.1 A Wire 2 I = 0.4 A


V = 1.8 V V = 1.8 V
d = 0.24 mm d = 0.48 mm
V.
(i) Calculate the resistance R of each wire, using the equation R = —
I

wire 1, R = ..........................................

wire 2, R = .......................................... [1]


(ii) Based on the results for the two wires, which of the following statements is a
correct conclusion? Tick one box.
A wire with half the diameter has half the resistance.

A wire with half the diameter has twice the resistance.

A wire with half the diameter has one quarter the resistance.

A wire with half the diameter has four times the resistance.

[1]

© UCLES 2004 0625/06/O/N/04 [Turn over


12 For
Examiner’s
Use
(iii) Using your answers to (i) and (ii), calculate the expected resistance of a wire AB
of the same material if it has a diameter of 0.12 mm.

expected resistance = .............................................................. [1]

(c) What instrument would you use to measure the diameter of the wires as accurately as
possible?

.................................................................................................................................... [1]

Copyright Acknowledgements

Every reasonable effort has been made to trace all copyright holders where the publishers (i.e. UCLES) are aware that third-party material has been
reproduced. The publishers would be pleased to hear from anyone whose rights we have unwittingly infringed.

University of Cambridge International Examinations is part of the University of Cambridge Local Examinations Syndicate (UCLES), which is itself a
department of the University of Cambridge.

© UCLES 2004 0625/06/O/N/04


Centre Number Candidate Number Name

UNIVERSITY OF CAMBRIDGE INTERNATIONAL EXAMINATIONS


International General Certificate of Secondary Education
PHYSICS 0625/06
Paper 6 Alternative to Practical
May/June 2005

1 hour
Candidates answer on the Question Paper.
No Additional Materials are required.

READ THESE INSTRUCTIONS FIRST

Write your Centre number, candidate number and name on all the work you hand in.
Write in dark blue or black pen in the spaces provided on the Question Paper.
You may use a soft pencil for any diagrams, graphs or rough working.
Do not use staples, paper clips, highlighters, glue or correction fluid.

Answer all questions.


At the end of the examination, fasten all your work securely together.
The number of marks is given in brackets [ ] at the end of each question or part question.

For Examiner’s Use


1

2
If you have been given a label, look at the
details. If any details are incorrect or 3
missing, please fill in your correct details
in the space given at the top of this page. 4

Stick your personal label here, if 5


provided.
Total

This document consists of 10 printed pages and 2 blank pages.


SPA (MML 8431 5/04) S81555/2
© UCLES 2005 [Turn over
2 For
Examiner’s
Use
1 The IGCSE class is investigating the change in temperature of hot water as cold water is
added to it.

The students are provided with 100 cm3 of hot water and a supply of cold water at room
temperature.

(a) The thermometer in Fig. 1.1 shows the temperature of the cold water.

–10 0 10 20 30 40 50 60 70 80 90 100 110


C

Fig. 1.1

Record the temperature of the cold water, as shown in Fig. 1.1.

.................................................................................................................................... [1]

(b) A student records the temperature of the hot water. He then pours 20 cm3 of the cold
water into the beaker containing the hot water. He records the temperature  of the
mixture of hot and cold water and the volume V of cold water added. He then repeats
the process four times until he has added a total of 100 cm3 of cold water. The table
shows the readings.

V/ /

0 80.0

20 58.0

40 48.0

60 40.5

80 34.0

100 29.0

(i) Complete the column headings in the table. [1]

© UCLES 2005 0625/06/M/J/05


3 For
Examiner’s
Use
(ii) Use the data in the table to plot a graph of temperature  (y-axis) against volume
V (x-axis).

0 20 40 60 80 100
V / cm3
[5]

Question 1 continues on page 4

© UCLES 2005 0625/06/M/J/05 [Turn over


4 For
Examiner’s
Use
(c) A sketch graph of the readings taken by another student carrying out a similar
experiment is shown in Fig. 1.2.

The theoretical line shows the results expected by the student after calculating the
values of . The student assumed that all the heat lost by the hot water was gained by
the cold water when the cold water was poured into the beaker.

The other line shows the experimental results.

80

/C

70

60

theoretical line

50
experimental line

40

30
0 20 40 60 80 100
V / cm3

Fig. 1.2

The student carried out the experiment with care. Suggest a practical reason why the
experimental line differs from the theoretical line.

..........................................................................................................................................

.................................................................................................................................... [1]

© UCLES 2005 0625/06/M/J/05


5 For
Examiner’s
Use
2 (a) The table below shows some measurements taken by three IGCSE students. The
second column shows the values recorded by the three students. For each quantity,
underline the value most likely to be correct. The first one is done for you.

quantity measured recorded values

thickness of a metre rule 0.25 mm


2.5 mm
25 mm

volume of a test-tube 12 mm3


12 cm3
12 m3

current in a 12 V ray box 0.5 A


lamp at less than normal
brightness 5.0 A
50 A

the surface area of the 0.3 cm2


base of a 250 cm3 beaker
3 cm2
30 cm2

the mass of a wooden 0.112 kg


metre rule
1.12 kg
11.2 kg

the weight of an IGCSE 6N


student
60 N
600 N
[5]

(b) A student is to find a value of the resistance of a wire by experiment. Potential


difference V and current I can be recorded. The resistance is then calculated using the
equation
V
R = –– .
I
State, with a reason, one example of good experimental practice that the student could
use to obtain a reliable result.

statement .........................................................................................................................

reason ........................................................................................................................ [2]

© UCLES 2005 0625/06/M/J/05 [Turn over


6 For
Examiner’s
Use
3 A student investigates the resistance of wire in different circuit arrangements.

The circuit shown in Fig. 3.1 is used.

power
source

A B C D
10 20 30 40 50 60 70 80 90

V crocodile
clip metre rule

Fig. 3.1

The student measures the current I in the wire. She then measures the p.d. V across AB,
AC and AD.

The student’s readings are shown in the table below.

section of wire l / cm I/A V/V R/

AB 0.375 0.95

AC 0.375 1.50

AD 0.375 1.95

(a) Using Fig. 3.1, record in the table the length l of each section of wire. [1]

© UCLES 2005 0625/06/M/J/05


7 For
Examiner’s
Use
(b) On Fig. 3.2, show the positions of the pointers of the ammeter reading 0.375 A, and
the voltmeter reading 1.50 V.

0.2 0.3

0.1 A 0.4

0 0.5

2 3

1 V 4

0 5

Fig. 3.2
[2]

(c) Calculate the resistance R of the sections of wire AB, AC and AD using the equation

V
R = –– .
I
Record these values of R, to a suitable number of significant figures, in the table. [2]

(d) Complete the column heading for the R column of the table. [1]

(e) Use your results to predict the resistance of a 1.50 m length of the same wire. Show
your working.

resistance = ...................................................... [2]

© UCLES 2005 0625/06/M/J/05 [Turn over


8 For
Examiner’s
Use
4 A student investigates the period of oscillation of a mass attached between two springs.

The apparatus used is shown in Fig. 4.1.

clamp

spring

mass

spring

clamp

Fig. 4.1

A 400 g mass m is attached between two springs, displaced a small distance downwards,
and then released so that it oscillates. The time t taken for 10 complete oscillations of the
mass is recorded. The experiment is repeated using values for m of 300 g and 200 g. The
readings are shown in the table below.

T s
m/g t/s T/s ––
m / ––
g

400 9.0

300 7.8

200 6.3

(a) Calculate the period T of the oscillations. T is the time for one complete oscillation.
Enter the values in the table. [2]
T
(b) Calculate and enter in the table the values of — . [2]
m

© UCLES 2005 0625/06/M/J/05


9 For
Examiner’s
Use
(c) The student suggests that T should be directly proportional to m. State with a reason
whether the results in the table support this suggestion.

statement .........................................................................................................................

reason ..............................................................................................................................

.................................................................................................................................... [2]

(d) In this experiment, the mass oscillates rapidly so that it is difficult to take the times
accurately. A technique has been included in this experiment to obtain an accurate
value for the period T. State, briefly, what this technique is and any calculation involved
to obtain the T value.

..........................................................................................................................................

..........................................................................................................................................

.................................................................................................................................... [2]

(e) Another student carried out the same experiment using a wider range of masses.
Suggest why, when the mass was 900 g, it could not oscillate freely.

.................................................................................................................................... [1]

© UCLES 2005 0625/06/M/J/05 [Turn over


10 For
Examiner’s
Use
5 A student investigates the refraction of light through a transparent block.

He places the transparent block on a sheet of plain paper, largest face down, and draws a
line round the block. He draws a line to represent an incident ray and places two pins W
and X in the line. Fig. 5.1 shows the outline of the block and the incident ray.

X
A B

D C

Fig. 5.1

(a) On Fig. 5.1, draw a normal to line AB at the point where the incident ray meets the
block. The incident ray is drawn on the diagram. The positions of the two pins W and X
that mark the incident ray are shown. [1]

(b) Measure the angle of incidence i.

i = ........................................................ [1]

(c) Draw in the refracted ray with an angle of refraction of 20°. Continue this line until it
meets the line CD. [2]

(d) The ray emerges from the block in a direction that is parallel to the incident ray. Draw in
this emergent ray. [2]

(e) Two pins Y and Z are placed so that the pins W and X, viewed through the block, and
the pins Y and Z all appear exactly in line with each other. Mark on the diagram, with
the letters Y and Z, where you would place these two pins. [2]

© UCLES 2005 0625/06/M/J/05


11

BLANK PAGE

0625/06/M/J/05
12

BLANK PAGE

Permission to reproduce items where third-party owned material protected by copyright is included has been sought and cleared where possible. Every
reasonable effort has been made by the publisher (UCLES) to trace copyright holders, but if any items requiring clearance have unwittingly been included, the
publisher will be pleased to make amends at the earliest possible opportunity.

University of Cambridge International Examinations is part of the University of Cambridge Local Examinations Syndicate (UCLES), which is itself a department
of the University of Cambridge.

0625/06/M/J/05
Centre Number Candidate Number Name

UNIVERSITY OF CAMBRIDGE INTERNATIONAL EXAMINATIONS


International General Certificate of Secondary Education
PHYSICS 0625/06
Paper 6 Alternative to Practical
October/November 2005

1 hour
Candidates answer on the Question Paper.
No Additional Materials are required.

READ THESE INSTRUCTIONS FIRST

Write your Centre number, candidate number and name on all work you hand in.
Write in dark blue or black pen in the spaces provided on the Question Paper.
You may use a soft pencil for any diagrams, graphs or rough working.
Do not use staples, paper clips, highlighters, glue or correction fluid.

Answer all questions.


At the end of the examination, fasten all your work securely together.
The number of marks is given in brackets [ ] at the end of each question or part question.

For Examiner’s Use


1

2
If you have been given a label, look at the
details. If any details are incorrect or 3
missing, please fill in your correct details
in the space given at the top of this page. 4

Stick your personal label here, if 5


provided.
Total

This document consists of 10 printed pages and 2 blank pages.


(SP) MML 8431 5/04 S82205/3
© UCLES 2005 [Turn over
2 For
Examiner’s
Use
1 The IGCSE class is investigating the effect of a load on a rule attached to a spring.

The apparatus used is shown in Fig. 1.1.

clamp
metre rule
spring

zero end of rule taped to bench


mass


bench

Fig. 1.1

The rule has the zero end taped to the bench so that it does not slip. The rule is attached to
a spring at the 40.0 cm mark. The students hang masses, starting with a 10 g mass, on the
rule at the 90.0 cm mark. For each mass, they measure the angle  between the rule and
the bench.

One student’s readings are shown in the table.

m/ /

0 29

10 28

20 26

30 25

40 22

50 19

(a) Complete the column headings in the table. [1]

(b) A student suggests that  should be directly proportional to m. State, with a reason,
whether the readings in the table support this suggestion.

statement .........................................................................................................................

reason ..............................................................................................................................

.................................................................................................................................... [2]

© UCLES 2005 0625/06/O/N/05


3 For
Examiner’s
Use
(c) A student carries out this experiment using the 360 ° protractor shown in Fig. 1.2.

80 90 100 1
70 10
12
60 0
13
50 0

14
40

0
15
30

0 1
20

60 1
10

180 190 20 70
350 0
40

0
03

210
33

22
0
32

0
0 23
31 02
40
250 300
260 270 280 290

Fig. 1.2

Explain how the student could use this protractor to measure the angle  between the
metre rule and the bench. You may draw a diagram if you wish.

..........................................................................................................................................

.................................................................................................................................... [2]

(d) The range of angles measured in this experiment may be quite small. Using the same
apparatus and with the masses and spring in the same positions, suggest another
method of investigating as reliably as possible the extent by which the rule is pulled
down by the masses. This method must not use a protractor but an additional rule may
be used. You may draw a diagram if you wish.

..........................................................................................................................................

.................................................................................................................................... [2]
© UCLES 2005 0625/06/O/N/05 [Turn over
4 For
Examiner’s
Use
2 An IGCSE student investigates the resistance of resistance wire ABCD in three different
circuit arrangements.

The circuits are shown in Fig. 2.1.

power
source

circuit (i)
A B

C D

power
source

circuit (ii) A B

C D

power
source

circuit (iii)

Fig. 2.1

© UCLES 2005 0625/06/O/N/05


5 For
Examiner’s
Use
(a) Circuit (iii) is the same as circuit (ii) but with an additional connecting lead between A
and D.

On Fig. 2.1, complete the circuit diagram for circuit (iii) using the standard symbol for a
resistor to represent each section AB, BC and CD of the resistance wire. [3]

(b) The student measures and records the current I and the p.d. V in each circuit. The
student’s readings are shown in the table.

circuit I/ V/ R/

(i) 0.91 1.80

(ii) 0.45 1.80

(iii) 1.37 1.85

(i) Complete the column headings for each of the I, V and R columns of the table. [1]
(ii) Calculate the resistance R for each circuit using the equation

V
R = –– .
I
Record in the table the values of R to an appropriate number of significant figures.
[2]

(c) Look at the resistance values for circuits (i) and (ii). The sections of resistance wire
AB, BC and CD are all of the same length. Suggest a value for the resistance of the
whole wire ABCD. Explain briefly how you obtained your value.

value ................................................................................................................................

explanation ......................................................................................................................

.................................................................................................................................... [2]

© UCLES 2005 0625/06/O/N/05 [Turn over


6 For
Examiner’s
Use
3 The IGCSE class carries out an experiment to investigate the effect of insulation on the rate
of cooling of hot water.

The apparatus is shown in Fig. 3.1.

thermometer thermometer

insulation

water water

bench
beaker A beaker B

Fig. 3.1

The students each have two glass beakers A and B. Beaker B is insulated. They also have
a supply of hot water.

A student pours hot water into beaker A until it is approximately two thirds full and then
measures the temperature  of the hot water. He records this temperature in the table at
time t = 0 s. He then starts a stopwatch and records the temperature of the water at 30 s
intervals for a total of four minutes.

He repeats the experiment using beaker B. All the readings are shown in the tables below.

beaker A beaker B

t/ / t/ /

0 80 0 80

30 67 30 69

60 59 60 62

90 54 90 57

120 51 120 53

150 48 150 50

180 47 180 48

210 46 210 47

240 45 240 46

© UCLES 2005 0625/06/O/N/05


7 For
Examiner’s
Use
(a) Complete the column headings in the tables. [1]

(b) Use the readings for beaker A to plot a graph of temperature  (y-axis) against
time t (x-axis). Start the temperature scale at 40 °C. Draw the best-fit curve. [4]

0 20 40 60 80 100 120 140 160 180 200 220 240 260 280
t/s

(c) Use the readings for beaker B to plot another curve on the same graph axes that you
used in part (b). [2]

(d) The experiment you have just done was designed to investigate the effect of insulation
on the rate of cooling. Suggest two improvements that could be made to the design of
the experiment.

1. ......................................................................................................................................

..........................................................................................................................................

2. ......................................................................................................................................

.................................................................................................................................... [2]

© UCLES 2005 0625/06/O/N/05 [Turn over


8 For
Examiner’s
Use
4 An IGCSE student is investigating the reflection of light by a plane mirror.

M M'

A B

card

Fig. 4.1

On Fig. 4.1, the line MM' shows the position of the mirror that is standing on a sheet of
paper. The reflecting surface of the mirror is vertical. AB is a card that is standing vertically
and is parallel to the reflecting surface of the mirror.

(a) Draw a normal to the mirror such that the edge B of the card lies on the normal. [1]

(b) Measure the distance x along the normal between the line MM' and the edge B of the
card.

x = ......................................................... [1]

(c) Draw a line from the edge A of the card to the point where the normal meets the line
MM'. This represents an incident ray from the edge of the card. [1]

(d) Measure the angle i between the incident ray and the normal.

i = .......................................................... [1]

© UCLES 2005 0625/06/O/N/05


9 For
Examiner’s
Use
x
(e) Calculate the ratio –– where y = 5.0 cm, the length of the card.
y
x
–– = ................................................... [2]
y

(f) The angle of reflection is to be determined as accurately as possible. On Fig. 4.1, mark
with the letters X, Y and Z the points where the student would place three pins in order
to plot the reflected ray. [4]

© UCLES 2005 0625/06/O/N/05 [Turn over


10 For
Examiner’s
Use
5 (a) The IGCSE class carries out an experiment to investigate the rate of cooling from
100 °C of a range of hot liquids. Underline any of the following variables that are likely
to have a significant effect on the temperature readings. (You may underline one, two
or all three of the suggested variables.)

type and size of container


volume of liquid
temperature of the surroundings [2]

(b) In an experiment to find the resistance of a wire, the students record the current in the
wire and the potential difference across it. They then calculate the resistance.
Underline any of the following variables that are likely to have a significant effect on the
current and/or potential difference readings. (You may underline one, two or all three of
the suggested variables.)

atmospheric pressure
temperature of the wire
length of wire [2]

(c) In an experiment, a short pendulum oscillates rapidly. A student is asked to find the
period of oscillation T of the pendulum using a stopwatch. The student sets the
pendulum swinging and records the time for one oscillation. A technique for improving
the accuracy of the value obtained for the period T should be used in this experiment.
State, briefly, what this technique is and any calculation involved to obtain the value
of T.

..........................................................................................................................................

..........................................................................................................................................

.................................................................................................................................... [2]

© UCLES 2005 0625/06/O/N/05


11

BLANK PAGE

0625/06/O/N/05
12

BLANK PAGE

Permission to reproduce items where third-party owned material protected by copyright is included has been sought and cleared where possible. Every
reasonable effort has been made by the publisher (UCLES) to trace copyright holders, but if any items requiring clearance have unwittingly been included, the
publisher will be pleased to make amends at the earliest possible opportunity.

University of Cambridge International Examinations is part of the University of Cambridge Local Examinations Syndicate (UCLES), which is itself a department
of the University of Cambridge.

© UCLES 2005 0625/06/O/N/05


Centre Number Candidate Number Name

UNIVERSITY OF CAMBRIDGE INTERNATIONAL EXAMINATIONS


International General Certificate of Secondary Education
PHYSICS 0625/06
Paper 6 Alternative to Practical
May/June 2006

1 hour
Candidates answer on the Question Paper.
No Additional Materials are required.

READ THESE INSTRUCTIONS FIRST

Write your Centre number, candidate number and name on all work you hand in.
Write in dark blue or black pen.
You may use a soft pencil for any diagrams, graphs or rough working.
Do not use staples, paper clips, highlighters, glue or correction fluid.

Answer all questions.


At the end of the examination, fasten all your work securely together.
The number of marks is given in brackets [ ] at the end of each question or part question.

For Examiner’s Use


1

Total

This document consists of 12 printed pages.


MML 10813 4/05 T06423/2
© UCLES 2006 [Turn over
2 For
Examiner’s
Use
1 The IGCSE class is determining the density of a sample of card.

Each student has a stack of ten pieces of card, as shown in Fig. 1.1.

Fig. 1.1

(a) (i) On Fig. 1.1, measure the height h of the stack of card.

h = ................................................... [1]
(ii) Calculate the average thickness t of one piece of card.

t = .................................................... [2]

(b) (i) On Fig. 1.1, measure the length l and width w of the top piece of card.

l = ..........................................................

w = ................................................... [1]
(ii) Calculate the volume V of one piece of card using the equation
V = ltw .

V = ................................................... [2]

© UCLES 2006 0625/06/M/J/06


3 For
Examiner’s
Use
(c) Calculate the density d of the card using the equation

m
d = ––
V

where the mass m of one piece of card is 1.3 g.

d = ................................................... [2]

(d) A sample of corrugated card of the same length and width as the card in Fig. 1.1
consists of two thin sheets of card with an air gap in between. The sheets of card are
separated by paper, as shown in the cross-section in Fig. 1.2. The thickness y of the air
gap as shown in Fig. 1.2 is between 2 mm and 3 mm.

card

paper y

card

Fig. 1.2

Estimate the volume Va of air trapped within the corrugated card shown in Fig. 1.2.

Va = ................................................. [1]

© UCLES 2006 0625/06/M/J/06 [Turn over


4 For
Examiner’s
Use
2 The IGCSE class is investigating the resistance of lamps in different circuit arrangements.

Fig. 2.1 shows a picture of the circuit.

power variable
source resistor

lamp

0-2 V voltmeter
0-1 A ammeter

Fig. 2.1

(a) Draw a circuit diagram of the circuit shown in Fig. 2.1. Use standard circuit symbols.

[3]

© UCLES 2006 0625/06/M/J/06


5 For
Examiner’s
Use
(b) The current I through the lamp and the voltage V across the lamp are measured. Then
a second lamp is connected in parallel with the first. The total current I in the circuit
and the voltage V across the lamps are measured. The table below shows the
readings.

I/ V/ R/

0.24 1.39

0.45 1.30

(i) Complete the column headings for each of the I, V and R columns of the table. [1]
(ii) Calculate the resistance R in each case using the equation

V
R = –– .
I
Enter the results in the table. [2]

© UCLES 2006 0625/06/M/J/06 [Turn over


6 For
Examiner’s
Use
3 The IGCSE class is determining the refractive index of the material of a transparent block.
Fig. 3.1. shows the drawing that a student makes.

P1
N

P2
A F B

D C
G

P3

N
P4

sheet of
plain paper
eye

Fig. 3.1

© UCLES 2006 0625/06/M/J/06


7 For
Examiner’s
Use
The student places two pins P1 and P2 on line EF to mark an incident ray. Then she places
the block on the paper and observes the images of P1 and P2 through side CD of the block
so that the images of P1 and P2 appear one behind the other. She places two pins P3 and
P4 between her eye and the block so that P3 and P4 and the images of P1 and P2, seen
through the block, appear one behind the other.

(a) (i) Draw a line joining the positions of P3 and P4. Continue the line until it meets CD.
Label this point H.
(ii) Measure the distance a between G and H.

a = ................................................... [1]
(iii) Draw the line HF.
(iv) Measure the length b of the line HF.

b = ................................................... [1]
(v) Extend the straight line EF within the outline of the block to a point I. The
distance FI must be exactly equal to b.
(vi) From I draw a line that meets NN at a right angle. Label this position J.
(vii) Measure the length c of the line JI.

c = .................................................... [3]
(viii) Calculate the refractive index n of the material of the block using the equation

c
n = –– .
a

n = ................................................... [2]

(b) Suggest two improvements you would make to this experiment to ensure an accurate
result for the refractive index n.

1 .......................................................................................................................................

..........................................................................................................................................

2 .......................................................................................................................................

.................................................................................................................................... [2]

© UCLES 2006 0625/06/M/J/06 [Turn over


8 For
Examiner’s
Use
4 An IGCSE student is investigating the temperature rise of water in beakers heated by
different methods. The apparatus is shown in Fig. 4.1. Beaker A is heated electrically and
beaker B is heated with a Bunsen burner.

fixed voltage
power source

A
V

beaker A

beaker B

Bunsen burner

Fig. 4.1

© UCLES 2006 0625/06/M/J/06


9 For
Examiner’s
Use
The student first records room temperature.

(a) Fig. 4.2 shows the thermometer at room temperature.

—10 0 10 20 30 40 50 60 70 80 90 100 110


o
C

Fig. 4.2

(i) Write down the value of room temperature.

room temperature = ......................... [1]


(ii) The two beakers are heated from room temperature for the same length of time.
The new water temperature for beaker A is 30 °C and for beaker B is 28 °C.
Calculate the temperature rise of the water in each beaker.

temperature rise in beaker A = ...............................

temperature rise in beaker B = ......................... [1]

(b) The electrical heater and the Bunsen burner both have the same power and both
beakers were heated from room temperature for the same length of time. Suggest why
there is a difference in temperature rise between beaker A and beaker B.

..........................................................................................................................................

..........................................................................................................................................

.................................................................................................................................... [2]

(c) In order to keep the heating effect of the electrical heater constant throughout the
heating period, the student adjusts the current. Name the component in the circuit that
the student uses for this purpose.

.................................................................................................................................... [1]

© UCLES 2006 0625/06/M/J/06 [Turn over


10 For
Examiner’s
Use
5 The IGCSE class is determining the weight of a metre rule.

The apparatus is shown in Fig. 5.1.

N
newton
meter

metre
pivot rule

bench

Fig. 5.1

A metre rule is supported at one end by a pivot through the 1.0 cm mark. The other end is
supported at the 91.0 cm mark by a newton meter hanging from a clamp.

(a) Describe how you would check that the metre rule is horizontal. You may draw a
diagram if you wish.

..........................................................................................................................................

.................................................................................................................................... [1]

© UCLES 2006 0625/06/M/J/06


11 For
Examiner’s
Use
(b) The students record the force F shown on the newton meter and the distance d from
the pivot to the 91 cm mark. They then repeat the experiment several times using a
range of values of the distance d. The readings are shown in the table.

1 1
F/N d/m ––
d / ––
m

0.74 0.900

0.78 0.850

0.81 0.800

0.86 0.750

0.92 0.700

1
Calculate and record in the table the values of –– . [1]
d
1 1
(c) (i) On the graph grid below, plot a graph of F / N (y-axis) against ––
Start the y-axis at 0.7 and the x-axis at 1.0. d / –– (x-axis).
m [2]

(ii) Draw the line of best fit on your graph. [2]

Question 5 continues on the next page.

© UCLES 2006 0625/06/M/J/06 [Turn over


12 For
Examiner’s
Use
(iii) Determine the gradient G of the line.

G = ................................................... [3]

(d) Calculate the weight of the metre rule using the equation

G
W = ––
k
where k = 0.490 m.

W = .................................................. [2]

Permission to reproduce items where third-party owned material protected by copyright is included has been sought and cleared where possible. Every
reasonable effort has been made by the publisher (UCLES) to trace copyright holders, but if any items requiring clearance have unwittingly been
included, the publisher will be pleased to make amends at the earliest possible opportunity.

University of Cambridge International Examinations is part of the University of Cambridge Local Examinations Syndicate (UCLES), which is itself a
department of the University of Cambridge.

© UCLES 2006 0625/06/M/J/06


Centre Number Candidate Number Name

UNIVERSITY OF CAMBRIDGE INTERNATIONAL EXAMINATIONS


International General Certificate of Secondary Education
PHYSICS 0625/06
Paper 6 Alternative to Practical
October/November 2006

1 hour
Candidates answer on the Question Paper.
No Additional Materials are required.

READ THESE INSTRUCTIONS FIRST

Write your Centre number, candidate number and name on all work you hand in.
Write in dark blue or black pen.
You may use a soft pencil for any diagrams, graphs or rough working.
Do not use staples, paper clips, highlighters, glue or correction fluid.

Answer all questions.

At the end of the examination, fasten all your work securely together.
The number of marks is given in brackets [ ] at the end of each question or part question.

For Examiner’s Use


1

Total

This document consists of 11 printed pages and 1 blank page.


MML 10815 4/05 T06425/2
© UCLES 2006 [Turn over
2 For
Examiner’s
Use
1 The IGCSE class is determining the density of a type of wood.

The students are provided with a bundle of wooden rods, as shown in Fig. 1.1.

Fig. 1.1

(a) On Fig. 1.1, measure the length l of a rod.

l = ............................................... cm [1]

(b) A student winds five turns of string round the bundle and marks the beginning and the
end of the five turns. She then uses the metre rule to measure the distance x between
the marks. She records that x = 24.5 cm.
(i) Determine the circumference c of the bundle of rods.

c = ..........................................................
(ii) Calculate the volume V of the bundle of rods using the equation

2
V = c––l .
4

V = ................................................... [4]

© UCLES 2006 0625/06/O/N/06


3 For
Examiner’s
Use
(c) The equation used in (b)(ii) assumes that the bundle is a solid cylinder. However, there
are air gaps between the rods.
(i) Estimate the total volume Vr of the rods themselves.

Vr = ........................................................
(ii) Calculate the density d of the wood using the equation

m
d = ––
Vr

where the mass m of the bundle = 6.3 g.

d = ................................................... [3]

© UCLES 2006 0625/06/O/N/06 [Turn over


4 For
Examiner’s
Use
2 The IGCSE class is investigating the swing of a loaded metre rule.

The arrangement of the apparatus is shown in Fig. 2.1.

pivot

metre
rule
d

load

one complete swing

Fig. 2.1

The load is attached to the metre rule so that its centre is 90.0 cm from the pivot. The rule is
displaced a small distance to one side and allowed to swing. The time t taken for
10 complete swings is recorded. This is repeated using different values of the distance d.
The readings are shown in the table.

d/ t/ T/

90.0 18.35

85.0 17.87

80.0 17.53

75.0 17.06

70.0 16.72

© UCLES 2006 0625/06/O/N/06


5 For
Examiner’s
Use
(a) Complete the column headings in the table. [1]

(b) Calculate the period T for each value of d. The period is the time taken for one
complete swing. Enter the values in the table. [2]

(c) On the grid below, plot a graph of T / s (y-axis) against d / cm (x-axis). Start the x-axis
at d = 70.0 cm and the y-axis at a suitable value of T / s to make best use of the graph
grid. [5]

(d) A student suggests that T is proportional to d. State whether or not the results support
this suggestion and give a reason for your answer.

statement .........................................................................................................................

..........................................................................................................................................

reason ..............................................................................................................................

.................................................................................................................................... [1]

(e) Explain why the student takes the time for ten swings and then calculates the time for
one swing (the period), rather than just measuring the time for one swing.

..........................................................................................................................................

.................................................................................................................................... [1]

© UCLES 2006 0625/06/O/N/06 [Turn over


6 For
Examiner’s
Use
3 The IGCSE class is investigating reflection in a plane mirror. Fig. 3.1 shows a ray diagram
that a student is constructing.

paper

G
E F

mirror

J K

eye

Fig. 3.1

© UCLES 2006 0625/06/O/N/06


7 For
Examiner’s
Use
(a) (i) Draw a normal GH to line EF.
(ii) Mark a point A on line GJ so that the distance AG is 11.5 cm.
(iii) Measure the angle of incidence i between line GJ and the normal.

i = ..................................................... [3]

(b) The student pushes two pins into the paper on line GJ, one at point A, and the other at
a point B nearer to the mirror. He views the images of the pins from the direction
indicated in Fig. 3.1. He then pushes in two pins on line GK between his eye and the
mirror so that these two pins and the images of the pins on line GJ appear exactly one
behind the other.
(i) On Fig. 3.1, mark suitable positions for the pins on lines GJ and GK. Label the
marks with letters B, C and D.
(ii) To obtain an accurate result for this experiment, would you view the tops, bases or
central parts of the pins when lining them up? Give a reason for your answer.

I would view ..............................................................................................................

reason ......................................................................................................................

..................................................................................................................................

............................................................................................................................ [3]

© UCLES 2006 0625/06/O/N/06 [Turn over


8 For
Examiner’s
Use
4 The IGCSE class is investigating conditions affecting the rate of cooling of a beaker of hot
water.

(a) The students start by measuring room temperature. Record the value of room
temperature as shown on the thermometer in Fig. 4.1.

—10 0 10 20 30 40 50 60 70 80 90 100 110


o
C

Fig. 4.1

temperature = .................................. [2]

(b) The students are provided with hot water in beakers as shown in Fig. 4.2. Beaker A is
insulated and beaker B has a lid.

thermometer thermometer

lid

insulation

water water

Beaker A Beaker B

Fig. 4.2

The students measure and record the temperature θ of the water in each beaker every
30 s for a total of five minutes. One student’s readings are shown in the tables.

© UCLES 2006 0625/06/O/N/06


9 For
Examiner’s
Use
beaker A beaker B

time / s θ / °C time / s θ / °C
0 83.0 0 82.0

30 82.0 30 82.0

60 81.0 60 81.0

90 79.5 90 80.0

120 79.0 120 79.0

150 77.0 150 78.0

180 75.0 180 76.0

210 74.0 210 75.0

240 72.0 240 74.0

270 71.0 270 73.0

300 70.0 300 72.0

(i) Look at the temperature readings in the tables. State whether the insulation round
beaker A or the lid on beaker B or neither is most effective in keeping the water
hot. By reference to readings in the tables, justify your answer.

statement .................................................................................................................

justification ...............................................................................................................

............................................................................................................................ [2]
(ii) Suggest a suitable material for the insulation around beaker A.

............................................................................................................................ [1]
(iii) To obtain reliable results in this experiment, it is important that variables are
controlled. State three variables that should be controlled in this experiment.

variable 1 ..................................................................................................................

variable 2 ..................................................................................................................

variable 3 ............................................................................................................ [3]

© UCLES 2006 0625/06/O/N/06 [Turn over


10 For
Examiner’s
Use
5 A student is investigating the relationship between potential difference V across a resistor
and the current I in it. Fig. 5.1 shows the apparatus that the student is using.

switch power variable


source resistor

lamp
resistor

ammeter

voltmeter

Fig. 5.1

(a) Draw the circuit diagram of the circuit shown in Fig. 5.1. Use standard circuit symbols.

[3]

(b) The student is using a lamp to show when the current is switched on.

Why is it unnecessary to use the lamp?

..........................................................................................................................................

.................................................................................................................................... [1]

© UCLES 2006 0625/06/O/N/06


11 For
Examiner’s
Use
(c) State which piece of apparatus in the circuit is used to control the size of the current.

.................................................................................................................................... [1]

(d) The student removes the lamp from the circuit. He is told that the resistance of a
conductor is constant if the temperature of the conductor is constant. He knows that
the current in the resistor has a heating effect. Suggest two ways in which the student
could minimise the heating effect of the current in the resistor.

1. ......................................................................................................................................

2. ................................................................................................................................ [2]

(e) Fig. 5.2 shows a variable resistor with the sliding contact in two different positions.

metal bar sliding contact in position A

coil of resistance wire

sliding contact in position B


metal bar

coil of resistance wire

Fig. 5.2

State which position, A or B, shows the higher resistance setting. Explain your answer.

statement ............................................

explanation ......................................................................................................................

.................................................................................................................................... [1]

© UCLES 2006 0625/06/O/N/06


12

BLANK PAGE

Permission to reproduce items where third-party owned material protected by copyright is included has been sought and cleared where possible. Every
reasonable effort has been made by the publisher (UCLES) to trace copyright holders, but if any items requiring clearance have unwittingly been included, the
publisher will be pleased to make amends at the earliest possible opportunity.

University of Cambridge International Examinations is part of the University of Cambridge Local Examinations Syndicate (UCLES), which is itself a department
of the University of Cambridge.

0625/06/O/N/06
UNIVERSITY OF CAMBRIDGE INTERNATIONAL EXAMINATIONS
International General Certificate of Secondary Education
*4019966178*

PHYSICS 0625/06
Paper 6 Alternative to Practical May/June 2007
1 hour
Candidates answer on the Question Paper.
No Additional Materials are required.

READ THESE INSTRUCTIONS FIRST

Write your Centre number, candidate number and name on all work you hand in.
Write in dark blue or black pen.
You may use a soft pencil for any diagrams, graphs or rough working.
Do not use staples, paper clips, highlighters, glue or correction fluid.
DO NOT WRITE ON ANY BARCODES.

Answer all questions.

At the end of the examination, fasten all your work securely together.
The number of marks is given in brackets [ ] at the end of each question or part question.

For Examiner’s Use

Total

This document consists of 10 printed pages and 2 blank pages.

MML 13215 4/06 T25816/4


© UCLES 2007 [Turn over
2 For
Examiner’s
Use
1 The IGCSE class is investigating the temperature changes that occur when hot and cold
water are mixed.

(a) A student pours 50 cm3 of water into a beaker. He then measures the temperature 1
of the water in the beaker. Write down the value of 1 shown on the thermometer in
Fig. 1.1.

°C
–10

10

20

30

40

50

60

70

80

90

100

110
Fig. 1.1

1 = ................................................. [2]

(b) The student then measures the temperature 2 of some hot water. He pours 50 cm3
of this hot water into the beaker of water at room temperature. He then records the
temperature 3 of the water in the beaker. His readings are

2 = 76 °C,

3 = 42 °C.

Calculate

(i) the temperature rise of the cold water, ......................................

(ii) the temperature fall of the hot water. .........................................


[2]

(c) A theoretical calculation based on the equation

thermal energy lost by hot water = thermal energy gained by cold water

predicts a higher value for the temperature 3 than the value that is obtained by this
experiment. Suggest

(i) a practical explanation for the difference in values,

..................................................................................................................................

(ii) two practical improvements that you could make to the procedure for this experiment
to obtain a result that is closer to the theoretical result.

1. ..............................................................................................................................

2. ........................................................................................................................ [3]

[Total: 7]

© UCLES 2007 0625/06/M/J/07


3 For
Examiner’s
Use
2 A student is investigating the position of a sheet of card that is hanging from a pivot.

Fig. 2.1 shows the apparatus drawn full size.

metre rule
stand pivot
pointer

card

h0

1 2 3 4 5 6
A

mass
0

Fig. 2.1

© UCLES 2007 0625/06/M/J/07 [Turn over


4 For
Examiner’s
Use
(a) On Fig. 2.1 measure the distance d between the centre of the hole labelled 1 and the
edge of the card at A. Record this value in the table.

hole d / mm h / mm b / mm

1 140

2 135

3 132

4 128

5 124

6 120

[3]

(b) Repeat step (a) for each of the remaining holes 2 – 6.

(c) On Fig. 2.1 measure the height h0 of the pointer above the bench.

h0 = .....................................................
[1]

(d) A student hangs a 10 g mass from the hole 1 in the card. She records the height h of
the end of the pointer above the bench. She then repeats this procedure by hanging the
mass from each hole in turn. Her results are shown in the table above.

(e) Calculate the differences in heights b using the equation

b = (h – h0)

and record the results in the table above.

© UCLES 2007 0625/06/M/J/07


5 For
Examiner’s
Use
(f) Plot the graph of b / mm (y-axis) against d / mm (x-axis).

45

b / mm

40

35

30

25

20
[4]

(g) The student suggests that b is directly proportional to d. By reference to your graph,
state whether or not the results support the student’s suggestion. Give a reason for your
answer.

Statement ........................................................................................................................

Reason ............................................................................................................................

..........................................................................................................................................

.................................................................................................................................... [2]

(h) It is important when recording the heights that the rule is vertical. State briefly how you
would check that the rule is vertical.

..........................................................................................................................................

.................................................................................................................................... [1]

[Total: 11]

© UCLES 2007 0625/06/M/J/07 [Turn over


6 For
Examiner’s
Use
3 The IGCSE class is investigating the resistance of a wire. The circuit is as shown in Fig. 3.1.

power
source

A B

C D
V

Fig. 3.1

(a) A student uses the switches to connect the wire AB into the circuit and records the p.d.
V across the wire between A and B. He also records the current I in the wire.

The student then repeats the measurements using the wire CD in place of wire AB.

The readings are shown in the table below.

wire V/ I/ R/

AB 1.9 0.24

CD 1.9 0.96

[3]

(i) Calculate the resistance R of each wire, using the equation

R = V / I.

Record the values in the table.

(ii) Complete the column headings in the table.

© UCLES 2007 0625/06/M/J/07


7 For
Examiner’s
Use
(b) The two wires AB and CD are made of the same material and are of the same length.
The diameter of wire CD is twice the diameter of wire AB.

(i) Look at the results in the table. Below are four possible relationships between R
and the diameter d of the wire. Tick the relationship that best matches the results.

R is proportional to d

1
R is proportional to
d

R is proportional to d 2

1
R is proportional to
d2

(ii) Explain briefly how the results support your answer in part (b)(i).

..................................................................................................................................

..................................................................................................................................

..................................................................................................................................

............................................................................................................................ [2]

(c) Following this experiment, the student wishes to investigate whether two lamps in parallel
with each other have a smaller combined resistance than the two lamps in series. Draw
one circuit diagram showing

(i) two lamps in parallel with each other connected to a power source,

(ii) an ammeter to measure the total current in the circuit,

(iii) a voltmeter to measure the potential difference across the two lamps.

[3]

[Total: 8]

© UCLES 2007 0625/06/M/J/07 [Turn over


8 For
Examiner’s
Use
4 The IGCSE class is investigating the formation of images by a lens.

Fig. 4.1 shows the apparatus that is being used.

illuminated screen
object
lens

Fig. 4.1

(a) A student places the screen at a distance d = 0.800 m from the illuminated object. She
adjusts the position of the lens until a clearly focused magnified image is formed on the
screen. She measures the distance x between the centre of the lens and the screen.
Without moving the illuminated object or the screen, she moves the lens towards the
screen until a second clearly focused (but diminished) image is formed on the screen.
She measures the distance y between the centre of the lens and the screen. She repeats
the experiment with the distance d increased to 0.900 m. The readings are shown in the
table.

x/m y/m d/m f/m

0.205 0.600 0.800

0.180 0.720 0.900

(i) For each set of readings calculate the focal length f of the lens using the equation
xy
f= .
d

Enter the values in the table.

© UCLES 2007 0625/06/M/J/07


9 For
Examiner’s
Use
(ii) Calculate the average value of the focal length f.

average value of the focal length f = ................................................ [4]

(b) Suggest two precautions that can be taken in this experiment in order to obtain an
accurate result.

1. ......................................................................................................................................

..........................................................................................................................................

2. ......................................................................................................................................

.................................................................................................................................... [2]

(c) The illuminated object is triangular in shape, as shown in Fig. 4.2.

Fig. 4.2

In the space below, sketch the appearance of one of the images on the screen.

[1]

[Total: 7]

© UCLES 2007 0625/06/M/J/07 [Turn over


10 For
Examiner’s
Use
5 (a) An IGCSE student is investigating the relationship between the extension of a spring
of unstretched length l 0 and the load hung on the spring. The apparatus is shown in
Fig. 5.1 below. The spring is shown larger than its actual size.

clamp

l0

set of ten
0.1 N weights

Fig. 5.1

Consider the readings that the student should take and write appropriate column
headings, with units, in the table below.

l 0 = 25 mm

0.0 25 0

0.1 30 5

0.2 36 11

0.3 43 18

0.4 50 25
[4]

(b) The student decides to repeat the experiment using a spring made of a different metal
in order to study how the extension may be affected by the metal from which the spring
is made. To make a fair comparison, other variables must be kept constant. Suggest
three variables that the student should keep constant.

1. ......................................................................................................................................

2. ......................................................................................................................................

3. ................................................................................................................................ [3]

[Total: 7]

© UCLES 2007 0625/06/M/J/07


11

BLANK PAGE

0625/06/M/J/07
12

BLANK PAGE

Permission to reproduce items where third-party owned material protected by copyright is included has been sought and cleared where possible. Every
reasonable effort has been made by the publisher (UCLES) to trace copyright holders, but if any items requiring clearance have unwittingly been included, the
publisher will be pleased to make amends at the earliest possible opportunity.

University of Cambridge International Examinations is part of the Cambridge Assessment Group. Cambridge Assessment is the brand name of University of
Cambridge Local Examinations Syndicate (UCLES), which is itself a department of the University of Cambridge.

0625/06/M/J/07
UNIVERSITY OF CAMBRIDGE INTERNATIONAL EXAMINATIONS
International General Certificate of Secondary Education
*6786650342*

PHYSICS 0625/06
Paper 6 Alternative to Practical October/November 2007
1 hour
Candidates answer on the Question Paper.
No Additional Materials are required.

Write your Centre number, candidate number and name on all work you hand in.
Write in dark blue or black pen.
You may use a soft pencil for any diagrams, graphs or rough working.
Do not use staples, paper clips, highlighters, glue or correction fluid.
DO NOT WRITE IN ANY BARCODES.

Answer all questions.

At the end of the examination, fasten all your work securely together.
The number of marks is given in brackets [ ] at the end of each question or part question.

For Examiner’s Use

Total

This document consists of 11 printed pages and 1 blank page.

SPA (MML 13215 4/06) T25854/5


© UCLES 2007 [Turn over
2 For
Examiner’s
Use
1 The IGCSE class is investigating the rate of cooling of hot water.

(a) A student measures room temperature. Write down the value of room temperature θ0
shown on the thermometer in Fig. 1.1.

°C
–10

10

20

30

40

50

60

70

80

90

100

110
Fig. 1.1

θ0 = ................................................. [1]

(b) He then pours hot water into a beaker until it is about two-thirds full. He measures and
records the temperature θ of the hot water and at the same time starts a stopwatch.
As the water cools, he records the temperature every 30 s for a total of five minutes. His
readings are shown in the table below.

t/ θ/
0 68.0

30 53.0

60 45.0

90 40.0

120 36.5

150 33.5

180 32.0

210 30.0

240 29.0

270 28.5

300 28.0

(i) Complete the column headings in the table.

(ii) Calculate the temperature fall T1 in the first minute of the experiment.

T1 = ....................................................

(iii) Calculate the temperature fall T2 in the final minute of the experiment.

T2 = ....................................................
[3]

© UCLES 2007 0625/06/O/N/07


3 For
Examiner’s
Use
(c) Theory suggests that the rate of cooling of the hot water at any time depends on the
difference between the temperature of the water at that time and room temperature.

(i) State and explain whether your answers in (b) support this theory.

Statement .................................................................................................................

Explanation ..............................................................................................................

............................................................................................................................ [1]

(ii) Suggest three variables that you would attempt to keep constant if this theory were
to be investigated further.

1. ...............................................................................................................................

2. ...............................................................................................................................

3. ......................................................................................................................... [3]

(d) Suggest one addition you could make to the apparatus provided in order to reduce the
rate of evaporation of the water in the beaker.

.................................................................................................................................... [1]

[Total: 9]

© UCLES 2007 0625/06/O/N/07 [Turn over


4 For
Examiner’s
Use
2 The IGCSE class is investigating a simple balance.

The diagram below shows the apparatus.

metre rule
stand pivot
pointer 150 mm

hook 140 mm

130 mm

paper 120 mm

clip 110 mm

100 mm
card
90 mm

80 mm

70 mm

60 mm

50 mm

40 mm

30 mm

20 mm

10 mm

0 mm

Fig. 2.1

(a) A student records the height h0 of the pointer above the bench. She then hangs a paper
clip on the hook and records the new height h of the pointer above the bench. Next she
records the heights of the pointer above the bench using different numbers N of paper
clips. The readings are shown in the table below.

h0 = 100 mm

N h / mm d / mm

1 108

2 114

3 120

4 125

5 134

6 141

© UCLES 2007 0625/06/O/N/07


5 For
Examiner’s
Use
Calculate the height differences d using the equation

d = (h – h0)

and enter them in the table. [2]

(b) (i) Plot the graph of d / mm (y-axis) against N (x-axis).

(ii) Use your graph to predict the value of d if a nail with the same mass as 4.6 paper
clips were to be hung from the hook in place of the paper clips. Show clearly on the
graph how you obtained your value.

d = ................................................. [6]

[Total: 8]

© UCLES 2007 0625/06/O/N/07 [Turn over


6 For
Examiner’s
Use
3 The IGCSE class is investigating the potential difference across lamps and the currents in
the lamps.

Fig. 3.1 shows the circuit that is being used.

power
source

lamp 1

lamp 2

lamp 3

Fig. 3.1

(a) A student uses the ammeter to record the current I in the wire connecting the power
source to the rest of the circuit. He then moves the ammeter to new positions in the
circuit and measures the current in each lamp in turn. The positions of the pointer on the
ammeter scale are shown below.

(i) (ii)

0.2 0.3 0.2 0.3


0.1 0.4 0.1 0.4
A A
0 0.5 0 0.5

current I = .............................................. current I1 in lamp 1 = .............................

(iii) (iv)

0.2 0.3 0.2 0.3


0.1 0.4 0.1 0.4
A A
0 0.5 0 0.5

current I2 in lamp 2 = ............................. current I3 in lamp 3 = .............................

Write down the ammeter readings I, I1, I2 and I3. [3]


© UCLES 2007 0625/06/O/N/07
7 For
Examiner’s
Use
(b) Theory suggests that I = I1 + I2 + I3. State whether or not your readings support this
theory. Give a reason for your answer.

Statement ........................................................................................................................

Reason ............................................................................................................................

..........................................................................................................................................

.................................................................................................................................... [1]

(c) To test the theory further, you would need to vary the value of I. State how you would
vary I.

..........................................................................................................................................

.................................................................................................................................... [1]

(d) The student uses a voltmeter to measure the potential difference V across the lamps.

His reading is V = 1.6 V.

(i) Calculate the resistance R of the lamps arranged in parallel, using the equation

R = V / I,

where I is the value of the current in (a)(i).

R = .....................................................

(ii) On Fig. 3.1, add the symbol for the voltmeter connected to measure the potential
difference across the lamps. [3]

[Total: 8]

© UCLES 2007 0625/06/O/N/07 [Turn over


8 For
Examiner’s
Use
4 The IGCSE class is investigating the refraction of light through a transparent block.

Fig. 4.1 shows the apparatus used.

eye

transparent
block

h
optics pin

sheet of x
paper

Fig. 4.1

(a) A student looks down through the transparent block at the image of a line drawn on the
sheet of paper. She carefully places the point of the optics pin exactly in line with the
image.

(i) On Fig. 4.1, measure the vertical distance x between the paper and the pin.

x = .....................................................

(ii) On Fig. 4.1, measure the height h of the transparent block.

h = .....................................................

(iii) Calculate the refractive index n of the material of the block using the equation
h
n = ––––– .
h –x

n = ................................................ [5]

© UCLES 2007 0625/06/O/N/07


9 For
Examiner’s
Use
(b) To obtain a reliable value for the vertical distance x between the paper and the pin, it is
important that the pin is horizontal. Explain briefly with the aid of a diagram how you
would check that the pin is horizontal.

..........................................................................................................................................

.................................................................................................................................... [1]

[Total: 6]

© UCLES 2007 0625/06/O/N/07 [Turn over


10 For
Examiner’s
Use
5 (a) An IGCSE student is investigating the differences in density of small pieces of different
rocks. She is using an electronic balance to measure the mass of each sample and
using the ‘displacement method’ to determine the volume of each sample. Fig. 5.1 shows
the displacement method.

cm3 cm3

100 100

80 80

60 60

40 40 rock sample

20 20

V1 V2

Fig. 5.1

(i) Write down the volume shown in each measuring cylinder.

V1 = .....................................................

V2 = .....................................................

(ii) Calculate the volume V of the rock sample.

V = .....................................................

(iii) Calculate the density of sample A using the equation


m
density = ––– ,
V
where the mass m of the sample of rock is 109 g.

density = .....................................................
[4]

© UCLES 2007 0625/06/O/N/07


11 For
Examiner’s
Use
(b) The table shows the readings that the student obtains for samples of rocks B and C.
Complete the table by

(i) inserting the appropriate column headings with units,


m
(ii) calculating the densities using the equation density = ––– .
V

sample m/g V/ density /

B 193 84 50 34

C 130 93 50 43

[4]

(c) Explain briefly how you would determine the density of sand grains.

..........................................................................................................................................

.................................................................................................................................... [1]

[Total: 9]

© UCLES 2007 0625/06/O/N/07


12

BLANK PAGE

Permission to reproduce items where third-party owned material protected by copyright is included has been sought and cleared where possible. Every
reasonable effort has been made by the publisher (UCLES) to trace copyright holders, but if any items requiring clearance have unwittingly been included, the
publisher will be pleased to make amends at the earliest possible opportunity.

University of Cambridge International Examinations is part of the Cambridge Assessment Group. Cambridge Assessment is the brand name of University of
Cambridge Local Examinations Syndicate (UCLES), which is itself a department of the University of Cambridge.

0625/06/O/N/07
UNIVERSITY OF CAMBRIDGE INTERNATIONAL EXAMINATIONS
International General Certificate of Secondary Education
*4130189347*

PHYSICS 0625/06
Paper 6 Alternative to Practical May/June 2008
1 hour
Candidates answer on the Question Paper.
No Additional Materials are required.

READ THESE INSTRUCTIONS FIRST

Write your Centre number, candidate number and name on all work you hand in.
Write in dark blue or black pen.
You may use a soft pencil for any diagrams, graphs or rough working.
Do not use staples, paper clips, highlighters, glue or correction fluid.
DO NOT WRITE IN ANY BARCODES.

Answer all questions.

At the end of the examination, fasten all your work securely together.
The number of marks is given in brackets [ ] at the end of each question or part question.

For Examiner’s Use

Total

This document consists of 12 printed pages and 4 blank pages.

SP (SLM/CGW) T40877/4
© UCLES 2008 [Turn over
2

1 An IGCSE student is determining the density of a solid metal cylinder using a balancing For
method. Fig. 1.1. shows the apparatus. Examiner’s
Use
metre
rule cylinder
a b

bench
pivot

Fig. 1.1

He places the cylinder on the metre rule so that its centre is directly above the 10.0 cm mark.
The rule is placed on the pivot so that the rule is as near as possible to being balanced.

He measures and records the distance a from the centre of the rule to the pivot and the
distance b from the centre of the cylinder to the pivot. He repeats the experiment with the
same cylinder at different positions on the rule.

The readings are shown in Table 1.1.

Table 1.1

a/ b/ M/

12.6 27.4

11.0 24.0

9.5 20.5

(a) (i) Complete the column headings in Table 1.1.

(ii) For each set of readings, calculate the mass M of the cylinder using the equation
ka
M= .
b
The value of k is the mass of the rule which is 108 g.

Enter the results in Table 1.1.


[3]

© UCLES 2008 0625/06/M/J/08


3

(b) The cylinder completely covers the marks on the metre rule. Describe, with the aid of For
a diagram, how you would judge that the centre of the cylinder is directly above the Examiner’s
10.0 cm mark. Use

..........................................................................................................................................

...................................................................................................................................... [1]

(c) Use your answers in Table 1.1 to calculate and record the average of the three values
for M. Show your working.

average value for M = .................................................. [2]

© UCLES 2008 0625/06/M/J/08 [Turn over


4

(d) Fig. 1.2 shows the cylinder placed flat on the bench and viewed from one side. For
Examiner’s
Use

Fig. 1.2

(i) On the diagram, measure the diameter d and the thickness t of the cylinder.

d = ......................................................

t = ......................................................

(ii) Calculate the volume V of the cylinder using the equation

V = πd t .
2

V = ......................................................

(iii) Calculate the density ρ of the cylinder using the equation

ρ = M.
V

ρ = .................................................. [3]

[Total: 9]

© UCLES 2008 0625/06/M/J/08


5

BLANK PAGE

0625/06/M/J/08 [Turn over


6

2 The IGCSE class is comparing the combined resistance of resistors in different circuit For
arrangements. The first circuit is shown in Fig. 2.1. Examiner’s
Use
power
source

V A

A B

Circuit 1

Fig. 2.1

(a) The current I in the circuit and the p.d. V across the three resistors are measured and
recorded. Three more circuit arrangements are used. For each arrangement, a student
disconnects the resistors and then reconnects them between points A and B as shown
in Figs. 2.2–2.4.

A B
Circuit 2

Fig. 2.2

A B
A B

Circuit 3 Circuit 4

Fig. 2.3 Fig. 2.4

The voltage and current readings are shown in the Table 2.1.

Table 2.1

Circuit V/ I/ R/

1 1.87 1.68

2 1.84 0.84

3 1.87 0.37

4 1.91 0.20

(i) Complete the column headings for each of the V, I and R columns of Table 2.1.

© UCLES 2008 0625/06/M/J/08


7

(ii) For each circuit, calculate the combined resistance R of the three resistors using For
the equation Examiner’s
Use

R = V.
I

Record these values of R in Table 2.1. [3]

(b) Theory suggests that, if all three resistors have the same resistance under all conditions,
the combined resistance in circuit 1 will be one half of the combined resistance in
circuit 2.

(i) State whether, within the limits of experimental accuracy, your results support this
theory. Justify your answer by reference to the results.

statement .................................................................................................................

justification ................................................................................................................

..................................................................................................................................

(ii) Suggest one precaution you could take to ensure that the readings are as accurate
as possible.

..................................................................................................................................

.............................................................................................................................. [3]

[Total: 6]

© UCLES 2008 0625/06/M/J/08 [Turn over


8

3 A student is investigating the effect of surface area exposed to the air on the rate of cooling For
of hot water. Examiner’s
Use

thermometer

thermometer

100 cm3

beaker water

100 cm3 measuring


cylinder
water

A B

Fig. 3.1

The student is provided with two containers. The beaker is labelled A and the measuring
cylinder is labelled B. Each container contains 100 cm3 of hot water. He records the
temperature of the water at 30 s intervals for a total of four minutes. Table 3.1 shows the
readings of time t and temperature θ.

Table 3.1
container A container B
(beaker) (measuring cylinder)
t /s θ / °C θ / °C
0 85 85

30 76 79

60 68 74

90 63 69

120 59 66

150 56 63

180 54 61

210 52 59

240 51 58

© UCLES 2008 0625/06/M/J/08


9

(a) (i) Use the data in Table 3.1 to plot a graph of θ / °C (y-axis) against t /s (x-axis) for the For
beaker. Draw the best-fit curve. Examiner’s
Use

(ii) Use the data for the measuring cylinder to plot another curve on the same graph
axes that you used for part (a)(i).

0 20 40 60 80 100 120 140 160 180 200 220 240


t /s
[6]

(b) The experiment is designed to investigate the effect of the surface area exposed to the
air on the rate of cooling. State briefly the effect of a larger surface area on the rate of
cooling. Justify your answer by reference to your graph.

statement ..........................................................................................................................

justification ........................................................................................................................

...................................................................................................................................... [2]

[Total: 8]

© UCLES 2008 0625/06/M/J/08 [Turn over


10

4 A student is determining a quantity called the refractive index of the material of a transparent For
block. Examiner’s
Use

Fig. 4.1 shows the ray-tracing sheet that the student is producing. ABCD is the outline of the
transparent block, drawn on the ray-tracing sheet.

A B

D C
P3

P4

Fig. 4.1

(a) (i) Draw the normal NN' to side AB, extended to cross side DC, so that the normal is
2.0 cm from A. Label the point F where NN' crosses AB. Label the point G where
NN' crosses DC.

(ii) Draw the line EF at an angle of 30° to the normal and to the left of the normal NN'.
E is a point outside the block and above AB on the ray-tracing sheet.
[3]

© UCLES 2008 0625/06/M/J/08


11

(b) Read the following passage, taken from the student’s notebook and then answer the For
questions that follow. Examiner’s
Use

I placed two pins P1 and P2 on line EF.


I observed the images of P1 and P2 through side CD of the block so that
the images of P1 and P2 appeared one behind the other. I placed two
more pins P3 and P4 between my eye and the block so that P3, P4 and the
images of P1 and P2, seen through the block, appeared one behind the
other. I marked the positions of P1, P2, P3 and P4.

(i) Draw a line joining the positions of P3 and P4. Continue the line until it meets CD.
Label this point H.

(ii) Measure and record the length a of the line GH.

a = ......................................................

(iii) Draw the line HF.

(iv) Measure and record the length b of the line HF.

b = .................................................. [3]

(c) Extend the straight line EF through the outline of the block to a point J. The point J must
be at least 5 cm from the block. The line EJ crosses the line CD. Label this point K.

(i) Measure and record the length c of the line GK.

c = ......................................................

(ii) Measure and record the length d of the line FK.

d = ......................................................

(iii) Calculate the refractive index n of the material of the block using the equation
cb
n= .
ad

n = .................................................. [3]

[Total: 9]

© UCLES 2008 0625/06/M/J/08 [Turn over


12

5 An IGCSE student has carried out a timing experiment using a simple pendulum. She plotted For
a graph of T 2/s2 against l /m. T is the time for one swing of the pendulum and l is the length Examiner’s
of the pendulum. The graph is shown below. Use

T 2 / s2

0
0 0.2 0.4 0.6 0.8 1.0 1.1
l /m

(a) (i) Determine the gradient G of the graph. Show clearly on the graph how you obtained
the necessary information.

G = ......................................................

(ii) Calculate the acceleration g of free fall using the equation


2
g = 4π .
G

g = ...............................................m/s2

(iii) The student could have calculated the acceleration of free fall g from just one set of
readings. State the purpose of taking sufficient readings to plot a graph.

..................................................................................................................................

.............................................................................................................................. [5]
© UCLES 2008 0625/06/M/J/08
13

(b) The student next studies the relationship between the mass m of the pendulum and the For
time for one swing T. The readings are shown in Table 5.1. Examiner’s
Use

Table 5.1

m /g T /s

50 1.58

100 1.60

150 1.61

200 1.57

250 1.59

(i) Suggest two variables that must be kept constant to make the experiment a fair
test.

1. ..............................................................................................................................

2. ..............................................................................................................................

(ii) Study the readings in the table and complete the following sentence.

Within the limits of experimental accuracy, the readings show that the mass m of

the pendulum ....................................................................................................... [3]

[Total: 8]

© UCLES 2008 0625/06/M/J/08


14

BLANK PAGE

0625/06/M/J/08
15

BLANK PAGE

0625/06/M/J/08
16

BLANK PAGE

Permission to reproduce items where third-party owned material protected by copyright is included has been sought and cleared where possible. Every
reasonable effort has been made by the publisher (UCLES) to trace copyright holders, but if any items requiring clearance have unwittingly been included, the
publisher will be pleased to make amends at the earliest possible opportunity.

University of Cambridge International Examinations is part of the Cambridge Assessment Group. Cambridge Assessment is the brand name of University of
Cambridge Local Examinations Syndicate (UCLES), which is itself a department of the University of Cambridge.

0625/06/M/J/08
UNIVERSITY OF CAMBRIDGE INTERNATIONAL EXAMINATIONS
International General Certificate of Secondary Education
*2092097264*

PHYSICS 0625/06
Paper 6 Alternative to Practical October/November 2008
1 hour
Candidates answer on the Question Paper.
No Additional Materials are required.

READ THESE INSTRUCTIONS FIRST

Write your Centre number, candidate number and name on all work you hand in.
Write in dark blue or black pen.
You may use a soft pencil for any diagrams, graphs or rough working.
Do not use staples, paper clips, highlighters, glue or correction fluid.
DO NOT WRITE IN ANY BARCODES.

Answer all questions.

At the end of the examination, fasten all your work securely together.
The number of marks is given in brackets [ ] at the end of each question or part question.

For Examiner’s Use

Total

This document consists of 12 printed pages.

SP (SLM/CGW) T73783/6 R
© UCLES 2008 [Turn over
2

1 An IGCSE student is determining the density of the metal from which a load is made. For
Examiner’s
The apparatus is shown in Fig. 1.1. Use

metre rule

spring
S0

beaker

water

bench

Fig. 1.1

(a) The student records the scale reading S0 on the metre rule at the bottom of the spring,
as shown in Fig. 1.1.

S0 = 37.4 cm

Describe briefly how the student can avoid a parallax error when taking the scale
reading.

..........................................................................................................................................

...................................................................................................................................... [1]

© UCLES 2008 0625/06/O/N/08


3

(b) He then hangs the load on the spring as shown in Fig. 1. 2. He records the new scale For
reading S1. Examiner’s
Use

0 metre rule

spring
S1
load

bench

Fig. 1.2

S1 = 40.5 cm

(i) Calculate the extension e1 of the spring using the equation

e1 = (S1 – S0).

e1 = ......................................................

The student carefully raises the beaker under the load until it is completely under water.
The load does not touch the sides or base of the beaker. He records the new scale
reading S2.

S2 = 39.8 cm

(ii) Calculate the extension e2 of the spring using the equation e2 = (S2 – S0).

e2 = ......................................................
[2]

© UCLES 2008 0625/06/O/N/08 [Turn over


4

(c) Calculate the density ρ of the material of the load using the equation For
Examiner’s
e1 Use
ρ= ×k
(e1 – e2)
where k = 1.00 g/cm3.

ρ = ................................................... [3]

(d) A second load, made from the same material and with the same mass, is too long to be
completely submerged in the water.

Suggest whether

(i) the value obtained for e2 would be greater, smaller or the same as that obtained in
part (b) (ii),

..................................................................................................................................

(ii) the value obtained for ρ would be greater, smaller or the same as that obtained in
part (c).

..................................................................................................................................
[2]

[Total: 8]

© UCLES 2008 0625/06/O/N/08


5

2 The IGCSE class is investigating the potential difference across, and the current in, wires. For
Examiner’s
The apparatus is shown in Fig. 2.1. Use

power source

A S B

C D

Fig. 2.1

(a) Draw a circuit diagram of the apparatus. Use standard circuit symbols.
(The circuit includes two identical resistance wires AB and CD. Use the standard symbol
for a resistance to represent each of these wires.)
This circuit is called circuit 1.

[3]

© UCLES 2008 0625/06/O/N/08 [Turn over


6

For circuit 1, the student places the contact S on the resistance wire AB at a distance of For
0.500 m from A. He measures the p.d. V across the wire between A and S and the current I Examiner’s
in the circuit. Use

The student then records the measurements for circuits 2 and 3, shown in Fig. 2.2 and
Fig. 2.3.

power source

A S B

C D
circuit 2

Fig. 2.2

power source

A S B

C D
circuit 3

Fig. 2.3

© UCLES 2008 0625/06/O/N/08


7

The voltage V and current I for all three circuits are shown in Table 2.1. For
Examiner’s
Table 2.1 Use

Circuit V/ I/
1 0.83 0.53
2 0.75 0.95
3 0.41 0.28

(b) Complete the column headings in the table. [1]

(c) Theory suggests that,

1. in circuits 1 and 2, the values of potential difference V will be equal,

2. the value of potential difference V in circuit 3 will be half that in circuit 1 or circuit 2.

(i) State whether, within the limits of experimental accuracy, the results support these
predictions.

Justify your statement by reference to the results.

Prediction 1 ...............................................................................................................

..................................................................................................................................

Prediction 2 ...............................................................................................................

.............................................................................................................................. [2]

(ii) Suggest one reason, other than a change in temperature of the wires, why the
results may not support the theory.

..................................................................................................................................

.............................................................................................................................. [1]

[Total: 7]

© UCLES 2008 0625/06/O/N/08 [Turn over


8

3 The IGCSE class is investigating the change in temperature of hot water as cold water is For
added to the hot water. Examiner’s
Use

A student measures and records the temperature θ of the hot water before adding any of the
cold water available.

He then pours 20 cm3 of the cold water into the beaker containing the hot water. He measures
and records the temperature θ of the mixture of hot and cold water.

He repeats this procedure four times until he has added a total of 100 cm3 of cold water.

The temperature readings are shown in Table 3.1. V is the volume of cold water added.

Table 3.1

V/ θ/
0 82

68

58

50

45

42

(a) (i) Complete the column headings in the table.

(ii) Enter the values for the volume of cold water added.
[2]

© UCLES 2008 0625/06/O/N/08


9

(b) Use the data in the table to plot a graph of temperature (y-axis) against volume (x-axis). For
Draw the best-fit curve. Examiner’s
Use

[4]

(c) During this experiment, some heat is lost from the hot water to the surroundings. Also,
each time the cold water is added, it is added in quite large volumes and at random
times.

Suggest two improvements you could make to the procedure to give a graph that more
accurately shows the pattern of temperature change of the hot water, due to addition of
cold water alone.

1. .....................................................................................................................................

..........................................................................................................................................

2. .....................................................................................................................................

...................................................................................................................................... [2]

[Total: 8]

© UCLES 2008 0625/06/O/N/08 [Turn over


10

4 An IGCSE student is determining the focal length of a converging lens. For


The apparatus is shown in Fig. 4.1. Examiner’s
Use

screen

x y

lens

illuminated
object

Fig. 4.1

(a) The student places the lens at a distance x = 25.0 cm from the illuminated object. She
places the screen close to the lens and then moves it away from the lens until a sharply
focused image is formed on the screen. She measures and records the distance y
between the lens and the screen.

y = 37.1 cm

Calculate the focal length f of the lens using the equation


xy .
f=
(x + y)

f = ................................................... [2]

(b) She then repeats the procedure with the lens at a distance x = 30.0 cm from the
illuminated object.

Fig. 4.1 shows this position of the apparatus. It is a scale diagram.

(i) On Fig. 4.1, measure the distance xs between the lens and the illuminated object.
Also on Fig. 4.1, measure the distance ys between the lens and the screen.

xs = ......................................................

ys = ......................................................

© UCLES 2008 0625/06/O/N/08


11

(ii) Calculate the actual distance y between the lens and the screen. For
Examiner’s
Use

y = ......................................................

(iii) Calculate the focal length f using the new values of x and y.

f = ......................................................

(iv) Calculate the average value of f. Show your working.

average value of f = ......................................................


[7]

(c) The illuminated object has the shape shown below.

Draw a diagram to show the appearance of the focused image in (b) on the screen.

[1]

[Total: 10]

© UCLES 2008 0625/06/O/N/08 [Turn over


12

5 (a) Table 5.1 shows some measurements taken by three IGCSE students. The second For
column shows the values recorded by the three students. For each quantity, underline Examiner’s
the value most likely to be correct. Use

The first one is done for you.

Table 5.1

Quantity measured Recorded values


The mass of a wooden 0.112 kg
metre rule 1.12 kg
11.2 kg
The weight of an empty 250 cm3 0.7 N
glass beaker 7.0 N
70 N
The volume of one sheet of this 0.6 cm3
examination paper 6.0 cm3
60 cm3
The time taken for one swing of a 0.14 s
simple pendulum of length 0.5 m 1.4 s
14 s
The pressure exerted on the ground 0.4 N/cm2
by a student standing on one foot 4.0 N/cm2
40 N/cm2
[4]

(b) (i) A student is to find the value of the resistance of a wire by experiment. Potential
difference V and current I can be recorded. The resistance is then calculated using
the equation R = V/ I.

The student knows that an increase in temperature will affect the resistance of the
wire. Assuming that variations in room temperature will not have a significant effect,
suggest two ways by which the student could minimise temperature increases in
the wire during the experiment.

1. ..............................................................................................................................

2. .......................................................................................................................... [2]

(ii) Name the circuit component that the student could use to control the current.

.............................................................................................................................. [1]

[Total: 7]

Permission to reproduce items where third-party owned material protected by copyright is included has been sought and cleared where possible. Every
reasonable effort has been made by the publisher (UCLES) to trace copyright holders, but if any items requiring clearance have unwittingly been included, the
publisher will be pleased to make amends at the earliest possible opportunity.

University of Cambridge International Examinations is part of the Cambridge Assessment Group. Cambridge Assessment is the brand name of University of
Cambridge Local Examinations Syndicate (UCLES), which is itself a department of the University of Cambridge.

© UCLES 2008 0625/06/O/N/08


UNIVERSITY OF CAMBRIDGE INTERNATIONAL EXAMINATIONS
International General Certificate of Secondary Education
*6752477947*

PHYSICS 0625/06
Paper 6 Alternative to Practical May/June 2009
1 hour
Candidates answer on the Question Paper.
No Additional Materials are required.

READ THESE INSTRUCTIONS FIRST

Write your Centre number, candidate number and name on all work you hand in.
Write in dark blue or black pen.
You may use a pencil for any diagrams, graphs or rough working.
Do not use staples, paper clips, highlighters, glue or correction fluid.
DO NOT WRITE IN ANY BARCODES.

Answer all questions.

At the end of the examination, fasten all your work securely together.
The number of marks is given in brackets [ ] at the end of each question or part question.

For Examiner’s Use

Total

This document consists of 11 printed pages and 1 blank page.

SPA FF/DT T76304/3


© UCLES 2009 [Turn over
2

1 An IGCSE student is making measurements as accurately as possible in order to determine For


the density of glass. Examiner’s
Use

Fig. 1.1 shows a glass test-tube drawn actual size.

Fig. 1.1

(a) (i) Use your rule to measure, in cm, the external diameter d of the test-tube.

d = ....................................... cm

© UCLES 2009 0625/06/M/J/09


3

(ii) Use your rule to measure, in cm, the length x of the test-tube. For
Examiner’s
x = ............................................ Use

(iii) Draw a labelled diagram to show how you would use two rectangular blocks of wood
and your rule to measure the length x of the test-tube as accurately as possible.

[4]

(b) The mass m of the test-tube is 31.2 g.

(i) Calculate the external volume Ve of the test-tube using the equation

πd 2X .
Ve =
4

Ve = ............................................

© UCLES 2009 0625/06/M/J/09 [Turn over


4

(ii) The student then fills the test-tube with water and pours the water into a measuring For
cylinder. Fig. 1.2 shows the measuring cylinder. Examiner’s
Use

cm3
100

90

80

70

60

50

40
water
30

20

10

Fig. 1.2

Record the volume reading Vi from the measuring cylinder. This is the internal
volume of the test-tube.

Vi = ............................................

(iii) Calculate the density ρ of the glass from which the test-tube is made using the
equation

m
ρ= .
(Ve − Vi)

ρ = ....................................... [4]

[Total: 8]

© UCLES 2009 0625/06/M/J/09


5

BLANK PAGE

0625/06/M/J/09 [Turn over


6

2 The IGCSE class is investigating the cooling of thermometer bulbs under different conditions. For
Examiner’s
The students are provided with two thermometers A and B. Thermometer B has cotton wool Use

wrapped around the bulb. Fig. 2.1 shows thermometer A.

thermometer A

stand

water

Fig. 2.1

The students measure the temperature θ of the hot water in the beaker. Fig. 2.2 shows the
thermometer reading.

-10 0 10 20 30 40 50 60 70 80 90 100 100°C

thermometer A

Fig. 2.2

(a) Record in Table 2.1 at time t = 0 s the temperature θ shown in Fig. 2.2.

(b) The students remove the thermometer from the water, starting the stopclock at the same
time. Table 2.1 shows the temperature of the thermometer bulb at 30 s intervals. The
experiment is repeated using thermometer B which has cotton wool wrapped around
the thermometer bulb.

© UCLES 2009 0625/06/M/J/09


7

Complete Table 2.1 by inserting the appropriate unit in the time and in the temperature For
column headings. Examiner’s
Use

Table 2.1

Thermometer A Thermometer B

t/ θ/ θ/

0 81

30 51 72

60 43 58

90 37 49

120 34 43

150 30 38

180 28 34

210 27 31
[2]

(c) Suggest which thermometer cooled more quickly at first. Justify your answer by reference
to the readings.

statement .........................................................................................................................

justification .......................................................................................................................

..................................................................................................................................... [2]

(d) To make a fair comparison between the rates of cooling of the two thermometer bulbs
under different conditions (in this experiment one thermometer bulb is covered with cotton
wool), it is important to control other experimental conditions. Suggest two conditions that
should be controlled in this experiment.

1. ......................................................................................................................................

2. ................................................................................................................................. [2]

[Total: 6]

© UCLES 2009 0625/06/M/J/09 [Turn over


8

3 The IGCSE class is investigating the resistance of a wire. For


Examiner’s
The circuit is shown in Fig. 3.1. Use

power
source

A
x
A B
C

Fig. 3.1

AB is a resistance wire. The students place the sliding contact C on the resistance wire AB
at a distance x = 0.100 m from A. They switch on and measure the p.d. V across the wire
between A and C. They also measure the current I in the wire. The value of I is 0.38 A.

They repeat the procedure several times using different values of x. The readings are shown
in Table 3.1. The current I is 0.38 A for each value of x.

Table 3.1

x/m V /V R/Ω

0.100 0.21

0.300 0.59

0.500 1.04

0.700 1.42

0.900 1.87
[2]

(a) Calculate the resistance R of the section AC of the wire for each value of x using the
equation R = V– . Record the values of R in the table.
I

© UCLES 2009 0625/06/M/J/09


9

(b) Use the results in Table 3.1 to plot a graph of R / Ω (y-axis) against x / m (x-axis). Draw For
the best fit line. Examiner’s
Use

[5]

(c) Within the limits of experimental accuracy, what do you conclude about the variation of
resistance with distance along the wire? Justify your conclusion by reference to your
graph.

statement .........................................................................................................................

justification .......................................................................................................................

..................................................................................................................................... [2]

(d) Using your graph, determine the value for R when x = 0.750 m. Show clearly on your
graph how you obtained the necessary information.

R = ....................................... [2]

(e) A variable that may be difficult to control in this experiment is the heating effect of the
current, which affects the resistance of the wire. Suggest how you would minimise the
heating effect.

..........................................................................................................................................

..................................................................................................................................... [1]

[Total: 12]

© UCLES 2009 0625/06/M/J/09 [Turn over


10

4 An IGCSE student is determining the focal length of a lens. For


Examiner’s
Fig. 4.1 shows the experimental set-up. The student positions the illuminated object and the Use

lens and then moves the screen away from the lens until a sharply focused image of the
object is formed on the screen.

illuminated
u v
object screen

Fig. 4.1

(a) Using your rule, measure on Fig. 4.1 the distance u, in cm, from the centre of the lens to
the illuminated object and the distance v from the centre of the lens to the screen.

u = ...........................................

v =............................................ [2]

(b) (i) Fig. 4.1 is drawn one fifth actual size. Calculate the actual distance x from the
illuminated object to the centre of the lens and the actual distance y from the centre of
the lens to the screen.

Record these values in Table 4.1. The first pair of readings obtained by the student
has already been entered in the table.

Table 4.1

x / cm y / cm f / cm

57.0 15.0

[3]

(ii) Calculate for both pairs of readings the focal length f of the lens using the equation

xy .
f=
(x + y )

Record the values of f in Table 4.1.

© UCLES 2009 0625/06/M/J/09


11

(c) Calculate the average value of the focal length. For


Examiner’s
Use

average value for the focal length = ....................................... [2]

(d) State two precautions you would take in the laboratory in order to obtain reliable
measurements.

1. ......................................................................................................................................

2. ................................................................................................................................. [2]

[Total: 9]

© UCLES 2009 0625/06/M/J/09 [Turn over


12

5 An IGCSE student is investigating moments using a simple balancing experiment. For


Examiner’s
He uses a pivot on a bench as shown in Fig. 5.1. Use

pivot bench

Fig. 5.1

First, the student balances the metre rule, without loads, on the pivot. He finds that it does
not balance at the 50.0 cm mark, as he expects, but it balances at the 49.7 cm mark.

Load Q is a metal cylinder with diameter a little larger than the width of the metre rule, so
that it covers the markings on the rule. Load Q is placed carefully on the balanced metre rule
with its centre at the 84.2 cm mark. The rule does not slip on the pivot.

(a) Draw on Fig. 5.1 the metre rule with load Q on it. [2]

(b) Explain, using a labelled diagram, how the student would ensure that the metre rule
reading at the centre of Q is 84.2 cm.

[2]

(c) Calculate the distance between the pivot and the centre of load Q.

distance = ....................................... [1]


[Total: 5]

Permission to reproduce items where third-party owned material protected by copyright is included has been sought and cleared where possible. Every reasonable
effort has been made by the publisher (UCLES) to trace copyright holders, but if any items requiring clearance have unwittingly been included, the publisher will
be pleased to make amends at the earliest possible opportunity.

University of Cambridge International Examinations is part of the Cambridge Assessment Group. Cambridge Assessment is the brand name of University of
Cambridge Local Examinations Syndicate (UCLES), which is itself a department of the University of Cambridge.
© UCLES 2009 0625/06/M/J/09
UNIVERSITY OF CAMBRIDGE INTERNATIONAL EXAMINATIONS
International General Certificate of Secondary Education
*8413808010*

PHYSICS 0625/06
Paper 6 Alternative to Practical October/November 2009
1 hour
Candidates answer on the Question Paper.
No Additional Materials are required.

READ THESE INSTRUCTIONS FIRST

Write your Centre number, candidate number and name on all the work you hand in.
Write in dark blue or black pen.
You may use a soft pencil for any diagrams, graphs or rough working.
Do not use staples, paper clips, highlighters, glue or correction fluid.
DO NOT WRITE IN ANY BARCODES.

Answer all questions.

At the end of the examination, fasten all your work securely together.
The number of marks is given in brackets [ ] at the end of each question or part question.

For Examiner’s Use

Total

This document consists of 14 printed pages and 2 blank pages.

DCA (SHW 00210 3/08) 12805/3


© UCLES 2009 [Turn over
2

1 The IGCSE class is investigating the period of oscillation of a simple pendulum. For
Examiner’s
Fig. 1.1 shows the set-up. Use

bob
d
floor
one complete
oscillation

Fig. 1.1 Fig. 1.2

(a) (i) On Fig. 1.1, measure the vertical distance d from the floor to the bottom of the
pendulum bob.

d = ......................................................

(ii) Fig. 1.1 is drawn one twentieth actual size. Calculate the actual distance x from
the floor to the bottom of the pendulum bob. Enter this value in the top row of
Table 1.1.

The students displace the pendulum bob slightly and release it so that it swings. They
measure and record in Table 1.1 the time t for 20 complete oscillations of the pendulum
(see Fig. 1.2).

Table 1.1

x / cm t/s T/s T 2 / s2

20.0

20.0 19.0

30.0 17.9

40.0 16.8

50.0 15.5

[4]

© UCLES 2009 0625/06/O/N/09


3

(b) (i) Calculate the period T of the pendulum for each set of readings. The period is the For
time for one complete oscillation. Enter the values in Table 1.1. Examiner’s
Use

(ii) Calculate the values of T 2. Enter the T 2 values in Table 1.1.

(c) Use your values from Table 1.1 to plot a graph of T 2 / s2 (y-axis) against x / cm (x-axis).
Draw the best-fit line.

[5]

© UCLES 2009 0625/06/O/N/09 [Turn over


4

(d) State whether or not your graph shows that T 2 is directly proportional to x. Justify your For
statement by reference to the graph. Examiner’s
Use

statement .........................................................................................................................

justification .......................................................................................................................

.................................................................................................................................... [1]

[Total: 10]

© UCLES 2009 0625/06/O/N/09


5

BLANK PAGE

0625/06/O/N/09 [Turn over


6

2 An IGCSE student is investigating the cooling of thermometer bulbs under different For
conditions. Examiner’s
Use

He places a thermometer in a beaker of hot water and records the temperature h of the hot
water.

–10 0 10 20 30 40 50 60 70 80 90 100 110 °C

Fig. 2.1

(a) Fig. 2.1 shows the thermometer. Write down the value of h that it shows.

h ................................................ [1]

He then moves the thermometer until the thermometer bulb is just above the surface of the
water (position A) and immediately starts a stopclock.

He records the time t and the temperature reading  every 30 s. The readings are shown in
Table 2.1.

Table 2.1

Position A Position B

t/ / /
30 65 56

60 58 47

90 54 40

120 52 35

150 50 32

180 48 30

(b) Complete the column headings in the table. [1]

© UCLES 2009 0625/06/O/N/09


7

The student replaces the thermometer in the hot water and then moves the thermometer For
15 cm away from the beaker to position B and immediately starts the stopclock. He records Examiner’s
the time t and the temperature reading  every 30 s. The readings are shown in Table 2.1. Use

(c) State in which position the thermometer bulb cooled more quickly. Justify your answer
by reference to the readings.

statement .........................................................................................................................

justification .......................................................................................................................

.................................................................................................................................... [1]

(d) To make a fair comparison between the rates of cooling of the thermometer bulbs in
the two positions, it is important to control other experimental conditions. Suggest two
conditions that should be controlled in this experiment.

1. ......................................................................................................................................

2. ................................................................................................................................ [2]

[Total: 5]

© UCLES 2009 0625/06/O/N/09 [Turn over


8

3 The IGCSE class is comparing the combined resistance of lamps arranged either in series For
or in parallel. Examiner’s
Use

The circuit shown in Fig. 3.1 is used.

power
source

Fig. 3.1

A student measures and records the current I in the circuit and the p.d. V across the two
lamps.

Fig. 3.2 shows the readings on the two meters.

0.2 0.3
0.1 0.4

0 0.5
A

2 3
1 4

0 5
V
Fig. 3.2

© UCLES 2009 0625/06/O/N/09


9

(a) (i) Write the voltage and current readings in Table 3.1, below. For
Examiner’s
(ii) Complete the column headings in Table 3.1. Use
[3]

(b) The student then sets up the circuit shown in Fig. 3.3 and records the readings. These
readings have already been entered in Table 3.1.

power
source

Fig. 3.3

For each set of readings in the table, calculate the combined resistance R of the two
lamps using the equation R = V / I. Record the values of R in Table 3.1. [2]

Table 3.1

V/ I/ R/

Circuit of Fig. 3.1

Circuit of Fig. 3.3 1.8 0.52

(c) Using the values of resistance you have obtained, calculate the ratio y of the resistances
using the equation
resistance of lamps in series
y= .
resistance of lamps in parallel

y = ......................................................
[2]

© UCLES 2009 0625/06/O/N/09 [Turn over


10

(d) Fig. 3.4 shows a circuit including two motors A and B. For
Examiner’s
Use

power source

ammeter
motor A
A

variable resistor
motor B

Fig. 3.4

(i) Draw a diagram of the circuit using standard circuit symbols. The circuit symbol for
a motor is:

© UCLES 2009 0625/06/O/N/09


11

(ii) An engineer wishes to measure the voltage across motor A. For


Examiner’s
1. On Fig. 3.4, mark with the letters X and Y where the engineer should connect Use

the voltmeter.

2. State the purpose of the variable resistor.

...........................................................................................................................

...........................................................................................................................
[3]

[Total: 10]

© UCLES 2009 0625/06/O/N/09 [Turn over


12

4 An IGCSE student is determining the focal length of a lens by two different methods. For
Examiner’s
The set-up for Method 1 is shown in Fig. 4.1. Use

object
illuminated screen mirror
object f

lens

Fig. 4.1

The student moves the lens and the mirror slowly towards the object screen until a sharply
focused image is obtained on the object screen as shown in Fig. 4.2.

image

Fig. 4.2

(a) On Fig. 4.1, use your rule to measure the distance f between the lens and the object
screen. This is the focal length of the lens.

f = ................................................ [2]

(b) For Method 2, the student takes measurements of the diameter d and maximum
thickness t of the lens. Use your rule to take measurements on Fig. 4.3.

Fig. 4.3

© UCLES 2009 0625/06/O/N/09


13

(i) Determine an average value for the diameter d of the lens. Record your readings in For
the space below. Examiner’s
Use

d = ......................................................

(ii) Measure the maximum thickness t of the lens.

t = ......................................................

(iii) Draw a diagram to show how, in the laboratory you would use two rectangular
blocks of wood and a metre rule to measure the thickness of the lens as accurately
as possible.

(iv) Theory shows that, for a perfectly formed lens, the focal length is given by the
formula
d2
f= where k = 4.16.
kt
Calculate the focal length f of the lens using this formula.

f = ......................................................
[7]

© UCLES 2009 0625/06/O/N/09 [Turn over


14

(c) Explain whether your results from Methods 1 and 2 support the theory in part (b)(iv). For
Examiner’s
.......................................................................................................................................... Use

..........................................................................................................................................

.................................................................................................................................... [1]

[Total: 10]

© UCLES 2009 0625/06/O/N/09


15

5 An IGCSE student is carrying out an optics experiment. For


Examiner’s
The experiment involves using a lens to focus the image of an illuminated object onto a Use

screen.

(a) Complete the diagram below to show the apparatus you would use. Include a metre rule
to measure the distances between the object and the lens and between the lens and
the screen. The illuminated object is drawn for you.

illuminated
object

lamp

card

[3]

(b) State two precautions that you would take to obtain accurate results in this experiment.

1. ......................................................................................................................................

..........................................................................................................................................

2. ......................................................................................................................................

.................................................................................................................................... [2]

[Total: 5]

© UCLES 2009 0625/06/O/N/09


16

BLANK PAGE

Permission to reproduce items where third-party owned material protected by copyright is included has been sought and cleared where possible. Every
reasonable effort has been made by the publisher (UCLES) to trace copyright holders, but if any items requiring clearance have unwittingly been included, the
publisher will be pleased to make amends at the earliest possible opportunity.

University of Cambridge International Examinations is part of the Cambridge Assessment Group. Cambridge Assessment is the brand name of University of
Cambridge Local Examinations Syndicate (UCLES), which is itself a department of the University of Cambridge.
0625/06/O/N/09
UNIVERSITY OF CAMBRIDGE INTERNATIONAL EXAMINATIONS
International General Certificate of Secondary Education
*5429162966*

PHYSICS 0625/61
Paper 6 Alternative to Practical May/June 2010
1 hour
Candidates answer on the Question Paper.
No Additional Materials are required.

READ THESE INSTRUCTIONS FIRST

Write your Centre number, candidate number and name on all the work you hand in.
Write in dark blue or black pen.
You may use a pencil for any diagrams, graphs or rough working.
Do not use staples, paper clips, highlighters, glue or correction fluid.
DO NOT WRITE IN ANY BARCODES.

Answer all questions.

At the end of the examination, fasten all your work securely together.
The number of marks is given in brackets [ ] at the end of each question or part question.

For Examiner’s Use

Total

This document consists of 11 printed pages and 1 blank page.

DC (KN/SW) 17179/5
© UCLES 2010 [Turn over
2

1 An IGCSE student is investigating the stretching of springs. For


Examiner’s
Fig. 1.1 shows the apparatus used for the first part of the experiment. Use

clamp

spring A spring B
l l

200 g mass
200 g mass

Fig. 1.1

The unstretched length lA of spring A is 15 mm.

The unstretched length lB of spring B is 16 mm.

(a) The student hangs a 200 g mass on each spring, as shown in Fig. 1.1.

(i) On Fig. 1.1 measure the new length l of spring A.

l = ..................................... mm

(ii) Calculate the extension eA of the spring using the equation eA = (l – lA).

eA = ..................................... mm

(iii) On Fig. 1.1 measure the new length l of spring B.

l = ..................................... mm

(iv) Calculate the extension eB of the spring using the equation eB = (l – lB).

eB = ..................................... mm
[2]
© UCLES 2010 0625/61/M/J/10
3

(b) The student then sets up the apparatus as shown in Fig. 1.2. For
Examiner’s
Use

spring A
l l spring B

rod

load

Fig. 1.2

(i) On Fig. 1.2 measure the new length of each of the springs.

spring A: l = ..................................... mm

spring B: l = ..................................... mm

(ii) Calculate the extension of each spring using the appropriate equation from
part (a).
spring A: e = ..................................... mm

spring B: e = ..................................... mm

(iii) Calculate the average of these two extensions eav . Show your working.

eav = ......................................mm
[3]
(c) It is suggested that (eA + eB)/4 = eav .

State whether your results support this theory and justify your answer with reference to
the results.

Statement ........................................................................................................................

Justification ......................................................................................................................

..................................................................................................................................... [2]

(d) Describe briefly one precaution that you would take to obtain accurate length
measurements.

..........................................................................................................................................

..........................................................................................................................................

..................................................................................................................................... [1]
© UCLES 2010 0625/61/M/J/10 [Turn over
4

2 The IGCSE class is investigating the cooling of water. For


Examiner’s
Fig. 2.1. shows the apparatus used. Use

thermometer

hot
water

Fig. 2.1

Hot water is poured into the beaker and temperature readings are taken as the water cools.

Table 2.1 shows the readings taken by one student.

Table 2.1

t /s θ / °C
0 85

30 78

60 74

90 71

120 69

150 67

300 63

(a) (i) Using the information in the table, calculate the temperature change T1 of the water
in the first 150 s.

T1 = ...........................................

© UCLES 2010 0625/61/M/J/10


5

(ii) Using the information in the table, calculate the temperature change T2 of the water For
in the final 150 s. Examiner’s
Use

T2 = ............................................
[3]

(b) Plot a graph of θ / °C (y-axis) against t / s (x-axis) for the first 150 s. [5]

0 20 40 60 80 100 120 140 160


t /s

(c) During the experiment the rate of temperature change decreases.

(i) Describe briefly how the results that you have calculated in part (a) show this
trend.

..................................................................................................................................

..................................................................................................................................

(ii) Describe briefly how the graph line shows this trend.

..................................................................................................................................

..................................................................................................................................
[2]

© UCLES 2010 0625/61/M/J/10 [Turn over


6

3 The IGCSE class is investigating the effect of the length of resistance wire in a circuit on the For
potential difference across a lamp. Examiner’s
Use

(a) Fig. 3.1 shows the circuit without the voltmeter. Complete the circuit diagram to show the
voltmeter connected in the circuit to measure the potential difference across the lamp.

power
source

l
A B
sliding
contact

Fig. 3.1
[2]

(b) A student switches on and places the sliding contact on the resistance wire at a distance
l = 0.200 m from end A. He records the value of l and the potential difference V across
the lamp.
He then repeats the procedure using a range of values of l. Table 3.1 shows the
readings.

Table 3.1

V
l/m V /V –
l /
0.200 1.67

0.400 1.43

0.600 1.25

0.800 1.11

1.000 1.00

(i) For each pair of readings in the table calculate and record in the table the value
of V
–.
l
(ii) Complete the table by writing in the unit for V
–.
l [3]

© UCLES 2010 0625/61/M/J/10


7

(c) A student suggests that the potential difference V across the lamp is directly proportional For
to the length l of resistance wire in the circuit. State whether or not you agree with this Examiner’s
suggestion and justify your answer by reference to the results. Use

Statement ........................................................................................................................

Justification ......................................................................................................................

......................................................................................................................................[2]

(d) State one precaution that you would take in order to obtain accurate readings of V in
this experiment.

..........................................................................................................................................

..........................................................................................................................................

......................................................................................................................................[1]

© UCLES 2010 0625/61/M/J/10 [Turn over


8

4 An IGCSE student is investigating reflection from a plane mirror. For


Examiner’s
Use

E G F

P2 P5

P6

P1
sheet of
paper
J K
H

Fig. 4.1

The student is using a sheet of plain paper on a pin board. Fig. 4.1 shows the sheet of paper.
The straight line EF shows the position of the reflecting surface of a plane mirror standing
vertically on the sheet of paper. Line GH is a normal to line EF. Line JG marks an incident ray
and line GK is the corresponding reflected ray. The student marks the position of the incident
ray with two pins (P1 and P2) and uses two more pins (P3 and P4) to find the direction of the
reflected ray.

(a) (i) On Fig. 4.1 mark with two neat crosses, labelled P3 and P4, suitable positions for
the pins to find the direction of the reflected ray.

(ii) On Fig. 4.1 measure the angle of incidence i.

i = ............................................

(iii) On Fig. 4.1 measure the angle of reflection r1.

r1 = ............................................
[3]

© UCLES 2010 0625/61/M/J/10


9

(b) (i) On Fig. 4.1 draw a line E'GF' such that the angle θ between this line and the line For
EGF is 10°. Start with E' below the line EGF. The straight line E'F' shows a new Examiner’s
position of the reflecting surface of the plane mirror standing vertically on the sheet Use

of paper.
The points labelled P5 and P6 mark the positions of two pins placed so that P5, P6
and the images of P1 and P2 appear in line with each other. P1 and P2 have not
been moved since the original set-up.

(ii) Using a ruler, draw a line joining the points labelled P5 and P6, and continue this
line to meet the line E'F'.

(iii) Measure the angle of reflection r2 between line GH and the line joining the points
labelled P5 and P6.

r2 = ............................................

(iv) Calculate the angle α through which the reflected ray has moved.

α = ............................................

(v) Calculate the difference between 2θ and α.


θ is the angle between the two positions of the mirror.

difference between 2θ and α = ............................................


[3]

(c) Theory suggests that if the mirror is moved through an angle θ then the reflected ray will
move through an angle of 2θ.
State whether your result supports the theory and justify your answer by reference to
the result.

Statement ........................................................................................................................

Justification ......................................................................................................................

......................................................................................................................................[2]

© UCLES 2010 0625/61/M/J/10 [Turn over


10

5 The IGCSE class is investigating the swing of a loaded metre rule. For
Examiner’s
The arrangement of the apparatus is shown in Fig. 5.1. Use

pivot at 10 cm mark

metre rule

load at 90 cm mark

Fig. 5.1

A student displaces the rule a small distance to one side and allows it to swing. The time t
taken for 10 complete swings is recorded. She calculates the time T taken for one swing. She
repeats the procedure using different values of the distance d.

The readings are shown in the Table 5.1.

Table 5.1

0.900 18.4 1.84

0.850 17.9 1.79

0.800 17.5 1.75

0.750 17.1 1.71

0.700 16.7 1.67

(a) Complete the column headings in the table. [3]

© UCLES 2010 0625/61/M/J/10


11

(b) Explain why the student takes the time for ten swings and then calculates the time for For
one swing, rather than just measuring the time for one swing. Examiner’s
Use

..........................................................................................................................................

......................................................................................................................................[1]

(c) The student tries to find a relationship between T and d. She first suggests that T × d is
a constant.

(i) Calculate the values of T × d and enter the values in the final column of the table.

(ii) State whether or not the results support this suggestion and give a reason for your
answer.

Statement .................................................................................................................

..................................................................................................................................

Reason .....................................................................................................................

..................................................................................................................................
[2]

© UCLES 2010 0625/61/M/J/10


12

BLANK PAGE

Permission to reproduce items where third-party owned material protected by copyright is included has been sought and cleared where possible. Every
reasonable effort has been made by the publisher (UCLES) to trace copyright holders, but if any items requiring clearance have unwittingly been included, the
publisher will be pleased to make amends at the earliest possible opportunity.

University of Cambridge International Examinations is part of the Cambridge Assessment Group. Cambridge Assessment is the brand name of University of
Cambridge Local Examinations Syndicate (UCLES), which is itself a department of the University of Cambridge.
© UCLES 2010 0625/61/M/J/10
w
w
w
.X
tr
me
eP
ap
UNIVERSITY OF CAMBRIDGE INTERNATIONAL EXAMINATIONS

er
s
International General Certificate of Secondary Education

.c
om
*0199664493*

PHYSICS 0625/61
Paper 6 Alternative to Practical October/November 2010
1 hour
Candidates answer on the Question Paper
No Additional Materials are required.

READ THESE INSTRUCTIONS FIRST

Write your Centre number, candidate number and name in the spaces at the top of the page.
Write in dark blue or black pen.
You may use a pencil for any diagrams, graphs or rough working.
Do not use staples, paper clips, highlighters, glue or correction fluid.
DO NOT WRITE IN ANY BARCODES.

Answer all questions.

At the end of the examination, fasten all your work securely together.
The number of marks is given in brackets [ ] at the end of each question or part question.

For Examiner’s Use

Total

This document consists of 10 printed pages and 2 blank pages.

DC (LEO/DJ) 25086/5
© UCLES 2010 [Turn over
2

1 A student is determining the position of the centre of mass of an object using a balancing For
method. Examiner’s
Use

Fig. 1.1 shows the apparatus used.

50.0 cm
mark
mass m d load of mass x

pivot
75.0 cm
mark

Fig. 1.1

A load of mass x is taped to the metre rule so that one side of the base is exactly on the
75.0 cm mark. The student places a mass m of 30 g on the rule and adjusts its position so
that the rule is as near as possible to being balanced with the 50.0 cm mark exactly over the
pivot, as shown in Fig. 1.1.

The student records the distance d from the centre of the 30 g mass to the 50.0 cm mark on
the rule. He then repeats the procedure using different masses. The readings are shown in
Table 1.1.

Table 1.1

1 1
m /g d / cm
d cm
30 45.0

40 34.0

50 27.0

60 22.5

70 19.3

(a) For each value of d, calculate 1/d and enter the values in the table. [2]

© UCLES 2010 0625/61/O/N/10


3

1 1
(b) Plot a graph of m / g (y-axis) against (x-axis). For
d cm Examiner’s
Use

[4]

(c) Determine the gradient G of the graph. Show clearly on the graph how you obtained the
necessary information.

G = .................................................. [2]

(d) Determine the horizontal distance z from the 75.0 cm mark on the rule to the centre of
mass of the load using the equation
z = G – k,
where k = 1250 g cm and x = 50 g. x

z = ................................................. [2]

[Total: 10]

© UCLES 2010 0625/61/O/N/10 [Turn over


4

2 The IGCSE class is investigating the rate of heating and cooling of a thermometer bulb. For
Examiner’s
The apparatus used is shown in Fig. 2.1. Use

thermometer

lid

water

Fig. 2.1

(a) Record the room temperature θr shown on the thermometer.

–10 0 10 20 30 40 50 60 70 80 90 100 110 °C

θr = ................................................. [1]

© UCLES 2010 0625/61/O/N/10


5

(b) For the cooling experiment, a student places the thermometer into hot water as shown in For
Fig. 2.1. When the temperature shown on the thermometer stops rising, she records the Examiner’s
temperature θ at time t = 0 s. She removes the thermometer from the water, immediately Use

starts a stopclock, and records the temperature shown on the thermometer at 30 s


intervals. The readings are shown in Table 2.1.

For the heating experiment, the student takes another thermometer and records the
temperature θ shown on the thermometer at time t = 0 s. She places the thermometer in
the beaker of hot water, immediately starts the stopclock, and records the temperature
shown by the thermometer at 10 s intervals. The readings are shown in Table 2.2.

Table 2.1 Table 2.2

t/ θ/ t/ θ/
0 74 0 25

30 60 10 69

60 52 20 80

90 45 30 81

120 39 40 81

150 35 50 82

180 33 60 82

(i) Complete the column headings in both tables. [1]

(ii) Estimate the time that would be taken in the cooling experiment for the thermometer
to cool from the reading at time t = 0 s to room temperature θr .

estimated time = ................................................. [1]

(c) State in which table the initial rate of temperature change is the greater. Justify your
answer by reference to your readings.

The initial rate of temperature change is greater in Table .................................................

justification .......................................................................................................................

...................................................................................................................................... [1]

(d) If one of these experiments were to be repeated in order to determine an average


temperature for each time, it would be important to control the conditions. Suggest two
such conditions that should be controlled.

1. ......................................................................................................................................

2. .................................................................................................................................. [2]

[Total: 6]

© UCLES 2010 0625/61/O/N/10 [Turn over


6

3 The IGCSE class is investigating the current in a circuit when different resistors are connected For
in the circuit. Examiner’s
Use

The circuit is shown in Fig. 3.1. The circuit contains a resistor X, and there is a gap in the
circuit between points A and B that is used for adding extra resistors to the circuit.

power source X
A

A B

Fig. 3.1

(a) A student connects points A and B together, switches on and measures the current I0 in
the circuit.

The reading is shown on the ammeter in Fig. 3.2.

0.4 0.6
0.2 0.8

0 A 1.0

Fig. 3.2

Write down the ammeter reading.

I0 = .................................................. [1]

© UCLES 2010 0625/61/O/N/10


7

(b) The student connects a 3.3 Ω resistor between points A and B, switches on and records For
the current I. He repeats the procedure with a 4.7 Ω resistor and then a 6.8 Ω resistor. Examiner’s
Use

Finally he connects the 3.3 Ω resistor and the 6.8 Ω resistor in series between points A
and B, and records the current I.

(i) Complete the column headings in the table. [1]

R/ I/
3.3 0.23

4.7 0.21

6.8 0.18

0.15

(ii) Write the combined resistance of the 3.3 Ω resistor and the 6.8 Ω resistor in series
in the space in the resistance column of the table. [1]

(c) Theory suggests that the current will be 0.5 I0 when the total resistance in the circuit
is twice the value of the resistance of resistor X. Use the readings in the table, and the
value of I0 from (a), to estimate the resistance of resistor X.

estimate of the resistance of resistor X = .................................................. [2]

(d) On Fig. 3.1 draw two resistors in parallel connected between A and B and also a
voltmeter connected to measure the potential difference across resistor X. [3]

[Total: 8]

© UCLES 2010 0625/61/O/N/10 [Turn over


8

4 The IGCSE class is investigating the reflection of light by a mirror as seen through a For
transparent block. Examiner’s
Use

Fig. 4.1 shows a student’s ray-trace sheet.

mirror

A B
N

transparent
block

E
D P3 C

P4

N'
F
eye

sheet of
paper

Fig. 4.1

© UCLES 2010 0625/61/O/N/10


9

(a) A student draws the outline of the transparent block ABCD on the ray-trace sheet. He For
draws the normal NN' to side CD. He draws the incident ray EF at an angle of incidence Examiner’s
i = 20°. He pushes two pins P1 and P2 into line EF and places the block on the sheet Use

of paper. He then observes the images of P1 and P2 through side CD of the block from
the direction indicated by the eye in Fig. 4.1 so that the images of P1 and P2 appear one
behind the other. He pushes two pins P3 and P4 into the surface, between his eye and
the block, so that P3, P4 and the images of P1 and P2, seen through the block, appear in
line. (The plane mirror along side AB of the block reflects the light.)

The positions of P3 and P4 are marked on Fig. 4.1.

(i) On line EF, mark with neat crosses (x) suitable positions for the pins P1 and P2.

(ii) Continue the line EF so that it crosses CD and extends as far as side AB.

(iii) Draw a line joining the positions of P4 and P3. Continue the line so that it crosses
CD and extends as far as side AB. Label the point G where this line crosses the
line from P1 and P2. [4]

(iv) Measure the acute angle θ between the lines meeting at G.

θ = ......................................................

(v) Calculate the difference (θ – 2i ).

(θ – 2i ) = ................................................. [2]

(b) The student repeats the procedure using an angle of incidence i = 30° and records the
value of θ as 62°.

(i) Calculate the difference (θ – 2i).

(θ – 2i ) = ......................................................

(ii) Theory suggests that θ = 2i . State whether the results support the theory and justify
your answer by reference to the results.

statement .................................................................................................................

justification ...............................................................................................................

.............................................................................................................................. [3]

(c) To place the pins as accurately as possible, the student views the bases of the pins.
Explain briefly why viewing the bases of the pins, rather than the tops of the pins,
improves the accuracy of the experiment.

..........................................................................................................................................

..........................................................................................................................................

...................................................................................................................................... [1]

[Total: 10]
© UCLES 2010 0625/61/O/N/10 [Turn over
10

5 The IGCSE class is investigating the time taken for ice cubes to melt when placed in water. For
Examiner’s
Each student is able to use Use

glass beakers,
a thermometer,
a stopclock,
a measuring cylinder,
an electronic balance,
a supply of ice cubes of different sizes,
a supply of cold water,
a stirrer,
a method of heating the water
and any other common laboratory apparatus that may be useful.

A student decides to investigate the effect of the mass of ice cubes on the time they take to
melt in water.

(a) Suggest three possible variables that should be kept constant in this investigation.

1. ......................................................................................................................................

2. ......................................................................................................................................

3. .................................................................................................................................. [3]

(b) In the table below, write the names of three items of apparatus that are necessary in
order to take readings in this investigation. In the second column of the table write the
quantity that the item measures.

item of apparatus quantity measured

[3]

[Total: 6]

© UCLES 2010 0625/61/O/N/10


11

BLANK PAGE

© UCLES 2010 0625/61/O/N/10


12

BLANK PAGE

Permission to reproduce items where third-party owned material protected by copyright is included has been sought and cleared where possible. Every
reasonable effort has been made by the publisher (UCLES) to trace copyright holders, but if any items requiring clearance have unwittingly been included, the
publisher will be pleased to make amends at the earliest possible opportunity.

University of Cambridge International Examinations is part of the Cambridge Assessment Group. Cambridge Assessment is the brand name of University of
Cambridge Local Examinations Syndicate (UCLES), which is itself a department of the University of Cambridge.

© UCLES 2010 0625/61/O/N/10


w
w
w
.X
tr
me
eP
ap
UNIVERSITY OF CAMBRIDGE INTERNATIONAL EXAMINATIONS

er
s
International General Certificate of Secondary Education

.c
om
* 9 0 9 6 7 1 7 3 6 0 *

PHYSICS 0625/61
Paper 6 Alternative to Practical May/June 2011
1 hour
Candidates answer on the Question Paper.
No Additional Materials are required.

READ THESE INSTRUCTIONS FIRST

Write your Centre number, candidate number and name on all the work you hand in.
Write in dark blue or black pen.
You may use a pencil for any diagrams, graphs or rough working.
Do not use staples, paper clips, highlighters, glue or correction fluid.
DO NOT WRITE IN ANY BARCODES.

Answer all questions.

At the end of the examination, fasten all your work securely together.
The number of marks is given in brackets [ ] at the end of each question or part question.

For Examiner’s Use

Total

This document consists of 10 printed pages and 2 blank pages.

DC (LEO/SW) 28913/4
© UCLES 2011 [Turn over
2

1 An IGCSE student is determining the position of the centre of mass of a triangular card. For
Examiner’s
The apparatus is shown in Fig. 1.1. Use

pin or nail
cork
A

stand

card

C
plumbline

Fig. 1.1

(a) The student hangs the card on the nail through hole A. He checks that the card is able
to swing freely and then hangs the plumbline from the nail so that it is close to, but not
touching, the card. When the card and plumbline are still, he makes a small mark at the
edge of the card where the plumbline crosses the edge. He removes the card and draws
a line from the mark to hole A.

He repeats the procedure using holes B and C.

Fig.1.2 is a drawing of the card.

© UCLES 2011 0625/61/M/J/11


3

For
Examiner’s
Use
A

C B

Fig.1.2

On Fig.1.2, the position of each of the marks the student makes is shown with a small
cross. On Fig. 1.2, draw in the lines between the positions of the holes A, B and C and
the corresponding crosses on the card. [2]

(b) If the experiment is completely accurate, the centre of mass of the card is at the position
where the three lines meet. On Fig. 1.2, judge the best position for the centre of mass,
marking it with a small cross. Draw a line from this position to the right-angled corner of
the card and measure the distance a between the centre of mass and the right-angled
corner of the card.

a = ........................................[3]

(c) In this experiment, it is important that the card is able to swing freely. For this reason, the
plumbline should not touch the card but be a small distance from it. This could cause an
inaccuracy in marking the card at the correct position. Describe how you would minimise
this possible inaccuracy. You may draw a diagram.

..........................................................................................................................................

......................................................................................................................................[1]

[Total: 6]

© UCLES 2011 0625/61/M/J/11 [Turn over


4

2 The IGCSE class is investigating the rate of cooling of water. For


Examiner’s
The apparatus is shown in Fig. 2.1. Use

thermometer

beaker

water

Fig. 2.1

(a) Record room temperature θR as shown on the thermometer in Fig. 2.2.

°C
–10

100

110
0

10

20

30

40

50

60

70

80

90

Fig. 2.2

θR = ........................................[1]

(b) The beaker contains 200 cm3 of hot water. A student takes temperature readings as the
water cools, as shown in Table 2.1.

Table 2.1

t/ θ/
0 79

30 65

60 58

90 55

120 53

150 52

180 51

© UCLES 2011 0625/61/M/J/11


5

(i) Complete the column headings in Table 2.1. For


Examiner’s
(ii) Calculate the temperature fall T1 during the first 30 s of cooling. Use

T1 = ............................................

(iii) Calculate the temperature fall T2 during the final 30 s of cooling.

T2 = ........................................[3]

(c) Plot the graph of temperature (y-axis) against time (x-axis).

[5]

(d) (i) State how the rate of cooling in the first 30 s differs from that in the final 30 s.

..................................................................................................................................

..................................................................................................................................

(ii) Explain how the graph line shows this difference.

..................................................................................................................................

..............................................................................................................................[2]

[Total: 11]

© UCLES 2011 0625/61/M/J/11 [Turn over


6

3 The IGCSE class is measuring the currents in lamps in different circuits. For
Examiner’s
The first circuit is shown in Fig. 3.1. Use

power
source

lamp 1
A

lamp 2

Fig. 3.1

(a) A student records the potential difference V across the lamps and the current I in
lamp 1. She rearranges the circuit so that the ammeter is connected in series with
lamp 2 and again records the potential difference V across the lamps and the current I
in lamp 2.

The readings are shown in Table 3.1.

Table 3.1

V/ I/ R/

lamp 1 1.9 0.35

lamp 2 1.9 0.32

V
(i) Calculate the resistance R of each lamp, using the equation R = , and enter the
I
results in the table.

(ii) Add together the two values of R to calculate RS, the sum of the resistances of the
two lamps.

RS = ............................................

(iii) Complete the column headings in the table.

[3]

© UCLES 2011 0625/61/M/J/11


7

(b) The student rearranges the circuit so that the lamps and the ammeter are in series. She For
does not change the position of the voltmeter. Examiner’s
Use

She records the readings on the voltmeter and the ammeter.

voltmeter reading.......1.9 V

ammeter reading.......0.23 A

(i) Draw a circuit diagram of the rearranged circuit using conventional symbols.

(ii) Use the voltmeter and ammeter readings to calculate RT , the combined resistance
of the two lamps in series.

RT = ........................................[3]

(c) A student suggests that the values of RS and RT should be equal. State whether the
results support this suggestion and justify your statement by reference to the calculated
values.

statement .........................................................................................................................

justification .......................................................................................................................

.....................................................................................................................................[2]

(d) State, without reference to the values of resistance that you have calculated, one piece
of evidence that the student can observe during the experiment that shows that the
temperature of the lamp filaments changes.

..........................................................................................................................................

......................................................................................................................................[1]

[Total: 9]

© UCLES 2011 0625/61/M/J/11 [Turn over


8

4 The IGCSE class is investigating reflection of light using a plane mirror. For
Examiner’s
A student has set up a ray trace sheet and this is shown in Fig. 4.1. The line MR shows the Use

position of a plane mirror.

mirror
N
M R

P3

P4

eye

Fig. 4.1

(a) (i) Draw a normal to line MR at N.

(ii) Draw a line 10 cm long that is parallel to line MR and 12 cm below it. The ends of
this line must be at the same distance from the edges of the page as the ends of
line MR. Label this line CD with C directly below M. [3]

(b) The student places a pin P1 so that it stands vertically at C. He places another pin P2 as
close as possible to the point N.

(i) Draw a line from C to N.

(ii) Measure and record the angle of incidence i between the line CN and the normal.

i = ........................................[2]

© UCLES 2011 0625/61/M/J/11


9

(c) The student views the image in the mirror of the pin P1 from the direction indicated by For
the eye in Fig. 4.1. He places two pins P3 and P4 some distance apart so that pins P4, Examiner’s
P3, P2 and the image of P1 all appear exactly one behind the other. The positions of P3 Use

and P4 are shown on Fig. 4.1.

(i) Draw in the line joining the positions of P3 and P4. Continue the line until it meets
the normal.

(ii) Measure and record the angle of reflection r between the normal and line P3P4.

r = ........................................[2]

(d) Several students found that, in spite of carrying out this experiment with reasonable
care, the measured value of the angle of reflection r was not exactly the same as the
value obtained from theory.

Suggest two possible causes of this inaccuracy.

1. ......................................................................................................................................

..........................................................................................................................................

2. ......................................................................................................................................

......................................................................................................................................[2]

[Total: 9]

© UCLES 2011 0625/61/M/J/11 [Turn over


10

5 Table 5.1 shows some measurements taken by three IGCSE students. The second column For
shows the values recorded by the three students. For each quantity, underline the value most Examiner’s
likely to be correct. Use

The first one is done for you.

Table 5.1

quantity measured recorded values


the mass of a wooden metre rule 0.112 kg
1.12 kg
11.2 kg
the diameter of a test tube 0.15 cm
1.5 cm
15 cm
the volume of a coffee cup 10 cm3
100 cm3
1000 cm3
the area of a computer keyboard 0.07 m2
0.70 m2
7.0 m2
the current in a 1.5 V torch lamp at normal 0.12 A
brightness 12 A
120 A
the circumference of a 250 cm3 beaker 2.3 cm
23 cm
230 cm
[5]

[Total: 5]

© UCLES 2011 0625/61/M/J/11


11

BLANK PAGE

© UCLES 2011 0625/61/M/J/11


12

BLANK PAGE

Permission to reproduce items where third-party owned material protected by copyright is included has been sought and cleared where possible. Every
reasonable effort has been made by the publisher (UCLES) to trace copyright holders, but if any items requiring clearance have unwittingly been included, the
publisher will be pleased to make amends at the earliest possible opportunity.

University of Cambridge International Examinations is part of the Cambridge Assessment Group. Cambridge Assessment is the brand name of University of
Cambridge Local Examinations Syndicate (UCLES), which is itself a department of the University of Cambridge.

© UCLES 2011 0625/61/M/J/11


w
w
w
.X
tr
me
eP
ap
UNIVERSITY OF CAMBRIDGE INTERNATIONAL EXAMINATIONS

er
s
International General Certificate of Secondary Education

.c
om
* 1 3 1 3 1 3 5 5 9 2 *

PHYSICS 0625/61
Paper 6 Alternative to Practical October/November 2011
1 hour
Candidates answer on the Question Paper
No Additional Materials are required.

READ THESE INSTRUCTIONS FIRST

Write your Centre number, candidate number and name in the spaces at the top of the page.
Write in dark blue or black pen.
You may use a pencil for any diagrams, graphs or rough working.
Do not use staples, paper clips, highlighters, glue or correction fluid.
DO NOT WRITE IN ANY BARCODES.

Answer all questions.

At the end of the examination, fasten all your work securely together.
The number of marks is given in brackets [ ] at the end of each question or part question.

This document consists of 12 printed pages.

DC (SJF/CGW) 34182/5
© UCLES 2011 [Turn over
2

1 An IGCSE student is determining the weight of a metre rule.

Fig. 1.1 shows the apparatus.

d x y
50.0 cm mark
metre rule

X
pivot

Fig. 1.1

X is a 1.0 N load.

The student places the load X on the rule so that its centre is at d = 5.0 cm from the zero end of
the rule, as shown in Fig.1.1. He adjusts the position of the rule so that it is as near as possible to
being balanced, with the 50.0 cm mark to the right of the pivot.

He measures and records the distance x from the centre of the load X to the pivot, and the
distance y from the pivot to the 50.0 cm mark on the rule. He repeats the procedure using d values
of 10.0 cm, 15.0 cm, 20.0 cm and 25.0 cm. The readings of d, x and y are shown in Table 1.1.

Table 1.1

d / cm x / cm y / cm
5.0 23.7 21.3
10.0 21.0 19.1
15.0 18.5 16.3
20.0 16.0 14.1
25.0 13.9 12.0

© UCLES 2011 0625/61/O/N/11


3

(a) Plot the graph of y / cm (y-axis) against x / cm (x-axis). You do not need to include the origin
(0,0) on your graph.

[4]

(b) Determine the gradient G of the graph. Show clearly on the graph how you obtained the
necessary information.

G = .......................................................... [2]

(c) Calculate the weight W of the metre rule using the equation W = L , where L = 1.0 N.
G

W = .......................................................... [1]

© UCLES 2011 0625/61/O/N/11 [Turn over


4

(d) The calculation of W is based on the assumption that the centre of mass of the rule is at the
50.0 cm mark.

(i) Describe briefly how you would determine the position of the centre of mass of the rule.

...........................................................................................................................................

...........................................................................................................................................

(ii) Describe how you would modify the experiment if the centre of mass was at the 49.7 cm
mark.

...........................................................................................................................................

.......................................................................................................................................[2]

[Total: 9]

© UCLES 2011 0625/61/O/N/11


5

2 The IGCSE class is investigating temperature changes when cold water and hot water are mixed.

(a) A student records the temperature θc of 100 cm3 of cold water and the temperature θh of
100 cm3 of hot water.

–10 0 10 20 30 40 50 60 70 80 90 100 110 °C

Fig. 2.1

Write down the temperature θc shown on the thermometer in Fig. 2.1.

θc = .......................................................... [2]

(b) The hot water is at a temperature θh = 86 °C.

Calculate θav , the average of θc and θh.

average θav = .......................................................... [1]

(c) The student adds 100 cm3 of the hot water to the cold water. She records the temperature θm
of the mixture of hot and cold water, θm = 48 °C.

State two precautions (other than repeating the experiment) that the student could take to
ensure the reliability of her value of the temperature θm.

1. ...............................................................................................................................................

2. ...............................................................................................................................................
[2]

(d) Suggest a practical reason in this experiment for the temperature of the mixture θm being
different from the average value θav , even when the student has taken the precautions you
suggested in (c).

...................................................................................................................................................

...............................................................................................................................................[1]

(e) Suggest a modification to the experiment which should reduce the difference between θm
and θav.

...................................................................................................................................................

...............................................................................................................................................[1]

(f) The student decides to repeat the experiment to check the readings. Suggest one possible
variable that she should keep constant.

...............................................................................................................................................[1]

[Total: 8]

© UCLES 2011 0625/61/O/N/11 [Turn over


6

3 The IGCSE class is investigating the current in resistors in a circuit.

The circuit is shown in Fig. 3.1.

power
source

A A D
B

Fig. 3.1

(a) A student measures the current IA at the position A shown by the ammeter, and then at
positions B (IB), C (IC) and D (ID).

The readings are:

IA= 0.28 A IB = 0.13 A IC = 0.14 A ID = 0.27 A

Theory suggests that IA = IB + IC and ID = IB + IC.

(i) Calculate IB + IC.

IB + IC = ...............................................................

(ii) State whether the experimental results support the theory. Justify your statement by
reference to the readings.

statement ..........................................................................................................................

justification ........................................................................................................................

...........................................................................................................................................

...........................................................................................................................................
[3]

© UCLES 2011 0625/61/O/N/11


7

(b) The student suggests repeating the experiment to confirm her conclusion. She connects a
variable resistor (rheostat) in series with the switch. State the purpose of the variable resistor.

...................................................................................................................................................

...............................................................................................................................................[1]

(c) The student connects a voltmeter and records the potential difference V across the
combination of the three resistors.

(i) On Fig. 3.1, draw in the voltmeter connected as described, using the standard symbol for
a voltmeter. [1]

(ii) Write down the voltmeter reading shown on Fig. 3.2.

4 5 6
3 7
2 8
1 9
0 V 10

Fig. 3.2

V = ......................................................... [1]

(iii) Calculate the resistance R of the combination of the three resistors using the equation
V
R= .
I

R = .......................................................... [2]

[Total: 8]

© UCLES 2011 0625/61/O/N/11 [Turn over


8

4 An IGCSE student is investigating reflection of light in a plane mirror.

Fig. 4.1 shows the student’s ray trace sheet.

ray trace
sheet

M R

P2

P3

eye

Fig. 4.1

(a) The line MR shows the position of a mirror.

(i) Draw a normal to this line that passes through its centre. Label the normal NL. Label the
point at which NL crosses MR with the letter B.
[1]

© UCLES 2011 0625/61/O/N/11


9

(ii) Draw a line 8 cm long from B at an angle of incidence i = 40 ° to the normal below MR
and to the left of the normal. Label the end of this line A. Record the angle of incidence i
in the first row of Table 4.1.

Table 4.1

i /° r /°

34 33
[2]

(b) Fig. 4.2 shows the mirror which is made of polished metal and has a vertical line drawn on it.

mirror

line

Fig. 4.2

The student places the mirror, with its reflecting face vertical, on MR. The lower end of the
line on the mirror is at point B. He places a pin P1 at A. He views the line on the mirror and
the image of pin P1 from the direction indicated by the eye in Fig. 4.1. He places two pins P2
and P3 some distance apart so that pins P3, P2, the image of P1, and the line on the mirror all
appear exactly one behind the other. The positions of P2 and P3 are shown.

(i) Draw the line joining the positions of P2 and P3. Continue the line until it meets the
normal.
(ii) Measure, and record in the first row of Table 4.1, the angle of reflection r between the
normal and the line passing through P2 and P3.
[2]

(c) The student draws a line parallel to MR and 2 cm above it. He places the mirror on this line
and repeats the procedure without changing the position of pin P1. His readings for i and r are
shown in the table.

In spite of carrying out this experiment with reasonable care, it is possible that the values
of the angle of reflection r will not be exactly the same as the values obtained from theory.
Suggest two possible causes of this inaccuracy.

1. ...............................................................................................................................................

...................................................................................................................................................

2. ...............................................................................................................................................

...............................................................................................................................................[2]

© UCLES 2011 0625/61/O/N/11 [Turn over


10

(d) The student was asked to list precautions that should be taken with this experiment in order
to obtain readings that are as accurate as possible. Table 4.2 shows the suggestions.

Place a tick (✓) in the second column of the table next to each correctly suggested
precaution.

Table 4.2

suggested precaution

avoid parallax (line of sight) errors when taking readings with the protractor

carry out the experiment in a darkened room

draw the lines so that they are as thin as possible

keep room temperature constant

place pins P2 and P3 as far apart as possible

use only two or three significant figures for the final answers
[3]

[Total: 10]

© UCLES 2011 0625/61/O/N/11


11

5 The IGCSE class is carrying out an experiment to determine the speed of sound in air.

Fig. 5.1 indicates the method used. The experiment is conducted outside the school building.

student A student B
drum stopwatch

Fig. 5.1 (not to scale)

Student A strikes a drum once as loudly as possible. Student B stands some distance away from
student A and starts a stopwatch when she sees the drum being hit. She stops the stopwatch
when she hears the sound. She records the time interval t in Table 5.1. The experiment is repeated
several times. She calculates the speed of sound v and enters the values in the table.

Table 5.1

t/s v / (m / s)
0.87 344.83
0.92 326.09
0.84 357.14
0.83 361.45
0.86 338.84

(a) Suggest a suitable distance d for students to use when carrying out this experiment.

d = .......................................................... [1]

(b) Suggest a suitable instrument for measuring the distance d.

...............................................................................................................................................[1]

(c) Calculate the average value vav for the speed of sound from the results in the table. Show
your working.

vav = .......................................................... [2]

© UCLES 2011 0625/61/O/N/11 [Turn over


12

(d) The student has recorded the values for the speed of sound v to five significant figures.
State whether this is a suitable number of significant figures for the speed of sound in air in
this experiment. Give a reason for your answer.

statement ..................................................................................................................................

reason .......................................................................................................................................

...............................................................................................................................................[1]

[Total: 5]

Permission to reproduce items where third-party owned material protected by copyright is included has been sought and cleared where possible. Every
reasonable effort has been made by the publisher (UCLES) to trace copyright holders, but if any items requiring clearance have unwittingly been included, the
publisher will be pleased to make amends at the earliest possible opportunity.

University of Cambridge International Examinations is part of the Cambridge Assessment Group. Cambridge Assessment is the brand name of University of
Cambridge Local Examinations Syndicate (UCLES), which is itself a department of the University of Cambridge.

© UCLES 2011 0625/61/O/N/11


w
w
w
.X
tr
me
eP
ap
UNIVERSITY OF CAMBRIDGE INTERNATIONAL EXAMINATIONS

er
s
International General Certificate of Secondary Education

.c
om
* 4 4 6 8 1 0 1 8 2 6 *

PHYSICS 0625/61
Paper 6 Alternative to Practical May/June 2012
1 hour
Candidates answer on the Question Paper.
No Additional Materials are required.

READ THESE INSTRUCTIONS FIRST

Write your Centre number, candidate number and name on all the work you hand in.
Write in dark blue or black pen.
You may use a pencil for any diagrams or graphs.
Do not use staples, paper clips, highlighters, glue or correction fluid.
DO NOT WRITE IN ANY BARCODES.

Answer all questions.

At the end of the examination, fasten all your work securely together.
The number of marks is given in brackets [ ] at the end of each question or part question.

For Examiner’s Use

Total

This document consists of 12 printed pages.

DC (NF/CGW) 42235/7
© UCLES 2012 [Turn over
2

1 An IGCSE student is determining the mass of a metre rule using a balancing method. For
Examiner’s
Fig. 1.1 shows the apparatus. Use

d x y 50.0 cm mark
metre rule

M
pivot

Fig. 1.1

Mass M is placed on the rule. The position of the pivot is adjusted until the rule balances.

(a) The student chooses a mass M which is similar to the mass of the metre rule. Suggest a
suitable value for the mass.

suitable mass = ................................................. [1]

(b) The mass is cylindrical and has a diameter slightly larger than the width of the metre rule.

Describe briefly how you would place the mass so that its centre of mass is exactly over
the 90.0 cm mark on the metre rule. You should draw a diagram and mark the position of
the centre of mass on the cylinder.

..........................................................................................................................................

..........................................................................................................................................

..................................................................................................................................... [2]

(c) From your experience of carrying out balancing experiments of this type, suggest one
difficulty that you are likely to come across that could make the final result inaccurate.

..........................................................................................................................................

..........................................................................................................................................

..................................................................................................................................... [1]

© UCLES 2012 0625/61/M/J/12


3

(d) The student takes a reading of x and the corresponding reading of y. He then calculates For
the mass of the metre rule. Examiner’s
Use

Suggest how you would improve the reliability of the value of the mass of the metre rule,
using this method.

..........................................................................................................................................

..................................................................................................................................... [1]

(e) Another student carries out a similar experiment to determine the mass of a 50 cm metal
strip. She calculates the mass and writes down “mass = 234.872 g”.

She checks the mass on an accurate balance. The value is 235 g. She thinks she must
have made a mistake in her experiment.

Write a brief comment on the accuracy of her experimental result.

..........................................................................................................................................

..........................................................................................................................................

..................................................................................................................................... [1]

[Total: 6]

© UCLES 2012 0625/61/M/J/12 [Turn over


4

2 The IGCSE class is investigating the heating of a thermometer bulb. For


Examiner’s
The apparatus is shown in Figs. 2.1, 2.2 and 2.3. Use

thermometer

card
metre rule
lamp

Fig. 2.1

(a) Record the value of room temperature θ R shown on the thermometer.

-10 0 10 20 30 40 50 60 70 80 90 100 110


°C

Fig. 2.2

θ R = ................................................. [1]

(b) A student switches on the lamp and places the thermometer so that its bulb is a horizontal
distance d = 100 mm from the surface of the lamp, as shown in Fig. 2.1. She records the
distance d between the thermometer bulb and the surface of the lamp. She also records
the temperature θ shown on the thermometer. She repeats the procedure using values
of d of 80 mm, 60 mm, 40 mm, 20 mm and 10 mm. The temperature readings are shown
in Table 2.1.

(i) Record the d values in the table.


(ii) Complete the column headings in the table.

Table 2.1

d / 00 θ / 00

52

56

61

67

75

86

[2]

© UCLES 2012 0625/61/M/J/12


5

(c) The student moves the thermometer away from the lamp and waits for about a minute For
for the thermometer to cool. She places the thermometer so that its bulb is a vertical Examiner’s
distance d V = 100 mm from the top surface of the lamp, as shown in Fig. 2.3. Use

thermometer

dv

lamp

Fig. 2.3

She records the temperature θ V shown on the thermometer: θ V = 55 °C.

Calculate the difference between θ V and the thermometer reading θH at a horizontal


distance of 100 mm from the lamp. State whether θ V is higher, lower or the same as θ H.

temperature difference = ......................................................

θ V is ................................................. [1]

(d) A student suggests that θ V will be higher than the thermometer reading θH because
thermal energy will travel by infra-red radiation and convection to the thermometer bulb
above the lamp but by infra-red radiation only when the bulb is to one side of the lamp.

If the experiment were to be repeated in order to investigate this suggestion it would


be important to control the conditions. Suggest two such conditions, relevant to this
investigation, that should be controlled.

1. ......................................................................................................................................

2. ......................................................................................................................................
[2]

(e) Briefly describe a precaution that you would take in this experiment in order to obtain a
reliable result.

..........................................................................................................................................

..........................................................................................................................................

..................................................................................................................................... [1]

[Total: 7]

© UCLES 2012 0625/61/M/J/12 [Turn over


6

3 The IGCSE class is determining the resistance of a fixed resistor in a circuit. For
Examiner’s
The circuit is shown in Fig. 3.1. Use

power
source

A
resistance
R d wire

Fig. 3.1

(a) A student places the sliding contact on the resistance wire at a distance d = 10.0 cm
from point A. He measures the current I in the circuit and the p.d. V across the resistor
R. He repeats the procedure using d values of 30.0 cm, 50.0 cm, 70.0 cm and 90.0 cm.

The readings are shown in Table 3.1.

Table 3.1

V/V I/A

10.0 1.7 1.13

30.0 1.3 0.87

50.0 1.0 0.67

70.0 0.8 0.53

90.0 0.7 0.47

(i) Complete the column headings in the table.

© UCLES 2012 0625/61/M/J/12


7

(ii) Plot a graph of V / V (y-axis) against I / A (x-axis). You do not need to include the For
origin (0, 0) on your graph. Examiner’s
Use

[5]

(iii) Determine the gradient G of the graph. Show clearly on the graph how you obtained
the necessary information.

G = ................................................. [3]

(b) The gradient G of the graph is numerically equal to the resistance R of the resistor R.

Write a value for the resistance R to a suitable number of significant figures for this
experiment.

R = ................................................. [2]

[Total: 10]

© UCLES 2012 0625/61/M/J/12 [Turn over


8

4 The IGCSE class is determining the focal length of a lens. For


Examiner’s
The apparatus is shown in Fig. 4.1. Use

illuminated
object screen
lens

x
d

Fig. 4.1

(a) A student places the lens between the object and the screen and close to the object.
She moves the lens towards the screen until a clearly focused, enlarged image is
formed on the screen.

(i) On Fig. 4.1, measure and record the distance d between the object and the screen.

d = ......................................................

(ii) On Fig. 4.1, measure and record the distance x between the centre of the lens and
the screen.

x = ......................................................
[2]

(iii) Fig. 4.1 is drawn one tenth actual size.

1. Calculate the actual distance D between the object and the screen.

D = ......................................................

2. Calculate the actual distance X between the centre of the lens and the screen.

X = ......................................................
[1]

(b) Without moving the illuminated object or the screen, the student moves the lens towards
the screen until a clearly focused, diminished image is formed on the screen. She
measures the distance Y between the centre of the lens and the screen: Y = 19.0 cm.

XY
Calculate the focal length f of the lens using the equation f = .
D

f = ................................................. [2]
© UCLES 2012 0625/61/M/J/12
9

(c) The student turns the lens through an angle of 180° and repeats the procedure obtaining For
a value for the focal length f = 14.7 cm. Examiner’s
Use

Theory suggests that the two values of the focal length f should be the same. State
whether the results support this theory and justify your answer by reference to the
results.

statement .........................................................................................................................

justification .......................................................................................................................

..........................................................................................................................................
[2]

(d) Briefly describe a precaution that you would take in this experiment in order to obtain a
reliable result.

..........................................................................................................................................

..........................................................................................................................................

..................................................................................................................................... [1]

[Total: 8]

© UCLES 2012 0625/61/M/J/12 [Turn over


10

5 The IGCSE class is determining the internal volume of a test-tube using two displacement For
methods. Examiner’s
Use

The apparatus used is shown in Figs. 5.1, 5.2 and 5.3.

cm3 cm3
100 100
measuring
90 cylinder 90

80 80

70 70

60 60

50 test-tube 50

40 40

water 30 30

20 20

10 10

Fig. 5.1 Fig. 5.2

(a) (i) Fig. 5.1 shows water in a measuring cylinder. Record the volume V1 of the water.

V1 = ................................................. [1]

(ii) On Fig. 5.1, show clearly the line of sight that you would use to obtain an accurate
volume reading. [2]

(b) (i) A student lowers a test-tube, closed end first, into the water in the measuring
cylinder and pushes the tube down until it is filled with water. From Fig. 5.2, record
the new water level V2.

V2 = ......................................................

(ii) Calculate the volume VG of the glass of the test-tube using the equation VG = (V2 – V1).

VG = ......................................................
[2]

© UCLES 2012 0625/61/M/J/12


11

(c) The student removes the test-tube from the measuring cylinder and empties the water For
back into the measuring cylinder. He then puts the test-tube, open end first, into the Examiner’s
water in the measuring cylinder and carefully pushes it down with his finger until it is Use

covered with water as shown in Fig. 5.3.

finger

cm3
100

90

80

70

60

50 test-tube

40

30

20

10

Fig. 5.3

(i) Record the new water level V3.

V3 = ......................................................

(ii) Calculate the increase in water level (V3 – V1).

(V3 – V1) = ......................................................

(iii) Calculate the volume VA of air in the test-tube using the equation VA = (V3 – V1) – VG.

VA = ......................................................
[1]

© UCLES 2012 0625/61/M/J/12 [Turn over


12

(d) The student removes the test-tube from the measuring cylinder and fills the test-tube For
with water from a beaker. He pours the water from the test-tube into an empty measuring Examiner’s
cylinder and records the volume VW of water: Use

18 cm3
VW = ......................................................

The student has attempted to determine the internal volume of the test-tube by two
methods. His two values for the internal volume are VA and VW.

Assuming that the experiments have been carried out correctly and carefully and that
the measuring cylinder scale is accurate, suggest two reasons why the value VA may be
inaccurate and two reasons why the value VW may be inaccurate.

VA:

reason 1 ...........................................................................................................................

..........................................................................................................................................

reason 2 ...........................................................................................................................

..........................................................................................................................................

VW:

reason 1 ...........................................................................................................................

..........................................................................................................................................

reason 2 ...........................................................................................................................

..........................................................................................................................................
[3]

[Total: 9]

Permission to reproduce items where third-party owned material protected by copyright is included has been sought and cleared where possible. Every
reasonable effort has been made by the publisher (UCLES) to trace copyright holders, but if any items requiring clearance have unwittingly been included, the
publisher will be pleased to make amends at the earliest possible opportunity.

University of Cambridge International Examinations is part of the Cambridge Assessment Group. Cambridge Assessment is the brand name of University of
Cambridge Local Examinations Syndicate (UCLES), which is itself a department of the University of Cambridge.

© UCLES 2012 0625/61/M/J/12


w
w
w
.X
tr
me
eP
ap
UNIVERSITY OF CAMBRIDGE INTERNATIONAL EXAMINATIONS

er
s
International General Certificate of Secondary Education

.c
om
* 1 1 7 9 4 6 4 0 8 3 *

PHYSICS 0625/61
Paper 6 Alternative to Practical October/November 2012
1 hour
Candidates answer on the Question Paper
No Additional Materials are required.

READ THESE INSTRUCTIONS FIRST

Write your Centre number, candidate number and name in the spaces at the top of the page.
Write in dark blue or black pen.
You may use a pencil for any diagrams or graphs.
Do not use staples, paper clips, highlighters, glue or correction fluid.
DO NOT WRITE IN ANY BARCODES.

Answer all questions.

At the end of the examination, fasten all your work securely together.
The number of marks is given in brackets [ ] at the end of each question or part question.

For Examiner’s Use

Total

This document consists of 11 printed pages and 1 blank page.

DC (CW/SW) 50194/4
© UCLES 2012 [Turn over
2

1 The IGCSE class is investigating the stretching of a spring. For


Examiner’s
Fig. 1.1 shows the experimental set up. Use

clamp

spring

d0

bench

Fig. 1.1

(a) On Fig. 1.1, measure the vertical distance d0, in mm, between the bottom of the spring
and the surface of the bench.

d0 = .......................................... mm [1]

(b) The diagram is drawn 1/10th actual size. Calculate the actual distance D0, in mm,
between the bottom of the spring and the surface of the bench.

D0 = .......................................... mm [1]

(c) A student hangs a 1.0 N load on the spring. He measures and records the distance D
between the bottom of the spring and the surface of the bench, and the value of the
load L.

He repeats the procedure using loads of 2.0 N, 3.0 N, 4.0 N and 5.0 N. The distance
readings are shown in Table 1.1.

Calculate the extension e of the spring, for each set of readings, using the equation
e = (D0 – D). Record the values of L and e in Table 1.1.

Table 1.1

L/N D / mm e / mm

199

191

179

171

160
[2]

© UCLES 2012 0625/61/O/N/12


3

(d) Plot a graph of e / mm (y-axis) against L / N (x-axis). For


Examiner’s
Use

[4]

(e) Determine the gradient G of the graph. Show clearly on the graph how you obtained the
necessary information.

G = .................................................. [2]

(f) When making measurements, the student is careful to avoid a line-of-sight error.

Suggest one other precaution that the student should take when measuring the distance
D between the bottom of the spring and the surface of the bench.

..........................................................................................................................................

...................................................................................................................................... [1]

[Total: 11]

© UCLES 2012 0625/61/O/N/12 [Turn over


4

2 The IGCSE class is investigating the rate of cooling of water under different conditions. For
Examiner’s
The apparatus is shown in Fig. 2.1. Use

110 °C
100
90
80
70
60
50
thermometer

water 40
30
20
10
0
–10

Fig. 2.1 Fig. 2.2

(a) Record the value of room temperature θR shown on the thermometer in Fig. 2.2.

θR = .................................................. [1]

© UCLES 2012 0625/61/O/N/12


5

(b) A student pours 150 cm3 of hot water into a beaker. She measures the temperature θ of For
the water at time t = 0 and records it in a table. Examiner’s
Use

She starts a stopclock and records the temperature of the water at 30 s intervals until
she has a total of six values up to time t = 150 s. The readings are shown in Table 2.1.

She repeats the procedure, using 250 cm3 of hot water.

Table 2.1

volume of water

150 cm3 250 cm3

t/ θ/ θ/

0 84 85

30 79 79

60 74 75

90 70 72

120 68 70

150 66 68

(i) Complete the column headings in the table. [1]

(ii) State whether the rate of cooling is significantly faster, slower, or about the same
when using the larger volume of hot water. Justify your answer by reference to the
readings.

statement .................................................................................................................

justification ...............................................................................................................

..................................................................................................................................

..................................................................................................................................
[2]

(c) If this experiment were to be repeated in order to check the results, it would be important
to control the conditions. Suggest two such conditions that should be controlled.

1. ......................................................................................................................................

2. ......................................................................................................................................
[2]

[Total: 6]

© UCLES 2012 0625/61/O/N/12 [Turn over


6

3 The IGCSE class is investigating the potential differences across circuit components. For
Examiner’s
Fig. 3.1 shows the apparatus used. Use

power
pack

ammeter

lamp N

lamp L lamp M

voltmeter

Fig. 3.1

(a) Draw a circuit diagram of the circuit shown in Fig. 3.1, using standard symbols.

[3]

(b) A student records the current IA, the potential difference VL across lamp L and the
potential difference VM across lamp M.
0.65 A
IA = ......................................................
0.9 V
VL = ......................................................
1.0 V
VM = ......................................................

(i) Calculate the potential difference VA across lamps L and M using the equation
VA = VL + VM.

VA = ......................................................

© UCLES 2012 0625/61/O/N/12


7

(ii) Calculate RA, the combined resistance of lamps L, M and N, using the equation For
V Examiner’s
RA = A .
IA Use

RA = ......................................................
[2]

(iii) On Fig. 3.2, draw a pointer showing the current IA = 0.65 A.

0.4 0.6
0.2 0.8

0 1.0
A

Fig. 3.2 [1]

(c) The student rearranges the circuit so that the three lamps are in series with each other.
He records the potential difference across each lamp in turn.

0.6 V
VL = ......................................................
0.7 V
VM = ......................................................
0.7 V
VN = ......................................................

Calculate the potential difference VB across the three lamps using the equation
VB = VL + VM + VN.

VB = ......................................................

(d) A student suggests that VA should be equal to VB.

State whether the results support this suggestion and justify your answer with reference
to the results.

statement .........................................................................................................................

justification .......................................................................................................................

..........................................................................................................................................
[2]

[Total: 8]

© UCLES 2012 0625/61/O/N/12 [Turn over


8

4 The IGCSE class is investigating the refraction of light passing through a transparent block. For
Examiner’s
The apparatus and ray-trace sheet are shown in Fig. 4.1. Use

A B

D C
P3

P4

ray-trace sheet eye

Fig. 4.1

© UCLES 2012 0625/61/O/N/12


9

(a) A student places the transparent block, largest face down, on the ray-trace sheet. She For
draws the outline of the block ABCD. Examiner’s
Use

(i) On Fig. 4.1, draw a normal at the centre of side AB. Label the point E where the
normal crosses AB.

(ii) Draw a line FE to the left of the normal and at an angle of incidence i = 30° to the
normal. [2]

(b) The student places two pins P1 and P2 on the line FE, placing one pin close to E. She
observes the images of P1 and P2 through side CD of the block so that the images of P1
and P2 appear one behind the other. She places two pins P3 and P4 between her eye
and the block so that P3 and P4, and the images of P1 and P2 seen through the block,
appear one behind the other.

(i) On Fig. 4.1, mark suitable positions for the pins P1 and P2. [1]

(ii) Draw a line joining the positions of P3 and P4. Continue the line until it meets CD
and label this point G.

(iii) Draw the line GE.


[1]

(c) (i) Measure and record the angle of refraction r between the line GE and the normal.

r = .................................................. [1]
i
(ii) Calculate the ratio r .

i
r = .................................................. [1]
(d) The student repeats the procedure but with the angle of incidence i = 40°. The angle of
refraction r = 26°.
i
(i) Calculate the ratio r .

i
r = .................................................. [1]
i
(ii) A student suggests that the ratio r should be a constant.

State and explain briefly whether your results support this suggestion.

..................................................................................................................................

..................................................................................................................................

.............................................................................................................................. [1]

[Total: 8]

© UCLES 2012 0625/61/O/N/12 [Turn over


10

5 (a) The IGCSE class has a range of apparatus available. Here is a list of some of the For
apparatus. Examiner’s
Use

ammeter

barometer

beaker

electronic balance

manometer

measuring cylinder

metre rule

newtonmeter (spring balance)

stopwatch

tape measure

thermometer

voltmeter

Complete Table 5.1 by inserting the name of one piece of apparatus from the list that is
the most suitable for measuring each quantity described.

Table 5.1

quantity to be measured most suitable apparatus

volume of water

a distance of about 50 m

the force required to lift a laboratory stool

the mass of a coin

the pressure of the laboratory gas supply

[5]

© UCLES 2012 0625/61/O/N/12


11

(b) The IGCSE class is carrying out a lens experiment. This involves using an illuminated For
object, a screen and a lens. Examiner’s
Use

Firstly, the distance between the illuminated object and the lens is measured with a
metre rule. Next, a clearly focused image is obtained on the screen.

(i) Explain briefly how you would avoid a parallax (line-of-sight) error when using the
metre rule.

..................................................................................................................................

..................................................................................................................................

.............................................................................................................................. [1]

(ii) State a precaution that you would take to ensure that the image is well focused.

..................................................................................................................................

..................................................................................................................................

.............................................................................................................................. [1]

[Total: 7]

© UCLES 2012 0625/61/O/N/12


12

BLANK PAGE

Permission to reproduce items where third-party owned material protected by copyright is included has been sought and cleared where possible. Every
reasonable effort has been made by the publisher (UCLES) to trace copyright holders, but if any items requiring clearance have unwittingly been included, the
publisher will be pleased to make amends at the earliest possible opportunity.

University of Cambridge International Examinations is part of the Cambridge Assessment Group. Cambridge Assessment is the brand name of University of
Cambridge Local Examinations Syndicate (UCLES), which is itself a department of the University of Cambridge.

© UCLES 2012 0625/61/O/N/12


Answer key for IGCSE Physics (0625/1) – Multiple Choice May 1999

Item Correct Answer Item Correct Answer


1 D 21 B
2 C 22 A
3 B 23 D
4 D 24 A
5 D 25 C
6 C 26 A
7 B 27 C
8 B 28 B
9 C 29 C
10 D 30 A
11 C 31 B
12 C 32 D
13 C 33 A
14 A 34 A
15 B 35 A
16 B 36 B
17 C 37 A
18 C 38 A
19 C 39 C
20 A 40 D

Back to questions
Answer key for IGCSE Physics (0625/1) – Multiple Choice November 1999

Item Correct Answer Item Correct Answer


1 A 21 D
2 C 22 D
3 C 23 C
4 D 24 D
5 C 25 D
6 B 26 B
7 C 27 C
8 B 28 A
9 C 29 A
10 B 30 B
11 B 31 A
12 C 32 D
13 B 33 C
14 A 34 A
15 A 35 D
16 B 36 D
17 B 37 C
18 B 38 C
19 A 39 B
20 D 40 A

Back to questions
Answer key for IGCSE Physics (0625/1) – Multiple Choice May/June 2000

Item Correct Answer Item Correct Answer


1 B 21 C
2 A 22 B
3 C 23 B
4 B 24 A
5 D 25 C
6 B 26 D
7 C 27 A
8 A 28 B
9 D 29 D
10 B 30 D
11 D 31 A
12 C 32 B
13 D 33 A
14 D 34 A
15 A 35 C
16 B 36 B
17 C 37 D
18 D 38 C
19 A 39 B
20 A 40 A

Back to questions
Answer key for IGCSE Physics (0625/1) – Multiple Choice November 2000

Item Correct Answer Item Correct Answer


1 D 21 D
2 B 22 B
3 D 23 B
4 C 24 B
5 C 25 B
6 B 26 C
7 B 27 D
8 D 28 A
9 A 29 C
10 D 30 B
11 B 31 C
12 B 32 D
13 D 33 A
14 D 34 D
15 C 35 A
16 A 36 D
17 B 37 D
18 D 38 C
19 C 39 A
20 B 40 B

Back to questions
Answer key for IGCSE Physics (0625/1) – Multiple Choice May/June 2001

Item Correct Answer Item Correct Answer


1 B 21 B
2 B 22 A
3 D 23 D
4 B 24 A
5 D 25 A
6 B 26 B
7 B 27 C
8 A 28 C
9 C 29 D
10 C 30 C
11 D 31 B
12 B 32 A
13 A 33 A
14 C 34 C
15 D 35 A
16 D 36 C
17 C 37 B
18 B 38 D
19 A 39 B
20 C 40 C

Back to questions
Answer key for IGCSE Physics (0625/1) – Multiple Choice October/November 2001

Item Correct Answer Item Correct Answer


1 A 21 A
2 A 22 B
3 B 23 A
4 A 24 B
5 B 25 C
6 B 26 D
7 C 27 D
8 D 28 C
9 C 29 B
10 D 30 C
11 D 31 A
12 D 32 C
13 A 33 C
14 D 34 A
15 A 35 A
16 A 36 C
17 B 37 B
18 B 38 B
19 A 39 D
20 C 40 C

Back to questions
Page 1 Mark Scheme Syllabus Paper
IGCSE EXAMINATIONS – JUNE 2003 0625 1

Question Question
Key Key
Number Number
1 A 21 D
2 B 22 D
3 B 23 B
4 C 24 B
5 D 25 B

6 C 26 D
7 A 27 A
8 D 28 A
9 B 29 B
10 B 30 D

11 A 31 C
12 C 32 D
13 B 33 A
14 D 34 A
15 B 35 C

16 A 36 B
17 A 37 D
18 A 38 A
19 B 39 D
20 D 40 B

TOTAL 40

Back to questions

© University of Cambridge Local Examinations Syndicate 2003


Page 1 Mark Scheme Syllabus Paper
IGCSE EXAMINATIONS – NOVEMBER 2003 0625 1

Question Question
Key Key
Number Number
1 D 21 A
2 C 22 D
3 A 23 C
4 C 24 B
5 C 25 A

6 B 26 B
7 C 27 B
8 A 28 B
9 C 29 B
10 D 30 D

11 D 31 C
12 B 32 C
13 D 33 B
14 D 34 B
15 D 35 B

16 A 36 C
17 D 37 A
18 A 38 A
19 B 39 C
20 B 40 A

TOTAL 40

Back to questions

© University of Cambridge Local Examinations Syndicate 2003


Page 1 Mark Scheme Syllabus Paper
PHYSICS - JUNE 2004 0625 1

Question Question
Key Key
Number Number
1 D 21 C
2 A 22 B
3 D 23 D
4 A 24 C
5 D 25 C

6 D 26 C
7 C 27 B
8 A 28 B
9 B 29 A
10 C 30 B

11 A 31 C
12 A 32 A
13 A 33 A
14 C 34 A
15 C 35 D

16 D 36 A
17 D 37 D
18 A 38 D
19 C 39 B
20 D 40 C

TOTAL 40

Back to questions

© University of Cambridge International Examinations 2004


Page 1 Mark Scheme Syllabus Paper
IGCSE – November 2004 0625 1

Question Question
Key Key
Number Number
1 B 21 C
2 B 22 C
3 D 23 B
4 C 24 B
5 D 25 A

6 D 26 D
7 D 27 C
8 C 28 C
9 D 29 D
10 A 30 C

11 D 31 A
12 A 32 C
13 C 33 A
14 B 34 B
15 C 35 C

16 B 36 C
17 C 37 A
18 B 38 D
19 B 39 C
20 A 40 C

Back to questions

© University of Cambridge International Examinations 2005


Page 1 Mark Scheme Syllabus Paper
IGCSE – JUNE 2005 0625 1

Question Question
Key Key
Number Number
1 B 21 A
2 D 22 A
3 C 23 B
4 D 24 B
5 B 25 B

6 B 26 C
7 A 27 A
8 C 28 C
9 C 29 D
10 D 30 C

11 B 31 D
12 B 32 B
13 D 33 B
14 C 34 D
15 D 35 B

16 D 36 A
17 C 37 B
18 A 38 A
19 D 39 C
20 C 40 C

© University of Cambridge International Examinations 2005


Page 1 Mark Scheme Syllabus Paper
IGCSE – NOVEMBER 2005 0625 1

Question Question
Key Key
Number Number

1 B 21 A
2 B 22 B
3 A 23 D
4 D 24 B
5 D 25 C

6 B 26 C
7 B 27 D
8 D 28 C
9 B 29 B
10 D 30 A

11 A 31 A
12 B 32 C
13 A 33 A
14 D 34 A
15 B 35 B

16 B 36 A
17 A 37 A
18 C 38 B
19 C 39 B
20 D 40 A

© University of Cambridge International Examinations 2005


Page 1 Mark Scheme Syllabus Paper
IGCSE – May/June 2006 0625 01

Question Question
Key Key
Number Number
1 A 21 B
2 C 22 C
3 C 23 C
4 B 24 B
5 B 25 B

6 D 26 C
7 A 27 B
8 A 28 B
9 B 29 D
10 A 30 A

11 B 31 C
12 D 32 C
13 B 33 D
14 D 34 C
15 C 35 A

16 A 36 A
17 B 37 C
18 A 38 D
19 D 39 C
20 D 40 D

© University of Cambridge International Examinations 2006


Page 2 Mark Scheme Syllabus Paper
GCE A/AS LEVEL - OCT/NOV 2006 0625 01

Question Question
Key Key
Number Number
1 B 21 D
2 B 22 B
3 D 23 A
4 D 24 C
5 B 25 A

6 C 26 C
7 C 27 A
8 A 28 C
9 B 29 B
10 D 30 A

11 A 31 D
12 B 32 B
13 D 33 A
14 C 34 A
15 D 35 D

16 C 36 B
17 B 37 A
18 C 38 B
19 D 39 D
20 C 40 C

© UCLES 2006
Page 2 Mark Scheme Syllabus Paper
IGCSE – May/June 2007 0625 01

Question Question
Key Key
Number Number
1 A 21 B
2 B 22 B
3 A 23 A
4 D 24 B
5 B 25 D

6 C 26 B
7 B 27 B
8 C 28 C
9 B 29 C
10 B 30 B

11 D 31 A
12 B 32 D
13 D 33 B
14 A 34 C
15 B 35 A

16 D 36 C
17 D 37 D
18 C 38 C
19 A 39 D
20 C 40 A

© UCLES 2007
Page 2 Mark Scheme Syllabus Paper
IGCSE – October/November 2007 0625 01

Question Question
Key Key
Number Number
1 C 21 B
2 A 22 C
3 A 23 C
4 D 24 C
5 B 25 A

6 B 26 C
7 C 27 C
8 C 28 B
9 A 29 C
10 C 30 C

11 C 31 B
12 D 32 B
13 D 33 B
14 D 34 A
15 A 35 B

16 B 36 D
17 B 37 D
18 C 38 C
19 B 39 A
20 D 40 C

© UCLES 2007
Page 2 Mark Scheme Syllabus Paper
IGCSE – May/June 2008 0625 01

Question Question
Key Key
Number Number
1 B 21 D
2 B 22 C
3 C 23 A
4 C 24 C
5 B 25 A

6 C 26 D
7 B 27 A
8 D 28 B
9 C 29 B
10 D 30 A

11 C 31 B
12 C 32 A
13 A 33 B
14 C 34 A
15 B 35 B

16 D 36 C
17 D 37 C
18 A 38 A
19 B 39 B
20 A 40 B

© UCLES 2008
Page 2 Mark Scheme Syllabus Paper
IGCSE – October/November 2008 0625 01

Question Question
Key Key
Number Number
1 C 21 D
2 D 22 C
3 B 23 A
4 B 24 C
5 B 25 C

6 A 26 B
7 A 27 C
8 C 28 B
9 A 29 B
10 A 30 C

11 D 31 C
12 B 32 D
13 C 33 B
14 C 34 B
15 C 35 A

16 B 36 B
17 D 37 C
18 C 38 B
19 B 39 C
20 D 40 D

© UCLES 2008
First variant Mark Scheme

Page 2 Mark Scheme: Teachers’ version Syllabus Paper


IGCSE – May/June 2009 0625 11

Question Question
Key Key
Number Number
1 C 21 D
2 C 22 B
3 B 23 A
4 B 24 C
5 D 25 C

6 A 26 A
7 A 27 C
8 B 28 A
9 C 29 B
10 C 30 C

11 B 31 D
12 A 32 A
13 C 33 B
14 A 34 C
15 D 35 B

16 A 36 B
17 A 37 A
18 C 38 B
19 C 39 C
20 D 40 C

© UCLES 2009
Page 2 Mark Scheme: Teachers’ version Syllabus Paper
IGCSE – October/November 2009 0625 11

Question Question
Key Key
Number Number
1 B 21 B
2 A 22 C
3 C 23 B
4 D 24 D
5 A 25 D

6 B 26 D
7 A 27 C
8 D 28 B
9 D 29 D
10 C 30 A

11 B 31 D
12 D 32 C
13 B 33 A
14 D 34 C
15 C 35 D

16 B 36 A
17 A 37 D
18 B 38 C
19 C 39 B
20 B 40 B

© UCLES 2009
Page 2 Mark Scheme: Teachers’ version Syllabus Paper
IGCSE – May/June 2010 0625 11

Question Question
Key Key
Number Number
1 C 21 A
2 A 22 D
3 B 23 C
4 B 24 D
5 B 25 A

6 A 26 B
7 D 27 D
8 C 28 B
9 B 29 B
10 A 30 D

11 A 31 C
12 A 32 B
13 D 33 C
14 D 34 A
15 C 35 D

16 C 36 A
17 C 37 D
18 A 38 B
19 D 39 B
20 C 40 B

© UCLES 2010
Page 2 Mark Scheme: Teachers’ version Syllabus Paper
IGCSE – October/November 2010 0625 11

Question Question
Key Key
Number Number
1 A 21 C
2 D 22 B
3 D 23 C
4 D 24 D
5 D 25 A

6 B 26 D
7 C 27 C
8 B 28 C
9 C 29 C
10 C 30 A

11 B 31 C
12 D 32 C
13 A 33 A
14 C 34 C
15 A 35 A

16 D 36 A
17 A 37 B
18 B 38 D
19 B 39 C
20 B 40 D

© UCLES 2010
Page 2 Mark Scheme: Teachers’ version Syllabus Paper
IGCSE – May/June 2011 0625 11

Question Question
Key Key
Number Number
1 A 21 D
2 B 22 D
3 A 23 C
4 D 24 C
5 A 25 A

6 A 26 A
7 C 27 B
8 D 28 B
9 B 29 D
10 D 30 D

11 C 31 A
12 D 32 B
13 D 33 C
14 C 34 A
15 C 35 D

16 A 36 B
17 A 37 C
18 B 38 B
19 B 39 C
20 C 40 C

© University of Cambridge International Examinations 2011


Page 2 Mark Scheme: Teachers’ version Syllabus Paper
IGCSE – October/November 2011 0625 11

Question Question
Key Key
Number Number
1 B 21 B
2 D 22 C
3 D 23 D
4 B 24 D
5 C 25 C

6 A 26 C
7 C 27 B
8 B 28 A
9 C 29 B
10 C 30 D

11 D 31 C
12 C 32 B
13 D 33 D
14 C 34 B
15 C 35 C

16 D 36 B
17 A 37 A
18 A 38 B
19 C 39 B
20 A 40 A

© University of Cambridge International Examinations 2011


Page 2 Mark Scheme: Teachers’ version Syllabus Paper
IGCSE – May/June 2012 0625 11

Question Question
Key Key
Number Number
1 A 21 D
2 C 22 A
3 B 23 C
4 D 24 C
5 B 25 C

6 D 26 D
7 A 27 A
8 C 28 D
9 C 29 C
10 B 30 D

11 C 31 C
12 B 32 D
13 A 33 B
14 A 34 C
15 D 35 B

16 A 36 D
17 D 37 C
18 A 38 B
19 D 39 B
20 B 40 B

© University of Cambridge International Examinations 2012


Page 2 Mark Scheme Syllabus Paper
IGCSE – October/November 2012 0625 11

Question Question
Key Key
Number Number
1 C 21 C
2 A 22 A
3 B 23 D
4 A 24 B
5 C 25 C

6 C 26 A
7 A 27 B
8 D 28 A
9 D 29 B
10 C 30 B

11 B 31 B
12 B 32 B
13 D 33 B
14 B 34 A
15 D 35 C

16 D 36 A
17 D 37 D
18 A 38 D
19 B 39 C
20 B 40 D

© Cambridge International Examinations 2012


Mark scheme for IGCSE Physics (0625/2) – Core Theory May 1999

1 speed = distance/time in any form 1


20/50 1
0.4 (cm/s) 1
3

2 (a)(i) W1 1
(ii) towards pivot, or equivalent 1
(b) pivot between W & W1 1
closer to W 1
4

3 (a) middle box ticked 1


(b) solid molecules vibrating / moving about a fixed position
gas molecules faster, move on one path until collision 2
3

4 (a) chemical 1
heat (or equiv.) 1
light (or equiv.) 1
(b) motion OR kinetic 1
position OR potential 1
(c) geothermal 1
6

5 (a) straight line 1


through origin 1
(b) calculate gradient OR R = V/I 1
3

6 (a) dispersion OR refraction 1


(b) spectrum 1
(c) red 1
(d) red 1
(ideally, “those colours which are not absorbed by the red filter”) 4

7 (a) coil turns 1


(b) current-carrying coil in magnetic field experiences turning effect
(allow l for reference to force on current-carrying conductor/coil) 2
(c) (electric) motor (ignore reference to d.c. or a.c.)
OR galvanometer etc 1
4

8 (a) idea of avoiding contamination 1


(b) idea of preventing radiation getting out 1
(c) idea of warning of the presence of radioactive material 1
3

9 (a)(i) volume = l x w x h 1
3
volume of paper = 0.3 x 0.2 x 0.05 OR 0.003 (m ) 1
(ii) density = mass/volume in any form 1
OR 2.4 or 2.4(stated)
0.3 x 0.2 x 0.05 candidate's volume 1
800 allow e.c.f. if working shown 1

-1-
3
kg/m 1
(b) 24N 1
(c) 24 OR candidate's weight substituted at top 1
0.3 x 0.2 OR 0.06 substituted at bottom 1
400 allow e.c.f. if working shown 1
2
N / m OR Pa 1
(d)(i) half (NOT “less”) 1
(ii) same 1
(iii) half (allow ecf from (i)) 1
(iv) half (allow ecf from (i)) 1
15

10 (a) ruler close to spring 1


measure original length
OR note reading of bottom of spring/other reference 1
hang load 1
measure final length
OR note new reading of bottom of spring etc 1
subtract 1
(b) straight line sloping up 1
through origin 1
(c) straight line through origin, but smaller slope 1
(d)(i) increases speed OR accelerates 1
(ii) same increase 1
same force 1

11 (a)(i) ON DIAGRAM
tube, closed at top 1
reservoir 1
mercury at suitable levels in tube & reservoir 1
appropriate meniscus on mercury (x3) 1
LABELLING
any 2 from: tube, reservoir, mercury, vacuum, scale 2
(ii) atmospheric pressure 1
(iii) h indicating mercury column 1
clear & correct indication of distance 1
(b)(i) 5 (cm) 1
(ii) idea of excess pressure 1
(iii) less 1
L.H. mercury higher than R.H. mercury 1
(iv) LH down, RH up BOTH 1
(v) bigger 1
15

12 (a)(i) A and H 1
(ii) G and F 1
(iii) B and E 1
(b)(i) any waveform 1
reasonably constant “period” (by eye) 1
reasonably constant amplitude (by eye) 1
(ii) vertical line 1
(c) different spacing 1
lady’s “waves” closer together than man’s “waves” 1
(allow 1 only for: man’s “waves” more irregular shape) 9

-2-
Mark scheme for IGCSE Physics (0625/2) – Core Theory November 1999

1 (a) volume = l x b x h 1
= 8 x 5 x 2 1
3
= 80 (cm ) 1
(b) line shown at 80 mark 1
4

2 (a) dot at intersection of AX and BY 1


(b) straight line through C and candidate’s centre of mass 1
2

3 (a) attraction 1
(b) repulsion 1
(c) attraction 1
(d) nothing 1
4

4 (a)(i) BC 1
(ii) had to lift own weight / overcome gravity 1
(b)(i) greater 1
(ii) same ( or greater) work 1
less time / faster / power = work/time 1
5

5 (a) series circuit using recognisable symbols 1


correct symbols for switch and cell / battery 1
correct symbol for battery 1
(b)(i) second bell in parallel with first 1
(ii) more current / amps / charge / energy from battery 1

6 (a) normal clearly and correctly shown 1


angle i correctly marked 1
angle r correctly marked 1
(b) ray bent upwards 1
parallel to incident ray by eye 1
5
7 Any two from
reference to convection
hot water rises
no convection if heated at the top 2
2

8 Any three from


nucleus at centre
electrons in outside nucleus
electrons in orbital / shell
number of electrons equals the charge on the nucleus 3
3

9 (a)(i) fA shown between slit and lens A 1


clear and correct indication of distance 1
(ii) fB shown between lens B and screen 1
clear and correct indication of distance 1
(b)(i) Any two of the following pairs
ray from top of O parallel to axis (by eye)
down through F (± _ mm)

ray from top of O through P (± _ mm)


continuing straight on

-1-
ray from top of O through F’ (± _ mm), not shown
parallel to axis after lens (by eye) 4
(Additional incorrect rays subtract 1 for each)
Both rays bending at centre line / refraction
at both surfaces 1
(ii) image in correct position 1
(iii) ecf for own image drawn
real 1
inverted 1
larger 1
13

10 (a) Any five relevant points


core
coil on right hand limb
coil on left hand limb
iron
primary
secondary
a.c.
input
output
laminations
etcetera 5
(b) good conductor / low resistance / reduction of energy loss 1
(c)(i) correct rearrangement of equation 1
correct substitution 1
VS = 30 (V) 1
(ii) zero 1
(d) the same / a little less because of losses 1
11

11 (a) any two different properties 2


appropriate means of detecting these properties 2
(b) idea of a reference point 1
idea of a physical event which occurs at one temperature 1
(c) steam / boiling water 1
100 1
melting ice / freezing water 1
0 1
any reference to standard pressure / pure / steam in above 1
(d) melting point correctly indicated 1
boiling point correctly indicated 1
correct region identified 1
14

12 (a) air molecules moving (not vibrating) 1


randomly 1
rapidly 1
(b) molecule moving towards the wall 1
molecule bouncing off the wall 1
(c) collision 1
with wall 1
many 1
force / push on wall 1
(d) move faster / gain more energy 1
more / harder collisions (with wall) 1
greater pressure 1
12

-2-
Mark scheme for IGCSE Physics (0625/2) – Core Theory May/June 2000

1 (a) weight 1
(b) mass 1
(c) greater density OR less air gaps between potatoes 1
3
2 (a) Any three from:
conduction
convection
radiation
evaporation 3
(b)(i) suitable procedure 1
(ii) correct form of heat loss for candidate’s procedure 1
5

3 (a) wind wire round iron bar 1


both ends of coil connected to battery 1
extra valid point e.g. cut off length of wire, strip ends etc. 1
(b) suitable check e.g. picking up iron(filings)
deflects compass needle
repels another magnet
move into coil connected to galvanometer 1
4

4 (a) refraction (allow slowing down of light/ angle of refraction) 1


(b)(i) normal correct, by eye 1
(ii) i “correctly” shown between candidate’s normal and ray 1
(c) (i) refraction away from normal 1
along ST 1
(ii) 1. (total internal) reflection 1
2.angle in water> critical angle 1
7

5 (a) idea that light travels faster than sound 1


extra detail
e.g. light travels extremely fast / much faster than sound /
8
instantaneously / at 3x 10 m/s 1

(b)(i) echo OR equivalent 1


(ii) idea of absorption of sound / dispersion / diffraction /
sound has travelled further / lower air density at altitude 1
(c) speed = distance/time in any form 1
(distance = ) approximately 330 x 4 1
1320(m) 1
7

6 (a)(i) smoke particles ticked 1


(ii) 3rd box ticked 1
(b) 4th box ticked 2
4

7 (a)(i) potential OR stored OR position OR gravitational (potential) 1


(ii) kinetic OR motion OR movement 1
(iii) turbine(s) OR coil(s) OR blade(s) OR rotor / fins / wheel 1
electrical / electricity 1

1
(b) Any two from:
waves
tides
steam
geothermal 2
6

8 (a)(i) 320 1
(ii) candidate’s value correctly plotted ± _ small square 1
(b)(i) 160 1
(ii) candidate’s value correctly plotted ± _ small square 1
(c) points plotted at (60,80)and (80,40) ± _ small square 1
(d) smooth curve through candidate’s points, by eye 1
(e) either of last two boxes ticked 1
(f) randomness OR idea of background 1
8

9 (a)(i) D 1
(ii) large(st) area 1
least pressure / pressure more spread out 1
(b) increases decreases stays the same
length AB √
width BC √
thickness CD √ 1
area touching the
horizontal surface √ 1
mass of sheet √ 1
weight of sheet √ 1
density of metal √ 1
pressure on surface √ 1
9

10 (a) R = V/l in any form 1


12/1.5 1
8_ 1
(b)(i) 12(V) 1
0.75 (A) 1
(ii) 16(_) OR 2 x candidate’s (a), correctly evaluated 1
(c) (i) 16 (_/m) OR candidate’s (b)(ii) 1
(ii) 64/candidate’s(ci) 1
4 (m) OR 64/candidate's (ci) correctly evaluated 1
9

11 (a) sound 1
(b) infra-red 1
(c) X-rays 1
(d)(i) no air / vacuum on the moon implied 1
1. sound needs a medium/air 1
2. radio waves don’t need a medium/air 1
(ii) radio / _ wave 1
7

12 (a)(i) B 1
(ii) greater force / weight / heavier 1
reference to formula or constancy of length 1
(b)(i) B 1

2
(ii) greater weight / force / heavier 1
reference to formula or constancy of height lifted 1
(c) weight (of hammer) OR mass OR force 1
time (to lift) 1
(d)(i) energy of motion OR K.E. 1
(ii) Any two from:
heat. OR frictional
sound
K.E. of nail 2
11

3
Mark scheme for IGCSE Physics (0625/2) – Core Theory November 2000

1 speeding up slowing down


steady speed at rest 4

2 (a) YES YES NO 3


(b) increase diameter (accept “size”) of handle (accept “tap”)
OR change to type B/C
OR “oil it” 1
4

3 (a)(i) 31 (m/s) 1
(ii) 6 OR 11 0R 22 (m/s) 1
(iii) 10 (s) 1
(b) distance = area under graph OR distance = average speed x time 1
_ x l0 x 15 1
75 (m) 1
6

4 (a) A, D, E, H ticked 4
(b)(i) E or H 1
(ii) evaporates more rapidly/easily 1
6

5 (a) internal /(random) thermal / kinetic / heat / motion / movement 1


(b)(i) beaker + water / second one / middle one 1
(ii) 1. 250 (g) 1
2. 250 (g) 1
3. 1050(J) 1
4. 200(J) 1
idea that sand requires less energy to raise its temp / temp of sand rises
more for same energy input / equivalent statement for water / water has
bigger SHC 1
7

6 (a)(i) normal correct, by eye 1


(ii) reflected ray correct, by eye 1
(iii) i and r identified by X’s
OR angles with mirror identified by X’s 1
(b) dot in correct position ±2mm 1
(c)(i) ear on right identified 1
(ii) 60 (cm) 1
6

7 (a) refraction OR deviation 1


(b)(i) different path (could be from 2nd surface) 1
below red path 1
starting at first surface 1
2 downward refractions, not below normal at first 1
(ii) dispersion 1
(c) T somewhere just above emergent ray 1
7

8 (a) density = mass/volume in any form 1


(b) 12.6/1.4 1
3
9 (g/cm ) 1

-1-
(c) no e.c.f. 1
(d) brass OR sensible deduction from candidate’s (b) 1
(e) aluminium 1
6
1 2
9 (a) R=R + R 1
60 (_) 1
(b)(i) p.d./current OR voltage/current OR volt/amp OR V/I OR 6/I etc 1
(ii) correct rearrangement 1
6/60 or candidate’s answer to (a) 1
0.1 ecf 1
A OR amp(s) 1
(c) (b)(ii) x 40 1
4 OR candidate’s correct evaluation 1
V or volt(s) 1
(d) answer to (c) or 4 (V) 1
11

10 (a)(i) 6 1
(ii) 90/6 OR 90/ candidate’s answer to (b)(i) 1
15cm e.c.f. 1
(iii) 1. (arcs of) circles 1
2. constant depth 1
same speed in all directions (or equiv.) OR same wave spacing 1
(iv) indication of vertical motion 1
up and down OR circles 1
(b)(i) transverse 1
longitudinal OR pressure 1
(ii) indication of motion along KL 1
backwards and forwards OR back and forth 1
12

11 (a)(i) passes a current / charge / electricity) (NOT conducts electricity”) OR


(some) electrons free to move about 1
(ii) does not pass a current / does not conduct electricity / all charges/electrons
fixed OR bad conductor 1
(b)(i) repel / move away 1
(ii) first and third boxes ticked 2
5

12 (a)(i) 84 or bottom one 1


(ii) 210 or top one 1
(iii) 84 OR bottom one 1
(iv) 210 - 84 OR 126 OR take bottom from top
OR take proton no. from nucleon no. 1
(b) 206 1
82 1
6

-2-
Mark scheme for IGCSE Physics (0625/2) – Core Theory May/June 2001

1 (a) 3.05 ticked 1


(b) 3.35 ticked 1
(c) 30 (minutes) e.c.f. 1
(d) 30/5 e.c.f.
6 (minutes) e.c.f 2
5

2 (a) some idea of turning/rotating 1


effect (of the force) OR force x (perpendicular) distance 1
(b) (i) same 1
(ii) X marked (on branch) at vertical through A ± 1/2 mm 1
4

3 (a) idea of air molecules moving 1


idea of air molecules striking something 1
idea of air molecules striking balloon 1
(b) (i) move faster OR more energy 1
(ii) increases 1
idea of "more collisions" OR "harder collisions" OR "greater
momentum change" OR "greater force" 1
6

4 orientation and size reasonably correct 1


vertical edge correct distance from mirror (by eye) 1
all three corners correct position (by eye) 1
3

5 (a) (i) A and G OR B and H OR C and I 1


(ii) B and C OR E and F OR H and I 1
(b) number of waves / oscillations / cycles / wavelengths / crests / troughs
per second / unit time / divided by time 2
(c) 1/0.2 1
5 (Hz) 1
6

6 (a) +ve and no charge circled 2


(b) does not conduct / bad conductor 1
electricity / charge / current / electrons 1
(c) any acceptable recognisable insulator 1
5

7 (a) 0.1 (A) 1


(b) IR 1
1.2 (V) c.a.o. 1
(c) voltmeter / multimeter set to volts range 1
(d) larger 1
(e) 12+5+3 1
20 (_) 1
7

8 (a) alternating / a.c. 1


N P/N s OR IS / Ip 1
iron NOT steel 1
(b) (i) 12Ns = 100/50 in any form 1

1
6 (V) 1
(ii) 6 (V) 1
(iii) 0 (V) OR "it won't work" 1
7

9 Graph special case:


4 for a perfect line but no visible points
3 for poor line through our points
(a) points correctly plotted ± _ small square 3
reasonable line through candidate's points 1
(b) distance = area under graph OR average velocity x time 1
_ x base x height OR _ x 3 x 15 1
22.5 (m) 1
(c) 25 (m/s) 1
(d) distance = speed x time (in any form) OR area under graph 1
25 x 3 e.c.f from (c) 1
75 (m) e.c.f. from (c) 1
11

10 (a) same 1
(b) greater 1
(c) (i) chemical 1
(ii) food 1
(d) potential OR PE OR gravitational OR position 1
(e) PE decreases 1
KE increases 1
energy turned to sound 1
idea of energy losses OR idea of sound becoming heat 1
9

11 (a) (i) 0.2 x 0.1 x 0.06 OR 20 x10 x 6 1


-3
1.2 x 10 OR 0.0012 OR 12000 1
3 3
m OR cm according to working 1
(ii) candidate’s volume x 2400 (kg) 1
2.88 (kg) 1
(iii) 1440 (kg) 1
(b) (i) less OR lighter OR mass in (a)(ii) is greater 1
(ii) different from brick 1
greater than brick 1
9

12 (a) (i) electrons / cathode rays (NOT _-particles) 1


something moving across screen / hitting the screen 1
idea of fluorescence / luminous / glows / mention of 1
fluorescent screen
(ii) idea of greater flow / energy of electrons / more electrons /
increased intensity of cathode rays / electrons move faster
OR increase anode voltage / increase electron gun voltage 1
(b) (i) (Y) input OR Y-plates 1
(ii) 1.Y gain OR volts/(cm) (NOT Y-shift) 1
2. time-base OR ms/(cm) NOT X-shift 1
3.vertical line OR dot moving up and down 1
8

2
Mark scheme for IGCSE Physics (0625/2) – Core Theory October/November 2001

1 (a) 16 (cm) 1
(b) 16/8 e.c.f. 1
2 (cm) e.c.f. 1
3

2 wider base 1
idea of lowering C of M e.g. heavier base, make it shorter 1
2

3 (a) heat it 1
(b) increases 1
(c) any suitable example 1
(d) any suitable example 1
4

4 (a) ray refracted down 1


parallel to incident ray (by eye) 1
(b)(i) Total Internal Refraction shown 1
Correct angle (by eye) 1
(ii) ray strikes surface at more than critical angle 1
(c) straight on at first surface 1
straight on at second surface 1
7

5 (a) should be refraction at first surface 1


should be dispersion at first surface 1
(b)(i) thermometer / thermocouple / bolometer / hand / thermopile / thermistor /
thermochromic paper / goggles
NOT I.R. detector 1
(ii) I. R. shown above red 1

(c) any two from:


radio (or TV), microwave, ultra-violet (or UV), x-rays, y-rays
(NOT cosmic) 2
6

6 (a) bar magnet held at each end of rod, implicit in answer 1


(i) magnet won't attract / repel (either end) 1
(ii) magnet attracts both ends 1
(iii) magnet repels (one end) OR "the one left after the others are identified" 1
(b)(i) iron (or ferromagnetic) 1
(ii) electricity / current (in coil) 1
(iii) switch off current OR equiv. NOT switch off electromagnet 1
7

7 (a) any two from:


mention of a fault / short-circuit
large current NOT increased current
overheating of cables
fire 2
(b) (electric) shock 1
water conducts / transfers electricity (NOT "water and
electricity don't mix") 1
(c) possible short circuit OR possible (electric) shock 1
5

8 (a) some indication that nucleus changes 1


any particle emitted 1

1
becomes a different atom / nucleus / nuclide / element 1
(b)(i) 5 (min) 1
(ii) 4 OR 20/candidate’s half-life correctly evaluated 1
(iii) 25 ± 5.... only 1
(c) 022 2
100 (NOT –1) 2
10

9 (a)(i) (milli)ammeter OR multimeter set to amp / current range 1


(ii) A e.c.f. 1
(iii) voltmeter OR multimeter set to volt / p.d. range 1
(iv) V e.c.f. 1
(b)(i) resistor / resistance 1
variable 1
RHEOSTAT (2) NOT potentiometer
(ii) vary current OR vary p.d. across R NOT vary resistance 1
(c)(i) find gradient OR choose one point and use p.d./current 1
(ii) correct figures from graph seen or implied 1
5 1
_ OR ohm 1
(d)(i) greater 1
(ii) straight line through origin 1
greater slope e.c.f. from (i) 1
14

10 (a)(i) aluminium 1
greatest density OR "because mass increases as density
increases 1
(ii) aluminium OR candidate’s (a) (i) 1
(iii) aluminium OR candidate’s (a) (i) 1
(b) A 1
smallest area 1
(c)(i) small area OR "because pressure increases as area
decreases" 1
great pressure (on sand) 1
(ii) any sensible suggestion involving larger area in contact with
sand (must involve chair and sand) 1
9

11 (a) temperature NOT °C 1


(b) mercury OR alcohol OR pentane 1
(c) 100 1
°C NOT C° or C 1
(d) stays the same OR "nothing" 1
(e) ice 1
pure OR melting 1
any detail of sensible apparatus 1
8

12 (i) bigger temperature difference 1


(ii) the glass is thin OR the window is a good conductor
OR because it isn't double-glazed 1
(b) sum of figures quoted 1
4 150 000 (J/hour) 1

(c) Any one of:


convection
radiation
through roof/ceiling
through floor

2
draughts
opening door/ window
NOT switching on air conditioning 1
5

3
om
s.c
er
ap
eP
em
tr
.X
w
w
w
Page 2 Mark Scheme Syllabus Paper
IGCSE EXAMINATIONS – JUNE 2003 0625 2

TARGET
QUESTION SCHEME MARK
GRADE
1 (a) 8 F B1

(b) EITHER OR
greater smaller
AND AND F B1
Thickness of rule thread stretched when 2
on rule
OR overlap at ends OR worn rule ends

2 (a) (i) 10 F B1

(ii) stretch OR shape (or suitable sketch) C B1

(b) (i) 120 F B1

(ii) up(wards) OR vertical (NOT vertically down) F B1


Accept arrow on diagram

(iii) increase size/area of blocks/larger blocks F B1


OR increase number of blocks 5
OR less soil in pot (NOT put pot on harder
ground)

3 (a) 0.97 – 0.51 F C1

0.46 F A1

(b) (i) 15 F B1

(ii) 515 e.c.f. C B1

(iii) D = M/V in any form, seen or implied F C1


(words/letters/mix)

EITHER OR OR

460 0.46 0.46 C C1


515 515 515 ´ 10-6 e.c.f.

0.8932… 8.932… ´ 10-4 893.2…No e.c.f. C C1


(any number of significant figures)

0.89 8.9 ´ 10-4 890 (e.c.f. for C A1


significant
figures)

g/cm3 kg/cm3 kg/m3 F B1


(0.89 kg/dm3 is (NOT 8.9-04) 9
OK)

© University of Cambridge Local Examinations Syndicate 2003


Page 3 Mark Scheme Syllabus Paper
IGCSE EXAMINATIONS – JUNE 2003 0625 2

4 (a) idea of air molecules moving (allow vibrating) F C1


(N.B. ‘collide’ = ‘moving’)

idea of air molecules striking something (condone F C1


themselves)

idea of air molecules striking walls C A1

(b) (i) moves down F B1

(ii) increases (e.c.f.) F M1

idea of more collisions must follow C A1


(per unit time) (e.c.f.) from (i) 6
OR P µ V1

5 (a) line starting at 0 oC F B1

reasonably horizontal line at any temp for > half the C M1 mark
time along-
side
horizontal from zero time as far as dotted line (ignore C graph
anything to R. of line) A1

(b) (i) water boils OR heat loss = C B1


heat supplied (NOT evaporates/
turns to gas) mark (i)
and (ii) C
(ii) gives water/molecules energy to together B1
escape OR break bonds 5
OR change state OR heat loss
from sides/surface/to air

6 (a) (i) normal correct, by eye F B1 mark


along-
(ii) reflected ray correct, by eye (ignore normal; F B1 side
ignore any arrow) diagram

(iii) both i and r correctly marked (condone F B1


sloppy normal and sloppy refracted ray)

(b) parallel to ray striking mirror 1 (allow incident ray) C B1


OR same direction (NOT equal/same as) 4
(N.B. sentence must be completed, i.e. no inference
from line on diagram)

© University of Cambridge Local Examinations Syndicate 2003


Page 4 Mark Scheme Syllabus Paper
IGCSE EXAMINATIONS – JUNE 2003 0625 2

7 (a) 680 1020 1360 1700 F B1

(b) 5 points plotted ± 1


2 small square (-1 e.e.o.o.) F B2
ignore 0,0 (e.c.f.)

reasonable line through his points – F B1


drawn with rule/thickness reasonable

(c) (i) flash F B1

light travels quickly OR sound travels slowly F C1


(accept figure)

light travels faster than sound (accept figure) F C1

light travels much faster than sound (accept C A1


figures)

(iii) 1400 - 1450 OR correct value from his graph F B1


± 12 square

clear and correct indication on graph of how F B1


obtained (minimum: dot at appropriate point) 10

8 (a) Charge(s) OR energy (NOT electricity (condone as C B1


extra), charged particles (condone as extra), current,
electrons (condone as extra), voltage)

(b) (i) 0 C B1

(ii) mention of 6V F B1

mention of rising OR not instantaneous (NOT C B1


‘reads’)

(iii) any realistic example of something turned F B1


on/off after a time lapse, e.g. electronic egg 6
timer, turn-off bedside radio

9 (a) (i) wire shown curved between A and B F C1 mark


along-
side
wire displaced all along between A and B, C A1 diagram
and reasonably smooth

(ii) idea of force (in any direction) F M1

on current/current-carrying conductor C A1

when in magnetic field C A1

© University of Cambridge Local Examinations Syndicate 2003


Page 5 Mark Scheme Syllabus Paper
IGCSE EXAMINATIONS – JUNE 2003 0625 2

(b) line curved in opposite direction F B1 mark


6 along-
side
diagram

10 (a) (i) electrons OR cathode rays (NOT beta- F B1


particles)

something ‘hitting’ the screen (NOT ‘form a F B1


spot’)

idea of fluorescence (of the screen, NOT ‘the C B1


gas’)

(ii) focus C B1

(iii) time base OR ms/cm C B1

(iv) electrons/cathode rays deflected (e.c.f. from F B1


(i); allow ‘attracted’ if intention clear)

something deflected horizontally C M1

some idea of repeated sweeps/back and C A1


forth

(b) (i) (y-)input (allow y-plates) F B1

(ii) 1. trace moves horizontally/sideways/left/right C B1

2. trace moves vertically/up/down C B1


11

11 (a) Connection to either side of cell, but not shorted out F B1

VR in series with lamp, and not shorted out OR F B1


correctly connected as a potential divider (condone
inclusion of a switch)

(b) (i) R1 + R2 F C1

12 F A1

(ii) 1. Resistance = p.d./current in any form F C1


(words/letters/mix)

6/12 e.c.f. C C1

0.5 or 12 e.c.f. C A1

© University of Cambridge Local Examinations Syndicate 2003


Page 6 Mark Scheme Syllabus Paper
IGCSE EXAMINATIONS – JUNE 2003 0625 2

2. his calculated current

his calculated current all 3 C B1

his calculated current

A OR amp OR ampere somewhere in (ii) F B1

(iii) voltmeter shown correctly connected (any C B1 mark


recognisable symbol; allow re-drawn circuit) 10 along-
side
diagram

12 (a) his weight F B1

(b) distance OR height F B1

(c) (i) 1000N climber OR heavier OR first F B1

(ii) his answer to (i) F B1

(d) (i) chemical (accept fuel) C B1

(ii) food (accept muscles) C B1

(iii) maintaining body function

heat loss any 1 C B1


K.E. 7
sounds

Mark first correct answer, condone extras

© University of Cambridge Local Examinations Syndicate 2003


Page 2 Mark Scheme Syllabus Paper
PHYSICS – NOVEMBER 2003 0625 2

TARGET
QUESTION SCHEME MARK
GRADE
1 (a) (i) G within block, to left of vertical through midpoint or AB F B1
(ii) Vertical line shown through A C B1
(b) A F M1
more stable (or equivalent statement) e.g. less likely to F A1
topple or “weight within base”
(c) so it does not topple over (or equivalent) F B1
5
2 reference mark on wheel *“(use stopwatch to)
time…” gets only one
datum line (could be “top” or “bottom”)
of these
*start timing/stopwatch as mark passes datum line
time a number of rotations (accept 1 here) 5C B5
time at least 20 rotations any 5
*stop stopwatch
divide time by number of rotations
repeat
make sure stopwatch at zero 5
3 gravitational OR potential OR PE OR GPE F B1
motion OR KE OR kinetic
heat/internal/thermal any order (–1 eeoo) 3F B3
sound
heat (accept potential) C B1
OR internal/thermal
NOT strain potential/NOT chemical potential
NOT sound, even as an extra 5
4 (a) vehicle 2 F M1
large(r) area (in contact with ground) C A1
low/less pressure C A1
less likely to sink/get stuck F A1
(b) (i) small area F C1
large pressure F B1
(ii) (weight spread over) large(r) area NOT body area C B1
small/less pressure C B1
reference to weight somewhere in (b) C B1
9
5 (a) (i) ray perpendicular to surface at A (by eye) F B1
(ii) normal at B correct (by eye) F B1
(iii) ray refracted down at B, but NOT along surface C B1
(iv) normal at D correct (by eye) F B1
(v) ray refracted up at D, but NOT along surface C B1
Page 3 Mark Scheme Syllabus Paper
PHYSICS – NOVEMBER 2003 0625 2

(b) converging OR will meet OR *one up, one down ALLOW * C B1


“opposite” *only if diagram acceptable

same deviation (or equivalent) OR “angles of refraction C B1


same”
(c) straight on OR split (depending on thickness of “ray”) OR no F B1
change (indirection) OR not refracted
8
6 (a) (i) speed F B1
(ii) frequency, ALLOW wavelength C B1
(iii) wavelength F B1
(b) gamma OR γ OR cosmic C B1

condone x-rays as an extra 4

7 (a) straight line sloping up to right F B1


through origin F B1
(b) (i) voltmeter OR multimeter on volts range (condone spelling) F B1
(ii) potential difference OR p.d. OR volts/voltage (no e.c.f.) F B1
(iii) ammeter OR multimeter on current/amps range (condone F B1
spelling)
(iv) current OR intensity OR amps/amperes OR ampage (no F B1
e.c.f.) NOT A
(v) evidence of 7.5 F C1
evidence of 0.3 F C1
7.5/0.3 OR V/I OR volts/current e.c.f. if written down C C1
25 e.c.f. only if V/I used C A1
Ω or ohm C B1
(vi) hisR/50 F C1
0.5 (Ω/m) e.c.f. C A1
13
8 (a) EITHER OR
iron filings (plotting) compass F B1
NOT “put” place near end of magnet F B1
sprinkle/spread/pour/scatter 0000 0000000 000000
tap card mark end(s) of compass C B1
further detail further detail C B1
(b) attraction of compass S pole
repulsion of compass N pole
attraction of S pole of another magnet F B1
any 1
repulsion of N pole of another magnet
attraction of Earth’s N pole
repulsion of Earth’s S pole 5

© University of Cambridge Local Examinations Syndicate 2003


Page 4 Mark Scheme Syllabus Paper
PHYSICS – NOVEMBER 2003 0625 2

9 (a) (i) decreases F M1


by 2 C A1
(ii) decreases F M1
by 2 C A1
(iii) decreases C B1
(b) 66 (yrs) F C1
evidence of 3 half-lives C C1
fraction 1/8 seen or implied C C1
400 C A1
9
10 (a) 1 3F B3
points plotted correctly ± small square (–1 eeoo) ignore
2
0,0 (–1 for very large blobs)
(b) 45 circled OR 2 circles ACCEPT point circled on graph F B1
(c) idea of stretching more than before (for same load increase) C B1
OR reference to elastic/proportional limit in some way
(d) EITHER OR
measure unloaded length idea of fixed end and free F B1
ALLOW “measure spring” end
NOT extension
measure loaded length NOT note position of free end, no F B1
extension load
subtract measure movt. free end, F B1
loaded
8
11 (a) (i) 100 F B1
(ii) 0 F B1
(iii) indication to the left of 0°C mark C B1
(b) expansion of a solid
expansion of a gas/pressure of a gas
current/pd/e.m.f. of a thermocouple
conductivity/resistance of a conductor/wire/thermistor….. any 1 C B1
colour of a hot wire
melting of a wax
NOT expansion of alcohol ACCEPT density of a liquid 4
12 (a) N1/N2 = V1/V2 in any form F C1
8000/N2 = 240/6 or correct substitution into correct equation F C1
200 ALLOW B1 for 20 if 800 used instead of F A1
8000 (working must be shown)
(b) (i) 200 e.c.f. i.e. his (a) F B1
(ii) 400 e.c.f. i.e. 2 x his (a), evaluated C B1
5

© University of Cambridge Local Examinations Syndicate 2003


Page 2 Mark Scheme Syllabus Paper
PHYSICS - JUNE 2004 0625 2

TARGET
QUESTION SCHEME MARK
GRADE

1 (a) 10 F B1

(b) division by 5 OR division by 6 F C1

2.0 OR 2 c.a.o C A1

(c) 10 × his(b) OR 11 × his(b) F C1

20 c.a.o C A1
5

2 (a) straight vertical arrow upwards to/from rail F B1

arrow to R of centre of rail F C1

arrow at R.H. end of rail (within 2 × width of resting F A1


block)

(b) moment ticked F B1

(c) reduce weight/mass OR shorten rail, lighter rail, F B1


thinner rail, open sideways, suitable long handle, 5
suitable 2 pulley system

3 (a) PQ or 0-50s or the horizontal part F B1


NOT just P or just Q

(b) changing speed (however indicated) NO e.c.f from F B1


(a). ACCEPT “acceleration”
but NOT “increasing speed”

(c) distance = area indicated in words or figures F B1


anywhere in (c)

(i) 20 × 50 F C1

1000 F A1

(ii) ½ × 20 × 50 OR ½ × his(i) C C1

500 C A1

(iii) his(i) + (ii) correctly evaluated F B1

(iv) his(iii)/100 OR total distance/total time stated F C1

correct evaluation F A1
10

© University of Cambridge International Examinations 2004


Page 3 Mark Scheme Syllabus Paper
PHYSICS - JUNE 2004 0625 2

4 (a) (horizontal) force F B1


allow F

distance (travelled from A to B) F B1


condone “perpendicular”
allow D OR d OR S

(b) goes faster OR less time F B1

accelerates C B1

(c) (i) 2nd person (however expressed) F B1

(ii) more work/energy OR bigger force OR pulls F B1


harder

smaller time OR greater speed C B1


(“more work/second” gets B1, B1) 7

5 (a) drops OR decreases OR cools down F B1

(b) idea of loss of molecules (from surface) OR F M1


molecules evaporate

more energetic/faster molecules C A1


(SPECIAL CASE remaining molecules slower B1)

(c) any sensible example where cooling is noticeable C B1


e.g. (feeling cold) after swimming, sweating, 4
refrigerators

6 (a) (increased) internal energy OR (increased) KE of C B1


molecules OR (increased) thermal/heat (energy)

(b) any mention of thermal capacity C C1

smaller thermal capacity C A1


3

7 (a) light wave fastest ) 2F B1+B1


)
water wave slowest )

(b) longitudinal F B1

transverse C B1

transverse F B1

(c) light wave ticked use + = 0 if extras F B1


6

© University of Cambridge International Examinations 2004


Page 4 Mark Scheme Syllabus Paper
PHYSICS - JUNE 2004 0625 2

8 A magnet OR magnetised F B1
B magnet OR magnetised F B1
C iron OR unmagnetised C B1
D aluminium C B1
4

9 (a) points plotted correctly ( ± ½ small square) 3F B3 (-1 eeoo)

(b) smooth curve through points by eye, not too thick F B1

(c) correct construction lines shown C B1


(allow dot on curve at correct place)

correct value from his graph, based on 800-400 F B1


( ± ½ square)

(d) (i) smaller F B1

(ii) the same OR no change C B1


8

10 (a) (i) less turns on secondary F B1


ACCEPT “because Np=4800 and Ns=200”
ACCEPT “sycoil < pycoil”
NOT “secondary < primary”

(ii) V2/V1 = N2/N1 in any form F C1

correct substitution F C1

10 F A1

(iii) 1. decreases F B1

2. runs slower OR will not work e.c.f. from F B1


(iii)1.

(b) ignore stage 1


from stage 2 onwards……

B )
E ) (3 marks for any 3)
A ) (2 marks for any 2) 3C B3
D ) (1 mark for any 1) 9

© University of Cambridge International Examinations 2004


Page 5 Mark Scheme Syllabus Paper
PHYSICS - JUNE 2004 0625 2

11 (a) (i) thermistor F B1

(ii) variable resistor (accept rheostat) F B1

(iii) light-dependent resistor F B1


(ACCEPT LDR)

(b) (i) 1. resistance = p.d./current OR R=V/I F B1


OR any correct reorganization
ACCEPT mixture of words and letters

2. 12/0.5 OR correct sub in his 1, if shown F C1

24 c.a.o F A1

Ω OR ohm C B1

(ii) 1. decreases F B1

2. idea of greater resistance F B1

3. dimmer OR does not glow/work/shine C B1


NOTE: NO e.c.f. in (ii) 10

12 (a) (i) beard tip to dot perpendicular to mirror (by F B1


eye)

distance beard tip to mirror = dist. mirror to F B1


dot (by eye)

(ii) reflected ray along line from eye to his dot C M1


(by eye)

incident ray from beard tip to join reflected C A1


ray at mirror

arrows from beard to eye C B1

(iii) virtual C B1

(iv) angle of incidence = angle of reflection OR F B1


i = r OR “they are equal” OR “sini = sinr”

(b) (i) right hand F B1

(ii) mark shown under L.H. eye on Fig. 11.2 F B1


9

© University of Cambridge International Examinations 2004


Page 1 Mark Scheme Syllabus Paper
IGCSE – November 2004 0625 2

Question Scheme Target


Mark
Grade
1 (a) 96 F B1
(b) top left eye circled F B1
(c) 100 – 96 e.c.f. F C1
4 e.c.f. F C1
4/80 e.c.f. F C1
0.05 e.c.f. F A1
6
2 (a) (i) 21 F B1
(ii) 1 increases F B1
2 falls )
) both e.c.f. F B1
3 rises )
(b) (i) same C B1
(ii) greater (accept comparison of directions) C B1
5
3 (a) (i) 0.6 x 0.5 x 0.2 F C1
0.06 F A1
3
m C B1
(ii) mass/volume F C1
7.2/his(i) F C1
120 e.c.f. F A1
kg/m3 C B1
(b) (i) “greater than” ticked C B1
(ii) Because of the air in the corrugated cardboard C B1
9
4 (a) (i) chemical )
internal OR heat OR thermal ) any 2
B1,
but also accept ) 2F
B1
nuclear OR kinetic OR potential for one of the
marks
(ii) radiation F B1
(b) (i) K.E. OR kinetic OR motion C B1
(ii) conduction F B1
(iii) 1 gravitational OR P.E. OR potential OR
F B1
position
2 chemical/fuel/food C B1
7

© University of Cambridge International Examinations 2005


Page 2 Mark Scheme Syllabus Paper
IGCSE – November 2004 0625 2

5 (a) bright specks OR (smoke) particles OR dots


F B1
NOT molecules, NOT air particles
something moving (however expressed) F M1
rapidly OR randomly OR zig-zag F A1
(b) air molecules OR air particles (NOT just “air”) C M1
collisions between smoke particles and air
C A1
mols/particles
invisible )
B1,
fast moving ) any 2 2C
B1
random )
(c) Idea of faster movement or more jerky movement F B1
8

6 (a) aluminum F B1
copper F B1
gold F B1
(b) insulator NOT non-conductor F B1
(c) (i) copper OR aluminum OR gold F B1
(ii) any of the insulators F B1
(d) convection F B1
(e) intelligent mention of convection currents C B1
(accept “to make sure always covered by water”
NOT “heat rises”)
8
7 (a) (i) speed = distance/time in any form F C1
correct substitution in his equation OR 1600/5 F C1
320 c.a.o. F A1
(ii) light travels instantaneously/very fast OR
reference to reaction time
NOT just faster than sound C B1
(b) explosion heard sooner/less than 5s )
second sound/echo/reflection (from ) any 2 B1,
2C
rock face) ) B1
louder )
6

© University of Cambridge International Examinations 2005


Page 3 Mark Scheme Syllabus Paper
IGCSE – November 2004 0625 2

8 repulsion F B1
repulsion F B1
attraction F B1
attraction C B1
attraction C B1
5
9 (a) (i) 3 )
F B1
3 ) both
(ii) 10 + 10 F C1
20 F A1
(b) (i) 1 1.5A ticked F B1
2 3A OR 3.0A C B1
(ii) 5 Ω ticked C B1
6
10 (a) electrons F B1
(b) neutrons OR atoms F B1
(c) electrons F B1
(d) electrons F B1
(e) protons C B1
neutrons C B1
(f) protons C B1
neutrons C B1
8
11 (a) (i) A or E F B1
(ii) CE or AC or 2.5 cm F B1
(b) real )
) B1,
inverted ) mark in pairs using ✓ + ✗ = 0 2f, 1C B1,
) B1
enlarged )
(c) any ray from 0’ to I’ refracting at mid-line of lens or
F B1
refractions at both surfaces
6

© University of Cambridge International Examinations 2005


Page 4 Mark Scheme Syllabus Paper
IGCSE – November 2004 0625 2

12 (a) (i) line from B to intersection F B1


(ii) dot at printed intersection F B1
(b) L.H. ornament C M1
lower C of M C A1
(c) block A F M1
narrower/smaller base F A1
6

© University of Cambridge International Examinations 2005


Page 1 Mark Scheme Syllabus Paper
IGCSE – JUNE 2005 0625 2

Question Scheme Target


Mark
Grade
1 (a) 100 – 20 F C1
level shown at 80 (ignore meniscus) ± 1mm (need
F A1
not actually draw surface)
(b) (i) 16 (cm) F B1
(ii) sub of (i) into formula F C1
sub of 100 into formula F C1
6.25 e.c.f. from (i) F A1
2
cm C B1
Accept working from any two correct values 7
2 (a) 2100 – 900 F C1
12 OR 10.06 (i.e. travel time) F A1
(b) 1500 (ignore unit) OR 15.00 OR ‘1500 to 1800’ F B1
(c) 1 F B1
(d) (i) find area of first trapezium (or equivalent) bold
C B1
area under graph
(ii) find total area C M1
divide by total time OR divide by 12 OR divide
C A1
by travel time
7
3 (a) it turns F M1
clockwise C A1
(b) (i) stationary OR nothing F B1
(ii) horizontal arrow (intention of from OR to pivot -
F M1
condone gap, if clear)
to left F A1
10 (N) C B1
6
4 (a) gravitational F B1
(b) kinetic F B1
(c) kinetic C B1
(d) electrical F B1
(e) internal ACCEPT heat condone valid extras C B1
5

© University of Cambridge International Examinations 2005


Page 2 Mark Scheme Syllabus Paper
IGCSE – JUNE 2005 0625 2

5 (a) (i) increased


F B1

(ii) molecules move faster/have more energy C B1


OR collisions more energetic
more frequent collisions (condone with each
C B1
other)/harder collisions
collisions with walls C B1
idea of collisions causing a force C B1
(b) (i) evaporation F B1
(ii) energy of molecules increases/molecules move
C B1
faster
molecules escape/break bonds F C1
faster/higher energy molecules escape C A1
9
6 (a) resistance = voltage/current in any form, letters,
F C1
words, figures, mix
330 6.3 2F A1+A1
(b) candidate’s 4 points correctly plotted ± ½ small
2F B2
square -1 e.e.o.o.
(c) (i) reasonable curve, smooth, not too thick F B1
(ii) resistance/temperature changes smoothly C B1
(d) (i) 1500 - 1700 OR candidates lowest point on
C B1
graph, quoted correctly
(ii) idea of lowest resistance C B1
9
7 (a) (from L to R) infra red (OR µwave)/ ultra violet/
4F B1 x 4
x rays/gamma rays
sound ticked F B1
5
8 (a) one 90° deviation at 1st mirror, by eye F C1
two 90° deviations at 1st mirror, by eye F A1
emerging rays parallel to incident rays, by eye F B1
(b) upside down OR inverted OR same size OR real C B1
4

© University of Cambridge International Examinations 2005


Page 3 Mark Scheme Syllabus Paper
IGCSE – JUNE 2005 0625 2

9 (a) speed = distance/time )


) in any form F C1
OR speed = 2 x distance/time )
2 x 249/332 F C1
1.5 F A1
(b) 0.75 and 2.25 ticked e.c.f. use + x = 0 for
2C B1 + B1
extras
Mark (b) independent of (a) 5
10 (a) (i) conductor OR metal F B1
(ii) metal (but not if answer to (i)) OR any named
F B1
metal OR any named liquid conductor
(ii) apply an e.m.f. OR apply a p.d. OR equivalent C B1
(b) (i) insulator NOT non-conductor (but condone as
F B1
extra)
(ii) any suitable example F B1
(c) insulator OR ‘(b)(i)’ e.c.f. from (b)(i) NOT just
F B1
the name of a type of insulator
6
11 (a) same ticked F B1
(b) 2 C B1
(c) R1 + R2 in any form, letters or numbers F C1
4 e.c.f. from (b) C A1
(d) (i) 1Ω ticked F B1
(ii) more than 0.3A ticked C B1
(e) (i) increases/stretches F B1
(ii) increases c.a.o. C B1
8
12 (a) 1 (unit) / neutral / zero / in nucleus
1/1800 (unit) OR tiny / -1 (unit) / in orbit
OR outside nucleus
1 4F,2C B1 x6
OR OR negligible NOT just negative OR
2000
in shell NOT zero
(b) (i) proton and neutron F B1
(ii) 1. 4 (units) C B1
2. (+)2 (units) NOT -2 C B1
9

© University of Cambridge International Examinations 2005


Page 1 Mark Scheme Syllabus Paper
IGCSE – NOVEMBER 2005 0625 2

MARK

1 (a) 8.5 (cm) B1

(b) 19.0 OR 19 (± 0.1) (cm) B1

(c) his (b) – his (a) C1

his correct subtraction A1

[4]

2 (a) distance/time C1

25/2 C1

12.5 A1

m/s B1

(b) less OR decreased OR slowing down B1

(c) more than ecf B1

[6]

3 (a) skate M1

small area (in contact with ice) A1

(b) large area )


wind causes large force on side of truck ) any 2 B1,B1
vehicle liable to blow over )
[4]

4 (a) 40 or 160 B1

(b) 720 B1

(c) W=Fxd C1

720 x 0.2 C1

144 A1

J OR joule B1

(d) his(c)/1.2 C1

his(c)/1.2 correctly evaluated C1

0.5 x his(c)/1.2 correctly evaluated A1


i.e. 60 gets C1, C1, A1 and 120 gets C1, C1, A0)
W OR watt OR J/s B1

[10]

© University of Cambridge International Examinations 2005


Page 2 Mark Scheme Syllabus Paper
IGCSE – NOVEMBER 2005 0625 2

5 (a) level in tube lower, or equivalent B1

(b) air has expanded (could be scored in (a), but not twice)
OR Kinetic Theory application to pressure B1

(c) any sensible comment B1


e.g. limited temp range, air bubbles out of tube, slow acting,
large volume of air, change in air pressure, no scale
[3]

6 (a) (i) current (in coil) B1

magnetic field (around coil) B1

(ii) magnetised OR attract B1

(b) current zero at first (even if only at origin) B1

horizontal first part B1

vertical rise somewhere B1

horizontal final part B1

[7]

7 (a) three rays parallel and horizontal B1

(b) (i) both principal foci marked B1

(ii) refraction at mid-line, then through F B1


(allow 2 surface refractions if lead back to mid-line)

(iii) ray through F to mid-line, then parallel B1


(allow as (ii))

(iv) image drawn between axis and intersection, perpendicular to axis C B1


(condone no labelling)

drawing accuracy mark for image 2 squares tall ± 2mm and


4 squares away ± 2mm B1

[6]

8 (a) (i) iron OR steel OR any ferromagnetic material B1


(B0 if magnetised stated)

(ii) 1. nothing ecf from (i) B1


2. nothing B1

(b) L.H. compass pointing to R B1

top compass pointing to L B1

© University of Cambridge International Examinations 2005


Page 3 Mark Scheme Syllabus Paper
IGCSE – NOVEMBER 2005 0625 2

bottom compass pointing to L B1

[6]

9 (a) 1 correct C1

2 correct C1

4 correct A1

(b) (i) all 6 components shown in series (any order) B1


ecf from (a) for symbols

(ii) voltmeter connected across cell, either our diag or his B1

(iii) both B1

(iv) 0.5 B1

(v) current stops OR ammeters read zero OR other bulb goes out B1

[8]

10 (a) 10 x 4 x 6.5 C1

260 (cm3) A1

(b) D = M/V in any form, words, letters, numbers, mixed C1

250/his V ecf if written down C1

0.961538 any no. of sig figs ecf C1

0.96 ecf A1

g/cm3 unless inconsistent with his figures B1

[7]

© University of Cambridge International Examinations 2005


Page 4 Mark Scheme Syllabus Paper
IGCSE – NOVEMBER 2005 0625 2

11 (a) electrons B1

(b) A B1

(c) (i) D B1

(ii) idea of detecting electrons/making spot visible B1

(d) deflects them B1

(e) no air OR no molecules OR no particles OR “nothing” B1

to stop/slow down/absorb the electrons/cathode rays B1

[7]

12 (a) (i) time taken for (B0 for half the time) B1

activity/count-rate/mass etc. B1

to decrease to half original value B1

(ii) radiation due to surroundings B1

(b) (i) 80 – 25 C1

55 cao A1

(ii) 1. 27.5 ecf B1

2. 52.5 ecf B1

(iii) 15 ± 1 ecf B1

(iv) background remains, even when source has decayed B1

(v) curve to the left of existing one B1

flattening out at 25 count/min B1

[12]

© University of Cambridge International Examinations 2005


Page 1 Mark Scheme Syllabus Paper
IGCSE – May/June 2006 0625 02

TARGET
GRADE MARK
1 (a) larger area F B1
smaller pressure F B1

(b) (i) get larger OR get firmer F B1

(ii) molecules move faster )


more collisions (per second) ) any 2 2C B1 + B1
pressure increased )

(c) (i) increases F B1

(ii) smaller volume F B1


more collisions (per second) C B1
anywhere in (b)(ii) or (c)(ii), collisions with walls C B1
9

2 (a) 6.0 ± 0.1 )


2.4 ± 0.1 ) -1 each error or omission 2F B2
3.1 ± 0.1 )

(b) AB x BC x CD OR l x b x h OR his figures shown multiplied F B1

(c) cm3 OR cu.cm OR cubic cm F B1


4

3 (a) P and Q F B1

(b) R and S F B1

(c) (i) D = M/V in any form, including our figures F B1

(ii) 57.5/25 C C1
2.3 C A1
g/cm3 C B1
6

4 (a) chemical, gravitational, internal, kinetic 2F,2C B1 x 4


(if more than 4 ticked, use  +  = 0 )

(b) kinetic NOT internal F B1

(c) potential F B1

(d) chemical C B1
7

5 (a) idea of greater speed F B1


idea of molecules further apart C B1

(b) (i) any suitable example involving expansion or contraction


e.g. thermometer, thermostat, bimetal strip, rivets, fitting steel tyres F B1

(ii) any suitable example involving expansion or contraction


e.g. expansion gaps in bridges etc, overhead cables, cracking glass C B1
4

© University of Cambridge International Examinations 2006


Page 2 Mark Scheme Syllabus Paper
IGCSE – May/June 2006 0625 02

6 (a) (i) wavelength labelled clearly ± 3mm F C1


wavelength labelled clearly ± 1mm C A1

(ii) horizontal line F M1


anywhere between top & bottom of wave pattern C A1

(b) (measure time for) (count number of) (measure time)


(a number of ) OR (waves passing ) OR (for 1 wave ) F M1
(waves to pass ) ( ) (to pass )
f = no. of waves OR in 1 second OR f = 1/T F A1
time 6

7 (a) ray bent down at first surface F M1


not below normal F A1
ray bent down at second surface C B1

(b) (i) dispersion ticked F B1

(ii) red C B1

(iii) violet C B1
(allow B1,B0 if red and violet both written but interchanged) 6

8 (a) end/point on magnet C B1


idea of pointing N (when freely suspended) F B1

(b) repulsive F B1

(c) (i) S at top and N at bottom F B1

(ii) disappears F B1
5

9 (a) strontium-90 F M1
decays most slowly OR longest half-life F A1

(b) (i) points correctly plotted ± ½ small square -1 each error or omission 3F B3

(ii) reasonable curve F B1

(iii) 8 (days) ± 0.5 OR his correct value ± 0.5 C B1


correct working shown on graph (minimum: dot on line) C B1
8

10 (a) (i) A and B (both) OR A and C (both) C B1

(ii) filament F B1

(iii) electrons ticked F B1

(iv) line along axis (by eye) OR conical beam along axis F B1

(v) light or glow indicated somehow F B1

(b) beam deflection shown F C1


beam deflected upwards C A1
indication of curve (condone curve outside electric field) C B1

(c) idea of no obstruction for cathode rays/electrons C B1


9

© University of Cambridge International Examinations 2006


Page 3 Mark Scheme Syllabus Paper
IGCSE – May/June 2006 0625 02

11 (a) (i) 10 x 2.5 F C1


25 (m) F A1

(ii) speed = distance/time in any form F C1


500/10 F M1
50 (s) F A1

(b) 75/2.5 C C1
30 (m/s) C A1

(c) accelerated F B1

(d) total distance = 3000 (m) F C1


total time = 150 (s) F C1
average speed = total distance/total time C C1
3000/150 C C1
20 (m/s) C A1
13

12 L1 joined to R3 or R1 F B1
L2 joined to R4 F B1
L4 joined to R1 F B1
3

© University of Cambridge International Examinations 2006


Page 3 Mark Scheme Syllabus Paper
IGCSE - OCT/NOV 2006 0625 2

QU. SCHEME MARK

1 (a) 55 (s) B1

(b) 55/5 ecf C1


11 (s) ecf A1
(c) EITHER OR OR OR

300/hour 1 takes 11s 5 takes 55s 1hour for 300 B1


= 5/min 300 take 3300s 300 take 60x55s 3600/300s for 1 B1
takes less less than 1hr less than 1hr 1 takes less
than 1 min for 300 for 300 time than B1
for 5 this

YES/NO ticked according to his working B1


7

2 2nd box ticked B1

3rd box ticked B1


(use  +  = 0 for extras) 2

3 (a) OP accelerating B1
PQ accelerating B1
QR constant speed B1
RS slowing down (however expressed) B1
(b) O and S (both) B1

(c) 6 (m/s) B1

(d) 70 (s) B1

(e) find area OPQRS (however expressed) B1


8

4 (a) (i) radiation B1

(ii) conduction B1

(b) (i) kinetic (however expressed) B1

potential (however expressed) B1

(ii) idea of energy loss or friction B1


5

© UCLES 2006
Page 4 Mark Scheme Syllabus Paper
IGCSE - OCT/NOV 2006 0625 2

5 (a) (magnitude of) force B1

distance (from fulcrum) B1

(b) (i) force B1

moment OR turning effect B1

(ii) F1 + F 2 + W B1

(iii) F B1
6

6 (a) (i) eye to image line perpendicular to mirror, by eye B1

object distance = image distance, by eye B1

(ii) normal correctly drawn, by eye B1

(iii) ray to bottom edge of mirror correct M1

reflected ray at correct angle to mirror, by eye


(condone sloppy normal) A1

(b) sensible attempt at explanation B1

(c) (i) 2 (m) (NO ecf) B1

(ii) distance lady to mirror = 3 (m) C1

distance moved = 2 (m) (NO ecf) A1

away from mirror/wall B1


10

7 (a) (i) large, OR accept any large example e.g. cliff B1

(ii) speed = distance/time OR speed = 2xdistance/time C1


(in any form)

correct substitution C1

480 (m) c.a.o. A1

(b) speed = 6/50 OR 3/50 C1

0.12 (m/s) OR 0.06 (m/s) A1


6

© UCLES 2006
Page 5 Mark Scheme Syllabus Paper
IGCSE - OCT/NOV 2006 0625 2

8 (a) (i) (group) 1 B1

(ii) (group) 2 B1

(iii) plastics OR glass OR ebonite B1

(b) top – and bottom + B1

(c) region/area/space etc. B1

charge B1

experiences a force B1
7

9 (a) good straight line through first 5 points, drawn with a rule B1

(b) intelligent attempt at a reason B1

(c) 67 – 40 C1

27 (mm) A1

(d) 2.4 – 2.5 (N) B1


5

10 (a) less turns on Sy OR more turns on Py B1

(b) voltage OR p.d. OR volts is less B1

(c) V1/V2 = N1/N2 in any form C1


correct substitution C1
12 (V) A1
(d) voltage too high OR bell would be damaged B1
6

11 (a) 92 B1

(b) orbit OR outside nucleus B1

(c) 146 B1

(d) nucleus B1

(e) decreases M1

by 2 A1
6

© UCLES 2006
Page 6 Mark Scheme Syllabus Paper
IGCSE - OCT/NOV 2006 0625 2

12 (a) reverse connections to ammeter or battery B1

(b) current OR amps OR amperes B1

(c) 4 components in series (ignore symbols) B1

4 recognisably correct symbols (ignore connections) B1

(d) (i) voltmeter OR multimeter on volts scale B1

(ii) voltmeter shown connected in parallel with resistor B1

(e) I = V/R in any form C1

correct substitution no ecf from wrong equation C1

0.4 (A) c.a.o. A1

(f) his value of (e) B1

(g) (i) 7.5 Ω ticked B1

(ii) increases current e.c.f. B1


12

© UCLES 2006
Page 3 Mark Scheme Syllabus Paper
IGCSE – May/June 2007 0625 02

TARGET MARK
GRADE

1 (a) seconds hand at 35 s F B1


minutes hand at or just to R of 60 (up to ½ division) C B1

(b) seconds hand at 55 s F B1


minutes hand between 4 and 5 C B1

(c) 4 minutes 20 s F B1
[Total: 5]

2 (a) speed = distance/time in any form OR 4800/12 F C1


400 (s) F A1

(b) straight line up to 12 m/s, 20s ± ½ small square F B1


horizontal line for 400 s (e.c.f. for start point and from (a)) F B1
straight line down to 0 m/s at 500 s F B1

(c) distance = ½ base x height


OR area under graph OR equation of motion F C1
accel. distance = 120 m C A1
decel. distance = 480 m C A1
NOTE: NO MARKS for using (d) and then going back to (c)
total distance = 120 + 4800 + 480 stated C A1

(d) average speed = total distance/total time


OR 5400/500 OR 5400/920 F C1
10.8 (m/s) OR 11 (m/s) c.a.o. F A1
[Total: 11]

3 (a) (i) indication of force at A F M1


upward vertical force OR upward force at rt. angles to card C A1

(ii) largest distance from hinge F B1

(b) when C of M lies outside base (idea of) F C1


when vertical through C of M lies outside base (idea of) C A1

(c) (i) less than F B1

(ii) idea of C of M of box raised OR matchbox less stable C B1


NOT matchbox is taller
[Total: 7]

© UCLES 2007
Page 4 Mark Scheme Syllabus Paper
IGCSE – May/June 2007 0625 02

4 (a) (i) large (bird) F M1

(ii) greater weight/mass/force of gravity/heavier F A1

(b) greater F B1
the same F B1

(c) small (bird) F B1

(d) lost/turned into/decreased (accept turned into KE) F M1


as heat ignore mention of sound C A1
[Total: 7]

5 (a) solid: 2, 3 and 6 ticked -1 each error ( use  +  = 0 for extras) F, C B2


gas: 1, 4 and 5 ticked -1 each error ( use  +  = 0 for extras) F, C B2

(b) molecules breaking free (of surface) NOT turns into a gas F M1
mention of higher energy/faster/mols near surface C A1
[Total: 6]

6 (a) [mark in pairs, use  +  = 0]


temp. decreasing F B1
volume increasing F B1

(b) (i) moved out/backwards/to the R F M1

(ii) idea of raised temp increases pressure,


therefore move piston out to decrease pressure C A1
[Total: 4]

7 (a) (i) (good) conductor OR equiv. NOT conductor of electricity F B1

(ii) poor conductor OR (good) insulator (allow electrical)


OR to stop your hand getting burned/prevent shock F B1

(b) (i) conduction F B1

(ii) any 2 of conduction, convection, radiation ticked F, C B1+B1


(-1 if evaporation ticked)

(c) equal to 40W C B1

[Total: 6]

© UCLES 2007
Page 5 Mark Scheme Syllabus Paper
IGCSE – May/June 2007 0625 02

8 (a) 50 F B1

(b) his (a) x 4 F C1


200 (Hz) e.c.f. F A1

(c) Yes, because it is between 20 – 20,000 Hz or more than 20 Hz C B1


allow e.c.f. from (b) answer must tally with (b)
[Total: 4]

9 (a) (i) series OR potential divider F B1

(ii) 12 (Ω) F B1

(iii) I = V/R in any form F C1


6/his (ii) F C1
0.5 e.c.f. F A1
A OR amp(s) OR ampere(s) OR a F B1

(iv) his (iii) x 10 F C1


5 (V) e.c.f. F A1

(v) his (iv) C B1

(b) (i) 1. 6 (V) C B1


2. 0 (V) C B1

(ii) C or clear mark positioned below A but not lower than bottom of C B1
the word contact”
allow e.c.f. only if 6 and 0 in (i) are reversed
[Total: 12]

10 (a) connect wire across/to millivoltmeter F B1


(any mention of connecting to electricity/battery gets B0 here)

move wire across magnetic field OR move magnet past wire


OR dip magnet into coil made of the wire
(condone connect to battery/electricity here) F B1

(b) millivoltmeter deflects F B1

(c) generator OR transformer OR induction coil


OR coil on a car OR microphone
NOT relay/motor/power station etc F B1

[Total: 4]

© UCLES 2007
Page 6 Mark Scheme Syllabus Paper
IGCSE – May/June 2007 0625 02

11 (a) dot to right of S: horiz. line from end/pole, to right (must not curve) F B1
dot to left of N: horiz. line from end/pole, to left (must not curve) F B1
dot by top LH corner: smooth curve from end/pole, above magnet,
to equivalent point at south end F B1
dot below magnet: smooth curve between N and S F M1
curve leaving & entering side of magnet, not ends C A1

(b) arrow clearly indicating N to S F B1

[Total: 6]

12 (a) (i) 2, 2, 0 (accept blank for 0) F B1

(ii) 0, 0, 1 (accept blank for 0) F B1

(b) protons: 11 F B1
neutrons: 13 C B1
electrons: same as his protons F B1

(c) (i) 0 C B1

(ii) -1 C B1

(iii) β OR electron OR e OR B OR beta C B1


NOT b
[Total: 8]

© UCLES 2007
Page 3 Mark Scheme Syllabus Paper
IGCSE – October/November 2007 0625 02

QU. SCHEME MARK

1 (a) 60 (cm3) B1

(b) liquid surface lower than in cylinder C1


liquid surface level with 15 cm3 (± 5 cm3) A1

(c) less B1

[Total: 4]

2 (a) 200,000 (m3) B1

(b) D = M/V in any form B1


his (a) x 1.3 C1
260,000 c.a.o. A1
kg B1

(c) decreases M1
air expands OR density decreases A1

(d) hot air rises B1

[Total: 8]

3 (a) 7.5 (cmHg) C1


75 (cmHg) A1
(give C1, A1 for 1.0006 x 105 Pa or 1 x 105 Pa (N/m2) if unit given)

(b) nothing OR (Torricellian) vacuum OR Hg vapour B1

(c) tube level lower B1


reservoir level higher (any amount) B1

(d) pressures on 2 surfaces equal (always) )


Hg levels equal (always) OR no Hg column ) any two B1+B1
no change when pressure changes )

[Total: 7]

© UCLES 2007
Page 4 Mark Scheme Syllabus Paper
IGCSE – October/November 2007 0625 02

QU. SCHEME MARK

4 (a) (i) arrow labelled W, vertically (by eye) down


from somewhere on either boat B1

(ii) arrow labelled F, down slope, between either boat and slipway B1

(b) (i) multiply W by (vertical) height raised OR Wh B1

(ii) multiply F by distance along slope OR Fs B1

(iii) add (i) and (ii) B1

(c) time taken B1

[Total: 6]

o
5 (a) C B1

(b) (i) ICE marked at 0 B1

(ii) STEAM marked at 100 B1

(c) expansion/volume/pressure OF a gas )


expansion/length OF a solid )
resistance OF a resistor/thermistor/wire )
bending OF a bimetal strip ) any 2 B1+B1
e.m.f/voltage OF a thermocouple )
colour OF a hot surface )
colour change OF certain chemicals )

[Total: 5]

6 (a) (i) uniform acceleration B1

(ii) 9 (m/s) B1

(iii) s = vt in any form C1


90 (m) OR 10 x his (ii), evaluated A1

(b) average speed is lower B1

[Total: 5]

© UCLES 2007
Page 5 Mark Scheme Syllabus Paper
IGCSE – October/November 2007 0625 02

QU. SCHEME MARK

7 (a) (i) 1.5 (cm) B1

(ii) circle centred on X, outside printed circle (circle need not be drawn
with a compass, but must be carefully drawn) M1
diameter 4.5 cm by eye A1

(b) sound longitudinal, water transverse )


sound wave faster (than water wave) ) any 2 B1,B1
different frequency/wavelength )

[Total: 5]

8 (a) (i) principal focus unambiguously marked B1


focal length approximately indicated C1
focal length precisely indicated, from pole to principal focus A1

(ii) any ray from X to Y, correctly refracted at lens B1

(b) [mark in pairs, using  +  = 0]


real B1
diminished B1
inverted B1
image distance less B1

(c) gets smaller B1


gets closer to lens B1

[Total: 10]

9 (a) points correctly plotted (±½ small square) –1 e.e.o.o. B2


smooth curve through his points B1
reasonable thickness B1

(b) (i) 5.3 – 6.1 B1

(ii) 0.9 – 1.7 B1

(c) R = V/I in any form C1


division by 25 or 25 x 10–3 somewhere C1

(i) answer between 220 and 240 B1

(ii) answer between 40 and 60 B1


Ω shown in either (i) or (ii) B1

(d) answer compatible with his (c) B1

[Total: 12]

© UCLES 2007
Page 6 Mark Scheme Syllabus Paper
IGCSE – October/November 2007 0625 02

QU. SCHEME MARK

10 (a) (i) shape appropriate outside coil (condone incomplete loops) M1


lines mostly parallel within coil A1
pattern roughly symmetrical A1
no lines touching or crossing A1

(ii) iron bar B1

(b) rods become magnetised M1


same direction A1
repel B1

[Total: 8]

11 (a) within range 18–20 (mins) B1

(b) (i) 922 or thereabouts B1

(ii) his (a) B1

(c) alpha OR beta B1

[Total: 4]

12 (a) electrons B1

(b) move M1
towards P1 A1

(c) idea of making both P3 and/or P4 positive B1


equal potential B1
(earthing of P1 and P2 not required for answer)

(d) fluorescent screen OR any other appropriate method B1

[Total: 6]

© UCLES 2007
Page 3 Mark Scheme Syllabus Paper
IGCSE – May/June 2008 0625 02

1 (a) (i) 9.2 ± 0.2 (cm) [B1]

(ii) Centre of mass at centre of rod anywhere between a line vertically above the ‘i’
and the and a line vertically above the left hand ‘1’ in ‘Fig. 1.1’, anywhere across
diameter including the surface but NOT outside the surface [B1]

(b) Centre of mass clearly to left of centre, inside the rod [M1]

anywhere between a line vertically above the ‘t’ in ‘to’ and a line vertically above the ‘t’
in the first ‘the’ AND on axis (by eye) [A1]

[Total: 4]

2 (a) (i) suitable scale, probably 5 small squares = 10 s, no awkward ratios [B1]

(ii) (if no scale written on graph, assume our scale)


straight line from origin [B1]
reaching 25 m/s after 10 s NOT horizontal from (0,25) – (10,25) [B1]
horizontal from 10 – 50 s [B1]
straight line down from end of his horizontal line [B1]
reaching axis at 70 s [B1]

(b) average speed = total distance/total time [C1]


1375/his 70 [C1]
19.64… e.c.f. any number of sig. figs [C1]
20 (m/s) e.c.f. [A1]

[Total: 10]

3 (a) clockwise: F3 [B1]

anticlockwise: F1 [B1]
F2 [B1]

(b) c [M1]
clockwise moment (accept moment on RH side) was too big [A1]
reduce moment by reducing distance [A1]
note: moment must be mentioned in both of the last 2 marks; accept turning effect,
torque and leverage as alternatives to moment

(c) any value bigger than 29 g and less than 30 g, but NOT 29 g or 30 g [B1]

[Total: 7]

© UCLES 2008
Page 4 Mark Scheme Syllabus Paper
IGCSE – May/June 2008 0625 02

4 (a) (i) P.E. (however expressed)


(e.g. GPE, gravitational, gravity, potential, positional) [B1]

(ii) chemical [B1]

(b) electrician AND because he is heavier/greater force/greater weight/greater gravity


force/ greater mass [B1]

(c) time AND either work done OR energy used OR equivalent


OR weight AND velocity/speed [B1]

[Total: 4]

5 (a) nucleus OR nuclei OR α-particle NOT nucleon or nuclide [B1]

(b) electron(s) OR e allow β-particle [B1]

(c) neutron(s) OR n [B1]


proton(s) OR p [B1]

(d) alpha OR α NOT a or A [B1]

(e) electron(s) OR e allow β-particles [B1]

[Total: 6]

6 (condone rays not drawn with a ruler, if reasonably straight)

(a) straight ray through centre of lens (±1 mm on axis by eye) (ignore any arrows) [B1]

(b) (i) ray correct, either through pole or


reasonably parallel to axis and then through F1 (±1 mm in either case)
NOTE: any refraction must be at centre line or at both surfaces [B1]

(ii) (condone image not labelled if it is clear where it is; condone image labelled as
‘object’ if image line clearly drawn)
image located at his intersection, even if intersection of incorrect rays [C1]
image drawn between axis and his intersection, and not beyond either [A1]

(c) clear indication of screen at candidate’s image, using vertical line [B1]

[Total: 5]

© UCLES 2008
Page 5 Mark Scheme Syllabus Paper
IGCSE – May/June 2008 0625 02

7 (a) gas )
solid ) any 1 correct [B1]
liquid ) remaining 2 both correct [B1]

i.e. gas, solid, liquid: 2 marks gas, liquid, solid: 1 mark liquid, solid, gas: 1 mark
liquid, gas, solid: 0 marks solid, liquid, gas: 0 marks solid, gas, liquid: 1 mark

(b) (i) liquid [B1]

(ii) idea that molecules/particles gain energy OR move faster (condone ‘vibrating’) [B1]
idea of molecules/particles becoming gaseous/breaking free [B1]

(iii) boiling, at one temperature only AND evaporation at any temperature [B1]
boiling throughout liquid AND evaporation at surface only [B1]

(c) (i) solid [B1]

(ii) 660 (°C) allow 659 (°C) NOT –660 (°C) [B1]

[Total: 9]

8 (a) (i) 1 ice point OR freezing point of water OR melting point of ice NOT just ‘freezing
point’ [B1]
2 ice OR freezing water [B1]
pure or melting or ice-water mix [B1]
3 0 (°C) OR 273 K OR 273 °K [B1]

(ii) 1 steam point OR boiling point of water


NOT just ‘boiling point’ [B1]
2 steam [B1]
boiling (water) OR standard pressure [B1]
3 100 (°C) OR 373 K OR 373 °K [B1]
°C OR K OR °K used in either of the parts 3 [B1]

(b) thermal capacity OR heat capacity, allow specific heat capacity [B1]

[Total: 10]

© UCLES 2008
Page 6 Mark Scheme Syllabus Paper
IGCSE – May/June 2008 0625 02

9 (a) correct symbol [B1]

(b) D A C B all 4 in correct order (allow B1 for any 2 in correct place) [B2]

(c) (note: mark 1 and 2 together) (1 mark max from any one line below)
too great a current might flow )
fuse might not melt NOT fuse won’t work ) any 2 [B1,B1]
fuse won’t protect OR appliance might be damaged )
wiring might overheat/melt or equivalent )
fire might be caused )
NOT circuit broken, NOT short circuit, NOT electric shock
[Total: 5]

10 (a) R1 + R2 in symbols or figures [C1]


60 (Ω) [A1]

(b) voltmeter correctly shown between X and Y (or equivalent), must be correct symbol [B1]

(c) (i) I = V/R [C1]


1.5/60 e.c.f from (a) [C1]
0.025 [A1]
A OR a OR amp(s) OR ampere(s) OR mA etc. [B1]

(ii) 1.5 (V) [B1]

(d) (i) decreases [B1]

(ii) decreases [B1]

(iii) 60 (Ω) e.c.f from (a) [B1]

[Total: 11]

11 (a) (i) no current in circuit OR no voltage in circuit [B1]


e.m.f. induced in AB is cancelled by e.m.f. induced in BC [B1]

(ii) idea of straightening out ABC OR rotate ABC (on its axis)
OR connect G across AB or CB [B1]

(b) any valid answer


e.g. transformer, induction coil, generator, dynamo, microphone, alternator, computer
NOT motor, relay
(use right + wrong = 0 for incorrect extras) [B1]

[Total: 4]

© UCLES 2008
Page 7 Mark Scheme Syllabus Paper
IGCSE – May/June 2008 0625 02

12 (a) anything less than, or equal to, 30 min [C1]


between 22 and 27 min, inclusive [A1]

(b) (i) iodine(-128) OR the second one [B1]

(ii) radon-220 OR the first one [M1]


NOTE: NOT radon-222
NOT just radon, unless mention of 55 s in ‘why’ section

shortest half-life OR decays most rapidly OR takes least time to decay


NOT ‘because it only has a half-life of 55 s’ [A1]
[Total: 5]

© UCLES 2008
Page 3 Mark Scheme Syllabus Paper
IGCSE – October/November 2008 0625 02

1 (a) attempt at subtraction C1


3 hrs 20 mins C1
200 mins A1

(b) 200/20 OR 200/10 e.c.f. from (a) C1


10 OR 20 e.c.f. A1 [5]

2 (a) L×B×H C1
2 × 1.5 × 1 C1
3 (m3) A1

(b) M=V×D C1
3000 (kg) e.c.f. A1
correct units in both (a) and (b) B1 [6]

3 (a) YZ OR WZ and WY OR XZ and XY B1

(b) (i) 4.2 – 4.6 (N) B1

(ii) his (i) B1


N B1

(iii) increases B1

(iv) goes up OR oscillates OR accelerates B1 [6]

4 (a) radio OR TV B1
ultra-violet B1

(b) long wavelength written at LH end B1

(c) any three acceptable uses B1,B1,B1

(d) red B1
violet (NOT blue, purple, mauve) B1 [8]

© UCLES 2008
Page 4 Mark Scheme Syllabus Paper
IGCSE – October/November 2008 0625 02

5 (a) (i) reflected B1

(ii) same frequency B1

(b) (i) 480 (m) B1

(ii) distance/time C1
his(i)/1.6 C1
300 (m/s) e.c.f. A1

(iii) measured distance incorrect )


measured time incorrect OR effect of reaction time ) any 2 B1,B1
any reference to temperature )
any reference to wind )
any reference to humidity )
any reference to pressure )
time interval very small to measure on a stopwatch )
[8]

6 (a) distance approximately marked (±5 mm) C1


distance accurately marked (±2 mm) A1

(b) (i) falls B1

(ii) air mols bombard surfaces B1


cause pressure on Hg B1

(c) Hg at same height as in LH tube (by eye) B1

(d) rises rises ) B1 × 4


falls stays the same )
[10]

7 (a) (i) normal correct, by eye B1

(ii) reflected ray in correct direction, by eye (condone poor normal) B1

(iii) i correctly labelled (condone poor normal) B1

(iv) r correctly labelled (condone poor normal) B1

(b) i=r B1

(c) (i) second mirror correct, by eye B1

(ii) ray parallel to original ray B1 [7]

© UCLES 2008
Page 5 Mark Scheme Syllabus Paper
IGCSE – October/November 2008 0625 02

8 (a) (i) N at left and S at right B1

(ii) attract e.c.f. B1

(b) (i) N at left and S at right B1

(ii) attract e.c.f. B1

(c) attract B1

(d) nothing B1 [6]

9 (a) a B1

(b) B B1

(c) I = V/R C1
6/4 A1
1.5 A1
A OR amp(s) OR ampere(s) B1

(d) connection 2 B1
both receive full voltage OR both run at full brightness B1
if one blows/switched off, other will continue working B1 [9]

10 (a) contact position shown at centre of potential divider, by eye B1

(b) current in coil )


core magnetised )
armature attracted ) any 3 B1,B1,B1
armature pivots )
armature pushes contacts closed )
[4]

© UCLES 2008
Page 6 Mark Scheme Syllabus Paper
IGCSE – October/November 2008 0625 02

11 (a) core B1

(b) a.c. B1

(c) VP/VS = NP/NS in any form C1


correct substitution C1
200 A1

(d) voltage too high OR lamp will blow B1 [6]

12 (a) 84 B1

(b) 218 B1

(c) (i) 2 B1

(ii) 4 B1
allow both marks for interchanged answers for (i) and (ii),
if answers to (a) and (b) were interchanged

(iii) alpha-particle B1 [5]

© UCLES 2008
Page 3 Mark Scheme: Teachers’ version Syllabus Paper
IGCSE – May/June 2009 0625 02

1 (a) 35 B1

(b) vehicles/time in any form, letters words or numbers C1


700/35 e.c.f. (a) C1
20 e.c.f. (a) A1 [4]

2 work force of gravity on a body

how big the body is

mass power of a given force

weight ÷ mass

weight amount of matter in a body

force × distance moved

density mass ÷ volume

the acceleration due to gravity B1×3 [3]

3 (a) 1500 B1

(b) second box ticked (use  +  = 0 for extras) B1

(c) constant speed B1

(d) award B1 from each of any 2 lines:

increased wind/air resistance OR headwind OR roof rack )


rough(er) ground OR flat tyre OR increased road resistance/friction ) B1 + B1
brakes applied )

IGNORE increased speed/changed car shape/increased load


IGNORE driver decided to stop [5]

4 (a) 88 – 92 B1

(b) his (a) B1

(c) 840 e.c.f. (b) B1

(d) left level up and right level down B1


L at 80 and R at 150 B1 [5]

© UCLES 2009
Page 4 Mark Scheme: Teachers’ version Syllabus Paper
IGCSE – May/June 2009 0625 02

5 (a) (i) rapid/rapid heat transfer/gain OR rapid reading/response


NOT sensitivity/temperature transfer B1

(ii) strength OR reduce chance of breaking OR to magnify the thread


ignore any mention of safety B1

(iii) sensitivity or equiv. (e.g. idea of large movement of thread) B1

(b) mercury OR alcohol B1

(c) 0 and 100 B1


°C on at least 1 temperature B1 [6]

6 (a) (i) decreasing OR getting lower/quieter/softer M1

(ii) amplitude/length of wave decreased OR waves got smaller


NOT wavelength decreased A1

(b) (i) nothing OR constant M1

(ii) waves equally spaced OR wavelength/period/T constant A1

(c) (i) 12 – 14 B1

(ii) 1. 300 (waves, oscillations, vibrations) every second B1


2. 1/300 (s) OR 0.0033 OR 0.003 with indication of recurring 3 B1
3. 1/his300 × his 12 OR his (1/300) C1
0.04 (s) e.c.f. A1

(d) (i) yes/ )


)

(ii) yes/ ) –1 e.e.o.o. B2


)

(iii) no/ ) [11]

© UCLES 2009
Page 5 Mark Scheme: Teachers’ version Syllabus Paper
IGCSE – May/June 2009 0625 02

7 (a) (i) 1. normal correct, by eye B1


2. i correctly labelled B1

(ii) i = r in any recognisable form accept incidence = refraction


NOT sin i = sin r B0 for refraction, refrection, reflaction B1

(iii) V B1

(iv) none B1

(b) be reasonably generous:


correct inversion M1
stem approx. parallel to card edge A1 [7]

8 (a) (i) iron (rod) B1

(ii) plastic (rod) B1

(b) S S N B1

(c) – somewhere on or near rod D, near end C


condone extra + or – signs unless contradict B1

(d) one needle pointing N, by eye C1


both needles pointing N, by eye A1 [6]

© UCLES 2009
Page 6 Mark Scheme: Teachers’ version Syllabus Paper
IGCSE – May/June 2009 0625 02

9 Apply max 1 un. pen. in (a) and (b) together. Apply at first instance of unit penalty.

(a) (i) 6 V B1

(ii) 50 mA OR 0.05 A B1

(b) R = V/I in any form, letters, words, numbers OR V/I C1


6/50 OR 6/0.05 e.c.f. (a) OR 0.12 Ω (0.12 Ω gets 2, 0.12 gets 1) C1
120 Ω c.a.o. accept V/A instead of Ω A1

(c) (i) increase resistance/ohms OR add another resistor


OR decrease e.m.f./voltage/p.d. C1
double resistance/ohms OR add another R (in series)
OR halve e.m.f./voltage/p.d. OR use 3 V cell/battery
OR remove one cell/battery OR use only 1 cell/battery A1

(ii) idea of breaking the circuit OR removing battery


OR make voltage zero OR switch off B1

(d) (i) infinite OR very large (if figure quoted, must be [25 A)
NOT just “higher” B1

(ii) idea of damage but NOT “blows up” C1


ammeter – coil burnt out OR pointer bent )
battery – overheats OR runs flat quickly ) any 1 A1
circuit – overheat/burn out/insulation melts )
NOT it trips out [11]

10 (a) XY would move up/anticlockwise/motion reversed/pan moves down B1

(b) (i) 1. sensible choice of F scale ) B1


) both lost if scales reversed
sensible choice of I scale ) B1
2. 4 points correctly plotted (± ½ small square) –1 e.e.o.o. B2
– B0 if ridiculous scale on either axis (e.g. non-linear, 3, 7, 9 etc.)
– can award both marks if scales interchanged but otherwise O.K.
– if any blob clearly >1 square diameter, then –1 for each (max 2)
3. reasonable straight line through his points, including 0,0 B1

(ii) 0.036 – 0.038 OR his correct value ± 0.0005 (B0 if ridiculous scale) B1

(c) (electric) motor OR ammeter OR galvanometer OR voltmeter


NOT generator/electronic balance B1 [8]

© UCLES 2009
Page 7 Mark Scheme: Teachers’ version Syllabus Paper
IGCSE – May/June 2009 0625 02

11 For (a), (b) and (c), mark the names, not the box

(a) CATHODE in bottom left box B1

(b) UP & DOWN in top middle box B1

(c) GLOWS in bottom right box B1

(d) battery shown connected across heater filament, any recognisable symbol
ignore extra wires if it would work B1

(e) electrons NOT beta particles NOT positive electron B1

(f) vacuum ticked (use  +  = 0 for extras) B1 [6]

12 (1) electron(s) electromagnetic radiation/waves/rays B1 + B1


OR e (ignore any prefix or suffix) NOT just rays etc.

~ 8000 units OR very large zero/nothing B1 + B1


NOT small/almost nothing
NOT – (dash)

negative allow – (dash) no charge OR zero/neutral B1 + B1


NOT negligible
NOT – (dash)

idea of not very (penetrating) idea of extremely (penetrating) B1 + B1


OR stopped (but if a substance is OR not stopped (but if a substance is
mentioned, it must be appropriate, mentioned, it must be appropriate)
not air)
NOT “not penetrating” NOT very/strongly/highly penetrating
NOT slowly penetrating NOT very fast penetrating [8]

© UCLES 2009
Page 3 Mark Scheme: Teachers’ version Syllabus Paper
IGCSE – October/November 2009 0625 02

1 (a) (i) 20 (cm3) )

(ii) 25 (cm3)±0.5 ) both B1 [1]

(b) 5 (cm3) e.c.f. B1 [1]

(c) 5/200 e.c.f. C1


0.025 (cm3) e.c.f. A1 [2]

[Total: 4]

2 (a) kinetic or K.E. or motion B1 [1]

(b) strain or elastic B1 [1]

(c) gravitational or P.E. or G.P.E. or potential B1 [1]

(d) weight /mass (of athlete) AND height/distance (of bar) B1 [1]

[Total: 4]

3 (a) (i) 1. increasing M1


steady or uniform A1
2. constant B1 [3]

(ii) horizontal straight line between A & B B1 [1]

(b) (i) line on axis between B & C B1 [1]

(ii) horizontal straight line between C & D M1


lower than that for AB A1 [2]

(c) zero distance or equiv. B1 [1]

[Total: 8]

4 (a) (i) moves to the left C1


accelerates to the left A1 [2]

(ii) arrow to the right B1


9N B1 [2]

(iii) blob on diagram clearly indicated as the C of M B1 [1]

© UCLES 2009
Page 4 Mark Scheme: Teachers’ version Syllabus Paper
IGCSE – October/November 2009 0625 02

(b) (i) rises B1 [1]

(ii) less stable B1 [1]

[Total: 7]

5 (a) (i) 1. cooling OR energy/heat lost seen anywhere in (i) B1


2. solidifying or temperature constant B1
3. cooling B1 [3]

(ii) first and last both ticked B1


middle ticked B1 [2]

(iii) solid accept ice/frozen B1 [1]

(b) rising curve of some sort C1


idea of mirror image of Fig. 6.1 A1 [2]

[Total: 8]

6 (a) same B1
greater at B B1
greater at B B1 [3]

(b) box 1 ticked ) B1


) use  +  =0 for extras
box 3 ticked ) B1 [2]

[Total: 5]

7 (a) q B1 [1]

(b) F marked close to point of image/object B1 [1]

(c) [mark in pairs, use  +  =0]


inverted B1
real B1 [2]

(d) same B1 [1]

(e) (i) nothing C1 [1]

(ii) image blurs A1 [1]

[Total: 7]

© UCLES 2009
Page 5 Mark Scheme: Teachers’ version Syllabus Paper
IGCSE – October/November 2009 0625 02

8 (a) one sound direct B1


one sound after reflection/echo B1 [2]

(b) first M1
second one suffers absorption,dispersion A1 [2]

(c) (i) s = vt in any form (seen somewhere in (c)) B1


time to hear 1st sound = 990/330 or 3 (s) B1 [2]

(ii) time to hear 2nd sound = (3 × 330)/330 or 9 (s) B1 [1]

(iii) interval = 6 (s) e.c.f. B1 [1]

[Total: 8]

9 (a) L.H. circuit – series AND R.H. circuit – parallel B1 [1]

(b) (i) 280 + 200 C1


480 (Ω) A1 [2]

(ii) I = V/R in any form C1


12/his (i) seen or 12/480 need not be seen C1
0.025 or 25 or 1/40 c.a.o. A1
A or mA as appropriate B1 [4]

(iii) his (ii) × 200 or proportion or potential divider calculation C1


5 (V) e.c.f. A1 [2]

(iv) connect voltmeter ) M1


) (could be shown on diag)
between A and B ) A1 [2]

[Total: 11]

10 (a) (i) core correctly labelled B1 [1]

(ii) iron B1 [1]

(iii) idea of magnetic linkage B1 [1]

(b) V1/V2 = N1/N2 in any form C1


correct substitution C1
120 (V) A1 [3]

[Total: 6]

© UCLES 2009
Page 6 Mark Scheme: Teachers’ version Syllabus Paper
IGCSE – October/November 2009 0625 02

11 no exposed wires )
)
no worn insulation )
)
no loose wires/connections )
)
no short circuits )
) any 3
plug correctly wired )
)
any idea about continuity check )
)
no sharp bends in cable ) B1 x 3 [3]

[Total: 3]

12 (a) 5 points correctly plotted (–1 e.e.o.o.) B2


reasonable curve through his points B1 [3]

(b) (i) between 30 and 35 or his correct value ± 5 B1 [1]

(ii) 2 (minutes) or his correct value ± 0.02 B1 [1]

(c) 2 (minutes) or his (b) (ii) B1 [1]

(d) (i) half-life too short B1 [1]

(ii) mark any correct 2, ignore the rest

long half life )


)
gamma-emitter )
)
good penetration ) any 2
)
similar particle size )
)
similar density ) B1+B1 [2]

[Total: 9]

© UCLES 2009
Page 3 Mark Scheme: Teachers’ version Syllabus Paper
IGCSE – May/June 2010 0625 21

1 (a) distance tape measure, trundle wheel, metre wheel B1, B1


OR laser measure NOT (metre) rule

time stopwatch/clock IGNORE just watch/clock


IGNORE just chronometer B1, B1

(b) speed = distance/time any arrangement, words or symbols B1


OR just distance/time IGNORE magic triangles

(c) (i) idea of acceleration/deceleration


OR some distance at lower speed/lorry stops B1

(ii) distance = speed × time in this form only, words, letters or numbers C1

66 × 20 OR 66 × ⅓ OR 66 × 20/60 C1

22 (km) c.a.o. condone 0.33 used to give appropriate answer A1


[9]

2 (a) 62.8 – 29.8 C1


33.0 (cm) OR 33 (cm) A1

(b) (i) 5.5 = constant × 33 e.c.f. C1

0.166 recurring e.c.f. ignore units


accept 1/6 or 0.16 or 0.166 or 0.167 or 0.17 or 0.2 NOT 0.20 A1

(ii) N/cm OR N/m OR n/cm OR n/m seen in (ii)


nothing else – mark independently of (i) B1
[5]

3 (a) I = U + W accept words or mixture of words/symbols B1

(b) (i) 850 (N) B1

(ii) force needed to accelerate load/get it started


OR if forces equal, then no movement B1

(iii) height OR distance (use  +  = 0 for extras) B1

(iv) time (use  +  = 0 for extras) B1

(c) greater than OR > OR stronger accept “double” etc B1


[6]

© UCLES 2010
Page 4 Mark Scheme: Teachers’ version Syllabus Paper
IGCSE – May/June 2010 0625 21

4 (a) (i) 1 nothing OR no change B1


2 quieter/softer OR loudness less/decreases B1

(ii) frequency control: none OR no adjustment no e.c.f. B1


amplitude control: increase (amplitude) no e.c.f.
allow turn clockwise/to right B1

(b) (i) echo OR reflection (of sound) OR bounced (back) B1

(ii) idea of sound taking a finite time to travel


OR idea of sound doesn’t travel infinitely fast
IGNORE sound has to travel to rock face and back B1
[6]

5 (a) X marked anywhere, above or below, on vertical anywhere through rod B1

(b) Y marked anywhere to right of X, but not beyond R.H. tip of parrot B1

(c) idea of topples/falls/loses balance C1


topples clockwise/to the right/to the front/forwards A1
[4]

6 (a) (i) radiation


evaporation any 2 B1, B1
convection

(ii) cardboard/it is a poor conductor/(good) insulator


air is trapped OR air is a poor conductor/(good) insulator any 2 B1, B1
reduced surface in contact with fingers

(b) (i) heat/energy to raise/lower/change temperature of a body


OR heat/energy to heat up a body B1
by 1 °C OR by 1K OR unit temp B1

(ii) low thermal capacity M1


less heat needed to raise temperature OR absorbs less heat A1
[8]

© UCLES 2010
Page 5 Mark Scheme: Teachers’ version Syllabus Paper
IGCSE – May/June 2010 0625 21

7 (a) (i) idea of heat concentrated in a small space


OR lots of wire in small space
OR to get required resistance in a small place B1

(ii) radiation B1

(b) (i) mark 1 and 2 together


240 and 100 in correct order B1
V and W in correct order B1

(ii) I = V/R OR I = W/V in any form, symbols or numbers C1


240/576 OR 100/240 C1
0.416 recurring,
accept 0.4 or 0.416 or 0.417 or 0.41 or 0.42 NOT 0.40 C1
A OR a OR amp(s) OR ampere(s) A1
[8]

8 (a) 10 (cm) B1

(b) gets smaller NOT gets lower B1


gets closer to lens/moves to left/moves closer to F1 B1

(c) (i) principal focus/foci OR focal/focus point(s)


NOT focal length NOT focus B1

(d) (ii) (ignore any arrows)

ray drawn from top of object, through F2, to lens


must pass through the stroke indicating F2 B1
single refraction clearly at centre line
OR two appropriate refractions at surfaces B1
travels parallel to axis after lens, by eye must be drawn with ruler B1
reaches top of image B1
[8]

9 (a) (i) water conducts/water lowers resistance B1


could get a shock (however expressed) B1

(ii) idea of cord insulating you from electricity OR cord not a conductor
OR idea of separates you from the electrics/live parts B1

(b) 10A ticked B1

(c) (i) large(r) current NOT more electricity B1

(ii) it/insulation/cable would overheat/melt OR cause fire


NOT blow up/damaged NOT fuse blows B1
[6]

© UCLES 2010
Page 6 Mark Scheme: Teachers’ version Syllabus Paper
IGCSE – May/June 2010 0625 21

10 (a) V1/V2 or N1/N2 or V1/N1 or V2/N2 in any form Allow full credit for use C1
substitution correct and seen of 25 turns to give 12V, M1
25 turns with working seen
A1
Y and Z (either order) B1

(b) 240 (V) B1

(c) core B1
iron NOT steel B1

(d) good conductor OR low resistance OR to reduce heating


OR for high efficiency IGNORE good/bad conductor of heat B1
[8]

11 (a) refraction OR slows down OR changes speed/wavelength OR bends


NOT reflaction or refrection B1
dispersion OR divides/splits into colours/wavelengths/frequencies B1

(b) (i) red If red and violet interchanged, B1


allow B1 only
(ii) violet NOT blue NOT purple B1

(c) (i) X at or above top of visible spectrum M1


middle of X clearly above top of visible spectrum but no more than
twice height of the letter A from top of visible spectrum, by eye A1

(ii) infra-red OR IR OR ir OR heat/thermal (radiation) B1


[7]

12 (a) (i) beta, gamma –1 e.e.o.o. B2

(ii) idea that radiation (from watch) can enter the body B1

(b) (i) bottom left box ticked –1 e.e.o.o. B1

(ii) locked cupboard OR lock (it)


OR storage in lead/suitable containers
IGNORE protective clothing/tongs etc B1
[5]

© UCLES 2010
w
w
w
.X
tr
me
eP
UNIVERSITY OF CAMBRIDGE INTERNATIONAL EXAMINATIONS

ap
er
International General Certificate of Secondary Education

s.c
om
MARK SCHEME for the October/November 2010 question paper
for the guidance of teachers

0625 PHYSICS
0625/21 Paper 2 (Core Theory), maximum raw mark 80

This mark scheme is published as an aid to teachers and candidates, to indicate the requirements of
the examination. It shows the basis on which Examiners were instructed to award marks. It does not
indicate the details of the discussions that took place at an Examiners’ meeting before marking began,
which would have considered the acceptability of alternative answers.

Mark schemes must be read in conjunction with the question papers and the report on the
examination.

• CIE will not enter into discussions or correspondence in connection with these mark schemes.

CIE is publishing the mark schemes for the October/November 2010 question papers for most IGCSE,
GCE Advanced Level and Advanced Subsidiary Level syllabuses and some Ordinary Level
syllabuses.
Page 2 Mark Scheme: Teachers’ version Syllabus Paper
IGCSE – October/November 2010 0625 21

NOTES ABOUT MARK SCHEME SYMBOLS & OTHER MATTERS

B marks are independent marks, which do not depend on any other marks. For a B mark to be
scored, the point to which it refers must actually be seen in the candidate's answer.

M marks are method marks upon which accuracy marks (A marks) later depend. For an M mark
to be scored, the point to which it refers must be seen in a candidate's answer. If a
candidate fails to score a particular M mark, then none of the dependent A marks can be
scored.

C marks are compensatory method marks which can be scored even if the points to which they
refer are not written down by the candidate, provided subsequent working gives
evidence that they must have known it. e.g. if an equation carries a C mark and the
candidate does not write down the actual equation but does correct working which
shows he knew the equation, then the C mark is scored.

A marks are accuracy or answer marks which either depend on an M mark, or which are one of
the ways which allow a C mark to be scored.

c.a.o. means "correct answer only".

e.c.f. means "error carried forward". This indicates that if a candidate has made an earlier
mistake and has carried his incorrect value forward to subsequent stages of working, he
may be given marks indicated by e.c.f. provided his subsequent working is correct,
bearing in mind his earlier mistake. This prevents a candidate being penalised more
than once for a particular mistake, but only applies to marks annotated "e.c.f."

e.e.o.o. means "each error or omission".

brackets ( ) around words or units in the mark scheme are intended to indicate wording used to
clarify the mark scheme, but the marks do not depend on seeing the words or units in
brackets.
e.g. 10 (J) means that the mark is scored for 10, regardless of the unit given.

underlining indicates that this must be seen in the answer offered, or something very similar.

un.pen. means "unit penalty". An otherwise correct answer will have one mark deducted if the
unit is wrong or missing. This only applies where specifically stated in the mark
scheme. Elsewhere, incorrect or missing units are condoned.

OR/or indicates alternative answers, any one of which is satisfactory for scoring the marks.

Spelling Be generous about spelling and use of English. If an answer can be understood to
mean what we want, give credit.

Significant Answers are acceptable to any number of significant figures ≥ 2, except if specified
figures otherwise, or if only 1 sig. fig. is appropriate.

Units Ignore units, except where a mark is specified for a particular unit.

Fractions These are only acceptable where specified.

Extras Ignore extras in answers if they are irrelevant; if they contradict an otherwise correct
response or are forbidden by mark scheme, use right + wrong = 0

Work which has been crossed out, but not replaced, should be marked as if it had not been crossed out.

© UCLES 2010
Page 3 Mark Scheme: Teachers’ version Syllabus Paper
IGCSE – October/November 2010 0625 21

1 (a) (i) 6 (cm) B1


5 (cm) B1

(ii) 6 × 5 × 2 ecf C1
60 (cm3) ecf A1

(b) D = M/V in any form, letters, words or numbers B1


53 C1
2.65 OR 2650 A1
g/cm3 OR kg/m3 (unit must be appropriate) B1

[Total: 8]

2 (a) distance/time in any form C1


960/8 OR 960/(8 × 60) C1
120 OR 2 A1
m/min OR m/s must correspond with value B1

(b) friction or air resistance or force accelerating/decelerating legs B1

[Total: 5]

3 (a) tidal B1
wave B1
hydroelectric accept waterfall B1
(any order)

(b) tidal wave hydroelectric


PE of rise and fall PE of rise and fall water stored at high level B1
flow through turbine rotates/moves floats flowing water drives turbine B1
turbine drives generator floats drive generator turbine drives generator B1

[Total: 6]

4 (a) focal length OR focal distance B1

(b) 4 rays all passing through F M1


appropriate refraction at both lens surfaces
OR all rays bent at lens mid-line A1

(c) focused image OR sharp image OR dot B1

(d) 4 dots OR out-of-focus/blurred/fuzzy image B1

[Total: 5]

© UCLES 2010
Page 4 Mark Scheme: Teachers’ version Syllabus Paper
IGCSE – October/November 2010 0625 21

5 (a) alpha and beta both underlined –1 e.e.o.o. B2

(b) gamma B1

(c) radio B1

(d) alpha B1

[Total: 5]

6 (a) conduction B1

(b) (i) convection B1

(ii) hot water expands OR hot water less dense B1


hot water rises (ignore anything about cold water falling) B1

(c) convection cannot occur B1


water is a poor conductor B1

[Total: 6]

7 (a) i correctly shown B1

(b) (i) ray shown in air at angle > 40° C1


angle same as in Fig. 7.1, by eye A1

(ii) ray reflected (MO if says along surface) M1


critical angle exceeded A1

[Total: 5]

8 (a) (i) one sound or equivalent (NOT an echo) B1

(ii) distance = speed × time in any form ...... condone factor of 2 C1


330 × 1.5 C1
495 (m) A1

(b) (i) idea of one sound direct


OR original sound B1
other sound by echo B1

(ii) 1.5 (s) B1


4.5 (s) B1

[Total: 8]

© UCLES 2010
Page 5 Mark Scheme: Teachers’ version Syllabus Paper
IGCSE – October/November 2010 0625 21

9 (a) (i) N at left end and S at right end (inside or outside magnet outline) M1
both N and S within magnet outline A1

(ii) attracted/moves towards magnet OR it becomes magnetised B1

(iii) nothing B1

(b) (i) pass current through coil/wire OR connect a battery across coil B1

(ii) iron NOT steel B1

(iii) can be very strong )


can be switched on & off easily ) any one B1
can reverse polarity easily )
adjustable strength )

[Total: 7]

10 (a) parallel B1

(b) I = V/R in any form C1


100/250 C1
0.4 (A) A1

(c) 12 (A) OR 30 × his (b), correctly evaluated B1

(d) parallel B1

(e) (i) none e.c.f. from (a) B1

(ii) none e.c.f. from (d) B1

[Total: 8]

11 (a) cell/battery shown M1


complete series circuit, including cell/battery A1
(ignore any switch, open or closed
ignore any other component, as long as a current would flow)

(b) (i) S and M on door and frame (either way) so they would be next to each other when door
closed B1
S on frame and M on door edge/door face close to edge B1

(ii) any suitable application B1


e.g. shop door, security door, lift door, fridge door, oven door

[Total: 5]

© UCLES 2010
Page 6 Mark Scheme: Teachers’ version Syllabus Paper
IGCSE – October/November 2010 0625 21

12 (a) yes B1
yes B1
no B1

(b) nucleus B1

(c) (i) 6 points correct ±½ small square –1 e.e.o.o. B2


thin, smooth curve through points B1

(ii) 8 ± 1 (mins) C1
108 ± 1 (mins) C1
100 ± 2 (mins) e.c.f. if working shown A1

(iii) half his (ii) e.c.f. B1

(d) his (ii) e.c.f. B1

[Total: 12]

© UCLES 2010
w
w
w
.X
tr
me
eP
UNIVERSITY OF CAMBRIDGE INTERNATIONAL EXAMINATIONS

ap
er
International General Certificate of Secondary Education

s.c
om
MARK SCHEME for the May/June 2011 question paper
for the guidance of teachers

0625 PHYSICS
0625/21 Paper 2 (Core Theory), maximum raw mark 80

This mark scheme is published as an aid to teachers and candidates, to indicate the requirements of
the examination. It shows the basis on which Examiners were instructed to award marks. It does not
indicate the details of the discussions that took place at an Examiners’ meeting before marking began,
which would have considered the acceptability of alternative answers.

Mark schemes must be read in conjunction with the question papers and the report on the
examination.

• Cambridge will not enter into discussions or correspondence in connection with these mark schemes.

Cambridge is publishing the mark schemes for the May/June 2011 question papers for most IGCSE,
GCE Advanced Level and Advanced Subsidiary Level syllabuses and some Ordinary Level
syllabuses.
Page 2 Mark Scheme: Teachers’ version Syllabus Paper
IGCSE – May/June 2011 0625 21

NOTES ABOUT MARK SCHEME SYMBOLS & OTHER MATTERS

B marks are independent marks, which do not depend on any other marks. For a B mark to be
scored, the point to which it refers must actually be seen in the candidate's answer.

M marks are method marks upon which accuracy marks (A marks) later depend. For an M mark
to be scored, the point to which it refers must be seen in a candidate's answer. If a
candidate fails to score a particular M mark, then none of the dependent A marks can be
scored.

C marks are compensatory method marks which can be scored even if the points to which they
refer are not written down by the candidate, provided subsequent working gives
evidence that they must have known it. e.g. if an equation carries a C mark and the
candidate does not write down the actual equation but does correct working which
shows he knew the equation, then the C mark is scored.

A marks are accuracy or answer marks which either depend on an M mark, or which are one of
the ways which allow a C mark to be scored.

c.a.o. means "correct answer only".

e.c.f. means "error carried forward". This indicates that if a candidate has made an earlier
mistake and has carried his incorrect value forward to subsequent stages of working, he
may be given marks indicated by e.c.f. provided his subsequent working is correct,
bearing in mind his earlier mistake. This prevents a candidate being penalised more
than once for a particular mistake, but only applies to marks annotated "e.c.f."

e.e.o.o. means "each error or omission".

brackets ( ) around words or units in the mark scheme are intended to indicate wording used to
clarify the mark scheme, but the marks do not depend on seeing the words or units in
brackets. e.g. 10 (J) means that the mark is scored for 10, regardless of the unit given.

underlining indicates that this must be seen in the answer offered, or something very similar.

OR/or indicates alternative answers, any one of which is satisfactory for scoring the marks.

Spelling Be generous about spelling and use of English. If an answer can be understood to
mean what we want, give credit.

Significant Answers are acceptable to any number of significant figures ≥ 2, except if specified
figures otherwise, or if only 1 sig.fig. is appropriate.

Units Incorrect units are not penalised, except where specified. More commonly, marks are
allocated for specific units.

Fractions These are only acceptable where specified.

Extras Ignore extras in answers if they are irrelevant; if they contradict an otherwise correct
response or are forbidden by mark scheme, use right + wrong = 0

Ignore Indicates that something which is not correct is disregarded and does not cause a right
plus wrong penalty.

Not/NOT Indicates that an incorrect answer is not to be disregarded, but cancels another
otherwise correct alternative offered by the candidate i.e. right plus wrong penalty
applies.

© University of Cambridge International Examinations 2011


Page 3 Mark Scheme: Teachers’ version Syllabus Paper
IGCSE – May/June 2011 0625 21

1 (a) 60.4 – 44.2 C1


16.2 (cm3) A1

(b) (density =) mass/volume in any form, letters, words, numbers C1


40.5/16.2 e.c.f. C1
2.5 e.c.f. A1
g / cm3 B1
(accept correct conversion kg / m3, with unit)

(c) 60.4 and 40.5 both ticked –1 e.e.o.o. B2 [8]

2 (a) molecules/particles/atoms moving (accept vibrating/oscillating) C1


molecules colliding (accept with each other) C1
molecules colliding with walls A1

(b) (i) LH graph – temperature/T/θ / °C/K on horizontal axis


RH graph – volume/V / m3/cm3 on horizontal axis M1

(ii) X on LH graph at intersection of line and vertical axis A1 [5]

3 (a) idea that non-renewable sources are finite / get used up B1

(b) (i) solar/sun/sunlight (ignore just light)


wind/éolienne accept windmill
waves (ignore sea)
tidal (ignore sea) any 1 M1
hydro(electric) (ignore water)
geothermal
biomass

(ii) high cost/low effectiveness


small output any 1 (ignore efficiency) A1
environmental impact
cannot be relied upon (wind/solar)

© University of Cambridge International Examinations 2011


Page 4 Mark Scheme: Teachers’ version Syllabus Paper
IGCSE – May/June 2011 0625 21

(c) (i) fossil fuel


coal
oil
petrol
(natural) gas any 1 M1
peat
nuclear
lignite

(ii) plentiful/regular/constant/reliable supply


cheap/cost effective any 1 A1
high output [5]

4 (a) cool air more dense OR cool air falls


OR warm air rises so it can be cooled B1

(b) energy/heat removed from store must be released outside store B1


heat developed by refrigeration unit B1

(c) reduce/prevent heat coming in from outside NOT cold getting out B1
reduce/prevent conduction NOT convection/radiation B1

(d) idea that heat gained from outside = heat removed by refrigeration unit B2
allow B1 for idea of thermostatic control [7]

5 (a) boxes 1 and 4 ticked –1 e.e.o.o. B2

(b) sound/wave reflected/bounces back (from surface) NOT just “returns” B1

(c) (i) cliff A B1

(ii) (s =) vt OR (s =) vt/2 in any form........... allow s = ut +½at2 C1


330 × 1.5 OR 495
OR 330 × 0.75 OR 247.5
OR 330 × 2.5 OR 825
OR 330 × 1.25 OR 412.5 C1
OR 330 × 4 OR 1320
OR 330 × 2
660 (m) A1

(iii) both echoes at the same time OR one echo OR louder B1


time value quoted between 1.5s and 2.5s B1 [9]

© University of Cambridge International Examinations 2011


Page 5 Mark Scheme: Teachers’ version Syllabus Paper
IGCSE – May/June 2011 0625 21

6 (a) ray bent down at 1st surface, but not beyond/along normal B1
ray bent down at 2nd surface, but not beyond/along surface B1
MAX 1 mark if any suggestion of a spectrum shown

(b) spot/dot/line AND of one colour accept a single named colour e.g. red B1

(c) spectrum/colours/light dispersed ignore rainbow C1


red at top and violet at bottom in words in space provided A1 [5]

7 (a) spheres closer together allow touching spheres B1

(b) (i) charging (of anything) by friction/rubbing B1


plastic/furniture (becomes) charged OR electron/charge transfer M1
plastic/furniture attracts dust/fluff A1

(ii) idea of charge leaking B1


water is a conductor B1 [6]

8 (a) (i) parallel B1

(ii) 4.2 (V) B1

(iii) V=IR in any form OR V/R C1


4.2 / 3 e.c.f. (ii) C1
1.4 e.c.f. (ii) A1
A OR amp(s) OR ampere(s) B1

(iv) 1. bigger OR the sum of the two currents OR 2 (A) B1


2. same/equal B1

(b) clear series connection of all 3 across battery in one circuit B1


clear parallel connection of all 3 across battery in other circuit, and must not be
shorted out B1
allow B1 max in (b) if correct series/parallel circuits both shown, but with more or
less than 3 resistors in either/both [10]

© University of Cambridge International Examinations 2011


Page 6 Mark Scheme: Teachers’ version Syllabus Paper
IGCSE – May/June 2011 0625 21

9 (a) all 3 lamps in parallel across battery + switch B2


(–1 if any lamps in series, –1 if connections across battery only)

(b) (i) molecules vibrate over bigger distance OR molecules separate


OR bigger space between molecules
NOT just “molecules need more space”
ignore breaking bonds B1

(ii) 1. bends ignore expands B1


bends/moves to the right/away from contact/outwards/towards invar strip B1
2. idea that something gets hot M1
idea that bimetallic strip/invar/brass bends/breaks circuit A1
idea that something cools (when no current) M1
idea that bimetallic strip/invar/brass straightens/makes contact A1 [9]

10 (a) (i) Fig. 10.1 B1

(ii) Fig. 10.3 B1

(b) 2 complete cycles, any shape (if full-wave rectified, must be 4 humps) B1
cyclical and equal amplitude above & below axis B1
uniform spacing B1
intention of sinusoidal shape accept sinusoidal full-wave rectification B1 [6]

11 (a) thermionic emission B1

(b) (i) S2 OR 2
any 1 correct B1
(ii) S1 OR 1 ignore mention of S2 B2
all 3 correct B2
(iii) S3 OR 3 ignore mention of S1 and/or S2

(c) reverse polarity of plates (however expressed)/make upper plate positive


OR correct description of use of magnet B1 [4]

12 (a) (radio)activity OR count rate OR counts/s OR particles emitted/s


OR rate of decay OR number of undecayed atoms/nuclei
OR radiation OR original number of atoms/nuclei B1
NOT mass/substance/material, unless clearly specified
to decrease to half (original value) NOT half the time B1

(b) (i) 53 ± 1 (s) B1

(ii) 84 ± 1 (s) B1

(iii) candidate’s (ii) + candidate’s (i) C1


correct evaluation of candidate’s (ii) + candidate’s (i) A1 [6]

© University of Cambridge International Examinations 2011


w
w
w
.X
tr
me
eP
UNIVERSITY OF CAMBRIDGE INTERNATIONAL EXAMINATIONS

ap
er
International General Certificate of Secondary Education

s.c
om
MARK SCHEME for the October/November 2011 question paper
for the guidance of teachers

0625 PHYSICS
0625/21 Paper 2 (Core Theory), maximum raw mark 80

This mark scheme is published as an aid to teachers and candidates, to indicate the requirements of
the examination. It shows the basis on which Examiners were instructed to award marks. It does not
indicate the details of the discussions that took place at an Examiners’ meeting before marking began,
which would have considered the acceptability of alternative answers.

Mark schemes must be read in conjunction with the question papers and the report on the
examination.

• Cambridge will not enter into discussions or correspondence in connection with these mark schemes.

Cambridge is publishing the mark schemes for the October/November 2011 question papers for most
IGCSE, GCE Advanced Level and Advanced Subsidiary Level syllabuses and some Ordinary Level
syllabuses.
Page 2 Mark Scheme: Teachers’ version Syllabus Paper
IGCSE – October/November 2011 0625 21

NOTES ABOUT MARK SCHEME SYMBOLS & OTHER MATTERS

B marks are independent marks, which do not depend on any other marks. For a B mark to be
scored, the point to which it refers must actually be seen in the candidate's answer.

M marks are method marks upon which accuracy marks (A marks) later depend. For an M mark
to be scored, the point to which it refers must be seen in a candidate's answer. If a
candidate fails to score a particular M mark, then none of the dependent A marks can be
scored.

C marks are compensatory method marks which can be scored even if the points to which they
refer are not written down by the candidate, provided subsequent working gives
evidence that they must have known it. e.g. if an equation carries a C mark and the
candidate does not write down the actual equation but does correct working which
shows he knew the equation, then the C mark is scored.

A marks are accuracy or answer marks which either depend on an M mark, or which are one of
the ways which allow a C mark to be scored.

c.a.o. means "correct answer only".

e.c.f. means "error carried forward". This indicates that if a candidate has made an earlier
mistake and has carried his incorrect value forward to subsequent stages of working, he
may be given marks indicated by e.c.f. provided his subsequent working is correct,
bearing in mind his earlier mistake. This prevents a candidate being penalised more
than once for a particular mistake, but only applies to marks annotated "e.c.f."

e.e.o.o. means "each error or omission".

brackets ( ) around words or units in the mark scheme are intended to indicate wording used to
clarify the mark scheme, but the marks do not depend on seeing the words or units in
brackets.
e.g. 10 (J) means that the mark is scored for 10, regardless of the unit given.

underlining indicates that this must be seen in the answer offered, or something very similar.

OR/or indicates alternative answers, any one of which is satisfactory for scoring the marks.

Spelling Be generous about spelling and use of English. If an answer can be understood to
mean what we want, give credit.

Significant Answers are acceptable to any number of significant figures ≥ 2, except if figures
specified otherwise, or if only 1 sig. fig. is appropriate.

Units Incorrect units are not penalised, except where specified. More commonly, marks are
allocated for specific units.

Fractions These are only acceptable where specified.

Extras Ignore extras in answers if they are irrelevant; if they contradict an otherwise correct
response or are forbidden by mark scheme, use right + wrong = 0

Ignore Indicates that something which is not correct is disregarded and does not cause a right
plus wrong penalty.

© University of Cambridge International Examinations 2011


Page 3 Mark Scheme: Teachers’ version Syllabus Paper
IGCSE – October/November 2011 0625 21

Not/NOT Indicates that an incorrect answer is not to be disregarded, but cancels another
otherwise correct alternative offered by the candidate i.e. right plus wrong penalty
applies.

© University of Cambridge International Examinations 2011


Page 4 Mark Scheme: Teachers’ version Syllabus Paper
IGCSE – October/November 2011 0625 21

1 (a) (i) BC B1

(ii) AB B1

(b) area under graph C1


0.5 × 15 × 5 C1
37.5 (m) A1 [5]

2 (a) tape measure OR trundle wheel OR laser measure IGNORE metre rule B1

(b) (i) clock OR watch (any sort) B1

(ii) set clock/watch to zero OR note start time OR start clock/watch/timing B1


(start clock/watch/timing) when wood seen to fall or equivalent B1
stop clock/watch/note time when wood reaches bridge 2 B1

(iii) speed = distance/ time in any form, letters, words, numbers C1


50/400 C1
0.125 A1
m/s B1 [9]

3 (a) (i) plumb-line (name or description) OR try-square and (horiz.) bench


OR spirit level B1

(ii) line joining A and D M1


line joining B and E M1
intersection clearly labelled G (dependent on scoring both M marks) A1

(b) X clearly on centre line B1


X clearly within semicircular portion, but not on surface B1 [6]

4 (a) dark specks OR bright specks NOT molecules/particles B1


moving C1
randomly/zigzag OR dancing about A1

(b) Brownian motion/movement B1

(c) invisible/too small to see/very small B1


moving fast/high kinetic energy B1
moving randomly/all directions B1 [7]

5 (a) 150 × 3 C1
450 (Hz) A1

© University of Cambridge International Examinations 2011


Page 5 Mark Scheme: Teachers’ version Syllabus Paper
IGCSE – October/November 2011 0625 21

(b) any figure between 20 and 50 inclusive


AND any figure between 15,000 and 25,000 inclusive B1

(c) increases/rises B1 [4]

6 (a) ultrasound B1

(b) (i) infra-red


visible all 4 correct B2
ultra-violet (any 2 correct B1)
X-rays

(ii) radio OR the top/first one B1

(iii) infra-red B1

(iv) X-rays OR gamma rays B1 [6]

7 (a) (i) needle inside coil B1


current through coil OR connect battery/power supply M1
direct current OR d.c.
OR a.c. and switch off before removing needle/ magnet A1

(ii) freely suspend/pivot and see which end points N (or equivalent)
OR see which end is repelled by N pole of a magnet B1

(b) 4+ smooth curves leaving one end and going to the other (ignore any arrows) B1
no lines crossing or meeting, even at ends B1 [6]

8 (a) battery/ammeter connected wrong way round


OR negative of battery should go to negative of ammeter B1

(b) correct symbols for battery, ammeter and rheostat


(allow common variants on battery/cell symbol) M1
all components in series A1

(c) voltmeter (any recognisable symbol) clearly in parallel with coil B1

(d) (i) 2.8 (A) and 12 (V) both B1

(ii) ammeter increases B1


voltmeter increases B1

(iii) 1.4 (A) OR half candidate’s original reading B1


6 (V) OR half candidate’s original reading B1 [9]

© University of Cambridge International Examinations 2011


Page 6 Mark Scheme: Teachers’ version Syllabus Paper
IGCSE – October/November 2011 0625 21

9 (a) transformer (ignore step-up/down) B1

(b) 132,000/22,000 OR 240/132,000 C1


X: 6 A1
Y: 0.001818 to at least 4 dec. pl. OR 1/550 NOT 550 A1

(c) less heat/energy loss


thinner/smaller cables
less copper used
less cable weight any 2 use  +  = 0 for incorrect extras B1+B1
less massive pylons
cheaper
smaller current [6]

10 (a) (electric) charge OR charged body B1


force B1

(b) A and B closer together allow touching M1


threads straight and equal angle (by eye) to vertical A1

(c) E horizontal to left


W vertically down all 3 marked on his diagram –1 e.e.o.o. B2
T up thread

(d) zero or 0 or nothing B1 [7]

11 (a) (i) filament/cathode clearly and correctly labelled B1

(ii) anode clearly and correctly labelled B1

(b) (i) battery shown connected across filament (no e.c.f.) B1

(ii) power supply connected between filament & anode (no e.c.f.) B1

(iii) straight path shown along axis (no e.c.f.) B1

(c) bright spot (or equivalent) B1

(d) spot moves down B1 [7]

12 (a) points correctly plotted (±½ small square) –1 e.e.o.o. B2


smooth curve through candidate’s points (by eye) B1

© University of Cambridge International Examinations 2011


Page 7 Mark Scheme: Teachers’ version Syllabus Paper
IGCSE – October/November 2011 0625 21

(b) (i) 1. in range 2.2–3.0 B1


2. in range 18.0–19.0 B1

(ii) 2 half-lives C1
(candidate’s 2 – candidate’s 1)/2 C1
7.5–8.6 (days) e.c.f. A1 [8]

© University of Cambridge International Examinations 2011


w
w
w
.X
tr
me
eP
UNIVERSITY OF CAMBRIDGE INTERNATIONAL EXAMINATIONS

ap
er
International General Certificate of Secondary Education

s.c
om
MARK SCHEME for the May/June 2012 question paper
for the guidance of teachers

0625 PHYSICS
0625/21 Paper 21 (Core Theory), maximum raw mark 80

This mark scheme is published as an aid to teachers and candidates, to indicate the requirements of
the examination. It shows the basis on which Examiners were instructed to award marks. It does not
indicate the details of the discussions that took place at an Examiners’ meeting before marking began,
which would have considered the acceptability of alternative answers.

Mark schemes must be read in conjunction with the question papers and the report on the
examination.

• Cambridge will not enter into discussions or correspondence in connection with these mark schemes.

Cambridge is publishing the mark schemes for the May/June 2012 question papers for most IGCSE,
GCE Advanced Level and Advanced Subsidiary Level syllabuses and some Ordinary Level
syllabuses.
Page 2 Mark Scheme: Teachers’ version Syllabus Paper
IGCSE – May/June 2012 0625 21

NOTES ABOUT MARK SCHEME

B marks are independent marks, which do not depend on any other marks. For a B mark to be
scored, the point to which it refers must actually be seen in the candidate's answer.

M marks are method marks upon which accuracy marks (A marks) later depend. For an M mark to
be scored, the point to which it refers must be seen in a candidate's answer. If a
candidate fails to score a particular M mark, then none of the dependent A marks can be
scored.

C marks are compensatory method marks which can be scored even if the points to which they
refer are not written down by the candidate, provided subsequent working gives
evidence that they must have known it, e.g. if an equation carries a C mark and the
candidate does not write down the actual equation but does correct working which
shows he knew the equation, then the C mark is scored.

A marks are accuracy or answer marks which either depend on an M mark, or which are one of
the ways which allow a C mark to be scored.

c.a.o. means “correct answer only”.

e.c.f. means “error carried forward”. This indicates that if a candidate has made an earlier
mistake and has carried his incorrect value forward to subsequent stages of working, he
may be given marks indicated by e.c.f. provided his subsequent working is correct,
bearing in mind his earlier mistake. This prevents a candidate being penalised more than
once for a particular mistake, but only applies to marks annotated “e.c.f.”

e.e.o.o. means “each error or omission”.

brackets ( ) around words or units in the mark scheme are intended to indicate wording used to
clarify the mark scheme, but the marks do not depend on seeing the words or units in
brackets, e.g. 10 (J) means that the mark is scored for 10, regardless of the unit given.

underlining indicates that this must be seen in the answer offered, or something very similar.

OR/or indicates alternative answers, any one of which is satisfactory for scoring the marks.

Spelling Be generous about spelling and use of English. If an answer can be understood to mean
what we want, give credit.

Significant figures
Answers are acceptable to any number of significant figures > 2, except if specified
otherwise, or if only 1 sig. fig. is appropriate.

Units Incorrect units are not penalised, except where specified. More commonly, marks are
allocated for specific units.

Fractions These are only acceptable where specified.

Extras Ignore extras in answers if they are irrelevant; if they contradict an otherwise correct
response or are forbidden by mark scheme, use right + wrong = 0

Ignore Indicates that something which is not correct is disregarded and does not cause a right
plus wrong penalty.

Not/NOT Indicates that an incorrect answer is not to be disregarded, but cancels another
otherwise correct alternative offered by the candidate i.e. right plus wrong penalty
applies.

© University of Cambridge International Examinations 2012


Page 3 Mark Scheme: Teachers’ version Syllabus Paper
IGCSE – May/June 2012 0625 21

1 (a) speed = distance ÷ time in any form OR (distance =) speed × time C1


80 × ½ OR 80 × 0.5 C1
40 (km) A1

(b) (i) First section of line:


horizontal line starting at zero time, any speed M1
at 80 km/hour A1
from 0 to 0.5 hour, no further A1

(ii) Second section of line:


straight line sloping down B1
line starting at end of previous section and ending at 1 hour
(condone not straight) B1
line ending at 30 km/hour B1

Third section of line:


vertical/near vertical line down to 0 at 1 hour B1
ignore further sections of graph [Total: 10]

2 (a) 84 – 53 C1
31 (cm3) A1

(b) 238 – 205 C1


33 (g) A1

(c) density = mass ÷ volume, however arranged B1


33 ÷ 31 e.c.f. (a) and (b) C1
1.0645161 correct to any no of sf > 2 don’t accept fractions A1
g/cm3 accept kg/m3 if clear attempt to convert to kg and m3 B1
[Total: 8]

3 (a) 70 000 (N) arrow to right accept labelled “thrust” B1


25 000 (N) arrow to left accept labelled “friction” B1

(b) (i) to left OR backward OR opposing motion B1

(ii) 45 000 (N) B1

(iii) air friction/air resistance/drag NOT wind/wheels/weight


NOT if any incorrect extra e.g. weight B1

(c) (i) accelerates OR speed increases OR moves faster M1

(ii) idea of unbalanced force e.g. forward force > backward force
NOT just forward force is bigger A1
[Total: 7]

© University of Cambridge International Examinations 2012


Page 4 Mark Scheme: Teachers’ version Syllabus Paper
IGCSE – May/June 2012 0625 21

4 (a) they/molecules/particles/atoms moving/vibrating/have KE C1


they/molecules/particles/atoms collide (condone with each other) C1
they/molecules/particles/atoms collide with walls A1
extra relevant information e.g. exert force, change of momentum, bounce back/off,
lots over an area, random/Brownian motion B1

(b) (i) decreases B1

(ii) increases B1
[Total: 6]

5 (a) changed/converted/transferred to other forms B1

(b) (i) 24 (kJ) B1

(ii) idea of wasted/lost C1


heat ignore sound A1

(iii) 696 OR 720 – candidate’s (i), correctly evaluated B1

(iv) idea of not very good no e.c.f.


accept “there is a lot of energy lost”, accept calculation
ignore “not 100%” B1
[Total: 6]

6 (a) EITHER
ray from tip of object through optical centre of lens M1
straight on after lens A1
OR
ray from tip of object through F2 and on to lens M1
parallel to axis after lens A1

(b) image drawn between candidate’s intersection and the axis B1

(c) same size B1


inverted no e.c.f. use  +  = 0 for size and orientation B1
real B1

(d) smaller B1
closer to lens/to the left B1
[Total: 8]

© University of Cambridge International Examinations 2012


Page 5 Mark Scheme: Teachers’ version Syllabus Paper
IGCSE – May/June 2012 0625 21

7 (a) infra-red B1

(b) infra-red B1

(c) X-rays B1

(d) microwaves B1
[Total: 4]

8 (a) (i) charge(s) OR electron(s) NOT ions B1

(ii) (an) ammeter B1

(iii) (a) voltmeter B1

(b) (R =) V/I in any form C1


9.6/0.8 C1
12 A1
Ω OR ohm(s) OR volt/amp OR volts per amp B1

(c) (i) increases B1

(ii) decreases OR e.c.f. from (i) B1


[Total: 9]

9 (a) coil clearly and unambiguously indicated B1

(b) increase strength/power of magnet


ignore increase magnetism/ignore add core
ignore magnets closer/bigger

increase current/voltage/energy from battery any 2 B1 + B1


accept stronger/more powerful battery

increase number of turns (in coil)


ignore bigger coil ignore rotations

(c) reverse current OR reverse magnet/field however expressed B1


[Total: 4]

© University of Cambridge International Examinations 2012


Page 6 Mark Scheme: Teachers’ version Syllabus Paper
IGCSE – May/June 2012 0625 21

10 (a) any variation of allow and B1

(b) (i) plug switch M1

(ii) exposed metal or equivalent OR not insulated OR (easy to get) shock A1

(c) (i) pull-cord switch B1

(ii) idea that water/moisture conducts ignore shock B1


covering/plastic/nylon is an insulator OR no metal is exposed B1

(d) 3 lamps connected in parallel with each other


NOT if shorted out by switch or extra wire B1
lamp combination (e.c.f.) in series with switch (e.c.f.) and supply
accept any recognisable symbol, accept closed switch B1
[Total: 8]

11 (a) any downward deflection and no upward deflection B1


curve, either all up or all down, from A to end of region between plates M1
straight on from end of region between plates, towards BC A1

(b) idea of deflection upwards/it goes upwards/it moves upwards no e.c.f.


ignore opposite direction/opposite path B1
[Total: 4]

12 (a) thorium OR Th OR 232 OR 90 B1

(b) technetium OR Tc OR 99(m) OR 43 B1

(c) barium OR Ba OR 139 OR 56 B1


silver OR Ag OR 110 OR 47 any 2
thorium OR Th OR 232 OR 90 B1

NOTE: technetium + anything scores 1 mark, “all of them” scores 1 mark

(d) silver OR Ag OR 110 OR 47 B1

(e) technetium OR Tc OR 99(m) OR 43 OR gamma


NOT any extras B1
[Total: 6]

© University of Cambridge International Examinations 2012


w
w
w
.X
tr
me
eP
CAMBRIDGE INTERNATIONAL EXAMINATIONS

ap
er
International General Certificate of Secondary Education

s.c
om
MARK SCHEME for the October/November 2012 series

0625 PHYSICS
0625/21 Paper 2 (Core Theory), maximum raw mark 80

This mark scheme is published as an aid to teachers and candidates, to indicate the requirements of
the examination. It shows the basis on which Examiners were instructed to award marks. It does not
indicate the details of the discussions that took place at an Examiners’ meeting before marking began,
which would have considered the acceptability of alternative answers.

Mark schemes should be read in conjunction with the question paper and the Principal Examiner
Report for Teachers.

Cambridge will not enter into discussions about these mark schemes.

Cambridge is publishing the mark schemes for the October/November 2012 series for most IGCSE,
GCE Advanced Level and Advanced Subsidiary Level components and some Ordinary Level
components.
Page 2 Mark Scheme Syllabus Paper
IGCSE – October/November 2012 0625 21

NOTES ABOUT MARK SCHEME SYMBOLS & OTHER MATTERS

B marks are independent marks, which do not depend on any other marks. For a B mark to be
scored, the point to which it refers must actually be seen in the candidate's answer.

M marks are method marks upon which accuracy marks (A marks) later depend. For an M mark to
be scored, the point to which it refers must be seen in a candidate's answer. If a
candidate fails to score a particular M mark, then none of the dependent A marks can be
scored.

C marks are compensatory method marks which can be scored even if the points to which they
refer are not written down by the candidate, provided subsequent working gives
evidence that they must have known it. e.g. if an equation carries a C mark and the
candidate does not write down the actual equation but does correct working which
shows he knew the equation, then the C mark is scored.

A marks are accuracy or answer marks which either depend on an M mark, or which are one of
the ways which allow a C mark to be scored.

c.a.o. means "correct answer only".

e.c.f. means "error carried forward". This indicates that if a candidate has made an earlier
mistake and has carried his incorrect value forward to subsequent stages of working, he
may be given marks indicated by e.c.f. provided his subsequent working is correct,
bearing in mind his earlier mistake. This prevents a candidate being penalised more than
once for a particular mistake, but only applies to marks annotated "e.c.f."

e.e.o.o. means "each error or omission".

brackets ( ) around words or units in the mark scheme are intended to indicate wording used to
clarify the mark scheme, but the marks do not depend on seeing the words or units in
brackets, e.g. 10 (J) means that the mark is scored for 10, regardless of the unit given.

underlining indicates that this must be seen in the answer offered, or something very similar.

OR/or indicates alternative answers, any one of which is satisfactory for scoring the marks.

o.w.t.t.e. means “or words to that effect”.

Spelling Be generous about spelling and use of English. If an answer can be understood to mean
what we want, give credit.

Significant figures
Answers are acceptable to any number of significant figures [ 2, except if specified
otherwise, or if only 1 significant figure is appropriate.

Units Incorrect units are not penalised, except where specified. More commonly, marks are
allocated for specific units.

Fractions These are only acceptable where specified.

Extras Ignore extras in answers if they are irrelevant; if they contradict an otherwise correct
response or are forbidden by mark scheme, use right + wrong = 0

© Cambridge International Examinations 2012


Page 3 Mark Scheme Syllabus Paper
IGCSE – October/November 2012 0625 21

Ignore Indicates that something which is not correct is disregarded and does not cause a right
plus wrong penalty.

Not/NOT Indicates that an incorrect answer is not to be disregarded, but cancels another
otherwise correct alternative offered by the candidate i.e. right plus wrong penalty
applies.

Work which has been crossed out, but not replaced, should be marked as if it had not been crossed
out.

© Cambridge International Examinations 2012


Page 4 Mark Scheme Syllabus Paper
IGCSE – October/November 2012 0625 21

1 (a) moment/torque
ignore turning force B1

(b) opposite direction B1


condone different direction(s)
larger or correct reverse argument (opening force is smaller) B1

(c) (apply) force further from hinge


OR oil/reduce friction/new hinge/use an assist mechanism/replace hinge(s) B1 [4]

2 (a) D = M / V in any form B1

(b) (i) length × width × height in any form


OR 2.5 ( × 104) × 6.0 ( × 103) × 3 ( × 10–6) i.e. ignore powers of 10 C1
4.5 × 10n any power of 10 C1
450 (m3) c.a.o. 4.5 x 102 A1

(ii) 900 × his 450 or correct sub into D = M / V C1


4.05 × 105 OR 405 000 (kg) e.c.f. A1 [6]

3 (a) speed = distance / time in any form OR distance / speed C1


80 / 320 C1
0.25 (s) A1

(b) (i) 0.45 OR his (a) + 0.2(0) correctly evaluated B2


(allow B1 only, 0.05 / his(a) – 0.2(0) OR 0.25 / his (a) alone)

(ii) start timing when he sees flash/smoke (accept any other


appropriate visual stimulus e.g. hand dropping as gun fires) B1

(c) 12.5 ± 0.2(s) Condone (1 min) 12.5 s OR 12.05 / 12.5 – 0.45 C1

12.95 OR 12.5 + his (b)(i) A1 [8]

© Cambridge International Examinations 2012


Page 5 Mark Scheme Syllabus Paper
IGCSE – October/November 2012 0625 21

4 (a) top box ticked B1

(b) elastic/strain/potential NOT gravitational PE B1

(c) kinetic B1
ignore heat

(d) gravitational/gravitational potential/GPE/PE B1


maximum B1
kinetic OR thermal/allow heat B1
thermal allow heat B1 [7]

5 (a) (i) move/vibrate/oscillate faster OR increase/gain KE B1


move (further) apart OR (they) separate B1

(ii) any 1 increases/enlarges/gets bigger/expands o.w.t.t.e. C1


all three increase A1

(b) nut/hole expands/enlarges


ignore particles expand/enlarge B1
bolt doesn’t expand (as much) B1 [6]

6 (a) (i) r correctly shown B1

(ii) bent up at first surface B1


bent up at second surface B1
straight line within prism B1

(iii) P clearly shown as the original point of entry B1

(b) (i) blue light refracted from same point at first surface B1
blue shown with greater refraction B1
blue light always below red light B1

(ii) dispersion B1 [9]

© Cambridge International Examinations 2012


Page 6 Mark Scheme Syllabus Paper
IGCSE – October/November 2012 0625 21

7 (a) arrow pointing to left B1

(b) rotates/turns/S pole goes away from magnet/repelled/ changes direction B1


N pole points to magnet/S Pole points to N Pole (of Earth)/turns through 180° B1
S Pole/N Pole points in opposite direction

(c) magnetic field/electromagnet(ism)/(ic) M1


caused by current A1 [5]

8 (a) its voltage/potential difference C1


condone volts
its e.m.f./electromotive force A1

(b) V = IR in any form OR V / R C1


4.5 / 180 C1
0.025 OR 2.5 × 10–2 OR 1 / 40 A1
A/amps/amp/a B1

(c) (i) two resistors shown in parallel (accept any symbol here) B1
condone faint lines through resistors (where attempted to rub out wire)

battery in series with resistances (allow any recognisable symbol here) B1


(even if resistances not in parallel)
all symbols correct (allow cell symbol for battery) B1
(allow rheostat for resistor condone old symbol)

(ii) 1. 4.5 (V) B1


ignore units
2. 0.025 OR his (b) B1
ignore units [11]

9 (a) switch correctly identified B1

(b) (i) moves/flows condone (current) flows OR stays the same B1


ignore nothing (happens)

(ii) increases/higher/greater M1
condone greater than zero
any indication of gradual increase A1

(c) remains the same OR decreases/goes back to zero (very) slowly i.e. ignore B1 [5]
decreases/getting smaller on their own.

© Cambridge International Examinations 2012


Page 7 Mark Scheme Syllabus Paper
IGCSE – October/November 2012 0625 21

10 (a) copper B1

(b) core B1

(c) Np / Ns = Vp / Vs in any form C1


8000/Ns = 240 / 6 OR 240 = 6 OR Ns = 6 C1
8000 Ns 8000 240
200 A1

(d) (i) lamp less bright/less than full brightness/wouldn’t light B1


(up properly)/ has less energy

(ii) lamp blows/bursts OR lamp too bright OR lamp B1 [7]


overheats/burns out OR much brighter/has more energy

11 (a) paper stops α C1


sheet of paper makes no difference to count rate A1

(b) Aluminium absorbs β allow aluminium stops β C1


Aluminium makes count rate decrease A1

(c) (10mm) lead / Pb stops all β OR only γ gets through (10 mm) lead / Pb B1
still some count rate with lead / Pb B1 [6]

12 (a) (i) (number of) protons + neutrons OR p + n B1


OR mass number/nucleon number

(ii) (number of) protons OR atomic number/ proton number B1


Ignore electrons

(b) (i) zero nucleons OR mass number is zero B1

(ii) negative charge OR requires a proton to be neutral B1

240 240
(c) (i) 94 Pu OR Pu OR 94 B1

250 250
(ii) 98 Cf OR 98 NOT just Cf B1 [6]

© Cambridge International Examinations 2012


Mark scheme for IGCSE Physics (0625/3) – Extended Theory May 1999

1 (a) Distance moved in one revolution is equal to the circumference 1


= 62.8/63 (in) 1
Time for 1 rev is 5(s) 1
Speed = 62.8/5 1
= 12.6 (m/s) 1
4 (max)

(b)(i) 12.6 (m/s) / same value / same number 1


Direction to the right /east / as marked on diagram 1
(ii) Velocity is a vector or has direction / speed is a scalar or has no direction 1
P is moving in a circle/constantly changing direction/not in a straight line 1
If direction changes velocity changes/speed does not or definition of
velocity/speed 1
4 (max)

(c) Rotation is taking place/direction changing 1


Force is the centripetal force/force needed for circular motion 1
Must act through centre otherwise motion not circular 1
3

(d)(i) 1. Any reference to air resistance 1


2. The water has hit the ground 1
(ii) 1. Velocity on hitting ground = (10 x 0.6) = 6 (m/s) 1
Distance=6/2 x 0.6 1
= 1.8 (m) 1
2. Horizontal distance = average velocity x time or area under graph 1
= (9.5 x 0.6) = 5.7 (m) 1
7

(e) Water spreads out 1


Cross-sectional area of water at R much larger than at Q 1
Pressure = force/area 1
Same mass/volume/s spread over larger area = Lower pressure or vice versa2
4 (max)

2 (a) Heat = mass x specific heat x temperature change 1


Heat = power x time 1
Heat from heater = heat in water 1
30 x l000 x l8 000 = 54000 x c x 2 2
= 5 000 (J/kg K) 1
6

(b)(i) Most energetic molecules leave the water surface (evaporation) 1


Carry away extra/latent heat 1
Water heats air molecules at surface (by conduction) 1
Air molecules carry heat away by convection 1
Molecules in water surface emit radiation into air 1
Energy carried away as wave energy 1
4 (max)

(ii) Some of the heat supplied does not end up in the water 1
Takes longer for same temperature rise 1
More heat supplied in longer time 1
7

1
3 (a) Scale, full size 1
Line at object height, refracted through lens to pass through focus 1
Ray through focus produced back to pass through “3 cm line” 1
Line through centre to locate object and image 1
Distance of object from lens, 2.5 cm to 2.9 cm 1
Distance of image from lens ,7.5 cm to 8.7 cm 1
6

(b)(i) Light of one colour / wavelength / frequency 1


8
(ii) 3 x 10 1
m/s 1
(iii) Formula quoted sin i / sin r = refractive index 1
o o
= sin 37 / sin 22 1
= 1.5 1
o
(iv) Beam continued using given angle of refraction (22 ) 1
7

4 (a)(i) Reasonable curve, either direction 1


Curve to positive plate (upwards) 1
(ii) Electrons are negatively charged
Unlike charges attract
Positive plate attracts electrons 2
(iii) Arrow, towards P, anywhere on the lines PQRS 1
5
13 -19
(b)(i) Total charge/s = 10 x 1.6 x 10 1
-6
= 1.6 x 10 (C) 1
(ii) Charge = current x time 1
-6
Current = charge/1(s) or 1.6 x 10 / 1 1
Amperes / A 1
4 (max)

(c)(i) Equation E = V It or = V q 1
E = 10000 x 2.1 x 600 (J) 1
7
= 1.3x10 (J) 1
(ii) Equation P = E/t 1
7
P = 1.3 x l0 /600 1
4
2.1 x 10 (W) 1
6

(d)(i) Path curved, up or down 1


Curved downwards 1
Fleming’s (left-hand) rule stated 1
(ii) Current = 12/100 1
= 0.12 (A) 1
(iii) Bigger deflection 1
in the opposite direction 1
7

5 (a)(i) lonisation means creating positively charged and negatively charged ions 1
from the air molecules between the gauze and the wire 1
The wire and the gauze are charged, one positively and the other negatively 1
The ionised air molecules (atoms) move to the opposite charge 1
Create a “surge” of current seen as a spark 1
(ii) Alpha, a huge amount, any quoted figures 1

2
Beta, a small amount compared to alpha, any quoted figures 1
Gamma, virtually none at all 1
6 (max)

(b) Beta: mass 1/1836 amu/very small/ negligible 1


constitution 1 electron 1
charge -1 unit 1
Gamma : mass zero 1
constitution waves/ wave energy 1
charge zero 1
4 (max)

3
Mark scheme for IGCSE Physics (0625/3) – Extended Theory November 1999
1 (a)(i) PE (for one fall) is mgh 1
= (200 x 10 x 6) 1
= 12 000 J 1
(ii) PE (lost) = KE (on impact) or v = 2gh or s = 2at 1
12000(J) or answer to (i) = _ mv or v = 2 x 10 x 6 or t = 1.1 s 1
v = 120 or v = gt 1
v = l0.95 or 11 m/s 1
7

(b)(i) (Energy to do) work/mechanical 1


energy to move the pile against resistance/ground forces 1
(Energy to) heat 1
(Energy to) sound (energy chains max 1, first answer only considered.)1
(ii) At impact kinetic energy/ momentum large 1
After impact kinetic energy/ momentum soon zero 1
Kinetic energy change/ momentum change large 1
Slows to rest in very short distance/time 1
KE lost/work = force x distance the pile moves / rate of change of
momentum = force 2
8 (max)

(c) lifting suspension etc/ pile deeper after each hit 1


need more p.e. each time 1
Rising mass gains k.e. 1
All lost at top 1
Power to stop/brake rising mass 1
All lost as heat 1
Efficiency of motor not 100% 1
Lost as heat 1
4 (max)

(d) Greater mass 1


Fall greater distance 1
Use motor to drive mass down/thinner or pointed pile 1
3

2 (a) Take temperatures before and after heating/ temperature difference


with thermocouple 1
Take the time that the heater is switched on/ time with stopwatch 1
Find in some way the mass of the block and the power of the heater 2
3 (max)

(b) Heat input of heater = heat gained by the block 1


Heat input of heater =200 x 1 50(J) 1
Heat gained by metal =1 x s x 50(J) 1
s = 600 (J/kgK)/ 0.6 (J/gE) 1
4

(c)(i) Outline e.g. 2 metals to suitable meter 1


Essential parts labelled 1
(ii) Temperature difference produces an emf/voltage/current 1
emf/current read by meter as a temperature 1
(iii) Temperature rapidly changing/small changes in temperature 1
measures high/low/wide range of temperatures 1
Not fragile like mercury in glass /can be made recording/remote
reading / thermometer not removed 1
6 (max)
o o o
3 (a) Each reflected angle 85 /10 /170 deviation at each reflection 1
o o
Total deviation 20 /160 1
2

(b) (Angle of incidence much) greater than the critical angle 1


So total internal reflection occurs 1
2

(c)(i) 1.50 = 1/sin c 1


sin c=0.6667 1
o o
c = 41.8 / 42 1
(ii) (Angle of incidence is) less than the critical angle 1
Refracted (out into the air)/ not totally internally reflected 1
4 (max)

(d)(i) Light of one colour/ wavelength 1


Light of one frequency 1
14 -7
(ii) Speed = 4 x l0 x 5 x l0 1
8
= 2 x l0 (m/s) 1
(iii) Speed in air/ speed in glass = refractive index 1
Speed in air / speed in glass x 1.50 1
5 (max)

4 (a)(i) Speed of rotation (of the coil/magnet) 1


is faster 1
The number of turns on the coil / number of coils (not transformer) 1
is greater 1
The strength of the magnet / magnetic field 1
is stronger 1
Any other correct factor identified 1
How factor change relates to the emf 1
4 (max)

(ii) The induced emf/current produced by a conductor cutting a


magnetic field 1
induced emf/current also interacts with the magnetic field 1
interaction produces a force which slows/opposes the moving conductor 1
7

(b)(i) Power input = power output / 400 x 80 = 30 000 x I 1


I = 1.1(A) 1
(ii) Reduce voltage 1
(iii) to value suitable for consumers / 240 V / 110 V 1
Current in these cables much reduced 1
(because) voltage increased 1
2 2
Heat = I Rt or Power = I R or heat in cables less 1
6 (max)

(c)(i) 3.1 (A) at X 1


0.8 (A) at Y 1
(ii) R = 110/any current value 1
R = (110/0.8) = 138 (ohm) 1
(iii) Any attempt at use of I/R formula for parallel resistors 1
1/R = 1/24 + 1/48 1
R = 16 (ohm) 1
2
(iv) Use of VI or I R for calculating power 1
2
= 110 x 4.6/4.6 x 24 1
= 506/508 (W) 1
2
(v) Use of energy = I Rt / VIt 1
= 30000 / 31000 (J) 1
10 (max)

5 (a) Diagram showing radioactive liquid in pipe, weld and detector in


correct places 1
three labels 1
Where weld thin reading rises 1
Radiation passes more easily through / less absorbed by thinner metal 1
3 (max)

(b)(i) Alpha - none passes through steel 1


Beta - some passes through steel 1
Gamma - most/all passes through steel 1
(ii) Accept either beta or gamma with some reasoning 1
Beta/gamma with clear reason for the choice 1
4 (max)

(c) Any 3 standard precautions which apply to this situation 3


(generally shielding/absorbing, distance, monitoring radiation) 3
Mark scheme for IGCSE Physics (0625/3) – Extended Theory May/June 2000

1 (a) Any three from:


velocity is a vector and speed is a scalar
vector or velocity has direction
scalar or speed has no direction or only magnitude
ref to vertical path as fixed direction 3
3

(b)(i) 1. 4.5 ± 0.1 s 1


2. any four from:
decelerates
uniformly
from high velocity at 0 s
to zero velocity(at 4.5 s)
accelerates (for the last 0.5 s) 4
2
(ii) 1. (—)10 m/s 1
2. 9.0 s or e.c.f. from 1 1
(iii) value stated as 45 m/s ± 1 m/s 1
8

2 (a) specific latent heat = heat supplied/mass melted 1


(mass melted =) 0.18 kg 1
(heat supplied =) 60 000 J 1
specific latent heat = 330 000 J/kg 1
4
(b) any 2 from:
molecules vibrate
pass energy from molecule to molecule
process is conduction 2
2

3 (a)(i) mass = 0.75 kg 1


(ii) weight =0.75 x 10 1
= 7.5 N 1
3

(b) Any 3 from:


(sum of) clockwise moments = (sum of) anticlockwise
moments about pivot
no net turning effect
needs extra moment (one side) to turn 3
3
2
(c) use of k.e. = _ mv 1
2
= _ x 0.5x1.2 1
=0.36 J 1
3

(d) any 2 from; (only) k.e. (before impact)


less or some k.e. after impact
heat + sound 2
2

1
4 (a) any 2 from:
random
collides with or bounces off sides
hits or bounces off other molecules
other relevant points/explanations 2
2

(b)(i) hits / bounces off ABCD at some time 1


(ii) hits / bounces off all sides at some time / chance of hitting all sides
equal (so equal pressure) 1
2

(c) use of p.v = c 1


5 5
p = (0.09/0.04 x 10 ) = 2.3 x l0 Pa 1
2
o
5 (a) hits at right angles to surface / angle i = 0 along normal 1
1

(b) velocity/speed/wavelength 1
increases at F,decreases at D 1
2
o
(c) correct angle marked / value 45 2
2
o
(d) refractive index = 1/sin 45 1
= 1.4(1) 1
2

6 (a)(i) place of higher pressure / air molecules closer together 1


(ii) place of lower pressure / air molecules further apart 1
2
(b) wavelength = 2.5 m 1
frequency = (330/2.5=) 130 Hz 1
2
(c) distance travelled in 1.2 s = (330 x 1.2 =) 396 m 1
distance = 200 m 1
2

7 (a) circuit with battery symbol and switch 1


and three from:
at least one straight line inside the coil
at least one loop at each side
at least one complete loop through the coil 2
full field
arrows correct 3
4

(b) use of charge/time 1


= 0.4 A 1
2

2
(c)(i) use of VIt 1
= 19(.2) J 1
(ii) resistance = 1.2/0.4 1
= 3 ohm 1
4

(d) e.m.f. = joules/coulomb 1


= 24/16 or 1.5 V 1
2

8 (a) any 3 from: primary (current) creates magnetic field


field is (constantly) changing
field in core links into secondary
changing field in or through secondary coil
induces a.c. in secondary coil, 3
3

(b) VpIp = VsIs


240 x Ip = 12 x 3.2
Ip (= 3.2/20) = 0.16 A 3
3

9 (a) deviation up marked P 1


deviation down marked R /or opposite way, both marked 1
both correct and reasonable curves 1
3

(b) Any three from:


in front/behind paper / at right angles to electric field 1
N-pole “in front” of diagram (to give field into paper)
magnetic field deflects beam “downwards”
by Fleming’s rule etc. 2
3
228 4 224
10 (a) 90 Th 2 He + 88 Ra 2
2

(b)(i) must be _ because _ and _ absorbed by Aluminium /


only _ not absorbed 2
(ii) _/_/_ 1
one reason e.g. range of any/all greater than 1.5 cm in air 1
4

(c)(i) atoms of gas gain / lose electron(s) / by or from colliding particles 1


(ii) (photons) not particles / no mass / no charge / very penetrating /
speed of light etc. 1
energy (of photons) not big enough 1
3

3
Mark scheme for IGCSE Physics (0625/3) – Extended Theory November 2000
1 (a) downward force through centre of mass 1
upward force along wire 1
downward force/weight = 5 N 1
upward force/tension = 5 N 1
4

(b) weight/gravity/pull of earth 1


tension/pull of wire(on mass) 1
2

(c)(i) continues straight line XY for any distance 1


line becomes curve of decreasing gradient to Z 1
(ii) 1.5kg 1
(iii) 12mm 1
4

2 (a) change in property/length/volume per degree 1


1
(b)(i) nitrogen 1
(ii) gases expand more/most 1
2

(c)(i) copper 1
(ii) small increase in length per degree/high melting point etc 1
2

(d) pointer movement not same for all degrees 1


effect different at different parts of the scale 1
2

3 (a) use of p = hdg 1


= 6 x 1000 x 10 1
= 60 000 Pa 1
3

(b) use of F = pA 1
(= 45 000 x 0.015) = 680 N 1
2

(c) use of work done = Fs 1


(= 550 x 4) = 2200 J 1
2

(d) at pulley (axles) 1


in person’s body 1
at block 1
work/energy used against friction force 2
or work/energy used in moving parts of the body eg. lifting arms 1
4 (max)

4 (a) use of v = f x wavelength, 1


(= 0.8 x 1.6) = 1.3 m/s 1
2

(b(i) 1.6 m or same 1


(ii) 0.8 Hz or same 1
2

(c) (part) circles 1


centred on middle of gap 1
not complete semicircles 1
3

5 (a) use of specific heat capacity =


heat supplied/mass x temperature change 1
= 15 500/0.45 x 8.2 2
= 4 200 J/kgK 1
4

(b)(i) compress gas/ move piston in 1


(ii) use of p x V = c 1
substitution correct 1
3
v = 0.0034 m 1
4

6 (a) ray correct through F 1


ray correct through C or ray through other F 1
rays produced back to form image 1
3

(b) sensible position, left of lens 1


1

(c) image length 4.5 ± 0.2 cm 1


Approximately x 3 [e.c.f. from (a)] 1
2

7 (a) use of R = V/I 1


6 ohm 1
2

(b) two in parallel = 3 ohm 1


circuit total = 9 ohm 1
2

(c) use of current = V/R 1


= 1.3 A 1
2

(d) voltage across R less than 12 V / low / 8V or current through R


less than 2 A (1.3 (A)
or lower voltage means less bright or lower current means less bright 1
current through P and Q equal / voltage across P and Q equal 1
current through P/Q less than through R or p.d. across P/Q less than
across R 1
3

(e) all three in parallel 1


1
8 (a) any sine wave 1
amplitude approx 7V 1
2 correct full waves 1
3

(b)(i) any mention of a magnetic field 1


conductor cutting field/lines 1
(so) induces emf (must be linked to cutting lines) 1
(ii) change in rate of cutting lines of force 1
coil rotates cuts one way then other 1
change in cutting direction reverses emf 1
4 (max)

9 (a)(i) electrons move from duster to rod 1


extra electrons on rod so -vely charged 1
(ii) hold rod close to plate 1
touch plate to earth it 1
remove rod leaves plate positively charged 1
4 (max)

(b) alpha particles (strongly) ionise air 1


make air conduct electrons to the plate 1
(or in terms of conduction and neutralisation of charg(e) 2
A o A
10 (a) zX e + Z+1Y
-1 1
correct mass no. and correct atomic number 1
2

(b)(i) some beta absorbed by paper, some / most pass through 1


thicker paper less pass through / lower reading 1
(ii) no gamma would be absorbed by paper 1
gamma are less safe 1
4

(c) smooth curve, either direction 1


curve towards bottom of page 1
2
Mark scheme for IGCSE Physics (0625/3) – Extended Theory May/June 2001

1 (a)(i) uniform acceleration 1


(ii) uniform speed / terminal velocity 1
2

(b) 20s 1
1

(c)(i) force of gravity / weight downwards 2


(ii) force of gravity / weight downwards and resistance / upthrust upwards 2
3 (max)

(d)(i) yes
(ii) no 1
1

(e) distance = area under graph / 0.5 x 5 x 48 1


= 120m 1
2

2 (a) momentum = mass x velocity / momentum = 800 x 20 1


= 16 000 1
kg m/s 1
2 (max)

(b)(i) momentum change = 16000 - 4000 1


force = 12 000/4 = 3000 N 2
(ii) deceleration = speed change / time 1
2
= 15/4 = 3.8m/s 2
6

3 (a) diagram showing balance and measuring cylinder 1


suitable labels 1
2

(b) balance reading for mass 1


two volume readings 1
2

(c) volume of stone from start and finish readings 1


density = mass / volume 1
2

4 (a) suitable random path 1


1

(b) air molecules hit particles 1


any two of:
molecules moving very fast / particles more massive / many hits
per particle / hits uneven 2
2 (max)

(c) any two of: bigger movements / faster movements / air molecules faster 2
2

5 (a) heat input = power x time / 50 x 600 1


temperature change for liquid = 45°C 1
heat gained by liquid = mass x s x temperature change / 200 x s x 45 1
s = 30 000 / 9 000 1

1
= 3.3 1
J/(g K) 1
5 (max)

(b)(i) any two of: heat lost to atmosphere / temperature rise less / bigger by
reference to equation 2
(ii) any one of: lagging / lid etc 1
3

6 (a)(i) any attempt shown to mark or measure critical angle 1


critical angle = 40° (not 42°) 1
(ii) incident angle at Q greater 1
o
>40 (> than candidate’s value i.e. e.c.f.) 1
4

(b) i = 32°, and r = 20° 1


R.I. = sin i / sin r 1
= sin 32° / sin 20° 1
= 1.55 1
4

7 (a) direction sound (wave) is travelling 1


1

(b)(i) pressure varies along sound wave / some particles closer than others 1
(ii) very few particles at P / P is a rarefaction 1
(iii) many particles at Q / Q is a compression 1
2 (max)

(c) vibrates / oscillates / moves backwards and forwards 1


along the direction of the wave 1
2

(d) 40 mm 1
1

8 (a)(i) current = power / voltage or = 0.6 / 1.5 1


= 0.4 A 1
(ii) resistance = voltage / current or 1.5 / 0.4 (e.c.f.) 1
= 3.8 ohm 1
(iii) charge = current x time or = 0.4 x 20 (e.c.f.) 1
=8C 1
5 (max)

(b)(i) two lengths in parallel 1


correctly shown on Fig. 8.2 1
(ii) two lengths in series 1
correctly shown on Fig. 8.3 1
3 (max)

(c) energy = l x l x R x t or V x l x t 1
= 0.6 x 0.6 x 5 x 20 1
= 36 J 1
3

9 (a)(i) any method of induction 1


any mention of magnetic field 1
changing / alternating 1
core transmits (changing) magnetic field from primary to secondary 1

2
(ii) no change in magnetic field 1
energy not transferred from primary to secondary 1
5 (max)

(b)(i) voltage ratio = turns ratio 1


2000 turns on primary 1
(ii) primary voltage x primary current = sec voltage x sec current 1
0.2 A 1
4

10 (a) each correct entry scores one mark 5


(b)(i) mass of nucleus (much) greater than alpha particle 1
after collision effect on alpha particle greater 1
(ii) positive charge 1
repulsion from the positive nucleus 1
(iii) most alpha particles pass straight through 1
most of atom space or nucleus much smaller than atom 1
5 (max)

3
Mark scheme for IGCSE Physics (0625/3) – Extended Theory October/November 2001
1 (a)(i) 10 m/s 1
(ii) 14 s 1
(iii) (distance is area under graph ) = 140 m 1
3

(b) deceleration = change in speed / time or 15 m/s in 8 s 1


2
= 1.9 m/s 1
2

(c)(i) arrow clearly towards centre 1


(ii) causes circular motion / prevents it going in a straight line 1
(iii) rails push on wheels / train or need force to produce acceleration 1
3

2 (a)(i) momentum = mass x velocity / 90 x 45 1


= 4050 kg m/s or Ns 1
(ii) average force = rate of change of momentum or force = ma or
= 4050/1.2 or 90 x 45/1.2 1
= 3380 N 1
4

(b) kinetic to heat (+ sound) 1


1

(c) k.e. = 0.5 x m v 1


0.5 x 90 x 2025 1
= 91 kJ 1
3

3 (a) ruler on pivot with one mass hanger on each side of the pivot 1
ruler, pivot and masses labelled 1
2

(b) any indication that masses and lengths from pivot measured 1
any indication of adjustment to achieve balance 1
2

(c) e.g. 100 g at 20 cm balances 50 g at 40 cm, two examples 2


one calculation e.g. 100 x 20 = 50 x 40 1
3

4 (a) (fast-moving) molecules hit M or each other 1


change of direction as a result of collisions stated or implied 1
2

(b) motion is random 1


movement keeps ‘doubling back’ so forward progress is slow 1
2

5 (a) energy needed to heat 1 g through 10 degrees = 10500/250 or energy


needed to heat 1 g through 100 degrees = 10500/25 = 420 J 2
energy needed to convert 1 g of water = 33900/15 = 2260 J 2
difference = 1840 J 1
5

(b) energy needed to separate the liquid molecules 1


because there are forces holding the molecules together 1
2

1
(c)(i) sensitivity, change in length / volume per degree or similar 1
(ii) range, lowest (temperature measured) to highest (large) or similar 1
(iii) linear scale, same distance between all degree intervals or similar 1
3

6 (a) names, refraction and diffraction 2


wavelength change, (smaller) and same / no change 1
frequency, same and same 1
4

(b)(i) (two) correct rays through lens 2


rays produced back to image 1
times bigger = 2 1
(ii) 1. Eye position suitable to view virtual image 1
2. magnifying glass or eyepiece 1
6

7 (a) A and B joined by (straight) line, all above the centre line 1
exterior loop A to B 1
arrow (internal) A to B and
arrow (external) B to A or one or more correct arrows on the loop 1
3

(b)(i) circle through C 1


arrow anticlockwise 1
(ii) lines can not touch or cross 1
(iii) 1. Strength same, direction opposite 1
2. Stronger field, same direction 1
5

8 (a) any use of W = V x I 1


X = 2.5 A; Y = 1.25 A
Z = 3.75 A (e.c.f.) 1
2

(b) attempt to use parallel resistance formula or Ohm’s law on full circuit 1
resistance = 64 ohm 2
3

(c)(i) total resistance = 288 ohm


current = 0.83 A 1
(ii) A, 80 V; B, 160 V 2
3

(d)(i) any point e.g. lamps require 240 V or voltage divided in series 1
one reference to values worked out by candidate 1
(ii) parallel circuit / switch in each line affects only 1 lamp etc 2
4

9 (a) connections correct 1


(b) 3.5 squares 1
1.4 V 1
(c) any sensible attempt e.g. takes less current / shows any variations
in value 1
1

10 (a) 24 and 12 on magnesium 1


0 and –1 on e 1
2

2
(b)(i) curve to positive 1
(ii) electron charge negative 1
negative attracted to positive 1
3

(c)(i) apparatus shown, beta source, detector / counter, paper in between 1


items above labelled 1
(ii) read detector, move paper and read again or use second sheet of paper 1
any change in reading means change in thickness 1
4

3
June 2003

INTERNATIONAL GCSE

MARK SCHEME

MAXIMUM MARK: 80

SYLLABUS/COMPONENT: 0625/03

PHYSICS
Paper 3 (Extended)
Page 1 Mark Scheme Syllabus Paper
IGCSE EXAMINATIONS – JUNE 2003 0625 3

1 (a) (i) force of gravity acts on masses/weight of masses B1


(ii) vector has direction/force has direction B1 2

(b) (i) spring 1 (more difficult) M1


any correct relevant pair of values A1
(ii) P marked at extension 25 mm to 28 mm A1
explanation in terms of end of proportionality B1
(iii) each graph read at 15 N, approx. 25 mm, 19 mm C1
difference correct, 6 mm +/- 1 mm A1 6
[8]

2 (a) change in speed is 1.5 m/s C1


deceleration = decrease in speed/time or 1.5/12 C1
a = (-/+) 0.125 m/s A1 3

(b) average speed = 1.75 m/s C1


distance = 21 m A1 2
[5]

3 (a) attempt to use triangle or parallelogram of forces M1


stated scale used A1
950 N and 1220 N in correct relative directions C1
correct resultant drawn in C1
weight = 1785 N [limits 1700 N to 1850 N] A1 5

(b) (i) work = force x distance or 1500 x 3.0 C1


work = 4500 J A1
(ii) power = work/time or 4500/2.5 C1
power = 1800 W A1 4
[9]

4 (a) air molecules hit dust particles M1


hits continuously/unevenly/hits cause movement in all
directions A1
air molecules fast moving/high energy B1 3

(b) any attempt to use p x v = constant or correct C1


proportion C1
fraction 2 x 80/25 seen A1 3
p = 6.4 x 10 (Pa)
[6]

5 (a) Y is a wire of different metal/not copper B1


Z is a galvanometer/millivoltmeter/miIIiammeter B1 2

(b) 2 junctions at different temperatures, accept one hot, B1


one cold
temperature difference causes e.m.f./voltage/current B1
reading of meter changes (with temperature) B1
1 junction at known temperature/need for calibration B1 max 3

(c) dull or black surface B1 1


[6]

© University of Cambridge Local Examinations Syndicate 2003


Page 2 Mark Scheme Syllabus Paper
IGCSE EXAMINATIONS – JUNE 2003 0625 3

6 (a) (i) incident ray, refracted ray and normal drawn C1


all correct and meeting at a point A1
(ii) angle of incidence and refraction correctly identified B1
(iii) values correct within agreed limits B1 4

(b) use of sini/sinr C1


correct substitution from candidates values C1
value correct within agreed limits from candidate's
values A1 3
[7]

7 (a) value 3 x 10 m/s A1 1

(b) speed of light (much) greater than speed of sound or


value for sound A1 1

(c) (i) source and receiver arrangement C1


with detail and labels A1
(ii) distance between source and receiver B1
time between flash and bang B1
(iii) speed = distance/time B1 max 4
[6]

8 (a) (i) use of charge = It or I = 90/45 C1


current = 2 A A1
(ii) resistance = voltage/current or 6/2 C1
resistance is 3 ohm A1
(iii) energy = Vit or Vq or 6 x 90 C1
energy is 540 J A1 6

(b) idea of energy transfer C1


is (6) J/C A1 2
[8]

9 (a) (i) power = VI or 24 X 2 C1


power is 48 W A1
(ii) voltage = power/current or 48/0.4 C1
voltage is 120 V A1 4

(b) (i) no/very little energy/power lost or energy/power in =


energy/power out B1
(ii) any mention of magnetic field B1
changing magnetic field B1
field passes through core or secondary coil B1
induces voltage in secondary coil B1
number of turns on secondary determines voltage
output B1 max 4
[8]

© University of Cambridge Local Examinations Syndicate 2003


Page 3 Mark Scheme Syllabus Paper
IGCSE EXAMINATIONS – JUNE 2003 0625 3

10 (a) (i) circular line of force around wire through P M1


arrow(s) on line anticlockwise - none wrong A1
(ii) arrow through Q to left A1 3

(b) (i) none/stays same B1


(ii) direction reverses B1 2

(c) at S - stronger B1
at T - same (strength) B1
at W - same (strength) B1 3
[8]

11 (a) (i) source, detector B1


named absorber/air and labels B1
(ii) take detector reading with no source (background) B1
detector reading with source, detector and air only B1
detector reading with appropriate named absorber
(including distance in air) B1
(iii) same reading with absorber(including air) as
background B1
so all alpha absorbed by cardboard/paper/air, others
would get through B1 max 6

(b) curved path stated or drawn B1


path at right angles to magnetic field B1
into paper B1 3
[9]

TOTAL 80

© University of Cambridge Local Examinations Syndicate 2003


November 2003

INTERNATIONAL GCSE

MARK SCHEME

MAXIMUM MARK: 80

SYLLABUS/COMPONENT: 0625/03

PHYSICS
Paper 3 (Extended)
Page 1 Mark Scheme Syllabus Paper
IGCSE EXAMINATIONS – NOVEMBER 2003 0625 3

1 (a) (i) 7(.0 s) A1


(ii) PQ or 0 – 2s or other correct description A1
distance = av. speed x time or area under graph C1
distance 11 x 2 m= 22 m A1 4
(b) (i) deceleration (now) uniform (test 2) B1
slower/lower (average) value/value between that of PQ and QR/takes longer B1
(or values) time to come to rest.
(ii) deceleration = change in speed/time or 15/8 C1
2
value = 1.9 m/s A1 4
(c) (i) graph shows constant acceleration B1
force = ma (and m is also constant) so force is constant B1
(ii) towards the centre of the motion/circle A1 3
[11]
2 (a) pressure = depth x g x density of water C1
pressure = 50 x 10 x 1000 C1
2
so value is 500 000 Pa or N/m A1 3
(b) force = pressure x area in any form C1
force = 500 000 x 0.15 x 0.07 C1
force = 5250 N A1 3
[6]
3 (a) one slightly nearer the centre than the other C1
20 kg is the nearer one to the pivot A1 2
(b) Clockwise moments = anticlockwise moments (about point/pivot) A1 1
(accept opposite directions and equal)
(c) 18x2.5=20xB C1
distance = 2.25(m) A1 2
[5]
4 (a) Some have extra/more energy than others B1
most energetic leave surface/ break liquid bonds etc B2 M2
(b) evaporation occurs strictly at the surface/at all temperature B1
boiling occurs throughout liquid/ at one temperature (at normal at. pr.)/100°C B1 2
(c) energy supplied = Wt /60 x 120 C1
sp.latent heat = energy/mass evaporated or 60 x 120/3.2 C1
value is 2250 J/g A1 3
[7]
5 (a) (i) nitrogen M1
(ii) copper-solid-molecules very tightly bonded together so separate little B1
water – liquid – molecules less tightly bonded/still small separation B1
nitrogen – gas – molecules “free” and not bonded so separate most B1 M3
(N.B. accept 2 bonding statements for 2 marks. 1 separation statement for 1 mark)

© University of Cambridge Local Examinations Syndicate 2003


Page 2 Mark Scheme Syllabus Paper
IGCSE EXAMINATIONS – NOVEMBER 2003 0625 3

(b) (i) size of movement/change in length of liquid column per degree B1


(ii) change in length (of liquid column) same for all degrees B1 2
[5]
6 (a) 3 more roughly circular B1
all drawn clearly circular, stop (well) clear of barrier and centred on slit B1
wavelength constant throughout, both sides of barrier B1 3
(b) wavelength – speed/frequency in any form C1
values substituted correctly C1
answer 6 x 10 m A1 3
[6]
7 (a) two dots, marked F, each 5.0 cm from the lens A2 2
(b) each correct ray one mark M2 2
(c) correct image, labeled I A1 1
(d) rays pass along the axis undeviated/object distance same for all object/rays meet at B1 1
same distance on image/image distance same for all image
(e) magnifying glass/eyepiece of telescope or microscope B1 1
[7]
8 (a) (i) 0-6 (V) positive and negative A1
(ii) all waves roughly 6V amplitude B1
3 waves approx. one wave every 0.1 s B1 3
(b) any mention of magnetic field B1
coils (forced to) cut magnetic field B1
includes e.m.f./voltage/current in the coils B1
as in Fleming’s R.H. rule B1 M3
(c) mechanical energy/work (in)/kinetic energy B1
electrical (out) (+ heat) (ignore sound) B1 2
[8]
9 (a) (i) regular (but)/not normal (sine) wave/several waves added together etc. B1
(ii) 1.6(V) A1
(iii) connect known voltage to Y plates (without any changes to C.R.O.) B1
read off against screen values B1 4
(b) (i) 6.1 (cm) (accept 6 or any value in range 6.0 to 6.2) A1
(ii) 50 ms for 10 cm or 5 ms per cm e.c.f. C1
so 6.1 x 5 ms or 31 ms A1
(iii) difference in time of runners finishing race or other timing between two closely B1 4
separated events.
[8]

© University of Cambridge Local Examinations Syndicate 2003


Page 3 Mark Scheme Syllabus Paper
IGCSE EXAMINATIONS – NOVEMBER 2003 0625 3

10 (a) current = power/voltage or 150/12 C1


value is 12.5 A A1 2
(b) (i) sum of currents at junction = current after junction/12.5 A = 5.0 A + I C1
value is 7.5 A A1
(ii) power = VI or is 7.5 x 12 e.c.f from (i) C1
value is 90 W A1
(iii) resistance = voltage/current or 12/7.5 e.c.f. from (i) but not from (a) C1
value is 1.6Ω A1 6
[8]
11 (a) top line correct, need 24 and 0 B1
bottom line correct, need 12 and –1 (accept β or e for electron B1 2

(b) particles take curved path (accept from diagram) B1


move between the poles at right angles to lines of force B1
move out of paper B1 3
(c) (i) use detector to pick up radiation (from isotope at points on/in body etc.) B1
high count where circulation good or v.v. explained B1
(ii) alpha particles all absorbed, none detected
beta particles may be largely absorbed, not penetrative enough
gamma rays reach detector/leave body any two B2 4
[9]
TOTAL 80

© University of Cambridge Local Examinations Syndicate 2003


June 2004

INTERNATIONAL GCSE

MARK SCHEME

MAXIMUM MARK: 80

SYLLABUS/COMPONENT: 0625/03

PHYSICS
Paper 3 (Extended)
Page 1 Mark Scheme Syllabus Paper
PHYSICS - JUNE 2004 0625 3

1 (a) (i) Acceleration / increase in speed M1


Uniform / constant or in a straight line A1
(ii) Uniform speed B1
Velocity changes / motion in a circle / accelerates B1 4
(b) Similarity: same value / 6m/s or velocity changing B1
Difference: opposite directions / up at E, down at C B1 2
(c) (i) Average speed x time / area under graph / 3 x 20 C1
60 m A1
(ii) 6 x 52 C1
312m A1 4
[10]

2 (a) 750 N A1 1

(b) p.e. lost / converted = mgh or weight x height C1


750 x 15 or 75 x10 x15 = 11250 (J) C1
p.e. lost = k.e. gained = 11250 (J) A1 3

(c) Any 3 of: heat in water / rock


(kinetic) energy of (moved) water / to make water move/
make waves
some k.e. still in (sinking) rock
sound energy on impact / of splash B3 3

(just heat and sound C1)


[7]
3 (a) (i) Extension proportional to load however expressed B1
(ii) Any relevant arithmetic to show direct proportion (or B1 2
straight line graph with values)
(b) (i) Work done = force x distance / 400 x 0.210 C1
84.0 J A1
(ii) (total) work/time or (24 x) 84/60 (apply e.c.f from (i) ) C1
33.6 W A1 4
[6]

4 (a) Water molecules at higher temps. have higher (av) k.e. B1


/ energy
Higher energy molecules (have greater chance to)
escape the surface B1
Higher energy molecules have energy to break liquid
“bonds” or separate liquid molecules or more
evaporation at 85°C (lowers level) B1 3

(b) Heat for evaporation = 34 500 – 600 = (33 900) C1


Sp. latent heat of evaporation = heat/mass evap. or
33 900 / 15 C1
2260 J/g (method and working correct, but no heat loss
used, 2/3) A1

(600 added or 34 500 used can score 2 max) 3

© University of Cambridge International Examinations 2004


Page 2 Mark Scheme Syllabus Paper
PHYSICS - JUNE 2004 0625 3

5 (a) (i) Thermopile / thermocouple / (blackened) thermometer /


infra red detector or use ammeter / voltmeter in supply
circuit B1
(ii) One of: same distance of plate to detector or use two
identical detectors or same time (after switching on) B1
(iii) Dull black better radiator / radiates more than silver / or
emits more heat / radiation B1
(iv) Infra red (i.r.) A1 4

(b) any correct example e.g. heating water or chimney M1


current clear and complete A1
direction shown correctly by arrows A1 3
[7]

6 (a) (i) Refraction at Q approx. correct, ray emerge from AB


parallel PQ B1
(ii) Angle of incidence correctly marked B1
Angle of refraction correctly marked B1

(can score even if incorrect / no refraction shown) 3

(b) (i) Refractive index = speed in air / speed in glass B1


(ii) Refractive index = (3 x 108 /2 x 108) = 1.5 B1 2

(c) (i) Wavelength = v/f or 3 x 108/6 x 1014 C1


Wavelength = 5 x 10-7 m A1 2
[7]

7 (a) C,R,C,R,C,R marked (or v.v.) along XY B1 1

(b) (i) Above normal / high air pressure or particles close B1


together
(ii) Below normal / low pressure or particles further apart B1 2

(c) Oscillation / vibration of particles / molecules (or


particles / molecules move to and fro) B1
Oscillation is along XY B1 2

(d) Time = distance / speed or (2x) 50/340 C1


Time = 0.29 s A1 2

© University of Cambridge International Examinations 2004


Page 3 Mark Scheme Syllabus Paper
PHYSICS - JUNE 2004 0625 3

8 (a) 1.52 kW A1 1

(b) (i) Each appliance is connected across 240 V supply or


equivalent B1
(ii) Any 2: all work on same voltage or on 240 V or mains
OR all have full/stated power OR each can be on or off
OR one goes off/breaks others stay on B2 3

(c) (i) Current = power/voltage or 200/240 C1


Current = 0.83 A A1
(ii) Energy = power x time or 1.2 x 3 C1
Energy = 3.6 kWh or 1.3 x 107 J A1
(iii) Current = 60/240 C1
R= V/I or 240/0.25 C1
R =960Ω A1 7
[11]
9 (a) Solenoid ends connected to meter, both labelled B1
One magnet in correct position to enter / leave
solenoid, labelled B1 2

(b) Push magnet into coil / pull out / move near end of coil B1 1

(c) (magnet has / produces) magnetic lines of force /


magnetic field B1
lines cut (coils of) solenoid / coils / wires B1 2

(d) (i) Pull magnet out of coil / reverse effect to answer (b) B1
(ii) Move magnet faster or effect in (a) faster B1 2
[7]

10 (a) Analogue, continuously increasing / decreasing


readings B1
Digital, readings increase / decrease by one unit B1 2

(b) (i) Transistors + other components such as resistors B1


(ii) Standard symbol, must have labeled inputs and output B1
(iii) Both inputs 0 (off), or either one input 0 (off), output 0
(off) B1
Both inputs 1 (on), output 1 (on) B1 4
OR correct truth table drawn (C1)
Some explanation of what truth table shows (A1)
[6]

11 (a) Particle 1 carries straight on B1


Particle 2 (slightly) deflected (less than 90°) B1
Particle 3 “turns back” / (deflected more than 90°) B1 3

(b) Nucleus is heavy /dense / all or most of mass in atom in


nucleus B1
Most of atom is space or nucleus is (very) small
cf. atom B1 2

(c) (mass) 4 B1 1
[6]

© University of Cambridge International Examinations 2004


Page 4 Mark Scheme Syllabus Paper
PHYSICS - JUNE 2004 0625 3

PAPER TOTAL = [80]

© University of Cambridge International Examinations 2004


November 2004

INTERNATIONAL GCSE

MARK SCHEME

MAXIMUM MARK: 80

SYLLABUS/COMPONENT: 0625/03
PHYSICS
(Extended Theory)
Page 1 Mark Scheme Syllabus Paper
IGCSE – November 2004 0625 3

1 (a) deceleration/slows down/speed reduces 1


deceleration uniform/comes to rest at 4 s 1 2

(b) (i) 40 (m/s) 1


(ii) 4 (s) 1 2
(c) speed falls from 0 to 40 m/s in 4 s 1
acceleration = change in speed/time taken or 40(m/s)/4(s) 1
acceleration = 10 m/s2 1 3

(d) distance = average speed x time or area of triangle under 1


graph
= 20 x 4 or 2 x 40 1
= 80 m 1 3
(10)

2 (a) pressure = hdg or 20 x 1000 x 10 1


= 2 x 105 Pa 1 2

(b) force = pressure x area or 2 x 105 x 0.5 e.c.f. 1


= 1 x 105 N 1 2

(c) potential energy (at water surface) 1


changed to kinetic energy (at pipe exit) 1 2
(6)

3 (a) one mark for each labelled diagram


both diagrams sensible but no labels max 1 2 2
(b) newtons/10 is kg or equivalent 1 1

(c) volume/level/reading of water then volume etc. water + rock 1 1

(d) difference in the two readings 1 1

(e) density = mass/volume 1 1


(6)

4 (a) (i) put hot junction in beaker (of hot water) 1


read temperature from galvo. in some way (calibration) 1 2

(ii) high/low temperatures stated or high/low values quoted or


temperature varying rapidly or small site/at point or remote
place (from meter) or in control systems any 2 2 2

(b) (i) raises the water temperature 1

(ii) provides latent heat or boils/evaporates water 1 2


(6)

© University of Cambridge International Examinations 2005


Page 2 Mark Scheme Syllabus Paper
IGCSE – November 2004 0625 3

5 (a) (i) any suitable random motion 1


molecules hit walls 1

(ii) 1.
rebound/bounce back or many hits per unit area or per unit
time or collisions create force 1
2.
(av) k.e./speed of molecules increases 1
more hits(/sec) or harder hits 1 5

(b) p 1 v 1 = p 2 v 2 quoted or any recognisable substitution 1


2 x 105 x 0 . 3 5 = 5 x 1 0 5 x v 1
volume = 0.14 (m3) 1 3
(8)
6 (a) expect two internal reflections at sensible angles 1 1
(b) angle of incidence at Y greater than critical angle 1
total internal reflection occurs 1 2

(c) (i) frequency = velocity/wavelength or 1.9 x 108/3.2 x 10-7 1


= 5.9 x 1014Hz 1

(ii) refractive index = 3/1.9 or 1.9/3 1


= 1.58 (no e.c.f.) 1 4
(7)
7 (a) I = V/R or 12/8 1
= 1.5 A 1 2

(b) (i) 1
10(Ω)

(ii) 2(Ω) 1 2

(c) power = VI or I2R or V2/R 1


= 72W 1 2

(d) (i) 12(V) 1

(ii) 6(V) 1 2

(e) (i) (resistance) less 1

(ii) (resistance) less 1 2


(10)
8 (a) diffraction 1 1

(b) plane waves in front of gap 1


curved end effect shown, reasonable curves 1
wavelength constant throughout and approximately same
as in Fig. 8.1 1
good quality i.e. end effect starts at correct points 1 4
(c) particles/water oscillate/vibrate/move up and down 1
at right angles to wave direction 1 2

(7)

© University of Cambridge International Examinations 2005


Page 3 Mark Scheme Syllabus Paper
IGCSE – November 2004 0625 3

9 (a) (i) two coils on continuous core (not allow coils joined) 1
primary coil to 240 V, secondary coil to 6 V 1
iron core, primary/input and secondary/output labelled 1

(ii) any values with correct 40:1 ratio, accept here or on


diagram 1 4

(b) power in = power out or 240 x I = 12 1


current = 0.05 A 1 2

(c) must be a changing magnetic field, only from a.c. 1 2


so that induction can take place 1 (8)

10 (a) (i) switch, relay or amplifier 1

(ii) any one of the three versions below, each 2 marks

1. vary base current 1


transistor switches on for Vbe >0.6 V 1

2. small change in base current 1


produces a large change in collector/emitter current 1

3. vary potential divider connected to transistor base 1


transistor switches on for Vbe > 0.6 V 1 3

(b) (i) standard symbol with 2 inputs and an output labelled 1

(ii) one or both inputs 1, output 1 (accept on, high for 1) 1


both inputs 0, output 0 (accept off, low for 0) 1 3
(6)
11 (a) correct equation i.e. Ra gives Rn + alpha particle or He 1
all numbers correct on Rn and He 1 2
(b) (i) radiation from surroundings/background radiation 1
(ii) 532 to 552 counts/min 1

(iii) 5/6 cm 1

(iv) beyond 5/6 cm no alpha, only background radiation 1 4


(6)

© University of Cambridge International Examinations 2005


June 2005

IGCSE

MARK SCHEME

MAXIMUM MARK: 80

SYLLABUS/COMPONENT: 0625/03
PHYSICS
Extended
Page 1 Mark Scheme Syllabus Paper
IGCSE – JUNE 2005 0625 3

1 (a) acceleration, speed increases B1


acceleration getting less B1
acc. zero/constant speed along RT or terminal velocity B1 3

(b) air resistance or friction (force) up (accept upthrust) B1


weight/(force of) gravity down B1 2

(c) air resistance (up) = weight (down) or two forces equal B1


no (net) force, no acceleration B1 2

(d) (i) distance = speed x time or 120 x 40 C1


distance = 4800 m A1
(ii) distance = average speed x time or 25 x 6 or area under graph C1
distance = 150 m A1 4
[11]

2 (a) time a number of swings (if number stated, >5) M1


time divided by [2 x number of swings] A1 2

(b) (i) weight of gravity and tension B1


(ii) force towards centre of circular motion or towards support point B1 2

(c) p.e. = mgh or 0.2 x 10 x 0.05 C1


= 0.1 J A1 2
[6]

3 (a) in a straight line or (vector) has direction B1 1

(b) f = ma or f = 3.0 x 2.0 C1


= 6(.0) N A1 2

(c) P = F/a or P = 120/0.05 C1


= 2400 N/m2 (or Pa) A1 2
[5]

4 (a) start temp. and final temp. or change in temperature B1


mass of iron B1
time heater on B1 3

(b) P x t, VIt or in words B1


= m x shc x cit or words B1 2

(c) (i) heat lost to surroundings/air B1


(ii) add lagging/insulate B1 2
[7]

© University of Cambridge International Examinations 2005


Page 2 Mark Scheme Syllabus Paper
IGCSE – JUNE 2005 0625 3

5 (a) air molecules hit particles or vice versa B1


air molecules have speed/moment/energy B1
hits uneven or from all directions B1
hits (by small molecules) can move a large particle or moves
particles small distances B1 4

(b) (i) most energetic/fastest molecules B1


(ii) need energy to overcome forces/break bonds/separate mols. B1
so work must be done/energy used as work B1 3
[7]

6 (a) along normal or angle i = 0 so angle r = 0 B1 1

(b) speed reduced, wavelength reduced, frequency unchanged


any two correct scores one mark B1
third correct scores second mark B1 2

(c) reflected at 30o B1


refracted at > 30o B1 2

(d) sin 30o/sin r = 0.67 C1


sin r = sin 30o/0.67 C1
r = 48o A1 3
[8]

7 (a) (i) x-rays or gamma rays B1


(ii) infra red or radio B1 2

(b) f = v/ λ or 3 x 108/ 1 x 10-12 C1


= 3 x 1020 Hz A1 2

(c) 3 x 108 m/s 1 1


[5]

8 (a) circuit which would work with supply and resistor B1


voltmeter in parallel and ammeter in series with resistor B1
variable resistor in series or means of changing p.d. across
resistor B1 3

(b) read ammeter and voltmeter B1


adjust rheostat/supply B1 2

(c) (i) I = V/R or V = IR or R = V/I or 0.5 = 6.0/3.0 + R C1


R = 9(.0) Ω A1
(ii) 60 C B1
(iii) P = VI or = I2R or P = v2/R or (0.5 x 3.0) x 0.5 C1
= 0.75 W A1 5
[10]

© University of Cambridge International Examinations 2005


Page 3 Mark Scheme Syllabus Paper
IGCSE – JUNE 2005 0625 3

9 (a) (i) to change a.c. to d.c. or rectify (a.c.) B1


(ii) full sine wave at least 1.5 full waves B1
half wave rectified at least two d.c. ‘bumps’ B1 3

(b) (i) correct symbol B1


(ii) when input high or 1, output low or 0 or off B1
when input low or 0 or off, output high or 1 or on B1 3
[6]

10 (a) 8 (mins) for value, no working shown B1


8 (mins) for value with suitable working or indication on graph B1 2

(b) (i) source, aluminium and detector, recognisable shapes B1


quality and all labels correct B1 2
(ii) count background B1
source and detector, no absorber, count taken B1
source, absorber and detector, count taken B1 3
[7]

11 (a) magnetic field and current at right angles causes B1


force on wire which deflects it B1 2
or
field around wire (B1)
interacts with the field of the magnet (B1)

(b) normal to/between poles, either way however expressed C1


out of paper A1 2

(c) converts electrical energy to work/k.e./movement energy B1 1

(d) (i) split rings and brushes or equivalent (e.g. leaning wires) B1

(ii) every half turn current passes from one ring to the other B1
so current flows opposite way around coil or commutates B1 3
[8]

© University of Cambridge International Examinations 2005


Page 1 Mark Scheme Syllabus Paper
IGCSE – November 2005 0625 3

1 (a) force of gravity on a mass or mg B1


mass/volume B1 [2]

(b) (i) hang object from spring balance, reading in N taken B1


divide reading in N by 10 or g B1

(iii) volume of water in cylinder or fill overflow can to top B1


add object find increase in volume or measure overflow B1 [4]
volume
{no credit for mass unless not scored in (i) and no credit for
density = mass/ volume unless not scored in a) }

(c) (i) 2N B1
left B1

(ii) F = ma or 2 = 0.5 a C1
a = 4.0 m/s2 A1 [4]
Total [10]

2 (a) upwards force = downwards force or no resultant force B1


opposing moments equal or A.C.M. = C. M. B1 [2]

(b) 30 x spring balance reading = 40 x 6.0 or equivalent C1


spring balance reading = 8.0 N A1 [2]

(c) 0.5 N B1
downwards B1 [2]
Total [6]

3 (a) P = hdg or 2 x 1000 x 10 C1


= 20 000 N/m2 or Pa A1 [2]

(b) p = f/a or 20 000 = 50/a C1


a = 0.0025 m2 A1 [2]

(c) potential energy of the water B1


converted to kinetic energy of water through outlet (and B1 [2]
heat) Total[6]

4 (a) turn on heater and wait until water starts dripping in beaker B1
empty beaker & replace, start watch B1
stop watch & remove beaker at same time B1
record time B1
find and record mass of water in beaker B1 [M4]

(b) 60 x t = 120 x 340 C1


t = 680 s A1 [2]

(c) (i) ice gains heat from surroundings/ice falls through funnel B1

(ii) lag or fit lid to funnel/place gauze in funnel bottom B1 [2]


Total [8]

© University of Cambridge International Examinations 2005


Page 2 Mark Scheme Syllabus Paper
IGCSE – November 2005 0625 3

5 (a) (i) random B1

(ii) hit and rebound B1 [2]

(b) (i) increase or further apart B1

(ii) increase or move faster B1 [2]

(c) (i) random, fast in gas to vibration in solid B1

(ii) long way apart in gas to very close or touching B1 [2]


Total [6]

6 (a) Sound reflects off wall B1 [1]

(b) 400 Hz B1 [1]

(c) λ = v/f or = 330/400 C1


= 0.83 m A1 [2]

(d) vibration/oscillation along line of/direction of wave B1 [1]


Total [5]

7 (a) (i) two approximately correct reflections B1


evidence of projecting back to image or use of equal
distance from the mirror, object and image B1

(ii) virtual B1
any one of upright, same size, same distance from mirror B1
[4]
(b) (i) ray 1 correct B1
ray 2 correct B1
image correctly located B1

(ii) eye symbol to right of lens B1


[4]
Total [8]

8 (a) force is produced B1


on any charge placed in the field B1 [2]

(b) at least 3 parallel, straight lines plate to plate, ignore end B1


effect
at least one correct arrow, none wrong B1 [2]

(c) q = It or 0.06 = I x 30 C1
I = 0.002 A or 2 mA A1 [2]

(d) E = Vit C1
= 1500 x 0.008 x 10 C1
= 120 J A1 [3]
Total [9]

© University of Cambridge International Examinations 2005


Page 3 Mark Scheme Syllabus Paper
IGCSE – November 2005 0625 3

9 (a) correct symbol B1


correct labels B1 [2]

(b) (i) low, OFF or 0

(ii) low, OFF or 0 need both correct B1 [1]

(c) (i) need 4 boxes correct for 2 marks, -1 for e.e.o.e. B2

(ii) no change B1 [3]


Total [6]

10 (a) (i) a.c. input causes constantly changing current through coil B1
magnetic field formed in or around coil B1
constantly changing magnetic field B1 [M2]

(ii) (changing) magnetic field transferred to secondary coil B1

(iii) (changing) magnetic field cuts secondary coil B1


induces e.m.f. B1 [3]

(b) more turns on secondary (than on primary) B1 [1]

(c) no transfer of magnetic field from primary to secondary B1 [1]

(d) Vp.Ip = Vs. Is or 100 x 0.4 = 200 x Is C1


Is = 0.2 A A1 [2]
Total [9]

11 (a)  β -source and detector suitably arranged B1


deflecting plates suitably arranged B1
additional detail e.g. slit or collimator, vacuum chamber,
circuit connected to deflecting plates B1 [3]

(b) at least 3 readings at right angles beyond & perp. to the M1


plates [2]
one near +ve, one near –ve and one in centre A1

(c) highest reading near +ve plate B1 [1]

(d) electrons negatively charged, attracted to +ve B1 [1]


Total [7]

© University of Cambridge International Examinations 2005


Page 2 Mark Scheme Syllabus Paper
IGCSE – May/June 2006 0625 03

1 (a) point 8,12 identified B1


straight line joining 0,0 and 8,12 B1
straight line joining 8,12 and 20,12 B1 3

(b) acceleration = change in v/change in t or 12/8 etc C1


= 1.5 m/s2 A1 2

(c) distance = area under graph between t = 20 and t = 25 C1


= 24 m to 28 m A1 2

(d) F = ma or 4000 x 1.2 C1


= 4800 N A1 2

(e) more passengers got on (so mass increased)


driver pressed accelerator less (so force decreased)
more traffic or going uphill any two lines B2 2
[11]

2 any closed triangle or parallelogram C1


forces in correct directions relative to each other C1
correct resultant indicated C1
resultant 7.7 N to 8.1 N A1 4
scale stated B1
resultant vertically upwards B1 2
[6]

3 (a) work = force x distance C1


= force of gravity/weight x (vertical) distance/height A1 2

(b) (i) work = (100 x 8) = 800 J A1


A1 2
(ii) power = (800/5) = 160 W

(iii) increases the k.e. of the water (ignore heat/sound) B1 1


[5]

4 (a) on surface/throughout; no bubbles/bubbles; all temps./b.p.;


s.v.p. < at. pressure; svp = at. pressure any two B2 2

(b) energy/work to separate molecules B1


(against) forces of attraction between water molecules B1 2
(to break bonds C1)
The k.e./speed of the molecules does not increase B1 1

(c) Wt = mL or 120 x 1 = 0.05 x L C1


L = 120/0.05 C1
L = 2400 J/g A1 3

[8]

© University of Cambridge International Examinations 2006


Page 3 Mark Scheme Syllabus Paper
IGCSE – May/June 2006 0625 03

5 (a) increase surface area of tank B1


blow air over surface/put in windy place B1 2

(b) (i) capillary tube longer or liquid with lower expansivity B1

(ii) capillary tube thinner/finer or liquid with higher expansivity


or bigger bulb B1 2

(c) p1v1 = p2v2 or 1 x 105 x 150 = p2 x50 C1


p2 = 3 x 105 (Pa) A1 2
[6]

6 (a) red ray refracted away from normal B1


violet ray refracted more than red ray in prism B1
violet ray further refracted from red ray to screen B1 3

(b) 1.52 = sin 40o/sin r M1


sin r = sin 40o/ 1.52 ( = 0.423) C1
r = 25o A1 3

(c) (i) 3 x 108 m/s A1

(ii) same as (i) A1 2


[8]

7 (a) Longitudinal or pressure waves B1 1

(b) a correct C marked B1


a correct R marked B1 2

(c) oscillation/vibration/backwards and forwards M1


along PY (consider pressure waves as alternative) A1 2

(d) wavelength = 340/200 C1


PX(= λ /2) = 0.85 m A1 2
[7]

8 (a) I = W/V or 9/6 C1


I = 1.5 A A1 2

(b) (i) 8 ohm A1

(ii) 6V A1 2

(c) (i) brightness decreases/dimmer B1

(ii) resistance of circuit greater B1


current through lamp falls B1 3

(d) (i) 4 ohm A1

(ii) 4 ohm A1 2
[9]

© University of Cambridge International Examinations 2006


Page 4 Mark Scheme Syllabus Paper
IGCSE – May/June 2006 0625 03

9 (a) primary and secondary coils on iron core labelled B1


240 V a.c. to primary, 12 V a.c. to secondary B1
turns ratio shown or stated 20:1, stepdown B1 3

(b) (i) must be constantly changing magnetic field B1

(ii) magnetic field of primary passes through core to secondary B1


magnetic field of secondary cuts coil, induces output B1 3

(c) (i) 18 W A1

(ii) 540 J A1 2
[8]

10 (a) bring rod close but not touching plate M1


touch metal plate with earth lead M1
remove lead and then rod A1 3

(b) (i) Q = 20 (mA) x 15 (s) C1


= 0.30 C A1

(ii) V = 20 (ma) x 10 (kΩ) C1


= 200 V A1 M3
[6]

11 line1 into paper B1


positive or +2 B1
line 2 out of paper or opposite of line 1 B1
negative or -1 B1
line 3 no deflection B1
no charge B1 6
[6]

© University of Cambridge International Examinations 2006


Page 2 Mark Scheme Syllabus Paper
IGCSE - OCT/NOV 2006 0625 03

1 (a) (i) t = v/g or 32/10 C1


= 3.2 s A1

(ii) straight line starting at zero, inclined C1


line joining 0,0 and 3.2, 32, accept c.f. from time (i) A1

(iii) 2.4 kg A1 [5]

(b) (i) take volume of water before use B1


(totally) immerse stone and take new volume B1
(Not clearly measured before and after C1)

(ii) hang rock from balance and take reading B1

(iii) density = mass/volume B1

(iv) need to tie "sinker" or cork or press cork down B1


need volume with sinker then volume with sinker and cork or just completely submerge
cork B1 [6]

[Total: 11]

2 (a) limit of proportionality (allow elastic limit) B1 [1]

(b) force is proportional to extension or in terms of doubling B1 [1]

(c) (up to Q extension proportional to force applied)


Q to R extension/unit force more however expressed B1 [1]

(d) k = force/extension or 8/2 or other correct ratio C1


= 4.0 N/mm A1 [2]

[Total: 5]

3 (a) p.e. lost = mgh or 1 x 10 x 7 C1


= 70 J A1 [2]

(b) 70 = 0.5 x m x v2 or ecf C1


v2 = 140 or 2 x p.e. C1
v = 12 m/s A1 [3]

(c) some p.e. changed to heat/sound/either one/work done against air resistance air/resistance acts
against the motion B1 [1]

[Total: 6]

4 (a) (i) 1 is 20°C


2 is 15 ± 1°C, need both correct for a mark A1

(ii) more heat lost at higher temperature B1 [2]

(b) heat in = 60 x 210 or Wt or 12 600 (J) C1


heat in water = m x s x ∆θ or 75 x s x 40 C1
s = 12600/75 x 40 C1
= 4.2 J/g °C A1 [4]

(c) outline correct, two wires with clear junction and a meter/datalogger/computer M1
labels, hot and cold junctions or clear, two different metals A1 [2]

[Total: 8]

© UCLES 2006
Page 3 Mark Scheme Syllabus Paper
IGCSE - OCT/NOV 2006 0625 03

5 (a) (i) conduction B1

(ii) particles/atoms/ions vibrate or electrons move and carry energy B1


pass on energy from one particle to the next B1
[3]
(b) four surfaces facing one heat source B1
suitable detector e.g. thermometer behind surface-read all 4 B1
precaution e.g. equal distance/time B1
(Can not score last two marks if experiment is totally wrong) [3]

[Total: 6]

6 (a) completed path B1 [1]

(b) any two correct, -1 each incorrect


virtual, inverted, same size as object B2 [2]

(c) angle of incidence zero/at right angles/along normal B1 [1]

(d) 1.5 = Va/Vg = 3x 108/Vg


C1
Vg = 2 x 108 m/s A1 [2]

(e) angle of incidence = 45°, so angle of reflection = 45°, so ray turns through 90°
OR angle i> angle c B1
so totally internally reflects B1 [2]

[Total: 8]

7 (a) straight not circular or WTTE


waves not same wavelength/same distance apart
waves should extend into shadow area (more) any 2 B2 [2]

(b) diagram showing large flat piece M1


with circular edges (ignore any wavelength changes) but straight part must be (very) nearly
equal to slit width A1 [2]

(c) speed = 1.2 x 8 C1


= 9.6 cm/s A1 [2]

[Total: 6]

8 (a) switch in correct position B1 [1]

(b) (i) rheostat/variable resistance symbol drawn B1

(ii) dot and R in line to 12 W lamp B1 [2]

(c) Question deleted

(d) R = V/I or 12/.3 C1


= 4Ω A1 [2]

(e) (i) parallel circuit/all lamps connected separately across the 12V B1

(ii) 4A A1 [2]

[Total: 7]

© UCLES 2006
Page 4 Mark Scheme Syllabus Paper
IGCSE - OCT/NOV 2006 0625 03

9 (a) (i) connections one to each plate M1


top one to +ve , bottom one to -ve A1 [2]
(New PSU drawn C1)

(ii) electrons negatively charged B1


one plate positively charged, one negatively charged B1
electrons attracted to +/repelled by – B1 [3]

(b) (i) time base applied to X plates stated or described B1

(ii) a.c. or varying voltage applied to Y plates B1 [2]

(c) 2 full waves, (equal about centre line) B1 [1]

[Total: 8]

10 (a) A – resistor B – LDR C – transistor D – lamp (–1 each incorrect) B2 [2]

(b) C B1 [1]

(c) resistance of LDR low in light, high in dark B1


increase of resistance/potential in circuit cause transistor to conduct (Vbe > 0.6 V) B1
switches lamp on B1 [3]

[Total: 6]

11 (a) (i) atoms interact with by particle/photon not radiation B1


electron(s) removed to form ions B1

(ii) much greater mass or size/slower speed/more ion pairs/cm/larger charge B1 [3]

(b) (i) any 2 correct B2

(ii) e.g. foil thickness described/outline diagram B1


foil too thick less reading/notes on diagram to show method B1
other examples will occur, must have two clear points:
e.g. 1. gamma rays aimed at cancer (not just radiation)
focused on tumour
e.g. 2. fission of heavy nucleus (accept named nuclide)
leads to more fissions/chain reaction [4]

[Total: 7]

© UCLES 2006
Page 3 Mark Scheme Syllabus Paper
IGCSE – May/June 2007 0625 03

1 (a) (i) straight arrow towards centre, by eye B1 [1]

(ii) force larger B1 [1]

(b) (i) straight arrow along tangent at P clockwise, by eye B1 [1]

(ii) friction between tyres and track provide centripetal force B1

friction too small (to provide required force) B1 [2]

(c) (i) constant speed/velocity OR uniform motion OR no acceln. B1 [1]


NOT constant motion

(ii) (3 × 25)/2 + (7 × 25) OR area under graph C1

212.5 cm any no s.f. ğ 2 A1 [2]

(iii) 25/3 or increase in speed/time C1

8.33 cm/s any no s.f. ğ 2 OR 8⅓ cm/s accept cm/s2 A1 [2]

[Total: 10]

2 (a) moment of W down/anticlockwise, moment of steam opposite C1

when moment of steam > moment of W, steam escapes


OR when clockwise moment > anticlockwise moment, steam escapes A1 [2]

(b) (i) 12 = 0.2 F C1

F = 60 N c.a.o. allow 60–61 for ans if working for 60 N shown A1 [2]

(ii) (P =) F/A or 60/0.0003 e.c.f. C1

2 × 105 Pa or 200 000 Pa e.c.f. (accept N/m2) OR 20 N/cm2 A1 [2]

[Total: 6]

© UCLES 2007
Page 4 Mark Scheme Syllabus Paper
IGCSE – May/June 2007 0625 03

3 (a) (i) work done = force x dist or 600 x 3 or 60 x 3 or fd or mgh C1

work = 1800 J c.a.o. accept j or Nm for unit A1 [2]

(ii) power = work/time or 1800/12 e.c.f. C1

power = 150 W e.c.f. accept J/s or NM/s for unit A1 [2]

(b) P.E. decreases/transformed (ignore mention of KE) C1

all the decrease becomes heat (ignore mention of sound) A1 [2]

[Total: 6]

4 (a) total mass before ice added B1

total mass after all ice melted B1 [2]

(b) (i) mass × sp ht cap × change in temp or 20 OR mcθ B1 [1]

(ii) mass (of melted ice) × sp latent ht OR ml


OR (heat gained by ice) = heat lost by water B1 [1]

(c) heat/mass or 12 800/30 C1

427 J/g OR 426667 J/kg any no s.f. ğ 2 A1 [2]

(d) heat gained from surroundings OR no lagging B1


heat needed to cool beaker/stirrer and thermometer ) any 2 +
too much ice added or similar point ) B1 [2]
allow stirring gives energy, allow evaporation/condensation
(ignore “mistakes when taking readings” or similar)

[Total: 8]

© UCLES 2007
Page 5 Mark Scheme Syllabus Paper
IGCSE – May/June 2007 0625 03

5 (a) (i) heat for the same time B1

take temps on both thermometers B1 [2]

(ii) dull black box temp > white box temp OR black is hotter etc. B1 [1]

(b) (i) large expansion/change in reading for small change in temp B1 [1]
NOT detect/respond to small temp changes

(ii) temperature rise small and/or small difference between them B1 [1]

(iii) distance between each degree on scale is the same B1 [1]

[Total: 6]

6 (a) (i) refracted ray, angle < i, emergent ray approx parallel to incident B1

(ii) reflected ray at equal angle to incident, by eye B1 [2]

(b) (i) 88–90° B1 [1]

(ii) 43° c.a.o. B1 [1]

(iii) n = sin (his90°)/sin (his43°) C1

1.466 or 1.47 or 1.5 c.a.o. any no s.f. ğ 2 A1 [2]

(c) n or his 1.5 = speed in air/speed in glass e.c.f. C1

speed in glass = 2(.0) × 108 m/s e.c.f. any no s.f. ğ 2 A1 [2]

[Total: 8]

© UCLES 2007
Page 6 Mark Scheme Syllabus Paper
IGCSE – May/June 2007 0625 03

7 (a) source of sound (e.g. gun/hooter), tape (100 m), stopwatch B1 [1]
NOT clock, metre rule (unless lab method)

(b) distance and time between “flash and bang” (must be clear) B1 [1]

(c) distance/time OR d/t OR 2d/t B1 [1]

(d) further apart/more accurate timer/repeat/any other B1 [1]

(e) speed of sound in air, tick 100 B1

speed of sound in water, tick 1000 B1 [2]

[Total: 6]

8 (a) connections such that all lamps will light B1

ammeter in correct position B1

variable resistor in correct position (condone poor symbol) B1

switch in appropriate position (could be 2 switches) B1 [4]

(b) (i) 3 A B1 [1]

(ii) 4Ω OR 12/his(i) correctly evaluated B1 [1]

(iii) 2Ω OR ½ × his(ii) correctly evaluated B1 [1]

(iv) 1080 J e.c.f. from (i) & (ii) if working shown B1 [1]

(c) lamps in series M1

less current/less p.d. (across 1 lamp)/voltage shared/higher resistance A1 [2]


NOT current shared
[Total: 10]

© UCLES 2007
Page 7 Mark Scheme Syllabus Paper
IGCSE – May/June 2007 0625 03

9 (a) current in spoke in magnetic field B1

causes force on spoke/wheel B1 [2]

(b) arrow to indicate anticlockwise motion B1 [1]

(c) outline of coil, pole pieces B1

d.c. supply connected to brushes B1

split rings connected to coil B1 [3]

(d) brushes connect to other split ring every half turn/coil vertical B1

reverses direction of current every half turn/coil vertical B1 [2]

[Total: 8]

10 (a) when temperature rises resistance falls (or v.v.) M1

p.d. across it falls or equivalent (or v.v.) A1

idea of causes transistor to switch on lamp (or lamp off) A1 [3]

(b) change value of R1/use variable res/swap R1 with something B1

brief explanation in terms of potential divider B1 [2]

(c) fire alarm/refrigerator fail light/other automatic lighting system B1 [1]

[Total: 6]

11 (a) A doubles back, either side B1

B carries on, slightly deflected B1

C carries straight on B1 [3]

(b) only (very) few scattered through large angles B1

most pass undeviated so most of atom space B1

scattering/deflection/repulsion due to concentrated


mass/charge/charge/nucleus B1 [3]

[Total: 6]

© UCLES 2007
Page 3 Mark Scheme Syllabus Paper
IGCSE – October/November 2007 0625 03

1 (a) (i) 1.6s to 1.8s ALLOW 4.2 – 6s ALLOW 4.4 – 6s NOT 2s NOT 4.0 – 6s B1

(ii) 6 – his (i), evaluated ALLOW 0 – 4.2s ALLOW 0 – 4.4s NOT 0 – 4s e.c.f. B1

(iii) his (i) × 20 C1


32 – 36m or his (i) × 20 evaluated
allow B1 only for 40m with no working A1

(iv) area under whole graph or ½vt + his(iii) C1


70 – 95m A1

(b) (i) weight of ball down and (air) resistance up


OR friction opposes weight )
upward/resistance/friction force increases
with time/distance/speed/as ball falls ) any 3 B1×3
net force reduces )
less force, so less acceleration )

(ii) up force = down force OR no resultant force OR air res. = weight B1


no net force, no acceleration/constant speed B1

[Total: 11]

2 (a) (i) down to R and up towards Q/S, then reverse OR equivalent


OR back towards Q, then reverse B1
continues backward and forward until stops (at R) B1

(ii) idea of energy loss OR because of friction NOT PE/KE B1

(b) (PE lost =) 1.2 × 0.5 OR 0.6 (J) OR 0.12 × 10 × 0.5 OR mgh OR wt × dist C1
i.e. evidence of mgh

0.5 × 0.12 × v2 = mgh OR 0.6 etc. e.c.f. C1


i.e. evidence of ½mv2

3.16 OR 3.2 m/s c.a.o. A1

[Total: 6]

© UCLES 2007
Page 4 Mark Scheme Syllabus Paper
IGCSE – October/November 2007 0625 03

3 (a) any logical method e.g.


extension is 2 cm for 8 N or 1 cm for 4 N C1
final extension is 3 cm C1
need 12 N to extend to 6 cm A1

(b) (i) shown on diagram:


distance from pivot to F OR value of weights OR dist from weights to pivot B1

(ii) force/weight of load × distance from pivot to force


(accept symbols if clear) B1

[Total: 5]

4 (a) (i) random B1


high speed (between collisions) B1

(ii) hit walls B1


many hits/unit area OR hit hard OR large force OR high energy
OR many hits/s OR hit very often B1

(b) particles vibrate (more) OR electrons gain energy B1


particle to particle transfer OR flow of free electrons B1

(c) 75 × 3200 OR ml C1
240 000 J OR 240 kJ OR 2.4 × 105J A1

[Total: 8]

5 (a) take readings of the detectors B1


fill box with water B1
take readings (again) B1

(b) dull black best AND shiny white worst B1

(c) two different metals B1


two junctions (could be at meter) hot and cold need not be indicated B1
any cell, max B1,B0
[Total: 6]

© UCLES 2007
Page 5 Mark Scheme Syllabus Paper
IGCSE – October/November 2007 0625 03

6 (a) mirror: 2 reflected rays approx correct M1


projected back to approx correct labelled image A1
note: images may be dots or lines
lens: ray through F, correct by eye M1
ray through centre OR ray through other F, correct by eye M1
projected back to approx correct (labelled) image A1

(b) (i) not produced by real rays crossing


OR cannot be caught on a screen
OR rays appear to come from image B1

(ii) upright/right way up/erect c.a.o. B1

(iii) lens image enlarged AND mirror image same size c.a.o.
OR (different) size OR (different) distance OR different side B1

[Total: 8]

7 (a) (i) diagram showing compressions and rarefactions


(could be either spaced vertical lines or dots, or coil or sine wave) B1
2C’s and 2R’s in approx correct place B1

(ii) wavelength correctly marked, by eye B1

(b) (i) all 3 in correct positions B1

(ii) radio (waves) B1

(iii) 3 × 108 m/s B1

[Total: 6]

© UCLES 2007
Page 6 Mark Scheme Syllabus Paper
IGCSE – October/November 2007 0625 03

8 (a) circuit 1 series AND circuit 2 parallel B1

(b) switch off each one separately )


one fails, other works )
both get full current/voltage/same voltage ) any 2 B1+B1
other good point e.g. more heat in parallel )
lower resistance )

(c) (total R =) 10 (Ω) C1


(V =) 12V A1

(d) 1/R = 1/4 + 1/6 (= 5/12) OR 1/R = 1/R1 + 1/R2 C1


2.4 (Ω) A1

(e) (i) 3(A) B1

(ii) 24W B1

(iii) 7200J e.c.f. (ii) B1

[Total: 10]

9 (a) when magnetic field cuts/cut by conductor/wire/coil/solenoid


OR change in magnetic field linked with coil etc. B1

current/e.m.f caused B1

(b) solenoid ends connected to meter/lamp note: any sign of a cell gets B0 B1
magnet indicated in suitable position on axis of solenoid B1

(c) insert/withdraw/move magnet into/out of solenoid B1


meter gives reading (as magnet moves) OR watch the meter OR lamp glows B1

(d) move magnet faster )


increase strength of magnet ) any 2 B1+B1
more turns on solenoid )
closer to solenoid )
[Total: 8]

© UCLES 2007
Page 7 Mark Scheme Syllabus Paper
IGCSE – October/November 2007 0625 03

10 (a) (i) low/0/off/no output B1

(ii) low/0/off/no output B1

(b) (i) temp sensor to NOT gate input, correct symbol B1


output of NOT gate (condone incorrect symbol) and humidity
sensor to AND inputs (condone labelled box for AND gate) B1

(ii) NOT low in, high out B1


AND both inputs high, high output B1
Note: B0, B0 for states on wrong diagram.

[Total: 6]

11 (a) detector, no source, no aluminium, take count OR take background B1


no aluminium, take count B1
aluminium, take count B1
subtract background/reading 1 from results B1

(b) count decreases as thickness of aluminium increases B1


6-10 sheets/several sheets/few mm,
count reduced to background count/β-particles stopped B1

[Total: 6]

© UCLES 2007
First variant Mark Scheme

Page 3 Mark Scheme Syllabus Paper


IGCSE – May/June 2008 0625 31

1 (a) (i) v/t or (v-u)/t or 28.5/3 or his correct ratio C1


9.3 to 9.5 m/s2 A1

(ii) area under graph or 0.5 × 3 × 28.5 or ½b×h C1


42 to 44 m (allow reasonable e.c.f.) A1

(iii) 15 m/s B1

(b) (plastic ball larger so) upward force/air resistance/drag more (or vice versa for rubber ball)
IGNORE wind resistance B1
rubber ball, this force not big enough to balance weight/gravity (force) B1
plastic ball, upward force/air resistance big enough to balance/equal weight/gravity
(force) B1

(c) mg or 0.05 × 10 or 50 x 10 accept 9.8 or 9.81 instead of 10 C1


0.5 N or 0.49N or 0.4905N nothing else A1
[10]

2 (a) fusion (of nuclei) CARE: NOT fission or fision ACCEPT fussion B1
condone radiation as an extra

(b) radiant/heat energy from Sun or radiation from Sun )


energy from Sun raises temperature of water/heats water/melts ice )
energy from Sun evaporates water ) any 3 B1 × 3
PE in cloud )
rain )
stored water has PE )

(c) (i) 25/100 for gas-fired or 30/90 for hydroelectric


or energy out/energy in or power out/power in B1

(ii) 30/90 or 1/3 or 33% is more than 25/100 or ¼ or 25%


OR lower input into hydroelectric station, but more output than gas-fired station B1
IGNORE hydroelectric losses less than gas-fired losses
[6]

© UCLES 2008
First variant Mark Scheme

Page 4 Mark Scheme Syllabus Paper


IGCSE – May/June 2008 0625 31

3 (a) mgh or 90 × 10 × 14 accept 9.8 or 9.81 instead of 10 C1


12 600 J or 12348 J or 12360.6 J nothing else A1

(b) PE lost = KE gained or mgh = ½mv2 C1


(v2 =) 280 e.c.f. or 274.4 or 274.68 C1
16.7 m/s e.c.f. or 16.565 m/s or 16.573 m/s NOTE: 16.8 m/s gets A0 A1

(c) energy lost or friction/air resistance/drag/wind resistance B1


[6]

4 (a) (pushing rubber cover) volume reduced M1


(when volume reduce), pressure goes up A1

(b) 1 × (105 ) × 60 = 1.5 × (105 ) × V C1


40 (cm3) C1
reduction in volume = 20 cm3 or 1/3 A1

(c) (ave) speed of mols/particles/atoms greater at high temp NOT energy/KE B1


stronger/more collisions with walls OR greater pressure B1
[7]

5 (a) SOLID higher temperature means higher energy/greater speed of


mols/particles/atoms B1
NOT more vibration NOT vibrate more

vibrations get bigger or movement greater/take up more space


or separation larger B1
GAS (ave) speed/energy of mols/particles/atoms greater B1
(ave) separation of mols/particles/atoms greater
or mols/particles/atoms take up more space
or increased pressure causes container to get bigger B1

(b) liquids: slightly more B1


gases: much more B1

(c) regular/uniform expansion or appropriate range (be generous if numbers quoted)


or expands a lot/large expansivity
or (relatively) non-toxic
or low freezing point/melting point
or measures low temperatures any 1 B1
IGNORE reacts to small temp change IGNORE high boiling point [7]

© UCLES 2008
First variant Mark Scheme

Page 5 Mark Scheme Syllabus Paper


IGCSE – May/June 2008 0625 31

6 (a) (for all rays, ignore any arrows, -1 for each incorrect extra ray)
correct ray through F1 ± 1mm on axis )
)
correct ray through F2 ± 1mm on axis ) any 2 B1, B1
)
ray through lens centre ± 1mm on axis )
image drawn between his intersection and axis B1

(b) virtual upright/erect magnified/enlarged further (from lens) any 3 B1 × 3


[6]

7 (a) (condone discontinuities at boundaries)

mirror:
equally spaced reflected waves, approx. same spacing as incident (by eye) B1
IGNORE reflected waves to left of arrowhead
correct angle to surface, by eye B1

block:
reduced wavelength in block B1
ACCEPT refracted waves to left of arrowhead
at sensible angle of refraction B1
CONDONE reflected waves shown as well as refracted

(b) (i) 3 × 108/speed in glass = 1.5 C1


2 × 108 m/s A1

(ii) sin70°/sinr = 1.5 C1


38.7895° to 2 or more sig figs A1
[8]

8 (a) all 4 lights in parallel with supply and none in series B1


master switch in a place where it will work (cannot score if no supply or if short
circuit) B1

one switch for 2 lights in living room AND one for bathroom
AND one for bedroom B1

(b) (i) W = V × I or 100 = 200 × I in any form C1


0.5 A or 0.5 a A1

(ii) I × t or 0.5 × 60 e.c.f. C1


30 C or 30 c e.c.f. A1

© UCLES 2008
First variant Mark Scheme

Page 6 Mark Scheme Syllabus Paper


IGCSE – May/June 2008 0625 31

(c) (i) 135 W B1

(ii) any power × any time (words or symbols or numbers) C1


NOTE: 280 (W) is the total power of lamps in house, so counts as “power”

486 000 J or 486 kJ or 0.135 kWh accept lower case units A1


NOTE: 45 × 3600 = 162000 J gets e.c.f. from (i)
[10]

9 (a) 3 complete circles about thick wire, roughly concentric on wire B1


clockwise or anticlockwise arrows on any 2 correct circles, and no contradictions B1

(b) (i) reduced B1

(ii) same OR none B1

(c) (i) thin wire is a current-carrying conductor in a magnetic field B1


field produced by current in thick wire B1
OR alternative approach:
( both wires produce a magnetic field B1 )
( fields interact B1 )

(ii) inwards/towards thick wire/to right/towards T1T2 B1

(iii) smaller force B1


[8]

10 (a) correct symbol, must show 3 connections, condone rounded “nose”, ignore width of the
shape, allow OR gate followed by NOT gate, correctly drawn B1

(b) if truth table is shown, mark the truth table and ignore the rest
either input 1, output 0 AND both inputs 1, output 0 B1
both inputs 0, output 1 accept high/low, on/off for both B1

(c) (i) one input is high/1 AND output is low/0 B1


IGNORE any reference to 2nd input

(ii) 1. on B1
2. off B1
[6]

© UCLES 2008
First variant Mark Scheme

Page 7 Mark Scheme Syllabus Paper


IGCSE – May/June 2008 0625 31

11 (a) number of protons 17 and 17 B1


number of neutrons 18 and 20 B1
number of electrons 17 and 17 B1

(b) alpha, beta, gamma words or symbols, any order NOT gamma particles B1

(c) (mark (i) and (ii) together)

(i) any correct use M1

(ii) simple correct explanation A1


[6]

© UCLES 2008
First variant Mark Scheme

Page 3 Mark Scheme Syllabus Paper


IGCSE – October/November 2008 0625 31

1 (a) (i) any mention of force or weight ignore mass C1


Force to left > force to right )
OR resultant force ) any 1 A1
OR unbalanced force )
OR weight > friction )

(ii) to overcome/compensate for friction/resistance B1

(b) 2/2.5 or 4/5 etc. or F/a or F = ma C1


0.8 kg A1

(c) 0.7/0.8 e.c.f. from (b) B1


0.875 (m/s2) e.c.f. from (b) could be scored on table (no unit needed) B1

(d) (i) v = at or 0.5 × 1.2 C1


0.6 m/s A1

(ii) any velocity × time or speed × time C1


0.36 m c.a.o. (note: 0.72 m gets C1, A0) A1 [11]

2 (a) two masses chosen with ratio 2:1 or 3:1 or 3:2 M1


chosen masses in correct holes to balance A1

(b) disc does not rotate/is balanced/in equilibrium/no movement B1


NOT spin the disc NOT anything to do with calculating moments
NOT when disturbed, returns to original position

(c) moment of one mass correct (ignore units)


accept mass × distance calculated B1
equal answers B1

(d) correct addition of masses/weights, including 200g B1


any mass correctly converted to N B1 [7]

3 (a) (i) hdg or 70 × 1050 × 10 C1


735 000 Pa or 7.35 × 105 Pa 2
accept N/m for Pa A1

(ii) 8.35 × 105 Pa OR his (a)(i) + 1.0 × 105 accept N/m2 for Pa B1

(b) pressure × area or P = F/A or 6.5 × 105 × 2.5 C1


1.625 × 106 N A1

(c) because density is less accept new calculation of pressure


OR because salt water is denser B1 [6]

© UCLES 2008
First variant Mark Scheme

Page 4 Mark Scheme Syllabus Paper


IGCSE – October/November 2008 0625 31

4 (a) typical random path drawn, at least 3 abrupt changes of direction B1

(b) air molecules hit dust particles in all directions/move it in all directions B1
just as likely to be up as down B1
(allow marks scored on diagram)

(c) random movements smaller OR slower movement


OR less energy OR movement decreases B1 [4]

5 (a) (i) funnel no longer giving heat to ice OR ice at M.P./constant temp
OR heater reached max temp B1

(ii) inside of large pieces could be well below freezing point )


OR smaller air gaps if pieces smaller ) any 1 B1
OR better contact between heater and ice )
OR to ensure heat from heater only goes to the ice )
OR larger surface area )
Ignore ice melts faster

(b) mass of beaker NOT mass of ice NOT mass of water B1


mass of beaker + water B1
(apply  +  = 0 for extras other than power & time)

(c) (mass of ice melted by heater = 16.3 – 2.1) = 14.2 g C1


ml in any form, words, symbols or numbers C1
Wt or Pt in any form, words, symbols or numbers accept VIt C1
338 J/g OR 338 000 J/kg c.a.o A1 [8]

6 (a) light of one colour/frequency/wavelength B1

(b) n = sinr/sini OR n = sini/sinr in any form C1


sinr/sin30 = 1.49 OR sinr = 1.49 × sin30 C1
48.0° – 48.2° A1

(c) ray at angle >30° and <60° to normal, by eye, correct way NO e.c.f. B1
Ignore any angles or labelling

(d) colours/spectrum would appear OR range of angles (ignore “rainbow”)


OR dispersion OR ray splits up B1

(e) 90° approx (accept any value 80° to 90°) B1

(f) (totally internally) reflected OR T.I.R. ignore not refracted B1 [8]

© UCLES 2008
First variant Mark Scheme

Page 5 Mark Scheme Syllabus Paper


IGCSE – October/November 2008 0625 31

7 (a) clear attempt at arcs of circles, at least 3 B1


same wavelength as incoming waves, by eye
(ignore shape ignore distance to first wave) B1
centre of curvature of arcs at centre of gap, by eye B1

(b) speed/wavelength or 20/2.5 or v = fλ C1


8 Hz or 8 s–1 or 8 waves/second A1

(c) his (b) or “the same” B1 [6]

8 (a) changes a.c. to d.c. OR rectifies a/c OR allows current to flow one way only
OR prevents current flowing backward B1

(b) I × t or 2 × 12 or 2 × 12 × 60 × 60 or amps × seconds C1


24 Ah or 86 400 C or 86 000 C A1

(c) emf = J/C OR energy converted/work done per unit charge/coulomb


OR W/A OR volts/p.d. when no current in circuit C1
12 J of energy are delivered/needed for every coulomb of charge
OR 12 W is the power to drive a current of 1 A A1

(d) (i) series connection shown, any recognisable symbols B1


(ii) total power = 16 W OR 8/6 C1
1.33 A accept fraction c.a.o. A1

(iii) any power × any time or 16 × 60 × 60 or IVt or 8 × 60 × 60 C1


57 600 J or 0.016 kWh or 28 800 J or 0.008 kWh A1 [10]

9 (a) pump water to higher level storage )


or heat water ) any one B1
or charge accumulators/batteries )
ignore charge capacitor NOT generator

(b) less/no energy/power/heat loss OR to reduce current


OR to allow thinner cables OR more efficient NOTHING ELSE B1

(c) I2R B1

(d) Ns/1200 = 32000/1100 OR N1/N2 = V1/V2 in any arrangement C1


34 880 or 34 900 or 34 909 or 34 910 or 35 000 A1

(e) input power = output power or V1I1 = V2I2 C1


current = power/voltage in any form, words, symbols or numbers C1
25 A A1 [8]

© UCLES 2008
First variant Mark Scheme

Page 6 Mark Scheme Syllabus Paper


IGCSE – October/November 2008 0625 31

10 (a) (i) LDR correctly identified B1

(ii) lamp correctly identified B1

(iii) transistor correctly identified B1

(b) (ignore anything that is in terms of currents)


resistance of LDR becomes high M1
LDR gets larger share of the voltage OR voltage across LDR gets bigger A1
transistor switches/turns lamp on A1 [6]

11 (a) A cathode OR electron gun


B Y plates OR vertical deflection plates
C X plates OR horizontal deflection plates
D screen OR fluorescent/phosphor OR tube NOT glass
4 correct B2, 3 or 2 correct B1 B2

(b) A; idea of releasing electrons/thermionic emission B1


B; move the electron beam vertically B1

(c) (i) y-plates/y-input or B NO e.c.f. B1

(ii) x-plates/x-input or C NO e.c.f. B1 [6]

© UCLES 2008
First variant Mark Scheme

Page 3 Mark Scheme: Teachers’ version Syllabus Paper


IGCSE – May/June 2009 0625 31

1 check zero on stopwatch OR repeat OR other sensible precaution B1


start stopwatch at some recognisable point in the cycle B1
stop stopwatch after at least 10 cycles OR count no. of cycles in at least 10 s B1
divide time by number of cycles B1 [4]

2 (a) water AND liquids expand more than solids B1

(b) steel M1
(steel) expands at same rate / has same expansion (as concrete) A1
different expansion AND cracks / breaks / damages / destroys concrete A1 [4]

3 (a) (i) straight line OR constant gradient / slope OR


change in speed with time constant OR speed proportional to time B1

(ii) increase in velocity / time OR a = v/t , symbols, words or numbers C1


0.75 m/s2 A1

(b) (i) decreases OR acceleration slows (down) NOT ‘it slows down’ C1

(ii) equal to forward / downward force / force down slope OR


constant / maximum OR (giving) no resultant force C1
equal to component of weight (down slope) A1

(iii) 1 graph starting at origin B1


curved from start AND decreasing gradient AND
horizontal final part B1

2 label A on any correct curved region B1


label B on horizontal region B1 [10]

4 (a) (i) (note: diagram may be drawn in any orientation)


sides correct length, by eye B1
forces drawn at 45°, by eye B1
parallelogram completed B1
correct diagonal drawn / correct resultant if intersecting arcs shown B1

(ii) magnitude: between 5500 N and 5700 B1


direction: between 28° and 32° B1

(b) (i) it has direction (as well as magnitude) B1

(ii) any example which is clearly a vector B1 [8]

© UCLES 2009
First variant Mark Scheme

Page 4 Mark Scheme: Teachers’ version Syllabus Paper


IGCSE – May/June 2009 0625 31

5 (a) (i) ½mv2 C1


½ × 7500 × 12 × 12 C1
540 000 J OR 540 kJ A1

(ii) W = E/t in any form B1


10% × his (a) C1
54 000 W OR 54 kW e.c.f. A1

(b) (i) 3750 kg B1

(ii) [If ecf from (i) and no other errors, maximum mark is 2]
mass: ½ OR correct sub in ½mv2 C1
speed: ½ OR 6750 (J) C1
fraction = ⅛ / 0.125 / 1:8 ? 12.5 % (c.a.o.) A1 [10]

6 (a) (i) P = F /A in any form, letters, words or numbers C1


1.4 × 106 Pa accept N/m2 A1

(ii) 84 N OR 84.0 N B1

(iii) same force over (much) smaller area B1


(much) bigger pressure B1

(b) (i) P = hdg in any form, letters, words or numbers C1


3 × 104 Pa OR 30 000 Pa OR 30 kPa accept N/m2 A1

(ii) his (i) B1 [8]

7 (a) Total penalty for use of ‘particles’ rather than ‘molecules’ is 1 mark.

(i) idea of some molecules gaining more KE B1


mols overcome attractive forces OR mols break free of surface B1

(ii) greater area B1


more mols escape (in given time) B1

(iii) increase temperature / supply more heat / make hotter )


blow air across surface, or equiv. ) any 2 B1 + B1
reduce humidity )
decrease pressure )

(b) water evaporates from cloth / water OR faster / more energetic


molecules evaporate )
less energetic mols left behind )
energy to evaporate taken from milk ) any 3 B1 × 3
evaporation produces cooling )
idea of cloth always being damp by soaking up water ) [9]

© UCLES 2009
First variant Mark Scheme

Page 5 Mark Scheme: Teachers’ version Syllabus Paper


IGCSE – May/June 2009 0625 31

8 (a) medium A because angle in air is bigger OR angle in A is smaller OR


refracts / bends away from normal / angle of refraction greater than angle
of incidence / total internal reflection only occurs in denser medium B1

(b) air: light travels faster in less dense medium OR air: air is less dense / rarer B1

(c) 42°–43° B1

(d) total internal reflection B1

(e) n = sin i / sin r OR n = sin r / sin i OR 1.49 = sin i / sin 35 C1


(allow 1.49 or refractive index instead of n in any of above)
58.719° to at least 2 s.f. Allow 58.71° A1

(f) n = speed in air / speed in medium in any arrangement


OR 1.49 = 3.0 × 108 / speed in medium A C1
2.01343 × 108 m/s to at least 2 s.f. A1 [8]

9 (a) half-wave rectification clearly indicated (any wave shape, repeated):


at least 2 humps with all spaces more than half width of hump, by eye. B1

(b) (i) A (c.a.o.) M1

(ii) For answers A and B only in (i), not C or D:


Route to resistor: correct arrow on one downwards diode and
nothing wrong on this route B1
Route from resistor: correct arrow on one downwards diode and
nothing wrong on this route B1 [4]

© UCLES 2009
First variant Mark Scheme

Page 6 Mark Scheme: Teachers’ version Syllabus Paper


IGCSE – May/June 2009 0625 31

10 (a) (i) 0 (A) / zero Unit penalty if wrong unit B1

(ii) 12 V B1

(b) (i) V / R OR V = IR in any form, letters, words or numbers C1


0.5 A A1

(ii) 8 × candidate’s (i) OR 8/24 × 12 C1


4 V OR 4.0 V e.c.f. A1

(c) 1/R1 + 1/R2 = 1/R OR R = R1R2 / (R1 + R2) in any form B1


5.3 (Ω) OR 5⅓ (Ω) OR 16/3 (Ω) C1
12 / candidate’s R C1
2.25 A c.a.o. A1

Alternatively: 12/16 (= 0.75) OR 12/8 (= 1.5) C1


12/16 (= 0.75) AND 12/8 (= 1.5) C1
Currents added C1
2.25 A c.a.o. A1 [10]

11 (a) ignore any extra ticks against α


β 3rd and 4th columns ticked
(use  +  = 0 for extras) i.e. 2 correct 2 marks
1 correct, nothing else 1 mark
1 correct, 1 wrong 1 mark
2 correct, 1 wrong 1 mark
2 correct, 2 or 3 wrong 0 marks B1 + B1
γ 1st column ticked (use  +  = 0 for extras) B1

(b) idea of in plane of page OR perpendicular to magnetic field C1


top to bottom of the page OR opposite direction of deflection of α OR
down the page A1
Ignore downwards. Ignore references to + or – plates, for both C1 and A1 [5]

© UCLES 2009
Page 4 Mark Scheme: Teachers’ version Syllabus Paper
IGCSE – October/November 2009 0625 31

1 (a) micrometer OR screw gauge OR vernier scale NOT vernier callipers B1

(b) 2.73 mm B1

(c) check/set zero )


close instrument on to paper )
not too tight/use ratchet ) any 3 B1 × 3
take reading of both scales )
use several sheets )
divide reading by no. of sheets )

[5]

2 (a) measuring cylinder with liquid B1


immerse statue B1
volume from difference of readings from measuring cylinder B1
OR
displacement can/equivalent/beaker, filled to overflowing with liquid (B1)
immerse statue (B1)
measure volume displaced with measuring cylinder (B1)

(b) (D =) M/V OR 600/65 B1


9.23 g/cm3 (minimum 2 s.f.) N.B. unit penalty applies B1
OR
(For gold) (M =) V × D OR 65 × 19 (B1)
1235 g (minimum 2 s.f.) N.B. unit penalty applies (B1)
OR
(For gold) (V =) M / D OR 600/19 (B1)
31.6 cm3 (minimum 2 s.f.) N.B. unit penalty applies (B1)

‘NO’ ticked if justified by previous work in (a) or (b).


e.c.f from wrong values above B1
[6]

3 (a) 5 points correctly plotted ±½ small square –1 e.e.o.o. (ignore 0,0) B2

(b) 3 N one, however identified OR 3rd value OR 4th value B1

(c) good straight line through origin and candidate’s remaining points B1

(d) straight line / constant gradient M1


does obey Hooke’s Law A1
OR
special case: obeys Hooke’s law because force ú extension or wtte B1

© UCLES 2009
Page 5 Mark Scheme: Teachers’ version Syllabus Paper
IGCSE – October/November 2009 0625 31

(e) graph becomes non-linear / curves / bends B1


Ignore reference to direction of curve or bend.

(f) will have exceeded / reached proportional / elastic limit


OR permanently deformed or equiv OR staightened
OR will have broken OR no longer elastic or wtte B1

[8]

4 (a) in direction of the force Do not accept forward on is own. B1

(b) changes direction / causes acceleration / stops straight line motion / keeps object
from leaving circle / keeps path circular / pulls object into circle B1

(c) (i) 1. 600 N B1


2. same as his 1. accept 600 N if no value given in (c) (i) 1. B1

(ii) ma OR 60 × 2.5 C1
150 N A1

(iii) 750 N e.c.f. from (c) (i) 2 and/or (c) (ii) B1

(iv) same as his (c) (i) 2 accept 600 N if no value given in (c) (i) 2. B1

[8]

5 (a) (P.E.) = mgh C1


12 × 10 × 3 Accept g = 9.8 or 9.81 C1
360 J g = 9.8 gives 352.8 J (minimum 2 s.f.) A1
g = 9.81 gives 353.16 J (minimum 2 s.f.)

(b) (P =) E/t C1
360/60 C1
6W 352.8 J gives 5.88 W 353.16 J gives 5.886 W (minimum 2 s.f.) A1

[6]

6 (a) (i) increases B1

(ii) pV = const in any form C1


1.05 (× 105) × 860 (× 10–6) = p × 645 (× 10–6) C1
1.4 × 105 Pa A1

© UCLES 2009
Page 6 Mark Scheme: Teachers’ version Syllabus Paper
IGCSE – October/November 2009 0625 31

(iii) F = pA in any form accept weight for F C1


EITHER increase in pressure = 0.35 × 105 (Pa) C1
0.35 × 105 × 5.0 × 10–3 C1
175 N (minimum 2 s.f.) c.a.o. A1
OR 1.05 × 105 × 5.0 × 10–3 or 525 N or 1.4 × 105 × 5.0 × 10–3 or 700 N (C1)
700 – 525 N e.c.f. from (a) (ii) (C1)
175 N (minimum 2 s.f.) c.a.o. (A1)

(b) (i) increases B1

(ii) no change B1

(iii) extra weight (on tray/piston) B1

(iv) increases B1

[12]

7 (a) EITHER OR
copper constantan
copper constantan
constantan copper B1

(b) galvanometer OR millivoltmeter OR milliammeter OR digital ammeter


OR digital voltmeter B1

(c) rapid response )


small area )
can measure high / low temperatures )
small thermal capacity (idea of) ) any 1 B1
remote reading )
large range )
data logging / continuous monitoring possible )
takes temperature of a surface )
N.B. (very) sensitive not accepted

[3]

8 (a) 2 cm (by eye) vertical object somewhere between F2 and lens


(condone no O, if clear) B1

(b) any two standard rays correctly drawn (no extrapolation needed) B1
correct rays extrapolated back to intersect B1
virtual image drawn at candidate’s intersection of extrapolated rays
(condone no I, if clear) B1

[4]

© UCLES 2009
Page 7 Mark Scheme: Teachers’ version Syllabus Paper
IGCSE – October/November 2009 0625 31

9 (a) (quantity of) heat/energy to raise temp by 1 °C/1degC/1K/unit temp rise M1


1 kg OR 1 g OR unit mass (Mention of change of state gets M0 A0) A1

(b) long time to heat up/cook )


long time to cool down ) any 1 B1
expensive to heat )
takes a lot of energy to heat up )

(c) (i) 1.8 degC OR 1.8 °C OR 1.8 K


AND 77.1 degC OR 77.1 °C OR 77.1K B1

(ii) (Q =) mcT in any form, seen anywhere B1


0.2 × 4200 × 1.8 e.c.f. from (c) (i) C1
1512 J (minimum 2 s.f.) c.a.o. A1

(iii) 1512 = 0.05 × c × 77.1 in any form e.c.f. from (c) (i) and/or (c) (ii) C1
392 J/kg K (N.B. must be to 3 sf ; A0 for wrong s.f.) e.c.f. A1

(iv) heat lost during transfer )


boiling water not at 100 °C / reason for not boiling
at 100 °C e.g. water not pure/ not standard pressure )
energy lost to cup etc. / surroundings ) any 1 B1
thermometer not accurate / sensitive enough )
temperature / mass(es) not accurately measured )

[10]

10 (a) (i) step-up transformer B1

(ii) less heat/energy/power loss (from lines) / thinner wires (possible) B1


OR lower current NOT more efficient

(b) P = V × I in any form, figures or symbols / (P =) VI C1


2.5 A A1

(c) P = I2R in any form, figures or symbols / (P =) I2R C1


18.75 W e.c.f. from (b) A1

(d) V = IR in any form, figures or symbols OR (V =) IR OR


P = V2 / R in any form, figures or symbols OR (P =) V2 / R OR V = (PR)1/2 C1

7.5 V e.c.f. from (b) or (c) A1

© UCLES 2009
Page 8 Mark Scheme: Teachers’ version Syllabus Paper
IGCSE – October/November 2009 0625 31

(e) 22,000 – 7.5 – 7.5 OR 22,000 – 7.5 ecf C1


21,985 V e.c.f. (minimum 4 s.f.in this case) A1
OR
55,000 – 37.5 = 54962.5 (C1)
54962.5 / 2.5 = 21985 V (minimum 4 s.f. in this case) (A1)

[10]

11 (a) A NOT or inverter B1


B AND B1

(b) (accept 1 or ON for HIGH, and 0 or OFF or NOT HIGH for LOW throughout)

(i) A – HIGH and B – LOW (both) no e.c.f. B1

(ii) A – HIGH and B – HIGH (both) no e.c.f. B1

(iii) A – LOW and B – LOW (both) no e.c.f. B1

(c) (i) B cannot provide enough power / current for lamp, or equiv.
OR allows remote lamp B1

(ii) the second one / dark and warm / HIGH, HIGH e.c.f. from (b) B1

(iii) warning if temperature in a closed / dark space (e.g. refrigerator, kiln) reaches
too high a value
N.B. “to switch on a lamp when it is dark and warm” not accepted B1

[8]

© UCLES 2009
Page 3 Mark Scheme: Teachers’ version Syllabus Paper
IGCSE – May/June 2010 0625 31

1 (a) decreases / braking / decelerating )


constant / steady / nothing ) all 3 B1
increases / accelerate )

(b) speed x time in any form, symbols, numbers or words


OR any area under graph used or stated C1
13 (m/s) OR 24 (s) seen or used in correct context C1
312 m A1

(c) rate of change of speed OR gradient of graph OR 18/12 C1

18 (m/s) OR 12 (s) seen or used in correct context C1


1.5 m/s2 A1

(d) same gradient / slope OR equal speed changes in equal times OR


allow graph symmetrical B1 [8]

2 (a) ½mv2 OR ½ × 900 x 302 C1


405 000 J A1

(b) force x distance OR 2000 x 30 C1


60 000 J OR 60 kJ A1

(c) 60 000 W OR 60 000 J/s OR 60kW OR 60 kJ/s ecf from (b) B1

(d) chemical B1

(e) idea of energy loss / heat / sound / inefficiency / energy used within car /
possibility of increase in P.E. Ignore work done against against friction B1 [7]

3 (a) 2nd statement re-written to include force in first gap and inversely
proportional to mass in second gap. NOT indirectly proportional B1

(b) F = ma OR in words in any correct arrangement B1

(c) (i) nothing OR continues as before OR same / constant velocity OR


same / constant speed & direction OR no acceleration B1

(ii) idea of retardation. Ignore stop. Ignore brakes. Ignore goes in


opposite direction B1

(iii) moves in (arc of a) circle or curve OR deflected OR turns OR


changes direction B1 [5]

© UCLES 2010
Page 4 Mark Scheme: Teachers’ version Syllabus Paper
IGCSE – May/June 2010 0625 31

4 (a) matt black B1

(b) (i) L down and R up, equal amounts (by eye) B1

(ii) on black side or on left (more) energy / heat absorbed OR greater


temp rise OR heats up quicker B1

on black side or on left greater expansion of air / greater pressure of air B1 [4]

5 (a) energy / heat required to change state / phase / any example of change of
state / phase M1

with no change in temperature / at a specified temperature A1


OR energy to break bonds between molecules /atoms M1
with no change in K.E. A1

(b) any time or range of time between 1.6 (min) and 14.0 (min) inclusive [no UP] B1

(c) turns substance to gas / vapour OR causes evaporation OR escape


from liquid C1

energy to break bonds/separate molecules/overcome intermolecular forces


Ignore move faster / PE increases A1

(d) (i) Pt / 2 × 4 / 2000 × 4 / 2 × 240 / 2000 × 240 / 8 / 8000 / 480 / 480000 C1


480 000 J OR 480 kJ A1

(ii) (θ =) 43 (°C) seen anywhere C1


Q = mcθ OR 480000 = m x 1760 × 43 in any form ecf. from (i) C1
6.34 kg or 6.3 kg ecf. A1 [10]

6 (a) (i) same / unchanged / nothing B1

(ii) reduced / slows down B1

(iii) reduced B1

(b) v = fλ in any form or in words [not numbers]


OR f =1/T in any form or in words [not numbers] B1
0.12 = f × 0.08 OR T = 0.08 / 0.12 C1
1.5 Hz / cycles per sec / c.p.s. / per s
[only 2 marks if B1 mark above not scored] A1

© UCLES 2010
Page 5 Mark Scheme: Teachers’ version Syllabus Paper
IGCSE – May/June 2010 0625 31

(c)

(ignore length of waves)


waves bending in correct direction (be generous) M1
A and B correct by eye, straight and parallel A1
C and D parallel to A and B by eye A1 [9]

7 (a) idea of light travelling (much) faster than sound B1

(b) (i) 4.0 (min) B1

(ii) always a (measurable) time difference / never zero time difference


Ignore time would be less B1

(iii) distance/time in any form, symbols, words, numbers OR 1200/3.6 C1


333.3 m/s to 2 or more sig figs A1

(iv) idea of light travelling instantaneously OR no wind


OR idea of lightning at ground level OR no obstruction to sound
Ignore echoes B1

(c)
light waves sound waves
longitudinal 
transverse 
electromagnetic 
mechanical 

–1 e.e.o.o. i.e. 1 mark subtracted from 3 for each error or omission B3 [9]

© UCLES 2010
Page 6 Mark Scheme: Teachers’ version Syllabus Paper
IGCSE – May/June 2010 0625 31

8 (a) (i) N1/N2 = V1/V2 in any form, symbols, words or numbers C1


12 (turns) [possible unit penalty] A1

(ii) mention of magnetic / electromagnetic field )


)
change of flux linkage / magnetism )
OR field lines being cut )
) any 3 B1 x 3
Induced current / emf / voltage )
)
Fewer coils in secondary so smaller emf / voltage
OR larger current )

(iii) heat in either coil / wires )


eddy currents in core / heat in core ) any 1 B1
magnetic leakage from core )
sound from core/coil )

(b) (i) 12 V d.c. OR low d.c.voltage B1

(ii) diode OR rectifier [Ignore extras unless wrong] B1

(c) V1I1 = V2I2 in any form, or words or numbers


OR power in = power out or equivalent C1

8A A1 [10]

9 (a) first finger – field / magnetism / flux )


second finger – current / charge flow (NOT electron flow) ) both B1

(b) (i) brush OR contact OR sliding connector B1


split ring OR commutator NOT slip ring B1

(ii) clockwise OR right side down OR left side up OR correct arrows


on figure NOT turn to the right B1

(iii) more current / more voltage / “stronger battery” / more power )


more turns on coil / more coils )
stronger magnet Ignore bigger magnets )
closer magnet / magnetic poles ) any 2 B1, B1
more magnets )
iron core ) [6]

© UCLES 2010
w
w
w
.X
tr
me
eP
UNIVERSITY OF CAMBRIDGE INTERNATIONAL EXAMINATIONS

ap
er
International General Certificate of Secondary Education

s.c
om
MARK SCHEME for the October/November 2010 question paper
for the guidance of teachers

0625 PHYSICS
0625/31 Paper 3 (Extended Theory), maximum raw mark 80

This mark scheme is published as an aid to teachers and candidates, to indicate the requirements of
the examination. It shows the basis on which Examiners were instructed to award marks. It does not
indicate the details of the discussions that took place at an Examiners’ meeting before marking began,
which would have considered the acceptability of alternative answers.

Mark schemes must be read in conjunction with the question papers and the report on the
examination.

• CIE will not enter into discussions or correspondence in connection with these mark schemes.

CIE is publishing the mark schemes for the October/November 2010 question papers for most IGCSE,
GCE Advanced Level and Advanced Subsidiary Level syllabuses and some Ordinary Level
syllabuses.
Page 2 Mark Scheme: Teachers’ version Syllabus Paper
IGCSE – October/November 2010 0625 31

NOTES ABOUT MARK SCHEME SYMBOLS & OTHER MATTERS

Points applicable to all answers

B marks are independent marks, which do not depend on any other marks. For a B mark to be
scored, the point to which it refers must actually be seen in the candidate's answer.

M marks are method marks upon which further marks depend. For an M mark to be scored, the
point to which it refers must be seen in a candidate's answer. If a candidate fails to
score a particular M mark, then none of the dependent marks can be scored.

C marks are compensatory method marks which can be scored even if the points to which they
refer are not written down by the candidate, provided subsequent working gives
evidence that they must have known it. e.g. if an equation carries a C mark and the
candidate does not write down the actual equation but does correct working which
shows he knew the equation, then the C mark is scored.

A marks are accuracy or answer marks which either depend on an M mark, or which are one of
the ways which allow a C mark to be scored.

e.e.o.o. means "each error or omission".

brackets ( ) around words or units in the mark scheme are intended to indicate wording used to
clarify the mark scheme, but the marks do not depend on seeing the words or units in
brackets.
e.g. 10 (J) means that the mark is scored for 10, regardless of the unit given.

underlining indicates that this must be seen in the answer offered, or something very similar.

OR/or indicates alternative answers, any one of which is satisfactory for scoring the marks.

Spelling Be generous about spelling and use of English. If an answer can be understood to
mean what we want, give credit.

Extras Ignore extras in answers if they are irrelevant; if they contradict an otherwise correct
response or are forbidden by mark scheme, use right + wrong = 0

Ignore Indicates that something which is not correct is disregarded and does not cause a right
plus wrong penalty.

Not/NOT Indicates that an incorrect answer is not to be disregarded, but cancels another
otherwise correct alternative offered by the candidate i.e. right plus wrong penalty
applies.

© UCLES 2010
Page 3 Mark Scheme: Teachers’ version Syllabus Paper
IGCSE – October/November 2010 0625 31

Points applicable to numerically worked answers only

Final If the final answer to a numerically worked question is correct, with the correct unit and
answers an acceptable number of significant figures, all the marks for that question are awarded.
The points which could have gained C marks need not be examined, even if wrong.

Ecf means "error carried forward". This indicates that if a candidate has made an earlier
mistake and has carried his incorrect value forward to subsequent stages of working, he
may be given marks indicated by ecf. provided his subsequent working is correct,
bearing in mind any earlier mistake. This prevents a candidate being penalised more
than once for a particular mistake, but only applies to marks annotated ecf.

Significant Answers are acceptable to any number of significant figures ≥ 2, except if specified
figures otherwise, or if only 1 sig. fig. is appropriate.

Units Deduct one mark for each incorrect or missing unit from an answer that would
otherwise gain all the marks available for that answer: maximum 1 per question.
No deduction is incurred if the unit is missing from the final answer but is shown correctly
in the working.

Arithmetic Deduct one mark if the only error in arriving at a final answer is an arithmetic one.
errors

Fractions These are only acceptable where specified.

Extras Ignore extras in answers if they are irrelevant; if they contradict an otherwise correct
response or are forbidden by the mark scheme, use right + wrong = 0

© UCLES 2010
Page 4 Mark Scheme: Teachers’ version Syllabus Paper
IGCSE – October/November 2010 0625 31

1 (a) (parallelogram or triangle may have any orientation)


NOT a copy of Fig. 1.1
two sides at right angles, by eye B1
one side longer than the other B1
diagonal or completion of triangle drawn and labelled “resultant” OR R
Ignore numerical values. Condone arrows in wrong direction B1

(b) 98 N – 102 N B1
(accept value found by calculation)

(c) (vertically) up/opposite to W NOT North B1

(d) his (b) OR correct value calculated B1


ignore mass

[Total: 6]

2 (a) constant velocity must be in a straight line/direction of motion is changing B1

(b) (i) if no force, then constant velocity in straight line OR force is needed
to change direction B1

body moving in circle is changing direction/velocity/accelerating


so force is needed B1

(ii) towards centre (of circle)/at right angles to motion/inwards B1

(iii) friction between tyres and road/reaction from banking of track B1

[Total: 5]

3 (a) (i) (P =) F/A in any form OR 1000/0.01 C1


100 000 Pa accept N/m2 A1

(ii) multiplication of either force or area by 4 C1


0.08 × his (i) OR 0.02 × his (i) C1
8000 N e.c.f. from (i) A1
(2000 N gets C0, C1, A1)

(b) his (ii) – 2000 correctly evaluated C1


600 kg e.c.f. A1

[Total: 7]

© UCLES 2010
Page 5 Mark Scheme: Teachers’ version Syllabus Paper
IGCSE – October/November 2010 0625 31

4 (a) heat/energy to raise/change temperature M1


of 1 kg/1g/unit mass through 1°C/1K A1
(mention of change of state scores zero)

(b) Q = mcθ (for θ accept t, T, ∆θ, ∆t, or ∆T) B1


23800 = 0.93 × c × (41.3 – 13.1) C1
907.5 or 907 or 908 or 910 J/(kg °C) or J/(kg K) at least 2 sig. figs A1
(for unit in (b) and (c)(i) condone no brackets and extra solidus)

(c) (i) 1212.9 or 1200 or 1210 or 1213 or 1214 J/(kg °C) or J/(kg K) B1

(ii) more energy lost (to surroundings) B1


(average) temperature is higher/initial temperature higher/no cooling
time allowed/temperature rise is lower/time of heating may be longer/
rate of heating may be lower B1

(d) insulate block/provide lid/cover with shiny foil )


start & finish same amount below & above room temperature ) any 2 B1 + B1
get heater up to temperature before inserting )
put oil in gap between heater & block )

[Total: 10]

5 (a) (i) (speed =) distance/time in any form, words, letters, numbers C1


0.15 m/s or 15 cm/s A1
(if answer only, 1 mark for either if no units)

(ii) (PE =) mgh OR mgh OR Wh symbols, words or numbers C1


100 J OR 98.1 J OR 98 J A1

(iii) his (ii)/40 OR his (ii)/4 C1


2.5 W OR 2.45 W e.c.f. from (ii) A1

(b) (input) greater/output less NOT a numerical factor B1

[Total: 7]

6 (a) incident ray in (more) dense medium )


angle of incidence greater than critical angle/42° ) any 3 B1 × 3
no light refracted )
reflected with i = r )

(b) reflection at Q only, no further reflections B2


(allow B1 only, if there is one further reflection at lower surface)
(give B0 for more than one further reflection) [Total: 5]

© UCLES 2010
Page 6 Mark Scheme: Teachers’ version Syllabus Paper
IGCSE – October/November 2010 0625 31

7 (a) (i) sound B1

(ii) particle OR mechanical OR compression OR longitudinal


OR matter wave B1

(iii) ultra violet/uv B1

(b) v = fλ OR λ = v/f B1
3.0 × 108/2.5 × 108 OR 3.0 x 108 = 2.5 × 108 λ C1
1.2 m A1

[Total: 6]

8 (a) capacitor/capacitance/condenser B1

(b) (i) 5 Ω B1

(ii) 5 and 20 both used OR 25 C1


R 1R 2
1/R = 1/R1 + 1/R2 OR (R =) seen or used C1
R1 + R 2
4Ω A1

(c) EITHER OR
ammeter reading falls (to zero) no current/reading M1

as capacitor charges P already charged/does not conduct d.c. A1

(d) Formula for calculation of I (I = V/R) OR P (P = V2/R) C1


Use of energy = power × time in any form C1
400 s A1

[Total: 10]

9 (a) (i) negative at LH end and positive at RH end B1

(ii) (+ve) charge on A attracts electrons/-ve charges/-ve ions


OR unlike charges attract (ignore reference to + charges) B1
electrons move to end X/towards A B1
(unbalanced) +ve charges (left) at end Y NOT repelled to Y B1

(iii) idea that each electron leaves behind an equal unbalanced proton
in nucleus/B has no net charge/B is neutral/idea that B has not
gained or lost any charges B1

(b) (i) nothing OR nothing implied B1

(ii) +ve charge cancelled/neutralised B1


by electrons/negative charges flowing up from earth B1

[Total: 8]

© UCLES 2010
Page 7 Mark Scheme: Teachers’ version Syllabus Paper
IGCSE – October/November 2010 0625 31

10 (a) idea of background radiation M1


random/different at different times NOT places A1

(b) A nothing OR background M1


reading doesn’t change (when source removed) A1

B gamma OR γ M1
gamma undeflected (by magnetic field) A1
uncharged/neutral OR electromagnetic radiation A1

C beta OR β B1
deflection is big/more deflection than alpha B1
low mass/much smaller than alpha B1

OR

beta OR β B1
negative B1
deflects according to left-hand rule B1

[Total: 10]

11 battery horizontal line across at least 4 squares M1


above or below horizontal centre line A1

a.c. supply alternating trace, any shape one or more cycles, at least
4 squares wide M1
above and below centre line, need not be symmetrical A1

a.c. supply only humps or only troughs seen, minimum 2 humps or troughs
+ diode M1
horizontal lines, approximately same width as humps or troughs,
separating humps or troughs A1

[Total: 6]

© UCLES 2010
Page 7 Mark Scheme: Teachers’ version Syllabus Paper
IGCSE – May/June 2010 0625 31

10 (a) proton number OR atomic number OR (number of) protons / electrons


OR position in periodic table OR chemical properties B1

(b) mass (number) OR nucleon number OR (number of) neutrons / nucleons


OR (number of) protons plus (number of) neutrons B1

(c) (i) mass (number) OR nucleon number OR (number of) nucleons


OR (number of) protons plus (number of) neutrons B1

(ii) proton number OR atomic number OR (number of) neutrons


OR (number of) protons / neutrons / electrons
OR position in periodic table OR chemical properties
OR a neutron changes into a proton B1 [4]

11 (a) (i) 4 Ω B1

(ii) IVt OR I2Rt OR V2t/R in any form or words or numbers


Condone t = 9 if substituted possible ecf from (i) C1
540 (s) C1
437.4 J possible ecf if 4 Ω from (i) used A1

(b) R = ρL/A OR R ∝ L/A OR R ∝ L and R ∝ 1/A or 1/d2 or 1/r2 C1

A2 = ¼A1 OR A2 = 0.25A1 C1
R2 = (0.45/0.3) × R1 OR (3/2) x R1 C1
⅜ OR 0.375 OR 37.5 % A1
OR
R = ρL/A OR R ∝ L/A OR R ∝ L and R ∝ 1/A or 1/d2 or 1/r2 C1

Resistance of thinner wire with same length as thicker wire = 4 × 4 = 16 Ω C1

Actual resistance of thinner wire = 1.8 /0.3 = 6.0 Ω C1

Ratio: L of thinner wire / L of thicker wire = 6.0 / 16 = 3/8 = 0.375 = 37.5 % A1 [8]

© UCLES 2010
Mark scheme for IGCSE Physics (0625/6) – Alternative to Practical May 1999

1 (a) Accurately drawn straight lines passing through all the relevant points 1
Pencil work, neat with thin lines not thicker than (circle line) 1
Correct labels 1
o o o
(b) Using candidate’s lines, d and e to nearest degree, d = 86 and e = 14 ± 1 2
(c) Candidate’s value accurate to 2 / 3 s.f. {6. 14} 1
(d)(i) E on reflected ray beyond P4 1
(ii) To prevent light from objects “behind” I reaching the eye {at E} 1
8

2 (a) Use of any suitable vertical, set-square / plumb-line 1


(b) Straw no longer horizontal
B moves downward 2
(c) 25.7 cm / 257 mm / 0.257 m 1
(d) Table with two columns 1
Headed with identification and unit for h 1
(e)(i) na = 9.44/9.5; nw = 4.4 to 4.5 1
(ii) upwards and smaller than weight / apparent loss of some weight 1
8
o
3 (a)(i) 26.8 C exact. 1
(ii) 0.2, unit not required 1
variation of 0.2 over such a long time is insignificant or similar 1
3
(b)(i) Expt l. 0cm
Expt 2 6.2
Expt 3 27.8 unit not required 1
3
(ii) 27.8/5 = 5.56/5.6 cm /h unit required 1
(c) Conclusion should establish, in the candidate’s own words that
Expt 1, no water leaks out from the tube so the rate of flow is zero 1
Expt 2, absorption by and evaporation from tissue results in a rate
of flow {1 .6} 1
Expt 3, Expt 2 + forced evaporation results in a larger rate of flow {x3.5} 1
8

4 (a) X = variable resistor / rheostat 1


Y = ammeter 1
(b) To change the value of the current 1
(c) Set X to maximum value, close switch 1
Adjust X to obtain desired value of I 1
Measure I and V 1
Repeat settings for a check / some worthy practical point e.g. tap analogue
meters or zero the meters. 1
(d) {max V = 9.5 x 0.1 = 0.95 V} FSD = 1 volt 1
8

5 (a)(i) Correct axes (h along y axis) labelled and with suitable scales 1
All plots to nearest 1/3 small square 1
Good line judgement. neat and thin line 1
(ii) Candidate uses the graph values correctly 1
(b) any two from:
reduces the chance of error / mentions parallax / reading can easily be
repeated or checked by someone else / can be made rigid allowing reading

1
to be taken at leisure 2
(c) So that the straw moves freely in the water / effect of meniscus on straw
always the same 1
(d) Straw would stick to the side of the container at the top of the meniscus 1
8

2
w
w
w
.X
tr
me
eP
UNIVERSITY OF CAMBRIDGE INTERNATIONAL EXAMINATIONS

ap
er
International General Certificate of Secondary Education

s.c
om
MARK SCHEME for the May/June 2011 question paper
for the guidance of teachers

0625 PHYSICS
0625/31 Paper 3 (Extended Theory), maximum raw mark 80

This mark scheme is published as an aid to teachers and candidates, to indicate the requirements of
the examination. It shows the basis on which Examiners were instructed to award marks. It does not
indicate the details of the discussions that took place at an Examiners’ meeting before marking began,
which would have considered the acceptability of alternative answers.

Mark schemes must be read in conjunction with the question papers and the report on the
examination.

• Cambridge will not enter into discussions or correspondence in connection with these mark schemes.

Cambridge is publishing the mark schemes for the May/June 2011 question papers for most IGCSE,
GCE Advanced Level and Advanced Subsidiary Level syllabuses and some Ordinary Level
syllabuses.
Page 2 Mark Scheme: Teachers’ version Syllabus Paper
IGCSE – May/June 2011 0625 31

Notes about Mark Scheme Symbols and Other Matters

B marks are independent marks, which do not depend on any other marks. For a B mark to be
scored, the point to which it refers must actually be seen in the candidate’s answer.
M marks are method marks upon which accuracy marks (A marks) later depend. For an M mark
to be scored, the point to which it refers must be seen in a candidate’s answer. If a
candidate fails to score a particular M mark, then none of the dependent A marks can be
scored.
C marks are compensatory method marks which can be scored even if the points to which they
refer are not written down by the candidate, provided subsequent working gives
evidence that they must have known it e.g. if an equation carries a C mark and the
candidate does not write down the actual equation but does correct working which
shows he knew the equation, then the C mark is scored.
A marks are accuracy or answer marks which either depend on an M mark, or which are one of
the ways which allow a C mark to be scored.
c.a.o. means “correct answer only”.
e.c.f. means “error carried forward”. This indicates that if a candidate has made an earlier
mistake and has carried his incorrect value forward to subsequent stages of working, he
may be given marks indicated by e.c.f. provided his subsequent working is correct,
bearing in mind his earlier mistake. This prevents a candidate being penalised more
than once for a particular mistake, but only applies to marks annotated “e.c.f.”
e.e.o.o. means “each error or omission”.
brackets ( ) around words or units in the mark scheme are intended to indicate wording used to
clarify the mark scheme, but the marks do not depend on seeing the words or units in
brackets e.g. 10 (J) means that the mark is scored for 10, regardless of the unit given.

underlining indicates that this must be seen in the answer offered, or something very similar.
OR/or indicates alternative answers, any one of which is satisfactory for scoring the marks.
Significant Answers are acceptable to any number of significant figures ≥ 2, except if specified
figures otherwise, or if only 1 sig. fig. is appropriate.
Units Deduct one mark for each incorrect or missing unit from an answer that would
otherwise gain all the marks available for that answer: maximum 1 per question.

Fractions These are only acceptable where specified.


Extras Ignore extras in answers if they are irrelevant; if they contradict an otherwise correct
response or are forbidden by mark scheme, use right + wrong = 0
Ignore Indicates that something which is not correct is disregarded and does not cause a right
plus wrong penalty.
Not/NOT Indicates that an incorrect answer is not to be disregarded, but cancels another
otherwise correct alternative offered by the candidate i.e. right plus wrong penalty
applies.

© University of Cambridge International Examinations 2011


Page 3 Mark Scheme: Teachers’ version Syllabus Paper
IGCSE – May/June 2011 0625 31

1 (a) all points correctly plotted ±½ small square B1


straight line of best fit for candidate’s points B1

(b) (i) candidate’s correct value with unit (± 0.2), (expect 1.2 N) B1

(ii) remains stationary / nothing happens / no acceleration NOT constant speed B1

(c) Correct data from candidates graph for ∆F and ∆m, used in ∆F/∆m B1

(d) (i) F = ma in any form, letters, words B1

(ii) gradient = F/a OR gradient = m ignore m=F/a C1


candidate’s (c) with correct unit A1

(e) straight line of positive gradient B1 [9]

2 (a) distance/height AND tape measure/(metre) rule(r) B1


weight OR load OR force
AND balance/scale(s) OR newton-meter/spring balance/force meter B1
time AND watch/clock/timer B1

(b) power = work/time OR energy/time in any form


OR Pt words or numbers seen anywhere e.g. 528 x 5 C1
(work =) force × distance in any form C1
11 A1

(c) efficiency = Eout/Ein OR Pout/Pin seen anywhere, clearly identified


OR 520 × (20/11) × 5
OR (work done =) 800 × 20 × 0.3 OR 800 × 20 × 30 OR 4800 (J) OR 720 (J) C1
(energy used =) 32,000 J A1 [8]

© University of Cambridge International Examinations 2011


Page 4 Mark Scheme: Teachers’ version Syllabus Paper
IGCSE – May/June 2011 0625 31

3 (a) (i) smaller because area smaller B1

(ii) smaller because depth/height smaller ignore less water B1

(b) (i) hρg OR 12 × 1000 × 10 C1


1.2 × 105 Pa OR 1.1772 × 105 Pa OR 1.176 × 105 Pa accept N/m2 A1

(ii) candidate’s (i) + 1.0 × 105 Pa correctly evaluated with unit (correct value
2.2 × 105) B1

(iii) p1V1 = p2V2 in any form C1


1.1 cm3
OR 0.5 × candidate’s (ii)/105 correctly evaluated A1

(iv) value in (iii) too small OR volume larger o.w.t.t.e. B1 [8]

4 (a) rheostat/variable resistor AND control/vary/change/ limit


current /resistance/power/voltage across heater B1

(b) (i) P = VI in any form OR (I=) P/V C1


1.25 A A1

(ii) (R =) V/I in any form words or numbers C1


(voltage across X =) 2.4 (V) OR 6 - 3.6 (V) C1
1.92 Ω e.c.f. from (b) (i) A1

(c) battery running down/going flat/energy of battery used up OR V or e.m.f. less


OR more/increasing resistance (of heater) NOT resistance of X increases B1

(d) (i) transformer condone step-up OR potential divider/potentiometer NOT extras B1

(ii) diode OR rectifier OR L.E.D. NOT extras B1 [9]

© University of Cambridge International Examinations 2011


Page 5 Mark Scheme: Teachers’ version Syllabus Paper
IGCSE – May/June 2011 0625 31

5 (a) (i) potential difference OR e.m.f. OR voltage ignore volts

(ii) frequency accept cycles/s ignore waves/s all 3 B1

(iii) power accept energy/s

(b) (i) case/frame/outside/base/parts that can be touched ignore metal parts B1

(ii) electric shock/electrocution/death by electricity o.w.t.t.e. ignore anything else B1


live wire touches case B1

(c) heaters in parallel with any supply


(M0 if no supply, clear break in circuit, short across supply or heater) M1
one switch controlling both heaters and one switch controlling one heater
OR one switch in series with each element A1

special case: heaters in series with supply and one switch shorting out one
resistor AND another switch in series with supply B2 [6]

6 (a) A and C B1

(b) (i) 4.2 × 1010 years B1

(ii) idea of decay OR changes proton/neutron/nucleon number


OR change into another nuclide/isotope/element/type of atom
OR emits α/β particle (ignore γ / radiation) B1

(iii) idea of insignificant change in activity during stated time up to 5 × 109 years
OR experiment time insignificant c.f. 1.4 × 1010 years OR long half life
OR long time to decay B1 [4]

© University of Cambridge International Examinations 2011


Page 6 Mark Scheme: Teachers’ version Syllabus Paper
IGCSE – May/June 2011 0625 31

7 (a) idea of fine ray/beam shone into (glass) block / pins appropriately placed
shown in diagram or described B1
angles i & r or C measured OR correct i & r or C marked on diagram B1
sini/sinr OR sinr/sini OR 1/sinC OR sinC B1
n = speed in air/speed in glass OR c/v = sini/sinr OR n = 1/sinC OR c/v = 1/sinC B1

(b) (i) v = fλ OR 240/1.9 × 105 OR T=d/s AND f=1/T B1


0.00126 Hz OR 0.0013 Hz NOT 0.0012 Hz
ignore more than 3 s.f. accept s-1 A1

(ii) distance = speed × time in any form accept s = 2d/t C1


(time for tremor =) 240 (s) or 4 mins also gives first C1 C1
(time for tsunami = ) 2500 (s) or 41 mins 40 s also gives first C1 C1
(warning time = ) 2260 (s) or 37 mins 40 s A1 [10]

8 (a) (i) total (internal) reflection OR reflection but no refraction/doesn’t emerge B1


angle (of incidence} > critical angle B1

(ii) initial reflection + 0 or 1 further reflection only, not at lower surface


must be straight and reach within 1cm of end B1

(b) (i) bends easily/less likely to break (ignore stronger) OR smaller pixels/
more detail/greater resolution/see smaller objects/wider field of view B1

(ii) light travels down/along/through fibres B1

(iii) light/image returns up/along/through fibres ignore cameras B1 [6]

9 (a) (i) down


down OR anti-clockwise both B1

(ii) BC is parallel to the field/doesn’t cut field or vice-versa/not at angle to field


ignore BC not perpendicular to field B1

(b) continues moving/turning NOT reverse/other direction M1


idea of moving things continue moving OR reference to Newton’s Laws
OR reference to momentum/KE/inertia NOT reference to force still acting A1

© University of Cambridge International Examinations 2011


Page 7 Mark Scheme: Teachers’ version Syllabus Paper
IGCSE – May/June 2011 0625 31

(c) more turns/several coils


iron core
increase current/voltage
stronger magnet
smaller air gap any 1 B1
curved poles
more efficient brushes
poles closer
use split-ring commutator [5]

10 (a) release of electrons due to heating/high temperature/heater B1

(b) X- and Y-plates labelled B1


anodes either order, labelled, either plates/cylinders with holes B1
closed tube of sensible shape
AND cathode AND anode(s) AND X- & Y- plates, all three features in correct
order
labels not needed for last mark but if given must be correct B1

(c) change current in filament/cathode/heater IGNORE limit


OR change temperature/heat/power/energy of filament/cathode/heater
OR change cathode-anode p.d./voltage
OR change charge/voltage of grid B1

(d) (i) (I=)Q/t in any form C1


0.0019 A OR 1.9 × 10-3 A OR 1.9 mA A1

(ii) (E=) VIt OR VQ in any form, words, symbols, numbers (accept t=5s) C1
190 J OR candidate’s I × 100 000 correctly evaluated A1 [9]

11 (a) Pt OR 1.2 × 104 × 9 OR 1.2 × 104 × (11 – 2) C1


(l=) E/m OR E/0.36 OR Pt/m OR Pt/0.36 C1
3 × 105 J/kg A1

(b) (i) liquid ignore vapour/gas/water A1

(ii) move around more rapidly / faster / more KE


ignore start to vibrate etc but accept starts to vibrate faster
move further apart / spreads out (NOT molecules expand) any 2 B1
break free / evaporate / overcome bonds / overcome forces of
attraction /escape / change state (accept boils)
convection (current) [6]

© University of Cambridge International Examinations 2011


w
w
w
.X
tr
me
eP
UNIVERSITY OF CAMBRIDGE INTERNATIONAL EXAMINATIONS

ap
er
International General Certificate of Secondary Education

s.c
om
MARK SCHEME for the October/November 2011 question paper
for the guidance of teachers

0625 PHYSICS
0625/31 Paper 3 (Extended Theory), maximum raw mark 80

This mark scheme is published as an aid to teachers and candidates, to indicate the requirements of
the examination. It shows the basis on which Examiners were instructed to award marks. It does not
indicate the details of the discussions that took place at an Examiners’ meeting before marking began,
which would have considered the acceptability of alternative answers.

Mark schemes must be read in conjunction with the question papers and the report on the
examination.

• Cambridge will not enter into discussions or correspondence in connection with these mark schemes.

Cambridge is publishing the mark schemes for the October/November 2011 question papers for most
IGCSE, GCE Advanced Level and Advanced Subsidiary Level syllabuses and some Ordinary Level
syllabuses.
Page 2 Mark Scheme: Teachers’ version Syllabus Paper
IGCSE – October/November 2011 0625 31

NOTES ABOUT MARK SCHEME SYMBOLS & OTHER MATTERS

M marks are method marks upon which further marks depend. For an M mark to be scored,
the point to which it refers must be seen in a candidate's answer. If a candidate
fails to score a particular M mark, then none of the dependent marks can be scored.

B marks: are independent marks, which do not depend on other marks. For a B mark to
scored, the point to which it refers must be seen specifically in the candidate’s
answers.

A marks In general A marks are awarded for final answers to numerical questions. If a final
numerical answer, eligible for A marks, is correct, with the correct unit and an
acceptable number of significant figures, all the marks for that question are normally
awarded.
It is very occasionally possible to arrive at a correct answer by an entirely wrong
approach. In these rare circumstances, do not award the A marks, but award C
marks on their merits.

C marks are compensatory marks in general applicable to numerical questions. These can be
scored even if the point to which they refer are not written down by the candidate,
provided subsequent working gives evidence that they must have known it.
For example, if an equation carries a C mark and the candidate does not write down
the actual equation but does correct substitution or working which shows he knew
the equation, then the C mark is scored.
A C mark is not awarded if a candidate makes two points which contradict each
other. Points which are wrong but irrelevant are ignored.

brackets ( ) around words or units in the mark scheme are intended to indicate wording used to
clarify the mark scheme, but the marks do not depend on seeing the words or units
in brackets.
e.g. 10 (J) means that the mark is scored for 10, regardless of the unit given.

underlining indicates that this must be seen in the answer offered, or something very similar.

OR / or indicates alternative answers, any one of which is satisfactory for scoring the marks.

e.e.o.o. means "each error or omission".

o.w.t.t.e. means “or words to that effect”.

Spelling Be generous about spelling and use of English. If an answer can be understood to
mean what we want, give credit.

Not/NOT Indicates that an incorrect answer is not to be disregarded, but cancels another
otherwise correct alternative offered by the candidate i.e. right plus wrong penalty
applies.

Ignore Indicates that something which is not correct or irrelevant is to be disregarded and
does not cause a right plus wrong penalty.

© University of Cambridge International Examinations 2011


Page 3 Mark Scheme: Teachers’ version Syllabus Paper
IGCSE – October/November 2011 0625 31

ecf meaning "error carried forward" is mainly applicable to numerical questions, but may
in particular circumstances be applied in non-numerical questions.
This indicates that if a candidate has made an earlier mistake and has carried an
incorrect value forward to subsequent stages of working, marks indicated by ecf
may be awarded, provided the subsequent working is correct, bearing in mind the
earlier mistake. This prevents a candidate being penalised more than once for a
particular mistake, but only applies to marks annotated ecf.

Sig. figs. Answers are normally acceptable to any number of significant figures ≥ 2. Any
exceptions to this general rule will be specified in the mark scheme. In general,
accept numerical answers, which, if reduced to two significant figures, would be
right.

Units Deduct one mark for each incorrect or missing unit from an answer that would
otherwise gain all the marks available for that answer: maximum 1 per
question. No deduction is incurred if the unit is missing from the final answer but is
shown correctly in the working.

Arithmetic errors Deduct one mark if the only error in arriving at a final answer is clearly an arithmetic
one.

Transcription Deduct one mark if the only error in arriving at a final answer is because given or
errors previously calculated data has clearly been misread but used correctly.

Fractions These are only acceptable where specified.

© University of Cambridge International Examinations 2011


Page 4 Mark Scheme: Teachers’ version Syllabus Paper
IGCSE – October/November 2011 0625 31

1 (a) acceleration = v – u OR ∆v (symbols used to be explained)


t t
OR change of velocity ÷ time
OR rate of change of velocity
OR change of velocity per second / in 1 sec (allow ‘in a certain time’) B1
accept speed for velocity

(b) (i) use of any area under graph C1


750 m A1

(ii) time = change of speed ÷ acceleration OR 30/0.60 C1


= 50 (s) A1
if working for t = 50 s not shown, allow 2 marks for correct use of 50 s
graph: along y-axis to 180 s / rise starts at 180 s B1
from x-axis rises to 30 m/s at 230 s / candidate’s calculated time B1
horizontal from top of slope to 280 s B1 [8]
allow ½ square tolerance at 180 s where relevant
allow ecf from wrong t

2 (a) two processes from:


vapour rising
condensation
rain falling
water falling from lake / through pipes
water turns turbine / generator
electricity generated. max B2

energy changes:
PE to KE matched to a process B1
KE to electricity energy for turbine / power station B1

(b) (i) (PE =) mgh OR 2 × 105 × 10 × 120 allow g = 9.8 or 9.81 C1


2.4 × 108 J A1

(ii) (KE of water =) ½mv2 OR ½ × 2 × 105 × 142 C1


1.96 × 107 J OR 2.0 × 107 J A1 [8]

© University of Cambridge International Examinations 2011


Page 5 Mark Scheme: Teachers’ version Syllabus Paper
IGCSE – October/November 2011 0625 31

3 (a) 1. no resultant force acts / no net force acts


OR total force up / in any direction = total force down / in opposite direction B1
allow sum of forces or resultant force for total force

2. no resultant moment / couple / torque acts


OR (sum of) clockwise moments and (sum of) anti-clockwise moments
(about any point / axis) balance B1

(b) (i) (anti-clockwise moment =) F × 2 C1


(total clockwise moment =) (120 × 33) + (20 × 15) = 4260 (N cm) C1
2130 N A1

(ii) 1990 N OR candidate’s (b)(i) – 140 N B1


force is downwards B1 [7]

4 (a) surfaces shown at realistic levels in dish and tube AND vertical height h between
levels clearly shown B1
top label: vacuum / mercury vapour B1
bottom label: mercury B1

(b) (P =) hdg OR 0.73 × 13600 × 10 C1


99280 Pa at least 2 s.f. B1

(c) one from:


abnormal weather / atmospheric conditions o.w.t.t.e.
air in space above mercury in tube
barometer is in a high altitude location o.w.t.t.e.
space above mercury is not a vacuum B1 [6]
ignore atmospheric pressure varies ignore temperature

5 (a) (i) most: gas


least: solid both required B1

(ii) because change of pressure (also) causes volume change (in a gas) B1
NOT ‘gas can be compressed’

(b) (i) two from:


expands uniformly (over required range)
remains liquid over required range
expands more than glass / has high expansivity / expansion
has (reasonably) low specific heat capacity.
has low freezing point / lower freezing point than mercury max B2

(ii) make (capillary) tube narrower (and longer) / thinner / smaller diameter B1
make bulb larger (and tube longer) B1
allow ‘bore’ for tube ignore ‘smaller’ ignore narrow thermometer

© University of Cambridge International Examinations 2011


Page 6 Mark Scheme: Teachers’ version Syllabus Paper
IGCSE – October/November 2011 0625 31

(c) allows fast(er) flow of heat to / from alcohol


OR allows fast response (to temperature change)
OR because glass is a poor conductor / good insulator (so needs to be thin for
fast response)
OR heat transfer more efficient / faster
OR glass takes up less heat B1 [7]
ignore reference to sensitivity ignore ‘easier’

6 (a) (i) 1. compressions and/or rarefactions closer together


OR more compressions and/or rarefactions B1
ignore wavelength shorter

2. layers closer together at compressions B1


layers farther apart at rarefactions B1
OR
compressions narrower (B1)
rarefactions wider (B1)
ignore wavelength shorter ignore ‘amplitude greater’ ignore ‘maximum
displacement greater’

(ii) distance between 2 compressions or 2 rarefactions shown with reasonable


accuracy B1

(b) time taken by sound in air = 200 / 343 = 0.583 s C1


time taken by sound in steel = 0.583 – 0.544 = 0.039 s C1
5128 m/s A1 [7]

7 (a) (i) light of a single wavelength / frequency ignore ‘one colour’ B1

(ii) n = sin i/sin r OR 1.52 = sin 50/sin r OR sin r = sin 50/1.52 C1


30.26º at least 2 s.f. A1

(iii) ray closer to normal in block B1


ray parallel to incident ray emerging from block B1

(b) (i) n = vA/vG OR n = 1.54/vG OR vG = 3 × 108/1.54 C1


1.948 × 108 m/s B1

(ii) ray with smaller angle of refraction than red in block i.e. violet ray under red ray B1
emerging ray parallel to incident ray B1 [9]

© University of Cambridge International Examinations 2011


Page 7 Mark Scheme: Teachers’ version Syllabus Paper
IGCSE – October/November 2011 0625 31

8 (a) any three from:


use a strong(er) magnet
increase the number of coils in the solenoid / turns of solenoid closer together
move the magnet fast(er).
place iron core in the solenoid
use thick(er) wire / low(er) resistance wire for solenoid max B3

(b) (i) NP/NS = VP/VS OR 200/800 = VP/24 OR VP = NPVS/NS


OR VP = 200 x 24/800 C1
6.0 V A1

(ii) IpVp = IsVs OR IpNp = IsNs OR IP = ISVS/VP OR IP = ISNS/NP


OR IP = (0.5 x 24)/6 OR IP = (0.5 x 800)/200 C1
2(.0) A
allow ecf from (b)(i) A1 [7]

9 (a) (i) 1. resistance is constant / doesn’t vary B1


2. resistance increases B1

(ii) 7 V B1

(b) resistance of resistor = 4/2.6 (= 1.54 Ω) C1


resistance of lamp = 4/3.6 (= 1.11 Ω) C1
1/R = 1/R1 + 1/R2 OR (R =) R1R2/(R1 + R2) OR either eq. with numbers C1
0.645 or 0.65 Ω A1
OR
current through resistor = 2.6 A (C1)
current through lamp = 3.6 A (C1)
total current = 2.6 + 3.6 = 6.2 A (C1)
0.645 Ω OR 0.65 Ω OR R = 4/sum of candidate’s currents (A1) [7]
accept R value based on no. of sig. figs. for resistors used by candidate

10 (a) (i) thermistor B1

(ii) lamp is ON at 20 oC / low temperature and OFF at 100 oC / high temperature B1

p.d. across B is high at 20 oC / low temperature B1


p.d. across B is low at 100 oC / high temperature B1
OR as temperature rises, p.d. across B falls (B2)

transistor acts as a switch for the lamp at a certain temperature


OR lamp is ON if there is current in base / collector
OR potential of base is high
OR lamp is OFF if there is no current in base / collector
OR potential of base is too low B1

© University of Cambridge International Examinations 2011


Page 8 Mark Scheme: Teachers’ version Syllabus Paper
IGCSE – October/November 2011 0625 31

(b) to switch on a warning light when temperature (required for a process) becomes
too low
OR to switch off a warning light when temperature (required for a process)
becomes high enough B1 [6]
example (e.g. freezer or incubator) not needed, but if given, explanation required

11 (a) (i) to heat the cathode / C B1

(ii) to emit electrons / to undergo thermionic emission (when heated) B1

(iii) to attract / accelerate electrons B1


to allow the electrons / beam to pass through to the screen / to focus the
beam / to direct the beam / produce a straight beam / to fix the beam current B1

(b) (i) p.d. / voltage / battery / power supply applied between / across plates B1
upper plate positive and lower plate negative B1

(ii) sketch showing: straight vertical lines from top plate to bottom plate B1
arrows pointing downwards / from + to – B1 [8]

© University of Cambridge International Examinations 2011


w
w
w
.X
tr
me
eP
UNIVERSITY OF CAMBRIDGE INTERNATIONAL EXAMINATIONS

ap
er
International General Certificate of Secondary Education

s.c
om
MARK SCHEME for the May/June 2012 question paper
for the guidance of teachers

0625 PHYSICS
0625/31 Paper 3 (Extended Theory), maximum raw mark 80

This mark scheme is published as an aid to teachers and candidates, to indicate the requirements of
the examination. It shows the basis on which Examiners were instructed to award marks. It does not
indicate the details of the discussions that took place at an Examiners’ meeting before marking began,
which would have considered the acceptability of alternative answers.

Mark schemes must be read in conjunction with the question papers and the report on the
examination.

• Cambridge will not enter into discussions or correspondence in connection with these mark schemes.

Cambridge is publishing the mark schemes for the May/June 2012 question papers for most IGCSE,
GCE Advanced Level and Advanced Subsidiary Level syllabuses and some Ordinary Level
syllabuses.
Page 2 Mark Scheme: Teachers’ version Syllabus Paper
IGCSE – May/June 2012 0625 31

NOTES ABOUT MARK SCHEME

M marks are method marks upon which further marks depend. For an M mark to be scored, the
point to which it refers must be seen in a candidate's answer. If a candidate fails to score
a particular M mark, then none of the dependent marks can be scored.

B marks are independent marks, which do not depend on other marks. For a B mark to be scored,
the point to which it refers must be seen specifically in the candidate’s answers.

A marks In general A marks are awarded for final answers to numerical questions.
If a final numerical answer, eligible for A marks, is correct, with the correct unit and an
acceptable number of significant figures, all the marks for that question are normally
awarded.
It is very occasionally possible to arrive at a correct answer by an entirely wrong
approach. In these rare circumstances, do not award the A marks, but award C marks on
their merits. However, correct numerical answers with no working shown gain all the
marks available.

C marks are compensatory marks in general applicable to numerical questions. These can be
scored even if the point to which they refer are not written down by the candidate,
provided subsequent working gives evidence that they must have known it. For
example, if an equation carries a C mark and the candidate does not write down the
actual equation but does correct substitution or working which shows he knew the
equation, then the C mark is scored. A C mark is not awarded if a candidate makes two
points which contradict each other. Points which are wrong but irrelevant are ignored.

brackets ( ) around words or units in the mark scheme are intended to indicate wording used to
clarify the mark scheme, but the marks do not depend on seeing the words or units in
brackets, e.g. 10 (J) means that the mark is scored for 10, regardless of the unit given.

underlining indicates that this must be seen in the answer offered, or something very similar.

OR / or indicates alternative answers, any one of which is satisfactory for scoring the marks.

e.e.o.o. means "each error or omission".

o.w.t.t.e. means “or words to that effect”.

Spelling Be generous about spelling and use of English. If an answer can be understood to mean
what we want, give credit. However, beware of and do not allow ambiguities, accidental
or deliberate: e.g. spelling which suggests confusion between reflection / refraction /
diffraction / thermistor / transistor / transformer.

Not/NOT Indicates that an incorrect answer is not to be disregarded, but cancels another
otherwise correct alternative offered by the candidate i.e. right plus wrong penalty
applies.

Ignore Indicates that something which is not correct or irrelevant is to be disregarded and does
not cause a right plus wrong penalty.

© University of Cambridge International Examinations 2012


Page 3 Mark Scheme: Teachers’ version Syllabus Paper
IGCSE – May/June 2012 0625 31

ecf meaning "error carried forward" is mainly applicable to numerical questions, but may in
particular circumstances be applied in non-numerical questions.
This indicates that if a candidate has made an earlier mistake and has carried an
incorrect value forward to subsequent stages of working, marks indicated by ecf may be
awarded, provided the subsequent working is correct, bearing in mind the earlier
mistake. This prevents a candidate being penalised more than once for a particular
mistake, but only applies to marks annotated ecf.

Significant Figures
Answers are normally acceptable to any number of significant figures ≥ 2. Accept
answers that round to give the correct answer to 2 s.f. Any exceptions to this general rule
will be specified in the mark scheme.

Units Deduct one mark for each incorrect or missing unit from a final answer that would
otherwise gain all the marks available for that answer: maximum 1 per question. No
deduction is incurred if the unit is missing from the final answer but is shown correctly in
the working.

Arithmetic errors
Deduct one mark if the only error in arriving at a final answer is clearly an arithmetic one.

Transcription errors
Deduct one mark if the only error in arriving at a final answer is because given or
previously calculated data has clearly been misread but used correctly.

Fractions e.g. ½, ¼ etc are only acceptable where specified.

© University of Cambridge International Examinations 2012


Page 4 Mark Scheme: Teachers’ version Syllabus Paper
IGCSE – May/June 2012 0625 31

1 (a) Period: 1.81 s OR 1.8 s as mean value


OR 1.8 s as most common reading / the mode B1

(b) Time a minimum of 2 (successive) oscillations B1


Divide result by the number of oscillations B1
OR
Count no. of oscillations in at least 20 s (B1)
Divide the time by the number of oscillations
OR Divide no. of oscillations by time and find reciprocal (B1)
2 of:
Repeat (several times) and find mean
Time with reference to fixed / fiducial point or top or bottom of oscillation
Check / set zero of stop-watch B2
Show knowledge of what is meant by one oscillation

[Total: 5]

2 (a) (i) Increasing speed / acceleration B1

(ii) Constant / steady / uniform speed or motion B1

(iii) Decreasing speed / deceleration / braking / slowing / stopping / negative


acceleration B1

(b) (i) (Total) distance / (total) time OR d / t OR 400 / 60 C1


6.67 m/s at least 2 s.f. A1

(ii) Mention of maximum gradient OR clear that whole or part of B to C is used C1


Use of correct data from graph to +/– ½ square C1
Answer rounds to 9.2 to 9.4 m/s, at least 2 s.f. A1

[Total: 8]

© University of Cambridge International Examinations 2012


Page 5 Mark Scheme: Teachers’ version Syllabus Paper
IGCSE – May/June 2012 0625 31

3 (a) Example: e.g. battery: (chemical to) electrical


engine: (chemical to) kinetic / mechanical
fire: (chemical to) thermal / heat
(human) body: (chemical to) heat / kinetic B1

(b) (i) (P =) IV OR in words OR 0.27 × 17 C1


= 4.59 W at least 2 s.f. A1

(ii) (K.E. =) efficiency × input OR 0.35 × 4.59 C1


= 1.61 J or Nm at least 2 s.f. A1

(iii) 1. d = m/V OR (m =) V × d OR in words OR 0.00014 × 1000 C1


= 0.14 kg A1

2. P.E. gained = K.E. lost OR mgh = ½ mv2


OR 0.14 × 10 × h = 1.61 OR 1.6 C1
h = 1.15 m OR 1.14 m at least 2 s.f. A1

OR
½ mv2 = 1.61 OR
v2 = 2 × 1.61 / 0.14 = 23 OR v2 = 2 × 1.6 / 0.14 = 22.86 (C1)
(h =) v2/2g = 23/20 = 1.15 m OR (h =) 22.86/20 = 1.14 m (A1)

[Total: 9]

4 (a) (p =) F/A OR in words OR 90/4.8 OR 90 / 0.00048 C1


= 18.75 N/cm2 OR 1.875 × 105 Pa OR 187500 Pa
OR 187.5 kPa OR 0.1875 MPa at least 2 s.f. A1

(b) Area of Y bigger (than area of X so force greater) B1

(c) Volume of oil moved at Y = volume of oil moved at X B1


Area of Y × distance moved by Y = Area of X × distance moved by X (so distance
move by Y smaller) B1
OR
Work done by piston X = work done on piston Y (B1)
Work = force × distance and F2 is greater than F1 so distance moved by Y smaller
(than distance moved by X) (B1)

(d) Air bubbles compress when pressure applied M1


More movement of piston X required for same movement of piston Y
OR Y moves less (for same movement of X)
OR Driver must push the brake pedal further / do more work
OR Pressure reduced / force on Y reduced
OR System is less efficient A1

[Total: 7]

© University of Cambridge International Examinations 2012


Page 6 Mark Scheme: Teachers’ version Syllabus Paper
IGCSE – May/June 2012 0625 31

5 (a) (i) e.g. freezing, solidification, condensation


OR example e.g. water to ice, steam to water, gas to solid B1

(ii) No change B1

(b) Heat/energy required to change temperature of the body B1


by 1 °C / 1 K / 1 unit / 1 deg B1
OR
mass (of body) × specific heat capacity (B2)

(c) (i) Q = mcθ OR in words OR 250 × 4.2 × 20 C1


= 21000 J A1

(ii) 21000 J OR same as (c)(i) B1

(iii) Q = mL OR m = Q/L OR either in words


OR 21000 = m × 330 OR m = 21000/330 C1
= 63.6 g at least 2 s.f. A1

[Total: 9]

6 (a) (i) Glass / flask receives heat / rises in temperature B1


Glass / flask expands B1

(ii) Heat flows through glass to water OR Water receives heat / thermal energy
from / conducted by glass OR Water temperature rises OR Water molecules
move faster / gain K.E. B1
Water expands / Water molecules move further apart B1

(iii) Glass / solid expands less OR water / liquid expands more B1

(b) Use a bigger flask OR a narrower tube


OR Use a solid and a liquid that expand more B1

[Total: 6]

© University of Cambridge International Examinations 2012


Page 7 Mark Scheme: Teachers’ version Syllabus Paper
IGCSE – May/June 2012 0625 31

7 (a) (Molecule) moves up and down / rises and falls


OR oscillates perpendicular to direction of wave
OR describes a circle B1

(b) (i) At least 3 circular arcs, angular spread greater than 90° (symmetrically above
and below slit B1
Centre of arcs at centre of slit and with same spacing (by eye) as incident
waves B1

(ii) Diffraction B1

(c) v = f × λ OR 12 = f × 1.4 OR f = v / λ OR f = 12 / 1.4 C1


f = 8.57 Hz / per s / waves or vibrations per s A1
at least 2 s.f.

[Total: 6]

8 (a) (i) Electron(s) B1

(ii) At least 2 + signs on left-hand side of S


Same number of – signs on right-hand side of S B1

(iii) Connect S to earth (with rod in place) M1


Remove connection of S to earth M1
Remove R / rod A1

(b) (i) Q = It OR I = Q / t OR in words OR I = 30/120 C1


= 0.25 A or C/s A1

(ii) E = IVt OR in words OR 0.25 × 1.5 × 106 × 120 C1


OR
E = QV OR in words OR 30 × 1.5 × 106 (C1)
E = 45000000 J / 4.5 × 107 J / 45 MJ / 12.5 kWh A1

[Total: 9]

© University of Cambridge International Examinations 2012


Page 8 Mark Scheme: Teachers’ version Syllabus Paper
IGCSE – May/June 2012 0625 31

9 (a) (i) I1 = I2 + I3 B1

(ii) I1 = I4 OR same B1

(b) (i) (V = IR = 0.80 × 3.0 =) 2.4 V A1

(ii) I = V/R in any algebraic form OR 2.4 / 2 OR (b)(i) / 2


OR any voltage divided by 2 C1
(I3 = V/R = 2.4 / 2 =) 1.2 A A1
OR
I3/I2 = 3/2 (C1)
I3 = 3/2 × 0.8 A = 1.2 A (A1)

(iii) (I2 + I3 OR Current through R = 0.8 + 1.2) = 2.0 (A)


OR 6 V / 2 A used C1
Parallel combination formula: 1/r = 1/r1 + 1/r2
OR (r =) r1r2/(r1 + r2) C1
Use of formula: combined resistance = 1.2 (Ω) C1
(R + 1.2 = 6/2 = 3.0 Ω R =) 1.8 Ω A1
OR
Current through R = 0.8 + 1.2 = 2.0 (A) (C1)
P.D. across R = 6.0 – 2.4 (C1)
= 3.6 (V) (C1)
R = 3.6 / 2.0 = 1.8 Ω (A1)

[Total: 9]

10 (a) (i) Parallel lines perpendicular to pole faces with arrows N to S B1

(ii) Arrow pointing to the right B1

(b) (i) Geiger (counter) / Geiger (tube) (+ scaler / ratemeter) / photographic plate /
scintillation counter / cloud chamber / luminescent or phosphorescent plate B1

(ii) Out of the plane of the paper B1

(iii) (Path is) a curve / circular / arc B1

(iv) (Air molecules are) ionised / lose electrons B1

[Total: 6]

© University of Cambridge International Examinations 2012


Page 9 Mark Scheme: Teachers’ version Syllabus Paper
IGCSE – May/June 2012 0625 31

11 (a) Transistor B1

(b) Resistor / variable resistor / rheostat identified B1


Light-dependent resistor / LDR identified B1
Resistor or alternative in gap A; LDR in gap B B1

(c) Thermistor / thermal resistor / heat or temperature dependent resistor identified B1


Thermistor (or alternative name) in gap A and resistor in gap B B1

[Total: 6]

© University of Cambridge International Examinations 2012


w
w
w
.X
tr
me
eP
CAMBRIDGE INTERNATIONAL EXAMINATIONS

ap
er
International General Certificate of Secondary Education

s.c
om
MARK SCHEME for the October/November 2012 series

0625 PHYSICS
0625/31 Paper 3 (Extended Theory), maximum raw mark 80

This mark scheme is published as an aid to teachers and candidates, to indicate the requirements of
the examination. It shows the basis on which Examiners were instructed to award marks. It does not
indicate the details of the discussions that took place at an Examiners’ meeting before marking began,
which would have considered the acceptability of alternative answers.

Mark schemes should be read in conjunction with the question paper and the Principal Examiner
Report for Teachers.

Cambridge will not enter into discussions about these mark schemes.

Cambridge is publishing the mark schemes for the October/November 2012 series for most IGCSE,
GCE Advanced Level and Advanced Subsidiary Level components and some Ordinary Level
components.
Page 2 Mark Scheme Syllabus Paper
IGCSE – October/November 2012 0625 31

NOTES ABOUT MARK SCHEME SYMBOLS & OTHER MATTERS

M marks are method marks upon which further marks depend. For an M mark to be scored, the
point to which it refers must be seen in a candidate's answer. If a candidate fails to
score a particular M mark, then none of the dependent marks can be scored.

B marks are independent marks, which do not depend on other marks. For a B mark to scored,
the point to which it refers must be seen specifically in the candidate’s answers.

A marks In general A marks are awarded for final answers to numerical questions. If a final
numerical answer, eligible for A marks, is correct, with the correct unit and an
acceptable number of significant figures, all the marks for that question are normally
awarded. It is very occasionally possible to arrive at a correct answer by an entirely
wrong approach. In these rare circumstances, do not award the A marks, but award C
marks on their merits. However, correct numerical answers with no working shown gain
all the marks available.

C marks are compensatory marks in general applicable to numerical questions. These can be
scored even if the point to which they refer are not written down by the candidate,
provided subsequent working gives evidence that they must have known it. For
example, if an equation carries a C mark and the candidate does not write down the
actual equation but does correct substitution or working which shows he knew the
equation, then the C mark is scored. A C mark is not awarded if a candidate makes two
points which contradict each other. Points which are wrong but irrelevant are ignored.

brackets ( ) around words or units in the mark scheme are intended to indicate wording used to
clarify the mark scheme, but the marks do not depend on seeing the words or units in
brackets.
e.g. 10 (J) means that the mark is scored for 10, regardless of the unit given.

underlining indicates that this must be seen in the answer offered, or something very similar.

OR / or indicates alternative answers, any one of which is satisfactory for scoring the marks.

e.e.o.o. means "each error or omission".

o.w.t.t.e. means “or words to that effect”.

c.a.o. correct answer only

Spelling Be generous about spelling and use of English. If an answer can be understood to
mean what we want, give credit. However, beware of and do not allow ambiguities,
accidental or deliberate: e.g. spelling which suggests confusion between reflection /
refraction / diffraction / thermistor / transistor / transformer.

Not/NOT Indicates that an incorrect answer is not to be disregarded, but cancels another
otherwise correct alternative offered by the candidate i.e. right plus wrong penalty
applies.

Ignore Indicates that something which is not correct or irrelevant is to be disregarded and
does not cause a right plus wrong penalty.

ecf meaning "error carried forward" is mainly applicable to numerical questions, but may in
particular circumstances be applied in non-numerical questions.

© Cambridge International Examinations 2012


Page 3 Mark Scheme Syllabus Paper
IGCSE – October/November 2012 0625 31

This indicates that if a candidate has made an earlier mistake and has carried an
incorrect value forward to subsequent stages of working, marks indicated by ecf may
be awarded, provided the subsequent working is correct, bearing in mind the earlier
mistake. This prevents a candidate being penalised more than once for a particular
mistake, but only applies to marks annotated ecf.

Sig. figs. Answers are normally acceptable to any number of significant figures [ 2. Any
exceptions to this general rule will be specified in the mark scheme. In general, accept
numerical answers, which, if reduced to two significant figures, would be right.

Units Deduct one mark for each incorrect or missing unit from an answer that would
otherwise gain all the marks available for that answer: maximum 1 per question.
No deduction is incurred if the unit is missing from the final answer but is shown
correctly in the working.

Arithmetic Deduct one mark if the only error in arriving at a final answer is clearly an arithmetic
errors one.

Transcription Deduct one mark if the only error in arriving at a final answer is because given or
errors previously calculated data has clearly been misread but used correctly.

Fractions e.g. ½, ¼, 1/10 etc are only acceptable where specified.

Crossed out Work which has been crossed out and not replaced but can easily be read, should
work be marked as if it had not been crossed out.

Use of NR (# key on the keyboard) Use this if the answer space for a question is completely blank
or contains no readable words, figures or symbols.

© Cambridge International Examinations 2012


Page 4 Mark Scheme Syllabus Paper
IGCSE – October/November 2012 0625 31

1 (a) (i) s = area under graph, stated or clearly used C1


= (½ × 18 × 10) + (120 × 18) + (½ × 18 × 20) Award if at least one term correct C1
= 90 + 2160 +180 C1
= 2430 m / 2.43 km at least 2 significant figures. *Unit penalty applies A1

(ii) v = u + at in any form OR (a=) gradient OR 18/10 C1


= 1.8 m/s2 *Unit penalty applies A1

(b) (F=) ma OR 1.1 × 105 × 1.8 ecf from (a)(ii) C1


= 1.98 × 105 N at least 2 significant figures. *Unit penalty applies A1

(c) driving force = friction/air resistance/drag B1 [9]

*Apply unit penalty once only

2 (a) Size / magnitude (NOT distance) and direction B1

(b) Vectors towards East and North with arrows correct by eye B1
Complete triangle or rectangle for candidate’s vectors B1
Resultant with correct arrow B1
Resultant 94 to 96 m/s by scale OR 95 m/s by calculation *Unit penalty applies B1
Angle measured 13.5° – 15.5° OR 15° by calculation *Unit penalty applies B1 [6]

*Apply unit penalty once only

3 (a) No resultant/net force OR no resultant force in any direction


OR no resultant force in any two perpendicular directions B1

No resultant/net moment/turning effect/couple/torque


OR (total) clockwise moment = (total) anticlockwise moment B1

Either order

(b) (i) F × 120 / F × 0.12 C1


= 20 × 500 OR 20 × 0.5 C1
F = 83.3 N at least 2 significant figures. Allow 831/3 *Unit penalty applies A1

(ii) F/A or in words OR 83.3/0.0036 ecf from (b)(i) C1


= 23100 Pa / N/m2 OR 2.31 N/cm2 OR 23.1 kPa *Unit penalty applies A1 [7]

*Apply unit penalty once only

4 (a) (The point in the body) where (all) the mass / weight / gravity acts / appears to act B1
(owtte)

(b) h is the height through which the centre of mass/rises


OR centre of mass/rises (much) less than 2.0 m

© Cambridge International Examinations 2012


Page 5 Mark Scheme Syllabus Paper
IGCSE – October/November 2012 0625 31

OR centre of mass/of athlete is above the ground level


OR centre of mass/gravity passes under bar B1

Allow centre of gravity in place of centre of mass

(c) Standing: has chemical energy B1


Run-up: kinetic energy gained B1
Pole bent: has strain / elastic energy B1
Rise: potential energy gained B1
Fall: kinetic energy gained B1
On mat: has thermal / heat / sound / strain / elastic energy B1 [8]

5 (a) (i) (Force exerted when) molecules hit wall / surface / solid (and rebound) B1
Allow (force) due to momentum change in collision

(ii) Molecules/atoms/particles collide with / push against walls B1


more (often) B1
(so) bigger force / push B1

NOT collide faster

(b) P1V1 = P2V2 OR PV = constant C1


8.0 × 105 × 5000 = 1 × 105 × V2 C1
V2 = 40 000 cm3 C1
Volume escaped = 40 000 – 5000 = 35 000 cm3 A1 [8]

6 (a) Heat required to change state of / melt 1 kg / 1 g / unit mass of solid (with no
change of temperature) B1

Allow specific example e.g. ice to water


NOT liquid to gas

© Cambridge International Examinations 2012


Page 6 Mark Scheme Syllabus Paper
IGCSE – October/November 2012 0625 31

(b) (i) d = m/V in any form OR (m =) V × d C1


OR (m =) 0.25 × 0.012 × 920
= 2.76 kg at least 2 significant figures. *Unit penalty applies A1

(ii) 60% of 250 = 150 (W/m2) OR 250 × 0.25 = 62.5 (J) C1


Heat absorbed in 1 s = 150 × 0.25 = 37.5 (J)
OR 60 % of 62.5 = 37.5 J OR J/s OR W *Unit penalty applies A1

Allow J/s or W because in one second.

(iii) Q = mL OR m = Q/L OR m = 37.5 / 3.3 × 105 ecf from (b)(ii) C1


m = 0.0001136 (kg) (in 1 s) C1
Time taken = 2.76/0.000114 = 24300 s at least 2 significant figures. *Unit
penalty applies A1
OR
P = Q/t OR t = Q/P OR t = mL/P (C1)
t = 2.76 × 3.3 × 105 / 37.5 (C1)
= 24300 s *Unit penalty applies (A1) [8]

*Apply unit penalty once only

7 (a) Faster / more energetic molecules escape / evaporate (from surface) B1


Molecules left (in liquid) have lower average speed / energy so temperature is
lower B1
OR
(Latent) heat needed to evaporate / leave the surface (B1)
comes from remaining liquid (B1)

(b) (i) Dull surface is better radiator / radiates faster


OR Shiny surface is poorer radiator / radiates slower B1

(ii) C hotter (than A) OR A cooler (than C) (so evaporates at a faster rate in C) B1

(iii) Less liquid in D OR more liquid in A B1

(iv) E has greater (surface) area / more open to air / is shallower B1


greater rate of loss of heat by evaporation / convection /
conduction / radiation B1 [7]

8 (a) (i) Diagram to show – boundary, normal and ray bending towards normal B1
Angle of incidence labelled i or 51° B1
Angle of refraction labelled r or 29° B1

(ii) n = sin i / sin r OR n = sin 51 / sin 29 C1


n = 1.603 at least 2 s.f. *Unit penalty applies A1

© Cambridge International Examinations 2012


Page 7 Mark Scheme Syllabus Paper
IGCSE – October/November 2012 0625 31

(b) Ray is totally internally reflected / undergoes TIR B1


Angle of incidence is more than / equal to the critical angle (of the glass) B1
OR
Ray travels along the boundary (B1)
Angle of incidence = critical angle (of the glass) (B1)
OR
Critical angle calculated as 38.6° ecf from (a)(ii) (B1)
Angle of incidence greater than critical angle (of the glass) (B1) [7]

9 (a) (i) In the opposite direction OR downwards B1


Faster / fast B1

(ii) No voltage/current induced B1


Currents/voltages (induced) in each half of XY are equal and in opposite
directions/oppose each other B1

(b) (i) Y-plates B1

(ii) Up and down (repeatedly) owtte B1

(iii) Off / zero B1 [7]

10 (a) (i) current

(ii) p.d. OR potential difference OR voltage B1

Both required

(b) R = R1 + R2 OR 1.2 + 3.6 OR 4.8 (k Ω) C1


I = 9.0 / 4.8 = 1.875 (mA) OR 9.0/4800 = 1.875 × 10–3 (A) C1
Voltmeter reading = 6.75 V *Unit penalty applies A1
OR
Voltmeter reading = [R1 / (R1 + R2)] V (C1)
= [3.6 / (1.2 + 3.6)] × 9.0 (C1)
= 6.75 V *Unit penalty applies (A1)

(c) (In fire) temperature of thermistor rises and its resistance falls B1
Current (through thermistor and relay coil) rises / flows B1
OR voltage / p.d. across / of relay coil rises
Magnetic field of relay closes switch (and bell rings) B1 [7]

*Apply unit penalty once only

© Cambridge International Examinations 2012


Page 8 Mark Scheme Syllabus Paper
IGCSE – October/November 2012 0625 31

11 (a) (i) alpha or α

(ii) beta or β

(iii) gamma or γ B2

Symbols must be clear


3 correct B2
2 correct B1

(b) (i) repulsion B1


α particle and (gold) nucleus / protons of (gold) nucleus have positive charges B1

(ii) Any two of:


Nucleus is very small (compared to size of atom) OR Most of atom is empty
space

Nucleus is positive / contains protons OR Nucleus has (all) the positive charge
of the atom

Nucleus is heavy OR Nucleus has most / all of the mass of the atom B2 [6]

Ignore neutrons

© Cambridge International Examinations 2012


Mark scheme for IGCSE Physics (0625/6) – Alternative to Practical November 1999

1 (a) 98, 83, 52 mm OR any other checkable value to ± 1 mm 1


corresponding values 68, 53, 21mm OR other checkable value to ± 1(mm) 1
Correct subtraction of candidate’s “Q” from candidate’s “P” (i.e. e.c.f) 1
Clear working throughout, accurate and in range (3/3.1) and good
presentation of h 1

(b) written answer / candidate’s diagram / from drawing on Fig 1.1:


ANY four points from:
Set perpendicular
zero on bench
close to tube
use of fiducial aid
position of eye to avoid parallax
read bottom of meniscus
use of background
repeats 4
8

2 (a) Four acceptable symbols (i.e. can be found in text books) and in series 1
Power-supply polarity labelled, correct diode connection 1
Open switch 1
(b)(i) Polarity of ammeter compatible with diode connections 1
Ammeter anywhere in series with other components 1
(ii) Yes box ticked plus current is same at every point in circuit 1
(c)(i) 3/100 A, 0.03 A or 30mA (unit required) 1
(ii) no other resistance in circuit / 3 volt across 100 _ / max resistance
is 100 _ 1
8

3 (a)(i) ratio of masses: 9 or 9:1, not other way round 1


ratio of areas: 8 - 8.1, or 8:1 etc 1
(ii) The height was the same for each area 1
“Use” of table, specific use or by just referring to areas, / balls / masses 1
Observation is consistent/true/same for all the masses or balls 1
(iii) as the mass decreases h increases 1
(b) Heaviest: each piece absorbs all of the KE / KE is transformed into other
forms of energy or into two other named forms / No KE of rebound 1
Lightest: each piece absorbs some KE/ some KE converted into other
forms etc / a “smaller” amount of KE of rebound 1
8

4 (a)(i) reduce/prevent conduction of “heat” into (the solution) / insulates the can 1
(ii) produce a uniform temperature 1
(iii) assists in accurate temperature measurements 1
0
(b)(i) 18.7 or 18.8 C 1
8.9 or 9 1
unit with one of the three temperatures 1
(ii) Any ONE point from
assists accuracy
helps avoiding parallax

1
more certain when the thread reaches the scale division 1
(c) Take the reading with the line of sight perpendicular to the scale,
marked on a diagram 1
8

5 (a)(i) Correct axes {y against x) and labelled 1


Axes scaled sensibly occupying at least half the grid with units and using
the required origin 1
Five correct plots, to the nearest 1/3 small graph square 2
A fine line and neat plots 1
(b)(i) Five correct subtractions {2.3., 2.1, 2.3, 2.1, 2.1.} 1
Showing y – x on diagram 1
(ii) They remain constant / approximately constant 1
(iii) (y-x) larger 1
8

2
Mark scheme for IGCSE Physics (0625/6) – Alternative to Practical May/June 2000

1 (a) neat, thin line OA 1


neat, thin line AP1P2 1
arrow shown, correct direction on either 1
(b) P2P1 extended to LL’ correct and neat 1
I labelled 1
(c) continuation of IA’ shown 1
(d) IR/OC shown 1
value 2.9 - 3.1 1
8

2 (a) Move centre of compass along line from magnet 1


Judged parallel to OX in some sensible way
(e.g. use lined paper with a line along OX or
use ruler and set square, etc) 1
Tap compass to prevent sticking 1
Mark either end of needle as near as possible to
compass then mark A between 1
Look directly down on to compass so centre is
on line 1
(b)(i) N pole marked (RH end) 1
(ii) moves gradually 1
to direction approx. 1
8

3 (a) 0-10 1
(b) correct position (between 6.2 and 6.4) 1
(c)(i) 3.4 _ 1
(ii) 0.2 _ 1
(d) correct symbol 1

(e) connect variable resistor in series 1


6

4 (a) suitable table (rows or columns) 1


mass unit shown 1
temperature unit shown 1
(b)(i) way of taking an average
or there will be errors in readings, so way of
showing expected results without the errors 1
(ii) 1: 0.8 1
2: 3 1
o
(iii) tick in box 1 and reason (e.g. difficult to measure temp to 1 C)
or heat losses involved
or easy to measure mass to better than l g) 1
7
5 (a) 1.096 (1.10)
1.068 (1.07)
1.006 (1.01)
0.866 ( 0.87) all correct 1
(b) greater accuracy 1
(c)(i) Graph:
scales right way round and suitable i.e. cover at least 1/2 grid no unsuitable
scales e.g. 10 sq = 0.3 1

1
scales labelled with quantity or unit 1
all plots correct to nearest square 2
(ii) ABCD labelled 1
(iii) T increases, 1
increase is greatest for larger values of d 1
(d) 1.l (+) 1
increased range OR largest difference in T value with larger d values 1
11

2
Mark scheme for IGCSE Physics (0625/6) – Alternative to Practical November 2000

1 (a)(i) use metre rule (or other straight edge) held horizontally
at height of rule near clamp 1
and measure height from floor 1
(OR measure height from floor near clamp 2)
(i.e. appreciation that even without weight, free end of rule is depressed)
(ii) metre rule and some means to ensure it is vertical (e.g. set-square) 1
(iii) means to more easily locate reading on vertical rule
(e.g. short rule, set-square)
OR fixing vertical rule in clamp & stand
OR convincing avoidance of parallax (action AND reason) 1
NOTE: (i), (ii) and (iii) may be scored directly from the diagram(s).
(b)(i) Take time (t) for several oscillations (N)
Calculate T from t/N 1
(ii) stopclock / stopwatch 1
(iii) make N as large as possible
OR repeat readings 1
(iv) use of fiducial mark at centre of oscillation
OR explanation of what fiducial mark means
OR shown on diagram 1
(mere use of word fiducial is not sufficient)
8

2 (a) w,x and wx tabulated 1


w and x with corresponding units 1
wx unit 1
correct wx value (2/3 sf) X3 1
all wx correct to + 2 mm 1
(b) As x increases, w decreases
OR w decreases faster than x increases 2

w / cm 4.4 2.8 1.9 0.95


x / cm 3.30 5.90 7.75 10.65
2
wx / cm 14.5 16.5 14.7 10.1

3 (a) need to add brass, not water, to cup 1


(b) (start clock)
at stated time remove brass 1
place brass in cup after time t 1

(OR hold thread, start clock at same time as removing brass


place brass in cup after time t)
(c) cool for 60s 1
record maximum temperature of water 1
(d)(i) use fresh water from tap

1
OR use thermometer to measure temperature and adjust 1
(ii) use 50 g – measure on balance 1
(e) ensure temp same throughout liquid 1
8

4 (a)(i) r angle of resistance 1


(ii) to obtain any value of current 1

(b)(i) 1.
1
2. move slider along line 1
3. to ensure good electrical contact 1
(ii) insulate 23 cm 1
correct end clear 1
OR
23cm

tape
7

5 (a) graph:
scales 1
labelled 1
plots correct to the nearest half square 2
line 1
(b)(i) y1 to the nearest half square 1
y2 to the nearest half square 1
ratios correct to 2/3 s.f. 1
(ii) force increases as separation decreases 1
x ratio is < 1; y ratio > 1 1
10

2
Mark scheme for IGCSE Physics (0625/6) – Alternative to Practical May/June 2001

1 (a) Graph:
Axes, labelled, y against x 1
Scales suitable 1
plots 6 correct 3
(OR 5 correct = 2, OR 4 correct = 1)
line judgement 1
string slipped on pulley / other sensible point 1
(b) lo and l1 clear and from same points 1
e = l1 – lo 1
9
o o o
2 a)(i) 7 C, 16 C, 12 C 2
C 1
(ii) lid 1
(iii) same volume / same initial temperature 1
(b)(i) A 1
(ii) would not have gone below room temperature 1
(iii) goes down to room temperature 1
8

3 (a)(i) 0.94, 0.35, 0.48 3


(ii) 1.77 (or sum of above) 1
unit 1
(iii) 2.69, 1.00, 1.37 correct to 2 / 3 s.f. 1
unit at least once 1
(b) diagram:
resistors in parallel 1
voltmeter correct 1
ammeter correct 1
10

4 (a) correct temperature 1


0
(b) ensure at 100 C 1
(c) heat not lost during transfer 1
(d)(i) 69.5°C 1
(ii) 10.5°C 1
(e) contains more heat 1
6

5 (a) 3.00, 1.47, 1.01 arithmetic 1


all to 2 / 3 s.f. 1
(b) suitable explanation / diagram e.g. mark on block 1
and measure to object with metre rule 1
(c) upside down 1
2.9 ± 0.1 cm 1
(d) centre of object and lens in line 1
7
Mark scheme for IGCSE Physics (0625/6) – Alternative to Practical October/November 2001

1 (a)(i) Graph:,

axes, labelled, d/mm against L / N 1


scales suitable 1
plots 6 correct 3
line judgement 1
(ii) reading to 1/2 square 1
clearly shown 1
(b) Any two from:
horizontal rule at top
vertical rule in centre
clamped rule in centre
rule in centre resting on floor 2
10

2 (a)(i) 0.22, 0.23, 0.17(5) correct 2


(ii) all correct 1
all to 2/3 s.f. 1
unit (at least once) 1
(b) circuit:
lamps in parallel 1
voltmeter correct 1
ammeter correct (for any one lamp) 1
8

3 (a)(i) h = 18(mm) 1
(ii) t = 1.2mm 1
(iii) correct arithmetic 1
unit 1
(iv) d correct arithmetic 1
2/3 s.f. 1
(b) mass on correct side 1
approximately twice as far from pivot (by eye)
(between 35 and 40) 1
8

4 (a)(i) 35, 23, 32 correct 1


(iii) Any two from:
same initial temperature
same size cans
same distance from heater
same time
same room temperature 2
(b)(i) R 1
(ii) 18 - 19°C 1
clear how obtained 1
6

5 (a)(i) lines correct, thin and neat 1


o o
(ii) 30 ± 1 1
(b) Any two from:
view base of pins
ensure vertical pins as far apart as possible
(or > 5cm)

1
use a third pin 2
(c)(i) 2.4 ± 0.1 cm 1
(ii) 6.8 ± 0.1 cm 1
(iii) n = 1.3 - 1.5 (from correct arithmetic) 1
no unit and 2/3 s.f. 1
8

2
June 2003

INTERNATIONAL GCSE

MARK SCHEME

MAXIMUM MARK: 40

SYLLABUS/COMPONENT: 0625/06

PHYSICS
Alternative to Practical
Page 1 Mark Scheme Syllabus Paper
IGCSE EXAMINATIONS – JUNE 2003 0625 6

1 (a) Seven correct values: 0, 2, 3, 6, 9, 10, 12 (-1 each error) 2

(b) Graph:
Scales, labelled, suitable size 1
Axes, right way round 1
Plots to ½ sq (-1 each error) 2

(c) Line shape 1


Line thickness 1

Triangle greater than ½ line and method used 1


Correct interpolation to ½ sq 1

TOTAL 10

2 (a) 36o (61o) 1

(b) Refracted ray drawn 1


22o (61o) 1
normal correct (by eye) 1
neat, thin, correct lines 1

(c) Correct refracted ray (by eye) with arrow 1

(d) Separation (LHS) at least 5cm 1


Separation (RHS) at least 5cm 1

TOTAL 8

3 (a) (i) Voltmeter across lamp 1

(ii) Variable resistor/rheostat 1

(b) Correct position 1

(c) V 1
A 1
W 1
correct R at 9.8V = 8.16666 (any sf) 1
all R to 2/3 sf 1
consistent 2 sf or consistent 3 sf 1

TOTAL 9

4 (a) (i) 6.8cm (68mm) 1

(ii) 6.8 1
unit, mm 1

(b) (i) 3.8/3.77 or 0.38/0.377 1


mm or cm as appropriate 1

© University of Cambridge Local Examinations Syndicate 2003


Page 2 Mark Scheme Syllabus Paper
IGCSE EXAMINATIONS – JUNE 2003 0625 6

(ii) 0.94/0.95 (or evidence of division by 4) 1

(iii) 0.75094/0.75095 1

(c) Thickness of string/thickness of marks on string/stretching of


string/metre rule measures to 1mm 1

TOTAL 8

5 (a) (i) polystyrene 1

(ii) Least steep curve (or numbers suitably quoted) 1

(b) Three from:


Thickness of insulator
Room temp.
Starting temp.
Mass/vol./amount of water
Using same can 3

TOTAL 5

© University of Cambridge Local Examinations Syndicate 2003


November 2003

INTERNATIONAL GCSE

MARK SCHEME

MAXIMUM MARK: 40

SYLLABUS/COMPONENT: 0625/06

PHYSICS
Alternative to Practical
Page 1 Mark Scheme Syllabus Paper
IGCSE EXAMINATIONS – NOVEMBER 2003 0625 6

1 (a) wind string round more than once 1


divide measured length by number of turns to find c 1

(b) (i) correct diagram, blocks parallel, one at each end 1


(ii) 119 mm OR 11.9 cm to 121 mm OR 12.1 cm 1

(c) V = 32.39 to 32.41 1


cm3 1

(d) (i) Vm = 0.5 – 2 cm3 1


(ii) correct calculation and 2/3 sf (ignore unit) 1

TOTAL 8

2 (a) (i)(ii) 2 neat continuous rays (thickness up to as EF) 1


(iii) normal where incident ray meets mirror (90° by eye) 1
(iv) i = 20° ± 1° (allow e.c.f. if mark for normal not scored) 1

(b) (i)(ii) lines complete and neat with AX correctly intersecting 1


(iii) AY = 5.9 – 6.1 cm AND YX = 5.5 + 0.3 cm 1

(c) any one from:


thickness of mirror
thickness of lines
thickness of pins
judgement of where lines cross 1

TOTAL 6

3 (a) pointer at 0.35 A 1

(b) (i) variable resistor/rheostat/potentiometer 1


(ii) V 1
A 1
Ω 1
One R correct 1
All R correct (6.129, 5.769, 4, correctly rounded) 1
Consistent sf for R (either all 2 sf or all 3 sf) 1
(iii) variable resistor/number of cells 1

(c) Voltmeter in parallel with resistors (or power source) 1


Ammeter next to X 1
Symbols correct and all connections drawn in 1

TOTAL 12

© University of Cambridge Local Examinations Syndicate 2003


Page 2 Mark Scheme Syllabus Paper
IGCSE EXAMINATIONS – NOVEMBER 2003 0625 6

4 (a) Scales: y-axis 1N = 4 cm; x-axis 1m/s2 = 4/5 cm right way round 1
Both axes labelled with quantity and unit 1
Plots to ½ sq (-1 each error or omission, minimum mark zero) 2
Line thickness less than 1 mm and no 'blob' plots 1
Well judged best fit single straight line 1

(b) Large triangle used (> ½ line) clear on graph 1


Interpolation to ½ sq (if large enough triangle present) 1
Value 1.38 – 1.48 1
kg and 2/3 sf 1

TOTAL 10

5 (a) Two from:


same volume of water
same starting temperature of water
same size/shape/type beakers
same thickness/mass/volume of insulator
same room temp 2

(b) 64°C (with unit) 1

(c) B 1

TOTAL 4

© University of Cambridge Local Examinations Syndicate 2003


June 2004

INTERNATIONAL GCSE

MARK SCHEME

MAXIMUM MARK: 40

SYLLABUS/COMPONENT: 0625/06

PHYSICS
Alternative to Practical
Page 1 Mark Scheme Syllabus Paper
PHYSICS - JUNE 2004 0625 6

1 (a) 0.63 – 0.65 (A) (strictly) 1


1.64 – 1.66 (V) (strictly) 1
3.32 (g) 1
150 (cm3) 1
8 (mm) or 0.8 (cm) 1
All units correct 1

(b) Remove electrodes from beaker 1


A method to ensure gap remains the same 1
(or other suitable suggestion e.g. measurement arrangement that
the beaker sits on)

(c) New variable (e.g. temperature, surface area / vol / size of


electrodes, power source setting, depth of immersion) 1

TOTAL 9

2 (a) All T values correct (0.34, 0.44, 0,49, 0.53, 0.60, 0.63) 1
All T values to 2 sf OR all to 3sf 1

(b) Graph:
Scales suitable 1
Scales labeled and with units 1
Plots correct to ½ sq (-1 each error) 2
Line judgement 1
Line thickness (and small, neat plots) 1

(c) T = 0.51 (s) correct answer only; NO ecf 1

(d) Statement: NO 1
Reason: line not through origin (or equivalent) 1

(allow mark if candidate describes str. line or constant gradient)

TOTAL 11

3 (a) Correct voltmeter 1


Correct ammeter 1

(b) R = 3.3, 2/3 sf 1


Unit Ω or ohm 1

(c) Circuit with correct parallel connections 1


Ammeter and ONE voltmeter correct 1
Variable resistor correct 1

TOTAL 7

© University of Cambridge International Examinations 2004


Page 2 Mark Scheme Syllabus Paper
PHYSICS - JUNE 2004 0625 6

4 (a) (i) x = 14 – 16mm 1


(ii) y = 76.5 – 78.5 mm 1
(iii) u = 75mm (ecf) and v = 390mm (ecf) 1
x,y,u and v all correct and with no unit 1
(iv) m = 5.2 (ecf) 2/3 sf and with no unit 1

(b) Upside down 1

Precaution 1 1
Precaution 2 1
(e.g. repeats, use mark on block supporting lens to show centre of
lens, place metre rule on bench to take readings or clamp rule in
position, use a dark area, explanation of how to avoid parallax
error, vertical screen/lens/both, centres of lens and object in line)

TOTAL 8

5 (a) 22 1

(b) (i) 14 (ecf) 1


(ii) 64 1
units all correct 1

(c) So that heat is not lost (wtte) 1

TOTAL 5

PAPER TOTAL = [40]

© University of Cambridge International Examinations 2004


November 2004

INTERNATIONAL GCSE

MARK SCHEME

MAXIMUM MARK: 40

SYLLABUS/COMPONENT: 0625/06
PHYSICS
Alternative to Practical
Page 1 Mark Scheme Syllabus Paper
IGCSE – November 2004 0625 6

1 (a) (i) 84 1

(ii) 50 1
o
both units correct C and cm3 (or ml) 1

(b) (i) 75 1

(ii) 15 (ecf) 1

(iii) source of error e.g. thickness of string/extension of string/diagonal


windings/identified parallax 1

(iv) improvement e.g. thinner string/inextensible string/parallel windings/


no gaps between windings/repeats and averages 1

(c) (i) 2.1 (cm) 1

(ii) 31.5 or 32 cm 2 (2/3 sf and unit required) 1

(d) time 1

another temperature 1

TOTAL 11

2 (a) (i) triangle seen 1

large triangle (> ½ line) 1

correct readings to ½ sq 1

G = 0.37 – 0.39 1

(ii) ρ = 2.63 (ecf) 1

2/3 sf and g/cm3 1

(b) increased accuracy 1

TOTAL 6

3 (a) (i) 2.15 – 2.25 1

(ii) 1.1 (+ both with correct unit, cm/mm) ecf 1

(b) (i) all correct 1 values, 91.1, 81.1, 71.1, etc 1

(ii) all correct T values, 1.93, 1.80, 1.67, 1.57, 1.41, 1.28 1

3/4sf for T 1

© University of Cambridge International Examinations 2005


Page 2 Mark Scheme Syllabus Paper
IGCSE – November 2004 0625 6

(c) Graph:

scales suitable T start at 1.0s, T: 10sq : 0.2s

1: 10sq : 20cm; both labelled

and correct way round 1

plots correct to ½ sq (-1 each error) 2

line judgement 1

line thickness 1

(d) 58 cm 1

TOTAL 11

4 (a) 4 pins at least one separation, separation > 5 cm 1

normal at 90o (by eye) 1

r = 19 – 21 1

i = 31 – 33 1

unit given for both 1

TOTAL 5

5 (a) (i) all R correct, 0.464, 0.976, 1.45, 1.88, 2.25 1

2/3 sf for R 1

(ii) V, cm, Ω 1

(b) (i) 18, 4.5 (ignore unit) 1

(ii) answer 4 1

(iii) 72 1

(c) micrometer 1

TOTAL 7

© University of Cambridge International Examinations 2005


June 2005

GCSE

MARK SCHEME

MAXIMUM MARK: 40

SYLLABUS/COMPONENT: 0625/06
PHYSICS
Alternative to Practical
Page 1 Mark Scheme Syllabus Paper
IGCSE – June 2005 0625 6

1 (a) 21oC (ignore unit) (20.9 acceptable) [1]

(b) (i) t in oC and V in cm3 [1]


(ii) θ axis labelled, with unit [1]
scale 10oC to 1 cm
or 0 - 100 in 25 sq steps or 20 - 80 in 10 sq steps [1]
correct plots to ½ sq (-1 each error) [2]
well judged best fit line [1]

(c) heat lost to surroundings or by evaporation [1]

[total: 8]

2 (a) 12 cm3 [1]


0.5 A [1]
30 cm2 [1]
0.112 kg [1]
600 N [1]

(b) repeats [1]


to spot anomalous results/to calculate average [1]
or series of different V and I, plot graph
or switch on/off, prevent temp rise
or low current, minimise temp rise
or avoidance of parallax, action and reason
or clean wires, resistance caused by dirt
or tap meter, prevent sticking
or check zero error, accuracy
(in each case the reason must support the statement
to gain the second mark)
[total: 7]

3 (a) l values 50, 75, 100 [1]

(b) 1.50 V shown correctly [1]


0.375 A shown correctly [1]

(c) 2.5(3); 4.0(0); 5.2(0) all correct [1]


all to 2sf or all to 3sf [1]

(d) Ω [1]

(e) R = 7.50 - 8.00 [2]


(or R = 6.60 - 7.49)

[total: 8]

© University of Cambridge International Examinations 2005


Page 2 Mark Scheme Syllabus Paper
IGCSE – June 2005 0625 6

4 (a) 0.90; 0.78; 0.63 (-1 each error, ignore sf) [2]

(b) 0.00225; 0.00260; 0.00315 all correct (ecf) [1]


all to 2sf or all to 3sf [1]

(c) NO [1]
T/m increases as m decreases (wtte) - if statement (no) correct [1]

(d) time n oscillations [1]


divide by n (n at least 3) [1]

(e) lower spring fully compressed (wtte) [1]

[total: 9]

5 (a) normal in correct position and at 90o (by eye) [1]

(b) i = 29 - 31 [1]

(c) refracted ray correct side of normal and at angle < i [1]
r = 18 - 22 [1]

(d) ray displaced and parallel to incident ray (by eye) [1]
all correct lines drawn neatly, not too thick, and forming
continuous path [1]

(e) two pins on emerging ray, labelled Y and Z [1]


pins at least 3 cm apart [1]

[total: 8]

© University of Cambridge International Examinations 2005


UNIVERSITY OF CAMBRIDGE INTERNATIONAL EXAMINATIONS
International General Certificate of Secondary Education

MARK SCHEME for the November 2005 question paper

0625 Physics
0625/06 Paper 6 Maximum mark 40

This mark scheme is published as an aid to teachers and students, to indicate the requirements of the
examination. It shows the basis on which Examiners were initially instructed to award marks. It does not
indicate the details of the discussions that took place at an Examiners’ meeting before marking began. Any
substantial changes to the mark scheme that arose from these discussions will be recorded in the published
Report on the Examination.

All Examiners are instructed that alternative correct answers and unexpected approaches in candidates’
scripts must be given marks that fairly reflect the relevant knowledge and skills demonstrated.

Mark schemes must be read in conjunction with the question papers and the Report on the Examination.

The minimum marks in these components needed for various grades were previously published with these
mark schemes, but are now instead included in the Report on the Examination for this session.

• CIE will not enter into discussion or correspondence in connection with these mark schemes.

CIE is publishing the mark schemes for the November 2005 question papers for most IGCSE and GCE
Advanced Level and Advanced Subsidiary Level syllabuses and some Ordinary Level syllabuses.
Page 1 Mark Scheme Syllabus Paper
IGCSE – NOVEMBER 2005 0625 6

1 (a) m in g and θ in degrees 1

(b) θ not directly proportional to m 1


as m increases θ decreases 1

(c) clear in words or diagram that 'centre point' of protractor


is at point where bottom edge of rule meets protractor
and 0 – 180 line is horizontal 1
similarly clear how 'dead space' is dealt with, e.g. protractor
stuck to edge of bench with 0 – 180 line at top of bench level
OR rule placed on block that is same height as 'dead space' 1

(d) words or diagram to show rule at end of metre rule


to measure height above bench level 1
clear that rule is vertical (e.g. use set square)
OR clamped at constant angle 1

TOTAL 7

2 (a) correct symbols for resistor, voltmeter and ammeter 1


correct connections between resistors AB and BC in series with
CD in parallel with both 1
voltmeter and ammeter correctly positioned 1

(b) I in A, V in V, R in Ω 1
1.98 or 2.0; 4.00 or 4.0; 1.06 or 1.1 1
all to 2 sf or 3 sf 1

(c) 5.9Ω – 6.1Ω 1


resistance proportional to length/
doubling length, doubled resistance/
3 x length will have 3 x resistance/
wtte 1

TOTAL 8

3 (a) θ in oC, t in s 1

(b) & (c) θ axis labelled 1


scale starts at 40 oC and 2 cm to 10 oC 1
plots correct to ½ sq (–1 each error) 2
well judged best fit curves 1
lines not too thick 1

(d) Two from:


e.g. use a lid
insulate the bottom of the beaker
use a container that is a good conductor (metal) 2

TOTAL 9

4 (a) normal in correct position and at 90o (by eye) 1

(b) 9.9 – 10.2 cm 1

© University of Cambridge International Examinations 2005


Page 2 Mark Scheme Syllabus Paper
IGCSE – NOVEMBER 2005 0625 6

(c) incident ray drawn in correctly 1

(d) 27o (± 2o) 1

(e) 2.0 (or correct from candidates x value) 1


2 or 3 sf and no unit 1

(f) X on incident ray close to mirror 1


Y and Z on reflected ray 1
Y – Z distance at least 5 cm 1
i = r (by eye) 1

TOTAL 10

5 (a) 1, 2 and 3 (–1 each error or omission) 2

(b) 2 and 3 (–1 each error or omission) 2

(c) time a number (n) oscillations 1


divide time by n 1

TOTAL 6

© University of Cambridge International Examinations 2005


UNIVERSITY OF CAMBRIDGE INTERNATIONAL EXAMINATIONS
International General Certificate of Secondary Education

MARK SCHEME for the May/June 2006 question paper

0625 PHYSICS

0625/06 Paper 6, maximum raw mark 40

These mark schemes are published as an aid to teachers and students, to indicate the requirements
of the examination. They show the basis on which Examiners were initially instructed to award marks.
They do not indicate the details of the discussions that took place at an Examiners’ meeting before
marking began. Any substantial changes to the mark scheme that arose from these discussions will
be recorded in the published Report on the Examination.

All Examiners are instructed that alternative correct answers and unexpected approaches in
candidates’ scripts must be given marks that fairly reflect the relevant knowledge and skills
demonstrated.

Mark schemes must be read in conjunction with the question papers and the Report on the
Examination.

The minimum marks in these components needed for various grades were previously published with
these mark schemes, but are now instead included in the Report on the Examination for this session.

• CIE will not enter into discussion or correspondence in connection with these mark schemes.

CIE is publishing the mark schemes for the May/June 2006 question papers for most IGCSE and
GCE Advanced Level and Advanced Subsidiary Level syllabuses and some Ordinary Level
syllabuses.
Page 1 Mark Scheme Syllabus Paper
IGCSE – May/June 2006 0625 06

1 (a) (i) 1.6 (cm) 16 (mm) [1]

(ii) 0.16 (cm) 1.6 (mm) [1]


both in cm (or mm) [1]

(b) (i) 1 = 5.8 cm and w = 6.0 cm (58 mm, 60 mm) [1]

(ii) V = 5.568 (or 5.57) [1]


V in cm3 (or mm3) [1]

(c) d = 0.233 (2/3 sf) [1]


d in g/cm3 (or g/mm3) [1]

(d) Va = 7/8/9/10 cm3 [1]

TOTAL 9

2 (a) correct ammeter and voltmeter symbols [1]


correct power source, variable resistor and lamp symbols [1]
correct circuit [1]

(b) (i) A; V; Ω [1]

(ii) 5.8 or 5.79 or 5.792; 2.9 or 2.89 or 2.889 [1]


consistent 2/3 sf [1]

TOTAL 6

3 (a) All lines present and neat, a = 1.5 cm [1]

(iv) b = 4.3 cm [1]

(iv) FI = 4.3 cm (or cand's a value) [1]

(v) IJ meets NN' at right angle (by eye) [1]

(vi) c correct to + 1 mm, 2.1 cm [1]

(vii) n calculation correct [1]


2/3 sf and no unit (1.4) [1]

(b) repeats and averages [1]


greater pin spacing [1]

TOTAL 9

© University of Cambridge International Examinations 2006


Page 2 Mark Scheme Syllabus Paper
IGCSE – May/June 2006 0625 06

4 (a) (i) 24(oC) [1]

(ii) 6(oC); 4(oC) (ecf) [1]

(b) Heat lost to surroundings [1]


round flame/to gauze/tripod [1]

(c) Variable resistor [1]

TOTAL 5

5 (a) description / diagram showing 2 equal heights from bench [1]

(b) 1.11(1); 1.18(1.176); 1.25(0); 1.33(3); 1.43(1.428) [1]

(c) (i) Axes suitable and labelled, false origin as instructed [1]
Plots correct to ½ small sq [1]

(ii) Well judged best fit line [1]


line suitably thin [1]

(iii) triangle method seen [1]


More than ½ line used [1]
Gradient value correct [1]

(d) Correct W value using cand's G [1]


2/3 sf and in N [1]

TOTAL 11

© University of Cambridge International Examinations 2006


UNIVERSITY OF CAMBRIDGE INTERNATIONAL EXAMINATIONS
International General Certificate of Secondary Education

MARK SCHEME for the October/November 2006 question paper

0625 PHYSICS
0625/06 Paper 6 (Alternative to Practical), maximum raw mark 40

This mark scheme is published as an aid to teachers and students, to indicate the requirements of the
examination. It shows the basis on which Examiners were instructed to award marks. It does not
indicate the details of the discussions that took place at an Examiners’ meeting before marking began.

All Examiners are instructed that alternative correct answers and unexpected approaches in
candidates’ scripts must be given marks that fairly reflect the relevant knowledge and skills
demonstrated.

Mark schemes must be read in conjunction with the question papers and the report on the
examination.

The grade thresholds for various grades are published in the report on the examination for most
IGCSE, GCE Advanced Level and Advanced Subsidiary Level syllabuses.

• CIE will not enter into discussions or correspondence in connection with these mark schemes.

CIE is publishing the mark schemes for the October/November 2006 question papers for most IGCSE,
GCE Advanced Level and Advanced Subsidiary Level syllabuses and some Ordinary Level
syllabuses.
Page 2 Mark Scheme Syllabus Paper
IGCSE - OCT/NOV 2006 0625 06

1 (a) 4.1 (cm) [1]

(b) (i) 4.9 (cm) [1]


both in correct unit [1]

(ii) 7.83(4) (ecf) [1]


cm3 [1]

(c) (i) 7/7.0/7.1/7.2/7.3/7.4/7.5


(ecf: less than V by up to 10% with equivalent sf) [1]

(ii) correct d value (0.84 – 0.90, no ecf) [1]


1/2/3 sf and g/cm3 [1]

[Total: 8]

2 (a) cm; s; s [1]

(b) 1.835; 1.787; 1.753; 1.706; 1.672 (accept 3 sf) [1]


consistent sf (3/4) [1]

(c) Axes suitable (plots occupy at least ½ grid)


and labelled, false origin as instructed [1]
Plots correct to ½ small sq (–1 each error) [2]
Well judged best fit line [1]
line suitably thin [1]

(d) No and not a straight line through the origin [1]

(e) greater accuracy (wtte) [1]

[Total: 10]

3 (a) (i) normal correct (by eye) (single, thin line) [1]

(ii) AG = 11.5 cm (+ 0.1) [1]

(iii) i = 26o/27o/28o (ignore unit) [1]

(b) (i) CD pin separation > 5 cm [1]

(ii) bases [1]


pins may not be vertical [1]

[Total: 6]

© UCLES 2006
Page 3 Mark Scheme Syllabus Paper
IGCSE - OCT/NOV 2006 0625 06

4 (a) 28oC value [1]


unit [1]

(b) B [1]
smaller temp drop [1]
(OR neither, insignificant difference)

(c) any suitable insulator [1]

(d) Any 3 from


initial temp
volume of water
size/shape of beaker
room temp/draughts/simultaneous timings
material of beaker
beakers on same surface [3]

[Total: 8]

5 (a) correct symbols for ammeter and voltmeter [1]


correct symbols for variable resistor, lamp and resistor [1]
circuit correct [1]

(b) ammeter will show current/voltmeter shows reading [1]

(c) variable resistor [1]

(d) (i) low current/increase R of variable resistor/


lower voltage/add another lamp [1]

(ii) switch off between readings [1]

(e) A, more resistance in circuit [1]

[Total: 8]

© UCLES 2006
UNIVERSITY OF CAMBRIDGE INTERNATIONAL EXAMINATIONS
International General Certificate of Secondary Education

MARK SCHEME for the May/June 2007 question paper

0625 PHYSICS
0625/06 Paper 6 (Alternative to Practical), maximum raw mark 40

This mark scheme is published as an aid to teachers and candidates, to indicate the requirements of
the examination. It shows the basis on which Examiners were instructed to award marks. It does not
indicate the details of the discussions that took place at an Examiners’ meeting before marking began.

All Examiners are instructed that alternative correct answers and unexpected approaches in
candidates’ scripts must be given marks that fairly reflect the relevant knowledge and skills
demonstrated.

Mark schemes must be read in conjunction with the question papers and the report on the
examination.

• CIE will not enter into discussions or correspondence in connection with these mark schemes.

CIE is publishing the mark schemes for the May/June 2007 question papers for most IGCSE, GCE
Advanced Level and Advanced Subsidiary Level syllabuses and some Ordinary Level syllabuses.
Page 2 Mark Scheme Syllabus Paper
IGCSE – May/June 2007 0625 6

1 (a) θ1 = 23 [1]
unit oC correctly written [1]

(b) 19 (°C) ecf [1]


34 (°C) ecf [1]

(c) (i) heat loss (to surroundings) [1]

(ii) any two from:


insulation / mat / foil
lid
speedier transfer
repeats
wait to record max temperature
stirring
include beaker in calculation [2]

[Total: 7]

2 (a) and (b) 6 d values [1]


correct values for d 5, 10, 15, 20, 25, 30 [1]

(c) h0 = 100mm (including unit, cm/m allowed) [1]

(e) correct values for b 40, 35, 32, 28, 24, 20 (ecf) [1]

(f) Graph:
correct d axis labelled with symbol / unit [1]
plots to nearest ½ sq (-1 each error or omission) [2]
best fit straight line [1]
single line, thin and best fit [1]

(g) no
line not through origin
OR when b increases, d decreases
OR negative gradient [1]

(h) use of set square / protractor / spirit level / plumbline [1]

[Total: 11]

© UCLES 2007
Page 3 Mark Scheme Syllabus Paper
IGCSE – May/June 2007 0625 6

3 (a) correct arithmetic for R values 7.92, 1.98 [1]


both R to 2sf OR both to 3sf [1]
all correct units: V, A, Ω [1]

(b) final box (ecf) [1]


second R (or I) about ¼ of first [1]

(c) lamp symbol correct [1]


ammeter and voltmeter symbols correct [1]
correct parallel circuit (ONE ammeter and ONE voltmeter, no extra components,
but accept switch if present, ignore power source or lack of) [1]

[Total: 8]

4 (a) correct arithmetic for f, 0.154, 0.144 (any sf) [1]


correct average f (0.149, ecf) [1]
average f to 2/3 sf [1]
correct unit for average f (m) [1]

(b) precautions:
any two from:
use darkened area (wtte)
metre rule on bench or clamped
object and lens same height from bench
mark on lens holder to show position of lens centre
take more readings
choosing mid point between acceptable positions
parallax, action and reason
lens/screen perpendicular to bench [2]

(c) inverted [1]

[Total: 7]

© UCLES 2007
Page 4 Mark Scheme Syllabus Paper
IGCSE – May/June 2007 0625 6

5 (a) weight / load / force / W / L / F [1]


length / l [1]
extension / e / x / (l – l0) [1]
units N, mm, mm [1]

(b) any three from


length of spring / l0
diameter/thickness of spring
range of loads
length of wire
diameter / thickness of wire
number of coils
coil spacing [3]
do NOT allow ‘size’ or room temperature

[Total: 7]

© UCLES 2007
UNIVERSITY OF CAMBRIDGE INTERNATIONAL EXAMINATIONS
International General Certificate of Secondary Education

MARK SCHEME for the October/November 2007 question paper

0625 PHYSICS
0625/06 Paper 6 (Alternative to Practical), maximum raw mark 40

This mark scheme is published as an aid to teachers and candidates, to indicate the requirements of
the examination. It shows the basis on which Examiners were instructed to award marks. It does not
indicate the details of the discussions that took place at an Examiners’ meeting before marking began.

All Examiners are instructed that alternative correct answers and unexpected approaches in
candidates’ scripts must be given marks that fairly reflect the relevant knowledge and skills
demonstrated.

Mark schemes must be read in conjunction with the question papers and the report on the
examination.

• CIE will not enter into discussions or correspondence in connection with these mark schemes.

CIE is publishing the mark schemes for the October/November 2007 question papers for most IGCSE,
GCE Advanced Level and Advanced Subsidiary Level syllabuses and some Ordinary Level
syllabuses.
Page 2 Mark Scheme Syllabus Paper
IGCSE – October/November 2007 0625 06

1 (a) 24 [1]

(b) s, °C [1]
23, 1 (-1 each error) [2]

(c) (i) reason consistent with results [1]

(ii) Three from:


room temp/draughts etc
volume
beaker
liquid
amount of stirring
surface area [3]

(d) lid [1]

[Total: 9]

2 (a) 8, 14, 20, 25, 34, 41 (-1 each error) [2]

(b) (i) Graph:


suitable scales labelled symbol/unit [1]
all plots to nearest ½ sq (-1 each error or omission) [2]
line thin and straight [1]

(ii) correct value (29mm – 31mm)to nearest ½ sq. [1]


clear how obtained [1]

[Total: 8]

3 (a) 0.41, 0.13, 0.14, 0.12(-1 each error) [2]


I in A at least once [1]

(b) statement (yes)


Reason – correct within limits of experimental accuracy [1]

(c) variable resistor/extra cell/variable power source/potential divider/potentiometer [1]

(d) (i) correct arithmetic for R 3.90 (ecf) [1]


unit and 2/3 sf [1]

(ii) voltmeter correct position and symbol [1]

[Total: 8]

© UCLES 2007
Page 3 Mark Scheme Syllabus Paper
IGCSE – October/November 2007 0625 06

4 (a) (i) x = 2.1, 2.2 [1]

(ii) h = 6.5, 6.6 [1]


x and h with same unit [1]

(iii) correct arithmetic for n1.47 – 1.51 (ecf) [1]


2/3 sf and no unit [1]

(b) two equal heights from bench (or other valid method) [1]

[Total: 6]

5 (a) (i) 50, 75/76 [1]

(ii) 25 (ecf) [1]


cm3 (at least once and not contradicted) [1]

(iii) density 4.36 (ecf) [1]

(b) V2, V1 [1]


cm3(at least once and not contradicted) [1]
density g/cm3 [1]
5.68, 3.02 both to 2/3 sf [1]

(c) Same method, lots of grains [1]

[Total: 9]

© UCLES 2007
UNIVERSITY OF CAMBRIDGE INTERNATIONAL EXAMINATIONS
International General Certificate of Secondary Education

MARK SCHEME for the May/June 2008 question paper

0625 PHYSICS
0625/06 Paper 6 (Alternative to Practical), maximum raw mark 40

This mark scheme is published as an aid to teachers and candidates, to indicate the requirements of
the examination. It shows the basis on which Examiners were instructed to award marks. It does not
indicate the details of the discussions that took place at an Examiners’ meeting before marking began.

All Examiners are instructed that alternative correct answers and unexpected approaches in
candidates’ scripts must be given marks that fairly reflect the relevant knowledge and skills
demonstrated.

Mark schemes must be read in conjunction with the question papers and the report on the
examination.

• CIE will not enter into discussions or correspondence in connection with these mark schemes.

CIE is publishing the mark schemes for the May/June 2008 question papers for most IGCSE, GCE
Advanced Level and Advanced Subsidiary Level syllabuses and some Ordinary Level syllabuses.
Page 2 Mark Scheme Syllabus Paper
IGCSE – May/June 2008 0625 06

1 (a) (i) cm, cm, g [1]

(ii) 49.66 (or 49.7), 49.50 (or 49.5), 50.05 (or 50.0) [1]
consistent significant figures (3 or 4) [1]

(b) clear explanation/diagram [1]

(c) correct method [1]


value 49.7 (ignore a fourth significant figure)
and allow ecf from (ii) [1]

(d) d = 1.8 (cm), t = 1.2 (cm) [1]


V = 3.05 (cm3) (ecf) [1]
ρ = 16.3 unit g/cm3, 2/3 significant figures (ecf) [1]

[Total: 9]

2 Table:

(a) Units V, A, Ω (symbol/word) [1]


R values 1.11, 2.19, 5.05, 9.55 [1]
Consistent 2 or consistent 3 sig fig for R [1]

(b) (i) Yes (if within 10%) No (if not) [M1]


Circuit 1 and circuit 2 compared [A1]

(ii) limit current (so temperature not increased)


OR switch off between readings
OR check for zero error
OR Repeats
OR Parallax error explained
OR Tapping meter [1]

[Total: 6]

3 Graph:
Temperature axis labelled θ/°C [1]
Suitable scales (plots occupy at least ½ grid) [1]
Plots correct to nearest ½ square (–1 each error) [2]
Lines well judged curves [1]
Lines thin [1]

(b) Statement:
larger surface area increases rate of cooling [1]
Justification:
Correct reference to gradients of lines or readings [1]

[Total: 8]

© UCLES 2008
Page 3 Mark Scheme Syllabus Paper
IGCSE – May/June 2008 0625 06

4 Trace:

(a) all lines present, thin, neat and in correct area [1]
normal at 90° (by eye)
and EF at 30° to normal (by eye) [1]
line KJ to at least beyond P4 [1]

(b) (i) a = 12–13 (mm) no ecf [1]

(ii) b = 40 (mm) no ecf [1]


a and b both with appropriate unit [1]

(c) (i) & (ii) c recorded and d = 44 (mm) [1]

(iii) correct calculation of n, value 1.43 (ecf) [1]


2/3 significant figures with no unit [1]

[Total: 9]

5 (a) (i) triangle method used


(whether or not shown on graph) [1]
Triangle using more than half line
and position indicated on graph [1]
Expect G = 4.00–4.35 (but allow correct working
from points read from beyond 1.0 on x axis) [1]
Expect g = 9.07–9.87 (ecf from G) [1]

(ii) greater accuracy/average value [1]

(b) (i) amplitude [1]


length [1]
(other possible correct responses shape/size of bob
and number of swings)

(ii) does not affect time [1]

[Total: 8]

© UCLES 2008
UNIVERSITY OF CAMBRIDGE INTERNATIONAL EXAMINATIONS
International General Certificate of Secondary Education

MARK SCHEME for the October/November 2008 question paper

0625 PHYSICS
0625/06 Paper 6 (Alternative to Practical), maximum raw mark 40

This mark scheme is published as an aid to teachers and candidates, to indicate the requirements of
the examination. It shows the basis on which Examiners were instructed to award marks. It does not
indicate the details of the discussions that took place at an Examiners’ meeting before marking began.

All Examiners are instructed that alternative correct answers and unexpected approaches in
candidates’ scripts must be given marks that fairly reflect the relevant knowledge and skills
demonstrated.

Mark schemes must be read in conjunction with the question papers and the report on the
examination.

• CIE will not enter into discussions or correspondence in connection with these mark schemes.

CIE is publishing the mark schemes for the October/November 2008 question papers for most IGCSE,
GCE Advanced Level and Advanced Subsidiary Level syllabuses and some Ordinary Level
syllabuses.
Page 2 Mark Scheme Syllabus Paper
IGCSE – October/November 2008 0625 6

1 (a) view perpendicular to (or straight in front of rule)/use of set square [1]

(b) (i) correct e1 value 3.1 and correct e2 value 2.4 [1]
e in cm [1]

(c) density 4.43 (ecf) [1]


2/3 significant figures [1]
g/cm3 [1]

(d) e2 greater [1]


ρ greater (or identical to e2 answer) (ecf) [1]

[Total: 8]

2 Diagram: correct symbols for ammeter and voltmeter [1]


correct symbols for resistor [1]
correct circuit arrangement [1]

Table: units V, A (symbol/word) [1]

(c) Prediction 1 Yes – close enough (or words to that effect)


OR No – not close enough (or words to that effect) [1]
Prediction 2 Yes – approximately half (or words to that effect) [1]

Resistance at connections
Internal resistance of source/other sensible suggestion [1]

[Total: 7]

3 Table
θ in °C, V in cm3 [1]
correct V 0, 20, 40, 60, 80, 100 [1]

Graph: axes labelled with symbol and unit [1]


axes suitable (e.g. not ‘3’ scale) and plots occupy more than ½ grid [1]
all plots correct (better than ½ sq) [1]
well judged, thin best fit line [1]

(c) 1. sensible comment about heat loss to the surroundings, e.g. use of insulation/lid [1]
2. sensible comment about adding water in a regulated, timed flow (including smaller
volumes/set time intervals/shorter intervals [1]

[Total: 8]

© UCLES 2008
Page 3 Mark Scheme Syllabus Paper
IGCSE – October/November 2008 0625 6

4 (a) f = 14.9(4), or 15 [1]


correct unit for f [1]

(b) (i) xs = 5.0(cm) and ys = 5.2(cm) [1]

(ii) factor of ×6 [1]


y = 31.2(cm) (ecf) [1]

(iii) 15.29, 15.3, 15 (ecf) [1]

(iv) correct method [1]


2 or 3 significant figures and correct unit [1]
average f 15.1 (correct answer only) [1]

(c) inverted image [1]

[Total: 10]

5 (a) 0.7 N [1]


6 cm3 [1]
1.4 s [1]
4.0 N/cm2 [1]

(b) (i) minimum current/turn down power supply/increase resistance [1]


switch off between readings/carry out without delay [1]

(ii) variable resistor/rheostat [1]

[Total: 7]

© UCLES 2008
UNIVERSITY OF CAMBRIDGE INTERNATIONAL EXAMINATIONS
International General Certificate of Secondary Education

MARK SCHEME for the May/June 2009 question paper


for the guidance of teachers

0625 PHYSICS
0625/06 Paper 6 (Alternative to Practical), maximum raw mark 40

This mark scheme is published as an aid to teachers and candidates, to indicate the requirements of
the examination. It shows the basis on which Examiners were instructed to award marks. It does not
indicate the details of the discussions that took place at an Examiners’ meeting before marking began,
which would have considered the acceptability of alternative answers.

Mark schemes must be read in conjunction with the question papers and the report on the
examination.

• CIE will not enter into discussions or correspondence in connection with these mark schemes.

CIE is publishing the mark schemes for the May/June 2009 question papers for most IGCSE, GCE
Advanced Level and Advanced Subsidiary Level syllabuses and some Ordinary Level syllabuses.
Page 2 Mark Scheme: Teachers’ version Syllabus Paper
IGCSE – May/June 2009 0625 06

1 (a) d 2.5 (cm) [1]


x 14.5 (cm) [1]
diagram showing blocks correctly placed across the ends [1]
rule position (or distance) shown correctly [1]

(b) (i) Ve 71.1 - 71.2 (cm3) ecf allowed [1]

(ii) measuring cylinder reading 56 (cm3) [1]

(iii) ρ 2.05–2.08 (or 2.1) ecf allowed [1]


g/cm3 and 2 or 3 significant figures [1]

[Total: 8]

2 (a) 87 (oC) [1]

(b) s, oC, oC [1]

(c) A ecf allowed [1]


justified by reference to readings (up to 90s) with comparison of drops in temperatures (with
numbers) given (ecf allowed) [1]

(d) Any two from:


starting temperature
room temperature
carry out at same time
same thermometer (words to that effect)
same position of thermometers
same time intervals [2]

[Total: 6]

3 (a) R values 0.553, 1.55, 2.74, 3.74, 4.92


(2,3,4 or more significant figures) [1]
Consistent 3 or consistent 4 significant figures for final four entries [1]

(b) Graph:
Axes labelled and scales suitable (must include origin) [1]
Plots correct to ½ square (–1 each error or omission) [2]
Well judged str. line taking account of all points and reaching an axis [1]
Thin line [1]

(c) Statement proportional (wtte) or as x increases, R increases [1]


Justification straight line through origin [1]

(d) Clear indication of method on graph [1]


Correct value to ½ square [1]

© UCLES 2009
Page 3 Mark Scheme: Teachers’ version Syllabus Paper
IGCSE – May/June 2009 0625 06

(e) low current/switch off between readings


or add (variable) resistor/lamp
or reduce voltage/power [1]

[Total: 12]

4 (a) 4.0 (cm) [1]


6.0 (cm) [1]

(b) 20, 30 ecf allowed [1]


f values 11.88 (11.9), 12.00 (12.0) [1]
f consistent 3 or more significant figures [1]

(c) average f 11.9, 11.94, 11.95, 12.0, 12 (cm) ecf allowed [1]
2/3 significant figures [1]

(h) Any two from


use of darkened room
slowly moving lens back and forth to get good image
clamp rule or place on bench
avoid parallax action given
object/lens/screen perpendicular to bench
object and lens same height from bench
repeats [2]

[Total: 9]

5 (a) Q correct position with suitable number(s) [1]


Rule correctly tilted, and on bench (or arrow to indicate) [1]

(b) Any two from:


Readings taken at either side/diameter of cylinder
Position of mid point found
Mark position of centre [2]

(c) 34.5 cm [1]

[Total: 5]

© UCLES 2009
UNIVERSITY OF CAMBRIDGE INTERNATIONAL EXAMINATIONS
International General Certificate of Secondary Education

MARK SCHEME for the October/November 2009 question paper


for the guidance of teachers

0625 PHYSICS
0625/06 Paper 6 (Alternative to Practical), maximum raw mark 40

This mark scheme is published as an aid to teachers and candidates, to indicate the requirements of
the examination. It shows the basis on which Examiners were instructed to award marks. It does not
indicate the details of the discussions that took place at an Examiners’ meeting before marking began,
which would have considered the acceptability of alternative answers.

Mark schemes must be read in conjunction with the question papers and the report on the
examination.

• CIE will not enter into discussions or correspondence in connection with these mark schemes.

CIE is publishing the mark schemes for the October/November 2009 question papers for most IGCSE,
GCE Advanced Level and Advanced Subsidiary Level syllabuses and some Ordinary Level
syllabuses.
Page 2 Mark Scheme: Teachers’ version Syllabus Paper
IGCSE – October/November 2009 0625 06

1 (a) (i) d 0.5 cm or 5mm [1]

(ii) x 10.0 [1]

(b) (i)–(iii)
table: T 1.0, 0.95, 0.895 (0.90, 0.9), 0.84, 0.775 (0.78) [1]
T2 1.00, 0.903, 0.801, 0.706, 0.601 (if T correct) [1]

(c) graph:
axes labelled [1]
scales suitable, plots occupying at least half grid [1]
plots all correct to ½ square [1]
well judged line [1]
thin line, 5 neat plots [1]

(d) statement NO and not through origin/


inverse/negative gradient/
x increases, T2 decreases/ wtte [1]

[Total: 10]

2 (a) 91 (oC) [1]

(b) t in s, both θ in oC [1]

(c) statement B and justified by reference to readings [1]

(d) any two from:


same starting temperature/temperature of hot water
constant room temperature/keep away from draughts/out of direct sunlight
same time intervals [2]

[Total: 5]

3 (a)–(c)
table:
V, A, Ω [1]
V 1.8 [1]
I 0.25 [1]
R values 7.20, 3.46(3.5) [1]
consistent significant figures for R (2 or more) [1]

(d) y 0.48, 0.49, 0.5 (ecf) [1]


2/3 significant figures and no unit [1]

© UCLES 2009
Page 3 Mark Scheme: Teachers’ version Syllabus Paper
IGCSE – October/November 2009 0625 06

(e) (i) correct symbols and circuit (ignore power source symbol) [1]

(ii) voltmeter position correct [1]

(iii) control current/voltage/resistance/speed of motor [1]

[Total: 10]

4 (a) f 14.95 + 0.05 (cm) [1]


unit to match number [1]

(b) more than one value shown [1]


d 6.5 + 0.1 [1]

(c) t 0.85 + 0.05 (cm) [1]


d and t both with correct unit [1]

(d) diagram showing blocks correctly placed [1]


rule shown correctly touching both blocks [1]

(e) f 10.9 – 13.1 (cm) (or 109 – 131 (mm)) [1]


no, too far out to be explained by experimental inaccuracy (wtte) [1]

[Total: 10]

5 (a) lens between object and screen (not mirror) [1]


lens at least 2 cm from object and screen [1]
metre rule on bench or clamped [1]

(b) any two from:


use of darkened room/brighter object
slowly moving lens back and forth to obtain good image
avoid parallax, action given
lining up object and lens
object and lens at same height from bench/object on principal axis
repeats
screen/lens perpendicular to bench
mark centre of lens position on block [2]

[Total: 5]

© UCLES 2009
UNIVERSITY OF CAMBRIDGE INTERNATIONAL EXAMINATIONS
International General Certificate of Secondary Education

MARK SCHEME for the May/June 2010 question paper


for the guidance of teachers

0625 PHYSICS
0625/61 Paper 6 (Alternative to Practical), maximum raw mark 40

This mark scheme is published as an aid to teachers and candidates, to indicate the requirements of
the examination. It shows the basis on which Examiners were instructed to award marks. It does not
indicate the details of the discussions that took place at an Examiners’ meeting before marking began,
which would have considered the acceptability of alternative answers.

Mark schemes must be read in conjunction with the question papers and the report on the
examination.

• CIE will not enter into discussions or correspondence in connection with these mark schemes.

CIE is publishing the mark schemes for the May/June 2010 question papers for most IGCSE, GCE
Advanced Level and Advanced Subsidiary Level syllabuses and some Ordinary Level syllabuses.
Page 2 Mark Scheme: Teachers’ version Syllabus Paper
IGCSE – May/June 2010 0625 61

1 (a) (i) l = 29 (mm) and l = 31 (mm) (allow 2.9 cm, 3.1 cm) [1]
eA = 14 (mm) and eB = 15 (mm) (ecf) (ignore minus signs) [1]

(b) (i) both l correct to (21.5 – 22) and 24 [1]

(ii) (6.5 – 7) and 8 (ecf) (ignore minus signs) [1]

(iii) eav = 7.5 (c.a.o.) [1]

(c) statement matches readings (expect YES) (ecf NO) [1]


justification matches statement and by reference to results
(expect within limits of experimental accuracy, wtte) (too different, wtte) [1]

(d) any one of:


avoidance of parallax error explained
use of horizontal aid
measuring to same point each time
repeats
wait for springs to stop moving [1]

[Total: 8]

2 (a) (i) T1 correct 18 [1]

(ii) T2 correct 4 [1]


unit oC (either position and not contradicted) [1]

(b) graph:
y-axis labelled [1]
plots occupying at least half of grid on suitable scale [1]
all plots correct to ½ square [1]
well judged single, smooth curve line, not ‘point-to-point’ [1]
thin line [1]

(c) (i) T2 < T1 (wtte) [1]

(ii) decreasing gradient (wtte) [1]

[Total: 10]

© UCLES 2010
Page 3 Mark Scheme: Teachers’ version Syllabus Paper
IGCSE – May/June 2010 0625 61

3 (a) correct symbol [1]


correct position [1]

(b) table:
V / l values correct 8.35, 3.58, 2.08, 1.39, 1.00 [1]
consistent 2 or 3 significant figures [1]
unit V/m [1]

(c) statement matches readings (expect NO) [1]


justification matches statement and by reference to results
V / l not constant, as l increases V decreases [1]

(d) any one of:


check for zero error
avoidance of parallax error explained
switch off between readings
repeats [1]

[Total: 8]

4 (a) (i) pins at least 5 cm apart [1]

(ii) i = 30 [1]

(iii) r1 = 31 [1]

(b) (i) & (ii) both lines correct area [1]

(iii)–(v) r2 correct to + 1o with unit [1]


difference = 1 or –1 (c.a.o.) [1]

(c) statement matches result (expect YES) (ecf NO) [1]


justification matches statement and by reference to result
(expect within limits of experimental accuracy, wtte) (too different, wtte) [1]

[Total: 8]

© UCLES 2010
Page 4 Mark Scheme: Teachers’ version Syllabus Paper
IGCSE – May/June 2010 0625 61

5 (a) column 1: d, m (or in words) [1]


columns 2 and 3: t, T (or in words) [1]
columns 2 and 3: s, s (or in words) [1]

(b) accuracy/reducing uncertainty/sensible comment on reaction time [1]

(c) (i) at least three correct values entered in table


1.66, 1.52, 1.40, 1.28, 1.17 (at least 2 significant figures) c.a.o [1]

(ii) statement matches result (expect NO) AND


justification matches statement and by reference to result
(expect decreasing, not equal, not constant, different, changing, wtte)
allow ecf from (i) [1]

[Total: 6]

© UCLES 2010
w
w
w
.X
tr
me
eP
UNIVERSITY OF CAMBRIDGE INTERNATIONAL EXAMINATIONS

ap
er
International General Certificate of Secondary Education

s.c
om
MARK SCHEME for the October/November 2010 question paper
for the guidance of teachers

0625 PHYSICS
0625/61 Paper 6 (Alternative to Practical), maximum raw mark 40

This mark scheme is published as an aid to teachers and candidates, to indicate the requirements of
the examination. It shows the basis on which Examiners were instructed to award marks. It does not
indicate the details of the discussions that took place at an Examiners’ meeting before marking began,
which would have considered the acceptability of alternative answers.

Mark schemes must be read in conjunction with the question papers and the report on the
examination.

• CIE will not enter into discussions or correspondence in connection with these mark schemes.

CIE is publishing the mark schemes for the October/November 2010 question papers for most IGCSE,
GCE Advanced Level and Advanced Subsidiary Level syllabuses and some Ordinary Level
syllabuses.
Page 2 Mark Scheme: Teachers’ version Syllabus Paper
IGCSE – October/November 2010 0625 61

1 (a) correct 1/d values 0.0222, 0.0294, 0.0370, 0.0444, 0.0518 [1]
all to 2 significant figures or all to 3 consistent significant figures [1]

(b) graph:
axes suitable and labelled [1]
all plots correct to ½ small square [1]
good line judgement (position) [1]
thin line, single, no blobs (quality) [1]

(c) gradient by triangle method using at least ½ candidate’s line [1]


clear, on graph, how obtained [1]

(d) z value 0.9 – 2.5 [1]


2 or 3 significant figures and unit cm given [1]

[Total: 10]

2 (a) θr 26 [1]

(b) (i) s and °C in both tables [1]

(ii) at least 300s and given to nearest 10s or in mins [1]

(c) Table 2.2 (heating) justified by two temperature differences compared,


must see 14 and 44/56 OR 74 to 60 and 25 to 69/81 [1]

(d) any two from:


same starting temperature
constant room temperature/avoid draughts/same place
same time intervals
same thermometer (wtte)
same mass/amount/volume of water
same beaker
lid always used [2]

[Total: 6]

© UCLES 2010
Page 3 Mark Scheme: Teachers’ version Syllabus Paper
IGCSE – October/November 2010 0625 61

3 (a) 0.3 – 0.31 [1]

(b) Ω, A [1]
10.1 [1]

(c) correct calculation of 0.5Io shown (ecf) [1]


10(Ω) [1]

(d) diagram:
resistors in parallel [1]
voltmeter symbol [1]
voltmeter position [1]

[Total 8]

4 (a) (i) – (iii)


EF extended correctly and neat [1]
P3P4 line drawn correctly and neat [1]
G labelled [1]
P1 and P2 at least 5cm apart [1]

(iv) and (v) 40 – 42 (ecf) [1]


(θ – 2i) correct (ecf) [1]

o
(b) (i) 2 and unit ( ) present at least once [1]

(ii) yes (or No, ecf) [1]


reference to ‘within limits of experimental accuracy’
(or close enough or wtte) [1]

(c) no concern about pins being vertical (or wtte) [1]

[Total: 10]

5 (a) any three from:


mass/volume/amount of water
room temperature
temperature of water
amount of stirring
size/shape of beaker
temperature of ice cube
number/mass/size of cubes [3]

(b) any three from:


stopclock: time
balance: mass
thermometer: temperature
measuring cylinder: volume (of water) [3]

[Total 6]

© UCLES 2010
w
w
w
.X
tr
me
eP
UNIVERSITY OF CAMBRIDGE INTERNATIONAL EXAMINATIONS

ap
er
International General Certificate of Secondary Education

s.c
om
MARK SCHEME for the May/June 2011 question paper
for the guidance of teachers

0625 PHYSICS
0625/61 Paper 6 (Alternative to Practical), maximum raw mark 40

This mark scheme is published as an aid to teachers and candidates, to indicate the requirements of
the examination. It shows the basis on which Examiners were instructed to award marks. It does not
indicate the details of the discussions that took place at an Examiners’ meeting before marking began,
which would have considered the acceptability of alternative answers.

Mark schemes must be read in conjunction with the question papers and the report on the
examination.

• Cambridge will not enter into discussions or correspondence in connection with these mark schemes.

Cambridge is publishing the mark schemes for the May/June 2011 question papers for most IGCSE,
GCE Advanced Level and Advanced Subsidiary Level syllabuses and some Ordinary Level
syllabuses.
Page 2 Mark Scheme: Teachers’ version Syllabus Paper
IGCSE – May/June 2011 0625 61

1. (a) Three straight lines in correct positions [1]


All lines continuous, straight, neat and thin [1]

(b) a = 4.2 – 4.4 (cm) no ecf [1]


Well-judged position in triangle [1]
Line correctly drawn [1]

(c) Viewing line directly in front of card (owtte) [1]

[Total: 6]

2. (a) 23 (ºC) [1]

(b) t in s, θ in ºC [1]

T1= 14 [1]
T2 = 1 [1]

(c) Graph:
Axes the right way round, both labelled with quantity, ignore unit [1]
Use of the scale temperature 50 – 80 and time 0 – 200 or 0 – 250, using the whole grid [1]
All seven plots correct to ½ small square [1]
Good line judgement [1]
Thin line [1]

(d) Greater rate of cooling in first 30 s (owtte) ecf possible [1]


Decreasing slope of graph (owtte) ecf possible [1]

[Total: 11]

© University of Cambridge International Examinations 2011


Page 3 Mark Scheme: Teachers’ version Syllabus Paper
IGCSE – May/June 2011 0625 61

3. (a) (i) 5.4 or 5.43 or 5.429 AND 5.9 or 5.94 or 5.938 [1]
R values both to 2 significant figures OR both to 3 significant figures, in table [1]

(iii) V, A , Ω [1]

(b) (i) Correct series circuit [1]


Correct symbols for ammeter, voltmeter and lamps [1]

(ii) RT = 8.26(Ω) [1]

(c) Statement: expect No (ecf available for Yes) [1]


Outside limits of experimental accuracy (owtte) [1]

(d) Brightness changes (owtte) [1]

[Total: 9]

4. (a) Normal in centre at 90o to MR [1]


CD drawn correctly [1]
Both neat and thin [1]

(b) (i) CN drawn correctly [1]

(ii) i = 23(o) ± 1(o) (ecf allowed) [1]

(c) (i) Line through P3 and P4 correct [1]


r = 21(o) ± 1(o) [1]

(d) Any two:


Thickness of lines
Thickness of mirror
Protractor can only be read to + 1o OR protractors are not that precise (owtte)
Thickness of pins [2]

[Total: 9]

5. 1.5 cm [1]
100 cm3 [1]
0.07 m2 [1]
0.12 A [1]
23 cm [1]

[Total: 5]

© University of Cambridge International Examinations 2011


Page 4 Mark Scheme: Teachers’ version Syllabus Paper
IGCSE – May/June 2011 0625 61

Abbreviations in the mark scheme:

ecf = error carried forward.


owtte = or words to that effect.
c.a.o. = correct answer only

© University of Cambridge International Examinations 2011


w
w
w
.X
tr
me
eP
UNIVERSITY OF CAMBRIDGE INTERNATIONAL EXAMINATIONS

ap
er
International General Certificate of Secondary Education

s.c
om
MARK SCHEME for the October/November 2011 question paper
for the guidance of teachers

0625 PHYSICS
0625/61 Paper 6 (Alternative to Practical), maximum raw mark 40

This mark scheme is published as an aid to teachers and candidates, to indicate the requirements of
the examination. It shows the basis on which Examiners were instructed to award marks. It does not
indicate the details of the discussions that took place at an Examiners’ meeting before marking began,
which would have considered the acceptability of alternative answers.

Mark schemes must be read in conjunction with the question papers and the report on the
examination.

• Cambridge will not enter into discussions or correspondence in connection with these mark schemes.

Cambridge is publishing the mark schemes for the October/November 2011 question papers for most
IGCSE, GCE Advanced Level and Advanced Subsidiary Level syllabuses and some Ordinary Level
syllabuses.
Page 2 Mark Scheme: Teachers’ version Syllabus Paper
IGCSE – October/November 2011 0625 61

1 (a) graph:
axes: the right way round, labelled x and y with unit cm [1]
scale: both 10 small squares = 2 cm
(either or both 20 small squares = 5 cm also acceptable) [1]
plots: all correct to ½ small square [1]
line: well-judged, best-fit, straight, thin, continuous line [1]

(b) correct triangle method using at least ½ candidate’s line, with method clearly indicated
on graph [1]
G = 0.94 – 1.00, no ecf [1]

(c) 1.0/(candidate’s G) calculation correct, 2 or 3 significant figures and unit N [1]

(d) (i) (where rule) balances on pivot o.w.t.t.e. [1]

(ii) take readings from 49.7 OR


adjust rule by adding weight until it balances at 50.0 cm mark [1]

[Total: 9]

2 (a) θc = 24 [1]
o
C [1]

(b) θav = 55 (oC) ecf from (a) [1]

(c) any two from:


stirring
waiting for temperature (to stabilise)
view thermometer at right angles o.w.t.t.e. [2]

(d) heat loss (to surroundings) o.w.t.t.e. [1]

(e) one from:


lagging beakers o.w.t.t.e.
use of lid
swifter transfer of water [1]

© University of Cambridge International Examinations 2011


Page 3 Mark Scheme: Teachers’ version Syllabus Paper
IGCSE – October/November 2011 0625 61

(f) one from:


amount of stirring o.w.t.t.e.
hot water temperature
cold water temperature
room temperature o.w.t.t.e.
transfer time [1]

[Total: 8]

3 (a) (i) 0.27 (A) [1]

(ii) expect YES (ecf: no) [1]


expect close enough / within limits of experimental accuracy o.w.t.t.e.
ecf: beyond limits of experimental accuracy o.w.t.t.e. [1]

(b) vary/control current/voltage [1]

(c) (i) voltmeter symbol correct and correctly connected across all three resistors [1]

(ii) 2.2 (V) [1]

(iii) R correctly evaluated


ecf from (ii) [1]
2 or 3 significant figures and unit Ω [1]

[Total: 8]

4 (a) (i) normal at 90o, at centre of MR and crossing MR [1]

(ii) AB is a continuous line from B, 8 cm long [1]


AB is at 40o to normal [1]

(b) (i) continuous, thin line that reaches normal and at least touches P2 and P3 dots [1]

(ii) r = 40 – 43(o) (no ecf) [1]

(c) any two from:


thickness of lines
thickness of protractor o.w.t.t.e. / accuracy of reading protractor
thickness of pins / pin holes [2]
accept thickness of mirror / glass in front of mirror

(d) ticks in boxes 1, 3, 5 (1 mark each)


(if more than 3 ticks, –1 for each tick in a wrong box to minimum of 0) [3]

[Total: 10]

© University of Cambridge International Examinations 2011


Page 4 Mark Scheme: Teachers’ version Syllabus Paper
IGCSE – October/November 2011 0625 61

5 (a) 200 m or more with unit [1]

(b) tape measure, trundle wheel or gps device [1]

(c) correct working seen [1]


345.67 (accept 345.66, 345, 346, 350) [1]

(d) (No), readings (time or distance) too inaccurate [1]

[Total: 5]

© University of Cambridge International Examinations 2011


w
w
w
.X
tr
me
eP
UNIVERSITY OF CAMBRIDGE INTERNATIONAL EXAMINATIONS

ap
er
International General Certificate of Secondary Education

s.c
om
MARK SCHEME for the May/June 2012 question paper
for the guidance of teachers

0625 PHYSICS
0625/61 Paper 6 (Alternative to Practical), maximum raw mark 40

This mark scheme is published as an aid to teachers and candidates, to indicate the requirements of
the examination. It shows the basis on which Examiners were instructed to award marks. It does not
indicate the details of the discussions that took place at an Examiners’ meeting before marking began,
which would have considered the acceptability of alternative answers.

Mark schemes must be read in conjunction with the question papers and the report on the
examination.

• Cambridge will not enter into discussions or correspondence in connection with these mark schemes.

Cambridge is publishing the mark schemes for the May/June 2012 question papers for most IGCSE,
GCE Advanced Level and Advanced Subsidiary Level syllabuses and some Ordinary Level
syllabuses.
Page 2 Mark Scheme: Teachers’ version Syllabus Paper
IGCSE – May/June 2012 0625 61

1 (a) 50–250 g (or 0.05–0.25 kg) correct unit required [1]

(b) Centre of mass marked close to centre of cylinder [1]


Clear indication of how centre of mass is placed above the 90.0 cm mark [1]

(c) Rule unlikely to exactly balance/ difficult to balance


OR rule could slide on pivot
OR mass could slide
OR centre of mass of rule not at 50.0 cm mark
OR rule not uniform1

Do not accept comments about poor/careless technique [1]

(d) Repeat readings (wtte) [1]


OR a reference to finding exact position of centre of mass of metre rule
OR a reference to dealing with centre of mass of rule not being at 50.0 cm mark

(e) Good/ fine/ reasonable/ same to 3 significant figures


OR Within limits of experimental accuracy (wtte)
OR Too many significant figures in experimental result [1]

[Total: 6]

2 (a) θR = 22(°C) [1]

(b) Table:
mm, °C [1]
Correct d values 100, 80, 60, 40, 20, 10 [1]

(c) Temperature difference = 3(°C), higher [1]

(d) Draughts [1]


Room temperature/humidity [1]

(e) One from:


Relevant avoidance of parallax explained, in using rule or thermometer
Waiting time between readings
Wait for steady thermometer reading
Allow lamp to cool/warm up
Repeats and average [1]

[Total: 7]

© University of Cambridge International Examinations 2012


Page 3 Mark Scheme: Teachers’ version Syllabus Paper
IGCSE – May/June 2012 0625 61

3 (a) (i) (cm, V, A) [no mark awarded]

(ii) Graph:
Axes correctly labelled with quantity and unit and correct way around [1]
Suitable scales – plots occupy at least half the grid [1]
All plots correct to ½ small square [1]
Good line judgement (ecf for curve if d plotted) [1]
Single, thin, continuous line [1]

(iii) Triangle using at least half of candidate’s line clearly indicated on graph [1]
Evidence of subtraction seen [1]
G value 1.5 when rounded to 2 significant figures [1]

(b) Same as G, rounded to 2 or 3 significant figures [1]


unit Ω/ohms [1]

[Total: 10]

4 (a) d in range 79 to 80 (mm), 7.9 to 8.0 (cm) [1]


x = 61 (mm) and consistent correct unit for both (mm or cm) [1]
D = 80 (cm), X = 61 (cm) ecf from (i) and (ii) [1]

(b) f = 14.5(cm) allow ecf from (a) [1]


2 or 3 significant figures and correct unit [1]

(c) Correct statement for results (expect Yes or wtte) [1]


Idea of within (or beyond) experimental accuracy or wtte [1]
Can only score if previous mark is scored

(d) Any one from:


Use of darkened room
How to avoid parallax when taking readings
Movement of lens back and forth to obtain clearest image
Mark lens holder to show position of centre of lens
Metre rule clamped or on bench
Object, lens and screen all perpendicular to bench
Object and lens same height above bench [1]

[Total: 8]

© University of Cambridge International Examinations 2012


Page 4 Mark Scheme: Teachers’ version Syllabus Paper
IGCSE – May/June 2012 0625 61

5 (a) V1 = 74 [1]
Line of sight perpendicular to scale [1]
Perpendicular line continues to measuring cylinder at surface level [1]

(b) V2 = 81, VG = 7 (ecf allowed) [1]


All volumes in cm3, unit given at least once, not contradicted [1]

(c) (V3 – V1) = 24, VA = 17 (ecf allowed) [1]

(d) Any three from:


VA: Finger increases V3 / tube not pushed in far enough
Some water in test-tube/air is compressed
VW: Water remaining in tube
Water remaining in measuring cylinder
Tube overfilled, wtte (surface tension effect) [3]
Either VA or VW (accept only once):
Measuring cylinder readings not very sensitive
Subtraction produces large percentage uncertainty

[Total: 9]

© University of Cambridge International Examinations 2012


w
w
w
.X
tr
me
eP
CAMBRIDGE INTERNATIONAL EXAMINATIONS

ap
er
International General Certificate of Secondary Education

s.c
om
MARK SCHEME for the October/November 2012 series

0625 PHYSICS
0625/61 Paper 6 (Alternative to Practical), maximum raw mark 40

This mark scheme is published as an aid to teachers and candidates, to indicate the requirements of
the examination. It shows the basis on which Examiners were instructed to award marks. It does not
indicate the details of the discussions that took place at an Examiners’ meeting before marking began,
which would have considered the acceptability of alternative answers.

Mark schemes should be read in conjunction with the question paper and the Principal Examiner
Report for Teachers.

Cambridge will not enter into discussions about these mark schemes.

Cambridge is publishing the mark schemes for the October/November 2012 series for most IGCSE,
GCE Advanced Level and Advanced Subsidiary Level components and some Ordinary Level
components.
Page 2 Mark Scheme Syllabus Paper
IGCSE – October/November 2012 0625 61

1 (a) d0 = 21 (mm) [1]

(b) Do = 210 (mm) or 10 × candidate’s (a) [1]

(c) L values 1.0, 2.0, 3.0, 4.0, 5.0 [1]


e values 1.0, 9.0, 21.0, 29.0, 40.0 [1]

(d) Graph:
Axes correctly labelled with quantity and unit and correct way around [1]
Suitable scales [1]
All plots correct to ½ small square
Good line judgement and a single, thin, continuous line [1]

(e) Triangle method used and shown on the graph [1]


Using at least half of line [1]

(f) Any one from:


Always measure from same point on spring (top or bottom of ring)
Wait for spring/weight to stop bouncing
Use of horizontal aid/ensure ruler is vertical
Bench surface not uniform [1]

[Total: 11]

2 (a) θR = 24(°C) [1]

(b) (i) Table:


s, °C, °C [1]

(ii) About the same [1]


Justified with reference to numbers in table [1]

(c) Any two from:


Volumes of water
Room temperature/draughts
Same beaker
Initial water temperature [2]

[Total: 6]

© Cambridge International Examinations 2012


Page 3 Mark Scheme Syllabus Paper
IGCSE – October/November 2012 0625 61

3 (a) Correct symbols for ammeter, voltmeter and lamps [1]


Ammeter and voltmeter in correct positions [1]
Correct parallel circuit [1]

(b) (i) and (ii) VA = 1.9(V) RA = 2.9(2) (Ω) [1]


Units V and Ω [1]

(iii) Pointer at correct position (0.65) [1]

(c) No mark awarded

(d) Statement matches readings (expect YES) [1]


Justified with idea of experimental inaccuracy [1]
(expect ‘close enough’, owtte)

[Total: 8]

4 (a) Trace:
Normal at 90° in correct position [1]
Angle of incidence = 30° ( ± 2°) [1]

(b) P1P2 distance [ 5.0 cm [1]


P3P4 line and line GE correctly and neatly drawn [1]

(c) (i) r = 18 or 19 or 20 [1]

(ii) i/r value correct [1]

(d) (i) i/r value 1.54 and both i/r values with no unit and to 2 or 3 significant figures [1]

(ii) Idea of within (or beyond) limits of experimental accuracy [1]

[Total: 8]

© Cambridge International Examinations 2012


Page 4 Mark Scheme Syllabus Paper
IGCSE – October/November 2012 0625 61

5 (a) Measuring cylinder


Tape measure
Newtonmeter (spring balance)
Electronic balance
Manometer

1 mark each [5]

(b) (i) Viewing scale perpendicularly (owtte) [1]

(ii) Any one from:


Moving lens back and forth
Dark area (owtte)
Object and lens at same height from bench
Object lens and screen at right angles to bench [1]

[Total: 7]

© Cambridge International Examinations 2012

Das könnte Ihnen auch gefallen